High School Teacher Eligibility Test- BOARD PROFESSIONAL...

50
1) 2) 3) PROFESSIONAL EXAMINATION BOARD High School Teacher Eligibility Test- 2018 1st Feb 2019 09:30am Topic:- GENERAL HINDI िनिलखत से कौन सा िवयािदबोधक िहै ? 1. ( ^ ) 2. ( , ) 3. ( ! ) 4. ( - ) Correct Answer :- ( ! ) वालासे बना िसे कौन सा है ? 1. बालपन 2. बाली 3. गाने वाला 4. बाला Correct Answer :- गाने वाला िका िवलोम _______ है। 1. ि2. िनिच 3. सिच 4. अिच Correct Answer :-

Transcript of High School Teacher Eligibility Test- BOARD PROFESSIONAL...

Page 1: High School Teacher Eligibility Test- BOARD PROFESSIONAL ...peb.mp.gov.in/results/RESULT_18/HST_RES18/Final_anwser_key/HST… · M a ndl a / मंड ल ... Under the Madhya Pradesh

1)

2)

3)

PROFESSIONAL EXAMINATIONBOARD

High School Teacher Eligibility Test-2018

1st Feb 2019 0930am

Topic- GENERAL HINDI

िनिलखत म स कौन सा िवयािदबोधक िच ह

1 ( ^ )

2 ( )

3 ( )

4 ( - )

Correct Answer -

( )

lsquoवालाrsquo य स बना श िन म स कौन सा ह

1 बालपन

2 बाली

3 गान वाला

4 बाला

Correct Answer -

गान वाला

िच का िवलोम _______ ह

1 िच

2 िनिच

3 सिच

4 अिच

Correct Answer -

4)

5)

6)

7)

अिच

बार-बार कही गई उ ___________ कहलाती ह

1 पनोकती

2 पन

3 पनकती

4 पनरोकती

Correct Answer -

पन

lsquoपािन परात को हाथ छयो निह ननन क जल सो पग धोयोrsquo म कौन सा अलकार ह

1 उपमा अलकार

2 पक अलकार

3 अितशयो अलकार

4 ष अलकार

Correct Answer -

अितशयो अलकार

सीस का श प या ह

1 शीस

2 सीश

3 शीत

4 शीश

Correct Answer -

शीश

lsquoआड़ हाथो लनाrsquo महावर का अथ ___________ ह

1 रह खलना

2 मरन क करीब होना

3 खरी-खोटी सनाना

4 अत मख

8)

9)

10)

Correct Answer -

खरी-खोटी सनाना

lsquolearningrsquo श द क िलए िह दी भाषा म उिचत पारभािषक श दावली का चयन कर

1 रटना

2 अिधगम

3 िवणता

4 सीखाना

Correct Answer -

अिधगम

चादी का पयायवाची ________ ह

1 धनरस

2 उदक

3 खनक

4 चहास

Correct Answer -

चहास

ldquoकर मसर नाचत नगन लख हलधर को ागrdquo

हिस- हिस गोपी िफर हस मन िपय सी भाग II - म कौन सा रस ह

1 हा रस

2 शात रस

3 भ रस

4 वा रस

Correct Answer -

हा रस

Topic- GENERAL ENGLISH

1)

2)

3)

Choose the option that best expresses the meaning of the highlighted idiomphrase

Anil How did the election go

Sunil Very exciting till the last minute Our candidate proved to be a dark horse

1 The candidate is not popular but won

2 His candidate lost the election though he was expected to win

3 The candidate backed out at the last minute

4 The candidate is popular and won

Correct Answer -

The candidate is not popular but won

A sentence with an underlined word is given below Choose the correct option which is closest in meaning tothe underlined word

The fruit of ones toil helps others to continue with the mission

1 achievement

2 hard work

3 attempts

4 sufferings

Correct Answer -

hard work

Out of the following options choose the correct form of verb that is in agreement with the subject for thegiven sentence

lsquoA Tale of Two Citiesrsquo ______ a wonderful book to read

1 are

2 were

3 is

4 have been

Correct Answer -

is

4)

5)

6)

Out of the following options choose the most appropriate tense that best fills in to complete the givensentence

This year our cricket team _________ all the matches played on the Indian soil

1 will win

2 was winning

3 wins

4 has been winning

Correct Answer -

has been winning

Choose the appropriate adverb clause from the options to complete the given sentence

He was very pleased _____

1 which you passed

2 which you had passed

3 that you passed

4 that you who passed

Correct Answer -

that you passed

Out of the following four options choose the incorrect sentence

1 Rohit is a charmer and you can find him selling newspapers on M G Road from 800 am every morning

2 Diabetes a leading cause of heart problems does not allow people to make it to the hospital in time after a heartattack

3 Symbiosis Institute is Indias premier approved business school

4 Nehrus vision continued to light up the entire nation even today

Correct Answer -

Nehrus vision continued to light up the entire nation even today

7)

8)

1)

Out of the following options choose the most appropriate usage to fill in the blank

I live a few yards ______ the bus stop

1 off

2 from

3 outside

4 away

Correct Answer -

from

Arrange the sentences P Q R and S to form a logical sequence between sentences 1 and 6

1 Jurors do not have to be intelligentP In 1933 in a sheep-stealing case in Wales two of the jurors were totally unable to understand Englishbut no one found that out until after the trialQ There is no test no examination and no enquiry about their abilitiesR The convicted found this out and complained about it to the Court of Criminal AppealS No one even takes the trouble to find out whether they can understand the English language6 Still the Court of Appeal felt that it must refuse to interfere

1 QPRS

2 SQRP

3 PRQS

4 QSPR

Correct Answer -

QSPR

Topic- GENERAL KNOWLEDGE

Which country will host the 2022 Football World Cup 2022 फटबॉल िव कप की मजबानी करन वाला दशकौन होगा

1 America अमरका

2 Canada कनाडा

3 Qatar कतर

4 Mexico मको

2)

3)

4)

5)

Correct Answer -

Qatar कतर

According to sex ratio trends of Census 2011 which district of Madhya Pradesh has been ranked numberOne

जनगणना 2011 म िलग अनपात क झानो क अनसार म दश क िकस िजल को नबर एक थान िदया गया ह

1 Seoni िसओनी

2 Ratlam रतलाम

3 Balaghat बालाघाट

4 Mandla मडला

Correct Answer -

Balaghat बालाघाट

The folk dance form called lsquoMatkirsquo is prevalent in which of the following regions of Madhya Pradesh

मटकी नामक लोक न प म यदश क िनिलखत म स िकस म चिलत ह

1 Bundelkhand बदलखड

2 Malwa मालवा

3 Vindhyachal िवाचल

4 Baghelkhand बघलख

Correct Answer -

Malwa मालवा

Who appoints the Chairman and the Members of the Union Public Service Commission

सघ लोक सवा आयोग क अ और सदो की िनय कौन करता ह

1 Chief Justice of India भारत क म य यायधीश

2 Prime Minister धान मी

3 President रा ट पित

4 Chief Election Commissioner म य चनाव आय त

Correct Answer -

President रा ट पित

6)

7)

8)

Under the Madhya Pradesh Ladli Laxmi Yojana the beneficiary is paid the total amount on attaining theage of ___ मदश लाडली ली योजना क तहत लाभाथ की आय ___ वष परी होन पर कल रािश का भगतान िकयाजाता ह

1 18

2 21

3 23

4 16

Correct Answer -

21

India has won the gold medal in hockey ______times at the Olympics

ओलिपक खलो म भारत न हॉकी म ण पदक _____ बार जीता ह

1 9

2 7

3 8

4 6

Correct Answer -

8

Where is the Madhya Pradesh Tribal Museum located at

म दश जनजातीय सहालय कहा थत ह

1 Indore इदौर

2 Jhabua झबआ

3 Khargone खरगोन

4 Bhopal भोपाल

Correct Answer -

Bhopal भोपाल

The fictional boy character of the lsquoThe Jungle Bookrsquo by author Rudyard Kipling was

लखक डयाड िकपिलग ारा lsquoद जगल बकrsquo का कािनक बालक चर था

1 Kaa का

2 Mowgli मोगली

9)

10)

1)

3 Shere Khan शर खान

4 Bagheera बघीरा

Correct Answer -

Mowgli मोगली

The recipient of which one of the following awards given by Madhya Pradesh Government gets the highest cashprize

िनिलखत परारो म स िकसक ाकता को म दश सरकार ारा उतम नकद परार िदया जाता ह

1 Mahatma Gandhi award महाा गाधी परार

2 Shikar Award िशखर परार

3 Sharad Joshi Award शरद जोशी परार

4 Kishore Kumar Award िकशोर कमार परार

Correct Answer -

Mahatma Gandhi award महाा गाधी परार

Who among the following was the Trinidadian writer of Indian descent

िनिलखत म स भारतीय मल क ििनदािदयन लखक कौन थ

1 RK Narayan आर क नारायण

2 Anita Desai अनीता दसाई

3 Rudyard Kipling डयाड िकपिलग

4 VS Naipaul वीएस नायपॉल

Correct Answer -

VS Naipaul वीएस नायपॉल

Topic- GENERAL REASONING

Find the missing number ल त स या ात कर

4 196 16 169 ____ 144 64

1 81

2 36

3 32

4 21

2)

3)

4)

Correct Answer -

36

Find the average औसत ात कर

25 35 45 55 65 amp 75

1 48

2 50

3 47

4 49

Correct Answer -

50

The below series uses a sequence of alphabets and numbers Identify the incorrect combination

नीच दी गई खला अरो एव साओ का अनम योग करती ह गलत सयोजन पहचान

(i) FT85DF77ER

(ii) FT85DF77ER

(iii) FT85DE77ER

(iv) FT85DF77ER

1 ii

2 iv

3 i

4 iii

Correct Answer -

iii

A and B together can complete a piece of work in 10 days and B alone can complete the same work in 20days In how many days can A alone complete the work

A और B एक साथ काम क एक भाग को 10 िदनो म समा त करत ह और B अकल उसी काम को 20 िदनो म समा त करता हिकतन िदनो म A अकल उस काम को समा त कर सकता ह

1 20 days 20 िदन

2 30 days 30 िदन

3 10 days 10 िदन

4 25 days 25 िदन

5)

6)

Correct Answer -

20 days 20 िदन

Which one of the following four addresses is NOT EXACTLY same as the one given below

िनिलखत िदए गए चार पतो म स नीच िदए गए पत क ठीक समान कौन सा एक नही ह

Ground Floor 59 Goulburn St

Sydney NSW 2000 Australia

+61 (02) 8987 3700

(i) Ground Floor 59 Goulburn St

Sydney NSW 2000 Australia

+61 (02) 8987 3700

(ii) Ground Floor 59 Goulburn St

Sydney NSW 2000 Australia

+61 (02) 8987 3700

(iii) Ground Floor 59 Govlburn St

Sqdney NSW 2000 Australia

+61 (02) 8987 3700

(iv) Ground Floor 59 Goulburn St

Sydney NSW 2000 Australia

+61 (02) 8987 3700

1 ii

2 iv

3 i

4 iii

Correct Answer -

iii

In a group of 75 people 32 of them like cold drink 56 of them like general water and each person likesat least one of the two drinks How many people like both

75 लोगो क समह म उनम स 32 को को िड क पसद ह उनम स 56 को सामा जल पसद ह और क को दो म स कम स कम एक पय पसद ह िकतन लोगो को दोनो पसद ह

1 11

2 13

7)

1)

2)

3 12

4 14

Correct Answer -

13

If A = 1 FAT = 27 then FAITH =

यिद A = 1 FAT = 27 तो FAITH =

1 41

2 40

3 42

4 44

Correct Answer -

44

Topic- PEDAGOGY

CANCELLED

Basic education became a major initiative because of the efforts of

ाथिमक िशा िन न क यास स एक मह वपण कायम बन गई

1 Dr Radhakrishnan डॉ राधाक णन

2 Dr Rajendra Prasad डॉ राज साद

3 Mahatma Gandhi महा मा गाधी

4 Rabindranath Tagore रबी नाथ टगोर

Correct Answer -

Mahatma Gandhi महा मा गाधी

The ldquoclause of phraserdquo is an unit of perception of लॉज ऑफ ज िन न क धारणा क एक इकाई ह

1 Image छिव

2 Concept अवधारणा

3 Language भाषा

4 Thought िवचार

Correct Answer -

3)

4)

5)

6)

Language भाषा

Which method is most suitable to study communication process among students

छाो क बीच सचार िया का अयन करन क िलए कौन सी िविध सबस उपय ह

1 Case Study मामल का अयन (कस टडी)

2 Systematic Observation वथत अवलोकन

3 Experimental Method योगाक िविध

4 Introspection आ-िनरीण (इट ोस शन)

Correct Answer -

Systematic Observation वथत अवलोकन

Genes in a human being are located in मानवो म जीन इनम थत होता ह

1 cytoplasm कोिशका (साइटो ला म)

2 ribosomes राइबोसोम

3 cell membranes कोिशका िझी

4 chromosomes गणस (ोमोसोम)

Correct Answer -

chromosomes गणस (ोमोसोम)

With smaller classes teachers are much more able to ____________

छोटी काओ क साथ िशक ____________ म अिधक सम होत ह

1 Go slow while teaching िशण क दौरान धीमी गित स जान

2 Narrate more personal experiences अिधक गत अनभवो को बतान

3 Adapt instruction to individual differences among students छाो क बीच गत मतभदो क िलए अनकल िनदश दन म

4 Make use of the extra space for extra-curricular activities पातर गितिविधयो क िलए अितर जगह का उपयोग करन

Correct Answer -

Adapt instruction to individual differences among students छाो क बीच गत मतभदो क िलए अनकल िनदश दन म

While engaging in a task the child gets bored This is a sign of

एक काम म होन पर बा ऊब जाता ह यह िन का सकत ह

1 the task requiring a professional approach काय को ावसाियक िकोण की आवकता ह

7)

8)

9)

2 the task becoming mechanically repetitive काय यािक प स दोहराव वाला ह

3 the child not being intelligent बा बमान नही ह

4 the child being incapable of learning बा सीखन म असमथ ह

Correct Answer -

the task becoming mechanically repetitive काय यािक प स दोहराव वाला ह

Educational Psychologists are more concerned with the learning in __________

शिणक मनोवािनक __________ म अिधगम क साथ अिधक िचितत होत ह

1 Formal environment औपचारक वातावरण

2 Informal environment अनौपचारक वातावरण

3 Physical environment भौितक वातावरण

4 Social environment सामािजक वातावरण

Correct Answer -

Formal environment औपचारक वातावरण

Special needs education is the type of education िवशष ज़रतो वाली िशा वह िशा होती ह जो

1 Given to person with disabilities अम य को दी जाती ह

2 Given to people from remote areas दर थ ो क लोगो को दी जाती ह

3 Provided to intelligent people बमान लोगो को दी जाती ह

4 Established by colonial masters औपिनविशक मखयाओ ारा थािपत की गई ह

Correct Answer -

Given to person with disabilities अम य को दी जाती ह

The Stanford-Binet scale of intelligence was first published in the year

ब क नफोड-िबनट कल को इस वष म पहली बार कािशत िकया गया था

1 1916

2 1903

3 1908

4 1900

Correct Answer -

1916

10)

11)

12)

13)

The term lsquofictional finalismrsquo was propounded by

पद lsquoकत योजनवादrsquo (िफ शनल फाइनिल म) इनक ारा ितपािदत िकया गया था

1 Skinner नर

2 Freud ायड

3 Adler एडलर

4 Pavlov पावलोव

Correct Answer -

Adler एडलर

Rational Emotive Behavior Therapy was propounded by

तक सगत भावनाक वहार थरपी िन क ारा ितपािदत की गई थी

1 Carl Jung काल यग

2 Carl Rogers काल रोजस

3 Aaron Beck आरोन बक

4 Albert Ellis अट एिलस

Correct Answer -

Albert Ellis अट एिलस

Which of the following indicates the quality of education in a school

िनिलखत म स या एक कल म िशा की गणव ता का सकतक ह

1 Text-books and Teaching-learning material पा-प तक तथा िशण व सीखन स सबिधत सामी

2 Infrastructural facilities at the school कल म आधारभत ढाच स सबिधत सिवधाए

3 Student achievement level िवािथयो का उपल तर

4 Classroom systems का की यव था

Correct Answer -

Student achievement level िवािथयो का उपल तर

Which of the following teachers can be identified with authoritarian teaching style

िनिलखत म स कौन सा िशक अिधकारवादी िशण शली क साथ पहचाना जा सकता ह

1 Laissez-faire teacher अब ध िशक

14)

15)

2 Democratic teacher लोकतीय िशक

3 Indifferent teacher िन प िशक

4 Direct instruction teacher िनदश िशक

Correct Answer -

Direct instruction teacher िनदश िशक

Who was the pioneer of classical conditioning

िचरितित ानकलन ( लािसकल कडीशिनग) क वतक कौन थ

1 Skinner नर

2 Pavlov पावलोव

3 Kohler कोहलर

4 Freud ायड

Correct Answer -

Pavlov पावलोव

Which of the following are true with reference to short term memory

1 Limited capacity

2 Brief storage of information

3 Unlimited capacity

4 Duration of storage less than twenty seconds

अ पकािलक मित क सदभ म िन न म स या स य ह

1 सीिमत मता

2 सचना का सि त भडारण

3 असीिमत मता

4 बीस सकड स कम भडारण की अविध

1 3 and 4 3 और 4

2 2 and 4 2 और 4

3 13 and 4 13 और 4

4 12 and 4 12 और 4

Correct Answer -

12 and 4 12 और 4

1)

2)

3)

4)

Topic- GEOGRAPHY

Which of the following ranges of population size is used to define Class-III city by Indian Census

भारतीय जनगणना ारा वग-III शहर को परभािषत करन क िलए िनिलखत म स िकस रज क जनसा आकारका उपयोग िकया जाता ह

1 20 000 to 49999 20 000 स 49999

2 30000 to 59999 30000 स 59999

3 24000 to 54999 24000 स 54999

4 50000 to 99999 50000 स 99999

Correct Answer -

20 000 to 49999 20 000 स 49999

Carbonaceous rocks which produce coal and oil belong to the category of rocks called_______

कोयल और तल का उादन करन वाली काबनय चान ______ नामक चानो की णी स सबिधत ह

1 metamorphic पातरत

2 sedimentary अवसादी

3 inorganic अजिवक

4 igneous आय

Correct Answer -

sedimentary अवसादी

The ruhr-complex is a major industrial centre in र-परसर िन का एक मख औोिगक क ह

1 North America उरी अमरका

2 Russia स

3 Germany जमनी

4 Europe यरोप

Correct Answer -

Germany जमनी

The term lsquoRegurrsquo refers to श lsquoरगरrsquo ______ स सबिधत ह

1 Deltaic alluvial soils डा जलोढ़ िमी

ि ी

5)

6)

2 Laterite soils लटराइट िमी

3 Red and yellow soils लाल और पीली िमी

4 Black cotton soils काली कपास िमी

Correct Answer -

Black cotton soils काली कपास िमी

Read the given statements and answer which of the following options isare correct

1 Lower the pressure greater the atmospheric disturbance

2 Air move from higher to low pressure

िदए गए कथन को पढ़ और उर द िक िन म स कौन सास िवक सही ह

1 िजतना दाब कम होगा वायमडलीय बाधाए उतनी अिधक होगी

2 वाय उ स िन दाब की ओर गित करती ह

1 Both Statements 1 and 2 are correct दोनो कथन 1 और 2 सही ह

2 Both Statements 1 and 2 are wrong दोनो कथन 1 और 2 गलत ह

3 Statement 1 is wrong and only Statement 2 is correct कथन 1 गलत ह और कवल कथन 2 सही ह

4 Statement 1 is correct and Statement 2 is wrong कथन 1 सही ह और कथन 2 गलत ह

Correct Answer -

Both Statements 1 and 2 are correct दोनो कथन 1 और 2 सही ह

CANCELLED

1 4 1 2 and 3 4 1 2 और 3

2 4 1 3 and 2 4 1 3 और 2

3 1 4 2 and 3 1 4 2 और 3

4 1 4 3 and 2 1 4 3 और 2

Correct Answer -

7)

8)

9)

1 4 3 and 2 1 4 3 और 2

CANCELLED

Karl Pearsonrsquos correlation co-efficient is काल िपयसन का सहसबध गणाक ह

1 Arithmec mean समार मा

2 Geometric mean गणोर मा

3 None of these इनम स कोई नही

4 Harmonic mean हराक मा

Correct Answer -

Geometric mean गणोर मा

CANCELLED

ldquoEach day is more or less the same the morning is clear and bright with a sea breeze as the Sun climbshigh in the sky heat mounts up dark clouds form then rain comes with thunder lighting But rain is soonoverrdquo Which of the following regions is described in the above passage

ldquoक िदन समान स अिधक या कम होता ह सम की हवा क साथ और उल सबह होती ह जस सयआकाश म ऊचा चढ़ता ह गम बढ़ जाती ह काल बादल बनत ह िफर िबजली क साथ बारश आती ह लिकनबारश जी ख हो जाती हlsquorsquo उपरो पा म िनिलखत म स िकन ो का वणन िकया गया ह

1 Equatorial भमरखीय

2 Equatorial भमरखीय

3 Savannah सवाना

4 Mediterranean आातरक (भमसागरीय)

5 Mediterranean आातरक (भमसागरीय)

6 Monsoon मानसन

7 Monsoon मानसन

Correct Answer -

Equatorial भमरखीय

Equatorial भमरखीय

CANCELLED

In which epoch of the geological history of the Earth dinosaurs reached their largest size

पी क भगभय इितहास क िकस यग म डायनासोर अपन सबस बड़ आकार तक पचि

10)

11)

1 Triassic ट ायिसक

2 Jurassic जरािसक

3 Cretaceous ीटशस

4 Permian पिमयन

Correct Answer -

Jurassic जरािसक

CANCELLED

A spring tide would occur in which of the following conditions

िनिलखत म स िकन थितयो म ार-भाटा आता ह

1 When the Sun Moon and Earth are in a straight line

जब सय चमा और पी एक सीधी रखा म होत ह

2 When the Moon and Earth are in right angle to each other

जब चमा और पी एक-दसर क दािहन कोण म होत ह

3 When the Earth and Moon are in right angle to the Sun

जब पी और चमा सय क दािहन कोण म होत ह

4 When the Sun and Moon are in right angle to each other

जब सय और चमा एक-दसर क दािहन कोण म होत ह

Correct Answer -

When the Sun Moon and Earth are in a straight line

जब सय चमा और पी एक सीधी रखा म होत ह

CANCELLED

An observe on the Earthrsquos surface always sees the same face of the moon because

एक पयवक को पी की सतह स हमशा चाद का एक ही फलक िदखाई दता ह ोिक

1 Its path of revolution around the earth is the same as that of the earth around the Sun

इसका पी क चारो ओर परमण का माग सय क चारो ओर पी क समान ही ह

2 Its period of revolution around the Earth is the same as its period of rotation around its own axis

इसकी पी क चारो ओर परमण की अविध उसकी अपनी धरी क चारो ओर घणन की अविध क समान ह

3 Its period of rotation is the same as that of the Earth इसकी घणन की अविध पी क समान ह

ी ि ी ी

12)

13)

14)

4 Its direct of rotation is the same as that of Earth घणन की िदशा पी क समान ही ह

Correct Answer -

Its period of revolution around the Earth is the same as its period of rotation around its own axis

इसकी पी क चारो ओर परमण की अविध उसकी अपनी धरी क चारो ओर घणन की अविध क समान ह

CANCELLED

The pebbles that are faceted by the sand-blasting and shaped polished by the wind abrasions are known as

पवन अपघषन ारा पॉिलश रत-िवोिटत और साच म ढला ककड़ ___________ क प म जाना जाता ह

1 Dreikanter िकोणक

2 Pediments िकोिनका

3 Inselberg इलबग

4 Dunes टीबा

Correct Answer -

Dreikanter िकोणक

CANCELLED

Astronomical unit is the average distance between खगोलीय इकाई ______ क बीच की औसत दरी ह

1 Earth and Mars पी और मगल

2 Earth and mercury पी और बध

3 Earth and moon पी और चमा

4 Earth and Sun पी और सय

Correct Answer -

Earth and Sun पी और सय

During cold weather season in the northern plains there will be an inflow of cyclonic disturbancesfrom the _________ directions

शीत मौसम क दौरान उरी मदानी इलाको म _________ िदशाओ स चवात सबधी गड़बड़ी का अतवाह होगा

1 East and Northwest पव और उरपिम

2 East and Northeast पव और पवर

3 West and East पिम और पव

ि औ ि

15)

16)

17)

4 West and Northwest पिम और उरपिम

Correct Answer -

West and Northwest पिम और उरपिम

During an earth quake the velocity of the body waves will________ along with the increase in densityof the material it is passing through

भकप क दौरान लहरो क ऊपरी भाग का वग घन म व क साथ-साथ ________ जो इसस गजरन वाली वको आग बढाएगी

1 not change नही बदलगा

2 increase initially and then decrease शआत म बढ़गा और िफर घटगा

3 increase बढ़गा

4 decrease घटगा

Correct Answer -

increase बढ़गा

The Clouded Leopard National park is situated in which of the following states

िनिलखत म िकस रा म धिमल तदआ रा ीय उान (ाउडड लपड नशनल पाक ) थत ह

1 Tripura िपरा

2 Uttar Pradesh उर दश

3 Assam असम

4 Mizoram िमजोरम

Correct Answer -

Tripura िपरा

Usually the land surfaces are heated more quickly than the water surfaces because _____________

आम तौर पर जल सतहो की तलना म भिम सतह अिधक तजी स गम होती ह ोिक _____________ ह

1 the specific heat of water is higher than land पानी की िविश ऊा भिम स अिधक

2 the specific heat of water is lesser than land पानी की िविश ऊा भिम स कम होती

3 the latent heat of water is higher than the land पानी की अतिनिहत ऊा भिम स अिधक

4 the land reflects more heat radiation than water भिम पानी की तलना म अिधक ऊा क िविकरण को पराविततकरती

18)

19)

20)

21)

Correct Answer -

the specific heat of water is higher than land पानी की िविश ऊा भिम स अिधक

The longest shore-line is along the state of सबस लबी समतटीय रखा िन रा क साथ ह

1 Maharashtra महारा

2 Orissa उड़ीसा

3 Kerala करल

4 Gujarat गजरात

Correct Answer -

Gujarat गजरात

The position when the Earth is farthest from the Sun is known as

जब पी सय स सबस दर होती ह तो उस थित को िन नाम स जाना जाता ह

1 Perihelion उपसौर

2 Vernal Equinox बसत िवषव

3 Aphelion अपसौर

4 Autumnal Equinox शराल िवषव

Correct Answer -

Aphelion अपसौर

The seasonal reversal of winds is the typical characteristic of

हवाओ का मौसमी परवतन ______ की सामा िवशषता ह

1 Mediterranean climates only कवल भमसागरीय जलवाय

2 All of the above climates उपय सभी मौसम

3 Monsoon climate only कवल मानसन जलवाय

4 Equatorial climate only कवल भमरखीय जलवाय

Correct Answer -

Monsoon climate only कवल मानसन जलवाय

In _________ rocks the minerals will occurs in beds or layers

ो ि ो ो

22)

23)

24)

______ चानो म खिनज तल या परतो म होत ह

1 metamorphic कायातरत

2 igneous and metamorphic आय और कायातरत

3 igneous आय

4 sedimentary अवसादी

Correct Answer -

sedimentary अवसादी

Black soil is ideal for the cultivation of cotton as कपास की खती क िलए काली िमी आदश ह ोिक

1 Its colour is black यह काली होती ह

2 It is found on plateau regions यह पठार ो म पायी जाती ह

3 It is made up of lava यह लावा स बनी होती ह

4 It can retain moisture यह नमी को बरकरार रख सकती ह

Correct Answer -

It can retain moisture यह नमी को बरकरार रख सकती ह

The National Survey and Mapping Organization of the country works under the Department of___________

दश का रा ीय सवण और मानिचण सगठन ___________ िवभाग क अतगत काय करता ह

1 Space अतर

2 Science and Technology िवान और तकनीक

3 Culture सित

4 Tourism पयटन

Correct Answer -

Science and Technology िवान और तकनीक

Palk strait separates India from पाक जलडमम भारत स _____ को अलग करता ह

1 Pakistan पािकान

2 Andaman Island अडमान ीप

3 China चीन

25)

26)

27)

4 Sri Lanka ीलका

Correct Answer -

Sri Lanka ीलका

Which among the following state is the major producer of Bauxite in India

िनिलखत म स कौन सा रा भारत म बॉाइट का मख उादक ह

1 Madhya Pradesh मदश

2 Rajasthan राजथान

3 Goa गोवा

4 Orissa उड़ीसा

Correct Answer -

Orissa उड़ीसा

Which of the following states DOES NOT share border with Chhattisgarh

िनिलखत म स कौन सा रा छीसगढ़ क साथ सीमा साझा नही करता ह

1 Telangana तलगाना

2 Uttar Pradesh उर दश

3 Bihar िबहार

4 Andhra Pradesh आ दश

Correct Answer -

Bihar िबहार

Which of the following statements is INCORRECT with respect to parallels of latitudes

अाश क समानातरो क सबध म िन निलखत म स कौन सा कथन गलत ह

1 A line joining places of equal latitude is known as parallel of largest

समान अाश क थानो को जोड़न वाली रखा को िवशालतम क समानातर क प म जाना जाता ह

2 They stat from equator and run parallels to it

व भम रखा स ारभ होत ह और इसक समानातर चलत ह

3 All parallels are equal in length सभी समातर लबाई म समान ह

4 All parallels are drawn as circles on the globe ोब पर सभी समानातर वो क प म खीच जात ह

28)

29)

30)

31)

Correct Answer -

All parallels are equal in length सभी समातर लबाई म समान ह

Which of the following Indian states is also known as a lsquoLand of Red river and Blue Hillsrsquo

िनिलखत म स िकस भारतीय रा को लाल नदी और नीली पहािड़यो की भिम क नाम स जाना जाता ह

1 Uttarkhand उराखड

2 Assam असम

3 Meghalaya मघालय

4 Arunachal Pradesh अणाचल दश

Correct Answer -

Assam असम

In spatial analysis of settlement Rn = 215 indicates which type of settlement arrangement

िनपटान क थािनक िवषण म Rn = 215 यह इिगत करता ह िक िकस कार की िनपटान वथा ह

1 Uniform यिनफॉम

2 Semi-Clustered समी- ल टर

3 Clustered ल टर

4 Random रडम

Correct Answer -

Uniform यिनफॉम

Who are known as the lsquoYellow Peoplersquo lsquoयलो पीपलrsquo क प म कौन जाना जाता ह

1 Mongoloids मोगोलोइडस

2 Nigroids नीोइडस

3 Australoids ऑ लॉइडस

4 Caucasoids कॉकसोइडस

Correct Answer -

Mongoloids मोगोलोइडस

ि ि ो ौ ि

32)

33)

34)

Who publishes the topographical map of India भारत क थलाकितक मानिच को कौन कािशत करता ह

1 Geographical Survey of India भारत का भौगोिलक सवण

2 Government of India भारत सरकार

3 Geological Survey of India भारत क भगभय सवण

4 Survey of India भारत का सवण

Correct Answer -

Survey of India भारत का सवण

Who among the following claimed geography to be the lsquoEcology of Manrsquo

िनिलखत म स िकसन भगोल को मन का पारथितकी कहा ह

1 Alfred Hener अड हटनर

2 Vidal-de la Blache वाइडल-िड लॉ ॉश

3 Oo Schluter ओटो टर

4 Harlan Barrow हरलन बारो

Correct Answer -

Harlan Barrow हरलन बारो

Who among the following is regarded as the founder of humanistic approach in geography

िनिलखत म स िकस भगोल म मानवतावादी िकोण का सथापक माना जाता ह

1 William Bunge िविलयम बग

2 Yi-Fu-Tuan यी-फ- यान

3 Brain JL Berry न जएल बरी

4 Richard Peet रचड पीट

Correct Answer -

Yi-Fu-Tuan यी-फ- यान

Who prepared Lorenz curve लोरज व िकसन तयार िकया

1 Geddes गडस

2 None of these इनम स कोई नही

3 Griffith Taylor ििफथ टलर

35)

36)

37)

4 Max U Lorenz म य लोरज

Correct Answer -

Max U Lorenz म य लोरज

Gulf Streams are the currents of which of the following oceans

खाड़ी की धाराए िनिलखत महासागरो म स िकसकी धाराए ह

1 North Atlantic Ocean उरी अटलािटक महासागर

2 North Pacific Ocean उरी शात महासागर

3 Arabian Sea अरब सागर

4 South Pacific Ocean दिण शात महासागर

Correct Answer -

North Atlantic Ocean उरी अटलािटक महासागर

Disintegration wearing away and removal of rock material is generally referred as

िशला पदाथ (रॉक सामी) का टटना िमटना और हटना आमतौर पर ________ क प म सदिभत िकया जाता ह

1 Shattering िवसकारक

2 Denudation अनाादन

3 Fault श

4 Decomposition िवयोजन

Correct Answer -

Denudation अनाादन

Variations in the length of day time and night from season to season are due to

मौसम स मौसम परवतन पर िदन क समय और रात क समय की अविध म िभताए िन कारण स होती ह

1 The Earthrsquos revolution round the Sun in an elliptical manner पी का दीघवाकार तरीक स सय क चारो घणन

2 The Earthrsquos rotation on its axis पी का इसकी धरी पर घणन

3 Revolution of the Earth on a tilted axis नत अ पर पी का घणन

4 Latitudinal position of the place थान की अाश थित

Correct Answer -

Revolution of the Earth on a tilted axis नत अ पर पी का घणन

38)

39)

40)

Point out the correct sequence of mountain ranges from north to south

उर स दिण तक पवत खलाओ क सही अनम को इिगत कर

1 Great Himalaya Middle Himalaya Outer Himalaya Trans Himalaya

महान िहमालय म िहमालय बा िहमालय परा िहमालय

2 Middle Himalaya Great Himalaya Trans Himalaya Outer Himalaya

म िहमालय महान िहमालय परा िहमालय बा िहमालय

3 Outer Himalaya Middle Himalaya Great Himalaya Trans Himalaya

बा िहमालय म िहमालय महान िहमालय परा िहमालय

4 Trans Himalaya Great Himalaya Middle Himalaya Outer Himalaya

परा िहमालय महान िहमालय म िहमालय बा िहमालय

Correct Answer -

Trans Himalaya Great Himalaya Middle Himalaya Outer Himalaya

परा िहमालय महान िहमालय म िहमालय बा िहमालय

Sunrsquos halo is produced by the refraction of light in सय का भामडल ______ म काश क अपवतन ाराउ होता ह

1 Ice crystals in Cirrro-Cumulus clouds पाभ-कपास मघो क बफ िल

2 Ice crystal in Cirrus clouds पाभमघो क बफ िल

3 Dust particles in Stratus clouds री मघो क धल कण

4 Water vapour in Stratus clouds री मघो क जल वा

Correct Answer -

Ice crystal in Cirrus clouds पाभमघो क बफ िल

Read the given statements and answer which of the following options isare correct

(1) The minerals present in the rocks exposed to atmosphere are not subjected to alteration

(2) Oxidation is one of the processes of chemical weathering

िदए गए कथनो को पढ़ और उर द िक िन म स कौन सास िवक सही ह

(1) वायमल स अनावत शल म उपथत खिनज परवतन क अधीन नही होता ह

(2) ऑीकरण रासायिनक अपय की ियाओ म स एक ह

ो ो

41)

42)

1 Both statements are wrong दोनो कथन गलत ह

2 Both statements are correct दोनो कथन सही ह

3 First statement is wrong and second statement is correct पहला कथन गलत ह और दसरा कथन सही ह

4 First statement is correct and second statement is wrong पहला कथन सही ह और दसरा कथन गलत ह

Correct Answer -

First statement is wrong and second statement is correct पहला कथन गलत ह और दसरा कथन सही ह

Read the given statements and answer which of the following options isare correct

1 Sunrsquos short waves enter the earth partially heating the atmosphere

2 Heated earth surface from the sun produces broader waves which interacts and heats the atmosphere

िदए गए कथन को पढ़ और उर द िक िन म स कौन सास िवक सही ह

1 सय की छोटी तरग पी म आिशक प स वश करती ह और वायमडल को ऊत करती ह

2 सय स ऊत पी की सतह िवारत तरग उ करती ह जो परर भाव डालती ह और वायमडल कोऊत करती ह

1 Both Statements 1 and 2 are correct दोनो कथन 1 और 2 सही ह

2 Both Statements 1 and 2 are wrong दोनो कथन 1 और 2 गलत ह

3 Statement 1 is wrong and only Statement 2 is correct कथन 1 गलत ह और कवल कथन 2 सही ह

4 Only statement 1 is correct कवल कथन 1 सही ह

Correct Answer -

Both Statements 1 and 2 are correct दोनो कथन 1 और 2 सही ह

Read the given statements and answer which of the following options isare correct

(1)The rocks that get changed due to heat and pressure are termed as metamorphic rocks

(2)Slate is one such type of metamorphic rock

िदए गए कथनो को पढ़ और उर द िक िन म स कौन सास िवक सही ह

(1) शल जो ऊा और दाब क कारण परवितत हो जात ह उ कायातरक शलो क प म जाना जाता ह

(2) ट एक तरह का कायातरक शल ह

1 Both statements are wrong दोनो कथन गलत ह

2 Both statements are correct दोनो कथन सही ह

3 First statement is wrong and second statement is correct पहला कथन गलत ह और दसरा कथन सही ह

ी औ

43)

44)

4 First statement is correct and second statement is wrong पहला कथन सही ह और दसरा कथन गलत ह

Correct Answer -

Both statements are correct दोनो कथन सही ह

Read the given statements and answer which of the following options isare correct

1 Higher temperature anomaly is observed in the northern hemisphere

2 Differential heating is absent in Northern Hemisphere

िदए गए कथनो को पढ़ और उर द िक िन म स कौन सास िवक सही ह

1 उरी गोलाध म उ तापमान िवसगित पायी जाती ह

2 उरी गोलाध म अतर ऊन अनपथत होती ह

1 Both Statements 1 and 2 are correct दोनो कथन 1 और 2 सही ह

2 Both Statements 1 and 2 are wrong दोनो कथन 1 और 2 गलत ह

3 Statement 1 is wrong and Statement 2 is correct कथन 1 गलत ह और कथन 2 सही ह

4 Statement 1 is correct and Statement 2 is wrong कथन 1 सही ह और कथन 2 गलत ह

Correct Answer -

Statement 1 is correct and Statement 2 is wrong कथन 1 सही ह और कथन 2 गलत ह

Read the given statements and answer which of the following options isare correct

(1) Plutonic rocks are intrusive type of igneous rocks

(2) It cools very slowly because the surrounding rock serves as insulation around the intrusion of magma

िदए गए कथनो को पढ़ और उर द िक िन म स कौन सास िवक सही ह

(1) िवतलीय शल अतवधी कार क आश शल ह

(2) यह बत धीर-धीर ठडा होता ह ोिक आस-पास क शल मा क अतवधन क चारो ओर रोधन क प म कायकरत ह

1 Both statements are wrong दोनो कथन गलत ह

2 Both statements are correct दोनो कथन सही ह

3 First statement is wrong and second statement is correct पहला कथन गलत ह और दसरा कथन सही ह

4 First statement is correct and second statement is wrong पहला कथन सही ह और दसरा कथन गलत ह

Correct Answer -

Both statements are correct दोनो कथन सही ह

45)

46)

47)

48)

The dust and ash material hurled from the volcanoes are termed as

ालामखी स िनकलन वाली धल और राख सामी को _______ क प म कहा जाता ह

1 Pyroclasc पाइरोाक

2 Hyperclastic हाइपराक

3 Hepiroclastic हिपरोाक

4 Cirroclastic िसरोाक

Correct Answer -

Pyroclasc पाइरोाक

The vertical difference in elevation between a low tide and high tide is referred as

कम ार और उ ार क बीच ऊचाई म लबवत अतर _____ स सदिभत होता ह

1 Tidal slope ारीय ढलान

2 Tidal elevation ारीय उयन

3 Tidal range ारीय परास

4 Tidal height ारीय ऊचाई

Correct Answer -

Tidal range ारीय परास

The maximum biodiversity is found in which of the following regions िनिलखत ो म स अिधकतमजव िविवधता िकसम पायी जाती ह

1 Amazon Basin अमज़न बिसन

2 East Indies ई इडीज

3 Congo Basin कागो बिसन

4 West indies व इडीज

Correct Answer -

Amazon Basin अमज़न बिसन

The cultivation of rice crop produces_______ चावल की फसल की खती ______ का उादन करती ह

1 SO2

49)

50)

51)

2 CH4

3 CFCs

4 CO2

Correct Answer -

CH4

The pressure system with higher pressure at the centre is called__________

क म उ दबाव वाली दबाव णाली को _______ कहा जाता ह

1 front अ

2 depression अवनमन

3 cyclone चवात

4 anti-cyclone ितचवात

Correct Answer -

anti-cyclone ितचवात

The Himalayan region is poor in mineral resources because िहमालयी खिनज ससाधनो म समनही ह ोिक

1 The displacement of rock strata has disturbed the arrangement of rocks and made it complex

शलीय परत क िवथापन न चानो की वथा को अवथत कर िदया ह और इस जिटल बना िदया ह

2 The climate conditions are not suitable for exploitation of minerals

जलवाय की थित खिनजो क दोहन क िलए उपय नही ह

3 The terrain makes explanation of minerals difficult and very costly due to transportation difficulties

भ-भाग परवहन की किठनाइयो क कारण खिनजो का दोहन मल और बत महगा बना दता ह

4 It is made up of crystalline rocks यह िलीय चानो स बना ह

Correct Answer -

The displacement of rock strata has disturbed the arrangement of rocks and made it complex

शलीय परत क िवथापन न चानो की वथा को अवथत कर िदया ह और इस जिटल बना िदया ह

The process through which the moisture is added to the atmosphere by vegetation is termed as

वह िया िजसक माम स वनित ारा वातावरण म नमी िमलायी जाती ह _______ क प म जानी जाती ह

52)

53)

54)

1 Condensation सघनन

2 Evapotranspiration वान-उजन

3 Radiation िविकरण

4 Precipitation वषण

Correct Answer -

Evapotranspiration वान-उजन

The process through which the terrestrial heat is transferred to air by direct contact is termed as

वह िया िजसम सपक ारा थलीय ऊा वाय म थानातरत हो जाती ह ______ क प म जानी जाती ह

1 Conduction चालन

2 Convection सवहन

3 Insolation आतपन

4 Radiation िविकरण

Correct Answer -

Conduction चालन

The largest area under mangroves is in which of the following statesunion territory

मोव क अतगत िनिलखत राोसघ शािसत दशो म स सबस बड़ा कौन सा ह

1 Andaman and Nicobar अमान और िनकोबार

2 Andhra Pradesh आ दश

3 West Bengal पिम बगाल

4 Gujarat गजरात

Correct Answer -

West Bengal पिम बगाल

The longitudinal transverse and surface waves in an earthquake originate from

भकप म दशातर अनथ और सतह तरग यहा उ होती ह

1 The focus on the surface of the Earth पी क सतह पर क -िबद म

2 The focus within the body of the Earth पी क भीतर क -िबद म

3 The epicenter within the body of the Earth पी क भीतर उपरक म

55)

56)

57)

4 The epicenter on the surface of the Earth पी क सतह पर उपरक म

Correct Answer -

The focus within the body of the Earth पी क भीतर क -िबद म

The down slope movement of material due to gravity is called______

गाकषण क कारण पदाथ की अनढाल गित को ______ कहा जाता ह

1 mass movement पदाथ सचलन

2 deposition िनप

3 erosion रण

4 volcanic movement ालामखीय सचलन

Correct Answer -

mass movement पदाथ सचलन

Shimla is cooler than Amritsar although both are on the same latitude This is because

िशमला म अमतसर स अिधक ठड ह हालािक दोनो समान अाश पर ह ऐसा ह ोिक

1 Shimla is at a greater height above sea level than Amritsar अमतसर की तलना म िशमला सम तल स अिधकऊचाई पर ह

2 Shimla is further north िशमला उर की ओर ह

3 Shimla is farther from the equator िशमला भम रखा स आग ह

4 Their longitudes differ उनकी दशातर रखाए िभ ह

Correct Answer -

Shimla is at a greater height above sea level than Amritsar अमतसर की तलना म िशमला सम तल स अिधकऊचाई पर ह

lsquoTempo of Urbanizationrsquo measures which of the following

lsquoशहरीकरण का टपोrsquo िनिलखत म स कौन सा उपाय ह

1 Speed of urbanizaon शहरीकरण की गित

2 None of the above इनम स कोई नही

3 Inequality of urbanizaon शहरीकरण की असमानता

4 Current level of urbanizaon शहरीकरण का वतमान र

Correct Answer -

58)

59)

60)

Speed of urbanizaon शहरीकरण की गित

Out of the following options choose the INCORRECT statement

िनिलखत िवको म स गलत कथन का चयन कर

1 The clear tracts in the equatorial region recover rapidly भम रखा म भभाग तजी स ठीक हो जात ह

2 The stable communities include a redwood forest a pine forest at high elevations

थर समदायो म एक रडवड वन उ ऊचाई पर एक दवदार वन शािमल ह

3 Any ecosystem moves towards maximum biomass and stability to survive

कोई भी पारथितकी त जीिवत रहन क िलए अिधकतम जवसहित और थरता की तरफ असर होता ह

4 Tropical rain forests near equator are stable ecosystems

भम रखा क पास उकिटबधीय वषा वन थर पारथितक त ह

Correct Answer -

The clear tracts in the equatorial region recover rapidly भम रखा म भभाग तजी स ठीक हो जात ह

Seasonal contrasts are maximum in मौसमी िवषमता अिधकतम ह

1 Mid latitudes म अाश म

2 Low attitudes िन अाश म

3 High latitudes उ अाश म

4 Subtropics उपोकिटबधीय म

Correct Answer -

Mid latitudes म अाश म

In India which type of forest among the following occupies the largest area

भारत म िनिलखत म स िकस कार क वन सबस बड़ा फल आािदत करत ह

1 Sub-tropical Dry Evergreen Forest उप उकिटबधीय श सदाबहार वन

2 Mountain Wet Temperate Forest पवतीय आ शीतो वन

3 Tropical Moist Deciduous Forest उकिटबधीय आ पणपाती वन

4 Tropical Wet Evergreen Forest उकिटबधीय आ सदाबहार वन

Correct Answer -

Tropical Moist Deciduous Forest उकिटबधीय आ पणपाती वन

61)

62)

63)

64)

What is the proportion of lsquoJuvenile Populationrsquo (0-14 years) in India as per 2011Census

2011 की जनगणना क अनसार भारत म जवनाइल पॉपलशन यानी िकशोर जनस या (0-14 वष) का अनपात ाह

1 3076 of total population कल जनस या का 3076

2 2764 of total population कल जनस या का 2764

3 2933 of total population कल जनस या का 2933

4 3354 of total population कल जनस या का 3354

Correct Answer -

3076 of total population कल जनस या का 3076

What is the Belfast famous for बलफा िकसक िलए मशर ह

1 Belt of cotton textile industry कपास व उोग क

2 Ship-building industry जहाज िनमाण उोग

3 Agricultural machinery किष उपकरण

4 Aero planes manufacturing वाययान िनमाण

Correct Answer -

Ship-building industry जहाज िनमाण उोग

What is the most important occupation in tropical monsoon lands

उकिटबधीय मॉनसन भिम म सबस महपण वसाय ा ह

1 Mining खनन

2 Cattle rearing मवशी पालन

3 Agriculture किष

4 Nomadic herding नोमािडक जड़ी-बिटया

Correct Answer -

Agriculture किष

What is the most important characteristics of the islands (Indian) located in the Arabian sea

अरब सागर म थत ीपो (भारतीय) की सबस महपण िवशषता ा ह

ी ो

65)

66)

67)

1 There are all of coral origins सभी कोरल मल क ह

2 There are all very small in size य सभी आकार म बत छोट ह

3 They have a very dry climate इनकी जलवाय बत श ह

4 They are extended parts of the mainland व महाीप क िवारत िह ह

Correct Answer -

There are all of coral origins सभी कोरल मल क ह

What do the basalt layers of the Deccan indicate डन की बसा परत ा इिगत करती ह

1 All of the above उपरो सभी

2 Huge volcanic eruptions in the distant past दरथ अतीत म िवशाल ालामखीय िवोट

3 The immense erosional activity of the rivers निदयो की िवशाल रण गितिविध

4 The influence of weathering मौसम का भाव

Correct Answer -

Huge volcanic eruptions in the distant past दरथ अतीत म िवशाल ालामखीय िवोट

In the structure of planet Earth below the mantle the core is mainly made up of_____

पी ह की सरचना म मटल क नीच कोर म प स______ स िनिमत होती ह

1 aluminium एमीिनयम

2 silicon िसिलकॉन

3 chromium ोिमयम

4 iron लोहा

Correct Answer -

iron लोहा

One of the major Mid Oceanic Ridge is found in मख म-महासागर चोिटयो म स एक ______ म पायाजाता ह

1 Mid Pacific Ocean म शात महासागर

2 Mid Atlantic Ocean म अटलािटक महासागर

3 Mid Indian Ocean म भारतीय महासागर

4 Mid Arctic Ocean म आक िटक महासागर

68)

69)

70)

71)

Correct Answer -

Mid Atlantic Ocean म अटलािटक महासागर

Magma that reaches the Earthrsquos surface and then solidifies is called________

मा जो पी की सतह तक पचती ह और िफर ठोस हो जाती ह ________कहलाती ह

1 quartz ाटज

2 lava लावा

3 granite नाइट

4 silicates िसिलकट

Correct Answer -

lava लावा

Isotherms are the lines of equal_______ समताप रखाए समान _______की रखाए होती ह

1 pressure दाब

2 temperature तापमान

3 rainfall वषा

4 height ऊचाई

Correct Answer -

temperature तापमान

Mark the correct sequence of passes in the Western Ghats from north to south

पिमी घाटो म उर स दिण तक दर क सही अनम को िचित कर

1 Thalghat Palghat Bhorghat थलगघाट पालघाट भोरघाट

2 Thalghat Bhorghat Palghat थलघाट भोरघाट पालघाट

3 Bhorghat Thalghat Palghat भोरघाट थलघाट पालघाट

4 Palghat Bhorghat Thalghat पालघाट भोरघाट थलघाट

Correct Answer -

Thalghat Bhorghat Palghat थलघाट भोरघाट पालघाट

Which of the following does not have influence over the climate in India

ि ि ि ी ी

72)

73)

िनिलखत म स िकसका भाव भारत की जलवाय पर नही पड़ता ह

1 Ocean currents सागर की लहर

2 Nearness to equator भम रखा स िनकटता

3 Monsoons मानसन

4 Presence of Indian ocean भारतीय महासागर की उपथित

Correct Answer -

Ocean currents सागर की लहर

Which of the following cloud types has the characteristics like vertical tall narrow and puffy

िनिलखत म स िकस कार क मघो म लबवत लबी सकीण और थलता जसी िवशषताए ह

1 Cumulonimbus तफानी मघ

2 Cumulus मघ पज

3 Cirrocumulus पाभ कपासी मघ

4 Nimbostratus वषारी मघ

Correct Answer -

Cumulus मघ पज

Which of the following statement is INCORRECT about Crude Birth Rate

िनिलखत स कौन सा कथन अशोिधत ज दर क बार म सही नही ह

1 It cannot be used for comparing fertility level between two countries with different population characteristics

इसका उपयोग िविभ जनसा िवशषताओ वाल दो दशो क बीच जनन र की तलना क िलए नही िकया जा सकता ह

2 It is a standardized measure of fertility

यह जनन मता का मानकीकत उपाय ह

3 It is effected by the age-sex composition of the population

यह आबादी की आय-िलग सरचना स भािवत होता ह

4 It is expressed per 1000 population in a given geographical unit

यह िकसी दी गई भौगोिलक इकाई म ित 1000 जनसा पर िकया जाता ह

Correct Answer -

It is a standardized measure of fertility

यह जनन मता का मानकीकत उपाय ह

74)

75)

76)

77)

Which of the following state in India experienced negative decadal growth rate during 2001 to 2011census

भारत म िनिलखत म स िकस रा म वष 2001 स 2011 की जनगणना क दौरान नकाराक िगरावट दर ई

1 Tripura िपरा

2 Nagaland नागालड

3 Haryana हरयाणा

4 Odisha ओिडसा

Correct Answer -

Nagaland नागालड

Which of the following is NOT a characteristic of peninsular rivers

िनिलखत म स कौन सी िवशषता ायीपीय निदयो म नही होती ह

1 Flow through shallow valleys उथल घािटयो क माम स वाह

2 Seasonal flow मौसमी वाह

3 Little erosional activity थोड़ी कटावदार गितिविध

4 Meandering tendency often shifting their beds घमावदार वि अर अपन तटो को थानातरत करना

Correct Answer -

Meandering tendency often shifting their beds घमावदार वि अर अपन तटो को थानातरत करना

Which of the following gases in the atmosphere absorbs heat from the Sunrsquos radiation and the Earthssurface

वायमडल म िनिलखत म स कौन सी गस सय क िविकरण और पी की सतह स ऊा को अवशोिषत करती ह

1 Neon िनयॉन

2 Carbon dioxide काबन डाइऑाइड

3 Argon आगन

4 Nitrogen नाइट ोजन

Correct Answer -

Carbon dioxide काबन डाइऑाइड

Which of the following kind of settlement pattern is found at the confluence of rivers

ि ि ि ि ो

78)

79)

80)

िनिलखत म स िकस कार का वथापन पटन निदयो क सगम पर पाया जाता ह

1 Triangular Paern िकोणीय पटन

2 Circular or Semi-Circular Paern परप या अध-परप पटन

3 Nebular Paern नबलर पटन

4 Star ndashShaped Paern ार-आकार का पटन

Correct Answer -

Triangular Paern िकोणीय पटन

Which one was not the objective of the Biosphere Reserve Projects launched by the UNESCO

यनो ारा श की गई सरित जवमडल परयोजनाओ का उ इनम स कौन सा नही था

1 To promote teaching and research िशण और अनसधान को बढ़ावा दना

2 To make agriculture sustainable किष को दीघकािलक बनाना

3 To conserve ecosystems पारथितक त को सरित करना

4 To conserve genetic diversity for a longtime लब समय तक अनवािशक िविवधता को सरित करना

Correct Answer -

To make agriculture sustainable किष को दीघकािलक बनाना

Which region of the Earth surface receives the highest amount of insulation

पी सतह का कौन सा तापावरोधन की उतम माा ा करता ह

1 Land mass थलखड

2 Savannah region सवाना

3 Water bodies जल िनकाय

4 Tropical desert उकिटबधीय रिगान

Correct Answer -

Tropical desert उकिटबधीय रिगान

Which one of the following is not a biodiversity hotspot

िनिलखत म स कौन सा जव िविवधता का म जगह नही ह

1 Eastern Himalaya पव िहमालय

2 Eastern Ghats पव घाट

81)

82)

83)

3 Indo-Myanmar भारत-ामार

4 Westerm Ghats पिमी घाट

Correct Answer -

Eastern Ghats पव घाट

Which one of the following is NOT a part of the World Network of Biosphere Reserves based on theUNESCO Man and Biosphere Programme

यनो मन और बायोीयर कायम क आधार पर िनिलखत म स कौन बायोीयर रजव क िव नटवक कािहा नही ह

1 Gulf of Mannar मार की खाड़ी

2 Seshachalam शषाचलम

3 Sunderban सदरबन

4 Nilgiri नीलिगर

Correct Answer -

Seshachalam शषाचलम

Which one of the following is an example of ldquodesert vegetationrdquo

िनिलखत म स कौन मथलीय वनित का एक उदाहरण ह

1 Mosses and lichens दलदल और शवाल

2 Temperate grassland समशीतो घास क मदान

3 Coniferous forest शकधारी वन

4 Acacia and cactus एकािसया और कस

Correct Answer -

Acacia and cactus एकािसया और कस

Which one of the following reflects more sunlight िनिलखत म स कौन सा सय की रोशनी को अिधकपरावितत करता ह

1 Paddy crop land धान फसल भिम

2 Land covered with fresh snow ताजा बफ स आािदत भिम

3 Sand desert रतीली रिगान

4 Prairie land यरी भिम

84)

85)

86)

87)

Correct Answer -

Land covered with fresh snow ताजा बफ स आािदत भिम

Which layer of the atmosphere is in contact with the surface of the earthrsquos oceans

वायमडल की कौन सी परत पी क महासागरो की सतह क सपक म ह

1 Stratosphere समताप मडल

2 Mesosphere म मडल

3 Hydrosphere जलमडल

4 Troposphere ोभ मडल

Correct Answer -

Troposphere ोभ मडल

Mediterranean Sea is a border of which of the following countries भम सागर िनिलखत दशो म सिकसकी सीमा ह

1 None of these इनम स कोई नही

2 Iraq इराक

3 Lebanon लबनान

4 Jordan जॉडन

Correct Answer -

Lebanon लबनान

Benguela ocean currents are found along which coast बगएला महासागर धाराए िकस तट क साथ पायीजाती ह

1 East Coast of South America दिण अमरका क पव तट

2 East Coast of Africa अीका क पव तट

3 West Coast of South America दिण अमरका क पिमी तट

4 West Coast of Africa अीका क पिमी तट

Correct Answer -

West Coast of Africa अीका क पिमी तट

88)

89)

90)

Due to tension a block of land on one side being pushed up or upthrown relative to the downthrown blockis referred as

तनाव क कारण नीच फ क ए खड क साप भिम का एक खड एक ओर स ऊपर धकला जाता ह या ऊपर की ओरफ का जाता ह यह _____ क प म सदिभत ह

1 Thrust fault प श

2 Normal fault सामा श

3 Reverse fault म श

4 Strike slip fault नितलब सपण श

Correct Answer -

Normal fault सामा श

Inter-tropical doldrums is a zone of ______ अतर-उकिटबधीय डोलड ______ का एक ह

1 Frontolysis टोलायिसस

2 Convergence अिभसरण

3 Inter-tropical divergence zone अतर-उकिटबधीय िवचलन

4 Local wind थानीय वाय

Correct Answer -

Convergence अिभसरण

The Horse Latitudes are regions located at about _____ north and south of the equator

हॉस अाश भम रखा क उर और दिण म लगभग _____ पर थत ह

1 30ndash60 degree Latitude 30-60 िडी अाश

2 0ndash5 degree Latitude 0-5 िडी अाश

3 30 degree Latitude 30 िडी अाश

4 60ndash90 degree Latitude 60-90 िडी अाश

Correct Answer -

30 degree Latitude 30 िडी अाश

Generally evaporation is high over which part of the Earth

आम तौर पर पी क िकस भाग पर वाीकरण अिधक होता ह

1 Equatorial maritime भमवत समीय ी ी

91)

92)

2 Equatorial continental भमवत महाीपीय

3 Polar maritime वीय समीय

4 Polar continental वीय महाीपीय

Correct Answer -

Equatorial maritime भमवत समीय

A very high temperature during summer in north western India leads to what type of climaticcondition in south

उर पिमी भारत म गम क दौरान बत अिधक तापमान होन क कारण दिण म िकस कार की जलवाय थितउ करता ह

1 Depression over arabian sea अरब सागर पर अवनमन

2 Failure monsoon मानसन िवफलता

3 Successful monsoon मानसन सफलता

4 Cyclones चवात

Correct Answer -

Successful monsoon मानसन सफलता

Lightning and thunder are the resultant effect when तिड़त और गजन परणामी भाव ह जब

1 Two massive clouds hit powerfully each other first lightning is produced and later sound is produced

दो बड़ बादल एक दसर स शशाली ढग स टकरात ह पहल आकाशीय िवदयत उ होता ह और बाद म िन उहोती ह

2 Two massive clouds come into contact with the powerful wind collision this results into first sound and thenlightning

दो बड़ बादल शशाली पवन सघ क सपक म आत ह इसका परणामप पहल िन और िफर आकाशीय िवदयतउ होता ह

3 None of the above उपरो म स कोई भी नही

4 A high density cloud contains positively and negatively charged electric ions and when this interacts light andsound are simultaneously produced

एक उ घन बादल म धनाक और ऋणाक आविशत िवदयत आयन होत ह और जब यह परर भाव डालत ह तोकाश और िन एक साथ उािदत होती ह

Correct Answer -

A high density cloud contains positively and negatively charged electric ions and when this interacts light andsound are simultaneously produced

औ ि ि ो औ ो

93)

94)

95)

एक उ घन बादल म धनाक और ऋणाक आविशत िवदयत आयन होत ह और जब यह परर भाव डालत ह तोकाश और िन एक साथ उािदत होती ह

Doon Valley is able to grow rice because दन घाटी चावल उगान म सम ह ोिक

1 Other crops cannot be grown वहा अ फसलो को उगाया नही जा सकता ह

2 People in the valley are rice eaters घाटी म लोग चावल खान वाल ह

3 There is a huge export demand of rice वहा चावल की भारी िनयात माग ह

4 It has warm summer and snow melt waters for irrigation

वहा गिमया गम होती ह िसचाई क िलए बफ का िपघला आ पानी होता ह

Correct Answer -

It has warm summer and snow melt waters for irrigation

वहा गिमया गम होती ह िसचाई क िलए बफ का िपघला आ पानी होता ह

CANCELLED

In the geological time scale the Mesozoic Era DOES NOT contains which of the following periods

भगभय समय पमान पर मजीवी यग म िन कालो म स कौन नही ह

1 Triassic ट ाइऐिसक

2 Jurassic जरिसक

3 Cretaceous चाकमय

4 Carboniferous काबनी

Correct Answer -

Carboniferous काबनी

96)

1 P-3 Q-4 R-2 S-1

2 P-3 Q-4 R-1 S-2

3 P-3 Q-4 R-1 S-2

4 P-4 Q-3 R-2 S-1

Correct Answer -

P-4 Q-3 R-2 S-1

1 P-3 Q-1 R-4 S-2

2 P-3 Q-4 R-1 S-2

3 P-3 Q-2 R-4 S-1

97)

98)

4 P-2 Q-1 R-4 S-3

Correct Answer -

P-3 Q-4 R-1 S-2

1 P-3 Q-1 R-4 S-2

2 P-2 Q-3 R-4 S-1

3 P-2 Q-1 R-3 S-4

4 P-4 Q-2 R-1 S-3

Correct Answer -

P-3 Q-1 R-4 S-2

99)

100)

1 P-3 Q-2 R-4 S-1

2 P-1 Q-2 R-3 S-4

3 P-2 Q-3 R-1 S-4

4 P-4 Q-3 R-2 S-1

Correct Answer -

P-2 Q-3 R-1 S-4

ldquoHuman geography is the study of changing relationship between the unresting man and the unstableearthrdquo was defined by

lsquolsquoमानव भगोल ाकल आदमी और अथर पी क बीच सबध परवतन का अयन हrdquo ______ ारा परभािषत िकया गयाथा

1 J Brunches ज चस

2 EC Semple ईसी सल

3 HJ Mackinder एच ज मिकदर

4 PV Blache पीवी च

Correct Answer -

EC Semple ईसी सल

Sedimentary rocks are finally and ultimately derived from the____________

अवसादी चान अततः ________ स ा की जाती ह

1 action of earth movements पी की गितिविधयो

2 marine deposit समी िनप

3 weathering of metamorphic rocks पातरत चानो क अपय

4 weathering of igneous rocks आय चानो क अपय

Correct Answer -

weathering of igneous rocks आय चानो क अपय

Page 2: High School Teacher Eligibility Test- BOARD PROFESSIONAL ...peb.mp.gov.in/results/RESULT_18/HST_RES18/Final_anwser_key/HST… · M a ndl a / मंड ल ... Under the Madhya Pradesh

4)

5)

6)

7)

अिच

बार-बार कही गई उ ___________ कहलाती ह

1 पनोकती

2 पन

3 पनकती

4 पनरोकती

Correct Answer -

पन

lsquoपािन परात को हाथ छयो निह ननन क जल सो पग धोयोrsquo म कौन सा अलकार ह

1 उपमा अलकार

2 पक अलकार

3 अितशयो अलकार

4 ष अलकार

Correct Answer -

अितशयो अलकार

सीस का श प या ह

1 शीस

2 सीश

3 शीत

4 शीश

Correct Answer -

शीश

lsquoआड़ हाथो लनाrsquo महावर का अथ ___________ ह

1 रह खलना

2 मरन क करीब होना

3 खरी-खोटी सनाना

4 अत मख

8)

9)

10)

Correct Answer -

खरी-खोटी सनाना

lsquolearningrsquo श द क िलए िह दी भाषा म उिचत पारभािषक श दावली का चयन कर

1 रटना

2 अिधगम

3 िवणता

4 सीखाना

Correct Answer -

अिधगम

चादी का पयायवाची ________ ह

1 धनरस

2 उदक

3 खनक

4 चहास

Correct Answer -

चहास

ldquoकर मसर नाचत नगन लख हलधर को ागrdquo

हिस- हिस गोपी िफर हस मन िपय सी भाग II - म कौन सा रस ह

1 हा रस

2 शात रस

3 भ रस

4 वा रस

Correct Answer -

हा रस

Topic- GENERAL ENGLISH

1)

2)

3)

Choose the option that best expresses the meaning of the highlighted idiomphrase

Anil How did the election go

Sunil Very exciting till the last minute Our candidate proved to be a dark horse

1 The candidate is not popular but won

2 His candidate lost the election though he was expected to win

3 The candidate backed out at the last minute

4 The candidate is popular and won

Correct Answer -

The candidate is not popular but won

A sentence with an underlined word is given below Choose the correct option which is closest in meaning tothe underlined word

The fruit of ones toil helps others to continue with the mission

1 achievement

2 hard work

3 attempts

4 sufferings

Correct Answer -

hard work

Out of the following options choose the correct form of verb that is in agreement with the subject for thegiven sentence

lsquoA Tale of Two Citiesrsquo ______ a wonderful book to read

1 are

2 were

3 is

4 have been

Correct Answer -

is

4)

5)

6)

Out of the following options choose the most appropriate tense that best fills in to complete the givensentence

This year our cricket team _________ all the matches played on the Indian soil

1 will win

2 was winning

3 wins

4 has been winning

Correct Answer -

has been winning

Choose the appropriate adverb clause from the options to complete the given sentence

He was very pleased _____

1 which you passed

2 which you had passed

3 that you passed

4 that you who passed

Correct Answer -

that you passed

Out of the following four options choose the incorrect sentence

1 Rohit is a charmer and you can find him selling newspapers on M G Road from 800 am every morning

2 Diabetes a leading cause of heart problems does not allow people to make it to the hospital in time after a heartattack

3 Symbiosis Institute is Indias premier approved business school

4 Nehrus vision continued to light up the entire nation even today

Correct Answer -

Nehrus vision continued to light up the entire nation even today

7)

8)

1)

Out of the following options choose the most appropriate usage to fill in the blank

I live a few yards ______ the bus stop

1 off

2 from

3 outside

4 away

Correct Answer -

from

Arrange the sentences P Q R and S to form a logical sequence between sentences 1 and 6

1 Jurors do not have to be intelligentP In 1933 in a sheep-stealing case in Wales two of the jurors were totally unable to understand Englishbut no one found that out until after the trialQ There is no test no examination and no enquiry about their abilitiesR The convicted found this out and complained about it to the Court of Criminal AppealS No one even takes the trouble to find out whether they can understand the English language6 Still the Court of Appeal felt that it must refuse to interfere

1 QPRS

2 SQRP

3 PRQS

4 QSPR

Correct Answer -

QSPR

Topic- GENERAL KNOWLEDGE

Which country will host the 2022 Football World Cup 2022 फटबॉल िव कप की मजबानी करन वाला दशकौन होगा

1 America अमरका

2 Canada कनाडा

3 Qatar कतर

4 Mexico मको

2)

3)

4)

5)

Correct Answer -

Qatar कतर

According to sex ratio trends of Census 2011 which district of Madhya Pradesh has been ranked numberOne

जनगणना 2011 म िलग अनपात क झानो क अनसार म दश क िकस िजल को नबर एक थान िदया गया ह

1 Seoni िसओनी

2 Ratlam रतलाम

3 Balaghat बालाघाट

4 Mandla मडला

Correct Answer -

Balaghat बालाघाट

The folk dance form called lsquoMatkirsquo is prevalent in which of the following regions of Madhya Pradesh

मटकी नामक लोक न प म यदश क िनिलखत म स िकस म चिलत ह

1 Bundelkhand बदलखड

2 Malwa मालवा

3 Vindhyachal िवाचल

4 Baghelkhand बघलख

Correct Answer -

Malwa मालवा

Who appoints the Chairman and the Members of the Union Public Service Commission

सघ लोक सवा आयोग क अ और सदो की िनय कौन करता ह

1 Chief Justice of India भारत क म य यायधीश

2 Prime Minister धान मी

3 President रा ट पित

4 Chief Election Commissioner म य चनाव आय त

Correct Answer -

President रा ट पित

6)

7)

8)

Under the Madhya Pradesh Ladli Laxmi Yojana the beneficiary is paid the total amount on attaining theage of ___ मदश लाडली ली योजना क तहत लाभाथ की आय ___ वष परी होन पर कल रािश का भगतान िकयाजाता ह

1 18

2 21

3 23

4 16

Correct Answer -

21

India has won the gold medal in hockey ______times at the Olympics

ओलिपक खलो म भारत न हॉकी म ण पदक _____ बार जीता ह

1 9

2 7

3 8

4 6

Correct Answer -

8

Where is the Madhya Pradesh Tribal Museum located at

म दश जनजातीय सहालय कहा थत ह

1 Indore इदौर

2 Jhabua झबआ

3 Khargone खरगोन

4 Bhopal भोपाल

Correct Answer -

Bhopal भोपाल

The fictional boy character of the lsquoThe Jungle Bookrsquo by author Rudyard Kipling was

लखक डयाड िकपिलग ारा lsquoद जगल बकrsquo का कािनक बालक चर था

1 Kaa का

2 Mowgli मोगली

9)

10)

1)

3 Shere Khan शर खान

4 Bagheera बघीरा

Correct Answer -

Mowgli मोगली

The recipient of which one of the following awards given by Madhya Pradesh Government gets the highest cashprize

िनिलखत परारो म स िकसक ाकता को म दश सरकार ारा उतम नकद परार िदया जाता ह

1 Mahatma Gandhi award महाा गाधी परार

2 Shikar Award िशखर परार

3 Sharad Joshi Award शरद जोशी परार

4 Kishore Kumar Award िकशोर कमार परार

Correct Answer -

Mahatma Gandhi award महाा गाधी परार

Who among the following was the Trinidadian writer of Indian descent

िनिलखत म स भारतीय मल क ििनदािदयन लखक कौन थ

1 RK Narayan आर क नारायण

2 Anita Desai अनीता दसाई

3 Rudyard Kipling डयाड िकपिलग

4 VS Naipaul वीएस नायपॉल

Correct Answer -

VS Naipaul वीएस नायपॉल

Topic- GENERAL REASONING

Find the missing number ल त स या ात कर

4 196 16 169 ____ 144 64

1 81

2 36

3 32

4 21

2)

3)

4)

Correct Answer -

36

Find the average औसत ात कर

25 35 45 55 65 amp 75

1 48

2 50

3 47

4 49

Correct Answer -

50

The below series uses a sequence of alphabets and numbers Identify the incorrect combination

नीच दी गई खला अरो एव साओ का अनम योग करती ह गलत सयोजन पहचान

(i) FT85DF77ER

(ii) FT85DF77ER

(iii) FT85DE77ER

(iv) FT85DF77ER

1 ii

2 iv

3 i

4 iii

Correct Answer -

iii

A and B together can complete a piece of work in 10 days and B alone can complete the same work in 20days In how many days can A alone complete the work

A और B एक साथ काम क एक भाग को 10 िदनो म समा त करत ह और B अकल उसी काम को 20 िदनो म समा त करता हिकतन िदनो म A अकल उस काम को समा त कर सकता ह

1 20 days 20 िदन

2 30 days 30 िदन

3 10 days 10 िदन

4 25 days 25 िदन

5)

6)

Correct Answer -

20 days 20 िदन

Which one of the following four addresses is NOT EXACTLY same as the one given below

िनिलखत िदए गए चार पतो म स नीच िदए गए पत क ठीक समान कौन सा एक नही ह

Ground Floor 59 Goulburn St

Sydney NSW 2000 Australia

+61 (02) 8987 3700

(i) Ground Floor 59 Goulburn St

Sydney NSW 2000 Australia

+61 (02) 8987 3700

(ii) Ground Floor 59 Goulburn St

Sydney NSW 2000 Australia

+61 (02) 8987 3700

(iii) Ground Floor 59 Govlburn St

Sqdney NSW 2000 Australia

+61 (02) 8987 3700

(iv) Ground Floor 59 Goulburn St

Sydney NSW 2000 Australia

+61 (02) 8987 3700

1 ii

2 iv

3 i

4 iii

Correct Answer -

iii

In a group of 75 people 32 of them like cold drink 56 of them like general water and each person likesat least one of the two drinks How many people like both

75 लोगो क समह म उनम स 32 को को िड क पसद ह उनम स 56 को सामा जल पसद ह और क को दो म स कम स कम एक पय पसद ह िकतन लोगो को दोनो पसद ह

1 11

2 13

7)

1)

2)

3 12

4 14

Correct Answer -

13

If A = 1 FAT = 27 then FAITH =

यिद A = 1 FAT = 27 तो FAITH =

1 41

2 40

3 42

4 44

Correct Answer -

44

Topic- PEDAGOGY

CANCELLED

Basic education became a major initiative because of the efforts of

ाथिमक िशा िन न क यास स एक मह वपण कायम बन गई

1 Dr Radhakrishnan डॉ राधाक णन

2 Dr Rajendra Prasad डॉ राज साद

3 Mahatma Gandhi महा मा गाधी

4 Rabindranath Tagore रबी नाथ टगोर

Correct Answer -

Mahatma Gandhi महा मा गाधी

The ldquoclause of phraserdquo is an unit of perception of लॉज ऑफ ज िन न क धारणा क एक इकाई ह

1 Image छिव

2 Concept अवधारणा

3 Language भाषा

4 Thought िवचार

Correct Answer -

3)

4)

5)

6)

Language भाषा

Which method is most suitable to study communication process among students

छाो क बीच सचार िया का अयन करन क िलए कौन सी िविध सबस उपय ह

1 Case Study मामल का अयन (कस टडी)

2 Systematic Observation वथत अवलोकन

3 Experimental Method योगाक िविध

4 Introspection आ-िनरीण (इट ोस शन)

Correct Answer -

Systematic Observation वथत अवलोकन

Genes in a human being are located in मानवो म जीन इनम थत होता ह

1 cytoplasm कोिशका (साइटो ला म)

2 ribosomes राइबोसोम

3 cell membranes कोिशका िझी

4 chromosomes गणस (ोमोसोम)

Correct Answer -

chromosomes गणस (ोमोसोम)

With smaller classes teachers are much more able to ____________

छोटी काओ क साथ िशक ____________ म अिधक सम होत ह

1 Go slow while teaching िशण क दौरान धीमी गित स जान

2 Narrate more personal experiences अिधक गत अनभवो को बतान

3 Adapt instruction to individual differences among students छाो क बीच गत मतभदो क िलए अनकल िनदश दन म

4 Make use of the extra space for extra-curricular activities पातर गितिविधयो क िलए अितर जगह का उपयोग करन

Correct Answer -

Adapt instruction to individual differences among students छाो क बीच गत मतभदो क िलए अनकल िनदश दन म

While engaging in a task the child gets bored This is a sign of

एक काम म होन पर बा ऊब जाता ह यह िन का सकत ह

1 the task requiring a professional approach काय को ावसाियक िकोण की आवकता ह

7)

8)

9)

2 the task becoming mechanically repetitive काय यािक प स दोहराव वाला ह

3 the child not being intelligent बा बमान नही ह

4 the child being incapable of learning बा सीखन म असमथ ह

Correct Answer -

the task becoming mechanically repetitive काय यािक प स दोहराव वाला ह

Educational Psychologists are more concerned with the learning in __________

शिणक मनोवािनक __________ म अिधगम क साथ अिधक िचितत होत ह

1 Formal environment औपचारक वातावरण

2 Informal environment अनौपचारक वातावरण

3 Physical environment भौितक वातावरण

4 Social environment सामािजक वातावरण

Correct Answer -

Formal environment औपचारक वातावरण

Special needs education is the type of education िवशष ज़रतो वाली िशा वह िशा होती ह जो

1 Given to person with disabilities अम य को दी जाती ह

2 Given to people from remote areas दर थ ो क लोगो को दी जाती ह

3 Provided to intelligent people बमान लोगो को दी जाती ह

4 Established by colonial masters औपिनविशक मखयाओ ारा थािपत की गई ह

Correct Answer -

Given to person with disabilities अम य को दी जाती ह

The Stanford-Binet scale of intelligence was first published in the year

ब क नफोड-िबनट कल को इस वष म पहली बार कािशत िकया गया था

1 1916

2 1903

3 1908

4 1900

Correct Answer -

1916

10)

11)

12)

13)

The term lsquofictional finalismrsquo was propounded by

पद lsquoकत योजनवादrsquo (िफ शनल फाइनिल म) इनक ारा ितपािदत िकया गया था

1 Skinner नर

2 Freud ायड

3 Adler एडलर

4 Pavlov पावलोव

Correct Answer -

Adler एडलर

Rational Emotive Behavior Therapy was propounded by

तक सगत भावनाक वहार थरपी िन क ारा ितपािदत की गई थी

1 Carl Jung काल यग

2 Carl Rogers काल रोजस

3 Aaron Beck आरोन बक

4 Albert Ellis अट एिलस

Correct Answer -

Albert Ellis अट एिलस

Which of the following indicates the quality of education in a school

िनिलखत म स या एक कल म िशा की गणव ता का सकतक ह

1 Text-books and Teaching-learning material पा-प तक तथा िशण व सीखन स सबिधत सामी

2 Infrastructural facilities at the school कल म आधारभत ढाच स सबिधत सिवधाए

3 Student achievement level िवािथयो का उपल तर

4 Classroom systems का की यव था

Correct Answer -

Student achievement level िवािथयो का उपल तर

Which of the following teachers can be identified with authoritarian teaching style

िनिलखत म स कौन सा िशक अिधकारवादी िशण शली क साथ पहचाना जा सकता ह

1 Laissez-faire teacher अब ध िशक

14)

15)

2 Democratic teacher लोकतीय िशक

3 Indifferent teacher िन प िशक

4 Direct instruction teacher िनदश िशक

Correct Answer -

Direct instruction teacher िनदश िशक

Who was the pioneer of classical conditioning

िचरितित ानकलन ( लािसकल कडीशिनग) क वतक कौन थ

1 Skinner नर

2 Pavlov पावलोव

3 Kohler कोहलर

4 Freud ायड

Correct Answer -

Pavlov पावलोव

Which of the following are true with reference to short term memory

1 Limited capacity

2 Brief storage of information

3 Unlimited capacity

4 Duration of storage less than twenty seconds

अ पकािलक मित क सदभ म िन न म स या स य ह

1 सीिमत मता

2 सचना का सि त भडारण

3 असीिमत मता

4 बीस सकड स कम भडारण की अविध

1 3 and 4 3 और 4

2 2 and 4 2 और 4

3 13 and 4 13 और 4

4 12 and 4 12 और 4

Correct Answer -

12 and 4 12 और 4

1)

2)

3)

4)

Topic- GEOGRAPHY

Which of the following ranges of population size is used to define Class-III city by Indian Census

भारतीय जनगणना ारा वग-III शहर को परभािषत करन क िलए िनिलखत म स िकस रज क जनसा आकारका उपयोग िकया जाता ह

1 20 000 to 49999 20 000 स 49999

2 30000 to 59999 30000 स 59999

3 24000 to 54999 24000 स 54999

4 50000 to 99999 50000 स 99999

Correct Answer -

20 000 to 49999 20 000 स 49999

Carbonaceous rocks which produce coal and oil belong to the category of rocks called_______

कोयल और तल का उादन करन वाली काबनय चान ______ नामक चानो की णी स सबिधत ह

1 metamorphic पातरत

2 sedimentary अवसादी

3 inorganic अजिवक

4 igneous आय

Correct Answer -

sedimentary अवसादी

The ruhr-complex is a major industrial centre in र-परसर िन का एक मख औोिगक क ह

1 North America उरी अमरका

2 Russia स

3 Germany जमनी

4 Europe यरोप

Correct Answer -

Germany जमनी

The term lsquoRegurrsquo refers to श lsquoरगरrsquo ______ स सबिधत ह

1 Deltaic alluvial soils डा जलोढ़ िमी

ि ी

5)

6)

2 Laterite soils लटराइट िमी

3 Red and yellow soils लाल और पीली िमी

4 Black cotton soils काली कपास िमी

Correct Answer -

Black cotton soils काली कपास िमी

Read the given statements and answer which of the following options isare correct

1 Lower the pressure greater the atmospheric disturbance

2 Air move from higher to low pressure

िदए गए कथन को पढ़ और उर द िक िन म स कौन सास िवक सही ह

1 िजतना दाब कम होगा वायमडलीय बाधाए उतनी अिधक होगी

2 वाय उ स िन दाब की ओर गित करती ह

1 Both Statements 1 and 2 are correct दोनो कथन 1 और 2 सही ह

2 Both Statements 1 and 2 are wrong दोनो कथन 1 और 2 गलत ह

3 Statement 1 is wrong and only Statement 2 is correct कथन 1 गलत ह और कवल कथन 2 सही ह

4 Statement 1 is correct and Statement 2 is wrong कथन 1 सही ह और कथन 2 गलत ह

Correct Answer -

Both Statements 1 and 2 are correct दोनो कथन 1 और 2 सही ह

CANCELLED

1 4 1 2 and 3 4 1 2 और 3

2 4 1 3 and 2 4 1 3 और 2

3 1 4 2 and 3 1 4 2 और 3

4 1 4 3 and 2 1 4 3 और 2

Correct Answer -

7)

8)

9)

1 4 3 and 2 1 4 3 और 2

CANCELLED

Karl Pearsonrsquos correlation co-efficient is काल िपयसन का सहसबध गणाक ह

1 Arithmec mean समार मा

2 Geometric mean गणोर मा

3 None of these इनम स कोई नही

4 Harmonic mean हराक मा

Correct Answer -

Geometric mean गणोर मा

CANCELLED

ldquoEach day is more or less the same the morning is clear and bright with a sea breeze as the Sun climbshigh in the sky heat mounts up dark clouds form then rain comes with thunder lighting But rain is soonoverrdquo Which of the following regions is described in the above passage

ldquoक िदन समान स अिधक या कम होता ह सम की हवा क साथ और उल सबह होती ह जस सयआकाश म ऊचा चढ़ता ह गम बढ़ जाती ह काल बादल बनत ह िफर िबजली क साथ बारश आती ह लिकनबारश जी ख हो जाती हlsquorsquo उपरो पा म िनिलखत म स िकन ो का वणन िकया गया ह

1 Equatorial भमरखीय

2 Equatorial भमरखीय

3 Savannah सवाना

4 Mediterranean आातरक (भमसागरीय)

5 Mediterranean आातरक (भमसागरीय)

6 Monsoon मानसन

7 Monsoon मानसन

Correct Answer -

Equatorial भमरखीय

Equatorial भमरखीय

CANCELLED

In which epoch of the geological history of the Earth dinosaurs reached their largest size

पी क भगभय इितहास क िकस यग म डायनासोर अपन सबस बड़ आकार तक पचि

10)

11)

1 Triassic ट ायिसक

2 Jurassic जरािसक

3 Cretaceous ीटशस

4 Permian पिमयन

Correct Answer -

Jurassic जरािसक

CANCELLED

A spring tide would occur in which of the following conditions

िनिलखत म स िकन थितयो म ार-भाटा आता ह

1 When the Sun Moon and Earth are in a straight line

जब सय चमा और पी एक सीधी रखा म होत ह

2 When the Moon and Earth are in right angle to each other

जब चमा और पी एक-दसर क दािहन कोण म होत ह

3 When the Earth and Moon are in right angle to the Sun

जब पी और चमा सय क दािहन कोण म होत ह

4 When the Sun and Moon are in right angle to each other

जब सय और चमा एक-दसर क दािहन कोण म होत ह

Correct Answer -

When the Sun Moon and Earth are in a straight line

जब सय चमा और पी एक सीधी रखा म होत ह

CANCELLED

An observe on the Earthrsquos surface always sees the same face of the moon because

एक पयवक को पी की सतह स हमशा चाद का एक ही फलक िदखाई दता ह ोिक

1 Its path of revolution around the earth is the same as that of the earth around the Sun

इसका पी क चारो ओर परमण का माग सय क चारो ओर पी क समान ही ह

2 Its period of revolution around the Earth is the same as its period of rotation around its own axis

इसकी पी क चारो ओर परमण की अविध उसकी अपनी धरी क चारो ओर घणन की अविध क समान ह

3 Its period of rotation is the same as that of the Earth इसकी घणन की अविध पी क समान ह

ी ि ी ी

12)

13)

14)

4 Its direct of rotation is the same as that of Earth घणन की िदशा पी क समान ही ह

Correct Answer -

Its period of revolution around the Earth is the same as its period of rotation around its own axis

इसकी पी क चारो ओर परमण की अविध उसकी अपनी धरी क चारो ओर घणन की अविध क समान ह

CANCELLED

The pebbles that are faceted by the sand-blasting and shaped polished by the wind abrasions are known as

पवन अपघषन ारा पॉिलश रत-िवोिटत और साच म ढला ककड़ ___________ क प म जाना जाता ह

1 Dreikanter िकोणक

2 Pediments िकोिनका

3 Inselberg इलबग

4 Dunes टीबा

Correct Answer -

Dreikanter िकोणक

CANCELLED

Astronomical unit is the average distance between खगोलीय इकाई ______ क बीच की औसत दरी ह

1 Earth and Mars पी और मगल

2 Earth and mercury पी और बध

3 Earth and moon पी और चमा

4 Earth and Sun पी और सय

Correct Answer -

Earth and Sun पी और सय

During cold weather season in the northern plains there will be an inflow of cyclonic disturbancesfrom the _________ directions

शीत मौसम क दौरान उरी मदानी इलाको म _________ िदशाओ स चवात सबधी गड़बड़ी का अतवाह होगा

1 East and Northwest पव और उरपिम

2 East and Northeast पव और पवर

3 West and East पिम और पव

ि औ ि

15)

16)

17)

4 West and Northwest पिम और उरपिम

Correct Answer -

West and Northwest पिम और उरपिम

During an earth quake the velocity of the body waves will________ along with the increase in densityof the material it is passing through

भकप क दौरान लहरो क ऊपरी भाग का वग घन म व क साथ-साथ ________ जो इसस गजरन वाली वको आग बढाएगी

1 not change नही बदलगा

2 increase initially and then decrease शआत म बढ़गा और िफर घटगा

3 increase बढ़गा

4 decrease घटगा

Correct Answer -

increase बढ़गा

The Clouded Leopard National park is situated in which of the following states

िनिलखत म िकस रा म धिमल तदआ रा ीय उान (ाउडड लपड नशनल पाक ) थत ह

1 Tripura िपरा

2 Uttar Pradesh उर दश

3 Assam असम

4 Mizoram िमजोरम

Correct Answer -

Tripura िपरा

Usually the land surfaces are heated more quickly than the water surfaces because _____________

आम तौर पर जल सतहो की तलना म भिम सतह अिधक तजी स गम होती ह ोिक _____________ ह

1 the specific heat of water is higher than land पानी की िविश ऊा भिम स अिधक

2 the specific heat of water is lesser than land पानी की िविश ऊा भिम स कम होती

3 the latent heat of water is higher than the land पानी की अतिनिहत ऊा भिम स अिधक

4 the land reflects more heat radiation than water भिम पानी की तलना म अिधक ऊा क िविकरण को पराविततकरती

18)

19)

20)

21)

Correct Answer -

the specific heat of water is higher than land पानी की िविश ऊा भिम स अिधक

The longest shore-line is along the state of सबस लबी समतटीय रखा िन रा क साथ ह

1 Maharashtra महारा

2 Orissa उड़ीसा

3 Kerala करल

4 Gujarat गजरात

Correct Answer -

Gujarat गजरात

The position when the Earth is farthest from the Sun is known as

जब पी सय स सबस दर होती ह तो उस थित को िन नाम स जाना जाता ह

1 Perihelion उपसौर

2 Vernal Equinox बसत िवषव

3 Aphelion अपसौर

4 Autumnal Equinox शराल िवषव

Correct Answer -

Aphelion अपसौर

The seasonal reversal of winds is the typical characteristic of

हवाओ का मौसमी परवतन ______ की सामा िवशषता ह

1 Mediterranean climates only कवल भमसागरीय जलवाय

2 All of the above climates उपय सभी मौसम

3 Monsoon climate only कवल मानसन जलवाय

4 Equatorial climate only कवल भमरखीय जलवाय

Correct Answer -

Monsoon climate only कवल मानसन जलवाय

In _________ rocks the minerals will occurs in beds or layers

ो ि ो ो

22)

23)

24)

______ चानो म खिनज तल या परतो म होत ह

1 metamorphic कायातरत

2 igneous and metamorphic आय और कायातरत

3 igneous आय

4 sedimentary अवसादी

Correct Answer -

sedimentary अवसादी

Black soil is ideal for the cultivation of cotton as कपास की खती क िलए काली िमी आदश ह ोिक

1 Its colour is black यह काली होती ह

2 It is found on plateau regions यह पठार ो म पायी जाती ह

3 It is made up of lava यह लावा स बनी होती ह

4 It can retain moisture यह नमी को बरकरार रख सकती ह

Correct Answer -

It can retain moisture यह नमी को बरकरार रख सकती ह

The National Survey and Mapping Organization of the country works under the Department of___________

दश का रा ीय सवण और मानिचण सगठन ___________ िवभाग क अतगत काय करता ह

1 Space अतर

2 Science and Technology िवान और तकनीक

3 Culture सित

4 Tourism पयटन

Correct Answer -

Science and Technology िवान और तकनीक

Palk strait separates India from पाक जलडमम भारत स _____ को अलग करता ह

1 Pakistan पािकान

2 Andaman Island अडमान ीप

3 China चीन

25)

26)

27)

4 Sri Lanka ीलका

Correct Answer -

Sri Lanka ीलका

Which among the following state is the major producer of Bauxite in India

िनिलखत म स कौन सा रा भारत म बॉाइट का मख उादक ह

1 Madhya Pradesh मदश

2 Rajasthan राजथान

3 Goa गोवा

4 Orissa उड़ीसा

Correct Answer -

Orissa उड़ीसा

Which of the following states DOES NOT share border with Chhattisgarh

िनिलखत म स कौन सा रा छीसगढ़ क साथ सीमा साझा नही करता ह

1 Telangana तलगाना

2 Uttar Pradesh उर दश

3 Bihar िबहार

4 Andhra Pradesh आ दश

Correct Answer -

Bihar िबहार

Which of the following statements is INCORRECT with respect to parallels of latitudes

अाश क समानातरो क सबध म िन निलखत म स कौन सा कथन गलत ह

1 A line joining places of equal latitude is known as parallel of largest

समान अाश क थानो को जोड़न वाली रखा को िवशालतम क समानातर क प म जाना जाता ह

2 They stat from equator and run parallels to it

व भम रखा स ारभ होत ह और इसक समानातर चलत ह

3 All parallels are equal in length सभी समातर लबाई म समान ह

4 All parallels are drawn as circles on the globe ोब पर सभी समानातर वो क प म खीच जात ह

28)

29)

30)

31)

Correct Answer -

All parallels are equal in length सभी समातर लबाई म समान ह

Which of the following Indian states is also known as a lsquoLand of Red river and Blue Hillsrsquo

िनिलखत म स िकस भारतीय रा को लाल नदी और नीली पहािड़यो की भिम क नाम स जाना जाता ह

1 Uttarkhand उराखड

2 Assam असम

3 Meghalaya मघालय

4 Arunachal Pradesh अणाचल दश

Correct Answer -

Assam असम

In spatial analysis of settlement Rn = 215 indicates which type of settlement arrangement

िनपटान क थािनक िवषण म Rn = 215 यह इिगत करता ह िक िकस कार की िनपटान वथा ह

1 Uniform यिनफॉम

2 Semi-Clustered समी- ल टर

3 Clustered ल टर

4 Random रडम

Correct Answer -

Uniform यिनफॉम

Who are known as the lsquoYellow Peoplersquo lsquoयलो पीपलrsquo क प म कौन जाना जाता ह

1 Mongoloids मोगोलोइडस

2 Nigroids नीोइडस

3 Australoids ऑ लॉइडस

4 Caucasoids कॉकसोइडस

Correct Answer -

Mongoloids मोगोलोइडस

ि ि ो ौ ि

32)

33)

34)

Who publishes the topographical map of India भारत क थलाकितक मानिच को कौन कािशत करता ह

1 Geographical Survey of India भारत का भौगोिलक सवण

2 Government of India भारत सरकार

3 Geological Survey of India भारत क भगभय सवण

4 Survey of India भारत का सवण

Correct Answer -

Survey of India भारत का सवण

Who among the following claimed geography to be the lsquoEcology of Manrsquo

िनिलखत म स िकसन भगोल को मन का पारथितकी कहा ह

1 Alfred Hener अड हटनर

2 Vidal-de la Blache वाइडल-िड लॉ ॉश

3 Oo Schluter ओटो टर

4 Harlan Barrow हरलन बारो

Correct Answer -

Harlan Barrow हरलन बारो

Who among the following is regarded as the founder of humanistic approach in geography

िनिलखत म स िकस भगोल म मानवतावादी िकोण का सथापक माना जाता ह

1 William Bunge िविलयम बग

2 Yi-Fu-Tuan यी-फ- यान

3 Brain JL Berry न जएल बरी

4 Richard Peet रचड पीट

Correct Answer -

Yi-Fu-Tuan यी-फ- यान

Who prepared Lorenz curve लोरज व िकसन तयार िकया

1 Geddes गडस

2 None of these इनम स कोई नही

3 Griffith Taylor ििफथ टलर

35)

36)

37)

4 Max U Lorenz म य लोरज

Correct Answer -

Max U Lorenz म य लोरज

Gulf Streams are the currents of which of the following oceans

खाड़ी की धाराए िनिलखत महासागरो म स िकसकी धाराए ह

1 North Atlantic Ocean उरी अटलािटक महासागर

2 North Pacific Ocean उरी शात महासागर

3 Arabian Sea अरब सागर

4 South Pacific Ocean दिण शात महासागर

Correct Answer -

North Atlantic Ocean उरी अटलािटक महासागर

Disintegration wearing away and removal of rock material is generally referred as

िशला पदाथ (रॉक सामी) का टटना िमटना और हटना आमतौर पर ________ क प म सदिभत िकया जाता ह

1 Shattering िवसकारक

2 Denudation अनाादन

3 Fault श

4 Decomposition िवयोजन

Correct Answer -

Denudation अनाादन

Variations in the length of day time and night from season to season are due to

मौसम स मौसम परवतन पर िदन क समय और रात क समय की अविध म िभताए िन कारण स होती ह

1 The Earthrsquos revolution round the Sun in an elliptical manner पी का दीघवाकार तरीक स सय क चारो घणन

2 The Earthrsquos rotation on its axis पी का इसकी धरी पर घणन

3 Revolution of the Earth on a tilted axis नत अ पर पी का घणन

4 Latitudinal position of the place थान की अाश थित

Correct Answer -

Revolution of the Earth on a tilted axis नत अ पर पी का घणन

38)

39)

40)

Point out the correct sequence of mountain ranges from north to south

उर स दिण तक पवत खलाओ क सही अनम को इिगत कर

1 Great Himalaya Middle Himalaya Outer Himalaya Trans Himalaya

महान िहमालय म िहमालय बा िहमालय परा िहमालय

2 Middle Himalaya Great Himalaya Trans Himalaya Outer Himalaya

म िहमालय महान िहमालय परा िहमालय बा िहमालय

3 Outer Himalaya Middle Himalaya Great Himalaya Trans Himalaya

बा िहमालय म िहमालय महान िहमालय परा िहमालय

4 Trans Himalaya Great Himalaya Middle Himalaya Outer Himalaya

परा िहमालय महान िहमालय म िहमालय बा िहमालय

Correct Answer -

Trans Himalaya Great Himalaya Middle Himalaya Outer Himalaya

परा िहमालय महान िहमालय म िहमालय बा िहमालय

Sunrsquos halo is produced by the refraction of light in सय का भामडल ______ म काश क अपवतन ाराउ होता ह

1 Ice crystals in Cirrro-Cumulus clouds पाभ-कपास मघो क बफ िल

2 Ice crystal in Cirrus clouds पाभमघो क बफ िल

3 Dust particles in Stratus clouds री मघो क धल कण

4 Water vapour in Stratus clouds री मघो क जल वा

Correct Answer -

Ice crystal in Cirrus clouds पाभमघो क बफ िल

Read the given statements and answer which of the following options isare correct

(1) The minerals present in the rocks exposed to atmosphere are not subjected to alteration

(2) Oxidation is one of the processes of chemical weathering

िदए गए कथनो को पढ़ और उर द िक िन म स कौन सास िवक सही ह

(1) वायमल स अनावत शल म उपथत खिनज परवतन क अधीन नही होता ह

(2) ऑीकरण रासायिनक अपय की ियाओ म स एक ह

ो ो

41)

42)

1 Both statements are wrong दोनो कथन गलत ह

2 Both statements are correct दोनो कथन सही ह

3 First statement is wrong and second statement is correct पहला कथन गलत ह और दसरा कथन सही ह

4 First statement is correct and second statement is wrong पहला कथन सही ह और दसरा कथन गलत ह

Correct Answer -

First statement is wrong and second statement is correct पहला कथन गलत ह और दसरा कथन सही ह

Read the given statements and answer which of the following options isare correct

1 Sunrsquos short waves enter the earth partially heating the atmosphere

2 Heated earth surface from the sun produces broader waves which interacts and heats the atmosphere

िदए गए कथन को पढ़ और उर द िक िन म स कौन सास िवक सही ह

1 सय की छोटी तरग पी म आिशक प स वश करती ह और वायमडल को ऊत करती ह

2 सय स ऊत पी की सतह िवारत तरग उ करती ह जो परर भाव डालती ह और वायमडल कोऊत करती ह

1 Both Statements 1 and 2 are correct दोनो कथन 1 और 2 सही ह

2 Both Statements 1 and 2 are wrong दोनो कथन 1 और 2 गलत ह

3 Statement 1 is wrong and only Statement 2 is correct कथन 1 गलत ह और कवल कथन 2 सही ह

4 Only statement 1 is correct कवल कथन 1 सही ह

Correct Answer -

Both Statements 1 and 2 are correct दोनो कथन 1 और 2 सही ह

Read the given statements and answer which of the following options isare correct

(1)The rocks that get changed due to heat and pressure are termed as metamorphic rocks

(2)Slate is one such type of metamorphic rock

िदए गए कथनो को पढ़ और उर द िक िन म स कौन सास िवक सही ह

(1) शल जो ऊा और दाब क कारण परवितत हो जात ह उ कायातरक शलो क प म जाना जाता ह

(2) ट एक तरह का कायातरक शल ह

1 Both statements are wrong दोनो कथन गलत ह

2 Both statements are correct दोनो कथन सही ह

3 First statement is wrong and second statement is correct पहला कथन गलत ह और दसरा कथन सही ह

ी औ

43)

44)

4 First statement is correct and second statement is wrong पहला कथन सही ह और दसरा कथन गलत ह

Correct Answer -

Both statements are correct दोनो कथन सही ह

Read the given statements and answer which of the following options isare correct

1 Higher temperature anomaly is observed in the northern hemisphere

2 Differential heating is absent in Northern Hemisphere

िदए गए कथनो को पढ़ और उर द िक िन म स कौन सास िवक सही ह

1 उरी गोलाध म उ तापमान िवसगित पायी जाती ह

2 उरी गोलाध म अतर ऊन अनपथत होती ह

1 Both Statements 1 and 2 are correct दोनो कथन 1 और 2 सही ह

2 Both Statements 1 and 2 are wrong दोनो कथन 1 और 2 गलत ह

3 Statement 1 is wrong and Statement 2 is correct कथन 1 गलत ह और कथन 2 सही ह

4 Statement 1 is correct and Statement 2 is wrong कथन 1 सही ह और कथन 2 गलत ह

Correct Answer -

Statement 1 is correct and Statement 2 is wrong कथन 1 सही ह और कथन 2 गलत ह

Read the given statements and answer which of the following options isare correct

(1) Plutonic rocks are intrusive type of igneous rocks

(2) It cools very slowly because the surrounding rock serves as insulation around the intrusion of magma

िदए गए कथनो को पढ़ और उर द िक िन म स कौन सास िवक सही ह

(1) िवतलीय शल अतवधी कार क आश शल ह

(2) यह बत धीर-धीर ठडा होता ह ोिक आस-पास क शल मा क अतवधन क चारो ओर रोधन क प म कायकरत ह

1 Both statements are wrong दोनो कथन गलत ह

2 Both statements are correct दोनो कथन सही ह

3 First statement is wrong and second statement is correct पहला कथन गलत ह और दसरा कथन सही ह

4 First statement is correct and second statement is wrong पहला कथन सही ह और दसरा कथन गलत ह

Correct Answer -

Both statements are correct दोनो कथन सही ह

45)

46)

47)

48)

The dust and ash material hurled from the volcanoes are termed as

ालामखी स िनकलन वाली धल और राख सामी को _______ क प म कहा जाता ह

1 Pyroclasc पाइरोाक

2 Hyperclastic हाइपराक

3 Hepiroclastic हिपरोाक

4 Cirroclastic िसरोाक

Correct Answer -

Pyroclasc पाइरोाक

The vertical difference in elevation between a low tide and high tide is referred as

कम ार और उ ार क बीच ऊचाई म लबवत अतर _____ स सदिभत होता ह

1 Tidal slope ारीय ढलान

2 Tidal elevation ारीय उयन

3 Tidal range ारीय परास

4 Tidal height ारीय ऊचाई

Correct Answer -

Tidal range ारीय परास

The maximum biodiversity is found in which of the following regions िनिलखत ो म स अिधकतमजव िविवधता िकसम पायी जाती ह

1 Amazon Basin अमज़न बिसन

2 East Indies ई इडीज

3 Congo Basin कागो बिसन

4 West indies व इडीज

Correct Answer -

Amazon Basin अमज़न बिसन

The cultivation of rice crop produces_______ चावल की फसल की खती ______ का उादन करती ह

1 SO2

49)

50)

51)

2 CH4

3 CFCs

4 CO2

Correct Answer -

CH4

The pressure system with higher pressure at the centre is called__________

क म उ दबाव वाली दबाव णाली को _______ कहा जाता ह

1 front अ

2 depression अवनमन

3 cyclone चवात

4 anti-cyclone ितचवात

Correct Answer -

anti-cyclone ितचवात

The Himalayan region is poor in mineral resources because िहमालयी खिनज ससाधनो म समनही ह ोिक

1 The displacement of rock strata has disturbed the arrangement of rocks and made it complex

शलीय परत क िवथापन न चानो की वथा को अवथत कर िदया ह और इस जिटल बना िदया ह

2 The climate conditions are not suitable for exploitation of minerals

जलवाय की थित खिनजो क दोहन क िलए उपय नही ह

3 The terrain makes explanation of minerals difficult and very costly due to transportation difficulties

भ-भाग परवहन की किठनाइयो क कारण खिनजो का दोहन मल और बत महगा बना दता ह

4 It is made up of crystalline rocks यह िलीय चानो स बना ह

Correct Answer -

The displacement of rock strata has disturbed the arrangement of rocks and made it complex

शलीय परत क िवथापन न चानो की वथा को अवथत कर िदया ह और इस जिटल बना िदया ह

The process through which the moisture is added to the atmosphere by vegetation is termed as

वह िया िजसक माम स वनित ारा वातावरण म नमी िमलायी जाती ह _______ क प म जानी जाती ह

52)

53)

54)

1 Condensation सघनन

2 Evapotranspiration वान-उजन

3 Radiation िविकरण

4 Precipitation वषण

Correct Answer -

Evapotranspiration वान-उजन

The process through which the terrestrial heat is transferred to air by direct contact is termed as

वह िया िजसम सपक ारा थलीय ऊा वाय म थानातरत हो जाती ह ______ क प म जानी जाती ह

1 Conduction चालन

2 Convection सवहन

3 Insolation आतपन

4 Radiation िविकरण

Correct Answer -

Conduction चालन

The largest area under mangroves is in which of the following statesunion territory

मोव क अतगत िनिलखत राोसघ शािसत दशो म स सबस बड़ा कौन सा ह

1 Andaman and Nicobar अमान और िनकोबार

2 Andhra Pradesh आ दश

3 West Bengal पिम बगाल

4 Gujarat गजरात

Correct Answer -

West Bengal पिम बगाल

The longitudinal transverse and surface waves in an earthquake originate from

भकप म दशातर अनथ और सतह तरग यहा उ होती ह

1 The focus on the surface of the Earth पी क सतह पर क -िबद म

2 The focus within the body of the Earth पी क भीतर क -िबद म

3 The epicenter within the body of the Earth पी क भीतर उपरक म

55)

56)

57)

4 The epicenter on the surface of the Earth पी क सतह पर उपरक म

Correct Answer -

The focus within the body of the Earth पी क भीतर क -िबद म

The down slope movement of material due to gravity is called______

गाकषण क कारण पदाथ की अनढाल गित को ______ कहा जाता ह

1 mass movement पदाथ सचलन

2 deposition िनप

3 erosion रण

4 volcanic movement ालामखीय सचलन

Correct Answer -

mass movement पदाथ सचलन

Shimla is cooler than Amritsar although both are on the same latitude This is because

िशमला म अमतसर स अिधक ठड ह हालािक दोनो समान अाश पर ह ऐसा ह ोिक

1 Shimla is at a greater height above sea level than Amritsar अमतसर की तलना म िशमला सम तल स अिधकऊचाई पर ह

2 Shimla is further north िशमला उर की ओर ह

3 Shimla is farther from the equator िशमला भम रखा स आग ह

4 Their longitudes differ उनकी दशातर रखाए िभ ह

Correct Answer -

Shimla is at a greater height above sea level than Amritsar अमतसर की तलना म िशमला सम तल स अिधकऊचाई पर ह

lsquoTempo of Urbanizationrsquo measures which of the following

lsquoशहरीकरण का टपोrsquo िनिलखत म स कौन सा उपाय ह

1 Speed of urbanizaon शहरीकरण की गित

2 None of the above इनम स कोई नही

3 Inequality of urbanizaon शहरीकरण की असमानता

4 Current level of urbanizaon शहरीकरण का वतमान र

Correct Answer -

58)

59)

60)

Speed of urbanizaon शहरीकरण की गित

Out of the following options choose the INCORRECT statement

िनिलखत िवको म स गलत कथन का चयन कर

1 The clear tracts in the equatorial region recover rapidly भम रखा म भभाग तजी स ठीक हो जात ह

2 The stable communities include a redwood forest a pine forest at high elevations

थर समदायो म एक रडवड वन उ ऊचाई पर एक दवदार वन शािमल ह

3 Any ecosystem moves towards maximum biomass and stability to survive

कोई भी पारथितकी त जीिवत रहन क िलए अिधकतम जवसहित और थरता की तरफ असर होता ह

4 Tropical rain forests near equator are stable ecosystems

भम रखा क पास उकिटबधीय वषा वन थर पारथितक त ह

Correct Answer -

The clear tracts in the equatorial region recover rapidly भम रखा म भभाग तजी स ठीक हो जात ह

Seasonal contrasts are maximum in मौसमी िवषमता अिधकतम ह

1 Mid latitudes म अाश म

2 Low attitudes िन अाश म

3 High latitudes उ अाश म

4 Subtropics उपोकिटबधीय म

Correct Answer -

Mid latitudes म अाश म

In India which type of forest among the following occupies the largest area

भारत म िनिलखत म स िकस कार क वन सबस बड़ा फल आािदत करत ह

1 Sub-tropical Dry Evergreen Forest उप उकिटबधीय श सदाबहार वन

2 Mountain Wet Temperate Forest पवतीय आ शीतो वन

3 Tropical Moist Deciduous Forest उकिटबधीय आ पणपाती वन

4 Tropical Wet Evergreen Forest उकिटबधीय आ सदाबहार वन

Correct Answer -

Tropical Moist Deciduous Forest उकिटबधीय आ पणपाती वन

61)

62)

63)

64)

What is the proportion of lsquoJuvenile Populationrsquo (0-14 years) in India as per 2011Census

2011 की जनगणना क अनसार भारत म जवनाइल पॉपलशन यानी िकशोर जनस या (0-14 वष) का अनपात ाह

1 3076 of total population कल जनस या का 3076

2 2764 of total population कल जनस या का 2764

3 2933 of total population कल जनस या का 2933

4 3354 of total population कल जनस या का 3354

Correct Answer -

3076 of total population कल जनस या का 3076

What is the Belfast famous for बलफा िकसक िलए मशर ह

1 Belt of cotton textile industry कपास व उोग क

2 Ship-building industry जहाज िनमाण उोग

3 Agricultural machinery किष उपकरण

4 Aero planes manufacturing वाययान िनमाण

Correct Answer -

Ship-building industry जहाज िनमाण उोग

What is the most important occupation in tropical monsoon lands

उकिटबधीय मॉनसन भिम म सबस महपण वसाय ा ह

1 Mining खनन

2 Cattle rearing मवशी पालन

3 Agriculture किष

4 Nomadic herding नोमािडक जड़ी-बिटया

Correct Answer -

Agriculture किष

What is the most important characteristics of the islands (Indian) located in the Arabian sea

अरब सागर म थत ीपो (भारतीय) की सबस महपण िवशषता ा ह

ी ो

65)

66)

67)

1 There are all of coral origins सभी कोरल मल क ह

2 There are all very small in size य सभी आकार म बत छोट ह

3 They have a very dry climate इनकी जलवाय बत श ह

4 They are extended parts of the mainland व महाीप क िवारत िह ह

Correct Answer -

There are all of coral origins सभी कोरल मल क ह

What do the basalt layers of the Deccan indicate डन की बसा परत ा इिगत करती ह

1 All of the above उपरो सभी

2 Huge volcanic eruptions in the distant past दरथ अतीत म िवशाल ालामखीय िवोट

3 The immense erosional activity of the rivers निदयो की िवशाल रण गितिविध

4 The influence of weathering मौसम का भाव

Correct Answer -

Huge volcanic eruptions in the distant past दरथ अतीत म िवशाल ालामखीय िवोट

In the structure of planet Earth below the mantle the core is mainly made up of_____

पी ह की सरचना म मटल क नीच कोर म प स______ स िनिमत होती ह

1 aluminium एमीिनयम

2 silicon िसिलकॉन

3 chromium ोिमयम

4 iron लोहा

Correct Answer -

iron लोहा

One of the major Mid Oceanic Ridge is found in मख म-महासागर चोिटयो म स एक ______ म पायाजाता ह

1 Mid Pacific Ocean म शात महासागर

2 Mid Atlantic Ocean म अटलािटक महासागर

3 Mid Indian Ocean म भारतीय महासागर

4 Mid Arctic Ocean म आक िटक महासागर

68)

69)

70)

71)

Correct Answer -

Mid Atlantic Ocean म अटलािटक महासागर

Magma that reaches the Earthrsquos surface and then solidifies is called________

मा जो पी की सतह तक पचती ह और िफर ठोस हो जाती ह ________कहलाती ह

1 quartz ाटज

2 lava लावा

3 granite नाइट

4 silicates िसिलकट

Correct Answer -

lava लावा

Isotherms are the lines of equal_______ समताप रखाए समान _______की रखाए होती ह

1 pressure दाब

2 temperature तापमान

3 rainfall वषा

4 height ऊचाई

Correct Answer -

temperature तापमान

Mark the correct sequence of passes in the Western Ghats from north to south

पिमी घाटो म उर स दिण तक दर क सही अनम को िचित कर

1 Thalghat Palghat Bhorghat थलगघाट पालघाट भोरघाट

2 Thalghat Bhorghat Palghat थलघाट भोरघाट पालघाट

3 Bhorghat Thalghat Palghat भोरघाट थलघाट पालघाट

4 Palghat Bhorghat Thalghat पालघाट भोरघाट थलघाट

Correct Answer -

Thalghat Bhorghat Palghat थलघाट भोरघाट पालघाट

Which of the following does not have influence over the climate in India

ि ि ि ी ी

72)

73)

िनिलखत म स िकसका भाव भारत की जलवाय पर नही पड़ता ह

1 Ocean currents सागर की लहर

2 Nearness to equator भम रखा स िनकटता

3 Monsoons मानसन

4 Presence of Indian ocean भारतीय महासागर की उपथित

Correct Answer -

Ocean currents सागर की लहर

Which of the following cloud types has the characteristics like vertical tall narrow and puffy

िनिलखत म स िकस कार क मघो म लबवत लबी सकीण और थलता जसी िवशषताए ह

1 Cumulonimbus तफानी मघ

2 Cumulus मघ पज

3 Cirrocumulus पाभ कपासी मघ

4 Nimbostratus वषारी मघ

Correct Answer -

Cumulus मघ पज

Which of the following statement is INCORRECT about Crude Birth Rate

िनिलखत स कौन सा कथन अशोिधत ज दर क बार म सही नही ह

1 It cannot be used for comparing fertility level between two countries with different population characteristics

इसका उपयोग िविभ जनसा िवशषताओ वाल दो दशो क बीच जनन र की तलना क िलए नही िकया जा सकता ह

2 It is a standardized measure of fertility

यह जनन मता का मानकीकत उपाय ह

3 It is effected by the age-sex composition of the population

यह आबादी की आय-िलग सरचना स भािवत होता ह

4 It is expressed per 1000 population in a given geographical unit

यह िकसी दी गई भौगोिलक इकाई म ित 1000 जनसा पर िकया जाता ह

Correct Answer -

It is a standardized measure of fertility

यह जनन मता का मानकीकत उपाय ह

74)

75)

76)

77)

Which of the following state in India experienced negative decadal growth rate during 2001 to 2011census

भारत म िनिलखत म स िकस रा म वष 2001 स 2011 की जनगणना क दौरान नकाराक िगरावट दर ई

1 Tripura िपरा

2 Nagaland नागालड

3 Haryana हरयाणा

4 Odisha ओिडसा

Correct Answer -

Nagaland नागालड

Which of the following is NOT a characteristic of peninsular rivers

िनिलखत म स कौन सी िवशषता ायीपीय निदयो म नही होती ह

1 Flow through shallow valleys उथल घािटयो क माम स वाह

2 Seasonal flow मौसमी वाह

3 Little erosional activity थोड़ी कटावदार गितिविध

4 Meandering tendency often shifting their beds घमावदार वि अर अपन तटो को थानातरत करना

Correct Answer -

Meandering tendency often shifting their beds घमावदार वि अर अपन तटो को थानातरत करना

Which of the following gases in the atmosphere absorbs heat from the Sunrsquos radiation and the Earthssurface

वायमडल म िनिलखत म स कौन सी गस सय क िविकरण और पी की सतह स ऊा को अवशोिषत करती ह

1 Neon िनयॉन

2 Carbon dioxide काबन डाइऑाइड

3 Argon आगन

4 Nitrogen नाइट ोजन

Correct Answer -

Carbon dioxide काबन डाइऑाइड

Which of the following kind of settlement pattern is found at the confluence of rivers

ि ि ि ि ो

78)

79)

80)

िनिलखत म स िकस कार का वथापन पटन निदयो क सगम पर पाया जाता ह

1 Triangular Paern िकोणीय पटन

2 Circular or Semi-Circular Paern परप या अध-परप पटन

3 Nebular Paern नबलर पटन

4 Star ndashShaped Paern ार-आकार का पटन

Correct Answer -

Triangular Paern िकोणीय पटन

Which one was not the objective of the Biosphere Reserve Projects launched by the UNESCO

यनो ारा श की गई सरित जवमडल परयोजनाओ का उ इनम स कौन सा नही था

1 To promote teaching and research िशण और अनसधान को बढ़ावा दना

2 To make agriculture sustainable किष को दीघकािलक बनाना

3 To conserve ecosystems पारथितक त को सरित करना

4 To conserve genetic diversity for a longtime लब समय तक अनवािशक िविवधता को सरित करना

Correct Answer -

To make agriculture sustainable किष को दीघकािलक बनाना

Which region of the Earth surface receives the highest amount of insulation

पी सतह का कौन सा तापावरोधन की उतम माा ा करता ह

1 Land mass थलखड

2 Savannah region सवाना

3 Water bodies जल िनकाय

4 Tropical desert उकिटबधीय रिगान

Correct Answer -

Tropical desert उकिटबधीय रिगान

Which one of the following is not a biodiversity hotspot

िनिलखत म स कौन सा जव िविवधता का म जगह नही ह

1 Eastern Himalaya पव िहमालय

2 Eastern Ghats पव घाट

81)

82)

83)

3 Indo-Myanmar भारत-ामार

4 Westerm Ghats पिमी घाट

Correct Answer -

Eastern Ghats पव घाट

Which one of the following is NOT a part of the World Network of Biosphere Reserves based on theUNESCO Man and Biosphere Programme

यनो मन और बायोीयर कायम क आधार पर िनिलखत म स कौन बायोीयर रजव क िव नटवक कािहा नही ह

1 Gulf of Mannar मार की खाड़ी

2 Seshachalam शषाचलम

3 Sunderban सदरबन

4 Nilgiri नीलिगर

Correct Answer -

Seshachalam शषाचलम

Which one of the following is an example of ldquodesert vegetationrdquo

िनिलखत म स कौन मथलीय वनित का एक उदाहरण ह

1 Mosses and lichens दलदल और शवाल

2 Temperate grassland समशीतो घास क मदान

3 Coniferous forest शकधारी वन

4 Acacia and cactus एकािसया और कस

Correct Answer -

Acacia and cactus एकािसया और कस

Which one of the following reflects more sunlight िनिलखत म स कौन सा सय की रोशनी को अिधकपरावितत करता ह

1 Paddy crop land धान फसल भिम

2 Land covered with fresh snow ताजा बफ स आािदत भिम

3 Sand desert रतीली रिगान

4 Prairie land यरी भिम

84)

85)

86)

87)

Correct Answer -

Land covered with fresh snow ताजा बफ स आािदत भिम

Which layer of the atmosphere is in contact with the surface of the earthrsquos oceans

वायमडल की कौन सी परत पी क महासागरो की सतह क सपक म ह

1 Stratosphere समताप मडल

2 Mesosphere म मडल

3 Hydrosphere जलमडल

4 Troposphere ोभ मडल

Correct Answer -

Troposphere ोभ मडल

Mediterranean Sea is a border of which of the following countries भम सागर िनिलखत दशो म सिकसकी सीमा ह

1 None of these इनम स कोई नही

2 Iraq इराक

3 Lebanon लबनान

4 Jordan जॉडन

Correct Answer -

Lebanon लबनान

Benguela ocean currents are found along which coast बगएला महासागर धाराए िकस तट क साथ पायीजाती ह

1 East Coast of South America दिण अमरका क पव तट

2 East Coast of Africa अीका क पव तट

3 West Coast of South America दिण अमरका क पिमी तट

4 West Coast of Africa अीका क पिमी तट

Correct Answer -

West Coast of Africa अीका क पिमी तट

88)

89)

90)

Due to tension a block of land on one side being pushed up or upthrown relative to the downthrown blockis referred as

तनाव क कारण नीच फ क ए खड क साप भिम का एक खड एक ओर स ऊपर धकला जाता ह या ऊपर की ओरफ का जाता ह यह _____ क प म सदिभत ह

1 Thrust fault प श

2 Normal fault सामा श

3 Reverse fault म श

4 Strike slip fault नितलब सपण श

Correct Answer -

Normal fault सामा श

Inter-tropical doldrums is a zone of ______ अतर-उकिटबधीय डोलड ______ का एक ह

1 Frontolysis टोलायिसस

2 Convergence अिभसरण

3 Inter-tropical divergence zone अतर-उकिटबधीय िवचलन

4 Local wind थानीय वाय

Correct Answer -

Convergence अिभसरण

The Horse Latitudes are regions located at about _____ north and south of the equator

हॉस अाश भम रखा क उर और दिण म लगभग _____ पर थत ह

1 30ndash60 degree Latitude 30-60 िडी अाश

2 0ndash5 degree Latitude 0-5 िडी अाश

3 30 degree Latitude 30 िडी अाश

4 60ndash90 degree Latitude 60-90 िडी अाश

Correct Answer -

30 degree Latitude 30 िडी अाश

Generally evaporation is high over which part of the Earth

आम तौर पर पी क िकस भाग पर वाीकरण अिधक होता ह

1 Equatorial maritime भमवत समीय ी ी

91)

92)

2 Equatorial continental भमवत महाीपीय

3 Polar maritime वीय समीय

4 Polar continental वीय महाीपीय

Correct Answer -

Equatorial maritime भमवत समीय

A very high temperature during summer in north western India leads to what type of climaticcondition in south

उर पिमी भारत म गम क दौरान बत अिधक तापमान होन क कारण दिण म िकस कार की जलवाय थितउ करता ह

1 Depression over arabian sea अरब सागर पर अवनमन

2 Failure monsoon मानसन िवफलता

3 Successful monsoon मानसन सफलता

4 Cyclones चवात

Correct Answer -

Successful monsoon मानसन सफलता

Lightning and thunder are the resultant effect when तिड़त और गजन परणामी भाव ह जब

1 Two massive clouds hit powerfully each other first lightning is produced and later sound is produced

दो बड़ बादल एक दसर स शशाली ढग स टकरात ह पहल आकाशीय िवदयत उ होता ह और बाद म िन उहोती ह

2 Two massive clouds come into contact with the powerful wind collision this results into first sound and thenlightning

दो बड़ बादल शशाली पवन सघ क सपक म आत ह इसका परणामप पहल िन और िफर आकाशीय िवदयतउ होता ह

3 None of the above उपरो म स कोई भी नही

4 A high density cloud contains positively and negatively charged electric ions and when this interacts light andsound are simultaneously produced

एक उ घन बादल म धनाक और ऋणाक आविशत िवदयत आयन होत ह और जब यह परर भाव डालत ह तोकाश और िन एक साथ उािदत होती ह

Correct Answer -

A high density cloud contains positively and negatively charged electric ions and when this interacts light andsound are simultaneously produced

औ ि ि ो औ ो

93)

94)

95)

एक उ घन बादल म धनाक और ऋणाक आविशत िवदयत आयन होत ह और जब यह परर भाव डालत ह तोकाश और िन एक साथ उािदत होती ह

Doon Valley is able to grow rice because दन घाटी चावल उगान म सम ह ोिक

1 Other crops cannot be grown वहा अ फसलो को उगाया नही जा सकता ह

2 People in the valley are rice eaters घाटी म लोग चावल खान वाल ह

3 There is a huge export demand of rice वहा चावल की भारी िनयात माग ह

4 It has warm summer and snow melt waters for irrigation

वहा गिमया गम होती ह िसचाई क िलए बफ का िपघला आ पानी होता ह

Correct Answer -

It has warm summer and snow melt waters for irrigation

वहा गिमया गम होती ह िसचाई क िलए बफ का िपघला आ पानी होता ह

CANCELLED

In the geological time scale the Mesozoic Era DOES NOT contains which of the following periods

भगभय समय पमान पर मजीवी यग म िन कालो म स कौन नही ह

1 Triassic ट ाइऐिसक

2 Jurassic जरिसक

3 Cretaceous चाकमय

4 Carboniferous काबनी

Correct Answer -

Carboniferous काबनी

96)

1 P-3 Q-4 R-2 S-1

2 P-3 Q-4 R-1 S-2

3 P-3 Q-4 R-1 S-2

4 P-4 Q-3 R-2 S-1

Correct Answer -

P-4 Q-3 R-2 S-1

1 P-3 Q-1 R-4 S-2

2 P-3 Q-4 R-1 S-2

3 P-3 Q-2 R-4 S-1

97)

98)

4 P-2 Q-1 R-4 S-3

Correct Answer -

P-3 Q-4 R-1 S-2

1 P-3 Q-1 R-4 S-2

2 P-2 Q-3 R-4 S-1

3 P-2 Q-1 R-3 S-4

4 P-4 Q-2 R-1 S-3

Correct Answer -

P-3 Q-1 R-4 S-2

99)

100)

1 P-3 Q-2 R-4 S-1

2 P-1 Q-2 R-3 S-4

3 P-2 Q-3 R-1 S-4

4 P-4 Q-3 R-2 S-1

Correct Answer -

P-2 Q-3 R-1 S-4

ldquoHuman geography is the study of changing relationship between the unresting man and the unstableearthrdquo was defined by

lsquolsquoमानव भगोल ाकल आदमी और अथर पी क बीच सबध परवतन का अयन हrdquo ______ ारा परभािषत िकया गयाथा

1 J Brunches ज चस

2 EC Semple ईसी सल

3 HJ Mackinder एच ज मिकदर

4 PV Blache पीवी च

Correct Answer -

EC Semple ईसी सल

Sedimentary rocks are finally and ultimately derived from the____________

अवसादी चान अततः ________ स ा की जाती ह

1 action of earth movements पी की गितिविधयो

2 marine deposit समी िनप

3 weathering of metamorphic rocks पातरत चानो क अपय

4 weathering of igneous rocks आय चानो क अपय

Correct Answer -

weathering of igneous rocks आय चानो क अपय

Page 3: High School Teacher Eligibility Test- BOARD PROFESSIONAL ...peb.mp.gov.in/results/RESULT_18/HST_RES18/Final_anwser_key/HST… · M a ndl a / मंड ल ... Under the Madhya Pradesh

8)

9)

10)

Correct Answer -

खरी-खोटी सनाना

lsquolearningrsquo श द क िलए िह दी भाषा म उिचत पारभािषक श दावली का चयन कर

1 रटना

2 अिधगम

3 िवणता

4 सीखाना

Correct Answer -

अिधगम

चादी का पयायवाची ________ ह

1 धनरस

2 उदक

3 खनक

4 चहास

Correct Answer -

चहास

ldquoकर मसर नाचत नगन लख हलधर को ागrdquo

हिस- हिस गोपी िफर हस मन िपय सी भाग II - म कौन सा रस ह

1 हा रस

2 शात रस

3 भ रस

4 वा रस

Correct Answer -

हा रस

Topic- GENERAL ENGLISH

1)

2)

3)

Choose the option that best expresses the meaning of the highlighted idiomphrase

Anil How did the election go

Sunil Very exciting till the last minute Our candidate proved to be a dark horse

1 The candidate is not popular but won

2 His candidate lost the election though he was expected to win

3 The candidate backed out at the last minute

4 The candidate is popular and won

Correct Answer -

The candidate is not popular but won

A sentence with an underlined word is given below Choose the correct option which is closest in meaning tothe underlined word

The fruit of ones toil helps others to continue with the mission

1 achievement

2 hard work

3 attempts

4 sufferings

Correct Answer -

hard work

Out of the following options choose the correct form of verb that is in agreement with the subject for thegiven sentence

lsquoA Tale of Two Citiesrsquo ______ a wonderful book to read

1 are

2 were

3 is

4 have been

Correct Answer -

is

4)

5)

6)

Out of the following options choose the most appropriate tense that best fills in to complete the givensentence

This year our cricket team _________ all the matches played on the Indian soil

1 will win

2 was winning

3 wins

4 has been winning

Correct Answer -

has been winning

Choose the appropriate adverb clause from the options to complete the given sentence

He was very pleased _____

1 which you passed

2 which you had passed

3 that you passed

4 that you who passed

Correct Answer -

that you passed

Out of the following four options choose the incorrect sentence

1 Rohit is a charmer and you can find him selling newspapers on M G Road from 800 am every morning

2 Diabetes a leading cause of heart problems does not allow people to make it to the hospital in time after a heartattack

3 Symbiosis Institute is Indias premier approved business school

4 Nehrus vision continued to light up the entire nation even today

Correct Answer -

Nehrus vision continued to light up the entire nation even today

7)

8)

1)

Out of the following options choose the most appropriate usage to fill in the blank

I live a few yards ______ the bus stop

1 off

2 from

3 outside

4 away

Correct Answer -

from

Arrange the sentences P Q R and S to form a logical sequence between sentences 1 and 6

1 Jurors do not have to be intelligentP In 1933 in a sheep-stealing case in Wales two of the jurors were totally unable to understand Englishbut no one found that out until after the trialQ There is no test no examination and no enquiry about their abilitiesR The convicted found this out and complained about it to the Court of Criminal AppealS No one even takes the trouble to find out whether they can understand the English language6 Still the Court of Appeal felt that it must refuse to interfere

1 QPRS

2 SQRP

3 PRQS

4 QSPR

Correct Answer -

QSPR

Topic- GENERAL KNOWLEDGE

Which country will host the 2022 Football World Cup 2022 फटबॉल िव कप की मजबानी करन वाला दशकौन होगा

1 America अमरका

2 Canada कनाडा

3 Qatar कतर

4 Mexico मको

2)

3)

4)

5)

Correct Answer -

Qatar कतर

According to sex ratio trends of Census 2011 which district of Madhya Pradesh has been ranked numberOne

जनगणना 2011 म िलग अनपात क झानो क अनसार म दश क िकस िजल को नबर एक थान िदया गया ह

1 Seoni िसओनी

2 Ratlam रतलाम

3 Balaghat बालाघाट

4 Mandla मडला

Correct Answer -

Balaghat बालाघाट

The folk dance form called lsquoMatkirsquo is prevalent in which of the following regions of Madhya Pradesh

मटकी नामक लोक न प म यदश क िनिलखत म स िकस म चिलत ह

1 Bundelkhand बदलखड

2 Malwa मालवा

3 Vindhyachal िवाचल

4 Baghelkhand बघलख

Correct Answer -

Malwa मालवा

Who appoints the Chairman and the Members of the Union Public Service Commission

सघ लोक सवा आयोग क अ और सदो की िनय कौन करता ह

1 Chief Justice of India भारत क म य यायधीश

2 Prime Minister धान मी

3 President रा ट पित

4 Chief Election Commissioner म य चनाव आय त

Correct Answer -

President रा ट पित

6)

7)

8)

Under the Madhya Pradesh Ladli Laxmi Yojana the beneficiary is paid the total amount on attaining theage of ___ मदश लाडली ली योजना क तहत लाभाथ की आय ___ वष परी होन पर कल रािश का भगतान िकयाजाता ह

1 18

2 21

3 23

4 16

Correct Answer -

21

India has won the gold medal in hockey ______times at the Olympics

ओलिपक खलो म भारत न हॉकी म ण पदक _____ बार जीता ह

1 9

2 7

3 8

4 6

Correct Answer -

8

Where is the Madhya Pradesh Tribal Museum located at

म दश जनजातीय सहालय कहा थत ह

1 Indore इदौर

2 Jhabua झबआ

3 Khargone खरगोन

4 Bhopal भोपाल

Correct Answer -

Bhopal भोपाल

The fictional boy character of the lsquoThe Jungle Bookrsquo by author Rudyard Kipling was

लखक डयाड िकपिलग ारा lsquoद जगल बकrsquo का कािनक बालक चर था

1 Kaa का

2 Mowgli मोगली

9)

10)

1)

3 Shere Khan शर खान

4 Bagheera बघीरा

Correct Answer -

Mowgli मोगली

The recipient of which one of the following awards given by Madhya Pradesh Government gets the highest cashprize

िनिलखत परारो म स िकसक ाकता को म दश सरकार ारा उतम नकद परार िदया जाता ह

1 Mahatma Gandhi award महाा गाधी परार

2 Shikar Award िशखर परार

3 Sharad Joshi Award शरद जोशी परार

4 Kishore Kumar Award िकशोर कमार परार

Correct Answer -

Mahatma Gandhi award महाा गाधी परार

Who among the following was the Trinidadian writer of Indian descent

िनिलखत म स भारतीय मल क ििनदािदयन लखक कौन थ

1 RK Narayan आर क नारायण

2 Anita Desai अनीता दसाई

3 Rudyard Kipling डयाड िकपिलग

4 VS Naipaul वीएस नायपॉल

Correct Answer -

VS Naipaul वीएस नायपॉल

Topic- GENERAL REASONING

Find the missing number ल त स या ात कर

4 196 16 169 ____ 144 64

1 81

2 36

3 32

4 21

2)

3)

4)

Correct Answer -

36

Find the average औसत ात कर

25 35 45 55 65 amp 75

1 48

2 50

3 47

4 49

Correct Answer -

50

The below series uses a sequence of alphabets and numbers Identify the incorrect combination

नीच दी गई खला अरो एव साओ का अनम योग करती ह गलत सयोजन पहचान

(i) FT85DF77ER

(ii) FT85DF77ER

(iii) FT85DE77ER

(iv) FT85DF77ER

1 ii

2 iv

3 i

4 iii

Correct Answer -

iii

A and B together can complete a piece of work in 10 days and B alone can complete the same work in 20days In how many days can A alone complete the work

A और B एक साथ काम क एक भाग को 10 िदनो म समा त करत ह और B अकल उसी काम को 20 िदनो म समा त करता हिकतन िदनो म A अकल उस काम को समा त कर सकता ह

1 20 days 20 िदन

2 30 days 30 िदन

3 10 days 10 िदन

4 25 days 25 िदन

5)

6)

Correct Answer -

20 days 20 िदन

Which one of the following four addresses is NOT EXACTLY same as the one given below

िनिलखत िदए गए चार पतो म स नीच िदए गए पत क ठीक समान कौन सा एक नही ह

Ground Floor 59 Goulburn St

Sydney NSW 2000 Australia

+61 (02) 8987 3700

(i) Ground Floor 59 Goulburn St

Sydney NSW 2000 Australia

+61 (02) 8987 3700

(ii) Ground Floor 59 Goulburn St

Sydney NSW 2000 Australia

+61 (02) 8987 3700

(iii) Ground Floor 59 Govlburn St

Sqdney NSW 2000 Australia

+61 (02) 8987 3700

(iv) Ground Floor 59 Goulburn St

Sydney NSW 2000 Australia

+61 (02) 8987 3700

1 ii

2 iv

3 i

4 iii

Correct Answer -

iii

In a group of 75 people 32 of them like cold drink 56 of them like general water and each person likesat least one of the two drinks How many people like both

75 लोगो क समह म उनम स 32 को को िड क पसद ह उनम स 56 को सामा जल पसद ह और क को दो म स कम स कम एक पय पसद ह िकतन लोगो को दोनो पसद ह

1 11

2 13

7)

1)

2)

3 12

4 14

Correct Answer -

13

If A = 1 FAT = 27 then FAITH =

यिद A = 1 FAT = 27 तो FAITH =

1 41

2 40

3 42

4 44

Correct Answer -

44

Topic- PEDAGOGY

CANCELLED

Basic education became a major initiative because of the efforts of

ाथिमक िशा िन न क यास स एक मह वपण कायम बन गई

1 Dr Radhakrishnan डॉ राधाक णन

2 Dr Rajendra Prasad डॉ राज साद

3 Mahatma Gandhi महा मा गाधी

4 Rabindranath Tagore रबी नाथ टगोर

Correct Answer -

Mahatma Gandhi महा मा गाधी

The ldquoclause of phraserdquo is an unit of perception of लॉज ऑफ ज िन न क धारणा क एक इकाई ह

1 Image छिव

2 Concept अवधारणा

3 Language भाषा

4 Thought िवचार

Correct Answer -

3)

4)

5)

6)

Language भाषा

Which method is most suitable to study communication process among students

छाो क बीच सचार िया का अयन करन क िलए कौन सी िविध सबस उपय ह

1 Case Study मामल का अयन (कस टडी)

2 Systematic Observation वथत अवलोकन

3 Experimental Method योगाक िविध

4 Introspection आ-िनरीण (इट ोस शन)

Correct Answer -

Systematic Observation वथत अवलोकन

Genes in a human being are located in मानवो म जीन इनम थत होता ह

1 cytoplasm कोिशका (साइटो ला म)

2 ribosomes राइबोसोम

3 cell membranes कोिशका िझी

4 chromosomes गणस (ोमोसोम)

Correct Answer -

chromosomes गणस (ोमोसोम)

With smaller classes teachers are much more able to ____________

छोटी काओ क साथ िशक ____________ म अिधक सम होत ह

1 Go slow while teaching िशण क दौरान धीमी गित स जान

2 Narrate more personal experiences अिधक गत अनभवो को बतान

3 Adapt instruction to individual differences among students छाो क बीच गत मतभदो क िलए अनकल िनदश दन म

4 Make use of the extra space for extra-curricular activities पातर गितिविधयो क िलए अितर जगह का उपयोग करन

Correct Answer -

Adapt instruction to individual differences among students छाो क बीच गत मतभदो क िलए अनकल िनदश दन म

While engaging in a task the child gets bored This is a sign of

एक काम म होन पर बा ऊब जाता ह यह िन का सकत ह

1 the task requiring a professional approach काय को ावसाियक िकोण की आवकता ह

7)

8)

9)

2 the task becoming mechanically repetitive काय यािक प स दोहराव वाला ह

3 the child not being intelligent बा बमान नही ह

4 the child being incapable of learning बा सीखन म असमथ ह

Correct Answer -

the task becoming mechanically repetitive काय यािक प स दोहराव वाला ह

Educational Psychologists are more concerned with the learning in __________

शिणक मनोवािनक __________ म अिधगम क साथ अिधक िचितत होत ह

1 Formal environment औपचारक वातावरण

2 Informal environment अनौपचारक वातावरण

3 Physical environment भौितक वातावरण

4 Social environment सामािजक वातावरण

Correct Answer -

Formal environment औपचारक वातावरण

Special needs education is the type of education िवशष ज़रतो वाली िशा वह िशा होती ह जो

1 Given to person with disabilities अम य को दी जाती ह

2 Given to people from remote areas दर थ ो क लोगो को दी जाती ह

3 Provided to intelligent people बमान लोगो को दी जाती ह

4 Established by colonial masters औपिनविशक मखयाओ ारा थािपत की गई ह

Correct Answer -

Given to person with disabilities अम य को दी जाती ह

The Stanford-Binet scale of intelligence was first published in the year

ब क नफोड-िबनट कल को इस वष म पहली बार कािशत िकया गया था

1 1916

2 1903

3 1908

4 1900

Correct Answer -

1916

10)

11)

12)

13)

The term lsquofictional finalismrsquo was propounded by

पद lsquoकत योजनवादrsquo (िफ शनल फाइनिल म) इनक ारा ितपािदत िकया गया था

1 Skinner नर

2 Freud ायड

3 Adler एडलर

4 Pavlov पावलोव

Correct Answer -

Adler एडलर

Rational Emotive Behavior Therapy was propounded by

तक सगत भावनाक वहार थरपी िन क ारा ितपािदत की गई थी

1 Carl Jung काल यग

2 Carl Rogers काल रोजस

3 Aaron Beck आरोन बक

4 Albert Ellis अट एिलस

Correct Answer -

Albert Ellis अट एिलस

Which of the following indicates the quality of education in a school

िनिलखत म स या एक कल म िशा की गणव ता का सकतक ह

1 Text-books and Teaching-learning material पा-प तक तथा िशण व सीखन स सबिधत सामी

2 Infrastructural facilities at the school कल म आधारभत ढाच स सबिधत सिवधाए

3 Student achievement level िवािथयो का उपल तर

4 Classroom systems का की यव था

Correct Answer -

Student achievement level िवािथयो का उपल तर

Which of the following teachers can be identified with authoritarian teaching style

िनिलखत म स कौन सा िशक अिधकारवादी िशण शली क साथ पहचाना जा सकता ह

1 Laissez-faire teacher अब ध िशक

14)

15)

2 Democratic teacher लोकतीय िशक

3 Indifferent teacher िन प िशक

4 Direct instruction teacher िनदश िशक

Correct Answer -

Direct instruction teacher िनदश िशक

Who was the pioneer of classical conditioning

िचरितित ानकलन ( लािसकल कडीशिनग) क वतक कौन थ

1 Skinner नर

2 Pavlov पावलोव

3 Kohler कोहलर

4 Freud ायड

Correct Answer -

Pavlov पावलोव

Which of the following are true with reference to short term memory

1 Limited capacity

2 Brief storage of information

3 Unlimited capacity

4 Duration of storage less than twenty seconds

अ पकािलक मित क सदभ म िन न म स या स य ह

1 सीिमत मता

2 सचना का सि त भडारण

3 असीिमत मता

4 बीस सकड स कम भडारण की अविध

1 3 and 4 3 और 4

2 2 and 4 2 और 4

3 13 and 4 13 और 4

4 12 and 4 12 और 4

Correct Answer -

12 and 4 12 और 4

1)

2)

3)

4)

Topic- GEOGRAPHY

Which of the following ranges of population size is used to define Class-III city by Indian Census

भारतीय जनगणना ारा वग-III शहर को परभािषत करन क िलए िनिलखत म स िकस रज क जनसा आकारका उपयोग िकया जाता ह

1 20 000 to 49999 20 000 स 49999

2 30000 to 59999 30000 स 59999

3 24000 to 54999 24000 स 54999

4 50000 to 99999 50000 स 99999

Correct Answer -

20 000 to 49999 20 000 स 49999

Carbonaceous rocks which produce coal and oil belong to the category of rocks called_______

कोयल और तल का उादन करन वाली काबनय चान ______ नामक चानो की णी स सबिधत ह

1 metamorphic पातरत

2 sedimentary अवसादी

3 inorganic अजिवक

4 igneous आय

Correct Answer -

sedimentary अवसादी

The ruhr-complex is a major industrial centre in र-परसर िन का एक मख औोिगक क ह

1 North America उरी अमरका

2 Russia स

3 Germany जमनी

4 Europe यरोप

Correct Answer -

Germany जमनी

The term lsquoRegurrsquo refers to श lsquoरगरrsquo ______ स सबिधत ह

1 Deltaic alluvial soils डा जलोढ़ िमी

ि ी

5)

6)

2 Laterite soils लटराइट िमी

3 Red and yellow soils लाल और पीली िमी

4 Black cotton soils काली कपास िमी

Correct Answer -

Black cotton soils काली कपास िमी

Read the given statements and answer which of the following options isare correct

1 Lower the pressure greater the atmospheric disturbance

2 Air move from higher to low pressure

िदए गए कथन को पढ़ और उर द िक िन म स कौन सास िवक सही ह

1 िजतना दाब कम होगा वायमडलीय बाधाए उतनी अिधक होगी

2 वाय उ स िन दाब की ओर गित करती ह

1 Both Statements 1 and 2 are correct दोनो कथन 1 और 2 सही ह

2 Both Statements 1 and 2 are wrong दोनो कथन 1 और 2 गलत ह

3 Statement 1 is wrong and only Statement 2 is correct कथन 1 गलत ह और कवल कथन 2 सही ह

4 Statement 1 is correct and Statement 2 is wrong कथन 1 सही ह और कथन 2 गलत ह

Correct Answer -

Both Statements 1 and 2 are correct दोनो कथन 1 और 2 सही ह

CANCELLED

1 4 1 2 and 3 4 1 2 और 3

2 4 1 3 and 2 4 1 3 और 2

3 1 4 2 and 3 1 4 2 और 3

4 1 4 3 and 2 1 4 3 और 2

Correct Answer -

7)

8)

9)

1 4 3 and 2 1 4 3 और 2

CANCELLED

Karl Pearsonrsquos correlation co-efficient is काल िपयसन का सहसबध गणाक ह

1 Arithmec mean समार मा

2 Geometric mean गणोर मा

3 None of these इनम स कोई नही

4 Harmonic mean हराक मा

Correct Answer -

Geometric mean गणोर मा

CANCELLED

ldquoEach day is more or less the same the morning is clear and bright with a sea breeze as the Sun climbshigh in the sky heat mounts up dark clouds form then rain comes with thunder lighting But rain is soonoverrdquo Which of the following regions is described in the above passage

ldquoक िदन समान स अिधक या कम होता ह सम की हवा क साथ और उल सबह होती ह जस सयआकाश म ऊचा चढ़ता ह गम बढ़ जाती ह काल बादल बनत ह िफर िबजली क साथ बारश आती ह लिकनबारश जी ख हो जाती हlsquorsquo उपरो पा म िनिलखत म स िकन ो का वणन िकया गया ह

1 Equatorial भमरखीय

2 Equatorial भमरखीय

3 Savannah सवाना

4 Mediterranean आातरक (भमसागरीय)

5 Mediterranean आातरक (भमसागरीय)

6 Monsoon मानसन

7 Monsoon मानसन

Correct Answer -

Equatorial भमरखीय

Equatorial भमरखीय

CANCELLED

In which epoch of the geological history of the Earth dinosaurs reached their largest size

पी क भगभय इितहास क िकस यग म डायनासोर अपन सबस बड़ आकार तक पचि

10)

11)

1 Triassic ट ायिसक

2 Jurassic जरािसक

3 Cretaceous ीटशस

4 Permian पिमयन

Correct Answer -

Jurassic जरािसक

CANCELLED

A spring tide would occur in which of the following conditions

िनिलखत म स िकन थितयो म ार-भाटा आता ह

1 When the Sun Moon and Earth are in a straight line

जब सय चमा और पी एक सीधी रखा म होत ह

2 When the Moon and Earth are in right angle to each other

जब चमा और पी एक-दसर क दािहन कोण म होत ह

3 When the Earth and Moon are in right angle to the Sun

जब पी और चमा सय क दािहन कोण म होत ह

4 When the Sun and Moon are in right angle to each other

जब सय और चमा एक-दसर क दािहन कोण म होत ह

Correct Answer -

When the Sun Moon and Earth are in a straight line

जब सय चमा और पी एक सीधी रखा म होत ह

CANCELLED

An observe on the Earthrsquos surface always sees the same face of the moon because

एक पयवक को पी की सतह स हमशा चाद का एक ही फलक िदखाई दता ह ोिक

1 Its path of revolution around the earth is the same as that of the earth around the Sun

इसका पी क चारो ओर परमण का माग सय क चारो ओर पी क समान ही ह

2 Its period of revolution around the Earth is the same as its period of rotation around its own axis

इसकी पी क चारो ओर परमण की अविध उसकी अपनी धरी क चारो ओर घणन की अविध क समान ह

3 Its period of rotation is the same as that of the Earth इसकी घणन की अविध पी क समान ह

ी ि ी ी

12)

13)

14)

4 Its direct of rotation is the same as that of Earth घणन की िदशा पी क समान ही ह

Correct Answer -

Its period of revolution around the Earth is the same as its period of rotation around its own axis

इसकी पी क चारो ओर परमण की अविध उसकी अपनी धरी क चारो ओर घणन की अविध क समान ह

CANCELLED

The pebbles that are faceted by the sand-blasting and shaped polished by the wind abrasions are known as

पवन अपघषन ारा पॉिलश रत-िवोिटत और साच म ढला ककड़ ___________ क प म जाना जाता ह

1 Dreikanter िकोणक

2 Pediments िकोिनका

3 Inselberg इलबग

4 Dunes टीबा

Correct Answer -

Dreikanter िकोणक

CANCELLED

Astronomical unit is the average distance between खगोलीय इकाई ______ क बीच की औसत दरी ह

1 Earth and Mars पी और मगल

2 Earth and mercury पी और बध

3 Earth and moon पी और चमा

4 Earth and Sun पी और सय

Correct Answer -

Earth and Sun पी और सय

During cold weather season in the northern plains there will be an inflow of cyclonic disturbancesfrom the _________ directions

शीत मौसम क दौरान उरी मदानी इलाको म _________ िदशाओ स चवात सबधी गड़बड़ी का अतवाह होगा

1 East and Northwest पव और उरपिम

2 East and Northeast पव और पवर

3 West and East पिम और पव

ि औ ि

15)

16)

17)

4 West and Northwest पिम और उरपिम

Correct Answer -

West and Northwest पिम और उरपिम

During an earth quake the velocity of the body waves will________ along with the increase in densityof the material it is passing through

भकप क दौरान लहरो क ऊपरी भाग का वग घन म व क साथ-साथ ________ जो इसस गजरन वाली वको आग बढाएगी

1 not change नही बदलगा

2 increase initially and then decrease शआत म बढ़गा और िफर घटगा

3 increase बढ़गा

4 decrease घटगा

Correct Answer -

increase बढ़गा

The Clouded Leopard National park is situated in which of the following states

िनिलखत म िकस रा म धिमल तदआ रा ीय उान (ाउडड लपड नशनल पाक ) थत ह

1 Tripura िपरा

2 Uttar Pradesh उर दश

3 Assam असम

4 Mizoram िमजोरम

Correct Answer -

Tripura िपरा

Usually the land surfaces are heated more quickly than the water surfaces because _____________

आम तौर पर जल सतहो की तलना म भिम सतह अिधक तजी स गम होती ह ोिक _____________ ह

1 the specific heat of water is higher than land पानी की िविश ऊा भिम स अिधक

2 the specific heat of water is lesser than land पानी की िविश ऊा भिम स कम होती

3 the latent heat of water is higher than the land पानी की अतिनिहत ऊा भिम स अिधक

4 the land reflects more heat radiation than water भिम पानी की तलना म अिधक ऊा क िविकरण को पराविततकरती

18)

19)

20)

21)

Correct Answer -

the specific heat of water is higher than land पानी की िविश ऊा भिम स अिधक

The longest shore-line is along the state of सबस लबी समतटीय रखा िन रा क साथ ह

1 Maharashtra महारा

2 Orissa उड़ीसा

3 Kerala करल

4 Gujarat गजरात

Correct Answer -

Gujarat गजरात

The position when the Earth is farthest from the Sun is known as

जब पी सय स सबस दर होती ह तो उस थित को िन नाम स जाना जाता ह

1 Perihelion उपसौर

2 Vernal Equinox बसत िवषव

3 Aphelion अपसौर

4 Autumnal Equinox शराल िवषव

Correct Answer -

Aphelion अपसौर

The seasonal reversal of winds is the typical characteristic of

हवाओ का मौसमी परवतन ______ की सामा िवशषता ह

1 Mediterranean climates only कवल भमसागरीय जलवाय

2 All of the above climates उपय सभी मौसम

3 Monsoon climate only कवल मानसन जलवाय

4 Equatorial climate only कवल भमरखीय जलवाय

Correct Answer -

Monsoon climate only कवल मानसन जलवाय

In _________ rocks the minerals will occurs in beds or layers

ो ि ो ो

22)

23)

24)

______ चानो म खिनज तल या परतो म होत ह

1 metamorphic कायातरत

2 igneous and metamorphic आय और कायातरत

3 igneous आय

4 sedimentary अवसादी

Correct Answer -

sedimentary अवसादी

Black soil is ideal for the cultivation of cotton as कपास की खती क िलए काली िमी आदश ह ोिक

1 Its colour is black यह काली होती ह

2 It is found on plateau regions यह पठार ो म पायी जाती ह

3 It is made up of lava यह लावा स बनी होती ह

4 It can retain moisture यह नमी को बरकरार रख सकती ह

Correct Answer -

It can retain moisture यह नमी को बरकरार रख सकती ह

The National Survey and Mapping Organization of the country works under the Department of___________

दश का रा ीय सवण और मानिचण सगठन ___________ िवभाग क अतगत काय करता ह

1 Space अतर

2 Science and Technology िवान और तकनीक

3 Culture सित

4 Tourism पयटन

Correct Answer -

Science and Technology िवान और तकनीक

Palk strait separates India from पाक जलडमम भारत स _____ को अलग करता ह

1 Pakistan पािकान

2 Andaman Island अडमान ीप

3 China चीन

25)

26)

27)

4 Sri Lanka ीलका

Correct Answer -

Sri Lanka ीलका

Which among the following state is the major producer of Bauxite in India

िनिलखत म स कौन सा रा भारत म बॉाइट का मख उादक ह

1 Madhya Pradesh मदश

2 Rajasthan राजथान

3 Goa गोवा

4 Orissa उड़ीसा

Correct Answer -

Orissa उड़ीसा

Which of the following states DOES NOT share border with Chhattisgarh

िनिलखत म स कौन सा रा छीसगढ़ क साथ सीमा साझा नही करता ह

1 Telangana तलगाना

2 Uttar Pradesh उर दश

3 Bihar िबहार

4 Andhra Pradesh आ दश

Correct Answer -

Bihar िबहार

Which of the following statements is INCORRECT with respect to parallels of latitudes

अाश क समानातरो क सबध म िन निलखत म स कौन सा कथन गलत ह

1 A line joining places of equal latitude is known as parallel of largest

समान अाश क थानो को जोड़न वाली रखा को िवशालतम क समानातर क प म जाना जाता ह

2 They stat from equator and run parallels to it

व भम रखा स ारभ होत ह और इसक समानातर चलत ह

3 All parallels are equal in length सभी समातर लबाई म समान ह

4 All parallels are drawn as circles on the globe ोब पर सभी समानातर वो क प म खीच जात ह

28)

29)

30)

31)

Correct Answer -

All parallels are equal in length सभी समातर लबाई म समान ह

Which of the following Indian states is also known as a lsquoLand of Red river and Blue Hillsrsquo

िनिलखत म स िकस भारतीय रा को लाल नदी और नीली पहािड़यो की भिम क नाम स जाना जाता ह

1 Uttarkhand उराखड

2 Assam असम

3 Meghalaya मघालय

4 Arunachal Pradesh अणाचल दश

Correct Answer -

Assam असम

In spatial analysis of settlement Rn = 215 indicates which type of settlement arrangement

िनपटान क थािनक िवषण म Rn = 215 यह इिगत करता ह िक िकस कार की िनपटान वथा ह

1 Uniform यिनफॉम

2 Semi-Clustered समी- ल टर

3 Clustered ल टर

4 Random रडम

Correct Answer -

Uniform यिनफॉम

Who are known as the lsquoYellow Peoplersquo lsquoयलो पीपलrsquo क प म कौन जाना जाता ह

1 Mongoloids मोगोलोइडस

2 Nigroids नीोइडस

3 Australoids ऑ लॉइडस

4 Caucasoids कॉकसोइडस

Correct Answer -

Mongoloids मोगोलोइडस

ि ि ो ौ ि

32)

33)

34)

Who publishes the topographical map of India भारत क थलाकितक मानिच को कौन कािशत करता ह

1 Geographical Survey of India भारत का भौगोिलक सवण

2 Government of India भारत सरकार

3 Geological Survey of India भारत क भगभय सवण

4 Survey of India भारत का सवण

Correct Answer -

Survey of India भारत का सवण

Who among the following claimed geography to be the lsquoEcology of Manrsquo

िनिलखत म स िकसन भगोल को मन का पारथितकी कहा ह

1 Alfred Hener अड हटनर

2 Vidal-de la Blache वाइडल-िड लॉ ॉश

3 Oo Schluter ओटो टर

4 Harlan Barrow हरलन बारो

Correct Answer -

Harlan Barrow हरलन बारो

Who among the following is regarded as the founder of humanistic approach in geography

िनिलखत म स िकस भगोल म मानवतावादी िकोण का सथापक माना जाता ह

1 William Bunge िविलयम बग

2 Yi-Fu-Tuan यी-फ- यान

3 Brain JL Berry न जएल बरी

4 Richard Peet रचड पीट

Correct Answer -

Yi-Fu-Tuan यी-फ- यान

Who prepared Lorenz curve लोरज व िकसन तयार िकया

1 Geddes गडस

2 None of these इनम स कोई नही

3 Griffith Taylor ििफथ टलर

35)

36)

37)

4 Max U Lorenz म य लोरज

Correct Answer -

Max U Lorenz म य लोरज

Gulf Streams are the currents of which of the following oceans

खाड़ी की धाराए िनिलखत महासागरो म स िकसकी धाराए ह

1 North Atlantic Ocean उरी अटलािटक महासागर

2 North Pacific Ocean उरी शात महासागर

3 Arabian Sea अरब सागर

4 South Pacific Ocean दिण शात महासागर

Correct Answer -

North Atlantic Ocean उरी अटलािटक महासागर

Disintegration wearing away and removal of rock material is generally referred as

िशला पदाथ (रॉक सामी) का टटना िमटना और हटना आमतौर पर ________ क प म सदिभत िकया जाता ह

1 Shattering िवसकारक

2 Denudation अनाादन

3 Fault श

4 Decomposition िवयोजन

Correct Answer -

Denudation अनाादन

Variations in the length of day time and night from season to season are due to

मौसम स मौसम परवतन पर िदन क समय और रात क समय की अविध म िभताए िन कारण स होती ह

1 The Earthrsquos revolution round the Sun in an elliptical manner पी का दीघवाकार तरीक स सय क चारो घणन

2 The Earthrsquos rotation on its axis पी का इसकी धरी पर घणन

3 Revolution of the Earth on a tilted axis नत अ पर पी का घणन

4 Latitudinal position of the place थान की अाश थित

Correct Answer -

Revolution of the Earth on a tilted axis नत अ पर पी का घणन

38)

39)

40)

Point out the correct sequence of mountain ranges from north to south

उर स दिण तक पवत खलाओ क सही अनम को इिगत कर

1 Great Himalaya Middle Himalaya Outer Himalaya Trans Himalaya

महान िहमालय म िहमालय बा िहमालय परा िहमालय

2 Middle Himalaya Great Himalaya Trans Himalaya Outer Himalaya

म िहमालय महान िहमालय परा िहमालय बा िहमालय

3 Outer Himalaya Middle Himalaya Great Himalaya Trans Himalaya

बा िहमालय म िहमालय महान िहमालय परा िहमालय

4 Trans Himalaya Great Himalaya Middle Himalaya Outer Himalaya

परा िहमालय महान िहमालय म िहमालय बा िहमालय

Correct Answer -

Trans Himalaya Great Himalaya Middle Himalaya Outer Himalaya

परा िहमालय महान िहमालय म िहमालय बा िहमालय

Sunrsquos halo is produced by the refraction of light in सय का भामडल ______ म काश क अपवतन ाराउ होता ह

1 Ice crystals in Cirrro-Cumulus clouds पाभ-कपास मघो क बफ िल

2 Ice crystal in Cirrus clouds पाभमघो क बफ िल

3 Dust particles in Stratus clouds री मघो क धल कण

4 Water vapour in Stratus clouds री मघो क जल वा

Correct Answer -

Ice crystal in Cirrus clouds पाभमघो क बफ िल

Read the given statements and answer which of the following options isare correct

(1) The minerals present in the rocks exposed to atmosphere are not subjected to alteration

(2) Oxidation is one of the processes of chemical weathering

िदए गए कथनो को पढ़ और उर द िक िन म स कौन सास िवक सही ह

(1) वायमल स अनावत शल म उपथत खिनज परवतन क अधीन नही होता ह

(2) ऑीकरण रासायिनक अपय की ियाओ म स एक ह

ो ो

41)

42)

1 Both statements are wrong दोनो कथन गलत ह

2 Both statements are correct दोनो कथन सही ह

3 First statement is wrong and second statement is correct पहला कथन गलत ह और दसरा कथन सही ह

4 First statement is correct and second statement is wrong पहला कथन सही ह और दसरा कथन गलत ह

Correct Answer -

First statement is wrong and second statement is correct पहला कथन गलत ह और दसरा कथन सही ह

Read the given statements and answer which of the following options isare correct

1 Sunrsquos short waves enter the earth partially heating the atmosphere

2 Heated earth surface from the sun produces broader waves which interacts and heats the atmosphere

िदए गए कथन को पढ़ और उर द िक िन म स कौन सास िवक सही ह

1 सय की छोटी तरग पी म आिशक प स वश करती ह और वायमडल को ऊत करती ह

2 सय स ऊत पी की सतह िवारत तरग उ करती ह जो परर भाव डालती ह और वायमडल कोऊत करती ह

1 Both Statements 1 and 2 are correct दोनो कथन 1 और 2 सही ह

2 Both Statements 1 and 2 are wrong दोनो कथन 1 और 2 गलत ह

3 Statement 1 is wrong and only Statement 2 is correct कथन 1 गलत ह और कवल कथन 2 सही ह

4 Only statement 1 is correct कवल कथन 1 सही ह

Correct Answer -

Both Statements 1 and 2 are correct दोनो कथन 1 और 2 सही ह

Read the given statements and answer which of the following options isare correct

(1)The rocks that get changed due to heat and pressure are termed as metamorphic rocks

(2)Slate is one such type of metamorphic rock

िदए गए कथनो को पढ़ और उर द िक िन म स कौन सास िवक सही ह

(1) शल जो ऊा और दाब क कारण परवितत हो जात ह उ कायातरक शलो क प म जाना जाता ह

(2) ट एक तरह का कायातरक शल ह

1 Both statements are wrong दोनो कथन गलत ह

2 Both statements are correct दोनो कथन सही ह

3 First statement is wrong and second statement is correct पहला कथन गलत ह और दसरा कथन सही ह

ी औ

43)

44)

4 First statement is correct and second statement is wrong पहला कथन सही ह और दसरा कथन गलत ह

Correct Answer -

Both statements are correct दोनो कथन सही ह

Read the given statements and answer which of the following options isare correct

1 Higher temperature anomaly is observed in the northern hemisphere

2 Differential heating is absent in Northern Hemisphere

िदए गए कथनो को पढ़ और उर द िक िन म स कौन सास िवक सही ह

1 उरी गोलाध म उ तापमान िवसगित पायी जाती ह

2 उरी गोलाध म अतर ऊन अनपथत होती ह

1 Both Statements 1 and 2 are correct दोनो कथन 1 और 2 सही ह

2 Both Statements 1 and 2 are wrong दोनो कथन 1 और 2 गलत ह

3 Statement 1 is wrong and Statement 2 is correct कथन 1 गलत ह और कथन 2 सही ह

4 Statement 1 is correct and Statement 2 is wrong कथन 1 सही ह और कथन 2 गलत ह

Correct Answer -

Statement 1 is correct and Statement 2 is wrong कथन 1 सही ह और कथन 2 गलत ह

Read the given statements and answer which of the following options isare correct

(1) Plutonic rocks are intrusive type of igneous rocks

(2) It cools very slowly because the surrounding rock serves as insulation around the intrusion of magma

िदए गए कथनो को पढ़ और उर द िक िन म स कौन सास िवक सही ह

(1) िवतलीय शल अतवधी कार क आश शल ह

(2) यह बत धीर-धीर ठडा होता ह ोिक आस-पास क शल मा क अतवधन क चारो ओर रोधन क प म कायकरत ह

1 Both statements are wrong दोनो कथन गलत ह

2 Both statements are correct दोनो कथन सही ह

3 First statement is wrong and second statement is correct पहला कथन गलत ह और दसरा कथन सही ह

4 First statement is correct and second statement is wrong पहला कथन सही ह और दसरा कथन गलत ह

Correct Answer -

Both statements are correct दोनो कथन सही ह

45)

46)

47)

48)

The dust and ash material hurled from the volcanoes are termed as

ालामखी स िनकलन वाली धल और राख सामी को _______ क प म कहा जाता ह

1 Pyroclasc पाइरोाक

2 Hyperclastic हाइपराक

3 Hepiroclastic हिपरोाक

4 Cirroclastic िसरोाक

Correct Answer -

Pyroclasc पाइरोाक

The vertical difference in elevation between a low tide and high tide is referred as

कम ार और उ ार क बीच ऊचाई म लबवत अतर _____ स सदिभत होता ह

1 Tidal slope ारीय ढलान

2 Tidal elevation ारीय उयन

3 Tidal range ारीय परास

4 Tidal height ारीय ऊचाई

Correct Answer -

Tidal range ारीय परास

The maximum biodiversity is found in which of the following regions िनिलखत ो म स अिधकतमजव िविवधता िकसम पायी जाती ह

1 Amazon Basin अमज़न बिसन

2 East Indies ई इडीज

3 Congo Basin कागो बिसन

4 West indies व इडीज

Correct Answer -

Amazon Basin अमज़न बिसन

The cultivation of rice crop produces_______ चावल की फसल की खती ______ का उादन करती ह

1 SO2

49)

50)

51)

2 CH4

3 CFCs

4 CO2

Correct Answer -

CH4

The pressure system with higher pressure at the centre is called__________

क म उ दबाव वाली दबाव णाली को _______ कहा जाता ह

1 front अ

2 depression अवनमन

3 cyclone चवात

4 anti-cyclone ितचवात

Correct Answer -

anti-cyclone ितचवात

The Himalayan region is poor in mineral resources because िहमालयी खिनज ससाधनो म समनही ह ोिक

1 The displacement of rock strata has disturbed the arrangement of rocks and made it complex

शलीय परत क िवथापन न चानो की वथा को अवथत कर िदया ह और इस जिटल बना िदया ह

2 The climate conditions are not suitable for exploitation of minerals

जलवाय की थित खिनजो क दोहन क िलए उपय नही ह

3 The terrain makes explanation of minerals difficult and very costly due to transportation difficulties

भ-भाग परवहन की किठनाइयो क कारण खिनजो का दोहन मल और बत महगा बना दता ह

4 It is made up of crystalline rocks यह िलीय चानो स बना ह

Correct Answer -

The displacement of rock strata has disturbed the arrangement of rocks and made it complex

शलीय परत क िवथापन न चानो की वथा को अवथत कर िदया ह और इस जिटल बना िदया ह

The process through which the moisture is added to the atmosphere by vegetation is termed as

वह िया िजसक माम स वनित ारा वातावरण म नमी िमलायी जाती ह _______ क प म जानी जाती ह

52)

53)

54)

1 Condensation सघनन

2 Evapotranspiration वान-उजन

3 Radiation िविकरण

4 Precipitation वषण

Correct Answer -

Evapotranspiration वान-उजन

The process through which the terrestrial heat is transferred to air by direct contact is termed as

वह िया िजसम सपक ारा थलीय ऊा वाय म थानातरत हो जाती ह ______ क प म जानी जाती ह

1 Conduction चालन

2 Convection सवहन

3 Insolation आतपन

4 Radiation िविकरण

Correct Answer -

Conduction चालन

The largest area under mangroves is in which of the following statesunion territory

मोव क अतगत िनिलखत राोसघ शािसत दशो म स सबस बड़ा कौन सा ह

1 Andaman and Nicobar अमान और िनकोबार

2 Andhra Pradesh आ दश

3 West Bengal पिम बगाल

4 Gujarat गजरात

Correct Answer -

West Bengal पिम बगाल

The longitudinal transverse and surface waves in an earthquake originate from

भकप म दशातर अनथ और सतह तरग यहा उ होती ह

1 The focus on the surface of the Earth पी क सतह पर क -िबद म

2 The focus within the body of the Earth पी क भीतर क -िबद म

3 The epicenter within the body of the Earth पी क भीतर उपरक म

55)

56)

57)

4 The epicenter on the surface of the Earth पी क सतह पर उपरक म

Correct Answer -

The focus within the body of the Earth पी क भीतर क -िबद म

The down slope movement of material due to gravity is called______

गाकषण क कारण पदाथ की अनढाल गित को ______ कहा जाता ह

1 mass movement पदाथ सचलन

2 deposition िनप

3 erosion रण

4 volcanic movement ालामखीय सचलन

Correct Answer -

mass movement पदाथ सचलन

Shimla is cooler than Amritsar although both are on the same latitude This is because

िशमला म अमतसर स अिधक ठड ह हालािक दोनो समान अाश पर ह ऐसा ह ोिक

1 Shimla is at a greater height above sea level than Amritsar अमतसर की तलना म िशमला सम तल स अिधकऊचाई पर ह

2 Shimla is further north िशमला उर की ओर ह

3 Shimla is farther from the equator िशमला भम रखा स आग ह

4 Their longitudes differ उनकी दशातर रखाए िभ ह

Correct Answer -

Shimla is at a greater height above sea level than Amritsar अमतसर की तलना म िशमला सम तल स अिधकऊचाई पर ह

lsquoTempo of Urbanizationrsquo measures which of the following

lsquoशहरीकरण का टपोrsquo िनिलखत म स कौन सा उपाय ह

1 Speed of urbanizaon शहरीकरण की गित

2 None of the above इनम स कोई नही

3 Inequality of urbanizaon शहरीकरण की असमानता

4 Current level of urbanizaon शहरीकरण का वतमान र

Correct Answer -

58)

59)

60)

Speed of urbanizaon शहरीकरण की गित

Out of the following options choose the INCORRECT statement

िनिलखत िवको म स गलत कथन का चयन कर

1 The clear tracts in the equatorial region recover rapidly भम रखा म भभाग तजी स ठीक हो जात ह

2 The stable communities include a redwood forest a pine forest at high elevations

थर समदायो म एक रडवड वन उ ऊचाई पर एक दवदार वन शािमल ह

3 Any ecosystem moves towards maximum biomass and stability to survive

कोई भी पारथितकी त जीिवत रहन क िलए अिधकतम जवसहित और थरता की तरफ असर होता ह

4 Tropical rain forests near equator are stable ecosystems

भम रखा क पास उकिटबधीय वषा वन थर पारथितक त ह

Correct Answer -

The clear tracts in the equatorial region recover rapidly भम रखा म भभाग तजी स ठीक हो जात ह

Seasonal contrasts are maximum in मौसमी िवषमता अिधकतम ह

1 Mid latitudes म अाश म

2 Low attitudes िन अाश म

3 High latitudes उ अाश म

4 Subtropics उपोकिटबधीय म

Correct Answer -

Mid latitudes म अाश म

In India which type of forest among the following occupies the largest area

भारत म िनिलखत म स िकस कार क वन सबस बड़ा फल आािदत करत ह

1 Sub-tropical Dry Evergreen Forest उप उकिटबधीय श सदाबहार वन

2 Mountain Wet Temperate Forest पवतीय आ शीतो वन

3 Tropical Moist Deciduous Forest उकिटबधीय आ पणपाती वन

4 Tropical Wet Evergreen Forest उकिटबधीय आ सदाबहार वन

Correct Answer -

Tropical Moist Deciduous Forest उकिटबधीय आ पणपाती वन

61)

62)

63)

64)

What is the proportion of lsquoJuvenile Populationrsquo (0-14 years) in India as per 2011Census

2011 की जनगणना क अनसार भारत म जवनाइल पॉपलशन यानी िकशोर जनस या (0-14 वष) का अनपात ाह

1 3076 of total population कल जनस या का 3076

2 2764 of total population कल जनस या का 2764

3 2933 of total population कल जनस या का 2933

4 3354 of total population कल जनस या का 3354

Correct Answer -

3076 of total population कल जनस या का 3076

What is the Belfast famous for बलफा िकसक िलए मशर ह

1 Belt of cotton textile industry कपास व उोग क

2 Ship-building industry जहाज िनमाण उोग

3 Agricultural machinery किष उपकरण

4 Aero planes manufacturing वाययान िनमाण

Correct Answer -

Ship-building industry जहाज िनमाण उोग

What is the most important occupation in tropical monsoon lands

उकिटबधीय मॉनसन भिम म सबस महपण वसाय ा ह

1 Mining खनन

2 Cattle rearing मवशी पालन

3 Agriculture किष

4 Nomadic herding नोमािडक जड़ी-बिटया

Correct Answer -

Agriculture किष

What is the most important characteristics of the islands (Indian) located in the Arabian sea

अरब सागर म थत ीपो (भारतीय) की सबस महपण िवशषता ा ह

ी ो

65)

66)

67)

1 There are all of coral origins सभी कोरल मल क ह

2 There are all very small in size य सभी आकार म बत छोट ह

3 They have a very dry climate इनकी जलवाय बत श ह

4 They are extended parts of the mainland व महाीप क िवारत िह ह

Correct Answer -

There are all of coral origins सभी कोरल मल क ह

What do the basalt layers of the Deccan indicate डन की बसा परत ा इिगत करती ह

1 All of the above उपरो सभी

2 Huge volcanic eruptions in the distant past दरथ अतीत म िवशाल ालामखीय िवोट

3 The immense erosional activity of the rivers निदयो की िवशाल रण गितिविध

4 The influence of weathering मौसम का भाव

Correct Answer -

Huge volcanic eruptions in the distant past दरथ अतीत म िवशाल ालामखीय िवोट

In the structure of planet Earth below the mantle the core is mainly made up of_____

पी ह की सरचना म मटल क नीच कोर म प स______ स िनिमत होती ह

1 aluminium एमीिनयम

2 silicon िसिलकॉन

3 chromium ोिमयम

4 iron लोहा

Correct Answer -

iron लोहा

One of the major Mid Oceanic Ridge is found in मख म-महासागर चोिटयो म स एक ______ म पायाजाता ह

1 Mid Pacific Ocean म शात महासागर

2 Mid Atlantic Ocean म अटलािटक महासागर

3 Mid Indian Ocean म भारतीय महासागर

4 Mid Arctic Ocean म आक िटक महासागर

68)

69)

70)

71)

Correct Answer -

Mid Atlantic Ocean म अटलािटक महासागर

Magma that reaches the Earthrsquos surface and then solidifies is called________

मा जो पी की सतह तक पचती ह और िफर ठोस हो जाती ह ________कहलाती ह

1 quartz ाटज

2 lava लावा

3 granite नाइट

4 silicates िसिलकट

Correct Answer -

lava लावा

Isotherms are the lines of equal_______ समताप रखाए समान _______की रखाए होती ह

1 pressure दाब

2 temperature तापमान

3 rainfall वषा

4 height ऊचाई

Correct Answer -

temperature तापमान

Mark the correct sequence of passes in the Western Ghats from north to south

पिमी घाटो म उर स दिण तक दर क सही अनम को िचित कर

1 Thalghat Palghat Bhorghat थलगघाट पालघाट भोरघाट

2 Thalghat Bhorghat Palghat थलघाट भोरघाट पालघाट

3 Bhorghat Thalghat Palghat भोरघाट थलघाट पालघाट

4 Palghat Bhorghat Thalghat पालघाट भोरघाट थलघाट

Correct Answer -

Thalghat Bhorghat Palghat थलघाट भोरघाट पालघाट

Which of the following does not have influence over the climate in India

ि ि ि ी ी

72)

73)

िनिलखत म स िकसका भाव भारत की जलवाय पर नही पड़ता ह

1 Ocean currents सागर की लहर

2 Nearness to equator भम रखा स िनकटता

3 Monsoons मानसन

4 Presence of Indian ocean भारतीय महासागर की उपथित

Correct Answer -

Ocean currents सागर की लहर

Which of the following cloud types has the characteristics like vertical tall narrow and puffy

िनिलखत म स िकस कार क मघो म लबवत लबी सकीण और थलता जसी िवशषताए ह

1 Cumulonimbus तफानी मघ

2 Cumulus मघ पज

3 Cirrocumulus पाभ कपासी मघ

4 Nimbostratus वषारी मघ

Correct Answer -

Cumulus मघ पज

Which of the following statement is INCORRECT about Crude Birth Rate

िनिलखत स कौन सा कथन अशोिधत ज दर क बार म सही नही ह

1 It cannot be used for comparing fertility level between two countries with different population characteristics

इसका उपयोग िविभ जनसा िवशषताओ वाल दो दशो क बीच जनन र की तलना क िलए नही िकया जा सकता ह

2 It is a standardized measure of fertility

यह जनन मता का मानकीकत उपाय ह

3 It is effected by the age-sex composition of the population

यह आबादी की आय-िलग सरचना स भािवत होता ह

4 It is expressed per 1000 population in a given geographical unit

यह िकसी दी गई भौगोिलक इकाई म ित 1000 जनसा पर िकया जाता ह

Correct Answer -

It is a standardized measure of fertility

यह जनन मता का मानकीकत उपाय ह

74)

75)

76)

77)

Which of the following state in India experienced negative decadal growth rate during 2001 to 2011census

भारत म िनिलखत म स िकस रा म वष 2001 स 2011 की जनगणना क दौरान नकाराक िगरावट दर ई

1 Tripura िपरा

2 Nagaland नागालड

3 Haryana हरयाणा

4 Odisha ओिडसा

Correct Answer -

Nagaland नागालड

Which of the following is NOT a characteristic of peninsular rivers

िनिलखत म स कौन सी िवशषता ायीपीय निदयो म नही होती ह

1 Flow through shallow valleys उथल घािटयो क माम स वाह

2 Seasonal flow मौसमी वाह

3 Little erosional activity थोड़ी कटावदार गितिविध

4 Meandering tendency often shifting their beds घमावदार वि अर अपन तटो को थानातरत करना

Correct Answer -

Meandering tendency often shifting their beds घमावदार वि अर अपन तटो को थानातरत करना

Which of the following gases in the atmosphere absorbs heat from the Sunrsquos radiation and the Earthssurface

वायमडल म िनिलखत म स कौन सी गस सय क िविकरण और पी की सतह स ऊा को अवशोिषत करती ह

1 Neon िनयॉन

2 Carbon dioxide काबन डाइऑाइड

3 Argon आगन

4 Nitrogen नाइट ोजन

Correct Answer -

Carbon dioxide काबन डाइऑाइड

Which of the following kind of settlement pattern is found at the confluence of rivers

ि ि ि ि ो

78)

79)

80)

िनिलखत म स िकस कार का वथापन पटन निदयो क सगम पर पाया जाता ह

1 Triangular Paern िकोणीय पटन

2 Circular or Semi-Circular Paern परप या अध-परप पटन

3 Nebular Paern नबलर पटन

4 Star ndashShaped Paern ार-आकार का पटन

Correct Answer -

Triangular Paern िकोणीय पटन

Which one was not the objective of the Biosphere Reserve Projects launched by the UNESCO

यनो ारा श की गई सरित जवमडल परयोजनाओ का उ इनम स कौन सा नही था

1 To promote teaching and research िशण और अनसधान को बढ़ावा दना

2 To make agriculture sustainable किष को दीघकािलक बनाना

3 To conserve ecosystems पारथितक त को सरित करना

4 To conserve genetic diversity for a longtime लब समय तक अनवािशक िविवधता को सरित करना

Correct Answer -

To make agriculture sustainable किष को दीघकािलक बनाना

Which region of the Earth surface receives the highest amount of insulation

पी सतह का कौन सा तापावरोधन की उतम माा ा करता ह

1 Land mass थलखड

2 Savannah region सवाना

3 Water bodies जल िनकाय

4 Tropical desert उकिटबधीय रिगान

Correct Answer -

Tropical desert उकिटबधीय रिगान

Which one of the following is not a biodiversity hotspot

िनिलखत म स कौन सा जव िविवधता का म जगह नही ह

1 Eastern Himalaya पव िहमालय

2 Eastern Ghats पव घाट

81)

82)

83)

3 Indo-Myanmar भारत-ामार

4 Westerm Ghats पिमी घाट

Correct Answer -

Eastern Ghats पव घाट

Which one of the following is NOT a part of the World Network of Biosphere Reserves based on theUNESCO Man and Biosphere Programme

यनो मन और बायोीयर कायम क आधार पर िनिलखत म स कौन बायोीयर रजव क िव नटवक कािहा नही ह

1 Gulf of Mannar मार की खाड़ी

2 Seshachalam शषाचलम

3 Sunderban सदरबन

4 Nilgiri नीलिगर

Correct Answer -

Seshachalam शषाचलम

Which one of the following is an example of ldquodesert vegetationrdquo

िनिलखत म स कौन मथलीय वनित का एक उदाहरण ह

1 Mosses and lichens दलदल और शवाल

2 Temperate grassland समशीतो घास क मदान

3 Coniferous forest शकधारी वन

4 Acacia and cactus एकािसया और कस

Correct Answer -

Acacia and cactus एकािसया और कस

Which one of the following reflects more sunlight िनिलखत म स कौन सा सय की रोशनी को अिधकपरावितत करता ह

1 Paddy crop land धान फसल भिम

2 Land covered with fresh snow ताजा बफ स आािदत भिम

3 Sand desert रतीली रिगान

4 Prairie land यरी भिम

84)

85)

86)

87)

Correct Answer -

Land covered with fresh snow ताजा बफ स आािदत भिम

Which layer of the atmosphere is in contact with the surface of the earthrsquos oceans

वायमडल की कौन सी परत पी क महासागरो की सतह क सपक म ह

1 Stratosphere समताप मडल

2 Mesosphere म मडल

3 Hydrosphere जलमडल

4 Troposphere ोभ मडल

Correct Answer -

Troposphere ोभ मडल

Mediterranean Sea is a border of which of the following countries भम सागर िनिलखत दशो म सिकसकी सीमा ह

1 None of these इनम स कोई नही

2 Iraq इराक

3 Lebanon लबनान

4 Jordan जॉडन

Correct Answer -

Lebanon लबनान

Benguela ocean currents are found along which coast बगएला महासागर धाराए िकस तट क साथ पायीजाती ह

1 East Coast of South America दिण अमरका क पव तट

2 East Coast of Africa अीका क पव तट

3 West Coast of South America दिण अमरका क पिमी तट

4 West Coast of Africa अीका क पिमी तट

Correct Answer -

West Coast of Africa अीका क पिमी तट

88)

89)

90)

Due to tension a block of land on one side being pushed up or upthrown relative to the downthrown blockis referred as

तनाव क कारण नीच फ क ए खड क साप भिम का एक खड एक ओर स ऊपर धकला जाता ह या ऊपर की ओरफ का जाता ह यह _____ क प म सदिभत ह

1 Thrust fault प श

2 Normal fault सामा श

3 Reverse fault म श

4 Strike slip fault नितलब सपण श

Correct Answer -

Normal fault सामा श

Inter-tropical doldrums is a zone of ______ अतर-उकिटबधीय डोलड ______ का एक ह

1 Frontolysis टोलायिसस

2 Convergence अिभसरण

3 Inter-tropical divergence zone अतर-उकिटबधीय िवचलन

4 Local wind थानीय वाय

Correct Answer -

Convergence अिभसरण

The Horse Latitudes are regions located at about _____ north and south of the equator

हॉस अाश भम रखा क उर और दिण म लगभग _____ पर थत ह

1 30ndash60 degree Latitude 30-60 िडी अाश

2 0ndash5 degree Latitude 0-5 िडी अाश

3 30 degree Latitude 30 िडी अाश

4 60ndash90 degree Latitude 60-90 िडी अाश

Correct Answer -

30 degree Latitude 30 िडी अाश

Generally evaporation is high over which part of the Earth

आम तौर पर पी क िकस भाग पर वाीकरण अिधक होता ह

1 Equatorial maritime भमवत समीय ी ी

91)

92)

2 Equatorial continental भमवत महाीपीय

3 Polar maritime वीय समीय

4 Polar continental वीय महाीपीय

Correct Answer -

Equatorial maritime भमवत समीय

A very high temperature during summer in north western India leads to what type of climaticcondition in south

उर पिमी भारत म गम क दौरान बत अिधक तापमान होन क कारण दिण म िकस कार की जलवाय थितउ करता ह

1 Depression over arabian sea अरब सागर पर अवनमन

2 Failure monsoon मानसन िवफलता

3 Successful monsoon मानसन सफलता

4 Cyclones चवात

Correct Answer -

Successful monsoon मानसन सफलता

Lightning and thunder are the resultant effect when तिड़त और गजन परणामी भाव ह जब

1 Two massive clouds hit powerfully each other first lightning is produced and later sound is produced

दो बड़ बादल एक दसर स शशाली ढग स टकरात ह पहल आकाशीय िवदयत उ होता ह और बाद म िन उहोती ह

2 Two massive clouds come into contact with the powerful wind collision this results into first sound and thenlightning

दो बड़ बादल शशाली पवन सघ क सपक म आत ह इसका परणामप पहल िन और िफर आकाशीय िवदयतउ होता ह

3 None of the above उपरो म स कोई भी नही

4 A high density cloud contains positively and negatively charged electric ions and when this interacts light andsound are simultaneously produced

एक उ घन बादल म धनाक और ऋणाक आविशत िवदयत आयन होत ह और जब यह परर भाव डालत ह तोकाश और िन एक साथ उािदत होती ह

Correct Answer -

A high density cloud contains positively and negatively charged electric ions and when this interacts light andsound are simultaneously produced

औ ि ि ो औ ो

93)

94)

95)

एक उ घन बादल म धनाक और ऋणाक आविशत िवदयत आयन होत ह और जब यह परर भाव डालत ह तोकाश और िन एक साथ उािदत होती ह

Doon Valley is able to grow rice because दन घाटी चावल उगान म सम ह ोिक

1 Other crops cannot be grown वहा अ फसलो को उगाया नही जा सकता ह

2 People in the valley are rice eaters घाटी म लोग चावल खान वाल ह

3 There is a huge export demand of rice वहा चावल की भारी िनयात माग ह

4 It has warm summer and snow melt waters for irrigation

वहा गिमया गम होती ह िसचाई क िलए बफ का िपघला आ पानी होता ह

Correct Answer -

It has warm summer and snow melt waters for irrigation

वहा गिमया गम होती ह िसचाई क िलए बफ का िपघला आ पानी होता ह

CANCELLED

In the geological time scale the Mesozoic Era DOES NOT contains which of the following periods

भगभय समय पमान पर मजीवी यग म िन कालो म स कौन नही ह

1 Triassic ट ाइऐिसक

2 Jurassic जरिसक

3 Cretaceous चाकमय

4 Carboniferous काबनी

Correct Answer -

Carboniferous काबनी

96)

1 P-3 Q-4 R-2 S-1

2 P-3 Q-4 R-1 S-2

3 P-3 Q-4 R-1 S-2

4 P-4 Q-3 R-2 S-1

Correct Answer -

P-4 Q-3 R-2 S-1

1 P-3 Q-1 R-4 S-2

2 P-3 Q-4 R-1 S-2

3 P-3 Q-2 R-4 S-1

97)

98)

4 P-2 Q-1 R-4 S-3

Correct Answer -

P-3 Q-4 R-1 S-2

1 P-3 Q-1 R-4 S-2

2 P-2 Q-3 R-4 S-1

3 P-2 Q-1 R-3 S-4

4 P-4 Q-2 R-1 S-3

Correct Answer -

P-3 Q-1 R-4 S-2

99)

100)

1 P-3 Q-2 R-4 S-1

2 P-1 Q-2 R-3 S-4

3 P-2 Q-3 R-1 S-4

4 P-4 Q-3 R-2 S-1

Correct Answer -

P-2 Q-3 R-1 S-4

ldquoHuman geography is the study of changing relationship between the unresting man and the unstableearthrdquo was defined by

lsquolsquoमानव भगोल ाकल आदमी और अथर पी क बीच सबध परवतन का अयन हrdquo ______ ारा परभािषत िकया गयाथा

1 J Brunches ज चस

2 EC Semple ईसी सल

3 HJ Mackinder एच ज मिकदर

4 PV Blache पीवी च

Correct Answer -

EC Semple ईसी सल

Sedimentary rocks are finally and ultimately derived from the____________

अवसादी चान अततः ________ स ा की जाती ह

1 action of earth movements पी की गितिविधयो

2 marine deposit समी िनप

3 weathering of metamorphic rocks पातरत चानो क अपय

4 weathering of igneous rocks आय चानो क अपय

Correct Answer -

weathering of igneous rocks आय चानो क अपय

Page 4: High School Teacher Eligibility Test- BOARD PROFESSIONAL ...peb.mp.gov.in/results/RESULT_18/HST_RES18/Final_anwser_key/HST… · M a ndl a / मंड ल ... Under the Madhya Pradesh

1)

2)

3)

Choose the option that best expresses the meaning of the highlighted idiomphrase

Anil How did the election go

Sunil Very exciting till the last minute Our candidate proved to be a dark horse

1 The candidate is not popular but won

2 His candidate lost the election though he was expected to win

3 The candidate backed out at the last minute

4 The candidate is popular and won

Correct Answer -

The candidate is not popular but won

A sentence with an underlined word is given below Choose the correct option which is closest in meaning tothe underlined word

The fruit of ones toil helps others to continue with the mission

1 achievement

2 hard work

3 attempts

4 sufferings

Correct Answer -

hard work

Out of the following options choose the correct form of verb that is in agreement with the subject for thegiven sentence

lsquoA Tale of Two Citiesrsquo ______ a wonderful book to read

1 are

2 were

3 is

4 have been

Correct Answer -

is

4)

5)

6)

Out of the following options choose the most appropriate tense that best fills in to complete the givensentence

This year our cricket team _________ all the matches played on the Indian soil

1 will win

2 was winning

3 wins

4 has been winning

Correct Answer -

has been winning

Choose the appropriate adverb clause from the options to complete the given sentence

He was very pleased _____

1 which you passed

2 which you had passed

3 that you passed

4 that you who passed

Correct Answer -

that you passed

Out of the following four options choose the incorrect sentence

1 Rohit is a charmer and you can find him selling newspapers on M G Road from 800 am every morning

2 Diabetes a leading cause of heart problems does not allow people to make it to the hospital in time after a heartattack

3 Symbiosis Institute is Indias premier approved business school

4 Nehrus vision continued to light up the entire nation even today

Correct Answer -

Nehrus vision continued to light up the entire nation even today

7)

8)

1)

Out of the following options choose the most appropriate usage to fill in the blank

I live a few yards ______ the bus stop

1 off

2 from

3 outside

4 away

Correct Answer -

from

Arrange the sentences P Q R and S to form a logical sequence between sentences 1 and 6

1 Jurors do not have to be intelligentP In 1933 in a sheep-stealing case in Wales two of the jurors were totally unable to understand Englishbut no one found that out until after the trialQ There is no test no examination and no enquiry about their abilitiesR The convicted found this out and complained about it to the Court of Criminal AppealS No one even takes the trouble to find out whether they can understand the English language6 Still the Court of Appeal felt that it must refuse to interfere

1 QPRS

2 SQRP

3 PRQS

4 QSPR

Correct Answer -

QSPR

Topic- GENERAL KNOWLEDGE

Which country will host the 2022 Football World Cup 2022 फटबॉल िव कप की मजबानी करन वाला दशकौन होगा

1 America अमरका

2 Canada कनाडा

3 Qatar कतर

4 Mexico मको

2)

3)

4)

5)

Correct Answer -

Qatar कतर

According to sex ratio trends of Census 2011 which district of Madhya Pradesh has been ranked numberOne

जनगणना 2011 म िलग अनपात क झानो क अनसार म दश क िकस िजल को नबर एक थान िदया गया ह

1 Seoni िसओनी

2 Ratlam रतलाम

3 Balaghat बालाघाट

4 Mandla मडला

Correct Answer -

Balaghat बालाघाट

The folk dance form called lsquoMatkirsquo is prevalent in which of the following regions of Madhya Pradesh

मटकी नामक लोक न प म यदश क िनिलखत म स िकस म चिलत ह

1 Bundelkhand बदलखड

2 Malwa मालवा

3 Vindhyachal िवाचल

4 Baghelkhand बघलख

Correct Answer -

Malwa मालवा

Who appoints the Chairman and the Members of the Union Public Service Commission

सघ लोक सवा आयोग क अ और सदो की िनय कौन करता ह

1 Chief Justice of India भारत क म य यायधीश

2 Prime Minister धान मी

3 President रा ट पित

4 Chief Election Commissioner म य चनाव आय त

Correct Answer -

President रा ट पित

6)

7)

8)

Under the Madhya Pradesh Ladli Laxmi Yojana the beneficiary is paid the total amount on attaining theage of ___ मदश लाडली ली योजना क तहत लाभाथ की आय ___ वष परी होन पर कल रािश का भगतान िकयाजाता ह

1 18

2 21

3 23

4 16

Correct Answer -

21

India has won the gold medal in hockey ______times at the Olympics

ओलिपक खलो म भारत न हॉकी म ण पदक _____ बार जीता ह

1 9

2 7

3 8

4 6

Correct Answer -

8

Where is the Madhya Pradesh Tribal Museum located at

म दश जनजातीय सहालय कहा थत ह

1 Indore इदौर

2 Jhabua झबआ

3 Khargone खरगोन

4 Bhopal भोपाल

Correct Answer -

Bhopal भोपाल

The fictional boy character of the lsquoThe Jungle Bookrsquo by author Rudyard Kipling was

लखक डयाड िकपिलग ारा lsquoद जगल बकrsquo का कािनक बालक चर था

1 Kaa का

2 Mowgli मोगली

9)

10)

1)

3 Shere Khan शर खान

4 Bagheera बघीरा

Correct Answer -

Mowgli मोगली

The recipient of which one of the following awards given by Madhya Pradesh Government gets the highest cashprize

िनिलखत परारो म स िकसक ाकता को म दश सरकार ारा उतम नकद परार िदया जाता ह

1 Mahatma Gandhi award महाा गाधी परार

2 Shikar Award िशखर परार

3 Sharad Joshi Award शरद जोशी परार

4 Kishore Kumar Award िकशोर कमार परार

Correct Answer -

Mahatma Gandhi award महाा गाधी परार

Who among the following was the Trinidadian writer of Indian descent

िनिलखत म स भारतीय मल क ििनदािदयन लखक कौन थ

1 RK Narayan आर क नारायण

2 Anita Desai अनीता दसाई

3 Rudyard Kipling डयाड िकपिलग

4 VS Naipaul वीएस नायपॉल

Correct Answer -

VS Naipaul वीएस नायपॉल

Topic- GENERAL REASONING

Find the missing number ल त स या ात कर

4 196 16 169 ____ 144 64

1 81

2 36

3 32

4 21

2)

3)

4)

Correct Answer -

36

Find the average औसत ात कर

25 35 45 55 65 amp 75

1 48

2 50

3 47

4 49

Correct Answer -

50

The below series uses a sequence of alphabets and numbers Identify the incorrect combination

नीच दी गई खला अरो एव साओ का अनम योग करती ह गलत सयोजन पहचान

(i) FT85DF77ER

(ii) FT85DF77ER

(iii) FT85DE77ER

(iv) FT85DF77ER

1 ii

2 iv

3 i

4 iii

Correct Answer -

iii

A and B together can complete a piece of work in 10 days and B alone can complete the same work in 20days In how many days can A alone complete the work

A और B एक साथ काम क एक भाग को 10 िदनो म समा त करत ह और B अकल उसी काम को 20 िदनो म समा त करता हिकतन िदनो म A अकल उस काम को समा त कर सकता ह

1 20 days 20 िदन

2 30 days 30 िदन

3 10 days 10 िदन

4 25 days 25 िदन

5)

6)

Correct Answer -

20 days 20 िदन

Which one of the following four addresses is NOT EXACTLY same as the one given below

िनिलखत िदए गए चार पतो म स नीच िदए गए पत क ठीक समान कौन सा एक नही ह

Ground Floor 59 Goulburn St

Sydney NSW 2000 Australia

+61 (02) 8987 3700

(i) Ground Floor 59 Goulburn St

Sydney NSW 2000 Australia

+61 (02) 8987 3700

(ii) Ground Floor 59 Goulburn St

Sydney NSW 2000 Australia

+61 (02) 8987 3700

(iii) Ground Floor 59 Govlburn St

Sqdney NSW 2000 Australia

+61 (02) 8987 3700

(iv) Ground Floor 59 Goulburn St

Sydney NSW 2000 Australia

+61 (02) 8987 3700

1 ii

2 iv

3 i

4 iii

Correct Answer -

iii

In a group of 75 people 32 of them like cold drink 56 of them like general water and each person likesat least one of the two drinks How many people like both

75 लोगो क समह म उनम स 32 को को िड क पसद ह उनम स 56 को सामा जल पसद ह और क को दो म स कम स कम एक पय पसद ह िकतन लोगो को दोनो पसद ह

1 11

2 13

7)

1)

2)

3 12

4 14

Correct Answer -

13

If A = 1 FAT = 27 then FAITH =

यिद A = 1 FAT = 27 तो FAITH =

1 41

2 40

3 42

4 44

Correct Answer -

44

Topic- PEDAGOGY

CANCELLED

Basic education became a major initiative because of the efforts of

ाथिमक िशा िन न क यास स एक मह वपण कायम बन गई

1 Dr Radhakrishnan डॉ राधाक णन

2 Dr Rajendra Prasad डॉ राज साद

3 Mahatma Gandhi महा मा गाधी

4 Rabindranath Tagore रबी नाथ टगोर

Correct Answer -

Mahatma Gandhi महा मा गाधी

The ldquoclause of phraserdquo is an unit of perception of लॉज ऑफ ज िन न क धारणा क एक इकाई ह

1 Image छिव

2 Concept अवधारणा

3 Language भाषा

4 Thought िवचार

Correct Answer -

3)

4)

5)

6)

Language भाषा

Which method is most suitable to study communication process among students

छाो क बीच सचार िया का अयन करन क िलए कौन सी िविध सबस उपय ह

1 Case Study मामल का अयन (कस टडी)

2 Systematic Observation वथत अवलोकन

3 Experimental Method योगाक िविध

4 Introspection आ-िनरीण (इट ोस शन)

Correct Answer -

Systematic Observation वथत अवलोकन

Genes in a human being are located in मानवो म जीन इनम थत होता ह

1 cytoplasm कोिशका (साइटो ला म)

2 ribosomes राइबोसोम

3 cell membranes कोिशका िझी

4 chromosomes गणस (ोमोसोम)

Correct Answer -

chromosomes गणस (ोमोसोम)

With smaller classes teachers are much more able to ____________

छोटी काओ क साथ िशक ____________ म अिधक सम होत ह

1 Go slow while teaching िशण क दौरान धीमी गित स जान

2 Narrate more personal experiences अिधक गत अनभवो को बतान

3 Adapt instruction to individual differences among students छाो क बीच गत मतभदो क िलए अनकल िनदश दन म

4 Make use of the extra space for extra-curricular activities पातर गितिविधयो क िलए अितर जगह का उपयोग करन

Correct Answer -

Adapt instruction to individual differences among students छाो क बीच गत मतभदो क िलए अनकल िनदश दन म

While engaging in a task the child gets bored This is a sign of

एक काम म होन पर बा ऊब जाता ह यह िन का सकत ह

1 the task requiring a professional approach काय को ावसाियक िकोण की आवकता ह

7)

8)

9)

2 the task becoming mechanically repetitive काय यािक प स दोहराव वाला ह

3 the child not being intelligent बा बमान नही ह

4 the child being incapable of learning बा सीखन म असमथ ह

Correct Answer -

the task becoming mechanically repetitive काय यािक प स दोहराव वाला ह

Educational Psychologists are more concerned with the learning in __________

शिणक मनोवािनक __________ म अिधगम क साथ अिधक िचितत होत ह

1 Formal environment औपचारक वातावरण

2 Informal environment अनौपचारक वातावरण

3 Physical environment भौितक वातावरण

4 Social environment सामािजक वातावरण

Correct Answer -

Formal environment औपचारक वातावरण

Special needs education is the type of education िवशष ज़रतो वाली िशा वह िशा होती ह जो

1 Given to person with disabilities अम य को दी जाती ह

2 Given to people from remote areas दर थ ो क लोगो को दी जाती ह

3 Provided to intelligent people बमान लोगो को दी जाती ह

4 Established by colonial masters औपिनविशक मखयाओ ारा थािपत की गई ह

Correct Answer -

Given to person with disabilities अम य को दी जाती ह

The Stanford-Binet scale of intelligence was first published in the year

ब क नफोड-िबनट कल को इस वष म पहली बार कािशत िकया गया था

1 1916

2 1903

3 1908

4 1900

Correct Answer -

1916

10)

11)

12)

13)

The term lsquofictional finalismrsquo was propounded by

पद lsquoकत योजनवादrsquo (िफ शनल फाइनिल म) इनक ारा ितपािदत िकया गया था

1 Skinner नर

2 Freud ायड

3 Adler एडलर

4 Pavlov पावलोव

Correct Answer -

Adler एडलर

Rational Emotive Behavior Therapy was propounded by

तक सगत भावनाक वहार थरपी िन क ारा ितपािदत की गई थी

1 Carl Jung काल यग

2 Carl Rogers काल रोजस

3 Aaron Beck आरोन बक

4 Albert Ellis अट एिलस

Correct Answer -

Albert Ellis अट एिलस

Which of the following indicates the quality of education in a school

िनिलखत म स या एक कल म िशा की गणव ता का सकतक ह

1 Text-books and Teaching-learning material पा-प तक तथा िशण व सीखन स सबिधत सामी

2 Infrastructural facilities at the school कल म आधारभत ढाच स सबिधत सिवधाए

3 Student achievement level िवािथयो का उपल तर

4 Classroom systems का की यव था

Correct Answer -

Student achievement level िवािथयो का उपल तर

Which of the following teachers can be identified with authoritarian teaching style

िनिलखत म स कौन सा िशक अिधकारवादी िशण शली क साथ पहचाना जा सकता ह

1 Laissez-faire teacher अब ध िशक

14)

15)

2 Democratic teacher लोकतीय िशक

3 Indifferent teacher िन प िशक

4 Direct instruction teacher िनदश िशक

Correct Answer -

Direct instruction teacher िनदश िशक

Who was the pioneer of classical conditioning

िचरितित ानकलन ( लािसकल कडीशिनग) क वतक कौन थ

1 Skinner नर

2 Pavlov पावलोव

3 Kohler कोहलर

4 Freud ायड

Correct Answer -

Pavlov पावलोव

Which of the following are true with reference to short term memory

1 Limited capacity

2 Brief storage of information

3 Unlimited capacity

4 Duration of storage less than twenty seconds

अ पकािलक मित क सदभ म िन न म स या स य ह

1 सीिमत मता

2 सचना का सि त भडारण

3 असीिमत मता

4 बीस सकड स कम भडारण की अविध

1 3 and 4 3 और 4

2 2 and 4 2 और 4

3 13 and 4 13 और 4

4 12 and 4 12 और 4

Correct Answer -

12 and 4 12 और 4

1)

2)

3)

4)

Topic- GEOGRAPHY

Which of the following ranges of population size is used to define Class-III city by Indian Census

भारतीय जनगणना ारा वग-III शहर को परभािषत करन क िलए िनिलखत म स िकस रज क जनसा आकारका उपयोग िकया जाता ह

1 20 000 to 49999 20 000 स 49999

2 30000 to 59999 30000 स 59999

3 24000 to 54999 24000 स 54999

4 50000 to 99999 50000 स 99999

Correct Answer -

20 000 to 49999 20 000 स 49999

Carbonaceous rocks which produce coal and oil belong to the category of rocks called_______

कोयल और तल का उादन करन वाली काबनय चान ______ नामक चानो की णी स सबिधत ह

1 metamorphic पातरत

2 sedimentary अवसादी

3 inorganic अजिवक

4 igneous आय

Correct Answer -

sedimentary अवसादी

The ruhr-complex is a major industrial centre in र-परसर िन का एक मख औोिगक क ह

1 North America उरी अमरका

2 Russia स

3 Germany जमनी

4 Europe यरोप

Correct Answer -

Germany जमनी

The term lsquoRegurrsquo refers to श lsquoरगरrsquo ______ स सबिधत ह

1 Deltaic alluvial soils डा जलोढ़ िमी

ि ी

5)

6)

2 Laterite soils लटराइट िमी

3 Red and yellow soils लाल और पीली िमी

4 Black cotton soils काली कपास िमी

Correct Answer -

Black cotton soils काली कपास िमी

Read the given statements and answer which of the following options isare correct

1 Lower the pressure greater the atmospheric disturbance

2 Air move from higher to low pressure

िदए गए कथन को पढ़ और उर द िक िन म स कौन सास िवक सही ह

1 िजतना दाब कम होगा वायमडलीय बाधाए उतनी अिधक होगी

2 वाय उ स िन दाब की ओर गित करती ह

1 Both Statements 1 and 2 are correct दोनो कथन 1 और 2 सही ह

2 Both Statements 1 and 2 are wrong दोनो कथन 1 और 2 गलत ह

3 Statement 1 is wrong and only Statement 2 is correct कथन 1 गलत ह और कवल कथन 2 सही ह

4 Statement 1 is correct and Statement 2 is wrong कथन 1 सही ह और कथन 2 गलत ह

Correct Answer -

Both Statements 1 and 2 are correct दोनो कथन 1 और 2 सही ह

CANCELLED

1 4 1 2 and 3 4 1 2 और 3

2 4 1 3 and 2 4 1 3 और 2

3 1 4 2 and 3 1 4 2 और 3

4 1 4 3 and 2 1 4 3 और 2

Correct Answer -

7)

8)

9)

1 4 3 and 2 1 4 3 और 2

CANCELLED

Karl Pearsonrsquos correlation co-efficient is काल िपयसन का सहसबध गणाक ह

1 Arithmec mean समार मा

2 Geometric mean गणोर मा

3 None of these इनम स कोई नही

4 Harmonic mean हराक मा

Correct Answer -

Geometric mean गणोर मा

CANCELLED

ldquoEach day is more or less the same the morning is clear and bright with a sea breeze as the Sun climbshigh in the sky heat mounts up dark clouds form then rain comes with thunder lighting But rain is soonoverrdquo Which of the following regions is described in the above passage

ldquoक िदन समान स अिधक या कम होता ह सम की हवा क साथ और उल सबह होती ह जस सयआकाश म ऊचा चढ़ता ह गम बढ़ जाती ह काल बादल बनत ह िफर िबजली क साथ बारश आती ह लिकनबारश जी ख हो जाती हlsquorsquo उपरो पा म िनिलखत म स िकन ो का वणन िकया गया ह

1 Equatorial भमरखीय

2 Equatorial भमरखीय

3 Savannah सवाना

4 Mediterranean आातरक (भमसागरीय)

5 Mediterranean आातरक (भमसागरीय)

6 Monsoon मानसन

7 Monsoon मानसन

Correct Answer -

Equatorial भमरखीय

Equatorial भमरखीय

CANCELLED

In which epoch of the geological history of the Earth dinosaurs reached their largest size

पी क भगभय इितहास क िकस यग म डायनासोर अपन सबस बड़ आकार तक पचि

10)

11)

1 Triassic ट ायिसक

2 Jurassic जरािसक

3 Cretaceous ीटशस

4 Permian पिमयन

Correct Answer -

Jurassic जरािसक

CANCELLED

A spring tide would occur in which of the following conditions

िनिलखत म स िकन थितयो म ार-भाटा आता ह

1 When the Sun Moon and Earth are in a straight line

जब सय चमा और पी एक सीधी रखा म होत ह

2 When the Moon and Earth are in right angle to each other

जब चमा और पी एक-दसर क दािहन कोण म होत ह

3 When the Earth and Moon are in right angle to the Sun

जब पी और चमा सय क दािहन कोण म होत ह

4 When the Sun and Moon are in right angle to each other

जब सय और चमा एक-दसर क दािहन कोण म होत ह

Correct Answer -

When the Sun Moon and Earth are in a straight line

जब सय चमा और पी एक सीधी रखा म होत ह

CANCELLED

An observe on the Earthrsquos surface always sees the same face of the moon because

एक पयवक को पी की सतह स हमशा चाद का एक ही फलक िदखाई दता ह ोिक

1 Its path of revolution around the earth is the same as that of the earth around the Sun

इसका पी क चारो ओर परमण का माग सय क चारो ओर पी क समान ही ह

2 Its period of revolution around the Earth is the same as its period of rotation around its own axis

इसकी पी क चारो ओर परमण की अविध उसकी अपनी धरी क चारो ओर घणन की अविध क समान ह

3 Its period of rotation is the same as that of the Earth इसकी घणन की अविध पी क समान ह

ी ि ी ी

12)

13)

14)

4 Its direct of rotation is the same as that of Earth घणन की िदशा पी क समान ही ह

Correct Answer -

Its period of revolution around the Earth is the same as its period of rotation around its own axis

इसकी पी क चारो ओर परमण की अविध उसकी अपनी धरी क चारो ओर घणन की अविध क समान ह

CANCELLED

The pebbles that are faceted by the sand-blasting and shaped polished by the wind abrasions are known as

पवन अपघषन ारा पॉिलश रत-िवोिटत और साच म ढला ककड़ ___________ क प म जाना जाता ह

1 Dreikanter िकोणक

2 Pediments िकोिनका

3 Inselberg इलबग

4 Dunes टीबा

Correct Answer -

Dreikanter िकोणक

CANCELLED

Astronomical unit is the average distance between खगोलीय इकाई ______ क बीच की औसत दरी ह

1 Earth and Mars पी और मगल

2 Earth and mercury पी और बध

3 Earth and moon पी और चमा

4 Earth and Sun पी और सय

Correct Answer -

Earth and Sun पी और सय

During cold weather season in the northern plains there will be an inflow of cyclonic disturbancesfrom the _________ directions

शीत मौसम क दौरान उरी मदानी इलाको म _________ िदशाओ स चवात सबधी गड़बड़ी का अतवाह होगा

1 East and Northwest पव और उरपिम

2 East and Northeast पव और पवर

3 West and East पिम और पव

ि औ ि

15)

16)

17)

4 West and Northwest पिम और उरपिम

Correct Answer -

West and Northwest पिम और उरपिम

During an earth quake the velocity of the body waves will________ along with the increase in densityof the material it is passing through

भकप क दौरान लहरो क ऊपरी भाग का वग घन म व क साथ-साथ ________ जो इसस गजरन वाली वको आग बढाएगी

1 not change नही बदलगा

2 increase initially and then decrease शआत म बढ़गा और िफर घटगा

3 increase बढ़गा

4 decrease घटगा

Correct Answer -

increase बढ़गा

The Clouded Leopard National park is situated in which of the following states

िनिलखत म िकस रा म धिमल तदआ रा ीय उान (ाउडड लपड नशनल पाक ) थत ह

1 Tripura िपरा

2 Uttar Pradesh उर दश

3 Assam असम

4 Mizoram िमजोरम

Correct Answer -

Tripura िपरा

Usually the land surfaces are heated more quickly than the water surfaces because _____________

आम तौर पर जल सतहो की तलना म भिम सतह अिधक तजी स गम होती ह ोिक _____________ ह

1 the specific heat of water is higher than land पानी की िविश ऊा भिम स अिधक

2 the specific heat of water is lesser than land पानी की िविश ऊा भिम स कम होती

3 the latent heat of water is higher than the land पानी की अतिनिहत ऊा भिम स अिधक

4 the land reflects more heat radiation than water भिम पानी की तलना म अिधक ऊा क िविकरण को पराविततकरती

18)

19)

20)

21)

Correct Answer -

the specific heat of water is higher than land पानी की िविश ऊा भिम स अिधक

The longest shore-line is along the state of सबस लबी समतटीय रखा िन रा क साथ ह

1 Maharashtra महारा

2 Orissa उड़ीसा

3 Kerala करल

4 Gujarat गजरात

Correct Answer -

Gujarat गजरात

The position when the Earth is farthest from the Sun is known as

जब पी सय स सबस दर होती ह तो उस थित को िन नाम स जाना जाता ह

1 Perihelion उपसौर

2 Vernal Equinox बसत िवषव

3 Aphelion अपसौर

4 Autumnal Equinox शराल िवषव

Correct Answer -

Aphelion अपसौर

The seasonal reversal of winds is the typical characteristic of

हवाओ का मौसमी परवतन ______ की सामा िवशषता ह

1 Mediterranean climates only कवल भमसागरीय जलवाय

2 All of the above climates उपय सभी मौसम

3 Monsoon climate only कवल मानसन जलवाय

4 Equatorial climate only कवल भमरखीय जलवाय

Correct Answer -

Monsoon climate only कवल मानसन जलवाय

In _________ rocks the minerals will occurs in beds or layers

ो ि ो ो

22)

23)

24)

______ चानो म खिनज तल या परतो म होत ह

1 metamorphic कायातरत

2 igneous and metamorphic आय और कायातरत

3 igneous आय

4 sedimentary अवसादी

Correct Answer -

sedimentary अवसादी

Black soil is ideal for the cultivation of cotton as कपास की खती क िलए काली िमी आदश ह ोिक

1 Its colour is black यह काली होती ह

2 It is found on plateau regions यह पठार ो म पायी जाती ह

3 It is made up of lava यह लावा स बनी होती ह

4 It can retain moisture यह नमी को बरकरार रख सकती ह

Correct Answer -

It can retain moisture यह नमी को बरकरार रख सकती ह

The National Survey and Mapping Organization of the country works under the Department of___________

दश का रा ीय सवण और मानिचण सगठन ___________ िवभाग क अतगत काय करता ह

1 Space अतर

2 Science and Technology िवान और तकनीक

3 Culture सित

4 Tourism पयटन

Correct Answer -

Science and Technology िवान और तकनीक

Palk strait separates India from पाक जलडमम भारत स _____ को अलग करता ह

1 Pakistan पािकान

2 Andaman Island अडमान ीप

3 China चीन

25)

26)

27)

4 Sri Lanka ीलका

Correct Answer -

Sri Lanka ीलका

Which among the following state is the major producer of Bauxite in India

िनिलखत म स कौन सा रा भारत म बॉाइट का मख उादक ह

1 Madhya Pradesh मदश

2 Rajasthan राजथान

3 Goa गोवा

4 Orissa उड़ीसा

Correct Answer -

Orissa उड़ीसा

Which of the following states DOES NOT share border with Chhattisgarh

िनिलखत म स कौन सा रा छीसगढ़ क साथ सीमा साझा नही करता ह

1 Telangana तलगाना

2 Uttar Pradesh उर दश

3 Bihar िबहार

4 Andhra Pradesh आ दश

Correct Answer -

Bihar िबहार

Which of the following statements is INCORRECT with respect to parallels of latitudes

अाश क समानातरो क सबध म िन निलखत म स कौन सा कथन गलत ह

1 A line joining places of equal latitude is known as parallel of largest

समान अाश क थानो को जोड़न वाली रखा को िवशालतम क समानातर क प म जाना जाता ह

2 They stat from equator and run parallels to it

व भम रखा स ारभ होत ह और इसक समानातर चलत ह

3 All parallels are equal in length सभी समातर लबाई म समान ह

4 All parallels are drawn as circles on the globe ोब पर सभी समानातर वो क प म खीच जात ह

28)

29)

30)

31)

Correct Answer -

All parallels are equal in length सभी समातर लबाई म समान ह

Which of the following Indian states is also known as a lsquoLand of Red river and Blue Hillsrsquo

िनिलखत म स िकस भारतीय रा को लाल नदी और नीली पहािड़यो की भिम क नाम स जाना जाता ह

1 Uttarkhand उराखड

2 Assam असम

3 Meghalaya मघालय

4 Arunachal Pradesh अणाचल दश

Correct Answer -

Assam असम

In spatial analysis of settlement Rn = 215 indicates which type of settlement arrangement

िनपटान क थािनक िवषण म Rn = 215 यह इिगत करता ह िक िकस कार की िनपटान वथा ह

1 Uniform यिनफॉम

2 Semi-Clustered समी- ल टर

3 Clustered ल टर

4 Random रडम

Correct Answer -

Uniform यिनफॉम

Who are known as the lsquoYellow Peoplersquo lsquoयलो पीपलrsquo क प म कौन जाना जाता ह

1 Mongoloids मोगोलोइडस

2 Nigroids नीोइडस

3 Australoids ऑ लॉइडस

4 Caucasoids कॉकसोइडस

Correct Answer -

Mongoloids मोगोलोइडस

ि ि ो ौ ि

32)

33)

34)

Who publishes the topographical map of India भारत क थलाकितक मानिच को कौन कािशत करता ह

1 Geographical Survey of India भारत का भौगोिलक सवण

2 Government of India भारत सरकार

3 Geological Survey of India भारत क भगभय सवण

4 Survey of India भारत का सवण

Correct Answer -

Survey of India भारत का सवण

Who among the following claimed geography to be the lsquoEcology of Manrsquo

िनिलखत म स िकसन भगोल को मन का पारथितकी कहा ह

1 Alfred Hener अड हटनर

2 Vidal-de la Blache वाइडल-िड लॉ ॉश

3 Oo Schluter ओटो टर

4 Harlan Barrow हरलन बारो

Correct Answer -

Harlan Barrow हरलन बारो

Who among the following is regarded as the founder of humanistic approach in geography

िनिलखत म स िकस भगोल म मानवतावादी िकोण का सथापक माना जाता ह

1 William Bunge िविलयम बग

2 Yi-Fu-Tuan यी-फ- यान

3 Brain JL Berry न जएल बरी

4 Richard Peet रचड पीट

Correct Answer -

Yi-Fu-Tuan यी-फ- यान

Who prepared Lorenz curve लोरज व िकसन तयार िकया

1 Geddes गडस

2 None of these इनम स कोई नही

3 Griffith Taylor ििफथ टलर

35)

36)

37)

4 Max U Lorenz म य लोरज

Correct Answer -

Max U Lorenz म य लोरज

Gulf Streams are the currents of which of the following oceans

खाड़ी की धाराए िनिलखत महासागरो म स िकसकी धाराए ह

1 North Atlantic Ocean उरी अटलािटक महासागर

2 North Pacific Ocean उरी शात महासागर

3 Arabian Sea अरब सागर

4 South Pacific Ocean दिण शात महासागर

Correct Answer -

North Atlantic Ocean उरी अटलािटक महासागर

Disintegration wearing away and removal of rock material is generally referred as

िशला पदाथ (रॉक सामी) का टटना िमटना और हटना आमतौर पर ________ क प म सदिभत िकया जाता ह

1 Shattering िवसकारक

2 Denudation अनाादन

3 Fault श

4 Decomposition िवयोजन

Correct Answer -

Denudation अनाादन

Variations in the length of day time and night from season to season are due to

मौसम स मौसम परवतन पर िदन क समय और रात क समय की अविध म िभताए िन कारण स होती ह

1 The Earthrsquos revolution round the Sun in an elliptical manner पी का दीघवाकार तरीक स सय क चारो घणन

2 The Earthrsquos rotation on its axis पी का इसकी धरी पर घणन

3 Revolution of the Earth on a tilted axis नत अ पर पी का घणन

4 Latitudinal position of the place थान की अाश थित

Correct Answer -

Revolution of the Earth on a tilted axis नत अ पर पी का घणन

38)

39)

40)

Point out the correct sequence of mountain ranges from north to south

उर स दिण तक पवत खलाओ क सही अनम को इिगत कर

1 Great Himalaya Middle Himalaya Outer Himalaya Trans Himalaya

महान िहमालय म िहमालय बा िहमालय परा िहमालय

2 Middle Himalaya Great Himalaya Trans Himalaya Outer Himalaya

म िहमालय महान िहमालय परा िहमालय बा िहमालय

3 Outer Himalaya Middle Himalaya Great Himalaya Trans Himalaya

बा िहमालय म िहमालय महान िहमालय परा िहमालय

4 Trans Himalaya Great Himalaya Middle Himalaya Outer Himalaya

परा िहमालय महान िहमालय म िहमालय बा िहमालय

Correct Answer -

Trans Himalaya Great Himalaya Middle Himalaya Outer Himalaya

परा िहमालय महान िहमालय म िहमालय बा िहमालय

Sunrsquos halo is produced by the refraction of light in सय का भामडल ______ म काश क अपवतन ाराउ होता ह

1 Ice crystals in Cirrro-Cumulus clouds पाभ-कपास मघो क बफ िल

2 Ice crystal in Cirrus clouds पाभमघो क बफ िल

3 Dust particles in Stratus clouds री मघो क धल कण

4 Water vapour in Stratus clouds री मघो क जल वा

Correct Answer -

Ice crystal in Cirrus clouds पाभमघो क बफ िल

Read the given statements and answer which of the following options isare correct

(1) The minerals present in the rocks exposed to atmosphere are not subjected to alteration

(2) Oxidation is one of the processes of chemical weathering

िदए गए कथनो को पढ़ और उर द िक िन म स कौन सास िवक सही ह

(1) वायमल स अनावत शल म उपथत खिनज परवतन क अधीन नही होता ह

(2) ऑीकरण रासायिनक अपय की ियाओ म स एक ह

ो ो

41)

42)

1 Both statements are wrong दोनो कथन गलत ह

2 Both statements are correct दोनो कथन सही ह

3 First statement is wrong and second statement is correct पहला कथन गलत ह और दसरा कथन सही ह

4 First statement is correct and second statement is wrong पहला कथन सही ह और दसरा कथन गलत ह

Correct Answer -

First statement is wrong and second statement is correct पहला कथन गलत ह और दसरा कथन सही ह

Read the given statements and answer which of the following options isare correct

1 Sunrsquos short waves enter the earth partially heating the atmosphere

2 Heated earth surface from the sun produces broader waves which interacts and heats the atmosphere

िदए गए कथन को पढ़ और उर द िक िन म स कौन सास िवक सही ह

1 सय की छोटी तरग पी म आिशक प स वश करती ह और वायमडल को ऊत करती ह

2 सय स ऊत पी की सतह िवारत तरग उ करती ह जो परर भाव डालती ह और वायमडल कोऊत करती ह

1 Both Statements 1 and 2 are correct दोनो कथन 1 और 2 सही ह

2 Both Statements 1 and 2 are wrong दोनो कथन 1 और 2 गलत ह

3 Statement 1 is wrong and only Statement 2 is correct कथन 1 गलत ह और कवल कथन 2 सही ह

4 Only statement 1 is correct कवल कथन 1 सही ह

Correct Answer -

Both Statements 1 and 2 are correct दोनो कथन 1 और 2 सही ह

Read the given statements and answer which of the following options isare correct

(1)The rocks that get changed due to heat and pressure are termed as metamorphic rocks

(2)Slate is one such type of metamorphic rock

िदए गए कथनो को पढ़ और उर द िक िन म स कौन सास िवक सही ह

(1) शल जो ऊा और दाब क कारण परवितत हो जात ह उ कायातरक शलो क प म जाना जाता ह

(2) ट एक तरह का कायातरक शल ह

1 Both statements are wrong दोनो कथन गलत ह

2 Both statements are correct दोनो कथन सही ह

3 First statement is wrong and second statement is correct पहला कथन गलत ह और दसरा कथन सही ह

ी औ

43)

44)

4 First statement is correct and second statement is wrong पहला कथन सही ह और दसरा कथन गलत ह

Correct Answer -

Both statements are correct दोनो कथन सही ह

Read the given statements and answer which of the following options isare correct

1 Higher temperature anomaly is observed in the northern hemisphere

2 Differential heating is absent in Northern Hemisphere

िदए गए कथनो को पढ़ और उर द िक िन म स कौन सास िवक सही ह

1 उरी गोलाध म उ तापमान िवसगित पायी जाती ह

2 उरी गोलाध म अतर ऊन अनपथत होती ह

1 Both Statements 1 and 2 are correct दोनो कथन 1 और 2 सही ह

2 Both Statements 1 and 2 are wrong दोनो कथन 1 और 2 गलत ह

3 Statement 1 is wrong and Statement 2 is correct कथन 1 गलत ह और कथन 2 सही ह

4 Statement 1 is correct and Statement 2 is wrong कथन 1 सही ह और कथन 2 गलत ह

Correct Answer -

Statement 1 is correct and Statement 2 is wrong कथन 1 सही ह और कथन 2 गलत ह

Read the given statements and answer which of the following options isare correct

(1) Plutonic rocks are intrusive type of igneous rocks

(2) It cools very slowly because the surrounding rock serves as insulation around the intrusion of magma

िदए गए कथनो को पढ़ और उर द िक िन म स कौन सास िवक सही ह

(1) िवतलीय शल अतवधी कार क आश शल ह

(2) यह बत धीर-धीर ठडा होता ह ोिक आस-पास क शल मा क अतवधन क चारो ओर रोधन क प म कायकरत ह

1 Both statements are wrong दोनो कथन गलत ह

2 Both statements are correct दोनो कथन सही ह

3 First statement is wrong and second statement is correct पहला कथन गलत ह और दसरा कथन सही ह

4 First statement is correct and second statement is wrong पहला कथन सही ह और दसरा कथन गलत ह

Correct Answer -

Both statements are correct दोनो कथन सही ह

45)

46)

47)

48)

The dust and ash material hurled from the volcanoes are termed as

ालामखी स िनकलन वाली धल और राख सामी को _______ क प म कहा जाता ह

1 Pyroclasc पाइरोाक

2 Hyperclastic हाइपराक

3 Hepiroclastic हिपरोाक

4 Cirroclastic िसरोाक

Correct Answer -

Pyroclasc पाइरोाक

The vertical difference in elevation between a low tide and high tide is referred as

कम ार और उ ार क बीच ऊचाई म लबवत अतर _____ स सदिभत होता ह

1 Tidal slope ारीय ढलान

2 Tidal elevation ारीय उयन

3 Tidal range ारीय परास

4 Tidal height ारीय ऊचाई

Correct Answer -

Tidal range ारीय परास

The maximum biodiversity is found in which of the following regions िनिलखत ो म स अिधकतमजव िविवधता िकसम पायी जाती ह

1 Amazon Basin अमज़न बिसन

2 East Indies ई इडीज

3 Congo Basin कागो बिसन

4 West indies व इडीज

Correct Answer -

Amazon Basin अमज़न बिसन

The cultivation of rice crop produces_______ चावल की फसल की खती ______ का उादन करती ह

1 SO2

49)

50)

51)

2 CH4

3 CFCs

4 CO2

Correct Answer -

CH4

The pressure system with higher pressure at the centre is called__________

क म उ दबाव वाली दबाव णाली को _______ कहा जाता ह

1 front अ

2 depression अवनमन

3 cyclone चवात

4 anti-cyclone ितचवात

Correct Answer -

anti-cyclone ितचवात

The Himalayan region is poor in mineral resources because िहमालयी खिनज ससाधनो म समनही ह ोिक

1 The displacement of rock strata has disturbed the arrangement of rocks and made it complex

शलीय परत क िवथापन न चानो की वथा को अवथत कर िदया ह और इस जिटल बना िदया ह

2 The climate conditions are not suitable for exploitation of minerals

जलवाय की थित खिनजो क दोहन क िलए उपय नही ह

3 The terrain makes explanation of minerals difficult and very costly due to transportation difficulties

भ-भाग परवहन की किठनाइयो क कारण खिनजो का दोहन मल और बत महगा बना दता ह

4 It is made up of crystalline rocks यह िलीय चानो स बना ह

Correct Answer -

The displacement of rock strata has disturbed the arrangement of rocks and made it complex

शलीय परत क िवथापन न चानो की वथा को अवथत कर िदया ह और इस जिटल बना िदया ह

The process through which the moisture is added to the atmosphere by vegetation is termed as

वह िया िजसक माम स वनित ारा वातावरण म नमी िमलायी जाती ह _______ क प म जानी जाती ह

52)

53)

54)

1 Condensation सघनन

2 Evapotranspiration वान-उजन

3 Radiation िविकरण

4 Precipitation वषण

Correct Answer -

Evapotranspiration वान-उजन

The process through which the terrestrial heat is transferred to air by direct contact is termed as

वह िया िजसम सपक ारा थलीय ऊा वाय म थानातरत हो जाती ह ______ क प म जानी जाती ह

1 Conduction चालन

2 Convection सवहन

3 Insolation आतपन

4 Radiation िविकरण

Correct Answer -

Conduction चालन

The largest area under mangroves is in which of the following statesunion territory

मोव क अतगत िनिलखत राोसघ शािसत दशो म स सबस बड़ा कौन सा ह

1 Andaman and Nicobar अमान और िनकोबार

2 Andhra Pradesh आ दश

3 West Bengal पिम बगाल

4 Gujarat गजरात

Correct Answer -

West Bengal पिम बगाल

The longitudinal transverse and surface waves in an earthquake originate from

भकप म दशातर अनथ और सतह तरग यहा उ होती ह

1 The focus on the surface of the Earth पी क सतह पर क -िबद म

2 The focus within the body of the Earth पी क भीतर क -िबद म

3 The epicenter within the body of the Earth पी क भीतर उपरक म

55)

56)

57)

4 The epicenter on the surface of the Earth पी क सतह पर उपरक म

Correct Answer -

The focus within the body of the Earth पी क भीतर क -िबद म

The down slope movement of material due to gravity is called______

गाकषण क कारण पदाथ की अनढाल गित को ______ कहा जाता ह

1 mass movement पदाथ सचलन

2 deposition िनप

3 erosion रण

4 volcanic movement ालामखीय सचलन

Correct Answer -

mass movement पदाथ सचलन

Shimla is cooler than Amritsar although both are on the same latitude This is because

िशमला म अमतसर स अिधक ठड ह हालािक दोनो समान अाश पर ह ऐसा ह ोिक

1 Shimla is at a greater height above sea level than Amritsar अमतसर की तलना म िशमला सम तल स अिधकऊचाई पर ह

2 Shimla is further north िशमला उर की ओर ह

3 Shimla is farther from the equator िशमला भम रखा स आग ह

4 Their longitudes differ उनकी दशातर रखाए िभ ह

Correct Answer -

Shimla is at a greater height above sea level than Amritsar अमतसर की तलना म िशमला सम तल स अिधकऊचाई पर ह

lsquoTempo of Urbanizationrsquo measures which of the following

lsquoशहरीकरण का टपोrsquo िनिलखत म स कौन सा उपाय ह

1 Speed of urbanizaon शहरीकरण की गित

2 None of the above इनम स कोई नही

3 Inequality of urbanizaon शहरीकरण की असमानता

4 Current level of urbanizaon शहरीकरण का वतमान र

Correct Answer -

58)

59)

60)

Speed of urbanizaon शहरीकरण की गित

Out of the following options choose the INCORRECT statement

िनिलखत िवको म स गलत कथन का चयन कर

1 The clear tracts in the equatorial region recover rapidly भम रखा म भभाग तजी स ठीक हो जात ह

2 The stable communities include a redwood forest a pine forest at high elevations

थर समदायो म एक रडवड वन उ ऊचाई पर एक दवदार वन शािमल ह

3 Any ecosystem moves towards maximum biomass and stability to survive

कोई भी पारथितकी त जीिवत रहन क िलए अिधकतम जवसहित और थरता की तरफ असर होता ह

4 Tropical rain forests near equator are stable ecosystems

भम रखा क पास उकिटबधीय वषा वन थर पारथितक त ह

Correct Answer -

The clear tracts in the equatorial region recover rapidly भम रखा म भभाग तजी स ठीक हो जात ह

Seasonal contrasts are maximum in मौसमी िवषमता अिधकतम ह

1 Mid latitudes म अाश म

2 Low attitudes िन अाश म

3 High latitudes उ अाश म

4 Subtropics उपोकिटबधीय म

Correct Answer -

Mid latitudes म अाश म

In India which type of forest among the following occupies the largest area

भारत म िनिलखत म स िकस कार क वन सबस बड़ा फल आािदत करत ह

1 Sub-tropical Dry Evergreen Forest उप उकिटबधीय श सदाबहार वन

2 Mountain Wet Temperate Forest पवतीय आ शीतो वन

3 Tropical Moist Deciduous Forest उकिटबधीय आ पणपाती वन

4 Tropical Wet Evergreen Forest उकिटबधीय आ सदाबहार वन

Correct Answer -

Tropical Moist Deciduous Forest उकिटबधीय आ पणपाती वन

61)

62)

63)

64)

What is the proportion of lsquoJuvenile Populationrsquo (0-14 years) in India as per 2011Census

2011 की जनगणना क अनसार भारत म जवनाइल पॉपलशन यानी िकशोर जनस या (0-14 वष) का अनपात ाह

1 3076 of total population कल जनस या का 3076

2 2764 of total population कल जनस या का 2764

3 2933 of total population कल जनस या का 2933

4 3354 of total population कल जनस या का 3354

Correct Answer -

3076 of total population कल जनस या का 3076

What is the Belfast famous for बलफा िकसक िलए मशर ह

1 Belt of cotton textile industry कपास व उोग क

2 Ship-building industry जहाज िनमाण उोग

3 Agricultural machinery किष उपकरण

4 Aero planes manufacturing वाययान िनमाण

Correct Answer -

Ship-building industry जहाज िनमाण उोग

What is the most important occupation in tropical monsoon lands

उकिटबधीय मॉनसन भिम म सबस महपण वसाय ा ह

1 Mining खनन

2 Cattle rearing मवशी पालन

3 Agriculture किष

4 Nomadic herding नोमािडक जड़ी-बिटया

Correct Answer -

Agriculture किष

What is the most important characteristics of the islands (Indian) located in the Arabian sea

अरब सागर म थत ीपो (भारतीय) की सबस महपण िवशषता ा ह

ी ो

65)

66)

67)

1 There are all of coral origins सभी कोरल मल क ह

2 There are all very small in size य सभी आकार म बत छोट ह

3 They have a very dry climate इनकी जलवाय बत श ह

4 They are extended parts of the mainland व महाीप क िवारत िह ह

Correct Answer -

There are all of coral origins सभी कोरल मल क ह

What do the basalt layers of the Deccan indicate डन की बसा परत ा इिगत करती ह

1 All of the above उपरो सभी

2 Huge volcanic eruptions in the distant past दरथ अतीत म िवशाल ालामखीय िवोट

3 The immense erosional activity of the rivers निदयो की िवशाल रण गितिविध

4 The influence of weathering मौसम का भाव

Correct Answer -

Huge volcanic eruptions in the distant past दरथ अतीत म िवशाल ालामखीय िवोट

In the structure of planet Earth below the mantle the core is mainly made up of_____

पी ह की सरचना म मटल क नीच कोर म प स______ स िनिमत होती ह

1 aluminium एमीिनयम

2 silicon िसिलकॉन

3 chromium ोिमयम

4 iron लोहा

Correct Answer -

iron लोहा

One of the major Mid Oceanic Ridge is found in मख म-महासागर चोिटयो म स एक ______ म पायाजाता ह

1 Mid Pacific Ocean म शात महासागर

2 Mid Atlantic Ocean म अटलािटक महासागर

3 Mid Indian Ocean म भारतीय महासागर

4 Mid Arctic Ocean म आक िटक महासागर

68)

69)

70)

71)

Correct Answer -

Mid Atlantic Ocean म अटलािटक महासागर

Magma that reaches the Earthrsquos surface and then solidifies is called________

मा जो पी की सतह तक पचती ह और िफर ठोस हो जाती ह ________कहलाती ह

1 quartz ाटज

2 lava लावा

3 granite नाइट

4 silicates िसिलकट

Correct Answer -

lava लावा

Isotherms are the lines of equal_______ समताप रखाए समान _______की रखाए होती ह

1 pressure दाब

2 temperature तापमान

3 rainfall वषा

4 height ऊचाई

Correct Answer -

temperature तापमान

Mark the correct sequence of passes in the Western Ghats from north to south

पिमी घाटो म उर स दिण तक दर क सही अनम को िचित कर

1 Thalghat Palghat Bhorghat थलगघाट पालघाट भोरघाट

2 Thalghat Bhorghat Palghat थलघाट भोरघाट पालघाट

3 Bhorghat Thalghat Palghat भोरघाट थलघाट पालघाट

4 Palghat Bhorghat Thalghat पालघाट भोरघाट थलघाट

Correct Answer -

Thalghat Bhorghat Palghat थलघाट भोरघाट पालघाट

Which of the following does not have influence over the climate in India

ि ि ि ी ी

72)

73)

िनिलखत म स िकसका भाव भारत की जलवाय पर नही पड़ता ह

1 Ocean currents सागर की लहर

2 Nearness to equator भम रखा स िनकटता

3 Monsoons मानसन

4 Presence of Indian ocean भारतीय महासागर की उपथित

Correct Answer -

Ocean currents सागर की लहर

Which of the following cloud types has the characteristics like vertical tall narrow and puffy

िनिलखत म स िकस कार क मघो म लबवत लबी सकीण और थलता जसी िवशषताए ह

1 Cumulonimbus तफानी मघ

2 Cumulus मघ पज

3 Cirrocumulus पाभ कपासी मघ

4 Nimbostratus वषारी मघ

Correct Answer -

Cumulus मघ पज

Which of the following statement is INCORRECT about Crude Birth Rate

िनिलखत स कौन सा कथन अशोिधत ज दर क बार म सही नही ह

1 It cannot be used for comparing fertility level between two countries with different population characteristics

इसका उपयोग िविभ जनसा िवशषताओ वाल दो दशो क बीच जनन र की तलना क िलए नही िकया जा सकता ह

2 It is a standardized measure of fertility

यह जनन मता का मानकीकत उपाय ह

3 It is effected by the age-sex composition of the population

यह आबादी की आय-िलग सरचना स भािवत होता ह

4 It is expressed per 1000 population in a given geographical unit

यह िकसी दी गई भौगोिलक इकाई म ित 1000 जनसा पर िकया जाता ह

Correct Answer -

It is a standardized measure of fertility

यह जनन मता का मानकीकत उपाय ह

74)

75)

76)

77)

Which of the following state in India experienced negative decadal growth rate during 2001 to 2011census

भारत म िनिलखत म स िकस रा म वष 2001 स 2011 की जनगणना क दौरान नकाराक िगरावट दर ई

1 Tripura िपरा

2 Nagaland नागालड

3 Haryana हरयाणा

4 Odisha ओिडसा

Correct Answer -

Nagaland नागालड

Which of the following is NOT a characteristic of peninsular rivers

िनिलखत म स कौन सी िवशषता ायीपीय निदयो म नही होती ह

1 Flow through shallow valleys उथल घािटयो क माम स वाह

2 Seasonal flow मौसमी वाह

3 Little erosional activity थोड़ी कटावदार गितिविध

4 Meandering tendency often shifting their beds घमावदार वि अर अपन तटो को थानातरत करना

Correct Answer -

Meandering tendency often shifting their beds घमावदार वि अर अपन तटो को थानातरत करना

Which of the following gases in the atmosphere absorbs heat from the Sunrsquos radiation and the Earthssurface

वायमडल म िनिलखत म स कौन सी गस सय क िविकरण और पी की सतह स ऊा को अवशोिषत करती ह

1 Neon िनयॉन

2 Carbon dioxide काबन डाइऑाइड

3 Argon आगन

4 Nitrogen नाइट ोजन

Correct Answer -

Carbon dioxide काबन डाइऑाइड

Which of the following kind of settlement pattern is found at the confluence of rivers

ि ि ि ि ो

78)

79)

80)

िनिलखत म स िकस कार का वथापन पटन निदयो क सगम पर पाया जाता ह

1 Triangular Paern िकोणीय पटन

2 Circular or Semi-Circular Paern परप या अध-परप पटन

3 Nebular Paern नबलर पटन

4 Star ndashShaped Paern ार-आकार का पटन

Correct Answer -

Triangular Paern िकोणीय पटन

Which one was not the objective of the Biosphere Reserve Projects launched by the UNESCO

यनो ारा श की गई सरित जवमडल परयोजनाओ का उ इनम स कौन सा नही था

1 To promote teaching and research िशण और अनसधान को बढ़ावा दना

2 To make agriculture sustainable किष को दीघकािलक बनाना

3 To conserve ecosystems पारथितक त को सरित करना

4 To conserve genetic diversity for a longtime लब समय तक अनवािशक िविवधता को सरित करना

Correct Answer -

To make agriculture sustainable किष को दीघकािलक बनाना

Which region of the Earth surface receives the highest amount of insulation

पी सतह का कौन सा तापावरोधन की उतम माा ा करता ह

1 Land mass थलखड

2 Savannah region सवाना

3 Water bodies जल िनकाय

4 Tropical desert उकिटबधीय रिगान

Correct Answer -

Tropical desert उकिटबधीय रिगान

Which one of the following is not a biodiversity hotspot

िनिलखत म स कौन सा जव िविवधता का म जगह नही ह

1 Eastern Himalaya पव िहमालय

2 Eastern Ghats पव घाट

81)

82)

83)

3 Indo-Myanmar भारत-ामार

4 Westerm Ghats पिमी घाट

Correct Answer -

Eastern Ghats पव घाट

Which one of the following is NOT a part of the World Network of Biosphere Reserves based on theUNESCO Man and Biosphere Programme

यनो मन और बायोीयर कायम क आधार पर िनिलखत म स कौन बायोीयर रजव क िव नटवक कािहा नही ह

1 Gulf of Mannar मार की खाड़ी

2 Seshachalam शषाचलम

3 Sunderban सदरबन

4 Nilgiri नीलिगर

Correct Answer -

Seshachalam शषाचलम

Which one of the following is an example of ldquodesert vegetationrdquo

िनिलखत म स कौन मथलीय वनित का एक उदाहरण ह

1 Mosses and lichens दलदल और शवाल

2 Temperate grassland समशीतो घास क मदान

3 Coniferous forest शकधारी वन

4 Acacia and cactus एकािसया और कस

Correct Answer -

Acacia and cactus एकािसया और कस

Which one of the following reflects more sunlight िनिलखत म स कौन सा सय की रोशनी को अिधकपरावितत करता ह

1 Paddy crop land धान फसल भिम

2 Land covered with fresh snow ताजा बफ स आािदत भिम

3 Sand desert रतीली रिगान

4 Prairie land यरी भिम

84)

85)

86)

87)

Correct Answer -

Land covered with fresh snow ताजा बफ स आािदत भिम

Which layer of the atmosphere is in contact with the surface of the earthrsquos oceans

वायमडल की कौन सी परत पी क महासागरो की सतह क सपक म ह

1 Stratosphere समताप मडल

2 Mesosphere म मडल

3 Hydrosphere जलमडल

4 Troposphere ोभ मडल

Correct Answer -

Troposphere ोभ मडल

Mediterranean Sea is a border of which of the following countries भम सागर िनिलखत दशो म सिकसकी सीमा ह

1 None of these इनम स कोई नही

2 Iraq इराक

3 Lebanon लबनान

4 Jordan जॉडन

Correct Answer -

Lebanon लबनान

Benguela ocean currents are found along which coast बगएला महासागर धाराए िकस तट क साथ पायीजाती ह

1 East Coast of South America दिण अमरका क पव तट

2 East Coast of Africa अीका क पव तट

3 West Coast of South America दिण अमरका क पिमी तट

4 West Coast of Africa अीका क पिमी तट

Correct Answer -

West Coast of Africa अीका क पिमी तट

88)

89)

90)

Due to tension a block of land on one side being pushed up or upthrown relative to the downthrown blockis referred as

तनाव क कारण नीच फ क ए खड क साप भिम का एक खड एक ओर स ऊपर धकला जाता ह या ऊपर की ओरफ का जाता ह यह _____ क प म सदिभत ह

1 Thrust fault प श

2 Normal fault सामा श

3 Reverse fault म श

4 Strike slip fault नितलब सपण श

Correct Answer -

Normal fault सामा श

Inter-tropical doldrums is a zone of ______ अतर-उकिटबधीय डोलड ______ का एक ह

1 Frontolysis टोलायिसस

2 Convergence अिभसरण

3 Inter-tropical divergence zone अतर-उकिटबधीय िवचलन

4 Local wind थानीय वाय

Correct Answer -

Convergence अिभसरण

The Horse Latitudes are regions located at about _____ north and south of the equator

हॉस अाश भम रखा क उर और दिण म लगभग _____ पर थत ह

1 30ndash60 degree Latitude 30-60 िडी अाश

2 0ndash5 degree Latitude 0-5 िडी अाश

3 30 degree Latitude 30 िडी अाश

4 60ndash90 degree Latitude 60-90 िडी अाश

Correct Answer -

30 degree Latitude 30 िडी अाश

Generally evaporation is high over which part of the Earth

आम तौर पर पी क िकस भाग पर वाीकरण अिधक होता ह

1 Equatorial maritime भमवत समीय ी ी

91)

92)

2 Equatorial continental भमवत महाीपीय

3 Polar maritime वीय समीय

4 Polar continental वीय महाीपीय

Correct Answer -

Equatorial maritime भमवत समीय

A very high temperature during summer in north western India leads to what type of climaticcondition in south

उर पिमी भारत म गम क दौरान बत अिधक तापमान होन क कारण दिण म िकस कार की जलवाय थितउ करता ह

1 Depression over arabian sea अरब सागर पर अवनमन

2 Failure monsoon मानसन िवफलता

3 Successful monsoon मानसन सफलता

4 Cyclones चवात

Correct Answer -

Successful monsoon मानसन सफलता

Lightning and thunder are the resultant effect when तिड़त और गजन परणामी भाव ह जब

1 Two massive clouds hit powerfully each other first lightning is produced and later sound is produced

दो बड़ बादल एक दसर स शशाली ढग स टकरात ह पहल आकाशीय िवदयत उ होता ह और बाद म िन उहोती ह

2 Two massive clouds come into contact with the powerful wind collision this results into first sound and thenlightning

दो बड़ बादल शशाली पवन सघ क सपक म आत ह इसका परणामप पहल िन और िफर आकाशीय िवदयतउ होता ह

3 None of the above उपरो म स कोई भी नही

4 A high density cloud contains positively and negatively charged electric ions and when this interacts light andsound are simultaneously produced

एक उ घन बादल म धनाक और ऋणाक आविशत िवदयत आयन होत ह और जब यह परर भाव डालत ह तोकाश और िन एक साथ उािदत होती ह

Correct Answer -

A high density cloud contains positively and negatively charged electric ions and when this interacts light andsound are simultaneously produced

औ ि ि ो औ ो

93)

94)

95)

एक उ घन बादल म धनाक और ऋणाक आविशत िवदयत आयन होत ह और जब यह परर भाव डालत ह तोकाश और िन एक साथ उािदत होती ह

Doon Valley is able to grow rice because दन घाटी चावल उगान म सम ह ोिक

1 Other crops cannot be grown वहा अ फसलो को उगाया नही जा सकता ह

2 People in the valley are rice eaters घाटी म लोग चावल खान वाल ह

3 There is a huge export demand of rice वहा चावल की भारी िनयात माग ह

4 It has warm summer and snow melt waters for irrigation

वहा गिमया गम होती ह िसचाई क िलए बफ का िपघला आ पानी होता ह

Correct Answer -

It has warm summer and snow melt waters for irrigation

वहा गिमया गम होती ह िसचाई क िलए बफ का िपघला आ पानी होता ह

CANCELLED

In the geological time scale the Mesozoic Era DOES NOT contains which of the following periods

भगभय समय पमान पर मजीवी यग म िन कालो म स कौन नही ह

1 Triassic ट ाइऐिसक

2 Jurassic जरिसक

3 Cretaceous चाकमय

4 Carboniferous काबनी

Correct Answer -

Carboniferous काबनी

96)

1 P-3 Q-4 R-2 S-1

2 P-3 Q-4 R-1 S-2

3 P-3 Q-4 R-1 S-2

4 P-4 Q-3 R-2 S-1

Correct Answer -

P-4 Q-3 R-2 S-1

1 P-3 Q-1 R-4 S-2

2 P-3 Q-4 R-1 S-2

3 P-3 Q-2 R-4 S-1

97)

98)

4 P-2 Q-1 R-4 S-3

Correct Answer -

P-3 Q-4 R-1 S-2

1 P-3 Q-1 R-4 S-2

2 P-2 Q-3 R-4 S-1

3 P-2 Q-1 R-3 S-4

4 P-4 Q-2 R-1 S-3

Correct Answer -

P-3 Q-1 R-4 S-2

99)

100)

1 P-3 Q-2 R-4 S-1

2 P-1 Q-2 R-3 S-4

3 P-2 Q-3 R-1 S-4

4 P-4 Q-3 R-2 S-1

Correct Answer -

P-2 Q-3 R-1 S-4

ldquoHuman geography is the study of changing relationship between the unresting man and the unstableearthrdquo was defined by

lsquolsquoमानव भगोल ाकल आदमी और अथर पी क बीच सबध परवतन का अयन हrdquo ______ ारा परभािषत िकया गयाथा

1 J Brunches ज चस

2 EC Semple ईसी सल

3 HJ Mackinder एच ज मिकदर

4 PV Blache पीवी च

Correct Answer -

EC Semple ईसी सल

Sedimentary rocks are finally and ultimately derived from the____________

अवसादी चान अततः ________ स ा की जाती ह

1 action of earth movements पी की गितिविधयो

2 marine deposit समी िनप

3 weathering of metamorphic rocks पातरत चानो क अपय

4 weathering of igneous rocks आय चानो क अपय

Correct Answer -

weathering of igneous rocks आय चानो क अपय

Page 5: High School Teacher Eligibility Test- BOARD PROFESSIONAL ...peb.mp.gov.in/results/RESULT_18/HST_RES18/Final_anwser_key/HST… · M a ndl a / मंड ल ... Under the Madhya Pradesh

4)

5)

6)

Out of the following options choose the most appropriate tense that best fills in to complete the givensentence

This year our cricket team _________ all the matches played on the Indian soil

1 will win

2 was winning

3 wins

4 has been winning

Correct Answer -

has been winning

Choose the appropriate adverb clause from the options to complete the given sentence

He was very pleased _____

1 which you passed

2 which you had passed

3 that you passed

4 that you who passed

Correct Answer -

that you passed

Out of the following four options choose the incorrect sentence

1 Rohit is a charmer and you can find him selling newspapers on M G Road from 800 am every morning

2 Diabetes a leading cause of heart problems does not allow people to make it to the hospital in time after a heartattack

3 Symbiosis Institute is Indias premier approved business school

4 Nehrus vision continued to light up the entire nation even today

Correct Answer -

Nehrus vision continued to light up the entire nation even today

7)

8)

1)

Out of the following options choose the most appropriate usage to fill in the blank

I live a few yards ______ the bus stop

1 off

2 from

3 outside

4 away

Correct Answer -

from

Arrange the sentences P Q R and S to form a logical sequence between sentences 1 and 6

1 Jurors do not have to be intelligentP In 1933 in a sheep-stealing case in Wales two of the jurors were totally unable to understand Englishbut no one found that out until after the trialQ There is no test no examination and no enquiry about their abilitiesR The convicted found this out and complained about it to the Court of Criminal AppealS No one even takes the trouble to find out whether they can understand the English language6 Still the Court of Appeal felt that it must refuse to interfere

1 QPRS

2 SQRP

3 PRQS

4 QSPR

Correct Answer -

QSPR

Topic- GENERAL KNOWLEDGE

Which country will host the 2022 Football World Cup 2022 फटबॉल िव कप की मजबानी करन वाला दशकौन होगा

1 America अमरका

2 Canada कनाडा

3 Qatar कतर

4 Mexico मको

2)

3)

4)

5)

Correct Answer -

Qatar कतर

According to sex ratio trends of Census 2011 which district of Madhya Pradesh has been ranked numberOne

जनगणना 2011 म िलग अनपात क झानो क अनसार म दश क िकस िजल को नबर एक थान िदया गया ह

1 Seoni िसओनी

2 Ratlam रतलाम

3 Balaghat बालाघाट

4 Mandla मडला

Correct Answer -

Balaghat बालाघाट

The folk dance form called lsquoMatkirsquo is prevalent in which of the following regions of Madhya Pradesh

मटकी नामक लोक न प म यदश क िनिलखत म स िकस म चिलत ह

1 Bundelkhand बदलखड

2 Malwa मालवा

3 Vindhyachal िवाचल

4 Baghelkhand बघलख

Correct Answer -

Malwa मालवा

Who appoints the Chairman and the Members of the Union Public Service Commission

सघ लोक सवा आयोग क अ और सदो की िनय कौन करता ह

1 Chief Justice of India भारत क म य यायधीश

2 Prime Minister धान मी

3 President रा ट पित

4 Chief Election Commissioner म य चनाव आय त

Correct Answer -

President रा ट पित

6)

7)

8)

Under the Madhya Pradesh Ladli Laxmi Yojana the beneficiary is paid the total amount on attaining theage of ___ मदश लाडली ली योजना क तहत लाभाथ की आय ___ वष परी होन पर कल रािश का भगतान िकयाजाता ह

1 18

2 21

3 23

4 16

Correct Answer -

21

India has won the gold medal in hockey ______times at the Olympics

ओलिपक खलो म भारत न हॉकी म ण पदक _____ बार जीता ह

1 9

2 7

3 8

4 6

Correct Answer -

8

Where is the Madhya Pradesh Tribal Museum located at

म दश जनजातीय सहालय कहा थत ह

1 Indore इदौर

2 Jhabua झबआ

3 Khargone खरगोन

4 Bhopal भोपाल

Correct Answer -

Bhopal भोपाल

The fictional boy character of the lsquoThe Jungle Bookrsquo by author Rudyard Kipling was

लखक डयाड िकपिलग ारा lsquoद जगल बकrsquo का कािनक बालक चर था

1 Kaa का

2 Mowgli मोगली

9)

10)

1)

3 Shere Khan शर खान

4 Bagheera बघीरा

Correct Answer -

Mowgli मोगली

The recipient of which one of the following awards given by Madhya Pradesh Government gets the highest cashprize

िनिलखत परारो म स िकसक ाकता को म दश सरकार ारा उतम नकद परार िदया जाता ह

1 Mahatma Gandhi award महाा गाधी परार

2 Shikar Award िशखर परार

3 Sharad Joshi Award शरद जोशी परार

4 Kishore Kumar Award िकशोर कमार परार

Correct Answer -

Mahatma Gandhi award महाा गाधी परार

Who among the following was the Trinidadian writer of Indian descent

िनिलखत म स भारतीय मल क ििनदािदयन लखक कौन थ

1 RK Narayan आर क नारायण

2 Anita Desai अनीता दसाई

3 Rudyard Kipling डयाड िकपिलग

4 VS Naipaul वीएस नायपॉल

Correct Answer -

VS Naipaul वीएस नायपॉल

Topic- GENERAL REASONING

Find the missing number ल त स या ात कर

4 196 16 169 ____ 144 64

1 81

2 36

3 32

4 21

2)

3)

4)

Correct Answer -

36

Find the average औसत ात कर

25 35 45 55 65 amp 75

1 48

2 50

3 47

4 49

Correct Answer -

50

The below series uses a sequence of alphabets and numbers Identify the incorrect combination

नीच दी गई खला अरो एव साओ का अनम योग करती ह गलत सयोजन पहचान

(i) FT85DF77ER

(ii) FT85DF77ER

(iii) FT85DE77ER

(iv) FT85DF77ER

1 ii

2 iv

3 i

4 iii

Correct Answer -

iii

A and B together can complete a piece of work in 10 days and B alone can complete the same work in 20days In how many days can A alone complete the work

A और B एक साथ काम क एक भाग को 10 िदनो म समा त करत ह और B अकल उसी काम को 20 िदनो म समा त करता हिकतन िदनो म A अकल उस काम को समा त कर सकता ह

1 20 days 20 िदन

2 30 days 30 िदन

3 10 days 10 िदन

4 25 days 25 िदन

5)

6)

Correct Answer -

20 days 20 िदन

Which one of the following four addresses is NOT EXACTLY same as the one given below

िनिलखत िदए गए चार पतो म स नीच िदए गए पत क ठीक समान कौन सा एक नही ह

Ground Floor 59 Goulburn St

Sydney NSW 2000 Australia

+61 (02) 8987 3700

(i) Ground Floor 59 Goulburn St

Sydney NSW 2000 Australia

+61 (02) 8987 3700

(ii) Ground Floor 59 Goulburn St

Sydney NSW 2000 Australia

+61 (02) 8987 3700

(iii) Ground Floor 59 Govlburn St

Sqdney NSW 2000 Australia

+61 (02) 8987 3700

(iv) Ground Floor 59 Goulburn St

Sydney NSW 2000 Australia

+61 (02) 8987 3700

1 ii

2 iv

3 i

4 iii

Correct Answer -

iii

In a group of 75 people 32 of them like cold drink 56 of them like general water and each person likesat least one of the two drinks How many people like both

75 लोगो क समह म उनम स 32 को को िड क पसद ह उनम स 56 को सामा जल पसद ह और क को दो म स कम स कम एक पय पसद ह िकतन लोगो को दोनो पसद ह

1 11

2 13

7)

1)

2)

3 12

4 14

Correct Answer -

13

If A = 1 FAT = 27 then FAITH =

यिद A = 1 FAT = 27 तो FAITH =

1 41

2 40

3 42

4 44

Correct Answer -

44

Topic- PEDAGOGY

CANCELLED

Basic education became a major initiative because of the efforts of

ाथिमक िशा िन न क यास स एक मह वपण कायम बन गई

1 Dr Radhakrishnan डॉ राधाक णन

2 Dr Rajendra Prasad डॉ राज साद

3 Mahatma Gandhi महा मा गाधी

4 Rabindranath Tagore रबी नाथ टगोर

Correct Answer -

Mahatma Gandhi महा मा गाधी

The ldquoclause of phraserdquo is an unit of perception of लॉज ऑफ ज िन न क धारणा क एक इकाई ह

1 Image छिव

2 Concept अवधारणा

3 Language भाषा

4 Thought िवचार

Correct Answer -

3)

4)

5)

6)

Language भाषा

Which method is most suitable to study communication process among students

छाो क बीच सचार िया का अयन करन क िलए कौन सी िविध सबस उपय ह

1 Case Study मामल का अयन (कस टडी)

2 Systematic Observation वथत अवलोकन

3 Experimental Method योगाक िविध

4 Introspection आ-िनरीण (इट ोस शन)

Correct Answer -

Systematic Observation वथत अवलोकन

Genes in a human being are located in मानवो म जीन इनम थत होता ह

1 cytoplasm कोिशका (साइटो ला म)

2 ribosomes राइबोसोम

3 cell membranes कोिशका िझी

4 chromosomes गणस (ोमोसोम)

Correct Answer -

chromosomes गणस (ोमोसोम)

With smaller classes teachers are much more able to ____________

छोटी काओ क साथ िशक ____________ म अिधक सम होत ह

1 Go slow while teaching िशण क दौरान धीमी गित स जान

2 Narrate more personal experiences अिधक गत अनभवो को बतान

3 Adapt instruction to individual differences among students छाो क बीच गत मतभदो क िलए अनकल िनदश दन म

4 Make use of the extra space for extra-curricular activities पातर गितिविधयो क िलए अितर जगह का उपयोग करन

Correct Answer -

Adapt instruction to individual differences among students छाो क बीच गत मतभदो क िलए अनकल िनदश दन म

While engaging in a task the child gets bored This is a sign of

एक काम म होन पर बा ऊब जाता ह यह िन का सकत ह

1 the task requiring a professional approach काय को ावसाियक िकोण की आवकता ह

7)

8)

9)

2 the task becoming mechanically repetitive काय यािक प स दोहराव वाला ह

3 the child not being intelligent बा बमान नही ह

4 the child being incapable of learning बा सीखन म असमथ ह

Correct Answer -

the task becoming mechanically repetitive काय यािक प स दोहराव वाला ह

Educational Psychologists are more concerned with the learning in __________

शिणक मनोवािनक __________ म अिधगम क साथ अिधक िचितत होत ह

1 Formal environment औपचारक वातावरण

2 Informal environment अनौपचारक वातावरण

3 Physical environment भौितक वातावरण

4 Social environment सामािजक वातावरण

Correct Answer -

Formal environment औपचारक वातावरण

Special needs education is the type of education िवशष ज़रतो वाली िशा वह िशा होती ह जो

1 Given to person with disabilities अम य को दी जाती ह

2 Given to people from remote areas दर थ ो क लोगो को दी जाती ह

3 Provided to intelligent people बमान लोगो को दी जाती ह

4 Established by colonial masters औपिनविशक मखयाओ ारा थािपत की गई ह

Correct Answer -

Given to person with disabilities अम य को दी जाती ह

The Stanford-Binet scale of intelligence was first published in the year

ब क नफोड-िबनट कल को इस वष म पहली बार कािशत िकया गया था

1 1916

2 1903

3 1908

4 1900

Correct Answer -

1916

10)

11)

12)

13)

The term lsquofictional finalismrsquo was propounded by

पद lsquoकत योजनवादrsquo (िफ शनल फाइनिल म) इनक ारा ितपािदत िकया गया था

1 Skinner नर

2 Freud ायड

3 Adler एडलर

4 Pavlov पावलोव

Correct Answer -

Adler एडलर

Rational Emotive Behavior Therapy was propounded by

तक सगत भावनाक वहार थरपी िन क ारा ितपािदत की गई थी

1 Carl Jung काल यग

2 Carl Rogers काल रोजस

3 Aaron Beck आरोन बक

4 Albert Ellis अट एिलस

Correct Answer -

Albert Ellis अट एिलस

Which of the following indicates the quality of education in a school

िनिलखत म स या एक कल म िशा की गणव ता का सकतक ह

1 Text-books and Teaching-learning material पा-प तक तथा िशण व सीखन स सबिधत सामी

2 Infrastructural facilities at the school कल म आधारभत ढाच स सबिधत सिवधाए

3 Student achievement level िवािथयो का उपल तर

4 Classroom systems का की यव था

Correct Answer -

Student achievement level िवािथयो का उपल तर

Which of the following teachers can be identified with authoritarian teaching style

िनिलखत म स कौन सा िशक अिधकारवादी िशण शली क साथ पहचाना जा सकता ह

1 Laissez-faire teacher अब ध िशक

14)

15)

2 Democratic teacher लोकतीय िशक

3 Indifferent teacher िन प िशक

4 Direct instruction teacher िनदश िशक

Correct Answer -

Direct instruction teacher िनदश िशक

Who was the pioneer of classical conditioning

िचरितित ानकलन ( लािसकल कडीशिनग) क वतक कौन थ

1 Skinner नर

2 Pavlov पावलोव

3 Kohler कोहलर

4 Freud ायड

Correct Answer -

Pavlov पावलोव

Which of the following are true with reference to short term memory

1 Limited capacity

2 Brief storage of information

3 Unlimited capacity

4 Duration of storage less than twenty seconds

अ पकािलक मित क सदभ म िन न म स या स य ह

1 सीिमत मता

2 सचना का सि त भडारण

3 असीिमत मता

4 बीस सकड स कम भडारण की अविध

1 3 and 4 3 और 4

2 2 and 4 2 और 4

3 13 and 4 13 और 4

4 12 and 4 12 और 4

Correct Answer -

12 and 4 12 और 4

1)

2)

3)

4)

Topic- GEOGRAPHY

Which of the following ranges of population size is used to define Class-III city by Indian Census

भारतीय जनगणना ारा वग-III शहर को परभािषत करन क िलए िनिलखत म स िकस रज क जनसा आकारका उपयोग िकया जाता ह

1 20 000 to 49999 20 000 स 49999

2 30000 to 59999 30000 स 59999

3 24000 to 54999 24000 स 54999

4 50000 to 99999 50000 स 99999

Correct Answer -

20 000 to 49999 20 000 स 49999

Carbonaceous rocks which produce coal and oil belong to the category of rocks called_______

कोयल और तल का उादन करन वाली काबनय चान ______ नामक चानो की णी स सबिधत ह

1 metamorphic पातरत

2 sedimentary अवसादी

3 inorganic अजिवक

4 igneous आय

Correct Answer -

sedimentary अवसादी

The ruhr-complex is a major industrial centre in र-परसर िन का एक मख औोिगक क ह

1 North America उरी अमरका

2 Russia स

3 Germany जमनी

4 Europe यरोप

Correct Answer -

Germany जमनी

The term lsquoRegurrsquo refers to श lsquoरगरrsquo ______ स सबिधत ह

1 Deltaic alluvial soils डा जलोढ़ िमी

ि ी

5)

6)

2 Laterite soils लटराइट िमी

3 Red and yellow soils लाल और पीली िमी

4 Black cotton soils काली कपास िमी

Correct Answer -

Black cotton soils काली कपास िमी

Read the given statements and answer which of the following options isare correct

1 Lower the pressure greater the atmospheric disturbance

2 Air move from higher to low pressure

िदए गए कथन को पढ़ और उर द िक िन म स कौन सास िवक सही ह

1 िजतना दाब कम होगा वायमडलीय बाधाए उतनी अिधक होगी

2 वाय उ स िन दाब की ओर गित करती ह

1 Both Statements 1 and 2 are correct दोनो कथन 1 और 2 सही ह

2 Both Statements 1 and 2 are wrong दोनो कथन 1 और 2 गलत ह

3 Statement 1 is wrong and only Statement 2 is correct कथन 1 गलत ह और कवल कथन 2 सही ह

4 Statement 1 is correct and Statement 2 is wrong कथन 1 सही ह और कथन 2 गलत ह

Correct Answer -

Both Statements 1 and 2 are correct दोनो कथन 1 और 2 सही ह

CANCELLED

1 4 1 2 and 3 4 1 2 और 3

2 4 1 3 and 2 4 1 3 और 2

3 1 4 2 and 3 1 4 2 और 3

4 1 4 3 and 2 1 4 3 और 2

Correct Answer -

7)

8)

9)

1 4 3 and 2 1 4 3 और 2

CANCELLED

Karl Pearsonrsquos correlation co-efficient is काल िपयसन का सहसबध गणाक ह

1 Arithmec mean समार मा

2 Geometric mean गणोर मा

3 None of these इनम स कोई नही

4 Harmonic mean हराक मा

Correct Answer -

Geometric mean गणोर मा

CANCELLED

ldquoEach day is more or less the same the morning is clear and bright with a sea breeze as the Sun climbshigh in the sky heat mounts up dark clouds form then rain comes with thunder lighting But rain is soonoverrdquo Which of the following regions is described in the above passage

ldquoक िदन समान स अिधक या कम होता ह सम की हवा क साथ और उल सबह होती ह जस सयआकाश म ऊचा चढ़ता ह गम बढ़ जाती ह काल बादल बनत ह िफर िबजली क साथ बारश आती ह लिकनबारश जी ख हो जाती हlsquorsquo उपरो पा म िनिलखत म स िकन ो का वणन िकया गया ह

1 Equatorial भमरखीय

2 Equatorial भमरखीय

3 Savannah सवाना

4 Mediterranean आातरक (भमसागरीय)

5 Mediterranean आातरक (भमसागरीय)

6 Monsoon मानसन

7 Monsoon मानसन

Correct Answer -

Equatorial भमरखीय

Equatorial भमरखीय

CANCELLED

In which epoch of the geological history of the Earth dinosaurs reached their largest size

पी क भगभय इितहास क िकस यग म डायनासोर अपन सबस बड़ आकार तक पचि

10)

11)

1 Triassic ट ायिसक

2 Jurassic जरािसक

3 Cretaceous ीटशस

4 Permian पिमयन

Correct Answer -

Jurassic जरािसक

CANCELLED

A spring tide would occur in which of the following conditions

िनिलखत म स िकन थितयो म ार-भाटा आता ह

1 When the Sun Moon and Earth are in a straight line

जब सय चमा और पी एक सीधी रखा म होत ह

2 When the Moon and Earth are in right angle to each other

जब चमा और पी एक-दसर क दािहन कोण म होत ह

3 When the Earth and Moon are in right angle to the Sun

जब पी और चमा सय क दािहन कोण म होत ह

4 When the Sun and Moon are in right angle to each other

जब सय और चमा एक-दसर क दािहन कोण म होत ह

Correct Answer -

When the Sun Moon and Earth are in a straight line

जब सय चमा और पी एक सीधी रखा म होत ह

CANCELLED

An observe on the Earthrsquos surface always sees the same face of the moon because

एक पयवक को पी की सतह स हमशा चाद का एक ही फलक िदखाई दता ह ोिक

1 Its path of revolution around the earth is the same as that of the earth around the Sun

इसका पी क चारो ओर परमण का माग सय क चारो ओर पी क समान ही ह

2 Its period of revolution around the Earth is the same as its period of rotation around its own axis

इसकी पी क चारो ओर परमण की अविध उसकी अपनी धरी क चारो ओर घणन की अविध क समान ह

3 Its period of rotation is the same as that of the Earth इसकी घणन की अविध पी क समान ह

ी ि ी ी

12)

13)

14)

4 Its direct of rotation is the same as that of Earth घणन की िदशा पी क समान ही ह

Correct Answer -

Its period of revolution around the Earth is the same as its period of rotation around its own axis

इसकी पी क चारो ओर परमण की अविध उसकी अपनी धरी क चारो ओर घणन की अविध क समान ह

CANCELLED

The pebbles that are faceted by the sand-blasting and shaped polished by the wind abrasions are known as

पवन अपघषन ारा पॉिलश रत-िवोिटत और साच म ढला ककड़ ___________ क प म जाना जाता ह

1 Dreikanter िकोणक

2 Pediments िकोिनका

3 Inselberg इलबग

4 Dunes टीबा

Correct Answer -

Dreikanter िकोणक

CANCELLED

Astronomical unit is the average distance between खगोलीय इकाई ______ क बीच की औसत दरी ह

1 Earth and Mars पी और मगल

2 Earth and mercury पी और बध

3 Earth and moon पी और चमा

4 Earth and Sun पी और सय

Correct Answer -

Earth and Sun पी और सय

During cold weather season in the northern plains there will be an inflow of cyclonic disturbancesfrom the _________ directions

शीत मौसम क दौरान उरी मदानी इलाको म _________ िदशाओ स चवात सबधी गड़बड़ी का अतवाह होगा

1 East and Northwest पव और उरपिम

2 East and Northeast पव और पवर

3 West and East पिम और पव

ि औ ि

15)

16)

17)

4 West and Northwest पिम और उरपिम

Correct Answer -

West and Northwest पिम और उरपिम

During an earth quake the velocity of the body waves will________ along with the increase in densityof the material it is passing through

भकप क दौरान लहरो क ऊपरी भाग का वग घन म व क साथ-साथ ________ जो इसस गजरन वाली वको आग बढाएगी

1 not change नही बदलगा

2 increase initially and then decrease शआत म बढ़गा और िफर घटगा

3 increase बढ़गा

4 decrease घटगा

Correct Answer -

increase बढ़गा

The Clouded Leopard National park is situated in which of the following states

िनिलखत म िकस रा म धिमल तदआ रा ीय उान (ाउडड लपड नशनल पाक ) थत ह

1 Tripura िपरा

2 Uttar Pradesh उर दश

3 Assam असम

4 Mizoram िमजोरम

Correct Answer -

Tripura िपरा

Usually the land surfaces are heated more quickly than the water surfaces because _____________

आम तौर पर जल सतहो की तलना म भिम सतह अिधक तजी स गम होती ह ोिक _____________ ह

1 the specific heat of water is higher than land पानी की िविश ऊा भिम स अिधक

2 the specific heat of water is lesser than land पानी की िविश ऊा भिम स कम होती

3 the latent heat of water is higher than the land पानी की अतिनिहत ऊा भिम स अिधक

4 the land reflects more heat radiation than water भिम पानी की तलना म अिधक ऊा क िविकरण को पराविततकरती

18)

19)

20)

21)

Correct Answer -

the specific heat of water is higher than land पानी की िविश ऊा भिम स अिधक

The longest shore-line is along the state of सबस लबी समतटीय रखा िन रा क साथ ह

1 Maharashtra महारा

2 Orissa उड़ीसा

3 Kerala करल

4 Gujarat गजरात

Correct Answer -

Gujarat गजरात

The position when the Earth is farthest from the Sun is known as

जब पी सय स सबस दर होती ह तो उस थित को िन नाम स जाना जाता ह

1 Perihelion उपसौर

2 Vernal Equinox बसत िवषव

3 Aphelion अपसौर

4 Autumnal Equinox शराल िवषव

Correct Answer -

Aphelion अपसौर

The seasonal reversal of winds is the typical characteristic of

हवाओ का मौसमी परवतन ______ की सामा िवशषता ह

1 Mediterranean climates only कवल भमसागरीय जलवाय

2 All of the above climates उपय सभी मौसम

3 Monsoon climate only कवल मानसन जलवाय

4 Equatorial climate only कवल भमरखीय जलवाय

Correct Answer -

Monsoon climate only कवल मानसन जलवाय

In _________ rocks the minerals will occurs in beds or layers

ो ि ो ो

22)

23)

24)

______ चानो म खिनज तल या परतो म होत ह

1 metamorphic कायातरत

2 igneous and metamorphic आय और कायातरत

3 igneous आय

4 sedimentary अवसादी

Correct Answer -

sedimentary अवसादी

Black soil is ideal for the cultivation of cotton as कपास की खती क िलए काली िमी आदश ह ोिक

1 Its colour is black यह काली होती ह

2 It is found on plateau regions यह पठार ो म पायी जाती ह

3 It is made up of lava यह लावा स बनी होती ह

4 It can retain moisture यह नमी को बरकरार रख सकती ह

Correct Answer -

It can retain moisture यह नमी को बरकरार रख सकती ह

The National Survey and Mapping Organization of the country works under the Department of___________

दश का रा ीय सवण और मानिचण सगठन ___________ िवभाग क अतगत काय करता ह

1 Space अतर

2 Science and Technology िवान और तकनीक

3 Culture सित

4 Tourism पयटन

Correct Answer -

Science and Technology िवान और तकनीक

Palk strait separates India from पाक जलडमम भारत स _____ को अलग करता ह

1 Pakistan पािकान

2 Andaman Island अडमान ीप

3 China चीन

25)

26)

27)

4 Sri Lanka ीलका

Correct Answer -

Sri Lanka ीलका

Which among the following state is the major producer of Bauxite in India

िनिलखत म स कौन सा रा भारत म बॉाइट का मख उादक ह

1 Madhya Pradesh मदश

2 Rajasthan राजथान

3 Goa गोवा

4 Orissa उड़ीसा

Correct Answer -

Orissa उड़ीसा

Which of the following states DOES NOT share border with Chhattisgarh

िनिलखत म स कौन सा रा छीसगढ़ क साथ सीमा साझा नही करता ह

1 Telangana तलगाना

2 Uttar Pradesh उर दश

3 Bihar िबहार

4 Andhra Pradesh आ दश

Correct Answer -

Bihar िबहार

Which of the following statements is INCORRECT with respect to parallels of latitudes

अाश क समानातरो क सबध म िन निलखत म स कौन सा कथन गलत ह

1 A line joining places of equal latitude is known as parallel of largest

समान अाश क थानो को जोड़न वाली रखा को िवशालतम क समानातर क प म जाना जाता ह

2 They stat from equator and run parallels to it

व भम रखा स ारभ होत ह और इसक समानातर चलत ह

3 All parallels are equal in length सभी समातर लबाई म समान ह

4 All parallels are drawn as circles on the globe ोब पर सभी समानातर वो क प म खीच जात ह

28)

29)

30)

31)

Correct Answer -

All parallels are equal in length सभी समातर लबाई म समान ह

Which of the following Indian states is also known as a lsquoLand of Red river and Blue Hillsrsquo

िनिलखत म स िकस भारतीय रा को लाल नदी और नीली पहािड़यो की भिम क नाम स जाना जाता ह

1 Uttarkhand उराखड

2 Assam असम

3 Meghalaya मघालय

4 Arunachal Pradesh अणाचल दश

Correct Answer -

Assam असम

In spatial analysis of settlement Rn = 215 indicates which type of settlement arrangement

िनपटान क थािनक िवषण म Rn = 215 यह इिगत करता ह िक िकस कार की िनपटान वथा ह

1 Uniform यिनफॉम

2 Semi-Clustered समी- ल टर

3 Clustered ल टर

4 Random रडम

Correct Answer -

Uniform यिनफॉम

Who are known as the lsquoYellow Peoplersquo lsquoयलो पीपलrsquo क प म कौन जाना जाता ह

1 Mongoloids मोगोलोइडस

2 Nigroids नीोइडस

3 Australoids ऑ लॉइडस

4 Caucasoids कॉकसोइडस

Correct Answer -

Mongoloids मोगोलोइडस

ि ि ो ौ ि

32)

33)

34)

Who publishes the topographical map of India भारत क थलाकितक मानिच को कौन कािशत करता ह

1 Geographical Survey of India भारत का भौगोिलक सवण

2 Government of India भारत सरकार

3 Geological Survey of India भारत क भगभय सवण

4 Survey of India भारत का सवण

Correct Answer -

Survey of India भारत का सवण

Who among the following claimed geography to be the lsquoEcology of Manrsquo

िनिलखत म स िकसन भगोल को मन का पारथितकी कहा ह

1 Alfred Hener अड हटनर

2 Vidal-de la Blache वाइडल-िड लॉ ॉश

3 Oo Schluter ओटो टर

4 Harlan Barrow हरलन बारो

Correct Answer -

Harlan Barrow हरलन बारो

Who among the following is regarded as the founder of humanistic approach in geography

िनिलखत म स िकस भगोल म मानवतावादी िकोण का सथापक माना जाता ह

1 William Bunge िविलयम बग

2 Yi-Fu-Tuan यी-फ- यान

3 Brain JL Berry न जएल बरी

4 Richard Peet रचड पीट

Correct Answer -

Yi-Fu-Tuan यी-फ- यान

Who prepared Lorenz curve लोरज व िकसन तयार िकया

1 Geddes गडस

2 None of these इनम स कोई नही

3 Griffith Taylor ििफथ टलर

35)

36)

37)

4 Max U Lorenz म य लोरज

Correct Answer -

Max U Lorenz म य लोरज

Gulf Streams are the currents of which of the following oceans

खाड़ी की धाराए िनिलखत महासागरो म स िकसकी धाराए ह

1 North Atlantic Ocean उरी अटलािटक महासागर

2 North Pacific Ocean उरी शात महासागर

3 Arabian Sea अरब सागर

4 South Pacific Ocean दिण शात महासागर

Correct Answer -

North Atlantic Ocean उरी अटलािटक महासागर

Disintegration wearing away and removal of rock material is generally referred as

िशला पदाथ (रॉक सामी) का टटना िमटना और हटना आमतौर पर ________ क प म सदिभत िकया जाता ह

1 Shattering िवसकारक

2 Denudation अनाादन

3 Fault श

4 Decomposition िवयोजन

Correct Answer -

Denudation अनाादन

Variations in the length of day time and night from season to season are due to

मौसम स मौसम परवतन पर िदन क समय और रात क समय की अविध म िभताए िन कारण स होती ह

1 The Earthrsquos revolution round the Sun in an elliptical manner पी का दीघवाकार तरीक स सय क चारो घणन

2 The Earthrsquos rotation on its axis पी का इसकी धरी पर घणन

3 Revolution of the Earth on a tilted axis नत अ पर पी का घणन

4 Latitudinal position of the place थान की अाश थित

Correct Answer -

Revolution of the Earth on a tilted axis नत अ पर पी का घणन

38)

39)

40)

Point out the correct sequence of mountain ranges from north to south

उर स दिण तक पवत खलाओ क सही अनम को इिगत कर

1 Great Himalaya Middle Himalaya Outer Himalaya Trans Himalaya

महान िहमालय म िहमालय बा िहमालय परा िहमालय

2 Middle Himalaya Great Himalaya Trans Himalaya Outer Himalaya

म िहमालय महान िहमालय परा िहमालय बा िहमालय

3 Outer Himalaya Middle Himalaya Great Himalaya Trans Himalaya

बा िहमालय म िहमालय महान िहमालय परा िहमालय

4 Trans Himalaya Great Himalaya Middle Himalaya Outer Himalaya

परा िहमालय महान िहमालय म िहमालय बा िहमालय

Correct Answer -

Trans Himalaya Great Himalaya Middle Himalaya Outer Himalaya

परा िहमालय महान िहमालय म िहमालय बा िहमालय

Sunrsquos halo is produced by the refraction of light in सय का भामडल ______ म काश क अपवतन ाराउ होता ह

1 Ice crystals in Cirrro-Cumulus clouds पाभ-कपास मघो क बफ िल

2 Ice crystal in Cirrus clouds पाभमघो क बफ िल

3 Dust particles in Stratus clouds री मघो क धल कण

4 Water vapour in Stratus clouds री मघो क जल वा

Correct Answer -

Ice crystal in Cirrus clouds पाभमघो क बफ िल

Read the given statements and answer which of the following options isare correct

(1) The minerals present in the rocks exposed to atmosphere are not subjected to alteration

(2) Oxidation is one of the processes of chemical weathering

िदए गए कथनो को पढ़ और उर द िक िन म स कौन सास िवक सही ह

(1) वायमल स अनावत शल म उपथत खिनज परवतन क अधीन नही होता ह

(2) ऑीकरण रासायिनक अपय की ियाओ म स एक ह

ो ो

41)

42)

1 Both statements are wrong दोनो कथन गलत ह

2 Both statements are correct दोनो कथन सही ह

3 First statement is wrong and second statement is correct पहला कथन गलत ह और दसरा कथन सही ह

4 First statement is correct and second statement is wrong पहला कथन सही ह और दसरा कथन गलत ह

Correct Answer -

First statement is wrong and second statement is correct पहला कथन गलत ह और दसरा कथन सही ह

Read the given statements and answer which of the following options isare correct

1 Sunrsquos short waves enter the earth partially heating the atmosphere

2 Heated earth surface from the sun produces broader waves which interacts and heats the atmosphere

िदए गए कथन को पढ़ और उर द िक िन म स कौन सास िवक सही ह

1 सय की छोटी तरग पी म आिशक प स वश करती ह और वायमडल को ऊत करती ह

2 सय स ऊत पी की सतह िवारत तरग उ करती ह जो परर भाव डालती ह और वायमडल कोऊत करती ह

1 Both Statements 1 and 2 are correct दोनो कथन 1 और 2 सही ह

2 Both Statements 1 and 2 are wrong दोनो कथन 1 और 2 गलत ह

3 Statement 1 is wrong and only Statement 2 is correct कथन 1 गलत ह और कवल कथन 2 सही ह

4 Only statement 1 is correct कवल कथन 1 सही ह

Correct Answer -

Both Statements 1 and 2 are correct दोनो कथन 1 और 2 सही ह

Read the given statements and answer which of the following options isare correct

(1)The rocks that get changed due to heat and pressure are termed as metamorphic rocks

(2)Slate is one such type of metamorphic rock

िदए गए कथनो को पढ़ और उर द िक िन म स कौन सास िवक सही ह

(1) शल जो ऊा और दाब क कारण परवितत हो जात ह उ कायातरक शलो क प म जाना जाता ह

(2) ट एक तरह का कायातरक शल ह

1 Both statements are wrong दोनो कथन गलत ह

2 Both statements are correct दोनो कथन सही ह

3 First statement is wrong and second statement is correct पहला कथन गलत ह और दसरा कथन सही ह

ी औ

43)

44)

4 First statement is correct and second statement is wrong पहला कथन सही ह और दसरा कथन गलत ह

Correct Answer -

Both statements are correct दोनो कथन सही ह

Read the given statements and answer which of the following options isare correct

1 Higher temperature anomaly is observed in the northern hemisphere

2 Differential heating is absent in Northern Hemisphere

िदए गए कथनो को पढ़ और उर द िक िन म स कौन सास िवक सही ह

1 उरी गोलाध म उ तापमान िवसगित पायी जाती ह

2 उरी गोलाध म अतर ऊन अनपथत होती ह

1 Both Statements 1 and 2 are correct दोनो कथन 1 और 2 सही ह

2 Both Statements 1 and 2 are wrong दोनो कथन 1 और 2 गलत ह

3 Statement 1 is wrong and Statement 2 is correct कथन 1 गलत ह और कथन 2 सही ह

4 Statement 1 is correct and Statement 2 is wrong कथन 1 सही ह और कथन 2 गलत ह

Correct Answer -

Statement 1 is correct and Statement 2 is wrong कथन 1 सही ह और कथन 2 गलत ह

Read the given statements and answer which of the following options isare correct

(1) Plutonic rocks are intrusive type of igneous rocks

(2) It cools very slowly because the surrounding rock serves as insulation around the intrusion of magma

िदए गए कथनो को पढ़ और उर द िक िन म स कौन सास िवक सही ह

(1) िवतलीय शल अतवधी कार क आश शल ह

(2) यह बत धीर-धीर ठडा होता ह ोिक आस-पास क शल मा क अतवधन क चारो ओर रोधन क प म कायकरत ह

1 Both statements are wrong दोनो कथन गलत ह

2 Both statements are correct दोनो कथन सही ह

3 First statement is wrong and second statement is correct पहला कथन गलत ह और दसरा कथन सही ह

4 First statement is correct and second statement is wrong पहला कथन सही ह और दसरा कथन गलत ह

Correct Answer -

Both statements are correct दोनो कथन सही ह

45)

46)

47)

48)

The dust and ash material hurled from the volcanoes are termed as

ालामखी स िनकलन वाली धल और राख सामी को _______ क प म कहा जाता ह

1 Pyroclasc पाइरोाक

2 Hyperclastic हाइपराक

3 Hepiroclastic हिपरोाक

4 Cirroclastic िसरोाक

Correct Answer -

Pyroclasc पाइरोाक

The vertical difference in elevation between a low tide and high tide is referred as

कम ार और उ ार क बीच ऊचाई म लबवत अतर _____ स सदिभत होता ह

1 Tidal slope ारीय ढलान

2 Tidal elevation ारीय उयन

3 Tidal range ारीय परास

4 Tidal height ारीय ऊचाई

Correct Answer -

Tidal range ारीय परास

The maximum biodiversity is found in which of the following regions िनिलखत ो म स अिधकतमजव िविवधता िकसम पायी जाती ह

1 Amazon Basin अमज़न बिसन

2 East Indies ई इडीज

3 Congo Basin कागो बिसन

4 West indies व इडीज

Correct Answer -

Amazon Basin अमज़न बिसन

The cultivation of rice crop produces_______ चावल की फसल की खती ______ का उादन करती ह

1 SO2

49)

50)

51)

2 CH4

3 CFCs

4 CO2

Correct Answer -

CH4

The pressure system with higher pressure at the centre is called__________

क म उ दबाव वाली दबाव णाली को _______ कहा जाता ह

1 front अ

2 depression अवनमन

3 cyclone चवात

4 anti-cyclone ितचवात

Correct Answer -

anti-cyclone ितचवात

The Himalayan region is poor in mineral resources because िहमालयी खिनज ससाधनो म समनही ह ोिक

1 The displacement of rock strata has disturbed the arrangement of rocks and made it complex

शलीय परत क िवथापन न चानो की वथा को अवथत कर िदया ह और इस जिटल बना िदया ह

2 The climate conditions are not suitable for exploitation of minerals

जलवाय की थित खिनजो क दोहन क िलए उपय नही ह

3 The terrain makes explanation of minerals difficult and very costly due to transportation difficulties

भ-भाग परवहन की किठनाइयो क कारण खिनजो का दोहन मल और बत महगा बना दता ह

4 It is made up of crystalline rocks यह िलीय चानो स बना ह

Correct Answer -

The displacement of rock strata has disturbed the arrangement of rocks and made it complex

शलीय परत क िवथापन न चानो की वथा को अवथत कर िदया ह और इस जिटल बना िदया ह

The process through which the moisture is added to the atmosphere by vegetation is termed as

वह िया िजसक माम स वनित ारा वातावरण म नमी िमलायी जाती ह _______ क प म जानी जाती ह

52)

53)

54)

1 Condensation सघनन

2 Evapotranspiration वान-उजन

3 Radiation िविकरण

4 Precipitation वषण

Correct Answer -

Evapotranspiration वान-उजन

The process through which the terrestrial heat is transferred to air by direct contact is termed as

वह िया िजसम सपक ारा थलीय ऊा वाय म थानातरत हो जाती ह ______ क प म जानी जाती ह

1 Conduction चालन

2 Convection सवहन

3 Insolation आतपन

4 Radiation िविकरण

Correct Answer -

Conduction चालन

The largest area under mangroves is in which of the following statesunion territory

मोव क अतगत िनिलखत राोसघ शािसत दशो म स सबस बड़ा कौन सा ह

1 Andaman and Nicobar अमान और िनकोबार

2 Andhra Pradesh आ दश

3 West Bengal पिम बगाल

4 Gujarat गजरात

Correct Answer -

West Bengal पिम बगाल

The longitudinal transverse and surface waves in an earthquake originate from

भकप म दशातर अनथ और सतह तरग यहा उ होती ह

1 The focus on the surface of the Earth पी क सतह पर क -िबद म

2 The focus within the body of the Earth पी क भीतर क -िबद म

3 The epicenter within the body of the Earth पी क भीतर उपरक म

55)

56)

57)

4 The epicenter on the surface of the Earth पी क सतह पर उपरक म

Correct Answer -

The focus within the body of the Earth पी क भीतर क -िबद म

The down slope movement of material due to gravity is called______

गाकषण क कारण पदाथ की अनढाल गित को ______ कहा जाता ह

1 mass movement पदाथ सचलन

2 deposition िनप

3 erosion रण

4 volcanic movement ालामखीय सचलन

Correct Answer -

mass movement पदाथ सचलन

Shimla is cooler than Amritsar although both are on the same latitude This is because

िशमला म अमतसर स अिधक ठड ह हालािक दोनो समान अाश पर ह ऐसा ह ोिक

1 Shimla is at a greater height above sea level than Amritsar अमतसर की तलना म िशमला सम तल स अिधकऊचाई पर ह

2 Shimla is further north िशमला उर की ओर ह

3 Shimla is farther from the equator िशमला भम रखा स आग ह

4 Their longitudes differ उनकी दशातर रखाए िभ ह

Correct Answer -

Shimla is at a greater height above sea level than Amritsar अमतसर की तलना म िशमला सम तल स अिधकऊचाई पर ह

lsquoTempo of Urbanizationrsquo measures which of the following

lsquoशहरीकरण का टपोrsquo िनिलखत म स कौन सा उपाय ह

1 Speed of urbanizaon शहरीकरण की गित

2 None of the above इनम स कोई नही

3 Inequality of urbanizaon शहरीकरण की असमानता

4 Current level of urbanizaon शहरीकरण का वतमान र

Correct Answer -

58)

59)

60)

Speed of urbanizaon शहरीकरण की गित

Out of the following options choose the INCORRECT statement

िनिलखत िवको म स गलत कथन का चयन कर

1 The clear tracts in the equatorial region recover rapidly भम रखा म भभाग तजी स ठीक हो जात ह

2 The stable communities include a redwood forest a pine forest at high elevations

थर समदायो म एक रडवड वन उ ऊचाई पर एक दवदार वन शािमल ह

3 Any ecosystem moves towards maximum biomass and stability to survive

कोई भी पारथितकी त जीिवत रहन क िलए अिधकतम जवसहित और थरता की तरफ असर होता ह

4 Tropical rain forests near equator are stable ecosystems

भम रखा क पास उकिटबधीय वषा वन थर पारथितक त ह

Correct Answer -

The clear tracts in the equatorial region recover rapidly भम रखा म भभाग तजी स ठीक हो जात ह

Seasonal contrasts are maximum in मौसमी िवषमता अिधकतम ह

1 Mid latitudes म अाश म

2 Low attitudes िन अाश म

3 High latitudes उ अाश म

4 Subtropics उपोकिटबधीय म

Correct Answer -

Mid latitudes म अाश म

In India which type of forest among the following occupies the largest area

भारत म िनिलखत म स िकस कार क वन सबस बड़ा फल आािदत करत ह

1 Sub-tropical Dry Evergreen Forest उप उकिटबधीय श सदाबहार वन

2 Mountain Wet Temperate Forest पवतीय आ शीतो वन

3 Tropical Moist Deciduous Forest उकिटबधीय आ पणपाती वन

4 Tropical Wet Evergreen Forest उकिटबधीय आ सदाबहार वन

Correct Answer -

Tropical Moist Deciduous Forest उकिटबधीय आ पणपाती वन

61)

62)

63)

64)

What is the proportion of lsquoJuvenile Populationrsquo (0-14 years) in India as per 2011Census

2011 की जनगणना क अनसार भारत म जवनाइल पॉपलशन यानी िकशोर जनस या (0-14 वष) का अनपात ाह

1 3076 of total population कल जनस या का 3076

2 2764 of total population कल जनस या का 2764

3 2933 of total population कल जनस या का 2933

4 3354 of total population कल जनस या का 3354

Correct Answer -

3076 of total population कल जनस या का 3076

What is the Belfast famous for बलफा िकसक िलए मशर ह

1 Belt of cotton textile industry कपास व उोग क

2 Ship-building industry जहाज िनमाण उोग

3 Agricultural machinery किष उपकरण

4 Aero planes manufacturing वाययान िनमाण

Correct Answer -

Ship-building industry जहाज िनमाण उोग

What is the most important occupation in tropical monsoon lands

उकिटबधीय मॉनसन भिम म सबस महपण वसाय ा ह

1 Mining खनन

2 Cattle rearing मवशी पालन

3 Agriculture किष

4 Nomadic herding नोमािडक जड़ी-बिटया

Correct Answer -

Agriculture किष

What is the most important characteristics of the islands (Indian) located in the Arabian sea

अरब सागर म थत ीपो (भारतीय) की सबस महपण िवशषता ा ह

ी ो

65)

66)

67)

1 There are all of coral origins सभी कोरल मल क ह

2 There are all very small in size य सभी आकार म बत छोट ह

3 They have a very dry climate इनकी जलवाय बत श ह

4 They are extended parts of the mainland व महाीप क िवारत िह ह

Correct Answer -

There are all of coral origins सभी कोरल मल क ह

What do the basalt layers of the Deccan indicate डन की बसा परत ा इिगत करती ह

1 All of the above उपरो सभी

2 Huge volcanic eruptions in the distant past दरथ अतीत म िवशाल ालामखीय िवोट

3 The immense erosional activity of the rivers निदयो की िवशाल रण गितिविध

4 The influence of weathering मौसम का भाव

Correct Answer -

Huge volcanic eruptions in the distant past दरथ अतीत म िवशाल ालामखीय िवोट

In the structure of planet Earth below the mantle the core is mainly made up of_____

पी ह की सरचना म मटल क नीच कोर म प स______ स िनिमत होती ह

1 aluminium एमीिनयम

2 silicon िसिलकॉन

3 chromium ोिमयम

4 iron लोहा

Correct Answer -

iron लोहा

One of the major Mid Oceanic Ridge is found in मख म-महासागर चोिटयो म स एक ______ म पायाजाता ह

1 Mid Pacific Ocean म शात महासागर

2 Mid Atlantic Ocean म अटलािटक महासागर

3 Mid Indian Ocean म भारतीय महासागर

4 Mid Arctic Ocean म आक िटक महासागर

68)

69)

70)

71)

Correct Answer -

Mid Atlantic Ocean म अटलािटक महासागर

Magma that reaches the Earthrsquos surface and then solidifies is called________

मा जो पी की सतह तक पचती ह और िफर ठोस हो जाती ह ________कहलाती ह

1 quartz ाटज

2 lava लावा

3 granite नाइट

4 silicates िसिलकट

Correct Answer -

lava लावा

Isotherms are the lines of equal_______ समताप रखाए समान _______की रखाए होती ह

1 pressure दाब

2 temperature तापमान

3 rainfall वषा

4 height ऊचाई

Correct Answer -

temperature तापमान

Mark the correct sequence of passes in the Western Ghats from north to south

पिमी घाटो म उर स दिण तक दर क सही अनम को िचित कर

1 Thalghat Palghat Bhorghat थलगघाट पालघाट भोरघाट

2 Thalghat Bhorghat Palghat थलघाट भोरघाट पालघाट

3 Bhorghat Thalghat Palghat भोरघाट थलघाट पालघाट

4 Palghat Bhorghat Thalghat पालघाट भोरघाट थलघाट

Correct Answer -

Thalghat Bhorghat Palghat थलघाट भोरघाट पालघाट

Which of the following does not have influence over the climate in India

ि ि ि ी ी

72)

73)

िनिलखत म स िकसका भाव भारत की जलवाय पर नही पड़ता ह

1 Ocean currents सागर की लहर

2 Nearness to equator भम रखा स िनकटता

3 Monsoons मानसन

4 Presence of Indian ocean भारतीय महासागर की उपथित

Correct Answer -

Ocean currents सागर की लहर

Which of the following cloud types has the characteristics like vertical tall narrow and puffy

िनिलखत म स िकस कार क मघो म लबवत लबी सकीण और थलता जसी िवशषताए ह

1 Cumulonimbus तफानी मघ

2 Cumulus मघ पज

3 Cirrocumulus पाभ कपासी मघ

4 Nimbostratus वषारी मघ

Correct Answer -

Cumulus मघ पज

Which of the following statement is INCORRECT about Crude Birth Rate

िनिलखत स कौन सा कथन अशोिधत ज दर क बार म सही नही ह

1 It cannot be used for comparing fertility level between two countries with different population characteristics

इसका उपयोग िविभ जनसा िवशषताओ वाल दो दशो क बीच जनन र की तलना क िलए नही िकया जा सकता ह

2 It is a standardized measure of fertility

यह जनन मता का मानकीकत उपाय ह

3 It is effected by the age-sex composition of the population

यह आबादी की आय-िलग सरचना स भािवत होता ह

4 It is expressed per 1000 population in a given geographical unit

यह िकसी दी गई भौगोिलक इकाई म ित 1000 जनसा पर िकया जाता ह

Correct Answer -

It is a standardized measure of fertility

यह जनन मता का मानकीकत उपाय ह

74)

75)

76)

77)

Which of the following state in India experienced negative decadal growth rate during 2001 to 2011census

भारत म िनिलखत म स िकस रा म वष 2001 स 2011 की जनगणना क दौरान नकाराक िगरावट दर ई

1 Tripura िपरा

2 Nagaland नागालड

3 Haryana हरयाणा

4 Odisha ओिडसा

Correct Answer -

Nagaland नागालड

Which of the following is NOT a characteristic of peninsular rivers

िनिलखत म स कौन सी िवशषता ायीपीय निदयो म नही होती ह

1 Flow through shallow valleys उथल घािटयो क माम स वाह

2 Seasonal flow मौसमी वाह

3 Little erosional activity थोड़ी कटावदार गितिविध

4 Meandering tendency often shifting their beds घमावदार वि अर अपन तटो को थानातरत करना

Correct Answer -

Meandering tendency often shifting their beds घमावदार वि अर अपन तटो को थानातरत करना

Which of the following gases in the atmosphere absorbs heat from the Sunrsquos radiation and the Earthssurface

वायमडल म िनिलखत म स कौन सी गस सय क िविकरण और पी की सतह स ऊा को अवशोिषत करती ह

1 Neon िनयॉन

2 Carbon dioxide काबन डाइऑाइड

3 Argon आगन

4 Nitrogen नाइट ोजन

Correct Answer -

Carbon dioxide काबन डाइऑाइड

Which of the following kind of settlement pattern is found at the confluence of rivers

ि ि ि ि ो

78)

79)

80)

िनिलखत म स िकस कार का वथापन पटन निदयो क सगम पर पाया जाता ह

1 Triangular Paern िकोणीय पटन

2 Circular or Semi-Circular Paern परप या अध-परप पटन

3 Nebular Paern नबलर पटन

4 Star ndashShaped Paern ार-आकार का पटन

Correct Answer -

Triangular Paern िकोणीय पटन

Which one was not the objective of the Biosphere Reserve Projects launched by the UNESCO

यनो ारा श की गई सरित जवमडल परयोजनाओ का उ इनम स कौन सा नही था

1 To promote teaching and research िशण और अनसधान को बढ़ावा दना

2 To make agriculture sustainable किष को दीघकािलक बनाना

3 To conserve ecosystems पारथितक त को सरित करना

4 To conserve genetic diversity for a longtime लब समय तक अनवािशक िविवधता को सरित करना

Correct Answer -

To make agriculture sustainable किष को दीघकािलक बनाना

Which region of the Earth surface receives the highest amount of insulation

पी सतह का कौन सा तापावरोधन की उतम माा ा करता ह

1 Land mass थलखड

2 Savannah region सवाना

3 Water bodies जल िनकाय

4 Tropical desert उकिटबधीय रिगान

Correct Answer -

Tropical desert उकिटबधीय रिगान

Which one of the following is not a biodiversity hotspot

िनिलखत म स कौन सा जव िविवधता का म जगह नही ह

1 Eastern Himalaya पव िहमालय

2 Eastern Ghats पव घाट

81)

82)

83)

3 Indo-Myanmar भारत-ामार

4 Westerm Ghats पिमी घाट

Correct Answer -

Eastern Ghats पव घाट

Which one of the following is NOT a part of the World Network of Biosphere Reserves based on theUNESCO Man and Biosphere Programme

यनो मन और बायोीयर कायम क आधार पर िनिलखत म स कौन बायोीयर रजव क िव नटवक कािहा नही ह

1 Gulf of Mannar मार की खाड़ी

2 Seshachalam शषाचलम

3 Sunderban सदरबन

4 Nilgiri नीलिगर

Correct Answer -

Seshachalam शषाचलम

Which one of the following is an example of ldquodesert vegetationrdquo

िनिलखत म स कौन मथलीय वनित का एक उदाहरण ह

1 Mosses and lichens दलदल और शवाल

2 Temperate grassland समशीतो घास क मदान

3 Coniferous forest शकधारी वन

4 Acacia and cactus एकािसया और कस

Correct Answer -

Acacia and cactus एकािसया और कस

Which one of the following reflects more sunlight िनिलखत म स कौन सा सय की रोशनी को अिधकपरावितत करता ह

1 Paddy crop land धान फसल भिम

2 Land covered with fresh snow ताजा बफ स आािदत भिम

3 Sand desert रतीली रिगान

4 Prairie land यरी भिम

84)

85)

86)

87)

Correct Answer -

Land covered with fresh snow ताजा बफ स आािदत भिम

Which layer of the atmosphere is in contact with the surface of the earthrsquos oceans

वायमडल की कौन सी परत पी क महासागरो की सतह क सपक म ह

1 Stratosphere समताप मडल

2 Mesosphere म मडल

3 Hydrosphere जलमडल

4 Troposphere ोभ मडल

Correct Answer -

Troposphere ोभ मडल

Mediterranean Sea is a border of which of the following countries भम सागर िनिलखत दशो म सिकसकी सीमा ह

1 None of these इनम स कोई नही

2 Iraq इराक

3 Lebanon लबनान

4 Jordan जॉडन

Correct Answer -

Lebanon लबनान

Benguela ocean currents are found along which coast बगएला महासागर धाराए िकस तट क साथ पायीजाती ह

1 East Coast of South America दिण अमरका क पव तट

2 East Coast of Africa अीका क पव तट

3 West Coast of South America दिण अमरका क पिमी तट

4 West Coast of Africa अीका क पिमी तट

Correct Answer -

West Coast of Africa अीका क पिमी तट

88)

89)

90)

Due to tension a block of land on one side being pushed up or upthrown relative to the downthrown blockis referred as

तनाव क कारण नीच फ क ए खड क साप भिम का एक खड एक ओर स ऊपर धकला जाता ह या ऊपर की ओरफ का जाता ह यह _____ क प म सदिभत ह

1 Thrust fault प श

2 Normal fault सामा श

3 Reverse fault म श

4 Strike slip fault नितलब सपण श

Correct Answer -

Normal fault सामा श

Inter-tropical doldrums is a zone of ______ अतर-उकिटबधीय डोलड ______ का एक ह

1 Frontolysis टोलायिसस

2 Convergence अिभसरण

3 Inter-tropical divergence zone अतर-उकिटबधीय िवचलन

4 Local wind थानीय वाय

Correct Answer -

Convergence अिभसरण

The Horse Latitudes are regions located at about _____ north and south of the equator

हॉस अाश भम रखा क उर और दिण म लगभग _____ पर थत ह

1 30ndash60 degree Latitude 30-60 िडी अाश

2 0ndash5 degree Latitude 0-5 िडी अाश

3 30 degree Latitude 30 िडी अाश

4 60ndash90 degree Latitude 60-90 िडी अाश

Correct Answer -

30 degree Latitude 30 िडी अाश

Generally evaporation is high over which part of the Earth

आम तौर पर पी क िकस भाग पर वाीकरण अिधक होता ह

1 Equatorial maritime भमवत समीय ी ी

91)

92)

2 Equatorial continental भमवत महाीपीय

3 Polar maritime वीय समीय

4 Polar continental वीय महाीपीय

Correct Answer -

Equatorial maritime भमवत समीय

A very high temperature during summer in north western India leads to what type of climaticcondition in south

उर पिमी भारत म गम क दौरान बत अिधक तापमान होन क कारण दिण म िकस कार की जलवाय थितउ करता ह

1 Depression over arabian sea अरब सागर पर अवनमन

2 Failure monsoon मानसन िवफलता

3 Successful monsoon मानसन सफलता

4 Cyclones चवात

Correct Answer -

Successful monsoon मानसन सफलता

Lightning and thunder are the resultant effect when तिड़त और गजन परणामी भाव ह जब

1 Two massive clouds hit powerfully each other first lightning is produced and later sound is produced

दो बड़ बादल एक दसर स शशाली ढग स टकरात ह पहल आकाशीय िवदयत उ होता ह और बाद म िन उहोती ह

2 Two massive clouds come into contact with the powerful wind collision this results into first sound and thenlightning

दो बड़ बादल शशाली पवन सघ क सपक म आत ह इसका परणामप पहल िन और िफर आकाशीय िवदयतउ होता ह

3 None of the above उपरो म स कोई भी नही

4 A high density cloud contains positively and negatively charged electric ions and when this interacts light andsound are simultaneously produced

एक उ घन बादल म धनाक और ऋणाक आविशत िवदयत आयन होत ह और जब यह परर भाव डालत ह तोकाश और िन एक साथ उािदत होती ह

Correct Answer -

A high density cloud contains positively and negatively charged electric ions and when this interacts light andsound are simultaneously produced

औ ि ि ो औ ो

93)

94)

95)

एक उ घन बादल म धनाक और ऋणाक आविशत िवदयत आयन होत ह और जब यह परर भाव डालत ह तोकाश और िन एक साथ उािदत होती ह

Doon Valley is able to grow rice because दन घाटी चावल उगान म सम ह ोिक

1 Other crops cannot be grown वहा अ फसलो को उगाया नही जा सकता ह

2 People in the valley are rice eaters घाटी म लोग चावल खान वाल ह

3 There is a huge export demand of rice वहा चावल की भारी िनयात माग ह

4 It has warm summer and snow melt waters for irrigation

वहा गिमया गम होती ह िसचाई क िलए बफ का िपघला आ पानी होता ह

Correct Answer -

It has warm summer and snow melt waters for irrigation

वहा गिमया गम होती ह िसचाई क िलए बफ का िपघला आ पानी होता ह

CANCELLED

In the geological time scale the Mesozoic Era DOES NOT contains which of the following periods

भगभय समय पमान पर मजीवी यग म िन कालो म स कौन नही ह

1 Triassic ट ाइऐिसक

2 Jurassic जरिसक

3 Cretaceous चाकमय

4 Carboniferous काबनी

Correct Answer -

Carboniferous काबनी

96)

1 P-3 Q-4 R-2 S-1

2 P-3 Q-4 R-1 S-2

3 P-3 Q-4 R-1 S-2

4 P-4 Q-3 R-2 S-1

Correct Answer -

P-4 Q-3 R-2 S-1

1 P-3 Q-1 R-4 S-2

2 P-3 Q-4 R-1 S-2

3 P-3 Q-2 R-4 S-1

97)

98)

4 P-2 Q-1 R-4 S-3

Correct Answer -

P-3 Q-4 R-1 S-2

1 P-3 Q-1 R-4 S-2

2 P-2 Q-3 R-4 S-1

3 P-2 Q-1 R-3 S-4

4 P-4 Q-2 R-1 S-3

Correct Answer -

P-3 Q-1 R-4 S-2

99)

100)

1 P-3 Q-2 R-4 S-1

2 P-1 Q-2 R-3 S-4

3 P-2 Q-3 R-1 S-4

4 P-4 Q-3 R-2 S-1

Correct Answer -

P-2 Q-3 R-1 S-4

ldquoHuman geography is the study of changing relationship between the unresting man and the unstableearthrdquo was defined by

lsquolsquoमानव भगोल ाकल आदमी और अथर पी क बीच सबध परवतन का अयन हrdquo ______ ारा परभािषत िकया गयाथा

1 J Brunches ज चस

2 EC Semple ईसी सल

3 HJ Mackinder एच ज मिकदर

4 PV Blache पीवी च

Correct Answer -

EC Semple ईसी सल

Sedimentary rocks are finally and ultimately derived from the____________

अवसादी चान अततः ________ स ा की जाती ह

1 action of earth movements पी की गितिविधयो

2 marine deposit समी िनप

3 weathering of metamorphic rocks पातरत चानो क अपय

4 weathering of igneous rocks आय चानो क अपय

Correct Answer -

weathering of igneous rocks आय चानो क अपय

Page 6: High School Teacher Eligibility Test- BOARD PROFESSIONAL ...peb.mp.gov.in/results/RESULT_18/HST_RES18/Final_anwser_key/HST… · M a ndl a / मंड ल ... Under the Madhya Pradesh

7)

8)

1)

Out of the following options choose the most appropriate usage to fill in the blank

I live a few yards ______ the bus stop

1 off

2 from

3 outside

4 away

Correct Answer -

from

Arrange the sentences P Q R and S to form a logical sequence between sentences 1 and 6

1 Jurors do not have to be intelligentP In 1933 in a sheep-stealing case in Wales two of the jurors were totally unable to understand Englishbut no one found that out until after the trialQ There is no test no examination and no enquiry about their abilitiesR The convicted found this out and complained about it to the Court of Criminal AppealS No one even takes the trouble to find out whether they can understand the English language6 Still the Court of Appeal felt that it must refuse to interfere

1 QPRS

2 SQRP

3 PRQS

4 QSPR

Correct Answer -

QSPR

Topic- GENERAL KNOWLEDGE

Which country will host the 2022 Football World Cup 2022 फटबॉल िव कप की मजबानी करन वाला दशकौन होगा

1 America अमरका

2 Canada कनाडा

3 Qatar कतर

4 Mexico मको

2)

3)

4)

5)

Correct Answer -

Qatar कतर

According to sex ratio trends of Census 2011 which district of Madhya Pradesh has been ranked numberOne

जनगणना 2011 म िलग अनपात क झानो क अनसार म दश क िकस िजल को नबर एक थान िदया गया ह

1 Seoni िसओनी

2 Ratlam रतलाम

3 Balaghat बालाघाट

4 Mandla मडला

Correct Answer -

Balaghat बालाघाट

The folk dance form called lsquoMatkirsquo is prevalent in which of the following regions of Madhya Pradesh

मटकी नामक लोक न प म यदश क िनिलखत म स िकस म चिलत ह

1 Bundelkhand बदलखड

2 Malwa मालवा

3 Vindhyachal िवाचल

4 Baghelkhand बघलख

Correct Answer -

Malwa मालवा

Who appoints the Chairman and the Members of the Union Public Service Commission

सघ लोक सवा आयोग क अ और सदो की िनय कौन करता ह

1 Chief Justice of India भारत क म य यायधीश

2 Prime Minister धान मी

3 President रा ट पित

4 Chief Election Commissioner म य चनाव आय त

Correct Answer -

President रा ट पित

6)

7)

8)

Under the Madhya Pradesh Ladli Laxmi Yojana the beneficiary is paid the total amount on attaining theage of ___ मदश लाडली ली योजना क तहत लाभाथ की आय ___ वष परी होन पर कल रािश का भगतान िकयाजाता ह

1 18

2 21

3 23

4 16

Correct Answer -

21

India has won the gold medal in hockey ______times at the Olympics

ओलिपक खलो म भारत न हॉकी म ण पदक _____ बार जीता ह

1 9

2 7

3 8

4 6

Correct Answer -

8

Where is the Madhya Pradesh Tribal Museum located at

म दश जनजातीय सहालय कहा थत ह

1 Indore इदौर

2 Jhabua झबआ

3 Khargone खरगोन

4 Bhopal भोपाल

Correct Answer -

Bhopal भोपाल

The fictional boy character of the lsquoThe Jungle Bookrsquo by author Rudyard Kipling was

लखक डयाड िकपिलग ारा lsquoद जगल बकrsquo का कािनक बालक चर था

1 Kaa का

2 Mowgli मोगली

9)

10)

1)

3 Shere Khan शर खान

4 Bagheera बघीरा

Correct Answer -

Mowgli मोगली

The recipient of which one of the following awards given by Madhya Pradesh Government gets the highest cashprize

िनिलखत परारो म स िकसक ाकता को म दश सरकार ारा उतम नकद परार िदया जाता ह

1 Mahatma Gandhi award महाा गाधी परार

2 Shikar Award िशखर परार

3 Sharad Joshi Award शरद जोशी परार

4 Kishore Kumar Award िकशोर कमार परार

Correct Answer -

Mahatma Gandhi award महाा गाधी परार

Who among the following was the Trinidadian writer of Indian descent

िनिलखत म स भारतीय मल क ििनदािदयन लखक कौन थ

1 RK Narayan आर क नारायण

2 Anita Desai अनीता दसाई

3 Rudyard Kipling डयाड िकपिलग

4 VS Naipaul वीएस नायपॉल

Correct Answer -

VS Naipaul वीएस नायपॉल

Topic- GENERAL REASONING

Find the missing number ल त स या ात कर

4 196 16 169 ____ 144 64

1 81

2 36

3 32

4 21

2)

3)

4)

Correct Answer -

36

Find the average औसत ात कर

25 35 45 55 65 amp 75

1 48

2 50

3 47

4 49

Correct Answer -

50

The below series uses a sequence of alphabets and numbers Identify the incorrect combination

नीच दी गई खला अरो एव साओ का अनम योग करती ह गलत सयोजन पहचान

(i) FT85DF77ER

(ii) FT85DF77ER

(iii) FT85DE77ER

(iv) FT85DF77ER

1 ii

2 iv

3 i

4 iii

Correct Answer -

iii

A and B together can complete a piece of work in 10 days and B alone can complete the same work in 20days In how many days can A alone complete the work

A और B एक साथ काम क एक भाग को 10 िदनो म समा त करत ह और B अकल उसी काम को 20 िदनो म समा त करता हिकतन िदनो म A अकल उस काम को समा त कर सकता ह

1 20 days 20 िदन

2 30 days 30 िदन

3 10 days 10 िदन

4 25 days 25 िदन

5)

6)

Correct Answer -

20 days 20 िदन

Which one of the following four addresses is NOT EXACTLY same as the one given below

िनिलखत िदए गए चार पतो म स नीच िदए गए पत क ठीक समान कौन सा एक नही ह

Ground Floor 59 Goulburn St

Sydney NSW 2000 Australia

+61 (02) 8987 3700

(i) Ground Floor 59 Goulburn St

Sydney NSW 2000 Australia

+61 (02) 8987 3700

(ii) Ground Floor 59 Goulburn St

Sydney NSW 2000 Australia

+61 (02) 8987 3700

(iii) Ground Floor 59 Govlburn St

Sqdney NSW 2000 Australia

+61 (02) 8987 3700

(iv) Ground Floor 59 Goulburn St

Sydney NSW 2000 Australia

+61 (02) 8987 3700

1 ii

2 iv

3 i

4 iii

Correct Answer -

iii

In a group of 75 people 32 of them like cold drink 56 of them like general water and each person likesat least one of the two drinks How many people like both

75 लोगो क समह म उनम स 32 को को िड क पसद ह उनम स 56 को सामा जल पसद ह और क को दो म स कम स कम एक पय पसद ह िकतन लोगो को दोनो पसद ह

1 11

2 13

7)

1)

2)

3 12

4 14

Correct Answer -

13

If A = 1 FAT = 27 then FAITH =

यिद A = 1 FAT = 27 तो FAITH =

1 41

2 40

3 42

4 44

Correct Answer -

44

Topic- PEDAGOGY

CANCELLED

Basic education became a major initiative because of the efforts of

ाथिमक िशा िन न क यास स एक मह वपण कायम बन गई

1 Dr Radhakrishnan डॉ राधाक णन

2 Dr Rajendra Prasad डॉ राज साद

3 Mahatma Gandhi महा मा गाधी

4 Rabindranath Tagore रबी नाथ टगोर

Correct Answer -

Mahatma Gandhi महा मा गाधी

The ldquoclause of phraserdquo is an unit of perception of लॉज ऑफ ज िन न क धारणा क एक इकाई ह

1 Image छिव

2 Concept अवधारणा

3 Language भाषा

4 Thought िवचार

Correct Answer -

3)

4)

5)

6)

Language भाषा

Which method is most suitable to study communication process among students

छाो क बीच सचार िया का अयन करन क िलए कौन सी िविध सबस उपय ह

1 Case Study मामल का अयन (कस टडी)

2 Systematic Observation वथत अवलोकन

3 Experimental Method योगाक िविध

4 Introspection आ-िनरीण (इट ोस शन)

Correct Answer -

Systematic Observation वथत अवलोकन

Genes in a human being are located in मानवो म जीन इनम थत होता ह

1 cytoplasm कोिशका (साइटो ला म)

2 ribosomes राइबोसोम

3 cell membranes कोिशका िझी

4 chromosomes गणस (ोमोसोम)

Correct Answer -

chromosomes गणस (ोमोसोम)

With smaller classes teachers are much more able to ____________

छोटी काओ क साथ िशक ____________ म अिधक सम होत ह

1 Go slow while teaching िशण क दौरान धीमी गित स जान

2 Narrate more personal experiences अिधक गत अनभवो को बतान

3 Adapt instruction to individual differences among students छाो क बीच गत मतभदो क िलए अनकल िनदश दन म

4 Make use of the extra space for extra-curricular activities पातर गितिविधयो क िलए अितर जगह का उपयोग करन

Correct Answer -

Adapt instruction to individual differences among students छाो क बीच गत मतभदो क िलए अनकल िनदश दन म

While engaging in a task the child gets bored This is a sign of

एक काम म होन पर बा ऊब जाता ह यह िन का सकत ह

1 the task requiring a professional approach काय को ावसाियक िकोण की आवकता ह

7)

8)

9)

2 the task becoming mechanically repetitive काय यािक प स दोहराव वाला ह

3 the child not being intelligent बा बमान नही ह

4 the child being incapable of learning बा सीखन म असमथ ह

Correct Answer -

the task becoming mechanically repetitive काय यािक प स दोहराव वाला ह

Educational Psychologists are more concerned with the learning in __________

शिणक मनोवािनक __________ म अिधगम क साथ अिधक िचितत होत ह

1 Formal environment औपचारक वातावरण

2 Informal environment अनौपचारक वातावरण

3 Physical environment भौितक वातावरण

4 Social environment सामािजक वातावरण

Correct Answer -

Formal environment औपचारक वातावरण

Special needs education is the type of education िवशष ज़रतो वाली िशा वह िशा होती ह जो

1 Given to person with disabilities अम य को दी जाती ह

2 Given to people from remote areas दर थ ो क लोगो को दी जाती ह

3 Provided to intelligent people बमान लोगो को दी जाती ह

4 Established by colonial masters औपिनविशक मखयाओ ारा थािपत की गई ह

Correct Answer -

Given to person with disabilities अम य को दी जाती ह

The Stanford-Binet scale of intelligence was first published in the year

ब क नफोड-िबनट कल को इस वष म पहली बार कािशत िकया गया था

1 1916

2 1903

3 1908

4 1900

Correct Answer -

1916

10)

11)

12)

13)

The term lsquofictional finalismrsquo was propounded by

पद lsquoकत योजनवादrsquo (िफ शनल फाइनिल म) इनक ारा ितपािदत िकया गया था

1 Skinner नर

2 Freud ायड

3 Adler एडलर

4 Pavlov पावलोव

Correct Answer -

Adler एडलर

Rational Emotive Behavior Therapy was propounded by

तक सगत भावनाक वहार थरपी िन क ारा ितपािदत की गई थी

1 Carl Jung काल यग

2 Carl Rogers काल रोजस

3 Aaron Beck आरोन बक

4 Albert Ellis अट एिलस

Correct Answer -

Albert Ellis अट एिलस

Which of the following indicates the quality of education in a school

िनिलखत म स या एक कल म िशा की गणव ता का सकतक ह

1 Text-books and Teaching-learning material पा-प तक तथा िशण व सीखन स सबिधत सामी

2 Infrastructural facilities at the school कल म आधारभत ढाच स सबिधत सिवधाए

3 Student achievement level िवािथयो का उपल तर

4 Classroom systems का की यव था

Correct Answer -

Student achievement level िवािथयो का उपल तर

Which of the following teachers can be identified with authoritarian teaching style

िनिलखत म स कौन सा िशक अिधकारवादी िशण शली क साथ पहचाना जा सकता ह

1 Laissez-faire teacher अब ध िशक

14)

15)

2 Democratic teacher लोकतीय िशक

3 Indifferent teacher िन प िशक

4 Direct instruction teacher िनदश िशक

Correct Answer -

Direct instruction teacher िनदश िशक

Who was the pioneer of classical conditioning

िचरितित ानकलन ( लािसकल कडीशिनग) क वतक कौन थ

1 Skinner नर

2 Pavlov पावलोव

3 Kohler कोहलर

4 Freud ायड

Correct Answer -

Pavlov पावलोव

Which of the following are true with reference to short term memory

1 Limited capacity

2 Brief storage of information

3 Unlimited capacity

4 Duration of storage less than twenty seconds

अ पकािलक मित क सदभ म िन न म स या स य ह

1 सीिमत मता

2 सचना का सि त भडारण

3 असीिमत मता

4 बीस सकड स कम भडारण की अविध

1 3 and 4 3 और 4

2 2 and 4 2 और 4

3 13 and 4 13 और 4

4 12 and 4 12 और 4

Correct Answer -

12 and 4 12 और 4

1)

2)

3)

4)

Topic- GEOGRAPHY

Which of the following ranges of population size is used to define Class-III city by Indian Census

भारतीय जनगणना ारा वग-III शहर को परभािषत करन क िलए िनिलखत म स िकस रज क जनसा आकारका उपयोग िकया जाता ह

1 20 000 to 49999 20 000 स 49999

2 30000 to 59999 30000 स 59999

3 24000 to 54999 24000 स 54999

4 50000 to 99999 50000 स 99999

Correct Answer -

20 000 to 49999 20 000 स 49999

Carbonaceous rocks which produce coal and oil belong to the category of rocks called_______

कोयल और तल का उादन करन वाली काबनय चान ______ नामक चानो की णी स सबिधत ह

1 metamorphic पातरत

2 sedimentary अवसादी

3 inorganic अजिवक

4 igneous आय

Correct Answer -

sedimentary अवसादी

The ruhr-complex is a major industrial centre in र-परसर िन का एक मख औोिगक क ह

1 North America उरी अमरका

2 Russia स

3 Germany जमनी

4 Europe यरोप

Correct Answer -

Germany जमनी

The term lsquoRegurrsquo refers to श lsquoरगरrsquo ______ स सबिधत ह

1 Deltaic alluvial soils डा जलोढ़ िमी

ि ी

5)

6)

2 Laterite soils लटराइट िमी

3 Red and yellow soils लाल और पीली िमी

4 Black cotton soils काली कपास िमी

Correct Answer -

Black cotton soils काली कपास िमी

Read the given statements and answer which of the following options isare correct

1 Lower the pressure greater the atmospheric disturbance

2 Air move from higher to low pressure

िदए गए कथन को पढ़ और उर द िक िन म स कौन सास िवक सही ह

1 िजतना दाब कम होगा वायमडलीय बाधाए उतनी अिधक होगी

2 वाय उ स िन दाब की ओर गित करती ह

1 Both Statements 1 and 2 are correct दोनो कथन 1 और 2 सही ह

2 Both Statements 1 and 2 are wrong दोनो कथन 1 और 2 गलत ह

3 Statement 1 is wrong and only Statement 2 is correct कथन 1 गलत ह और कवल कथन 2 सही ह

4 Statement 1 is correct and Statement 2 is wrong कथन 1 सही ह और कथन 2 गलत ह

Correct Answer -

Both Statements 1 and 2 are correct दोनो कथन 1 और 2 सही ह

CANCELLED

1 4 1 2 and 3 4 1 2 और 3

2 4 1 3 and 2 4 1 3 और 2

3 1 4 2 and 3 1 4 2 और 3

4 1 4 3 and 2 1 4 3 और 2

Correct Answer -

7)

8)

9)

1 4 3 and 2 1 4 3 और 2

CANCELLED

Karl Pearsonrsquos correlation co-efficient is काल िपयसन का सहसबध गणाक ह

1 Arithmec mean समार मा

2 Geometric mean गणोर मा

3 None of these इनम स कोई नही

4 Harmonic mean हराक मा

Correct Answer -

Geometric mean गणोर मा

CANCELLED

ldquoEach day is more or less the same the morning is clear and bright with a sea breeze as the Sun climbshigh in the sky heat mounts up dark clouds form then rain comes with thunder lighting But rain is soonoverrdquo Which of the following regions is described in the above passage

ldquoक िदन समान स अिधक या कम होता ह सम की हवा क साथ और उल सबह होती ह जस सयआकाश म ऊचा चढ़ता ह गम बढ़ जाती ह काल बादल बनत ह िफर िबजली क साथ बारश आती ह लिकनबारश जी ख हो जाती हlsquorsquo उपरो पा म िनिलखत म स िकन ो का वणन िकया गया ह

1 Equatorial भमरखीय

2 Equatorial भमरखीय

3 Savannah सवाना

4 Mediterranean आातरक (भमसागरीय)

5 Mediterranean आातरक (भमसागरीय)

6 Monsoon मानसन

7 Monsoon मानसन

Correct Answer -

Equatorial भमरखीय

Equatorial भमरखीय

CANCELLED

In which epoch of the geological history of the Earth dinosaurs reached their largest size

पी क भगभय इितहास क िकस यग म डायनासोर अपन सबस बड़ आकार तक पचि

10)

11)

1 Triassic ट ायिसक

2 Jurassic जरािसक

3 Cretaceous ीटशस

4 Permian पिमयन

Correct Answer -

Jurassic जरािसक

CANCELLED

A spring tide would occur in which of the following conditions

िनिलखत म स िकन थितयो म ार-भाटा आता ह

1 When the Sun Moon and Earth are in a straight line

जब सय चमा और पी एक सीधी रखा म होत ह

2 When the Moon and Earth are in right angle to each other

जब चमा और पी एक-दसर क दािहन कोण म होत ह

3 When the Earth and Moon are in right angle to the Sun

जब पी और चमा सय क दािहन कोण म होत ह

4 When the Sun and Moon are in right angle to each other

जब सय और चमा एक-दसर क दािहन कोण म होत ह

Correct Answer -

When the Sun Moon and Earth are in a straight line

जब सय चमा और पी एक सीधी रखा म होत ह

CANCELLED

An observe on the Earthrsquos surface always sees the same face of the moon because

एक पयवक को पी की सतह स हमशा चाद का एक ही फलक िदखाई दता ह ोिक

1 Its path of revolution around the earth is the same as that of the earth around the Sun

इसका पी क चारो ओर परमण का माग सय क चारो ओर पी क समान ही ह

2 Its period of revolution around the Earth is the same as its period of rotation around its own axis

इसकी पी क चारो ओर परमण की अविध उसकी अपनी धरी क चारो ओर घणन की अविध क समान ह

3 Its period of rotation is the same as that of the Earth इसकी घणन की अविध पी क समान ह

ी ि ी ी

12)

13)

14)

4 Its direct of rotation is the same as that of Earth घणन की िदशा पी क समान ही ह

Correct Answer -

Its period of revolution around the Earth is the same as its period of rotation around its own axis

इसकी पी क चारो ओर परमण की अविध उसकी अपनी धरी क चारो ओर घणन की अविध क समान ह

CANCELLED

The pebbles that are faceted by the sand-blasting and shaped polished by the wind abrasions are known as

पवन अपघषन ारा पॉिलश रत-िवोिटत और साच म ढला ककड़ ___________ क प म जाना जाता ह

1 Dreikanter िकोणक

2 Pediments िकोिनका

3 Inselberg इलबग

4 Dunes टीबा

Correct Answer -

Dreikanter िकोणक

CANCELLED

Astronomical unit is the average distance between खगोलीय इकाई ______ क बीच की औसत दरी ह

1 Earth and Mars पी और मगल

2 Earth and mercury पी और बध

3 Earth and moon पी और चमा

4 Earth and Sun पी और सय

Correct Answer -

Earth and Sun पी और सय

During cold weather season in the northern plains there will be an inflow of cyclonic disturbancesfrom the _________ directions

शीत मौसम क दौरान उरी मदानी इलाको म _________ िदशाओ स चवात सबधी गड़बड़ी का अतवाह होगा

1 East and Northwest पव और उरपिम

2 East and Northeast पव और पवर

3 West and East पिम और पव

ि औ ि

15)

16)

17)

4 West and Northwest पिम और उरपिम

Correct Answer -

West and Northwest पिम और उरपिम

During an earth quake the velocity of the body waves will________ along with the increase in densityof the material it is passing through

भकप क दौरान लहरो क ऊपरी भाग का वग घन म व क साथ-साथ ________ जो इसस गजरन वाली वको आग बढाएगी

1 not change नही बदलगा

2 increase initially and then decrease शआत म बढ़गा और िफर घटगा

3 increase बढ़गा

4 decrease घटगा

Correct Answer -

increase बढ़गा

The Clouded Leopard National park is situated in which of the following states

िनिलखत म िकस रा म धिमल तदआ रा ीय उान (ाउडड लपड नशनल पाक ) थत ह

1 Tripura िपरा

2 Uttar Pradesh उर दश

3 Assam असम

4 Mizoram िमजोरम

Correct Answer -

Tripura िपरा

Usually the land surfaces are heated more quickly than the water surfaces because _____________

आम तौर पर जल सतहो की तलना म भिम सतह अिधक तजी स गम होती ह ोिक _____________ ह

1 the specific heat of water is higher than land पानी की िविश ऊा भिम स अिधक

2 the specific heat of water is lesser than land पानी की िविश ऊा भिम स कम होती

3 the latent heat of water is higher than the land पानी की अतिनिहत ऊा भिम स अिधक

4 the land reflects more heat radiation than water भिम पानी की तलना म अिधक ऊा क िविकरण को पराविततकरती

18)

19)

20)

21)

Correct Answer -

the specific heat of water is higher than land पानी की िविश ऊा भिम स अिधक

The longest shore-line is along the state of सबस लबी समतटीय रखा िन रा क साथ ह

1 Maharashtra महारा

2 Orissa उड़ीसा

3 Kerala करल

4 Gujarat गजरात

Correct Answer -

Gujarat गजरात

The position when the Earth is farthest from the Sun is known as

जब पी सय स सबस दर होती ह तो उस थित को िन नाम स जाना जाता ह

1 Perihelion उपसौर

2 Vernal Equinox बसत िवषव

3 Aphelion अपसौर

4 Autumnal Equinox शराल िवषव

Correct Answer -

Aphelion अपसौर

The seasonal reversal of winds is the typical characteristic of

हवाओ का मौसमी परवतन ______ की सामा िवशषता ह

1 Mediterranean climates only कवल भमसागरीय जलवाय

2 All of the above climates उपय सभी मौसम

3 Monsoon climate only कवल मानसन जलवाय

4 Equatorial climate only कवल भमरखीय जलवाय

Correct Answer -

Monsoon climate only कवल मानसन जलवाय

In _________ rocks the minerals will occurs in beds or layers

ो ि ो ो

22)

23)

24)

______ चानो म खिनज तल या परतो म होत ह

1 metamorphic कायातरत

2 igneous and metamorphic आय और कायातरत

3 igneous आय

4 sedimentary अवसादी

Correct Answer -

sedimentary अवसादी

Black soil is ideal for the cultivation of cotton as कपास की खती क िलए काली िमी आदश ह ोिक

1 Its colour is black यह काली होती ह

2 It is found on plateau regions यह पठार ो म पायी जाती ह

3 It is made up of lava यह लावा स बनी होती ह

4 It can retain moisture यह नमी को बरकरार रख सकती ह

Correct Answer -

It can retain moisture यह नमी को बरकरार रख सकती ह

The National Survey and Mapping Organization of the country works under the Department of___________

दश का रा ीय सवण और मानिचण सगठन ___________ िवभाग क अतगत काय करता ह

1 Space अतर

2 Science and Technology िवान और तकनीक

3 Culture सित

4 Tourism पयटन

Correct Answer -

Science and Technology िवान और तकनीक

Palk strait separates India from पाक जलडमम भारत स _____ को अलग करता ह

1 Pakistan पािकान

2 Andaman Island अडमान ीप

3 China चीन

25)

26)

27)

4 Sri Lanka ीलका

Correct Answer -

Sri Lanka ीलका

Which among the following state is the major producer of Bauxite in India

िनिलखत म स कौन सा रा भारत म बॉाइट का मख उादक ह

1 Madhya Pradesh मदश

2 Rajasthan राजथान

3 Goa गोवा

4 Orissa उड़ीसा

Correct Answer -

Orissa उड़ीसा

Which of the following states DOES NOT share border with Chhattisgarh

िनिलखत म स कौन सा रा छीसगढ़ क साथ सीमा साझा नही करता ह

1 Telangana तलगाना

2 Uttar Pradesh उर दश

3 Bihar िबहार

4 Andhra Pradesh आ दश

Correct Answer -

Bihar िबहार

Which of the following statements is INCORRECT with respect to parallels of latitudes

अाश क समानातरो क सबध म िन निलखत म स कौन सा कथन गलत ह

1 A line joining places of equal latitude is known as parallel of largest

समान अाश क थानो को जोड़न वाली रखा को िवशालतम क समानातर क प म जाना जाता ह

2 They stat from equator and run parallels to it

व भम रखा स ारभ होत ह और इसक समानातर चलत ह

3 All parallels are equal in length सभी समातर लबाई म समान ह

4 All parallels are drawn as circles on the globe ोब पर सभी समानातर वो क प म खीच जात ह

28)

29)

30)

31)

Correct Answer -

All parallels are equal in length सभी समातर लबाई म समान ह

Which of the following Indian states is also known as a lsquoLand of Red river and Blue Hillsrsquo

िनिलखत म स िकस भारतीय रा को लाल नदी और नीली पहािड़यो की भिम क नाम स जाना जाता ह

1 Uttarkhand उराखड

2 Assam असम

3 Meghalaya मघालय

4 Arunachal Pradesh अणाचल दश

Correct Answer -

Assam असम

In spatial analysis of settlement Rn = 215 indicates which type of settlement arrangement

िनपटान क थािनक िवषण म Rn = 215 यह इिगत करता ह िक िकस कार की िनपटान वथा ह

1 Uniform यिनफॉम

2 Semi-Clustered समी- ल टर

3 Clustered ल टर

4 Random रडम

Correct Answer -

Uniform यिनफॉम

Who are known as the lsquoYellow Peoplersquo lsquoयलो पीपलrsquo क प म कौन जाना जाता ह

1 Mongoloids मोगोलोइडस

2 Nigroids नीोइडस

3 Australoids ऑ लॉइडस

4 Caucasoids कॉकसोइडस

Correct Answer -

Mongoloids मोगोलोइडस

ि ि ो ौ ि

32)

33)

34)

Who publishes the topographical map of India भारत क थलाकितक मानिच को कौन कािशत करता ह

1 Geographical Survey of India भारत का भौगोिलक सवण

2 Government of India भारत सरकार

3 Geological Survey of India भारत क भगभय सवण

4 Survey of India भारत का सवण

Correct Answer -

Survey of India भारत का सवण

Who among the following claimed geography to be the lsquoEcology of Manrsquo

िनिलखत म स िकसन भगोल को मन का पारथितकी कहा ह

1 Alfred Hener अड हटनर

2 Vidal-de la Blache वाइडल-िड लॉ ॉश

3 Oo Schluter ओटो टर

4 Harlan Barrow हरलन बारो

Correct Answer -

Harlan Barrow हरलन बारो

Who among the following is regarded as the founder of humanistic approach in geography

िनिलखत म स िकस भगोल म मानवतावादी िकोण का सथापक माना जाता ह

1 William Bunge िविलयम बग

2 Yi-Fu-Tuan यी-फ- यान

3 Brain JL Berry न जएल बरी

4 Richard Peet रचड पीट

Correct Answer -

Yi-Fu-Tuan यी-फ- यान

Who prepared Lorenz curve लोरज व िकसन तयार िकया

1 Geddes गडस

2 None of these इनम स कोई नही

3 Griffith Taylor ििफथ टलर

35)

36)

37)

4 Max U Lorenz म य लोरज

Correct Answer -

Max U Lorenz म य लोरज

Gulf Streams are the currents of which of the following oceans

खाड़ी की धाराए िनिलखत महासागरो म स िकसकी धाराए ह

1 North Atlantic Ocean उरी अटलािटक महासागर

2 North Pacific Ocean उरी शात महासागर

3 Arabian Sea अरब सागर

4 South Pacific Ocean दिण शात महासागर

Correct Answer -

North Atlantic Ocean उरी अटलािटक महासागर

Disintegration wearing away and removal of rock material is generally referred as

िशला पदाथ (रॉक सामी) का टटना िमटना और हटना आमतौर पर ________ क प म सदिभत िकया जाता ह

1 Shattering िवसकारक

2 Denudation अनाादन

3 Fault श

4 Decomposition िवयोजन

Correct Answer -

Denudation अनाादन

Variations in the length of day time and night from season to season are due to

मौसम स मौसम परवतन पर िदन क समय और रात क समय की अविध म िभताए िन कारण स होती ह

1 The Earthrsquos revolution round the Sun in an elliptical manner पी का दीघवाकार तरीक स सय क चारो घणन

2 The Earthrsquos rotation on its axis पी का इसकी धरी पर घणन

3 Revolution of the Earth on a tilted axis नत अ पर पी का घणन

4 Latitudinal position of the place थान की अाश थित

Correct Answer -

Revolution of the Earth on a tilted axis नत अ पर पी का घणन

38)

39)

40)

Point out the correct sequence of mountain ranges from north to south

उर स दिण तक पवत खलाओ क सही अनम को इिगत कर

1 Great Himalaya Middle Himalaya Outer Himalaya Trans Himalaya

महान िहमालय म िहमालय बा िहमालय परा िहमालय

2 Middle Himalaya Great Himalaya Trans Himalaya Outer Himalaya

म िहमालय महान िहमालय परा िहमालय बा िहमालय

3 Outer Himalaya Middle Himalaya Great Himalaya Trans Himalaya

बा िहमालय म िहमालय महान िहमालय परा िहमालय

4 Trans Himalaya Great Himalaya Middle Himalaya Outer Himalaya

परा िहमालय महान िहमालय म िहमालय बा िहमालय

Correct Answer -

Trans Himalaya Great Himalaya Middle Himalaya Outer Himalaya

परा िहमालय महान िहमालय म िहमालय बा िहमालय

Sunrsquos halo is produced by the refraction of light in सय का भामडल ______ म काश क अपवतन ाराउ होता ह

1 Ice crystals in Cirrro-Cumulus clouds पाभ-कपास मघो क बफ िल

2 Ice crystal in Cirrus clouds पाभमघो क बफ िल

3 Dust particles in Stratus clouds री मघो क धल कण

4 Water vapour in Stratus clouds री मघो क जल वा

Correct Answer -

Ice crystal in Cirrus clouds पाभमघो क बफ िल

Read the given statements and answer which of the following options isare correct

(1) The minerals present in the rocks exposed to atmosphere are not subjected to alteration

(2) Oxidation is one of the processes of chemical weathering

िदए गए कथनो को पढ़ और उर द िक िन म स कौन सास िवक सही ह

(1) वायमल स अनावत शल म उपथत खिनज परवतन क अधीन नही होता ह

(2) ऑीकरण रासायिनक अपय की ियाओ म स एक ह

ो ो

41)

42)

1 Both statements are wrong दोनो कथन गलत ह

2 Both statements are correct दोनो कथन सही ह

3 First statement is wrong and second statement is correct पहला कथन गलत ह और दसरा कथन सही ह

4 First statement is correct and second statement is wrong पहला कथन सही ह और दसरा कथन गलत ह

Correct Answer -

First statement is wrong and second statement is correct पहला कथन गलत ह और दसरा कथन सही ह

Read the given statements and answer which of the following options isare correct

1 Sunrsquos short waves enter the earth partially heating the atmosphere

2 Heated earth surface from the sun produces broader waves which interacts and heats the atmosphere

िदए गए कथन को पढ़ और उर द िक िन म स कौन सास िवक सही ह

1 सय की छोटी तरग पी म आिशक प स वश करती ह और वायमडल को ऊत करती ह

2 सय स ऊत पी की सतह िवारत तरग उ करती ह जो परर भाव डालती ह और वायमडल कोऊत करती ह

1 Both Statements 1 and 2 are correct दोनो कथन 1 और 2 सही ह

2 Both Statements 1 and 2 are wrong दोनो कथन 1 और 2 गलत ह

3 Statement 1 is wrong and only Statement 2 is correct कथन 1 गलत ह और कवल कथन 2 सही ह

4 Only statement 1 is correct कवल कथन 1 सही ह

Correct Answer -

Both Statements 1 and 2 are correct दोनो कथन 1 और 2 सही ह

Read the given statements and answer which of the following options isare correct

(1)The rocks that get changed due to heat and pressure are termed as metamorphic rocks

(2)Slate is one such type of metamorphic rock

िदए गए कथनो को पढ़ और उर द िक िन म स कौन सास िवक सही ह

(1) शल जो ऊा और दाब क कारण परवितत हो जात ह उ कायातरक शलो क प म जाना जाता ह

(2) ट एक तरह का कायातरक शल ह

1 Both statements are wrong दोनो कथन गलत ह

2 Both statements are correct दोनो कथन सही ह

3 First statement is wrong and second statement is correct पहला कथन गलत ह और दसरा कथन सही ह

ी औ

43)

44)

4 First statement is correct and second statement is wrong पहला कथन सही ह और दसरा कथन गलत ह

Correct Answer -

Both statements are correct दोनो कथन सही ह

Read the given statements and answer which of the following options isare correct

1 Higher temperature anomaly is observed in the northern hemisphere

2 Differential heating is absent in Northern Hemisphere

िदए गए कथनो को पढ़ और उर द िक िन म स कौन सास िवक सही ह

1 उरी गोलाध म उ तापमान िवसगित पायी जाती ह

2 उरी गोलाध म अतर ऊन अनपथत होती ह

1 Both Statements 1 and 2 are correct दोनो कथन 1 और 2 सही ह

2 Both Statements 1 and 2 are wrong दोनो कथन 1 और 2 गलत ह

3 Statement 1 is wrong and Statement 2 is correct कथन 1 गलत ह और कथन 2 सही ह

4 Statement 1 is correct and Statement 2 is wrong कथन 1 सही ह और कथन 2 गलत ह

Correct Answer -

Statement 1 is correct and Statement 2 is wrong कथन 1 सही ह और कथन 2 गलत ह

Read the given statements and answer which of the following options isare correct

(1) Plutonic rocks are intrusive type of igneous rocks

(2) It cools very slowly because the surrounding rock serves as insulation around the intrusion of magma

िदए गए कथनो को पढ़ और उर द िक िन म स कौन सास िवक सही ह

(1) िवतलीय शल अतवधी कार क आश शल ह

(2) यह बत धीर-धीर ठडा होता ह ोिक आस-पास क शल मा क अतवधन क चारो ओर रोधन क प म कायकरत ह

1 Both statements are wrong दोनो कथन गलत ह

2 Both statements are correct दोनो कथन सही ह

3 First statement is wrong and second statement is correct पहला कथन गलत ह और दसरा कथन सही ह

4 First statement is correct and second statement is wrong पहला कथन सही ह और दसरा कथन गलत ह

Correct Answer -

Both statements are correct दोनो कथन सही ह

45)

46)

47)

48)

The dust and ash material hurled from the volcanoes are termed as

ालामखी स िनकलन वाली धल और राख सामी को _______ क प म कहा जाता ह

1 Pyroclasc पाइरोाक

2 Hyperclastic हाइपराक

3 Hepiroclastic हिपरोाक

4 Cirroclastic िसरोाक

Correct Answer -

Pyroclasc पाइरोाक

The vertical difference in elevation between a low tide and high tide is referred as

कम ार और उ ार क बीच ऊचाई म लबवत अतर _____ स सदिभत होता ह

1 Tidal slope ारीय ढलान

2 Tidal elevation ारीय उयन

3 Tidal range ारीय परास

4 Tidal height ारीय ऊचाई

Correct Answer -

Tidal range ारीय परास

The maximum biodiversity is found in which of the following regions िनिलखत ो म स अिधकतमजव िविवधता िकसम पायी जाती ह

1 Amazon Basin अमज़न बिसन

2 East Indies ई इडीज

3 Congo Basin कागो बिसन

4 West indies व इडीज

Correct Answer -

Amazon Basin अमज़न बिसन

The cultivation of rice crop produces_______ चावल की फसल की खती ______ का उादन करती ह

1 SO2

49)

50)

51)

2 CH4

3 CFCs

4 CO2

Correct Answer -

CH4

The pressure system with higher pressure at the centre is called__________

क म उ दबाव वाली दबाव णाली को _______ कहा जाता ह

1 front अ

2 depression अवनमन

3 cyclone चवात

4 anti-cyclone ितचवात

Correct Answer -

anti-cyclone ितचवात

The Himalayan region is poor in mineral resources because िहमालयी खिनज ससाधनो म समनही ह ोिक

1 The displacement of rock strata has disturbed the arrangement of rocks and made it complex

शलीय परत क िवथापन न चानो की वथा को अवथत कर िदया ह और इस जिटल बना िदया ह

2 The climate conditions are not suitable for exploitation of minerals

जलवाय की थित खिनजो क दोहन क िलए उपय नही ह

3 The terrain makes explanation of minerals difficult and very costly due to transportation difficulties

भ-भाग परवहन की किठनाइयो क कारण खिनजो का दोहन मल और बत महगा बना दता ह

4 It is made up of crystalline rocks यह िलीय चानो स बना ह

Correct Answer -

The displacement of rock strata has disturbed the arrangement of rocks and made it complex

शलीय परत क िवथापन न चानो की वथा को अवथत कर िदया ह और इस जिटल बना िदया ह

The process through which the moisture is added to the atmosphere by vegetation is termed as

वह िया िजसक माम स वनित ारा वातावरण म नमी िमलायी जाती ह _______ क प म जानी जाती ह

52)

53)

54)

1 Condensation सघनन

2 Evapotranspiration वान-उजन

3 Radiation िविकरण

4 Precipitation वषण

Correct Answer -

Evapotranspiration वान-उजन

The process through which the terrestrial heat is transferred to air by direct contact is termed as

वह िया िजसम सपक ारा थलीय ऊा वाय म थानातरत हो जाती ह ______ क प म जानी जाती ह

1 Conduction चालन

2 Convection सवहन

3 Insolation आतपन

4 Radiation िविकरण

Correct Answer -

Conduction चालन

The largest area under mangroves is in which of the following statesunion territory

मोव क अतगत िनिलखत राोसघ शािसत दशो म स सबस बड़ा कौन सा ह

1 Andaman and Nicobar अमान और िनकोबार

2 Andhra Pradesh आ दश

3 West Bengal पिम बगाल

4 Gujarat गजरात

Correct Answer -

West Bengal पिम बगाल

The longitudinal transverse and surface waves in an earthquake originate from

भकप म दशातर अनथ और सतह तरग यहा उ होती ह

1 The focus on the surface of the Earth पी क सतह पर क -िबद म

2 The focus within the body of the Earth पी क भीतर क -िबद म

3 The epicenter within the body of the Earth पी क भीतर उपरक म

55)

56)

57)

4 The epicenter on the surface of the Earth पी क सतह पर उपरक म

Correct Answer -

The focus within the body of the Earth पी क भीतर क -िबद म

The down slope movement of material due to gravity is called______

गाकषण क कारण पदाथ की अनढाल गित को ______ कहा जाता ह

1 mass movement पदाथ सचलन

2 deposition िनप

3 erosion रण

4 volcanic movement ालामखीय सचलन

Correct Answer -

mass movement पदाथ सचलन

Shimla is cooler than Amritsar although both are on the same latitude This is because

िशमला म अमतसर स अिधक ठड ह हालािक दोनो समान अाश पर ह ऐसा ह ोिक

1 Shimla is at a greater height above sea level than Amritsar अमतसर की तलना म िशमला सम तल स अिधकऊचाई पर ह

2 Shimla is further north िशमला उर की ओर ह

3 Shimla is farther from the equator िशमला भम रखा स आग ह

4 Their longitudes differ उनकी दशातर रखाए िभ ह

Correct Answer -

Shimla is at a greater height above sea level than Amritsar अमतसर की तलना म िशमला सम तल स अिधकऊचाई पर ह

lsquoTempo of Urbanizationrsquo measures which of the following

lsquoशहरीकरण का टपोrsquo िनिलखत म स कौन सा उपाय ह

1 Speed of urbanizaon शहरीकरण की गित

2 None of the above इनम स कोई नही

3 Inequality of urbanizaon शहरीकरण की असमानता

4 Current level of urbanizaon शहरीकरण का वतमान र

Correct Answer -

58)

59)

60)

Speed of urbanizaon शहरीकरण की गित

Out of the following options choose the INCORRECT statement

िनिलखत िवको म स गलत कथन का चयन कर

1 The clear tracts in the equatorial region recover rapidly भम रखा म भभाग तजी स ठीक हो जात ह

2 The stable communities include a redwood forest a pine forest at high elevations

थर समदायो म एक रडवड वन उ ऊचाई पर एक दवदार वन शािमल ह

3 Any ecosystem moves towards maximum biomass and stability to survive

कोई भी पारथितकी त जीिवत रहन क िलए अिधकतम जवसहित और थरता की तरफ असर होता ह

4 Tropical rain forests near equator are stable ecosystems

भम रखा क पास उकिटबधीय वषा वन थर पारथितक त ह

Correct Answer -

The clear tracts in the equatorial region recover rapidly भम रखा म भभाग तजी स ठीक हो जात ह

Seasonal contrasts are maximum in मौसमी िवषमता अिधकतम ह

1 Mid latitudes म अाश म

2 Low attitudes िन अाश म

3 High latitudes उ अाश म

4 Subtropics उपोकिटबधीय म

Correct Answer -

Mid latitudes म अाश म

In India which type of forest among the following occupies the largest area

भारत म िनिलखत म स िकस कार क वन सबस बड़ा फल आािदत करत ह

1 Sub-tropical Dry Evergreen Forest उप उकिटबधीय श सदाबहार वन

2 Mountain Wet Temperate Forest पवतीय आ शीतो वन

3 Tropical Moist Deciduous Forest उकिटबधीय आ पणपाती वन

4 Tropical Wet Evergreen Forest उकिटबधीय आ सदाबहार वन

Correct Answer -

Tropical Moist Deciduous Forest उकिटबधीय आ पणपाती वन

61)

62)

63)

64)

What is the proportion of lsquoJuvenile Populationrsquo (0-14 years) in India as per 2011Census

2011 की जनगणना क अनसार भारत म जवनाइल पॉपलशन यानी िकशोर जनस या (0-14 वष) का अनपात ाह

1 3076 of total population कल जनस या का 3076

2 2764 of total population कल जनस या का 2764

3 2933 of total population कल जनस या का 2933

4 3354 of total population कल जनस या का 3354

Correct Answer -

3076 of total population कल जनस या का 3076

What is the Belfast famous for बलफा िकसक िलए मशर ह

1 Belt of cotton textile industry कपास व उोग क

2 Ship-building industry जहाज िनमाण उोग

3 Agricultural machinery किष उपकरण

4 Aero planes manufacturing वाययान िनमाण

Correct Answer -

Ship-building industry जहाज िनमाण उोग

What is the most important occupation in tropical monsoon lands

उकिटबधीय मॉनसन भिम म सबस महपण वसाय ा ह

1 Mining खनन

2 Cattle rearing मवशी पालन

3 Agriculture किष

4 Nomadic herding नोमािडक जड़ी-बिटया

Correct Answer -

Agriculture किष

What is the most important characteristics of the islands (Indian) located in the Arabian sea

अरब सागर म थत ीपो (भारतीय) की सबस महपण िवशषता ा ह

ी ो

65)

66)

67)

1 There are all of coral origins सभी कोरल मल क ह

2 There are all very small in size य सभी आकार म बत छोट ह

3 They have a very dry climate इनकी जलवाय बत श ह

4 They are extended parts of the mainland व महाीप क िवारत िह ह

Correct Answer -

There are all of coral origins सभी कोरल मल क ह

What do the basalt layers of the Deccan indicate डन की बसा परत ा इिगत करती ह

1 All of the above उपरो सभी

2 Huge volcanic eruptions in the distant past दरथ अतीत म िवशाल ालामखीय िवोट

3 The immense erosional activity of the rivers निदयो की िवशाल रण गितिविध

4 The influence of weathering मौसम का भाव

Correct Answer -

Huge volcanic eruptions in the distant past दरथ अतीत म िवशाल ालामखीय िवोट

In the structure of planet Earth below the mantle the core is mainly made up of_____

पी ह की सरचना म मटल क नीच कोर म प स______ स िनिमत होती ह

1 aluminium एमीिनयम

2 silicon िसिलकॉन

3 chromium ोिमयम

4 iron लोहा

Correct Answer -

iron लोहा

One of the major Mid Oceanic Ridge is found in मख म-महासागर चोिटयो म स एक ______ म पायाजाता ह

1 Mid Pacific Ocean म शात महासागर

2 Mid Atlantic Ocean म अटलािटक महासागर

3 Mid Indian Ocean म भारतीय महासागर

4 Mid Arctic Ocean म आक िटक महासागर

68)

69)

70)

71)

Correct Answer -

Mid Atlantic Ocean म अटलािटक महासागर

Magma that reaches the Earthrsquos surface and then solidifies is called________

मा जो पी की सतह तक पचती ह और िफर ठोस हो जाती ह ________कहलाती ह

1 quartz ाटज

2 lava लावा

3 granite नाइट

4 silicates िसिलकट

Correct Answer -

lava लावा

Isotherms are the lines of equal_______ समताप रखाए समान _______की रखाए होती ह

1 pressure दाब

2 temperature तापमान

3 rainfall वषा

4 height ऊचाई

Correct Answer -

temperature तापमान

Mark the correct sequence of passes in the Western Ghats from north to south

पिमी घाटो म उर स दिण तक दर क सही अनम को िचित कर

1 Thalghat Palghat Bhorghat थलगघाट पालघाट भोरघाट

2 Thalghat Bhorghat Palghat थलघाट भोरघाट पालघाट

3 Bhorghat Thalghat Palghat भोरघाट थलघाट पालघाट

4 Palghat Bhorghat Thalghat पालघाट भोरघाट थलघाट

Correct Answer -

Thalghat Bhorghat Palghat थलघाट भोरघाट पालघाट

Which of the following does not have influence over the climate in India

ि ि ि ी ी

72)

73)

िनिलखत म स िकसका भाव भारत की जलवाय पर नही पड़ता ह

1 Ocean currents सागर की लहर

2 Nearness to equator भम रखा स िनकटता

3 Monsoons मानसन

4 Presence of Indian ocean भारतीय महासागर की उपथित

Correct Answer -

Ocean currents सागर की लहर

Which of the following cloud types has the characteristics like vertical tall narrow and puffy

िनिलखत म स िकस कार क मघो म लबवत लबी सकीण और थलता जसी िवशषताए ह

1 Cumulonimbus तफानी मघ

2 Cumulus मघ पज

3 Cirrocumulus पाभ कपासी मघ

4 Nimbostratus वषारी मघ

Correct Answer -

Cumulus मघ पज

Which of the following statement is INCORRECT about Crude Birth Rate

िनिलखत स कौन सा कथन अशोिधत ज दर क बार म सही नही ह

1 It cannot be used for comparing fertility level between two countries with different population characteristics

इसका उपयोग िविभ जनसा िवशषताओ वाल दो दशो क बीच जनन र की तलना क िलए नही िकया जा सकता ह

2 It is a standardized measure of fertility

यह जनन मता का मानकीकत उपाय ह

3 It is effected by the age-sex composition of the population

यह आबादी की आय-िलग सरचना स भािवत होता ह

4 It is expressed per 1000 population in a given geographical unit

यह िकसी दी गई भौगोिलक इकाई म ित 1000 जनसा पर िकया जाता ह

Correct Answer -

It is a standardized measure of fertility

यह जनन मता का मानकीकत उपाय ह

74)

75)

76)

77)

Which of the following state in India experienced negative decadal growth rate during 2001 to 2011census

भारत म िनिलखत म स िकस रा म वष 2001 स 2011 की जनगणना क दौरान नकाराक िगरावट दर ई

1 Tripura िपरा

2 Nagaland नागालड

3 Haryana हरयाणा

4 Odisha ओिडसा

Correct Answer -

Nagaland नागालड

Which of the following is NOT a characteristic of peninsular rivers

िनिलखत म स कौन सी िवशषता ायीपीय निदयो म नही होती ह

1 Flow through shallow valleys उथल घािटयो क माम स वाह

2 Seasonal flow मौसमी वाह

3 Little erosional activity थोड़ी कटावदार गितिविध

4 Meandering tendency often shifting their beds घमावदार वि अर अपन तटो को थानातरत करना

Correct Answer -

Meandering tendency often shifting their beds घमावदार वि अर अपन तटो को थानातरत करना

Which of the following gases in the atmosphere absorbs heat from the Sunrsquos radiation and the Earthssurface

वायमडल म िनिलखत म स कौन सी गस सय क िविकरण और पी की सतह स ऊा को अवशोिषत करती ह

1 Neon िनयॉन

2 Carbon dioxide काबन डाइऑाइड

3 Argon आगन

4 Nitrogen नाइट ोजन

Correct Answer -

Carbon dioxide काबन डाइऑाइड

Which of the following kind of settlement pattern is found at the confluence of rivers

ि ि ि ि ो

78)

79)

80)

िनिलखत म स िकस कार का वथापन पटन निदयो क सगम पर पाया जाता ह

1 Triangular Paern िकोणीय पटन

2 Circular or Semi-Circular Paern परप या अध-परप पटन

3 Nebular Paern नबलर पटन

4 Star ndashShaped Paern ार-आकार का पटन

Correct Answer -

Triangular Paern िकोणीय पटन

Which one was not the objective of the Biosphere Reserve Projects launched by the UNESCO

यनो ारा श की गई सरित जवमडल परयोजनाओ का उ इनम स कौन सा नही था

1 To promote teaching and research िशण और अनसधान को बढ़ावा दना

2 To make agriculture sustainable किष को दीघकािलक बनाना

3 To conserve ecosystems पारथितक त को सरित करना

4 To conserve genetic diversity for a longtime लब समय तक अनवािशक िविवधता को सरित करना

Correct Answer -

To make agriculture sustainable किष को दीघकािलक बनाना

Which region of the Earth surface receives the highest amount of insulation

पी सतह का कौन सा तापावरोधन की उतम माा ा करता ह

1 Land mass थलखड

2 Savannah region सवाना

3 Water bodies जल िनकाय

4 Tropical desert उकिटबधीय रिगान

Correct Answer -

Tropical desert उकिटबधीय रिगान

Which one of the following is not a biodiversity hotspot

िनिलखत म स कौन सा जव िविवधता का म जगह नही ह

1 Eastern Himalaya पव िहमालय

2 Eastern Ghats पव घाट

81)

82)

83)

3 Indo-Myanmar भारत-ामार

4 Westerm Ghats पिमी घाट

Correct Answer -

Eastern Ghats पव घाट

Which one of the following is NOT a part of the World Network of Biosphere Reserves based on theUNESCO Man and Biosphere Programme

यनो मन और बायोीयर कायम क आधार पर िनिलखत म स कौन बायोीयर रजव क िव नटवक कािहा नही ह

1 Gulf of Mannar मार की खाड़ी

2 Seshachalam शषाचलम

3 Sunderban सदरबन

4 Nilgiri नीलिगर

Correct Answer -

Seshachalam शषाचलम

Which one of the following is an example of ldquodesert vegetationrdquo

िनिलखत म स कौन मथलीय वनित का एक उदाहरण ह

1 Mosses and lichens दलदल और शवाल

2 Temperate grassland समशीतो घास क मदान

3 Coniferous forest शकधारी वन

4 Acacia and cactus एकािसया और कस

Correct Answer -

Acacia and cactus एकािसया और कस

Which one of the following reflects more sunlight िनिलखत म स कौन सा सय की रोशनी को अिधकपरावितत करता ह

1 Paddy crop land धान फसल भिम

2 Land covered with fresh snow ताजा बफ स आािदत भिम

3 Sand desert रतीली रिगान

4 Prairie land यरी भिम

84)

85)

86)

87)

Correct Answer -

Land covered with fresh snow ताजा बफ स आािदत भिम

Which layer of the atmosphere is in contact with the surface of the earthrsquos oceans

वायमडल की कौन सी परत पी क महासागरो की सतह क सपक म ह

1 Stratosphere समताप मडल

2 Mesosphere म मडल

3 Hydrosphere जलमडल

4 Troposphere ोभ मडल

Correct Answer -

Troposphere ोभ मडल

Mediterranean Sea is a border of which of the following countries भम सागर िनिलखत दशो म सिकसकी सीमा ह

1 None of these इनम स कोई नही

2 Iraq इराक

3 Lebanon लबनान

4 Jordan जॉडन

Correct Answer -

Lebanon लबनान

Benguela ocean currents are found along which coast बगएला महासागर धाराए िकस तट क साथ पायीजाती ह

1 East Coast of South America दिण अमरका क पव तट

2 East Coast of Africa अीका क पव तट

3 West Coast of South America दिण अमरका क पिमी तट

4 West Coast of Africa अीका क पिमी तट

Correct Answer -

West Coast of Africa अीका क पिमी तट

88)

89)

90)

Due to tension a block of land on one side being pushed up or upthrown relative to the downthrown blockis referred as

तनाव क कारण नीच फ क ए खड क साप भिम का एक खड एक ओर स ऊपर धकला जाता ह या ऊपर की ओरफ का जाता ह यह _____ क प म सदिभत ह

1 Thrust fault प श

2 Normal fault सामा श

3 Reverse fault म श

4 Strike slip fault नितलब सपण श

Correct Answer -

Normal fault सामा श

Inter-tropical doldrums is a zone of ______ अतर-उकिटबधीय डोलड ______ का एक ह

1 Frontolysis टोलायिसस

2 Convergence अिभसरण

3 Inter-tropical divergence zone अतर-उकिटबधीय िवचलन

4 Local wind थानीय वाय

Correct Answer -

Convergence अिभसरण

The Horse Latitudes are regions located at about _____ north and south of the equator

हॉस अाश भम रखा क उर और दिण म लगभग _____ पर थत ह

1 30ndash60 degree Latitude 30-60 िडी अाश

2 0ndash5 degree Latitude 0-5 िडी अाश

3 30 degree Latitude 30 िडी अाश

4 60ndash90 degree Latitude 60-90 िडी अाश

Correct Answer -

30 degree Latitude 30 िडी अाश

Generally evaporation is high over which part of the Earth

आम तौर पर पी क िकस भाग पर वाीकरण अिधक होता ह

1 Equatorial maritime भमवत समीय ी ी

91)

92)

2 Equatorial continental भमवत महाीपीय

3 Polar maritime वीय समीय

4 Polar continental वीय महाीपीय

Correct Answer -

Equatorial maritime भमवत समीय

A very high temperature during summer in north western India leads to what type of climaticcondition in south

उर पिमी भारत म गम क दौरान बत अिधक तापमान होन क कारण दिण म िकस कार की जलवाय थितउ करता ह

1 Depression over arabian sea अरब सागर पर अवनमन

2 Failure monsoon मानसन िवफलता

3 Successful monsoon मानसन सफलता

4 Cyclones चवात

Correct Answer -

Successful monsoon मानसन सफलता

Lightning and thunder are the resultant effect when तिड़त और गजन परणामी भाव ह जब

1 Two massive clouds hit powerfully each other first lightning is produced and later sound is produced

दो बड़ बादल एक दसर स शशाली ढग स टकरात ह पहल आकाशीय िवदयत उ होता ह और बाद म िन उहोती ह

2 Two massive clouds come into contact with the powerful wind collision this results into first sound and thenlightning

दो बड़ बादल शशाली पवन सघ क सपक म आत ह इसका परणामप पहल िन और िफर आकाशीय िवदयतउ होता ह

3 None of the above उपरो म स कोई भी नही

4 A high density cloud contains positively and negatively charged electric ions and when this interacts light andsound are simultaneously produced

एक उ घन बादल म धनाक और ऋणाक आविशत िवदयत आयन होत ह और जब यह परर भाव डालत ह तोकाश और िन एक साथ उािदत होती ह

Correct Answer -

A high density cloud contains positively and negatively charged electric ions and when this interacts light andsound are simultaneously produced

औ ि ि ो औ ो

93)

94)

95)

एक उ घन बादल म धनाक और ऋणाक आविशत िवदयत आयन होत ह और जब यह परर भाव डालत ह तोकाश और िन एक साथ उािदत होती ह

Doon Valley is able to grow rice because दन घाटी चावल उगान म सम ह ोिक

1 Other crops cannot be grown वहा अ फसलो को उगाया नही जा सकता ह

2 People in the valley are rice eaters घाटी म लोग चावल खान वाल ह

3 There is a huge export demand of rice वहा चावल की भारी िनयात माग ह

4 It has warm summer and snow melt waters for irrigation

वहा गिमया गम होती ह िसचाई क िलए बफ का िपघला आ पानी होता ह

Correct Answer -

It has warm summer and snow melt waters for irrigation

वहा गिमया गम होती ह िसचाई क िलए बफ का िपघला आ पानी होता ह

CANCELLED

In the geological time scale the Mesozoic Era DOES NOT contains which of the following periods

भगभय समय पमान पर मजीवी यग म िन कालो म स कौन नही ह

1 Triassic ट ाइऐिसक

2 Jurassic जरिसक

3 Cretaceous चाकमय

4 Carboniferous काबनी

Correct Answer -

Carboniferous काबनी

96)

1 P-3 Q-4 R-2 S-1

2 P-3 Q-4 R-1 S-2

3 P-3 Q-4 R-1 S-2

4 P-4 Q-3 R-2 S-1

Correct Answer -

P-4 Q-3 R-2 S-1

1 P-3 Q-1 R-4 S-2

2 P-3 Q-4 R-1 S-2

3 P-3 Q-2 R-4 S-1

97)

98)

4 P-2 Q-1 R-4 S-3

Correct Answer -

P-3 Q-4 R-1 S-2

1 P-3 Q-1 R-4 S-2

2 P-2 Q-3 R-4 S-1

3 P-2 Q-1 R-3 S-4

4 P-4 Q-2 R-1 S-3

Correct Answer -

P-3 Q-1 R-4 S-2

99)

100)

1 P-3 Q-2 R-4 S-1

2 P-1 Q-2 R-3 S-4

3 P-2 Q-3 R-1 S-4

4 P-4 Q-3 R-2 S-1

Correct Answer -

P-2 Q-3 R-1 S-4

ldquoHuman geography is the study of changing relationship between the unresting man and the unstableearthrdquo was defined by

lsquolsquoमानव भगोल ाकल आदमी और अथर पी क बीच सबध परवतन का अयन हrdquo ______ ारा परभािषत िकया गयाथा

1 J Brunches ज चस

2 EC Semple ईसी सल

3 HJ Mackinder एच ज मिकदर

4 PV Blache पीवी च

Correct Answer -

EC Semple ईसी सल

Sedimentary rocks are finally and ultimately derived from the____________

अवसादी चान अततः ________ स ा की जाती ह

1 action of earth movements पी की गितिविधयो

2 marine deposit समी िनप

3 weathering of metamorphic rocks पातरत चानो क अपय

4 weathering of igneous rocks आय चानो क अपय

Correct Answer -

weathering of igneous rocks आय चानो क अपय

Page 7: High School Teacher Eligibility Test- BOARD PROFESSIONAL ...peb.mp.gov.in/results/RESULT_18/HST_RES18/Final_anwser_key/HST… · M a ndl a / मंड ल ... Under the Madhya Pradesh

2)

3)

4)

5)

Correct Answer -

Qatar कतर

According to sex ratio trends of Census 2011 which district of Madhya Pradesh has been ranked numberOne

जनगणना 2011 म िलग अनपात क झानो क अनसार म दश क िकस िजल को नबर एक थान िदया गया ह

1 Seoni िसओनी

2 Ratlam रतलाम

3 Balaghat बालाघाट

4 Mandla मडला

Correct Answer -

Balaghat बालाघाट

The folk dance form called lsquoMatkirsquo is prevalent in which of the following regions of Madhya Pradesh

मटकी नामक लोक न प म यदश क िनिलखत म स िकस म चिलत ह

1 Bundelkhand बदलखड

2 Malwa मालवा

3 Vindhyachal िवाचल

4 Baghelkhand बघलख

Correct Answer -

Malwa मालवा

Who appoints the Chairman and the Members of the Union Public Service Commission

सघ लोक सवा आयोग क अ और सदो की िनय कौन करता ह

1 Chief Justice of India भारत क म य यायधीश

2 Prime Minister धान मी

3 President रा ट पित

4 Chief Election Commissioner म य चनाव आय त

Correct Answer -

President रा ट पित

6)

7)

8)

Under the Madhya Pradesh Ladli Laxmi Yojana the beneficiary is paid the total amount on attaining theage of ___ मदश लाडली ली योजना क तहत लाभाथ की आय ___ वष परी होन पर कल रािश का भगतान िकयाजाता ह

1 18

2 21

3 23

4 16

Correct Answer -

21

India has won the gold medal in hockey ______times at the Olympics

ओलिपक खलो म भारत न हॉकी म ण पदक _____ बार जीता ह

1 9

2 7

3 8

4 6

Correct Answer -

8

Where is the Madhya Pradesh Tribal Museum located at

म दश जनजातीय सहालय कहा थत ह

1 Indore इदौर

2 Jhabua झबआ

3 Khargone खरगोन

4 Bhopal भोपाल

Correct Answer -

Bhopal भोपाल

The fictional boy character of the lsquoThe Jungle Bookrsquo by author Rudyard Kipling was

लखक डयाड िकपिलग ारा lsquoद जगल बकrsquo का कािनक बालक चर था

1 Kaa का

2 Mowgli मोगली

9)

10)

1)

3 Shere Khan शर खान

4 Bagheera बघीरा

Correct Answer -

Mowgli मोगली

The recipient of which one of the following awards given by Madhya Pradesh Government gets the highest cashprize

िनिलखत परारो म स िकसक ाकता को म दश सरकार ारा उतम नकद परार िदया जाता ह

1 Mahatma Gandhi award महाा गाधी परार

2 Shikar Award िशखर परार

3 Sharad Joshi Award शरद जोशी परार

4 Kishore Kumar Award िकशोर कमार परार

Correct Answer -

Mahatma Gandhi award महाा गाधी परार

Who among the following was the Trinidadian writer of Indian descent

िनिलखत म स भारतीय मल क ििनदािदयन लखक कौन थ

1 RK Narayan आर क नारायण

2 Anita Desai अनीता दसाई

3 Rudyard Kipling डयाड िकपिलग

4 VS Naipaul वीएस नायपॉल

Correct Answer -

VS Naipaul वीएस नायपॉल

Topic- GENERAL REASONING

Find the missing number ल त स या ात कर

4 196 16 169 ____ 144 64

1 81

2 36

3 32

4 21

2)

3)

4)

Correct Answer -

36

Find the average औसत ात कर

25 35 45 55 65 amp 75

1 48

2 50

3 47

4 49

Correct Answer -

50

The below series uses a sequence of alphabets and numbers Identify the incorrect combination

नीच दी गई खला अरो एव साओ का अनम योग करती ह गलत सयोजन पहचान

(i) FT85DF77ER

(ii) FT85DF77ER

(iii) FT85DE77ER

(iv) FT85DF77ER

1 ii

2 iv

3 i

4 iii

Correct Answer -

iii

A and B together can complete a piece of work in 10 days and B alone can complete the same work in 20days In how many days can A alone complete the work

A और B एक साथ काम क एक भाग को 10 िदनो म समा त करत ह और B अकल उसी काम को 20 िदनो म समा त करता हिकतन िदनो म A अकल उस काम को समा त कर सकता ह

1 20 days 20 िदन

2 30 days 30 िदन

3 10 days 10 िदन

4 25 days 25 िदन

5)

6)

Correct Answer -

20 days 20 िदन

Which one of the following four addresses is NOT EXACTLY same as the one given below

िनिलखत िदए गए चार पतो म स नीच िदए गए पत क ठीक समान कौन सा एक नही ह

Ground Floor 59 Goulburn St

Sydney NSW 2000 Australia

+61 (02) 8987 3700

(i) Ground Floor 59 Goulburn St

Sydney NSW 2000 Australia

+61 (02) 8987 3700

(ii) Ground Floor 59 Goulburn St

Sydney NSW 2000 Australia

+61 (02) 8987 3700

(iii) Ground Floor 59 Govlburn St

Sqdney NSW 2000 Australia

+61 (02) 8987 3700

(iv) Ground Floor 59 Goulburn St

Sydney NSW 2000 Australia

+61 (02) 8987 3700

1 ii

2 iv

3 i

4 iii

Correct Answer -

iii

In a group of 75 people 32 of them like cold drink 56 of them like general water and each person likesat least one of the two drinks How many people like both

75 लोगो क समह म उनम स 32 को को िड क पसद ह उनम स 56 को सामा जल पसद ह और क को दो म स कम स कम एक पय पसद ह िकतन लोगो को दोनो पसद ह

1 11

2 13

7)

1)

2)

3 12

4 14

Correct Answer -

13

If A = 1 FAT = 27 then FAITH =

यिद A = 1 FAT = 27 तो FAITH =

1 41

2 40

3 42

4 44

Correct Answer -

44

Topic- PEDAGOGY

CANCELLED

Basic education became a major initiative because of the efforts of

ाथिमक िशा िन न क यास स एक मह वपण कायम बन गई

1 Dr Radhakrishnan डॉ राधाक णन

2 Dr Rajendra Prasad डॉ राज साद

3 Mahatma Gandhi महा मा गाधी

4 Rabindranath Tagore रबी नाथ टगोर

Correct Answer -

Mahatma Gandhi महा मा गाधी

The ldquoclause of phraserdquo is an unit of perception of लॉज ऑफ ज िन न क धारणा क एक इकाई ह

1 Image छिव

2 Concept अवधारणा

3 Language भाषा

4 Thought िवचार

Correct Answer -

3)

4)

5)

6)

Language भाषा

Which method is most suitable to study communication process among students

छाो क बीच सचार िया का अयन करन क िलए कौन सी िविध सबस उपय ह

1 Case Study मामल का अयन (कस टडी)

2 Systematic Observation वथत अवलोकन

3 Experimental Method योगाक िविध

4 Introspection आ-िनरीण (इट ोस शन)

Correct Answer -

Systematic Observation वथत अवलोकन

Genes in a human being are located in मानवो म जीन इनम थत होता ह

1 cytoplasm कोिशका (साइटो ला म)

2 ribosomes राइबोसोम

3 cell membranes कोिशका िझी

4 chromosomes गणस (ोमोसोम)

Correct Answer -

chromosomes गणस (ोमोसोम)

With smaller classes teachers are much more able to ____________

छोटी काओ क साथ िशक ____________ म अिधक सम होत ह

1 Go slow while teaching िशण क दौरान धीमी गित स जान

2 Narrate more personal experiences अिधक गत अनभवो को बतान

3 Adapt instruction to individual differences among students छाो क बीच गत मतभदो क िलए अनकल िनदश दन म

4 Make use of the extra space for extra-curricular activities पातर गितिविधयो क िलए अितर जगह का उपयोग करन

Correct Answer -

Adapt instruction to individual differences among students छाो क बीच गत मतभदो क िलए अनकल िनदश दन म

While engaging in a task the child gets bored This is a sign of

एक काम म होन पर बा ऊब जाता ह यह िन का सकत ह

1 the task requiring a professional approach काय को ावसाियक िकोण की आवकता ह

7)

8)

9)

2 the task becoming mechanically repetitive काय यािक प स दोहराव वाला ह

3 the child not being intelligent बा बमान नही ह

4 the child being incapable of learning बा सीखन म असमथ ह

Correct Answer -

the task becoming mechanically repetitive काय यािक प स दोहराव वाला ह

Educational Psychologists are more concerned with the learning in __________

शिणक मनोवािनक __________ म अिधगम क साथ अिधक िचितत होत ह

1 Formal environment औपचारक वातावरण

2 Informal environment अनौपचारक वातावरण

3 Physical environment भौितक वातावरण

4 Social environment सामािजक वातावरण

Correct Answer -

Formal environment औपचारक वातावरण

Special needs education is the type of education िवशष ज़रतो वाली िशा वह िशा होती ह जो

1 Given to person with disabilities अम य को दी जाती ह

2 Given to people from remote areas दर थ ो क लोगो को दी जाती ह

3 Provided to intelligent people बमान लोगो को दी जाती ह

4 Established by colonial masters औपिनविशक मखयाओ ारा थािपत की गई ह

Correct Answer -

Given to person with disabilities अम य को दी जाती ह

The Stanford-Binet scale of intelligence was first published in the year

ब क नफोड-िबनट कल को इस वष म पहली बार कािशत िकया गया था

1 1916

2 1903

3 1908

4 1900

Correct Answer -

1916

10)

11)

12)

13)

The term lsquofictional finalismrsquo was propounded by

पद lsquoकत योजनवादrsquo (िफ शनल फाइनिल म) इनक ारा ितपािदत िकया गया था

1 Skinner नर

2 Freud ायड

3 Adler एडलर

4 Pavlov पावलोव

Correct Answer -

Adler एडलर

Rational Emotive Behavior Therapy was propounded by

तक सगत भावनाक वहार थरपी िन क ारा ितपािदत की गई थी

1 Carl Jung काल यग

2 Carl Rogers काल रोजस

3 Aaron Beck आरोन बक

4 Albert Ellis अट एिलस

Correct Answer -

Albert Ellis अट एिलस

Which of the following indicates the quality of education in a school

िनिलखत म स या एक कल म िशा की गणव ता का सकतक ह

1 Text-books and Teaching-learning material पा-प तक तथा िशण व सीखन स सबिधत सामी

2 Infrastructural facilities at the school कल म आधारभत ढाच स सबिधत सिवधाए

3 Student achievement level िवािथयो का उपल तर

4 Classroom systems का की यव था

Correct Answer -

Student achievement level िवािथयो का उपल तर

Which of the following teachers can be identified with authoritarian teaching style

िनिलखत म स कौन सा िशक अिधकारवादी िशण शली क साथ पहचाना जा सकता ह

1 Laissez-faire teacher अब ध िशक

14)

15)

2 Democratic teacher लोकतीय िशक

3 Indifferent teacher िन प िशक

4 Direct instruction teacher िनदश िशक

Correct Answer -

Direct instruction teacher िनदश िशक

Who was the pioneer of classical conditioning

िचरितित ानकलन ( लािसकल कडीशिनग) क वतक कौन थ

1 Skinner नर

2 Pavlov पावलोव

3 Kohler कोहलर

4 Freud ायड

Correct Answer -

Pavlov पावलोव

Which of the following are true with reference to short term memory

1 Limited capacity

2 Brief storage of information

3 Unlimited capacity

4 Duration of storage less than twenty seconds

अ पकािलक मित क सदभ म िन न म स या स य ह

1 सीिमत मता

2 सचना का सि त भडारण

3 असीिमत मता

4 बीस सकड स कम भडारण की अविध

1 3 and 4 3 और 4

2 2 and 4 2 और 4

3 13 and 4 13 और 4

4 12 and 4 12 और 4

Correct Answer -

12 and 4 12 और 4

1)

2)

3)

4)

Topic- GEOGRAPHY

Which of the following ranges of population size is used to define Class-III city by Indian Census

भारतीय जनगणना ारा वग-III शहर को परभािषत करन क िलए िनिलखत म स िकस रज क जनसा आकारका उपयोग िकया जाता ह

1 20 000 to 49999 20 000 स 49999

2 30000 to 59999 30000 स 59999

3 24000 to 54999 24000 स 54999

4 50000 to 99999 50000 स 99999

Correct Answer -

20 000 to 49999 20 000 स 49999

Carbonaceous rocks which produce coal and oil belong to the category of rocks called_______

कोयल और तल का उादन करन वाली काबनय चान ______ नामक चानो की णी स सबिधत ह

1 metamorphic पातरत

2 sedimentary अवसादी

3 inorganic अजिवक

4 igneous आय

Correct Answer -

sedimentary अवसादी

The ruhr-complex is a major industrial centre in र-परसर िन का एक मख औोिगक क ह

1 North America उरी अमरका

2 Russia स

3 Germany जमनी

4 Europe यरोप

Correct Answer -

Germany जमनी

The term lsquoRegurrsquo refers to श lsquoरगरrsquo ______ स सबिधत ह

1 Deltaic alluvial soils डा जलोढ़ िमी

ि ी

5)

6)

2 Laterite soils लटराइट िमी

3 Red and yellow soils लाल और पीली िमी

4 Black cotton soils काली कपास िमी

Correct Answer -

Black cotton soils काली कपास िमी

Read the given statements and answer which of the following options isare correct

1 Lower the pressure greater the atmospheric disturbance

2 Air move from higher to low pressure

िदए गए कथन को पढ़ और उर द िक िन म स कौन सास िवक सही ह

1 िजतना दाब कम होगा वायमडलीय बाधाए उतनी अिधक होगी

2 वाय उ स िन दाब की ओर गित करती ह

1 Both Statements 1 and 2 are correct दोनो कथन 1 और 2 सही ह

2 Both Statements 1 and 2 are wrong दोनो कथन 1 और 2 गलत ह

3 Statement 1 is wrong and only Statement 2 is correct कथन 1 गलत ह और कवल कथन 2 सही ह

4 Statement 1 is correct and Statement 2 is wrong कथन 1 सही ह और कथन 2 गलत ह

Correct Answer -

Both Statements 1 and 2 are correct दोनो कथन 1 और 2 सही ह

CANCELLED

1 4 1 2 and 3 4 1 2 और 3

2 4 1 3 and 2 4 1 3 और 2

3 1 4 2 and 3 1 4 2 और 3

4 1 4 3 and 2 1 4 3 और 2

Correct Answer -

7)

8)

9)

1 4 3 and 2 1 4 3 और 2

CANCELLED

Karl Pearsonrsquos correlation co-efficient is काल िपयसन का सहसबध गणाक ह

1 Arithmec mean समार मा

2 Geometric mean गणोर मा

3 None of these इनम स कोई नही

4 Harmonic mean हराक मा

Correct Answer -

Geometric mean गणोर मा

CANCELLED

ldquoEach day is more or less the same the morning is clear and bright with a sea breeze as the Sun climbshigh in the sky heat mounts up dark clouds form then rain comes with thunder lighting But rain is soonoverrdquo Which of the following regions is described in the above passage

ldquoक िदन समान स अिधक या कम होता ह सम की हवा क साथ और उल सबह होती ह जस सयआकाश म ऊचा चढ़ता ह गम बढ़ जाती ह काल बादल बनत ह िफर िबजली क साथ बारश आती ह लिकनबारश जी ख हो जाती हlsquorsquo उपरो पा म िनिलखत म स िकन ो का वणन िकया गया ह

1 Equatorial भमरखीय

2 Equatorial भमरखीय

3 Savannah सवाना

4 Mediterranean आातरक (भमसागरीय)

5 Mediterranean आातरक (भमसागरीय)

6 Monsoon मानसन

7 Monsoon मानसन

Correct Answer -

Equatorial भमरखीय

Equatorial भमरखीय

CANCELLED

In which epoch of the geological history of the Earth dinosaurs reached their largest size

पी क भगभय इितहास क िकस यग म डायनासोर अपन सबस बड़ आकार तक पचि

10)

11)

1 Triassic ट ायिसक

2 Jurassic जरािसक

3 Cretaceous ीटशस

4 Permian पिमयन

Correct Answer -

Jurassic जरािसक

CANCELLED

A spring tide would occur in which of the following conditions

िनिलखत म स िकन थितयो म ार-भाटा आता ह

1 When the Sun Moon and Earth are in a straight line

जब सय चमा और पी एक सीधी रखा म होत ह

2 When the Moon and Earth are in right angle to each other

जब चमा और पी एक-दसर क दािहन कोण म होत ह

3 When the Earth and Moon are in right angle to the Sun

जब पी और चमा सय क दािहन कोण म होत ह

4 When the Sun and Moon are in right angle to each other

जब सय और चमा एक-दसर क दािहन कोण म होत ह

Correct Answer -

When the Sun Moon and Earth are in a straight line

जब सय चमा और पी एक सीधी रखा म होत ह

CANCELLED

An observe on the Earthrsquos surface always sees the same face of the moon because

एक पयवक को पी की सतह स हमशा चाद का एक ही फलक िदखाई दता ह ोिक

1 Its path of revolution around the earth is the same as that of the earth around the Sun

इसका पी क चारो ओर परमण का माग सय क चारो ओर पी क समान ही ह

2 Its period of revolution around the Earth is the same as its period of rotation around its own axis

इसकी पी क चारो ओर परमण की अविध उसकी अपनी धरी क चारो ओर घणन की अविध क समान ह

3 Its period of rotation is the same as that of the Earth इसकी घणन की अविध पी क समान ह

ी ि ी ी

12)

13)

14)

4 Its direct of rotation is the same as that of Earth घणन की िदशा पी क समान ही ह

Correct Answer -

Its period of revolution around the Earth is the same as its period of rotation around its own axis

इसकी पी क चारो ओर परमण की अविध उसकी अपनी धरी क चारो ओर घणन की अविध क समान ह

CANCELLED

The pebbles that are faceted by the sand-blasting and shaped polished by the wind abrasions are known as

पवन अपघषन ारा पॉिलश रत-िवोिटत और साच म ढला ककड़ ___________ क प म जाना जाता ह

1 Dreikanter िकोणक

2 Pediments िकोिनका

3 Inselberg इलबग

4 Dunes टीबा

Correct Answer -

Dreikanter िकोणक

CANCELLED

Astronomical unit is the average distance between खगोलीय इकाई ______ क बीच की औसत दरी ह

1 Earth and Mars पी और मगल

2 Earth and mercury पी और बध

3 Earth and moon पी और चमा

4 Earth and Sun पी और सय

Correct Answer -

Earth and Sun पी और सय

During cold weather season in the northern plains there will be an inflow of cyclonic disturbancesfrom the _________ directions

शीत मौसम क दौरान उरी मदानी इलाको म _________ िदशाओ स चवात सबधी गड़बड़ी का अतवाह होगा

1 East and Northwest पव और उरपिम

2 East and Northeast पव और पवर

3 West and East पिम और पव

ि औ ि

15)

16)

17)

4 West and Northwest पिम और उरपिम

Correct Answer -

West and Northwest पिम और उरपिम

During an earth quake the velocity of the body waves will________ along with the increase in densityof the material it is passing through

भकप क दौरान लहरो क ऊपरी भाग का वग घन म व क साथ-साथ ________ जो इसस गजरन वाली वको आग बढाएगी

1 not change नही बदलगा

2 increase initially and then decrease शआत म बढ़गा और िफर घटगा

3 increase बढ़गा

4 decrease घटगा

Correct Answer -

increase बढ़गा

The Clouded Leopard National park is situated in which of the following states

िनिलखत म िकस रा म धिमल तदआ रा ीय उान (ाउडड लपड नशनल पाक ) थत ह

1 Tripura िपरा

2 Uttar Pradesh उर दश

3 Assam असम

4 Mizoram िमजोरम

Correct Answer -

Tripura िपरा

Usually the land surfaces are heated more quickly than the water surfaces because _____________

आम तौर पर जल सतहो की तलना म भिम सतह अिधक तजी स गम होती ह ोिक _____________ ह

1 the specific heat of water is higher than land पानी की िविश ऊा भिम स अिधक

2 the specific heat of water is lesser than land पानी की िविश ऊा भिम स कम होती

3 the latent heat of water is higher than the land पानी की अतिनिहत ऊा भिम स अिधक

4 the land reflects more heat radiation than water भिम पानी की तलना म अिधक ऊा क िविकरण को पराविततकरती

18)

19)

20)

21)

Correct Answer -

the specific heat of water is higher than land पानी की िविश ऊा भिम स अिधक

The longest shore-line is along the state of सबस लबी समतटीय रखा िन रा क साथ ह

1 Maharashtra महारा

2 Orissa उड़ीसा

3 Kerala करल

4 Gujarat गजरात

Correct Answer -

Gujarat गजरात

The position when the Earth is farthest from the Sun is known as

जब पी सय स सबस दर होती ह तो उस थित को िन नाम स जाना जाता ह

1 Perihelion उपसौर

2 Vernal Equinox बसत िवषव

3 Aphelion अपसौर

4 Autumnal Equinox शराल िवषव

Correct Answer -

Aphelion अपसौर

The seasonal reversal of winds is the typical characteristic of

हवाओ का मौसमी परवतन ______ की सामा िवशषता ह

1 Mediterranean climates only कवल भमसागरीय जलवाय

2 All of the above climates उपय सभी मौसम

3 Monsoon climate only कवल मानसन जलवाय

4 Equatorial climate only कवल भमरखीय जलवाय

Correct Answer -

Monsoon climate only कवल मानसन जलवाय

In _________ rocks the minerals will occurs in beds or layers

ो ि ो ो

22)

23)

24)

______ चानो म खिनज तल या परतो म होत ह

1 metamorphic कायातरत

2 igneous and metamorphic आय और कायातरत

3 igneous आय

4 sedimentary अवसादी

Correct Answer -

sedimentary अवसादी

Black soil is ideal for the cultivation of cotton as कपास की खती क िलए काली िमी आदश ह ोिक

1 Its colour is black यह काली होती ह

2 It is found on plateau regions यह पठार ो म पायी जाती ह

3 It is made up of lava यह लावा स बनी होती ह

4 It can retain moisture यह नमी को बरकरार रख सकती ह

Correct Answer -

It can retain moisture यह नमी को बरकरार रख सकती ह

The National Survey and Mapping Organization of the country works under the Department of___________

दश का रा ीय सवण और मानिचण सगठन ___________ िवभाग क अतगत काय करता ह

1 Space अतर

2 Science and Technology िवान और तकनीक

3 Culture सित

4 Tourism पयटन

Correct Answer -

Science and Technology िवान और तकनीक

Palk strait separates India from पाक जलडमम भारत स _____ को अलग करता ह

1 Pakistan पािकान

2 Andaman Island अडमान ीप

3 China चीन

25)

26)

27)

4 Sri Lanka ीलका

Correct Answer -

Sri Lanka ीलका

Which among the following state is the major producer of Bauxite in India

िनिलखत म स कौन सा रा भारत म बॉाइट का मख उादक ह

1 Madhya Pradesh मदश

2 Rajasthan राजथान

3 Goa गोवा

4 Orissa उड़ीसा

Correct Answer -

Orissa उड़ीसा

Which of the following states DOES NOT share border with Chhattisgarh

िनिलखत म स कौन सा रा छीसगढ़ क साथ सीमा साझा नही करता ह

1 Telangana तलगाना

2 Uttar Pradesh उर दश

3 Bihar िबहार

4 Andhra Pradesh आ दश

Correct Answer -

Bihar िबहार

Which of the following statements is INCORRECT with respect to parallels of latitudes

अाश क समानातरो क सबध म िन निलखत म स कौन सा कथन गलत ह

1 A line joining places of equal latitude is known as parallel of largest

समान अाश क थानो को जोड़न वाली रखा को िवशालतम क समानातर क प म जाना जाता ह

2 They stat from equator and run parallels to it

व भम रखा स ारभ होत ह और इसक समानातर चलत ह

3 All parallels are equal in length सभी समातर लबाई म समान ह

4 All parallels are drawn as circles on the globe ोब पर सभी समानातर वो क प म खीच जात ह

28)

29)

30)

31)

Correct Answer -

All parallels are equal in length सभी समातर लबाई म समान ह

Which of the following Indian states is also known as a lsquoLand of Red river and Blue Hillsrsquo

िनिलखत म स िकस भारतीय रा को लाल नदी और नीली पहािड़यो की भिम क नाम स जाना जाता ह

1 Uttarkhand उराखड

2 Assam असम

3 Meghalaya मघालय

4 Arunachal Pradesh अणाचल दश

Correct Answer -

Assam असम

In spatial analysis of settlement Rn = 215 indicates which type of settlement arrangement

िनपटान क थािनक िवषण म Rn = 215 यह इिगत करता ह िक िकस कार की िनपटान वथा ह

1 Uniform यिनफॉम

2 Semi-Clustered समी- ल टर

3 Clustered ल टर

4 Random रडम

Correct Answer -

Uniform यिनफॉम

Who are known as the lsquoYellow Peoplersquo lsquoयलो पीपलrsquo क प म कौन जाना जाता ह

1 Mongoloids मोगोलोइडस

2 Nigroids नीोइडस

3 Australoids ऑ लॉइडस

4 Caucasoids कॉकसोइडस

Correct Answer -

Mongoloids मोगोलोइडस

ि ि ो ौ ि

32)

33)

34)

Who publishes the topographical map of India भारत क थलाकितक मानिच को कौन कािशत करता ह

1 Geographical Survey of India भारत का भौगोिलक सवण

2 Government of India भारत सरकार

3 Geological Survey of India भारत क भगभय सवण

4 Survey of India भारत का सवण

Correct Answer -

Survey of India भारत का सवण

Who among the following claimed geography to be the lsquoEcology of Manrsquo

िनिलखत म स िकसन भगोल को मन का पारथितकी कहा ह

1 Alfred Hener अड हटनर

2 Vidal-de la Blache वाइडल-िड लॉ ॉश

3 Oo Schluter ओटो टर

4 Harlan Barrow हरलन बारो

Correct Answer -

Harlan Barrow हरलन बारो

Who among the following is regarded as the founder of humanistic approach in geography

िनिलखत म स िकस भगोल म मानवतावादी िकोण का सथापक माना जाता ह

1 William Bunge िविलयम बग

2 Yi-Fu-Tuan यी-फ- यान

3 Brain JL Berry न जएल बरी

4 Richard Peet रचड पीट

Correct Answer -

Yi-Fu-Tuan यी-फ- यान

Who prepared Lorenz curve लोरज व िकसन तयार िकया

1 Geddes गडस

2 None of these इनम स कोई नही

3 Griffith Taylor ििफथ टलर

35)

36)

37)

4 Max U Lorenz म य लोरज

Correct Answer -

Max U Lorenz म य लोरज

Gulf Streams are the currents of which of the following oceans

खाड़ी की धाराए िनिलखत महासागरो म स िकसकी धाराए ह

1 North Atlantic Ocean उरी अटलािटक महासागर

2 North Pacific Ocean उरी शात महासागर

3 Arabian Sea अरब सागर

4 South Pacific Ocean दिण शात महासागर

Correct Answer -

North Atlantic Ocean उरी अटलािटक महासागर

Disintegration wearing away and removal of rock material is generally referred as

िशला पदाथ (रॉक सामी) का टटना िमटना और हटना आमतौर पर ________ क प म सदिभत िकया जाता ह

1 Shattering िवसकारक

2 Denudation अनाादन

3 Fault श

4 Decomposition िवयोजन

Correct Answer -

Denudation अनाादन

Variations in the length of day time and night from season to season are due to

मौसम स मौसम परवतन पर िदन क समय और रात क समय की अविध म िभताए िन कारण स होती ह

1 The Earthrsquos revolution round the Sun in an elliptical manner पी का दीघवाकार तरीक स सय क चारो घणन

2 The Earthrsquos rotation on its axis पी का इसकी धरी पर घणन

3 Revolution of the Earth on a tilted axis नत अ पर पी का घणन

4 Latitudinal position of the place थान की अाश थित

Correct Answer -

Revolution of the Earth on a tilted axis नत अ पर पी का घणन

38)

39)

40)

Point out the correct sequence of mountain ranges from north to south

उर स दिण तक पवत खलाओ क सही अनम को इिगत कर

1 Great Himalaya Middle Himalaya Outer Himalaya Trans Himalaya

महान िहमालय म िहमालय बा िहमालय परा िहमालय

2 Middle Himalaya Great Himalaya Trans Himalaya Outer Himalaya

म िहमालय महान िहमालय परा िहमालय बा िहमालय

3 Outer Himalaya Middle Himalaya Great Himalaya Trans Himalaya

बा िहमालय म िहमालय महान िहमालय परा िहमालय

4 Trans Himalaya Great Himalaya Middle Himalaya Outer Himalaya

परा िहमालय महान िहमालय म िहमालय बा िहमालय

Correct Answer -

Trans Himalaya Great Himalaya Middle Himalaya Outer Himalaya

परा िहमालय महान िहमालय म िहमालय बा िहमालय

Sunrsquos halo is produced by the refraction of light in सय का भामडल ______ म काश क अपवतन ाराउ होता ह

1 Ice crystals in Cirrro-Cumulus clouds पाभ-कपास मघो क बफ िल

2 Ice crystal in Cirrus clouds पाभमघो क बफ िल

3 Dust particles in Stratus clouds री मघो क धल कण

4 Water vapour in Stratus clouds री मघो क जल वा

Correct Answer -

Ice crystal in Cirrus clouds पाभमघो क बफ िल

Read the given statements and answer which of the following options isare correct

(1) The minerals present in the rocks exposed to atmosphere are not subjected to alteration

(2) Oxidation is one of the processes of chemical weathering

िदए गए कथनो को पढ़ और उर द िक िन म स कौन सास िवक सही ह

(1) वायमल स अनावत शल म उपथत खिनज परवतन क अधीन नही होता ह

(2) ऑीकरण रासायिनक अपय की ियाओ म स एक ह

ो ो

41)

42)

1 Both statements are wrong दोनो कथन गलत ह

2 Both statements are correct दोनो कथन सही ह

3 First statement is wrong and second statement is correct पहला कथन गलत ह और दसरा कथन सही ह

4 First statement is correct and second statement is wrong पहला कथन सही ह और दसरा कथन गलत ह

Correct Answer -

First statement is wrong and second statement is correct पहला कथन गलत ह और दसरा कथन सही ह

Read the given statements and answer which of the following options isare correct

1 Sunrsquos short waves enter the earth partially heating the atmosphere

2 Heated earth surface from the sun produces broader waves which interacts and heats the atmosphere

िदए गए कथन को पढ़ और उर द िक िन म स कौन सास िवक सही ह

1 सय की छोटी तरग पी म आिशक प स वश करती ह और वायमडल को ऊत करती ह

2 सय स ऊत पी की सतह िवारत तरग उ करती ह जो परर भाव डालती ह और वायमडल कोऊत करती ह

1 Both Statements 1 and 2 are correct दोनो कथन 1 और 2 सही ह

2 Both Statements 1 and 2 are wrong दोनो कथन 1 और 2 गलत ह

3 Statement 1 is wrong and only Statement 2 is correct कथन 1 गलत ह और कवल कथन 2 सही ह

4 Only statement 1 is correct कवल कथन 1 सही ह

Correct Answer -

Both Statements 1 and 2 are correct दोनो कथन 1 और 2 सही ह

Read the given statements and answer which of the following options isare correct

(1)The rocks that get changed due to heat and pressure are termed as metamorphic rocks

(2)Slate is one such type of metamorphic rock

िदए गए कथनो को पढ़ और उर द िक िन म स कौन सास िवक सही ह

(1) शल जो ऊा और दाब क कारण परवितत हो जात ह उ कायातरक शलो क प म जाना जाता ह

(2) ट एक तरह का कायातरक शल ह

1 Both statements are wrong दोनो कथन गलत ह

2 Both statements are correct दोनो कथन सही ह

3 First statement is wrong and second statement is correct पहला कथन गलत ह और दसरा कथन सही ह

ी औ

43)

44)

4 First statement is correct and second statement is wrong पहला कथन सही ह और दसरा कथन गलत ह

Correct Answer -

Both statements are correct दोनो कथन सही ह

Read the given statements and answer which of the following options isare correct

1 Higher temperature anomaly is observed in the northern hemisphere

2 Differential heating is absent in Northern Hemisphere

िदए गए कथनो को पढ़ और उर द िक िन म स कौन सास िवक सही ह

1 उरी गोलाध म उ तापमान िवसगित पायी जाती ह

2 उरी गोलाध म अतर ऊन अनपथत होती ह

1 Both Statements 1 and 2 are correct दोनो कथन 1 और 2 सही ह

2 Both Statements 1 and 2 are wrong दोनो कथन 1 और 2 गलत ह

3 Statement 1 is wrong and Statement 2 is correct कथन 1 गलत ह और कथन 2 सही ह

4 Statement 1 is correct and Statement 2 is wrong कथन 1 सही ह और कथन 2 गलत ह

Correct Answer -

Statement 1 is correct and Statement 2 is wrong कथन 1 सही ह और कथन 2 गलत ह

Read the given statements and answer which of the following options isare correct

(1) Plutonic rocks are intrusive type of igneous rocks

(2) It cools very slowly because the surrounding rock serves as insulation around the intrusion of magma

िदए गए कथनो को पढ़ और उर द िक िन म स कौन सास िवक सही ह

(1) िवतलीय शल अतवधी कार क आश शल ह

(2) यह बत धीर-धीर ठडा होता ह ोिक आस-पास क शल मा क अतवधन क चारो ओर रोधन क प म कायकरत ह

1 Both statements are wrong दोनो कथन गलत ह

2 Both statements are correct दोनो कथन सही ह

3 First statement is wrong and second statement is correct पहला कथन गलत ह और दसरा कथन सही ह

4 First statement is correct and second statement is wrong पहला कथन सही ह और दसरा कथन गलत ह

Correct Answer -

Both statements are correct दोनो कथन सही ह

45)

46)

47)

48)

The dust and ash material hurled from the volcanoes are termed as

ालामखी स िनकलन वाली धल और राख सामी को _______ क प म कहा जाता ह

1 Pyroclasc पाइरोाक

2 Hyperclastic हाइपराक

3 Hepiroclastic हिपरोाक

4 Cirroclastic िसरोाक

Correct Answer -

Pyroclasc पाइरोाक

The vertical difference in elevation between a low tide and high tide is referred as

कम ार और उ ार क बीच ऊचाई म लबवत अतर _____ स सदिभत होता ह

1 Tidal slope ारीय ढलान

2 Tidal elevation ारीय उयन

3 Tidal range ारीय परास

4 Tidal height ारीय ऊचाई

Correct Answer -

Tidal range ारीय परास

The maximum biodiversity is found in which of the following regions िनिलखत ो म स अिधकतमजव िविवधता िकसम पायी जाती ह

1 Amazon Basin अमज़न बिसन

2 East Indies ई इडीज

3 Congo Basin कागो बिसन

4 West indies व इडीज

Correct Answer -

Amazon Basin अमज़न बिसन

The cultivation of rice crop produces_______ चावल की फसल की खती ______ का उादन करती ह

1 SO2

49)

50)

51)

2 CH4

3 CFCs

4 CO2

Correct Answer -

CH4

The pressure system with higher pressure at the centre is called__________

क म उ दबाव वाली दबाव णाली को _______ कहा जाता ह

1 front अ

2 depression अवनमन

3 cyclone चवात

4 anti-cyclone ितचवात

Correct Answer -

anti-cyclone ितचवात

The Himalayan region is poor in mineral resources because िहमालयी खिनज ससाधनो म समनही ह ोिक

1 The displacement of rock strata has disturbed the arrangement of rocks and made it complex

शलीय परत क िवथापन न चानो की वथा को अवथत कर िदया ह और इस जिटल बना िदया ह

2 The climate conditions are not suitable for exploitation of minerals

जलवाय की थित खिनजो क दोहन क िलए उपय नही ह

3 The terrain makes explanation of minerals difficult and very costly due to transportation difficulties

भ-भाग परवहन की किठनाइयो क कारण खिनजो का दोहन मल और बत महगा बना दता ह

4 It is made up of crystalline rocks यह िलीय चानो स बना ह

Correct Answer -

The displacement of rock strata has disturbed the arrangement of rocks and made it complex

शलीय परत क िवथापन न चानो की वथा को अवथत कर िदया ह और इस जिटल बना िदया ह

The process through which the moisture is added to the atmosphere by vegetation is termed as

वह िया िजसक माम स वनित ारा वातावरण म नमी िमलायी जाती ह _______ क प म जानी जाती ह

52)

53)

54)

1 Condensation सघनन

2 Evapotranspiration वान-उजन

3 Radiation िविकरण

4 Precipitation वषण

Correct Answer -

Evapotranspiration वान-उजन

The process through which the terrestrial heat is transferred to air by direct contact is termed as

वह िया िजसम सपक ारा थलीय ऊा वाय म थानातरत हो जाती ह ______ क प म जानी जाती ह

1 Conduction चालन

2 Convection सवहन

3 Insolation आतपन

4 Radiation िविकरण

Correct Answer -

Conduction चालन

The largest area under mangroves is in which of the following statesunion territory

मोव क अतगत िनिलखत राोसघ शािसत दशो म स सबस बड़ा कौन सा ह

1 Andaman and Nicobar अमान और िनकोबार

2 Andhra Pradesh आ दश

3 West Bengal पिम बगाल

4 Gujarat गजरात

Correct Answer -

West Bengal पिम बगाल

The longitudinal transverse and surface waves in an earthquake originate from

भकप म दशातर अनथ और सतह तरग यहा उ होती ह

1 The focus on the surface of the Earth पी क सतह पर क -िबद म

2 The focus within the body of the Earth पी क भीतर क -िबद म

3 The epicenter within the body of the Earth पी क भीतर उपरक म

55)

56)

57)

4 The epicenter on the surface of the Earth पी क सतह पर उपरक म

Correct Answer -

The focus within the body of the Earth पी क भीतर क -िबद म

The down slope movement of material due to gravity is called______

गाकषण क कारण पदाथ की अनढाल गित को ______ कहा जाता ह

1 mass movement पदाथ सचलन

2 deposition िनप

3 erosion रण

4 volcanic movement ालामखीय सचलन

Correct Answer -

mass movement पदाथ सचलन

Shimla is cooler than Amritsar although both are on the same latitude This is because

िशमला म अमतसर स अिधक ठड ह हालािक दोनो समान अाश पर ह ऐसा ह ोिक

1 Shimla is at a greater height above sea level than Amritsar अमतसर की तलना म िशमला सम तल स अिधकऊचाई पर ह

2 Shimla is further north िशमला उर की ओर ह

3 Shimla is farther from the equator िशमला भम रखा स आग ह

4 Their longitudes differ उनकी दशातर रखाए िभ ह

Correct Answer -

Shimla is at a greater height above sea level than Amritsar अमतसर की तलना म िशमला सम तल स अिधकऊचाई पर ह

lsquoTempo of Urbanizationrsquo measures which of the following

lsquoशहरीकरण का टपोrsquo िनिलखत म स कौन सा उपाय ह

1 Speed of urbanizaon शहरीकरण की गित

2 None of the above इनम स कोई नही

3 Inequality of urbanizaon शहरीकरण की असमानता

4 Current level of urbanizaon शहरीकरण का वतमान र

Correct Answer -

58)

59)

60)

Speed of urbanizaon शहरीकरण की गित

Out of the following options choose the INCORRECT statement

िनिलखत िवको म स गलत कथन का चयन कर

1 The clear tracts in the equatorial region recover rapidly भम रखा म भभाग तजी स ठीक हो जात ह

2 The stable communities include a redwood forest a pine forest at high elevations

थर समदायो म एक रडवड वन उ ऊचाई पर एक दवदार वन शािमल ह

3 Any ecosystem moves towards maximum biomass and stability to survive

कोई भी पारथितकी त जीिवत रहन क िलए अिधकतम जवसहित और थरता की तरफ असर होता ह

4 Tropical rain forests near equator are stable ecosystems

भम रखा क पास उकिटबधीय वषा वन थर पारथितक त ह

Correct Answer -

The clear tracts in the equatorial region recover rapidly भम रखा म भभाग तजी स ठीक हो जात ह

Seasonal contrasts are maximum in मौसमी िवषमता अिधकतम ह

1 Mid latitudes म अाश म

2 Low attitudes िन अाश म

3 High latitudes उ अाश म

4 Subtropics उपोकिटबधीय म

Correct Answer -

Mid latitudes म अाश म

In India which type of forest among the following occupies the largest area

भारत म िनिलखत म स िकस कार क वन सबस बड़ा फल आािदत करत ह

1 Sub-tropical Dry Evergreen Forest उप उकिटबधीय श सदाबहार वन

2 Mountain Wet Temperate Forest पवतीय आ शीतो वन

3 Tropical Moist Deciduous Forest उकिटबधीय आ पणपाती वन

4 Tropical Wet Evergreen Forest उकिटबधीय आ सदाबहार वन

Correct Answer -

Tropical Moist Deciduous Forest उकिटबधीय आ पणपाती वन

61)

62)

63)

64)

What is the proportion of lsquoJuvenile Populationrsquo (0-14 years) in India as per 2011Census

2011 की जनगणना क अनसार भारत म जवनाइल पॉपलशन यानी िकशोर जनस या (0-14 वष) का अनपात ाह

1 3076 of total population कल जनस या का 3076

2 2764 of total population कल जनस या का 2764

3 2933 of total population कल जनस या का 2933

4 3354 of total population कल जनस या का 3354

Correct Answer -

3076 of total population कल जनस या का 3076

What is the Belfast famous for बलफा िकसक िलए मशर ह

1 Belt of cotton textile industry कपास व उोग क

2 Ship-building industry जहाज िनमाण उोग

3 Agricultural machinery किष उपकरण

4 Aero planes manufacturing वाययान िनमाण

Correct Answer -

Ship-building industry जहाज िनमाण उोग

What is the most important occupation in tropical monsoon lands

उकिटबधीय मॉनसन भिम म सबस महपण वसाय ा ह

1 Mining खनन

2 Cattle rearing मवशी पालन

3 Agriculture किष

4 Nomadic herding नोमािडक जड़ी-बिटया

Correct Answer -

Agriculture किष

What is the most important characteristics of the islands (Indian) located in the Arabian sea

अरब सागर म थत ीपो (भारतीय) की सबस महपण िवशषता ा ह

ी ो

65)

66)

67)

1 There are all of coral origins सभी कोरल मल क ह

2 There are all very small in size य सभी आकार म बत छोट ह

3 They have a very dry climate इनकी जलवाय बत श ह

4 They are extended parts of the mainland व महाीप क िवारत िह ह

Correct Answer -

There are all of coral origins सभी कोरल मल क ह

What do the basalt layers of the Deccan indicate डन की बसा परत ा इिगत करती ह

1 All of the above उपरो सभी

2 Huge volcanic eruptions in the distant past दरथ अतीत म िवशाल ालामखीय िवोट

3 The immense erosional activity of the rivers निदयो की िवशाल रण गितिविध

4 The influence of weathering मौसम का भाव

Correct Answer -

Huge volcanic eruptions in the distant past दरथ अतीत म िवशाल ालामखीय िवोट

In the structure of planet Earth below the mantle the core is mainly made up of_____

पी ह की सरचना म मटल क नीच कोर म प स______ स िनिमत होती ह

1 aluminium एमीिनयम

2 silicon िसिलकॉन

3 chromium ोिमयम

4 iron लोहा

Correct Answer -

iron लोहा

One of the major Mid Oceanic Ridge is found in मख म-महासागर चोिटयो म स एक ______ म पायाजाता ह

1 Mid Pacific Ocean म शात महासागर

2 Mid Atlantic Ocean म अटलािटक महासागर

3 Mid Indian Ocean म भारतीय महासागर

4 Mid Arctic Ocean म आक िटक महासागर

68)

69)

70)

71)

Correct Answer -

Mid Atlantic Ocean म अटलािटक महासागर

Magma that reaches the Earthrsquos surface and then solidifies is called________

मा जो पी की सतह तक पचती ह और िफर ठोस हो जाती ह ________कहलाती ह

1 quartz ाटज

2 lava लावा

3 granite नाइट

4 silicates िसिलकट

Correct Answer -

lava लावा

Isotherms are the lines of equal_______ समताप रखाए समान _______की रखाए होती ह

1 pressure दाब

2 temperature तापमान

3 rainfall वषा

4 height ऊचाई

Correct Answer -

temperature तापमान

Mark the correct sequence of passes in the Western Ghats from north to south

पिमी घाटो म उर स दिण तक दर क सही अनम को िचित कर

1 Thalghat Palghat Bhorghat थलगघाट पालघाट भोरघाट

2 Thalghat Bhorghat Palghat थलघाट भोरघाट पालघाट

3 Bhorghat Thalghat Palghat भोरघाट थलघाट पालघाट

4 Palghat Bhorghat Thalghat पालघाट भोरघाट थलघाट

Correct Answer -

Thalghat Bhorghat Palghat थलघाट भोरघाट पालघाट

Which of the following does not have influence over the climate in India

ि ि ि ी ी

72)

73)

िनिलखत म स िकसका भाव भारत की जलवाय पर नही पड़ता ह

1 Ocean currents सागर की लहर

2 Nearness to equator भम रखा स िनकटता

3 Monsoons मानसन

4 Presence of Indian ocean भारतीय महासागर की उपथित

Correct Answer -

Ocean currents सागर की लहर

Which of the following cloud types has the characteristics like vertical tall narrow and puffy

िनिलखत म स िकस कार क मघो म लबवत लबी सकीण और थलता जसी िवशषताए ह

1 Cumulonimbus तफानी मघ

2 Cumulus मघ पज

3 Cirrocumulus पाभ कपासी मघ

4 Nimbostratus वषारी मघ

Correct Answer -

Cumulus मघ पज

Which of the following statement is INCORRECT about Crude Birth Rate

िनिलखत स कौन सा कथन अशोिधत ज दर क बार म सही नही ह

1 It cannot be used for comparing fertility level between two countries with different population characteristics

इसका उपयोग िविभ जनसा िवशषताओ वाल दो दशो क बीच जनन र की तलना क िलए नही िकया जा सकता ह

2 It is a standardized measure of fertility

यह जनन मता का मानकीकत उपाय ह

3 It is effected by the age-sex composition of the population

यह आबादी की आय-िलग सरचना स भािवत होता ह

4 It is expressed per 1000 population in a given geographical unit

यह िकसी दी गई भौगोिलक इकाई म ित 1000 जनसा पर िकया जाता ह

Correct Answer -

It is a standardized measure of fertility

यह जनन मता का मानकीकत उपाय ह

74)

75)

76)

77)

Which of the following state in India experienced negative decadal growth rate during 2001 to 2011census

भारत म िनिलखत म स िकस रा म वष 2001 स 2011 की जनगणना क दौरान नकाराक िगरावट दर ई

1 Tripura िपरा

2 Nagaland नागालड

3 Haryana हरयाणा

4 Odisha ओिडसा

Correct Answer -

Nagaland नागालड

Which of the following is NOT a characteristic of peninsular rivers

िनिलखत म स कौन सी िवशषता ायीपीय निदयो म नही होती ह

1 Flow through shallow valleys उथल घािटयो क माम स वाह

2 Seasonal flow मौसमी वाह

3 Little erosional activity थोड़ी कटावदार गितिविध

4 Meandering tendency often shifting their beds घमावदार वि अर अपन तटो को थानातरत करना

Correct Answer -

Meandering tendency often shifting their beds घमावदार वि अर अपन तटो को थानातरत करना

Which of the following gases in the atmosphere absorbs heat from the Sunrsquos radiation and the Earthssurface

वायमडल म िनिलखत म स कौन सी गस सय क िविकरण और पी की सतह स ऊा को अवशोिषत करती ह

1 Neon िनयॉन

2 Carbon dioxide काबन डाइऑाइड

3 Argon आगन

4 Nitrogen नाइट ोजन

Correct Answer -

Carbon dioxide काबन डाइऑाइड

Which of the following kind of settlement pattern is found at the confluence of rivers

ि ि ि ि ो

78)

79)

80)

िनिलखत म स िकस कार का वथापन पटन निदयो क सगम पर पाया जाता ह

1 Triangular Paern िकोणीय पटन

2 Circular or Semi-Circular Paern परप या अध-परप पटन

3 Nebular Paern नबलर पटन

4 Star ndashShaped Paern ार-आकार का पटन

Correct Answer -

Triangular Paern िकोणीय पटन

Which one was not the objective of the Biosphere Reserve Projects launched by the UNESCO

यनो ारा श की गई सरित जवमडल परयोजनाओ का उ इनम स कौन सा नही था

1 To promote teaching and research िशण और अनसधान को बढ़ावा दना

2 To make agriculture sustainable किष को दीघकािलक बनाना

3 To conserve ecosystems पारथितक त को सरित करना

4 To conserve genetic diversity for a longtime लब समय तक अनवािशक िविवधता को सरित करना

Correct Answer -

To make agriculture sustainable किष को दीघकािलक बनाना

Which region of the Earth surface receives the highest amount of insulation

पी सतह का कौन सा तापावरोधन की उतम माा ा करता ह

1 Land mass थलखड

2 Savannah region सवाना

3 Water bodies जल िनकाय

4 Tropical desert उकिटबधीय रिगान

Correct Answer -

Tropical desert उकिटबधीय रिगान

Which one of the following is not a biodiversity hotspot

िनिलखत म स कौन सा जव िविवधता का म जगह नही ह

1 Eastern Himalaya पव िहमालय

2 Eastern Ghats पव घाट

81)

82)

83)

3 Indo-Myanmar भारत-ामार

4 Westerm Ghats पिमी घाट

Correct Answer -

Eastern Ghats पव घाट

Which one of the following is NOT a part of the World Network of Biosphere Reserves based on theUNESCO Man and Biosphere Programme

यनो मन और बायोीयर कायम क आधार पर िनिलखत म स कौन बायोीयर रजव क िव नटवक कािहा नही ह

1 Gulf of Mannar मार की खाड़ी

2 Seshachalam शषाचलम

3 Sunderban सदरबन

4 Nilgiri नीलिगर

Correct Answer -

Seshachalam शषाचलम

Which one of the following is an example of ldquodesert vegetationrdquo

िनिलखत म स कौन मथलीय वनित का एक उदाहरण ह

1 Mosses and lichens दलदल और शवाल

2 Temperate grassland समशीतो घास क मदान

3 Coniferous forest शकधारी वन

4 Acacia and cactus एकािसया और कस

Correct Answer -

Acacia and cactus एकािसया और कस

Which one of the following reflects more sunlight िनिलखत म स कौन सा सय की रोशनी को अिधकपरावितत करता ह

1 Paddy crop land धान फसल भिम

2 Land covered with fresh snow ताजा बफ स आािदत भिम

3 Sand desert रतीली रिगान

4 Prairie land यरी भिम

84)

85)

86)

87)

Correct Answer -

Land covered with fresh snow ताजा बफ स आािदत भिम

Which layer of the atmosphere is in contact with the surface of the earthrsquos oceans

वायमडल की कौन सी परत पी क महासागरो की सतह क सपक म ह

1 Stratosphere समताप मडल

2 Mesosphere म मडल

3 Hydrosphere जलमडल

4 Troposphere ोभ मडल

Correct Answer -

Troposphere ोभ मडल

Mediterranean Sea is a border of which of the following countries भम सागर िनिलखत दशो म सिकसकी सीमा ह

1 None of these इनम स कोई नही

2 Iraq इराक

3 Lebanon लबनान

4 Jordan जॉडन

Correct Answer -

Lebanon लबनान

Benguela ocean currents are found along which coast बगएला महासागर धाराए िकस तट क साथ पायीजाती ह

1 East Coast of South America दिण अमरका क पव तट

2 East Coast of Africa अीका क पव तट

3 West Coast of South America दिण अमरका क पिमी तट

4 West Coast of Africa अीका क पिमी तट

Correct Answer -

West Coast of Africa अीका क पिमी तट

88)

89)

90)

Due to tension a block of land on one side being pushed up or upthrown relative to the downthrown blockis referred as

तनाव क कारण नीच फ क ए खड क साप भिम का एक खड एक ओर स ऊपर धकला जाता ह या ऊपर की ओरफ का जाता ह यह _____ क प म सदिभत ह

1 Thrust fault प श

2 Normal fault सामा श

3 Reverse fault म श

4 Strike slip fault नितलब सपण श

Correct Answer -

Normal fault सामा श

Inter-tropical doldrums is a zone of ______ अतर-उकिटबधीय डोलड ______ का एक ह

1 Frontolysis टोलायिसस

2 Convergence अिभसरण

3 Inter-tropical divergence zone अतर-उकिटबधीय िवचलन

4 Local wind थानीय वाय

Correct Answer -

Convergence अिभसरण

The Horse Latitudes are regions located at about _____ north and south of the equator

हॉस अाश भम रखा क उर और दिण म लगभग _____ पर थत ह

1 30ndash60 degree Latitude 30-60 िडी अाश

2 0ndash5 degree Latitude 0-5 िडी अाश

3 30 degree Latitude 30 िडी अाश

4 60ndash90 degree Latitude 60-90 िडी अाश

Correct Answer -

30 degree Latitude 30 िडी अाश

Generally evaporation is high over which part of the Earth

आम तौर पर पी क िकस भाग पर वाीकरण अिधक होता ह

1 Equatorial maritime भमवत समीय ी ी

91)

92)

2 Equatorial continental भमवत महाीपीय

3 Polar maritime वीय समीय

4 Polar continental वीय महाीपीय

Correct Answer -

Equatorial maritime भमवत समीय

A very high temperature during summer in north western India leads to what type of climaticcondition in south

उर पिमी भारत म गम क दौरान बत अिधक तापमान होन क कारण दिण म िकस कार की जलवाय थितउ करता ह

1 Depression over arabian sea अरब सागर पर अवनमन

2 Failure monsoon मानसन िवफलता

3 Successful monsoon मानसन सफलता

4 Cyclones चवात

Correct Answer -

Successful monsoon मानसन सफलता

Lightning and thunder are the resultant effect when तिड़त और गजन परणामी भाव ह जब

1 Two massive clouds hit powerfully each other first lightning is produced and later sound is produced

दो बड़ बादल एक दसर स शशाली ढग स टकरात ह पहल आकाशीय िवदयत उ होता ह और बाद म िन उहोती ह

2 Two massive clouds come into contact with the powerful wind collision this results into first sound and thenlightning

दो बड़ बादल शशाली पवन सघ क सपक म आत ह इसका परणामप पहल िन और िफर आकाशीय िवदयतउ होता ह

3 None of the above उपरो म स कोई भी नही

4 A high density cloud contains positively and negatively charged electric ions and when this interacts light andsound are simultaneously produced

एक उ घन बादल म धनाक और ऋणाक आविशत िवदयत आयन होत ह और जब यह परर भाव डालत ह तोकाश और िन एक साथ उािदत होती ह

Correct Answer -

A high density cloud contains positively and negatively charged electric ions and when this interacts light andsound are simultaneously produced

औ ि ि ो औ ो

93)

94)

95)

एक उ घन बादल म धनाक और ऋणाक आविशत िवदयत आयन होत ह और जब यह परर भाव डालत ह तोकाश और िन एक साथ उािदत होती ह

Doon Valley is able to grow rice because दन घाटी चावल उगान म सम ह ोिक

1 Other crops cannot be grown वहा अ फसलो को उगाया नही जा सकता ह

2 People in the valley are rice eaters घाटी म लोग चावल खान वाल ह

3 There is a huge export demand of rice वहा चावल की भारी िनयात माग ह

4 It has warm summer and snow melt waters for irrigation

वहा गिमया गम होती ह िसचाई क िलए बफ का िपघला आ पानी होता ह

Correct Answer -

It has warm summer and snow melt waters for irrigation

वहा गिमया गम होती ह िसचाई क िलए बफ का िपघला आ पानी होता ह

CANCELLED

In the geological time scale the Mesozoic Era DOES NOT contains which of the following periods

भगभय समय पमान पर मजीवी यग म िन कालो म स कौन नही ह

1 Triassic ट ाइऐिसक

2 Jurassic जरिसक

3 Cretaceous चाकमय

4 Carboniferous काबनी

Correct Answer -

Carboniferous काबनी

96)

1 P-3 Q-4 R-2 S-1

2 P-3 Q-4 R-1 S-2

3 P-3 Q-4 R-1 S-2

4 P-4 Q-3 R-2 S-1

Correct Answer -

P-4 Q-3 R-2 S-1

1 P-3 Q-1 R-4 S-2

2 P-3 Q-4 R-1 S-2

3 P-3 Q-2 R-4 S-1

97)

98)

4 P-2 Q-1 R-4 S-3

Correct Answer -

P-3 Q-4 R-1 S-2

1 P-3 Q-1 R-4 S-2

2 P-2 Q-3 R-4 S-1

3 P-2 Q-1 R-3 S-4

4 P-4 Q-2 R-1 S-3

Correct Answer -

P-3 Q-1 R-4 S-2

99)

100)

1 P-3 Q-2 R-4 S-1

2 P-1 Q-2 R-3 S-4

3 P-2 Q-3 R-1 S-4

4 P-4 Q-3 R-2 S-1

Correct Answer -

P-2 Q-3 R-1 S-4

ldquoHuman geography is the study of changing relationship between the unresting man and the unstableearthrdquo was defined by

lsquolsquoमानव भगोल ाकल आदमी और अथर पी क बीच सबध परवतन का अयन हrdquo ______ ारा परभािषत िकया गयाथा

1 J Brunches ज चस

2 EC Semple ईसी सल

3 HJ Mackinder एच ज मिकदर

4 PV Blache पीवी च

Correct Answer -

EC Semple ईसी सल

Sedimentary rocks are finally and ultimately derived from the____________

अवसादी चान अततः ________ स ा की जाती ह

1 action of earth movements पी की गितिविधयो

2 marine deposit समी िनप

3 weathering of metamorphic rocks पातरत चानो क अपय

4 weathering of igneous rocks आय चानो क अपय

Correct Answer -

weathering of igneous rocks आय चानो क अपय

Page 8: High School Teacher Eligibility Test- BOARD PROFESSIONAL ...peb.mp.gov.in/results/RESULT_18/HST_RES18/Final_anwser_key/HST… · M a ndl a / मंड ल ... Under the Madhya Pradesh

6)

7)

8)

Under the Madhya Pradesh Ladli Laxmi Yojana the beneficiary is paid the total amount on attaining theage of ___ मदश लाडली ली योजना क तहत लाभाथ की आय ___ वष परी होन पर कल रािश का भगतान िकयाजाता ह

1 18

2 21

3 23

4 16

Correct Answer -

21

India has won the gold medal in hockey ______times at the Olympics

ओलिपक खलो म भारत न हॉकी म ण पदक _____ बार जीता ह

1 9

2 7

3 8

4 6

Correct Answer -

8

Where is the Madhya Pradesh Tribal Museum located at

म दश जनजातीय सहालय कहा थत ह

1 Indore इदौर

2 Jhabua झबआ

3 Khargone खरगोन

4 Bhopal भोपाल

Correct Answer -

Bhopal भोपाल

The fictional boy character of the lsquoThe Jungle Bookrsquo by author Rudyard Kipling was

लखक डयाड िकपिलग ारा lsquoद जगल बकrsquo का कािनक बालक चर था

1 Kaa का

2 Mowgli मोगली

9)

10)

1)

3 Shere Khan शर खान

4 Bagheera बघीरा

Correct Answer -

Mowgli मोगली

The recipient of which one of the following awards given by Madhya Pradesh Government gets the highest cashprize

िनिलखत परारो म स िकसक ाकता को म दश सरकार ारा उतम नकद परार िदया जाता ह

1 Mahatma Gandhi award महाा गाधी परार

2 Shikar Award िशखर परार

3 Sharad Joshi Award शरद जोशी परार

4 Kishore Kumar Award िकशोर कमार परार

Correct Answer -

Mahatma Gandhi award महाा गाधी परार

Who among the following was the Trinidadian writer of Indian descent

िनिलखत म स भारतीय मल क ििनदािदयन लखक कौन थ

1 RK Narayan आर क नारायण

2 Anita Desai अनीता दसाई

3 Rudyard Kipling डयाड िकपिलग

4 VS Naipaul वीएस नायपॉल

Correct Answer -

VS Naipaul वीएस नायपॉल

Topic- GENERAL REASONING

Find the missing number ल त स या ात कर

4 196 16 169 ____ 144 64

1 81

2 36

3 32

4 21

2)

3)

4)

Correct Answer -

36

Find the average औसत ात कर

25 35 45 55 65 amp 75

1 48

2 50

3 47

4 49

Correct Answer -

50

The below series uses a sequence of alphabets and numbers Identify the incorrect combination

नीच दी गई खला अरो एव साओ का अनम योग करती ह गलत सयोजन पहचान

(i) FT85DF77ER

(ii) FT85DF77ER

(iii) FT85DE77ER

(iv) FT85DF77ER

1 ii

2 iv

3 i

4 iii

Correct Answer -

iii

A and B together can complete a piece of work in 10 days and B alone can complete the same work in 20days In how many days can A alone complete the work

A और B एक साथ काम क एक भाग को 10 िदनो म समा त करत ह और B अकल उसी काम को 20 िदनो म समा त करता हिकतन िदनो म A अकल उस काम को समा त कर सकता ह

1 20 days 20 िदन

2 30 days 30 िदन

3 10 days 10 िदन

4 25 days 25 िदन

5)

6)

Correct Answer -

20 days 20 िदन

Which one of the following four addresses is NOT EXACTLY same as the one given below

िनिलखत िदए गए चार पतो म स नीच िदए गए पत क ठीक समान कौन सा एक नही ह

Ground Floor 59 Goulburn St

Sydney NSW 2000 Australia

+61 (02) 8987 3700

(i) Ground Floor 59 Goulburn St

Sydney NSW 2000 Australia

+61 (02) 8987 3700

(ii) Ground Floor 59 Goulburn St

Sydney NSW 2000 Australia

+61 (02) 8987 3700

(iii) Ground Floor 59 Govlburn St

Sqdney NSW 2000 Australia

+61 (02) 8987 3700

(iv) Ground Floor 59 Goulburn St

Sydney NSW 2000 Australia

+61 (02) 8987 3700

1 ii

2 iv

3 i

4 iii

Correct Answer -

iii

In a group of 75 people 32 of them like cold drink 56 of them like general water and each person likesat least one of the two drinks How many people like both

75 लोगो क समह म उनम स 32 को को िड क पसद ह उनम स 56 को सामा जल पसद ह और क को दो म स कम स कम एक पय पसद ह िकतन लोगो को दोनो पसद ह

1 11

2 13

7)

1)

2)

3 12

4 14

Correct Answer -

13

If A = 1 FAT = 27 then FAITH =

यिद A = 1 FAT = 27 तो FAITH =

1 41

2 40

3 42

4 44

Correct Answer -

44

Topic- PEDAGOGY

CANCELLED

Basic education became a major initiative because of the efforts of

ाथिमक िशा िन न क यास स एक मह वपण कायम बन गई

1 Dr Radhakrishnan डॉ राधाक णन

2 Dr Rajendra Prasad डॉ राज साद

3 Mahatma Gandhi महा मा गाधी

4 Rabindranath Tagore रबी नाथ टगोर

Correct Answer -

Mahatma Gandhi महा मा गाधी

The ldquoclause of phraserdquo is an unit of perception of लॉज ऑफ ज िन न क धारणा क एक इकाई ह

1 Image छिव

2 Concept अवधारणा

3 Language भाषा

4 Thought िवचार

Correct Answer -

3)

4)

5)

6)

Language भाषा

Which method is most suitable to study communication process among students

छाो क बीच सचार िया का अयन करन क िलए कौन सी िविध सबस उपय ह

1 Case Study मामल का अयन (कस टडी)

2 Systematic Observation वथत अवलोकन

3 Experimental Method योगाक िविध

4 Introspection आ-िनरीण (इट ोस शन)

Correct Answer -

Systematic Observation वथत अवलोकन

Genes in a human being are located in मानवो म जीन इनम थत होता ह

1 cytoplasm कोिशका (साइटो ला म)

2 ribosomes राइबोसोम

3 cell membranes कोिशका िझी

4 chromosomes गणस (ोमोसोम)

Correct Answer -

chromosomes गणस (ोमोसोम)

With smaller classes teachers are much more able to ____________

छोटी काओ क साथ िशक ____________ म अिधक सम होत ह

1 Go slow while teaching िशण क दौरान धीमी गित स जान

2 Narrate more personal experiences अिधक गत अनभवो को बतान

3 Adapt instruction to individual differences among students छाो क बीच गत मतभदो क िलए अनकल िनदश दन म

4 Make use of the extra space for extra-curricular activities पातर गितिविधयो क िलए अितर जगह का उपयोग करन

Correct Answer -

Adapt instruction to individual differences among students छाो क बीच गत मतभदो क िलए अनकल िनदश दन म

While engaging in a task the child gets bored This is a sign of

एक काम म होन पर बा ऊब जाता ह यह िन का सकत ह

1 the task requiring a professional approach काय को ावसाियक िकोण की आवकता ह

7)

8)

9)

2 the task becoming mechanically repetitive काय यािक प स दोहराव वाला ह

3 the child not being intelligent बा बमान नही ह

4 the child being incapable of learning बा सीखन म असमथ ह

Correct Answer -

the task becoming mechanically repetitive काय यािक प स दोहराव वाला ह

Educational Psychologists are more concerned with the learning in __________

शिणक मनोवािनक __________ म अिधगम क साथ अिधक िचितत होत ह

1 Formal environment औपचारक वातावरण

2 Informal environment अनौपचारक वातावरण

3 Physical environment भौितक वातावरण

4 Social environment सामािजक वातावरण

Correct Answer -

Formal environment औपचारक वातावरण

Special needs education is the type of education िवशष ज़रतो वाली िशा वह िशा होती ह जो

1 Given to person with disabilities अम य को दी जाती ह

2 Given to people from remote areas दर थ ो क लोगो को दी जाती ह

3 Provided to intelligent people बमान लोगो को दी जाती ह

4 Established by colonial masters औपिनविशक मखयाओ ारा थािपत की गई ह

Correct Answer -

Given to person with disabilities अम य को दी जाती ह

The Stanford-Binet scale of intelligence was first published in the year

ब क नफोड-िबनट कल को इस वष म पहली बार कािशत िकया गया था

1 1916

2 1903

3 1908

4 1900

Correct Answer -

1916

10)

11)

12)

13)

The term lsquofictional finalismrsquo was propounded by

पद lsquoकत योजनवादrsquo (िफ शनल फाइनिल म) इनक ारा ितपािदत िकया गया था

1 Skinner नर

2 Freud ायड

3 Adler एडलर

4 Pavlov पावलोव

Correct Answer -

Adler एडलर

Rational Emotive Behavior Therapy was propounded by

तक सगत भावनाक वहार थरपी िन क ारा ितपािदत की गई थी

1 Carl Jung काल यग

2 Carl Rogers काल रोजस

3 Aaron Beck आरोन बक

4 Albert Ellis अट एिलस

Correct Answer -

Albert Ellis अट एिलस

Which of the following indicates the quality of education in a school

िनिलखत म स या एक कल म िशा की गणव ता का सकतक ह

1 Text-books and Teaching-learning material पा-प तक तथा िशण व सीखन स सबिधत सामी

2 Infrastructural facilities at the school कल म आधारभत ढाच स सबिधत सिवधाए

3 Student achievement level िवािथयो का उपल तर

4 Classroom systems का की यव था

Correct Answer -

Student achievement level िवािथयो का उपल तर

Which of the following teachers can be identified with authoritarian teaching style

िनिलखत म स कौन सा िशक अिधकारवादी िशण शली क साथ पहचाना जा सकता ह

1 Laissez-faire teacher अब ध िशक

14)

15)

2 Democratic teacher लोकतीय िशक

3 Indifferent teacher िन प िशक

4 Direct instruction teacher िनदश िशक

Correct Answer -

Direct instruction teacher िनदश िशक

Who was the pioneer of classical conditioning

िचरितित ानकलन ( लािसकल कडीशिनग) क वतक कौन थ

1 Skinner नर

2 Pavlov पावलोव

3 Kohler कोहलर

4 Freud ायड

Correct Answer -

Pavlov पावलोव

Which of the following are true with reference to short term memory

1 Limited capacity

2 Brief storage of information

3 Unlimited capacity

4 Duration of storage less than twenty seconds

अ पकािलक मित क सदभ म िन न म स या स य ह

1 सीिमत मता

2 सचना का सि त भडारण

3 असीिमत मता

4 बीस सकड स कम भडारण की अविध

1 3 and 4 3 और 4

2 2 and 4 2 और 4

3 13 and 4 13 और 4

4 12 and 4 12 और 4

Correct Answer -

12 and 4 12 और 4

1)

2)

3)

4)

Topic- GEOGRAPHY

Which of the following ranges of population size is used to define Class-III city by Indian Census

भारतीय जनगणना ारा वग-III शहर को परभािषत करन क िलए िनिलखत म स िकस रज क जनसा आकारका उपयोग िकया जाता ह

1 20 000 to 49999 20 000 स 49999

2 30000 to 59999 30000 स 59999

3 24000 to 54999 24000 स 54999

4 50000 to 99999 50000 स 99999

Correct Answer -

20 000 to 49999 20 000 स 49999

Carbonaceous rocks which produce coal and oil belong to the category of rocks called_______

कोयल और तल का उादन करन वाली काबनय चान ______ नामक चानो की णी स सबिधत ह

1 metamorphic पातरत

2 sedimentary अवसादी

3 inorganic अजिवक

4 igneous आय

Correct Answer -

sedimentary अवसादी

The ruhr-complex is a major industrial centre in र-परसर िन का एक मख औोिगक क ह

1 North America उरी अमरका

2 Russia स

3 Germany जमनी

4 Europe यरोप

Correct Answer -

Germany जमनी

The term lsquoRegurrsquo refers to श lsquoरगरrsquo ______ स सबिधत ह

1 Deltaic alluvial soils डा जलोढ़ िमी

ि ी

5)

6)

2 Laterite soils लटराइट िमी

3 Red and yellow soils लाल और पीली िमी

4 Black cotton soils काली कपास िमी

Correct Answer -

Black cotton soils काली कपास िमी

Read the given statements and answer which of the following options isare correct

1 Lower the pressure greater the atmospheric disturbance

2 Air move from higher to low pressure

िदए गए कथन को पढ़ और उर द िक िन म स कौन सास िवक सही ह

1 िजतना दाब कम होगा वायमडलीय बाधाए उतनी अिधक होगी

2 वाय उ स िन दाब की ओर गित करती ह

1 Both Statements 1 and 2 are correct दोनो कथन 1 और 2 सही ह

2 Both Statements 1 and 2 are wrong दोनो कथन 1 और 2 गलत ह

3 Statement 1 is wrong and only Statement 2 is correct कथन 1 गलत ह और कवल कथन 2 सही ह

4 Statement 1 is correct and Statement 2 is wrong कथन 1 सही ह और कथन 2 गलत ह

Correct Answer -

Both Statements 1 and 2 are correct दोनो कथन 1 और 2 सही ह

CANCELLED

1 4 1 2 and 3 4 1 2 और 3

2 4 1 3 and 2 4 1 3 और 2

3 1 4 2 and 3 1 4 2 और 3

4 1 4 3 and 2 1 4 3 और 2

Correct Answer -

7)

8)

9)

1 4 3 and 2 1 4 3 और 2

CANCELLED

Karl Pearsonrsquos correlation co-efficient is काल िपयसन का सहसबध गणाक ह

1 Arithmec mean समार मा

2 Geometric mean गणोर मा

3 None of these इनम स कोई नही

4 Harmonic mean हराक मा

Correct Answer -

Geometric mean गणोर मा

CANCELLED

ldquoEach day is more or less the same the morning is clear and bright with a sea breeze as the Sun climbshigh in the sky heat mounts up dark clouds form then rain comes with thunder lighting But rain is soonoverrdquo Which of the following regions is described in the above passage

ldquoक िदन समान स अिधक या कम होता ह सम की हवा क साथ और उल सबह होती ह जस सयआकाश म ऊचा चढ़ता ह गम बढ़ जाती ह काल बादल बनत ह िफर िबजली क साथ बारश आती ह लिकनबारश जी ख हो जाती हlsquorsquo उपरो पा म िनिलखत म स िकन ो का वणन िकया गया ह

1 Equatorial भमरखीय

2 Equatorial भमरखीय

3 Savannah सवाना

4 Mediterranean आातरक (भमसागरीय)

5 Mediterranean आातरक (भमसागरीय)

6 Monsoon मानसन

7 Monsoon मानसन

Correct Answer -

Equatorial भमरखीय

Equatorial भमरखीय

CANCELLED

In which epoch of the geological history of the Earth dinosaurs reached their largest size

पी क भगभय इितहास क िकस यग म डायनासोर अपन सबस बड़ आकार तक पचि

10)

11)

1 Triassic ट ायिसक

2 Jurassic जरािसक

3 Cretaceous ीटशस

4 Permian पिमयन

Correct Answer -

Jurassic जरािसक

CANCELLED

A spring tide would occur in which of the following conditions

िनिलखत म स िकन थितयो म ार-भाटा आता ह

1 When the Sun Moon and Earth are in a straight line

जब सय चमा और पी एक सीधी रखा म होत ह

2 When the Moon and Earth are in right angle to each other

जब चमा और पी एक-दसर क दािहन कोण म होत ह

3 When the Earth and Moon are in right angle to the Sun

जब पी और चमा सय क दािहन कोण म होत ह

4 When the Sun and Moon are in right angle to each other

जब सय और चमा एक-दसर क दािहन कोण म होत ह

Correct Answer -

When the Sun Moon and Earth are in a straight line

जब सय चमा और पी एक सीधी रखा म होत ह

CANCELLED

An observe on the Earthrsquos surface always sees the same face of the moon because

एक पयवक को पी की सतह स हमशा चाद का एक ही फलक िदखाई दता ह ोिक

1 Its path of revolution around the earth is the same as that of the earth around the Sun

इसका पी क चारो ओर परमण का माग सय क चारो ओर पी क समान ही ह

2 Its period of revolution around the Earth is the same as its period of rotation around its own axis

इसकी पी क चारो ओर परमण की अविध उसकी अपनी धरी क चारो ओर घणन की अविध क समान ह

3 Its period of rotation is the same as that of the Earth इसकी घणन की अविध पी क समान ह

ी ि ी ी

12)

13)

14)

4 Its direct of rotation is the same as that of Earth घणन की िदशा पी क समान ही ह

Correct Answer -

Its period of revolution around the Earth is the same as its period of rotation around its own axis

इसकी पी क चारो ओर परमण की अविध उसकी अपनी धरी क चारो ओर घणन की अविध क समान ह

CANCELLED

The pebbles that are faceted by the sand-blasting and shaped polished by the wind abrasions are known as

पवन अपघषन ारा पॉिलश रत-िवोिटत और साच म ढला ककड़ ___________ क प म जाना जाता ह

1 Dreikanter िकोणक

2 Pediments िकोिनका

3 Inselberg इलबग

4 Dunes टीबा

Correct Answer -

Dreikanter िकोणक

CANCELLED

Astronomical unit is the average distance between खगोलीय इकाई ______ क बीच की औसत दरी ह

1 Earth and Mars पी और मगल

2 Earth and mercury पी और बध

3 Earth and moon पी और चमा

4 Earth and Sun पी और सय

Correct Answer -

Earth and Sun पी और सय

During cold weather season in the northern plains there will be an inflow of cyclonic disturbancesfrom the _________ directions

शीत मौसम क दौरान उरी मदानी इलाको म _________ िदशाओ स चवात सबधी गड़बड़ी का अतवाह होगा

1 East and Northwest पव और उरपिम

2 East and Northeast पव और पवर

3 West and East पिम और पव

ि औ ि

15)

16)

17)

4 West and Northwest पिम और उरपिम

Correct Answer -

West and Northwest पिम और उरपिम

During an earth quake the velocity of the body waves will________ along with the increase in densityof the material it is passing through

भकप क दौरान लहरो क ऊपरी भाग का वग घन म व क साथ-साथ ________ जो इसस गजरन वाली वको आग बढाएगी

1 not change नही बदलगा

2 increase initially and then decrease शआत म बढ़गा और िफर घटगा

3 increase बढ़गा

4 decrease घटगा

Correct Answer -

increase बढ़गा

The Clouded Leopard National park is situated in which of the following states

िनिलखत म िकस रा म धिमल तदआ रा ीय उान (ाउडड लपड नशनल पाक ) थत ह

1 Tripura िपरा

2 Uttar Pradesh उर दश

3 Assam असम

4 Mizoram िमजोरम

Correct Answer -

Tripura िपरा

Usually the land surfaces are heated more quickly than the water surfaces because _____________

आम तौर पर जल सतहो की तलना म भिम सतह अिधक तजी स गम होती ह ोिक _____________ ह

1 the specific heat of water is higher than land पानी की िविश ऊा भिम स अिधक

2 the specific heat of water is lesser than land पानी की िविश ऊा भिम स कम होती

3 the latent heat of water is higher than the land पानी की अतिनिहत ऊा भिम स अिधक

4 the land reflects more heat radiation than water भिम पानी की तलना म अिधक ऊा क िविकरण को पराविततकरती

18)

19)

20)

21)

Correct Answer -

the specific heat of water is higher than land पानी की िविश ऊा भिम स अिधक

The longest shore-line is along the state of सबस लबी समतटीय रखा िन रा क साथ ह

1 Maharashtra महारा

2 Orissa उड़ीसा

3 Kerala करल

4 Gujarat गजरात

Correct Answer -

Gujarat गजरात

The position when the Earth is farthest from the Sun is known as

जब पी सय स सबस दर होती ह तो उस थित को िन नाम स जाना जाता ह

1 Perihelion उपसौर

2 Vernal Equinox बसत िवषव

3 Aphelion अपसौर

4 Autumnal Equinox शराल िवषव

Correct Answer -

Aphelion अपसौर

The seasonal reversal of winds is the typical characteristic of

हवाओ का मौसमी परवतन ______ की सामा िवशषता ह

1 Mediterranean climates only कवल भमसागरीय जलवाय

2 All of the above climates उपय सभी मौसम

3 Monsoon climate only कवल मानसन जलवाय

4 Equatorial climate only कवल भमरखीय जलवाय

Correct Answer -

Monsoon climate only कवल मानसन जलवाय

In _________ rocks the minerals will occurs in beds or layers

ो ि ो ो

22)

23)

24)

______ चानो म खिनज तल या परतो म होत ह

1 metamorphic कायातरत

2 igneous and metamorphic आय और कायातरत

3 igneous आय

4 sedimentary अवसादी

Correct Answer -

sedimentary अवसादी

Black soil is ideal for the cultivation of cotton as कपास की खती क िलए काली िमी आदश ह ोिक

1 Its colour is black यह काली होती ह

2 It is found on plateau regions यह पठार ो म पायी जाती ह

3 It is made up of lava यह लावा स बनी होती ह

4 It can retain moisture यह नमी को बरकरार रख सकती ह

Correct Answer -

It can retain moisture यह नमी को बरकरार रख सकती ह

The National Survey and Mapping Organization of the country works under the Department of___________

दश का रा ीय सवण और मानिचण सगठन ___________ िवभाग क अतगत काय करता ह

1 Space अतर

2 Science and Technology िवान और तकनीक

3 Culture सित

4 Tourism पयटन

Correct Answer -

Science and Technology िवान और तकनीक

Palk strait separates India from पाक जलडमम भारत स _____ को अलग करता ह

1 Pakistan पािकान

2 Andaman Island अडमान ीप

3 China चीन

25)

26)

27)

4 Sri Lanka ीलका

Correct Answer -

Sri Lanka ीलका

Which among the following state is the major producer of Bauxite in India

िनिलखत म स कौन सा रा भारत म बॉाइट का मख उादक ह

1 Madhya Pradesh मदश

2 Rajasthan राजथान

3 Goa गोवा

4 Orissa उड़ीसा

Correct Answer -

Orissa उड़ीसा

Which of the following states DOES NOT share border with Chhattisgarh

िनिलखत म स कौन सा रा छीसगढ़ क साथ सीमा साझा नही करता ह

1 Telangana तलगाना

2 Uttar Pradesh उर दश

3 Bihar िबहार

4 Andhra Pradesh आ दश

Correct Answer -

Bihar िबहार

Which of the following statements is INCORRECT with respect to parallels of latitudes

अाश क समानातरो क सबध म िन निलखत म स कौन सा कथन गलत ह

1 A line joining places of equal latitude is known as parallel of largest

समान अाश क थानो को जोड़न वाली रखा को िवशालतम क समानातर क प म जाना जाता ह

2 They stat from equator and run parallels to it

व भम रखा स ारभ होत ह और इसक समानातर चलत ह

3 All parallels are equal in length सभी समातर लबाई म समान ह

4 All parallels are drawn as circles on the globe ोब पर सभी समानातर वो क प म खीच जात ह

28)

29)

30)

31)

Correct Answer -

All parallels are equal in length सभी समातर लबाई म समान ह

Which of the following Indian states is also known as a lsquoLand of Red river and Blue Hillsrsquo

िनिलखत म स िकस भारतीय रा को लाल नदी और नीली पहािड़यो की भिम क नाम स जाना जाता ह

1 Uttarkhand उराखड

2 Assam असम

3 Meghalaya मघालय

4 Arunachal Pradesh अणाचल दश

Correct Answer -

Assam असम

In spatial analysis of settlement Rn = 215 indicates which type of settlement arrangement

िनपटान क थािनक िवषण म Rn = 215 यह इिगत करता ह िक िकस कार की िनपटान वथा ह

1 Uniform यिनफॉम

2 Semi-Clustered समी- ल टर

3 Clustered ल टर

4 Random रडम

Correct Answer -

Uniform यिनफॉम

Who are known as the lsquoYellow Peoplersquo lsquoयलो पीपलrsquo क प म कौन जाना जाता ह

1 Mongoloids मोगोलोइडस

2 Nigroids नीोइडस

3 Australoids ऑ लॉइडस

4 Caucasoids कॉकसोइडस

Correct Answer -

Mongoloids मोगोलोइडस

ि ि ो ौ ि

32)

33)

34)

Who publishes the topographical map of India भारत क थलाकितक मानिच को कौन कािशत करता ह

1 Geographical Survey of India भारत का भौगोिलक सवण

2 Government of India भारत सरकार

3 Geological Survey of India भारत क भगभय सवण

4 Survey of India भारत का सवण

Correct Answer -

Survey of India भारत का सवण

Who among the following claimed geography to be the lsquoEcology of Manrsquo

िनिलखत म स िकसन भगोल को मन का पारथितकी कहा ह

1 Alfred Hener अड हटनर

2 Vidal-de la Blache वाइडल-िड लॉ ॉश

3 Oo Schluter ओटो टर

4 Harlan Barrow हरलन बारो

Correct Answer -

Harlan Barrow हरलन बारो

Who among the following is regarded as the founder of humanistic approach in geography

िनिलखत म स िकस भगोल म मानवतावादी िकोण का सथापक माना जाता ह

1 William Bunge िविलयम बग

2 Yi-Fu-Tuan यी-फ- यान

3 Brain JL Berry न जएल बरी

4 Richard Peet रचड पीट

Correct Answer -

Yi-Fu-Tuan यी-फ- यान

Who prepared Lorenz curve लोरज व िकसन तयार िकया

1 Geddes गडस

2 None of these इनम स कोई नही

3 Griffith Taylor ििफथ टलर

35)

36)

37)

4 Max U Lorenz म य लोरज

Correct Answer -

Max U Lorenz म य लोरज

Gulf Streams are the currents of which of the following oceans

खाड़ी की धाराए िनिलखत महासागरो म स िकसकी धाराए ह

1 North Atlantic Ocean उरी अटलािटक महासागर

2 North Pacific Ocean उरी शात महासागर

3 Arabian Sea अरब सागर

4 South Pacific Ocean दिण शात महासागर

Correct Answer -

North Atlantic Ocean उरी अटलािटक महासागर

Disintegration wearing away and removal of rock material is generally referred as

िशला पदाथ (रॉक सामी) का टटना िमटना और हटना आमतौर पर ________ क प म सदिभत िकया जाता ह

1 Shattering िवसकारक

2 Denudation अनाादन

3 Fault श

4 Decomposition िवयोजन

Correct Answer -

Denudation अनाादन

Variations in the length of day time and night from season to season are due to

मौसम स मौसम परवतन पर िदन क समय और रात क समय की अविध म िभताए िन कारण स होती ह

1 The Earthrsquos revolution round the Sun in an elliptical manner पी का दीघवाकार तरीक स सय क चारो घणन

2 The Earthrsquos rotation on its axis पी का इसकी धरी पर घणन

3 Revolution of the Earth on a tilted axis नत अ पर पी का घणन

4 Latitudinal position of the place थान की अाश थित

Correct Answer -

Revolution of the Earth on a tilted axis नत अ पर पी का घणन

38)

39)

40)

Point out the correct sequence of mountain ranges from north to south

उर स दिण तक पवत खलाओ क सही अनम को इिगत कर

1 Great Himalaya Middle Himalaya Outer Himalaya Trans Himalaya

महान िहमालय म िहमालय बा िहमालय परा िहमालय

2 Middle Himalaya Great Himalaya Trans Himalaya Outer Himalaya

म िहमालय महान िहमालय परा िहमालय बा िहमालय

3 Outer Himalaya Middle Himalaya Great Himalaya Trans Himalaya

बा िहमालय म िहमालय महान िहमालय परा िहमालय

4 Trans Himalaya Great Himalaya Middle Himalaya Outer Himalaya

परा िहमालय महान िहमालय म िहमालय बा िहमालय

Correct Answer -

Trans Himalaya Great Himalaya Middle Himalaya Outer Himalaya

परा िहमालय महान िहमालय म िहमालय बा िहमालय

Sunrsquos halo is produced by the refraction of light in सय का भामडल ______ म काश क अपवतन ाराउ होता ह

1 Ice crystals in Cirrro-Cumulus clouds पाभ-कपास मघो क बफ िल

2 Ice crystal in Cirrus clouds पाभमघो क बफ िल

3 Dust particles in Stratus clouds री मघो क धल कण

4 Water vapour in Stratus clouds री मघो क जल वा

Correct Answer -

Ice crystal in Cirrus clouds पाभमघो क बफ िल

Read the given statements and answer which of the following options isare correct

(1) The minerals present in the rocks exposed to atmosphere are not subjected to alteration

(2) Oxidation is one of the processes of chemical weathering

िदए गए कथनो को पढ़ और उर द िक िन म स कौन सास िवक सही ह

(1) वायमल स अनावत शल म उपथत खिनज परवतन क अधीन नही होता ह

(2) ऑीकरण रासायिनक अपय की ियाओ म स एक ह

ो ो

41)

42)

1 Both statements are wrong दोनो कथन गलत ह

2 Both statements are correct दोनो कथन सही ह

3 First statement is wrong and second statement is correct पहला कथन गलत ह और दसरा कथन सही ह

4 First statement is correct and second statement is wrong पहला कथन सही ह और दसरा कथन गलत ह

Correct Answer -

First statement is wrong and second statement is correct पहला कथन गलत ह और दसरा कथन सही ह

Read the given statements and answer which of the following options isare correct

1 Sunrsquos short waves enter the earth partially heating the atmosphere

2 Heated earth surface from the sun produces broader waves which interacts and heats the atmosphere

िदए गए कथन को पढ़ और उर द िक िन म स कौन सास िवक सही ह

1 सय की छोटी तरग पी म आिशक प स वश करती ह और वायमडल को ऊत करती ह

2 सय स ऊत पी की सतह िवारत तरग उ करती ह जो परर भाव डालती ह और वायमडल कोऊत करती ह

1 Both Statements 1 and 2 are correct दोनो कथन 1 और 2 सही ह

2 Both Statements 1 and 2 are wrong दोनो कथन 1 और 2 गलत ह

3 Statement 1 is wrong and only Statement 2 is correct कथन 1 गलत ह और कवल कथन 2 सही ह

4 Only statement 1 is correct कवल कथन 1 सही ह

Correct Answer -

Both Statements 1 and 2 are correct दोनो कथन 1 और 2 सही ह

Read the given statements and answer which of the following options isare correct

(1)The rocks that get changed due to heat and pressure are termed as metamorphic rocks

(2)Slate is one such type of metamorphic rock

िदए गए कथनो को पढ़ और उर द िक िन म स कौन सास िवक सही ह

(1) शल जो ऊा और दाब क कारण परवितत हो जात ह उ कायातरक शलो क प म जाना जाता ह

(2) ट एक तरह का कायातरक शल ह

1 Both statements are wrong दोनो कथन गलत ह

2 Both statements are correct दोनो कथन सही ह

3 First statement is wrong and second statement is correct पहला कथन गलत ह और दसरा कथन सही ह

ी औ

43)

44)

4 First statement is correct and second statement is wrong पहला कथन सही ह और दसरा कथन गलत ह

Correct Answer -

Both statements are correct दोनो कथन सही ह

Read the given statements and answer which of the following options isare correct

1 Higher temperature anomaly is observed in the northern hemisphere

2 Differential heating is absent in Northern Hemisphere

िदए गए कथनो को पढ़ और उर द िक िन म स कौन सास िवक सही ह

1 उरी गोलाध म उ तापमान िवसगित पायी जाती ह

2 उरी गोलाध म अतर ऊन अनपथत होती ह

1 Both Statements 1 and 2 are correct दोनो कथन 1 और 2 सही ह

2 Both Statements 1 and 2 are wrong दोनो कथन 1 और 2 गलत ह

3 Statement 1 is wrong and Statement 2 is correct कथन 1 गलत ह और कथन 2 सही ह

4 Statement 1 is correct and Statement 2 is wrong कथन 1 सही ह और कथन 2 गलत ह

Correct Answer -

Statement 1 is correct and Statement 2 is wrong कथन 1 सही ह और कथन 2 गलत ह

Read the given statements and answer which of the following options isare correct

(1) Plutonic rocks are intrusive type of igneous rocks

(2) It cools very slowly because the surrounding rock serves as insulation around the intrusion of magma

िदए गए कथनो को पढ़ और उर द िक िन म स कौन सास िवक सही ह

(1) िवतलीय शल अतवधी कार क आश शल ह

(2) यह बत धीर-धीर ठडा होता ह ोिक आस-पास क शल मा क अतवधन क चारो ओर रोधन क प म कायकरत ह

1 Both statements are wrong दोनो कथन गलत ह

2 Both statements are correct दोनो कथन सही ह

3 First statement is wrong and second statement is correct पहला कथन गलत ह और दसरा कथन सही ह

4 First statement is correct and second statement is wrong पहला कथन सही ह और दसरा कथन गलत ह

Correct Answer -

Both statements are correct दोनो कथन सही ह

45)

46)

47)

48)

The dust and ash material hurled from the volcanoes are termed as

ालामखी स िनकलन वाली धल और राख सामी को _______ क प म कहा जाता ह

1 Pyroclasc पाइरोाक

2 Hyperclastic हाइपराक

3 Hepiroclastic हिपरोाक

4 Cirroclastic िसरोाक

Correct Answer -

Pyroclasc पाइरोाक

The vertical difference in elevation between a low tide and high tide is referred as

कम ार और उ ार क बीच ऊचाई म लबवत अतर _____ स सदिभत होता ह

1 Tidal slope ारीय ढलान

2 Tidal elevation ारीय उयन

3 Tidal range ारीय परास

4 Tidal height ारीय ऊचाई

Correct Answer -

Tidal range ारीय परास

The maximum biodiversity is found in which of the following regions िनिलखत ो म स अिधकतमजव िविवधता िकसम पायी जाती ह

1 Amazon Basin अमज़न बिसन

2 East Indies ई इडीज

3 Congo Basin कागो बिसन

4 West indies व इडीज

Correct Answer -

Amazon Basin अमज़न बिसन

The cultivation of rice crop produces_______ चावल की फसल की खती ______ का उादन करती ह

1 SO2

49)

50)

51)

2 CH4

3 CFCs

4 CO2

Correct Answer -

CH4

The pressure system with higher pressure at the centre is called__________

क म उ दबाव वाली दबाव णाली को _______ कहा जाता ह

1 front अ

2 depression अवनमन

3 cyclone चवात

4 anti-cyclone ितचवात

Correct Answer -

anti-cyclone ितचवात

The Himalayan region is poor in mineral resources because िहमालयी खिनज ससाधनो म समनही ह ोिक

1 The displacement of rock strata has disturbed the arrangement of rocks and made it complex

शलीय परत क िवथापन न चानो की वथा को अवथत कर िदया ह और इस जिटल बना िदया ह

2 The climate conditions are not suitable for exploitation of minerals

जलवाय की थित खिनजो क दोहन क िलए उपय नही ह

3 The terrain makes explanation of minerals difficult and very costly due to transportation difficulties

भ-भाग परवहन की किठनाइयो क कारण खिनजो का दोहन मल और बत महगा बना दता ह

4 It is made up of crystalline rocks यह िलीय चानो स बना ह

Correct Answer -

The displacement of rock strata has disturbed the arrangement of rocks and made it complex

शलीय परत क िवथापन न चानो की वथा को अवथत कर िदया ह और इस जिटल बना िदया ह

The process through which the moisture is added to the atmosphere by vegetation is termed as

वह िया िजसक माम स वनित ारा वातावरण म नमी िमलायी जाती ह _______ क प म जानी जाती ह

52)

53)

54)

1 Condensation सघनन

2 Evapotranspiration वान-उजन

3 Radiation िविकरण

4 Precipitation वषण

Correct Answer -

Evapotranspiration वान-उजन

The process through which the terrestrial heat is transferred to air by direct contact is termed as

वह िया िजसम सपक ारा थलीय ऊा वाय म थानातरत हो जाती ह ______ क प म जानी जाती ह

1 Conduction चालन

2 Convection सवहन

3 Insolation आतपन

4 Radiation िविकरण

Correct Answer -

Conduction चालन

The largest area under mangroves is in which of the following statesunion territory

मोव क अतगत िनिलखत राोसघ शािसत दशो म स सबस बड़ा कौन सा ह

1 Andaman and Nicobar अमान और िनकोबार

2 Andhra Pradesh आ दश

3 West Bengal पिम बगाल

4 Gujarat गजरात

Correct Answer -

West Bengal पिम बगाल

The longitudinal transverse and surface waves in an earthquake originate from

भकप म दशातर अनथ और सतह तरग यहा उ होती ह

1 The focus on the surface of the Earth पी क सतह पर क -िबद म

2 The focus within the body of the Earth पी क भीतर क -िबद म

3 The epicenter within the body of the Earth पी क भीतर उपरक म

55)

56)

57)

4 The epicenter on the surface of the Earth पी क सतह पर उपरक म

Correct Answer -

The focus within the body of the Earth पी क भीतर क -िबद म

The down slope movement of material due to gravity is called______

गाकषण क कारण पदाथ की अनढाल गित को ______ कहा जाता ह

1 mass movement पदाथ सचलन

2 deposition िनप

3 erosion रण

4 volcanic movement ालामखीय सचलन

Correct Answer -

mass movement पदाथ सचलन

Shimla is cooler than Amritsar although both are on the same latitude This is because

िशमला म अमतसर स अिधक ठड ह हालािक दोनो समान अाश पर ह ऐसा ह ोिक

1 Shimla is at a greater height above sea level than Amritsar अमतसर की तलना म िशमला सम तल स अिधकऊचाई पर ह

2 Shimla is further north िशमला उर की ओर ह

3 Shimla is farther from the equator िशमला भम रखा स आग ह

4 Their longitudes differ उनकी दशातर रखाए िभ ह

Correct Answer -

Shimla is at a greater height above sea level than Amritsar अमतसर की तलना म िशमला सम तल स अिधकऊचाई पर ह

lsquoTempo of Urbanizationrsquo measures which of the following

lsquoशहरीकरण का टपोrsquo िनिलखत म स कौन सा उपाय ह

1 Speed of urbanizaon शहरीकरण की गित

2 None of the above इनम स कोई नही

3 Inequality of urbanizaon शहरीकरण की असमानता

4 Current level of urbanizaon शहरीकरण का वतमान र

Correct Answer -

58)

59)

60)

Speed of urbanizaon शहरीकरण की गित

Out of the following options choose the INCORRECT statement

िनिलखत िवको म स गलत कथन का चयन कर

1 The clear tracts in the equatorial region recover rapidly भम रखा म भभाग तजी स ठीक हो जात ह

2 The stable communities include a redwood forest a pine forest at high elevations

थर समदायो म एक रडवड वन उ ऊचाई पर एक दवदार वन शािमल ह

3 Any ecosystem moves towards maximum biomass and stability to survive

कोई भी पारथितकी त जीिवत रहन क िलए अिधकतम जवसहित और थरता की तरफ असर होता ह

4 Tropical rain forests near equator are stable ecosystems

भम रखा क पास उकिटबधीय वषा वन थर पारथितक त ह

Correct Answer -

The clear tracts in the equatorial region recover rapidly भम रखा म भभाग तजी स ठीक हो जात ह

Seasonal contrasts are maximum in मौसमी िवषमता अिधकतम ह

1 Mid latitudes म अाश म

2 Low attitudes िन अाश म

3 High latitudes उ अाश म

4 Subtropics उपोकिटबधीय म

Correct Answer -

Mid latitudes म अाश म

In India which type of forest among the following occupies the largest area

भारत म िनिलखत म स िकस कार क वन सबस बड़ा फल आािदत करत ह

1 Sub-tropical Dry Evergreen Forest उप उकिटबधीय श सदाबहार वन

2 Mountain Wet Temperate Forest पवतीय आ शीतो वन

3 Tropical Moist Deciduous Forest उकिटबधीय आ पणपाती वन

4 Tropical Wet Evergreen Forest उकिटबधीय आ सदाबहार वन

Correct Answer -

Tropical Moist Deciduous Forest उकिटबधीय आ पणपाती वन

61)

62)

63)

64)

What is the proportion of lsquoJuvenile Populationrsquo (0-14 years) in India as per 2011Census

2011 की जनगणना क अनसार भारत म जवनाइल पॉपलशन यानी िकशोर जनस या (0-14 वष) का अनपात ाह

1 3076 of total population कल जनस या का 3076

2 2764 of total population कल जनस या का 2764

3 2933 of total population कल जनस या का 2933

4 3354 of total population कल जनस या का 3354

Correct Answer -

3076 of total population कल जनस या का 3076

What is the Belfast famous for बलफा िकसक िलए मशर ह

1 Belt of cotton textile industry कपास व उोग क

2 Ship-building industry जहाज िनमाण उोग

3 Agricultural machinery किष उपकरण

4 Aero planes manufacturing वाययान िनमाण

Correct Answer -

Ship-building industry जहाज िनमाण उोग

What is the most important occupation in tropical monsoon lands

उकिटबधीय मॉनसन भिम म सबस महपण वसाय ा ह

1 Mining खनन

2 Cattle rearing मवशी पालन

3 Agriculture किष

4 Nomadic herding नोमािडक जड़ी-बिटया

Correct Answer -

Agriculture किष

What is the most important characteristics of the islands (Indian) located in the Arabian sea

अरब सागर म थत ीपो (भारतीय) की सबस महपण िवशषता ा ह

ी ो

65)

66)

67)

1 There are all of coral origins सभी कोरल मल क ह

2 There are all very small in size य सभी आकार म बत छोट ह

3 They have a very dry climate इनकी जलवाय बत श ह

4 They are extended parts of the mainland व महाीप क िवारत िह ह

Correct Answer -

There are all of coral origins सभी कोरल मल क ह

What do the basalt layers of the Deccan indicate डन की बसा परत ा इिगत करती ह

1 All of the above उपरो सभी

2 Huge volcanic eruptions in the distant past दरथ अतीत म िवशाल ालामखीय िवोट

3 The immense erosional activity of the rivers निदयो की िवशाल रण गितिविध

4 The influence of weathering मौसम का भाव

Correct Answer -

Huge volcanic eruptions in the distant past दरथ अतीत म िवशाल ालामखीय िवोट

In the structure of planet Earth below the mantle the core is mainly made up of_____

पी ह की सरचना म मटल क नीच कोर म प स______ स िनिमत होती ह

1 aluminium एमीिनयम

2 silicon िसिलकॉन

3 chromium ोिमयम

4 iron लोहा

Correct Answer -

iron लोहा

One of the major Mid Oceanic Ridge is found in मख म-महासागर चोिटयो म स एक ______ म पायाजाता ह

1 Mid Pacific Ocean म शात महासागर

2 Mid Atlantic Ocean म अटलािटक महासागर

3 Mid Indian Ocean म भारतीय महासागर

4 Mid Arctic Ocean म आक िटक महासागर

68)

69)

70)

71)

Correct Answer -

Mid Atlantic Ocean म अटलािटक महासागर

Magma that reaches the Earthrsquos surface and then solidifies is called________

मा जो पी की सतह तक पचती ह और िफर ठोस हो जाती ह ________कहलाती ह

1 quartz ाटज

2 lava लावा

3 granite नाइट

4 silicates िसिलकट

Correct Answer -

lava लावा

Isotherms are the lines of equal_______ समताप रखाए समान _______की रखाए होती ह

1 pressure दाब

2 temperature तापमान

3 rainfall वषा

4 height ऊचाई

Correct Answer -

temperature तापमान

Mark the correct sequence of passes in the Western Ghats from north to south

पिमी घाटो म उर स दिण तक दर क सही अनम को िचित कर

1 Thalghat Palghat Bhorghat थलगघाट पालघाट भोरघाट

2 Thalghat Bhorghat Palghat थलघाट भोरघाट पालघाट

3 Bhorghat Thalghat Palghat भोरघाट थलघाट पालघाट

4 Palghat Bhorghat Thalghat पालघाट भोरघाट थलघाट

Correct Answer -

Thalghat Bhorghat Palghat थलघाट भोरघाट पालघाट

Which of the following does not have influence over the climate in India

ि ि ि ी ी

72)

73)

िनिलखत म स िकसका भाव भारत की जलवाय पर नही पड़ता ह

1 Ocean currents सागर की लहर

2 Nearness to equator भम रखा स िनकटता

3 Monsoons मानसन

4 Presence of Indian ocean भारतीय महासागर की उपथित

Correct Answer -

Ocean currents सागर की लहर

Which of the following cloud types has the characteristics like vertical tall narrow and puffy

िनिलखत म स िकस कार क मघो म लबवत लबी सकीण और थलता जसी िवशषताए ह

1 Cumulonimbus तफानी मघ

2 Cumulus मघ पज

3 Cirrocumulus पाभ कपासी मघ

4 Nimbostratus वषारी मघ

Correct Answer -

Cumulus मघ पज

Which of the following statement is INCORRECT about Crude Birth Rate

िनिलखत स कौन सा कथन अशोिधत ज दर क बार म सही नही ह

1 It cannot be used for comparing fertility level between two countries with different population characteristics

इसका उपयोग िविभ जनसा िवशषताओ वाल दो दशो क बीच जनन र की तलना क िलए नही िकया जा सकता ह

2 It is a standardized measure of fertility

यह जनन मता का मानकीकत उपाय ह

3 It is effected by the age-sex composition of the population

यह आबादी की आय-िलग सरचना स भािवत होता ह

4 It is expressed per 1000 population in a given geographical unit

यह िकसी दी गई भौगोिलक इकाई म ित 1000 जनसा पर िकया जाता ह

Correct Answer -

It is a standardized measure of fertility

यह जनन मता का मानकीकत उपाय ह

74)

75)

76)

77)

Which of the following state in India experienced negative decadal growth rate during 2001 to 2011census

भारत म िनिलखत म स िकस रा म वष 2001 स 2011 की जनगणना क दौरान नकाराक िगरावट दर ई

1 Tripura िपरा

2 Nagaland नागालड

3 Haryana हरयाणा

4 Odisha ओिडसा

Correct Answer -

Nagaland नागालड

Which of the following is NOT a characteristic of peninsular rivers

िनिलखत म स कौन सी िवशषता ायीपीय निदयो म नही होती ह

1 Flow through shallow valleys उथल घािटयो क माम स वाह

2 Seasonal flow मौसमी वाह

3 Little erosional activity थोड़ी कटावदार गितिविध

4 Meandering tendency often shifting their beds घमावदार वि अर अपन तटो को थानातरत करना

Correct Answer -

Meandering tendency often shifting their beds घमावदार वि अर अपन तटो को थानातरत करना

Which of the following gases in the atmosphere absorbs heat from the Sunrsquos radiation and the Earthssurface

वायमडल म िनिलखत म स कौन सी गस सय क िविकरण और पी की सतह स ऊा को अवशोिषत करती ह

1 Neon िनयॉन

2 Carbon dioxide काबन डाइऑाइड

3 Argon आगन

4 Nitrogen नाइट ोजन

Correct Answer -

Carbon dioxide काबन डाइऑाइड

Which of the following kind of settlement pattern is found at the confluence of rivers

ि ि ि ि ो

78)

79)

80)

िनिलखत म स िकस कार का वथापन पटन निदयो क सगम पर पाया जाता ह

1 Triangular Paern िकोणीय पटन

2 Circular or Semi-Circular Paern परप या अध-परप पटन

3 Nebular Paern नबलर पटन

4 Star ndashShaped Paern ार-आकार का पटन

Correct Answer -

Triangular Paern िकोणीय पटन

Which one was not the objective of the Biosphere Reserve Projects launched by the UNESCO

यनो ारा श की गई सरित जवमडल परयोजनाओ का उ इनम स कौन सा नही था

1 To promote teaching and research िशण और अनसधान को बढ़ावा दना

2 To make agriculture sustainable किष को दीघकािलक बनाना

3 To conserve ecosystems पारथितक त को सरित करना

4 To conserve genetic diversity for a longtime लब समय तक अनवािशक िविवधता को सरित करना

Correct Answer -

To make agriculture sustainable किष को दीघकािलक बनाना

Which region of the Earth surface receives the highest amount of insulation

पी सतह का कौन सा तापावरोधन की उतम माा ा करता ह

1 Land mass थलखड

2 Savannah region सवाना

3 Water bodies जल िनकाय

4 Tropical desert उकिटबधीय रिगान

Correct Answer -

Tropical desert उकिटबधीय रिगान

Which one of the following is not a biodiversity hotspot

िनिलखत म स कौन सा जव िविवधता का म जगह नही ह

1 Eastern Himalaya पव िहमालय

2 Eastern Ghats पव घाट

81)

82)

83)

3 Indo-Myanmar भारत-ामार

4 Westerm Ghats पिमी घाट

Correct Answer -

Eastern Ghats पव घाट

Which one of the following is NOT a part of the World Network of Biosphere Reserves based on theUNESCO Man and Biosphere Programme

यनो मन और बायोीयर कायम क आधार पर िनिलखत म स कौन बायोीयर रजव क िव नटवक कािहा नही ह

1 Gulf of Mannar मार की खाड़ी

2 Seshachalam शषाचलम

3 Sunderban सदरबन

4 Nilgiri नीलिगर

Correct Answer -

Seshachalam शषाचलम

Which one of the following is an example of ldquodesert vegetationrdquo

िनिलखत म स कौन मथलीय वनित का एक उदाहरण ह

1 Mosses and lichens दलदल और शवाल

2 Temperate grassland समशीतो घास क मदान

3 Coniferous forest शकधारी वन

4 Acacia and cactus एकािसया और कस

Correct Answer -

Acacia and cactus एकािसया और कस

Which one of the following reflects more sunlight िनिलखत म स कौन सा सय की रोशनी को अिधकपरावितत करता ह

1 Paddy crop land धान फसल भिम

2 Land covered with fresh snow ताजा बफ स आािदत भिम

3 Sand desert रतीली रिगान

4 Prairie land यरी भिम

84)

85)

86)

87)

Correct Answer -

Land covered with fresh snow ताजा बफ स आािदत भिम

Which layer of the atmosphere is in contact with the surface of the earthrsquos oceans

वायमडल की कौन सी परत पी क महासागरो की सतह क सपक म ह

1 Stratosphere समताप मडल

2 Mesosphere म मडल

3 Hydrosphere जलमडल

4 Troposphere ोभ मडल

Correct Answer -

Troposphere ोभ मडल

Mediterranean Sea is a border of which of the following countries भम सागर िनिलखत दशो म सिकसकी सीमा ह

1 None of these इनम स कोई नही

2 Iraq इराक

3 Lebanon लबनान

4 Jordan जॉडन

Correct Answer -

Lebanon लबनान

Benguela ocean currents are found along which coast बगएला महासागर धाराए िकस तट क साथ पायीजाती ह

1 East Coast of South America दिण अमरका क पव तट

2 East Coast of Africa अीका क पव तट

3 West Coast of South America दिण अमरका क पिमी तट

4 West Coast of Africa अीका क पिमी तट

Correct Answer -

West Coast of Africa अीका क पिमी तट

88)

89)

90)

Due to tension a block of land on one side being pushed up or upthrown relative to the downthrown blockis referred as

तनाव क कारण नीच फ क ए खड क साप भिम का एक खड एक ओर स ऊपर धकला जाता ह या ऊपर की ओरफ का जाता ह यह _____ क प म सदिभत ह

1 Thrust fault प श

2 Normal fault सामा श

3 Reverse fault म श

4 Strike slip fault नितलब सपण श

Correct Answer -

Normal fault सामा श

Inter-tropical doldrums is a zone of ______ अतर-उकिटबधीय डोलड ______ का एक ह

1 Frontolysis टोलायिसस

2 Convergence अिभसरण

3 Inter-tropical divergence zone अतर-उकिटबधीय िवचलन

4 Local wind थानीय वाय

Correct Answer -

Convergence अिभसरण

The Horse Latitudes are regions located at about _____ north and south of the equator

हॉस अाश भम रखा क उर और दिण म लगभग _____ पर थत ह

1 30ndash60 degree Latitude 30-60 िडी अाश

2 0ndash5 degree Latitude 0-5 िडी अाश

3 30 degree Latitude 30 िडी अाश

4 60ndash90 degree Latitude 60-90 िडी अाश

Correct Answer -

30 degree Latitude 30 िडी अाश

Generally evaporation is high over which part of the Earth

आम तौर पर पी क िकस भाग पर वाीकरण अिधक होता ह

1 Equatorial maritime भमवत समीय ी ी

91)

92)

2 Equatorial continental भमवत महाीपीय

3 Polar maritime वीय समीय

4 Polar continental वीय महाीपीय

Correct Answer -

Equatorial maritime भमवत समीय

A very high temperature during summer in north western India leads to what type of climaticcondition in south

उर पिमी भारत म गम क दौरान बत अिधक तापमान होन क कारण दिण म िकस कार की जलवाय थितउ करता ह

1 Depression over arabian sea अरब सागर पर अवनमन

2 Failure monsoon मानसन िवफलता

3 Successful monsoon मानसन सफलता

4 Cyclones चवात

Correct Answer -

Successful monsoon मानसन सफलता

Lightning and thunder are the resultant effect when तिड़त और गजन परणामी भाव ह जब

1 Two massive clouds hit powerfully each other first lightning is produced and later sound is produced

दो बड़ बादल एक दसर स शशाली ढग स टकरात ह पहल आकाशीय िवदयत उ होता ह और बाद म िन उहोती ह

2 Two massive clouds come into contact with the powerful wind collision this results into first sound and thenlightning

दो बड़ बादल शशाली पवन सघ क सपक म आत ह इसका परणामप पहल िन और िफर आकाशीय िवदयतउ होता ह

3 None of the above उपरो म स कोई भी नही

4 A high density cloud contains positively and negatively charged electric ions and when this interacts light andsound are simultaneously produced

एक उ घन बादल म धनाक और ऋणाक आविशत िवदयत आयन होत ह और जब यह परर भाव डालत ह तोकाश और िन एक साथ उािदत होती ह

Correct Answer -

A high density cloud contains positively and negatively charged electric ions and when this interacts light andsound are simultaneously produced

औ ि ि ो औ ो

93)

94)

95)

एक उ घन बादल म धनाक और ऋणाक आविशत िवदयत आयन होत ह और जब यह परर भाव डालत ह तोकाश और िन एक साथ उािदत होती ह

Doon Valley is able to grow rice because दन घाटी चावल उगान म सम ह ोिक

1 Other crops cannot be grown वहा अ फसलो को उगाया नही जा सकता ह

2 People in the valley are rice eaters घाटी म लोग चावल खान वाल ह

3 There is a huge export demand of rice वहा चावल की भारी िनयात माग ह

4 It has warm summer and snow melt waters for irrigation

वहा गिमया गम होती ह िसचाई क िलए बफ का िपघला आ पानी होता ह

Correct Answer -

It has warm summer and snow melt waters for irrigation

वहा गिमया गम होती ह िसचाई क िलए बफ का िपघला आ पानी होता ह

CANCELLED

In the geological time scale the Mesozoic Era DOES NOT contains which of the following periods

भगभय समय पमान पर मजीवी यग म िन कालो म स कौन नही ह

1 Triassic ट ाइऐिसक

2 Jurassic जरिसक

3 Cretaceous चाकमय

4 Carboniferous काबनी

Correct Answer -

Carboniferous काबनी

96)

1 P-3 Q-4 R-2 S-1

2 P-3 Q-4 R-1 S-2

3 P-3 Q-4 R-1 S-2

4 P-4 Q-3 R-2 S-1

Correct Answer -

P-4 Q-3 R-2 S-1

1 P-3 Q-1 R-4 S-2

2 P-3 Q-4 R-1 S-2

3 P-3 Q-2 R-4 S-1

97)

98)

4 P-2 Q-1 R-4 S-3

Correct Answer -

P-3 Q-4 R-1 S-2

1 P-3 Q-1 R-4 S-2

2 P-2 Q-3 R-4 S-1

3 P-2 Q-1 R-3 S-4

4 P-4 Q-2 R-1 S-3

Correct Answer -

P-3 Q-1 R-4 S-2

99)

100)

1 P-3 Q-2 R-4 S-1

2 P-1 Q-2 R-3 S-4

3 P-2 Q-3 R-1 S-4

4 P-4 Q-3 R-2 S-1

Correct Answer -

P-2 Q-3 R-1 S-4

ldquoHuman geography is the study of changing relationship between the unresting man and the unstableearthrdquo was defined by

lsquolsquoमानव भगोल ाकल आदमी और अथर पी क बीच सबध परवतन का अयन हrdquo ______ ारा परभािषत िकया गयाथा

1 J Brunches ज चस

2 EC Semple ईसी सल

3 HJ Mackinder एच ज मिकदर

4 PV Blache पीवी च

Correct Answer -

EC Semple ईसी सल

Sedimentary rocks are finally and ultimately derived from the____________

अवसादी चान अततः ________ स ा की जाती ह

1 action of earth movements पी की गितिविधयो

2 marine deposit समी िनप

3 weathering of metamorphic rocks पातरत चानो क अपय

4 weathering of igneous rocks आय चानो क अपय

Correct Answer -

weathering of igneous rocks आय चानो क अपय

Page 9: High School Teacher Eligibility Test- BOARD PROFESSIONAL ...peb.mp.gov.in/results/RESULT_18/HST_RES18/Final_anwser_key/HST… · M a ndl a / मंड ल ... Under the Madhya Pradesh

9)

10)

1)

3 Shere Khan शर खान

4 Bagheera बघीरा

Correct Answer -

Mowgli मोगली

The recipient of which one of the following awards given by Madhya Pradesh Government gets the highest cashprize

िनिलखत परारो म स िकसक ाकता को म दश सरकार ारा उतम नकद परार िदया जाता ह

1 Mahatma Gandhi award महाा गाधी परार

2 Shikar Award िशखर परार

3 Sharad Joshi Award शरद जोशी परार

4 Kishore Kumar Award िकशोर कमार परार

Correct Answer -

Mahatma Gandhi award महाा गाधी परार

Who among the following was the Trinidadian writer of Indian descent

िनिलखत म स भारतीय मल क ििनदािदयन लखक कौन थ

1 RK Narayan आर क नारायण

2 Anita Desai अनीता दसाई

3 Rudyard Kipling डयाड िकपिलग

4 VS Naipaul वीएस नायपॉल

Correct Answer -

VS Naipaul वीएस नायपॉल

Topic- GENERAL REASONING

Find the missing number ल त स या ात कर

4 196 16 169 ____ 144 64

1 81

2 36

3 32

4 21

2)

3)

4)

Correct Answer -

36

Find the average औसत ात कर

25 35 45 55 65 amp 75

1 48

2 50

3 47

4 49

Correct Answer -

50

The below series uses a sequence of alphabets and numbers Identify the incorrect combination

नीच दी गई खला अरो एव साओ का अनम योग करती ह गलत सयोजन पहचान

(i) FT85DF77ER

(ii) FT85DF77ER

(iii) FT85DE77ER

(iv) FT85DF77ER

1 ii

2 iv

3 i

4 iii

Correct Answer -

iii

A and B together can complete a piece of work in 10 days and B alone can complete the same work in 20days In how many days can A alone complete the work

A और B एक साथ काम क एक भाग को 10 िदनो म समा त करत ह और B अकल उसी काम को 20 िदनो म समा त करता हिकतन िदनो म A अकल उस काम को समा त कर सकता ह

1 20 days 20 िदन

2 30 days 30 िदन

3 10 days 10 िदन

4 25 days 25 िदन

5)

6)

Correct Answer -

20 days 20 िदन

Which one of the following four addresses is NOT EXACTLY same as the one given below

िनिलखत िदए गए चार पतो म स नीच िदए गए पत क ठीक समान कौन सा एक नही ह

Ground Floor 59 Goulburn St

Sydney NSW 2000 Australia

+61 (02) 8987 3700

(i) Ground Floor 59 Goulburn St

Sydney NSW 2000 Australia

+61 (02) 8987 3700

(ii) Ground Floor 59 Goulburn St

Sydney NSW 2000 Australia

+61 (02) 8987 3700

(iii) Ground Floor 59 Govlburn St

Sqdney NSW 2000 Australia

+61 (02) 8987 3700

(iv) Ground Floor 59 Goulburn St

Sydney NSW 2000 Australia

+61 (02) 8987 3700

1 ii

2 iv

3 i

4 iii

Correct Answer -

iii

In a group of 75 people 32 of them like cold drink 56 of them like general water and each person likesat least one of the two drinks How many people like both

75 लोगो क समह म उनम स 32 को को िड क पसद ह उनम स 56 को सामा जल पसद ह और क को दो म स कम स कम एक पय पसद ह िकतन लोगो को दोनो पसद ह

1 11

2 13

7)

1)

2)

3 12

4 14

Correct Answer -

13

If A = 1 FAT = 27 then FAITH =

यिद A = 1 FAT = 27 तो FAITH =

1 41

2 40

3 42

4 44

Correct Answer -

44

Topic- PEDAGOGY

CANCELLED

Basic education became a major initiative because of the efforts of

ाथिमक िशा िन न क यास स एक मह वपण कायम बन गई

1 Dr Radhakrishnan डॉ राधाक णन

2 Dr Rajendra Prasad डॉ राज साद

3 Mahatma Gandhi महा मा गाधी

4 Rabindranath Tagore रबी नाथ टगोर

Correct Answer -

Mahatma Gandhi महा मा गाधी

The ldquoclause of phraserdquo is an unit of perception of लॉज ऑफ ज िन न क धारणा क एक इकाई ह

1 Image छिव

2 Concept अवधारणा

3 Language भाषा

4 Thought िवचार

Correct Answer -

3)

4)

5)

6)

Language भाषा

Which method is most suitable to study communication process among students

छाो क बीच सचार िया का अयन करन क िलए कौन सी िविध सबस उपय ह

1 Case Study मामल का अयन (कस टडी)

2 Systematic Observation वथत अवलोकन

3 Experimental Method योगाक िविध

4 Introspection आ-िनरीण (इट ोस शन)

Correct Answer -

Systematic Observation वथत अवलोकन

Genes in a human being are located in मानवो म जीन इनम थत होता ह

1 cytoplasm कोिशका (साइटो ला म)

2 ribosomes राइबोसोम

3 cell membranes कोिशका िझी

4 chromosomes गणस (ोमोसोम)

Correct Answer -

chromosomes गणस (ोमोसोम)

With smaller classes teachers are much more able to ____________

छोटी काओ क साथ िशक ____________ म अिधक सम होत ह

1 Go slow while teaching िशण क दौरान धीमी गित स जान

2 Narrate more personal experiences अिधक गत अनभवो को बतान

3 Adapt instruction to individual differences among students छाो क बीच गत मतभदो क िलए अनकल िनदश दन म

4 Make use of the extra space for extra-curricular activities पातर गितिविधयो क िलए अितर जगह का उपयोग करन

Correct Answer -

Adapt instruction to individual differences among students छाो क बीच गत मतभदो क िलए अनकल िनदश दन म

While engaging in a task the child gets bored This is a sign of

एक काम म होन पर बा ऊब जाता ह यह िन का सकत ह

1 the task requiring a professional approach काय को ावसाियक िकोण की आवकता ह

7)

8)

9)

2 the task becoming mechanically repetitive काय यािक प स दोहराव वाला ह

3 the child not being intelligent बा बमान नही ह

4 the child being incapable of learning बा सीखन म असमथ ह

Correct Answer -

the task becoming mechanically repetitive काय यािक प स दोहराव वाला ह

Educational Psychologists are more concerned with the learning in __________

शिणक मनोवािनक __________ म अिधगम क साथ अिधक िचितत होत ह

1 Formal environment औपचारक वातावरण

2 Informal environment अनौपचारक वातावरण

3 Physical environment भौितक वातावरण

4 Social environment सामािजक वातावरण

Correct Answer -

Formal environment औपचारक वातावरण

Special needs education is the type of education िवशष ज़रतो वाली िशा वह िशा होती ह जो

1 Given to person with disabilities अम य को दी जाती ह

2 Given to people from remote areas दर थ ो क लोगो को दी जाती ह

3 Provided to intelligent people बमान लोगो को दी जाती ह

4 Established by colonial masters औपिनविशक मखयाओ ारा थािपत की गई ह

Correct Answer -

Given to person with disabilities अम य को दी जाती ह

The Stanford-Binet scale of intelligence was first published in the year

ब क नफोड-िबनट कल को इस वष म पहली बार कािशत िकया गया था

1 1916

2 1903

3 1908

4 1900

Correct Answer -

1916

10)

11)

12)

13)

The term lsquofictional finalismrsquo was propounded by

पद lsquoकत योजनवादrsquo (िफ शनल फाइनिल म) इनक ारा ितपािदत िकया गया था

1 Skinner नर

2 Freud ायड

3 Adler एडलर

4 Pavlov पावलोव

Correct Answer -

Adler एडलर

Rational Emotive Behavior Therapy was propounded by

तक सगत भावनाक वहार थरपी िन क ारा ितपािदत की गई थी

1 Carl Jung काल यग

2 Carl Rogers काल रोजस

3 Aaron Beck आरोन बक

4 Albert Ellis अट एिलस

Correct Answer -

Albert Ellis अट एिलस

Which of the following indicates the quality of education in a school

िनिलखत म स या एक कल म िशा की गणव ता का सकतक ह

1 Text-books and Teaching-learning material पा-प तक तथा िशण व सीखन स सबिधत सामी

2 Infrastructural facilities at the school कल म आधारभत ढाच स सबिधत सिवधाए

3 Student achievement level िवािथयो का उपल तर

4 Classroom systems का की यव था

Correct Answer -

Student achievement level िवािथयो का उपल तर

Which of the following teachers can be identified with authoritarian teaching style

िनिलखत म स कौन सा िशक अिधकारवादी िशण शली क साथ पहचाना जा सकता ह

1 Laissez-faire teacher अब ध िशक

14)

15)

2 Democratic teacher लोकतीय िशक

3 Indifferent teacher िन प िशक

4 Direct instruction teacher िनदश िशक

Correct Answer -

Direct instruction teacher िनदश िशक

Who was the pioneer of classical conditioning

िचरितित ानकलन ( लािसकल कडीशिनग) क वतक कौन थ

1 Skinner नर

2 Pavlov पावलोव

3 Kohler कोहलर

4 Freud ायड

Correct Answer -

Pavlov पावलोव

Which of the following are true with reference to short term memory

1 Limited capacity

2 Brief storage of information

3 Unlimited capacity

4 Duration of storage less than twenty seconds

अ पकािलक मित क सदभ म िन न म स या स य ह

1 सीिमत मता

2 सचना का सि त भडारण

3 असीिमत मता

4 बीस सकड स कम भडारण की अविध

1 3 and 4 3 और 4

2 2 and 4 2 और 4

3 13 and 4 13 और 4

4 12 and 4 12 और 4

Correct Answer -

12 and 4 12 और 4

1)

2)

3)

4)

Topic- GEOGRAPHY

Which of the following ranges of population size is used to define Class-III city by Indian Census

भारतीय जनगणना ारा वग-III शहर को परभािषत करन क िलए िनिलखत म स िकस रज क जनसा आकारका उपयोग िकया जाता ह

1 20 000 to 49999 20 000 स 49999

2 30000 to 59999 30000 स 59999

3 24000 to 54999 24000 स 54999

4 50000 to 99999 50000 स 99999

Correct Answer -

20 000 to 49999 20 000 स 49999

Carbonaceous rocks which produce coal and oil belong to the category of rocks called_______

कोयल और तल का उादन करन वाली काबनय चान ______ नामक चानो की णी स सबिधत ह

1 metamorphic पातरत

2 sedimentary अवसादी

3 inorganic अजिवक

4 igneous आय

Correct Answer -

sedimentary अवसादी

The ruhr-complex is a major industrial centre in र-परसर िन का एक मख औोिगक क ह

1 North America उरी अमरका

2 Russia स

3 Germany जमनी

4 Europe यरोप

Correct Answer -

Germany जमनी

The term lsquoRegurrsquo refers to श lsquoरगरrsquo ______ स सबिधत ह

1 Deltaic alluvial soils डा जलोढ़ िमी

ि ी

5)

6)

2 Laterite soils लटराइट िमी

3 Red and yellow soils लाल और पीली िमी

4 Black cotton soils काली कपास िमी

Correct Answer -

Black cotton soils काली कपास िमी

Read the given statements and answer which of the following options isare correct

1 Lower the pressure greater the atmospheric disturbance

2 Air move from higher to low pressure

िदए गए कथन को पढ़ और उर द िक िन म स कौन सास िवक सही ह

1 िजतना दाब कम होगा वायमडलीय बाधाए उतनी अिधक होगी

2 वाय उ स िन दाब की ओर गित करती ह

1 Both Statements 1 and 2 are correct दोनो कथन 1 और 2 सही ह

2 Both Statements 1 and 2 are wrong दोनो कथन 1 और 2 गलत ह

3 Statement 1 is wrong and only Statement 2 is correct कथन 1 गलत ह और कवल कथन 2 सही ह

4 Statement 1 is correct and Statement 2 is wrong कथन 1 सही ह और कथन 2 गलत ह

Correct Answer -

Both Statements 1 and 2 are correct दोनो कथन 1 और 2 सही ह

CANCELLED

1 4 1 2 and 3 4 1 2 और 3

2 4 1 3 and 2 4 1 3 और 2

3 1 4 2 and 3 1 4 2 और 3

4 1 4 3 and 2 1 4 3 और 2

Correct Answer -

7)

8)

9)

1 4 3 and 2 1 4 3 और 2

CANCELLED

Karl Pearsonrsquos correlation co-efficient is काल िपयसन का सहसबध गणाक ह

1 Arithmec mean समार मा

2 Geometric mean गणोर मा

3 None of these इनम स कोई नही

4 Harmonic mean हराक मा

Correct Answer -

Geometric mean गणोर मा

CANCELLED

ldquoEach day is more or less the same the morning is clear and bright with a sea breeze as the Sun climbshigh in the sky heat mounts up dark clouds form then rain comes with thunder lighting But rain is soonoverrdquo Which of the following regions is described in the above passage

ldquoक िदन समान स अिधक या कम होता ह सम की हवा क साथ और उल सबह होती ह जस सयआकाश म ऊचा चढ़ता ह गम बढ़ जाती ह काल बादल बनत ह िफर िबजली क साथ बारश आती ह लिकनबारश जी ख हो जाती हlsquorsquo उपरो पा म िनिलखत म स िकन ो का वणन िकया गया ह

1 Equatorial भमरखीय

2 Equatorial भमरखीय

3 Savannah सवाना

4 Mediterranean आातरक (भमसागरीय)

5 Mediterranean आातरक (भमसागरीय)

6 Monsoon मानसन

7 Monsoon मानसन

Correct Answer -

Equatorial भमरखीय

Equatorial भमरखीय

CANCELLED

In which epoch of the geological history of the Earth dinosaurs reached their largest size

पी क भगभय इितहास क िकस यग म डायनासोर अपन सबस बड़ आकार तक पचि

10)

11)

1 Triassic ट ायिसक

2 Jurassic जरािसक

3 Cretaceous ीटशस

4 Permian पिमयन

Correct Answer -

Jurassic जरािसक

CANCELLED

A spring tide would occur in which of the following conditions

िनिलखत म स िकन थितयो म ार-भाटा आता ह

1 When the Sun Moon and Earth are in a straight line

जब सय चमा और पी एक सीधी रखा म होत ह

2 When the Moon and Earth are in right angle to each other

जब चमा और पी एक-दसर क दािहन कोण म होत ह

3 When the Earth and Moon are in right angle to the Sun

जब पी और चमा सय क दािहन कोण म होत ह

4 When the Sun and Moon are in right angle to each other

जब सय और चमा एक-दसर क दािहन कोण म होत ह

Correct Answer -

When the Sun Moon and Earth are in a straight line

जब सय चमा और पी एक सीधी रखा म होत ह

CANCELLED

An observe on the Earthrsquos surface always sees the same face of the moon because

एक पयवक को पी की सतह स हमशा चाद का एक ही फलक िदखाई दता ह ोिक

1 Its path of revolution around the earth is the same as that of the earth around the Sun

इसका पी क चारो ओर परमण का माग सय क चारो ओर पी क समान ही ह

2 Its period of revolution around the Earth is the same as its period of rotation around its own axis

इसकी पी क चारो ओर परमण की अविध उसकी अपनी धरी क चारो ओर घणन की अविध क समान ह

3 Its period of rotation is the same as that of the Earth इसकी घणन की अविध पी क समान ह

ी ि ी ी

12)

13)

14)

4 Its direct of rotation is the same as that of Earth घणन की िदशा पी क समान ही ह

Correct Answer -

Its period of revolution around the Earth is the same as its period of rotation around its own axis

इसकी पी क चारो ओर परमण की अविध उसकी अपनी धरी क चारो ओर घणन की अविध क समान ह

CANCELLED

The pebbles that are faceted by the sand-blasting and shaped polished by the wind abrasions are known as

पवन अपघषन ारा पॉिलश रत-िवोिटत और साच म ढला ककड़ ___________ क प म जाना जाता ह

1 Dreikanter िकोणक

2 Pediments िकोिनका

3 Inselberg इलबग

4 Dunes टीबा

Correct Answer -

Dreikanter िकोणक

CANCELLED

Astronomical unit is the average distance between खगोलीय इकाई ______ क बीच की औसत दरी ह

1 Earth and Mars पी और मगल

2 Earth and mercury पी और बध

3 Earth and moon पी और चमा

4 Earth and Sun पी और सय

Correct Answer -

Earth and Sun पी और सय

During cold weather season in the northern plains there will be an inflow of cyclonic disturbancesfrom the _________ directions

शीत मौसम क दौरान उरी मदानी इलाको म _________ िदशाओ स चवात सबधी गड़बड़ी का अतवाह होगा

1 East and Northwest पव और उरपिम

2 East and Northeast पव और पवर

3 West and East पिम और पव

ि औ ि

15)

16)

17)

4 West and Northwest पिम और उरपिम

Correct Answer -

West and Northwest पिम और उरपिम

During an earth quake the velocity of the body waves will________ along with the increase in densityof the material it is passing through

भकप क दौरान लहरो क ऊपरी भाग का वग घन म व क साथ-साथ ________ जो इसस गजरन वाली वको आग बढाएगी

1 not change नही बदलगा

2 increase initially and then decrease शआत म बढ़गा और िफर घटगा

3 increase बढ़गा

4 decrease घटगा

Correct Answer -

increase बढ़गा

The Clouded Leopard National park is situated in which of the following states

िनिलखत म िकस रा म धिमल तदआ रा ीय उान (ाउडड लपड नशनल पाक ) थत ह

1 Tripura िपरा

2 Uttar Pradesh उर दश

3 Assam असम

4 Mizoram िमजोरम

Correct Answer -

Tripura िपरा

Usually the land surfaces are heated more quickly than the water surfaces because _____________

आम तौर पर जल सतहो की तलना म भिम सतह अिधक तजी स गम होती ह ोिक _____________ ह

1 the specific heat of water is higher than land पानी की िविश ऊा भिम स अिधक

2 the specific heat of water is lesser than land पानी की िविश ऊा भिम स कम होती

3 the latent heat of water is higher than the land पानी की अतिनिहत ऊा भिम स अिधक

4 the land reflects more heat radiation than water भिम पानी की तलना म अिधक ऊा क िविकरण को पराविततकरती

18)

19)

20)

21)

Correct Answer -

the specific heat of water is higher than land पानी की िविश ऊा भिम स अिधक

The longest shore-line is along the state of सबस लबी समतटीय रखा िन रा क साथ ह

1 Maharashtra महारा

2 Orissa उड़ीसा

3 Kerala करल

4 Gujarat गजरात

Correct Answer -

Gujarat गजरात

The position when the Earth is farthest from the Sun is known as

जब पी सय स सबस दर होती ह तो उस थित को िन नाम स जाना जाता ह

1 Perihelion उपसौर

2 Vernal Equinox बसत िवषव

3 Aphelion अपसौर

4 Autumnal Equinox शराल िवषव

Correct Answer -

Aphelion अपसौर

The seasonal reversal of winds is the typical characteristic of

हवाओ का मौसमी परवतन ______ की सामा िवशषता ह

1 Mediterranean climates only कवल भमसागरीय जलवाय

2 All of the above climates उपय सभी मौसम

3 Monsoon climate only कवल मानसन जलवाय

4 Equatorial climate only कवल भमरखीय जलवाय

Correct Answer -

Monsoon climate only कवल मानसन जलवाय

In _________ rocks the minerals will occurs in beds or layers

ो ि ो ो

22)

23)

24)

______ चानो म खिनज तल या परतो म होत ह

1 metamorphic कायातरत

2 igneous and metamorphic आय और कायातरत

3 igneous आय

4 sedimentary अवसादी

Correct Answer -

sedimentary अवसादी

Black soil is ideal for the cultivation of cotton as कपास की खती क िलए काली िमी आदश ह ोिक

1 Its colour is black यह काली होती ह

2 It is found on plateau regions यह पठार ो म पायी जाती ह

3 It is made up of lava यह लावा स बनी होती ह

4 It can retain moisture यह नमी को बरकरार रख सकती ह

Correct Answer -

It can retain moisture यह नमी को बरकरार रख सकती ह

The National Survey and Mapping Organization of the country works under the Department of___________

दश का रा ीय सवण और मानिचण सगठन ___________ िवभाग क अतगत काय करता ह

1 Space अतर

2 Science and Technology िवान और तकनीक

3 Culture सित

4 Tourism पयटन

Correct Answer -

Science and Technology िवान और तकनीक

Palk strait separates India from पाक जलडमम भारत स _____ को अलग करता ह

1 Pakistan पािकान

2 Andaman Island अडमान ीप

3 China चीन

25)

26)

27)

4 Sri Lanka ीलका

Correct Answer -

Sri Lanka ीलका

Which among the following state is the major producer of Bauxite in India

िनिलखत म स कौन सा रा भारत म बॉाइट का मख उादक ह

1 Madhya Pradesh मदश

2 Rajasthan राजथान

3 Goa गोवा

4 Orissa उड़ीसा

Correct Answer -

Orissa उड़ीसा

Which of the following states DOES NOT share border with Chhattisgarh

िनिलखत म स कौन सा रा छीसगढ़ क साथ सीमा साझा नही करता ह

1 Telangana तलगाना

2 Uttar Pradesh उर दश

3 Bihar िबहार

4 Andhra Pradesh आ दश

Correct Answer -

Bihar िबहार

Which of the following statements is INCORRECT with respect to parallels of latitudes

अाश क समानातरो क सबध म िन निलखत म स कौन सा कथन गलत ह

1 A line joining places of equal latitude is known as parallel of largest

समान अाश क थानो को जोड़न वाली रखा को िवशालतम क समानातर क प म जाना जाता ह

2 They stat from equator and run parallels to it

व भम रखा स ारभ होत ह और इसक समानातर चलत ह

3 All parallels are equal in length सभी समातर लबाई म समान ह

4 All parallels are drawn as circles on the globe ोब पर सभी समानातर वो क प म खीच जात ह

28)

29)

30)

31)

Correct Answer -

All parallels are equal in length सभी समातर लबाई म समान ह

Which of the following Indian states is also known as a lsquoLand of Red river and Blue Hillsrsquo

िनिलखत म स िकस भारतीय रा को लाल नदी और नीली पहािड़यो की भिम क नाम स जाना जाता ह

1 Uttarkhand उराखड

2 Assam असम

3 Meghalaya मघालय

4 Arunachal Pradesh अणाचल दश

Correct Answer -

Assam असम

In spatial analysis of settlement Rn = 215 indicates which type of settlement arrangement

िनपटान क थािनक िवषण म Rn = 215 यह इिगत करता ह िक िकस कार की िनपटान वथा ह

1 Uniform यिनफॉम

2 Semi-Clustered समी- ल टर

3 Clustered ल टर

4 Random रडम

Correct Answer -

Uniform यिनफॉम

Who are known as the lsquoYellow Peoplersquo lsquoयलो पीपलrsquo क प म कौन जाना जाता ह

1 Mongoloids मोगोलोइडस

2 Nigroids नीोइडस

3 Australoids ऑ लॉइडस

4 Caucasoids कॉकसोइडस

Correct Answer -

Mongoloids मोगोलोइडस

ि ि ो ौ ि

32)

33)

34)

Who publishes the topographical map of India भारत क थलाकितक मानिच को कौन कािशत करता ह

1 Geographical Survey of India भारत का भौगोिलक सवण

2 Government of India भारत सरकार

3 Geological Survey of India भारत क भगभय सवण

4 Survey of India भारत का सवण

Correct Answer -

Survey of India भारत का सवण

Who among the following claimed geography to be the lsquoEcology of Manrsquo

िनिलखत म स िकसन भगोल को मन का पारथितकी कहा ह

1 Alfred Hener अड हटनर

2 Vidal-de la Blache वाइडल-िड लॉ ॉश

3 Oo Schluter ओटो टर

4 Harlan Barrow हरलन बारो

Correct Answer -

Harlan Barrow हरलन बारो

Who among the following is regarded as the founder of humanistic approach in geography

िनिलखत म स िकस भगोल म मानवतावादी िकोण का सथापक माना जाता ह

1 William Bunge िविलयम बग

2 Yi-Fu-Tuan यी-फ- यान

3 Brain JL Berry न जएल बरी

4 Richard Peet रचड पीट

Correct Answer -

Yi-Fu-Tuan यी-फ- यान

Who prepared Lorenz curve लोरज व िकसन तयार िकया

1 Geddes गडस

2 None of these इनम स कोई नही

3 Griffith Taylor ििफथ टलर

35)

36)

37)

4 Max U Lorenz म य लोरज

Correct Answer -

Max U Lorenz म य लोरज

Gulf Streams are the currents of which of the following oceans

खाड़ी की धाराए िनिलखत महासागरो म स िकसकी धाराए ह

1 North Atlantic Ocean उरी अटलािटक महासागर

2 North Pacific Ocean उरी शात महासागर

3 Arabian Sea अरब सागर

4 South Pacific Ocean दिण शात महासागर

Correct Answer -

North Atlantic Ocean उरी अटलािटक महासागर

Disintegration wearing away and removal of rock material is generally referred as

िशला पदाथ (रॉक सामी) का टटना िमटना और हटना आमतौर पर ________ क प म सदिभत िकया जाता ह

1 Shattering िवसकारक

2 Denudation अनाादन

3 Fault श

4 Decomposition िवयोजन

Correct Answer -

Denudation अनाादन

Variations in the length of day time and night from season to season are due to

मौसम स मौसम परवतन पर िदन क समय और रात क समय की अविध म िभताए िन कारण स होती ह

1 The Earthrsquos revolution round the Sun in an elliptical manner पी का दीघवाकार तरीक स सय क चारो घणन

2 The Earthrsquos rotation on its axis पी का इसकी धरी पर घणन

3 Revolution of the Earth on a tilted axis नत अ पर पी का घणन

4 Latitudinal position of the place थान की अाश थित

Correct Answer -

Revolution of the Earth on a tilted axis नत अ पर पी का घणन

38)

39)

40)

Point out the correct sequence of mountain ranges from north to south

उर स दिण तक पवत खलाओ क सही अनम को इिगत कर

1 Great Himalaya Middle Himalaya Outer Himalaya Trans Himalaya

महान िहमालय म िहमालय बा िहमालय परा िहमालय

2 Middle Himalaya Great Himalaya Trans Himalaya Outer Himalaya

म िहमालय महान िहमालय परा िहमालय बा िहमालय

3 Outer Himalaya Middle Himalaya Great Himalaya Trans Himalaya

बा िहमालय म िहमालय महान िहमालय परा िहमालय

4 Trans Himalaya Great Himalaya Middle Himalaya Outer Himalaya

परा िहमालय महान िहमालय म िहमालय बा िहमालय

Correct Answer -

Trans Himalaya Great Himalaya Middle Himalaya Outer Himalaya

परा िहमालय महान िहमालय म िहमालय बा िहमालय

Sunrsquos halo is produced by the refraction of light in सय का भामडल ______ म काश क अपवतन ाराउ होता ह

1 Ice crystals in Cirrro-Cumulus clouds पाभ-कपास मघो क बफ िल

2 Ice crystal in Cirrus clouds पाभमघो क बफ िल

3 Dust particles in Stratus clouds री मघो क धल कण

4 Water vapour in Stratus clouds री मघो क जल वा

Correct Answer -

Ice crystal in Cirrus clouds पाभमघो क बफ िल

Read the given statements and answer which of the following options isare correct

(1) The minerals present in the rocks exposed to atmosphere are not subjected to alteration

(2) Oxidation is one of the processes of chemical weathering

िदए गए कथनो को पढ़ और उर द िक िन म स कौन सास िवक सही ह

(1) वायमल स अनावत शल म उपथत खिनज परवतन क अधीन नही होता ह

(2) ऑीकरण रासायिनक अपय की ियाओ म स एक ह

ो ो

41)

42)

1 Both statements are wrong दोनो कथन गलत ह

2 Both statements are correct दोनो कथन सही ह

3 First statement is wrong and second statement is correct पहला कथन गलत ह और दसरा कथन सही ह

4 First statement is correct and second statement is wrong पहला कथन सही ह और दसरा कथन गलत ह

Correct Answer -

First statement is wrong and second statement is correct पहला कथन गलत ह और दसरा कथन सही ह

Read the given statements and answer which of the following options isare correct

1 Sunrsquos short waves enter the earth partially heating the atmosphere

2 Heated earth surface from the sun produces broader waves which interacts and heats the atmosphere

िदए गए कथन को पढ़ और उर द िक िन म स कौन सास िवक सही ह

1 सय की छोटी तरग पी म आिशक प स वश करती ह और वायमडल को ऊत करती ह

2 सय स ऊत पी की सतह िवारत तरग उ करती ह जो परर भाव डालती ह और वायमडल कोऊत करती ह

1 Both Statements 1 and 2 are correct दोनो कथन 1 और 2 सही ह

2 Both Statements 1 and 2 are wrong दोनो कथन 1 और 2 गलत ह

3 Statement 1 is wrong and only Statement 2 is correct कथन 1 गलत ह और कवल कथन 2 सही ह

4 Only statement 1 is correct कवल कथन 1 सही ह

Correct Answer -

Both Statements 1 and 2 are correct दोनो कथन 1 और 2 सही ह

Read the given statements and answer which of the following options isare correct

(1)The rocks that get changed due to heat and pressure are termed as metamorphic rocks

(2)Slate is one such type of metamorphic rock

िदए गए कथनो को पढ़ और उर द िक िन म स कौन सास िवक सही ह

(1) शल जो ऊा और दाब क कारण परवितत हो जात ह उ कायातरक शलो क प म जाना जाता ह

(2) ट एक तरह का कायातरक शल ह

1 Both statements are wrong दोनो कथन गलत ह

2 Both statements are correct दोनो कथन सही ह

3 First statement is wrong and second statement is correct पहला कथन गलत ह और दसरा कथन सही ह

ी औ

43)

44)

4 First statement is correct and second statement is wrong पहला कथन सही ह और दसरा कथन गलत ह

Correct Answer -

Both statements are correct दोनो कथन सही ह

Read the given statements and answer which of the following options isare correct

1 Higher temperature anomaly is observed in the northern hemisphere

2 Differential heating is absent in Northern Hemisphere

िदए गए कथनो को पढ़ और उर द िक िन म स कौन सास िवक सही ह

1 उरी गोलाध म उ तापमान िवसगित पायी जाती ह

2 उरी गोलाध म अतर ऊन अनपथत होती ह

1 Both Statements 1 and 2 are correct दोनो कथन 1 और 2 सही ह

2 Both Statements 1 and 2 are wrong दोनो कथन 1 और 2 गलत ह

3 Statement 1 is wrong and Statement 2 is correct कथन 1 गलत ह और कथन 2 सही ह

4 Statement 1 is correct and Statement 2 is wrong कथन 1 सही ह और कथन 2 गलत ह

Correct Answer -

Statement 1 is correct and Statement 2 is wrong कथन 1 सही ह और कथन 2 गलत ह

Read the given statements and answer which of the following options isare correct

(1) Plutonic rocks are intrusive type of igneous rocks

(2) It cools very slowly because the surrounding rock serves as insulation around the intrusion of magma

िदए गए कथनो को पढ़ और उर द िक िन म स कौन सास िवक सही ह

(1) िवतलीय शल अतवधी कार क आश शल ह

(2) यह बत धीर-धीर ठडा होता ह ोिक आस-पास क शल मा क अतवधन क चारो ओर रोधन क प म कायकरत ह

1 Both statements are wrong दोनो कथन गलत ह

2 Both statements are correct दोनो कथन सही ह

3 First statement is wrong and second statement is correct पहला कथन गलत ह और दसरा कथन सही ह

4 First statement is correct and second statement is wrong पहला कथन सही ह और दसरा कथन गलत ह

Correct Answer -

Both statements are correct दोनो कथन सही ह

45)

46)

47)

48)

The dust and ash material hurled from the volcanoes are termed as

ालामखी स िनकलन वाली धल और राख सामी को _______ क प म कहा जाता ह

1 Pyroclasc पाइरोाक

2 Hyperclastic हाइपराक

3 Hepiroclastic हिपरोाक

4 Cirroclastic िसरोाक

Correct Answer -

Pyroclasc पाइरोाक

The vertical difference in elevation between a low tide and high tide is referred as

कम ार और उ ार क बीच ऊचाई म लबवत अतर _____ स सदिभत होता ह

1 Tidal slope ारीय ढलान

2 Tidal elevation ारीय उयन

3 Tidal range ारीय परास

4 Tidal height ारीय ऊचाई

Correct Answer -

Tidal range ारीय परास

The maximum biodiversity is found in which of the following regions िनिलखत ो म स अिधकतमजव िविवधता िकसम पायी जाती ह

1 Amazon Basin अमज़न बिसन

2 East Indies ई इडीज

3 Congo Basin कागो बिसन

4 West indies व इडीज

Correct Answer -

Amazon Basin अमज़न बिसन

The cultivation of rice crop produces_______ चावल की फसल की खती ______ का उादन करती ह

1 SO2

49)

50)

51)

2 CH4

3 CFCs

4 CO2

Correct Answer -

CH4

The pressure system with higher pressure at the centre is called__________

क म उ दबाव वाली दबाव णाली को _______ कहा जाता ह

1 front अ

2 depression अवनमन

3 cyclone चवात

4 anti-cyclone ितचवात

Correct Answer -

anti-cyclone ितचवात

The Himalayan region is poor in mineral resources because िहमालयी खिनज ससाधनो म समनही ह ोिक

1 The displacement of rock strata has disturbed the arrangement of rocks and made it complex

शलीय परत क िवथापन न चानो की वथा को अवथत कर िदया ह और इस जिटल बना िदया ह

2 The climate conditions are not suitable for exploitation of minerals

जलवाय की थित खिनजो क दोहन क िलए उपय नही ह

3 The terrain makes explanation of minerals difficult and very costly due to transportation difficulties

भ-भाग परवहन की किठनाइयो क कारण खिनजो का दोहन मल और बत महगा बना दता ह

4 It is made up of crystalline rocks यह िलीय चानो स बना ह

Correct Answer -

The displacement of rock strata has disturbed the arrangement of rocks and made it complex

शलीय परत क िवथापन न चानो की वथा को अवथत कर िदया ह और इस जिटल बना िदया ह

The process through which the moisture is added to the atmosphere by vegetation is termed as

वह िया िजसक माम स वनित ारा वातावरण म नमी िमलायी जाती ह _______ क प म जानी जाती ह

52)

53)

54)

1 Condensation सघनन

2 Evapotranspiration वान-उजन

3 Radiation िविकरण

4 Precipitation वषण

Correct Answer -

Evapotranspiration वान-उजन

The process through which the terrestrial heat is transferred to air by direct contact is termed as

वह िया िजसम सपक ारा थलीय ऊा वाय म थानातरत हो जाती ह ______ क प म जानी जाती ह

1 Conduction चालन

2 Convection सवहन

3 Insolation आतपन

4 Radiation िविकरण

Correct Answer -

Conduction चालन

The largest area under mangroves is in which of the following statesunion territory

मोव क अतगत िनिलखत राोसघ शािसत दशो म स सबस बड़ा कौन सा ह

1 Andaman and Nicobar अमान और िनकोबार

2 Andhra Pradesh आ दश

3 West Bengal पिम बगाल

4 Gujarat गजरात

Correct Answer -

West Bengal पिम बगाल

The longitudinal transverse and surface waves in an earthquake originate from

भकप म दशातर अनथ और सतह तरग यहा उ होती ह

1 The focus on the surface of the Earth पी क सतह पर क -िबद म

2 The focus within the body of the Earth पी क भीतर क -िबद म

3 The epicenter within the body of the Earth पी क भीतर उपरक म

55)

56)

57)

4 The epicenter on the surface of the Earth पी क सतह पर उपरक म

Correct Answer -

The focus within the body of the Earth पी क भीतर क -िबद म

The down slope movement of material due to gravity is called______

गाकषण क कारण पदाथ की अनढाल गित को ______ कहा जाता ह

1 mass movement पदाथ सचलन

2 deposition िनप

3 erosion रण

4 volcanic movement ालामखीय सचलन

Correct Answer -

mass movement पदाथ सचलन

Shimla is cooler than Amritsar although both are on the same latitude This is because

िशमला म अमतसर स अिधक ठड ह हालािक दोनो समान अाश पर ह ऐसा ह ोिक

1 Shimla is at a greater height above sea level than Amritsar अमतसर की तलना म िशमला सम तल स अिधकऊचाई पर ह

2 Shimla is further north िशमला उर की ओर ह

3 Shimla is farther from the equator िशमला भम रखा स आग ह

4 Their longitudes differ उनकी दशातर रखाए िभ ह

Correct Answer -

Shimla is at a greater height above sea level than Amritsar अमतसर की तलना म िशमला सम तल स अिधकऊचाई पर ह

lsquoTempo of Urbanizationrsquo measures which of the following

lsquoशहरीकरण का टपोrsquo िनिलखत म स कौन सा उपाय ह

1 Speed of urbanizaon शहरीकरण की गित

2 None of the above इनम स कोई नही

3 Inequality of urbanizaon शहरीकरण की असमानता

4 Current level of urbanizaon शहरीकरण का वतमान र

Correct Answer -

58)

59)

60)

Speed of urbanizaon शहरीकरण की गित

Out of the following options choose the INCORRECT statement

िनिलखत िवको म स गलत कथन का चयन कर

1 The clear tracts in the equatorial region recover rapidly भम रखा म भभाग तजी स ठीक हो जात ह

2 The stable communities include a redwood forest a pine forest at high elevations

थर समदायो म एक रडवड वन उ ऊचाई पर एक दवदार वन शािमल ह

3 Any ecosystem moves towards maximum biomass and stability to survive

कोई भी पारथितकी त जीिवत रहन क िलए अिधकतम जवसहित और थरता की तरफ असर होता ह

4 Tropical rain forests near equator are stable ecosystems

भम रखा क पास उकिटबधीय वषा वन थर पारथितक त ह

Correct Answer -

The clear tracts in the equatorial region recover rapidly भम रखा म भभाग तजी स ठीक हो जात ह

Seasonal contrasts are maximum in मौसमी िवषमता अिधकतम ह

1 Mid latitudes म अाश म

2 Low attitudes िन अाश म

3 High latitudes उ अाश म

4 Subtropics उपोकिटबधीय म

Correct Answer -

Mid latitudes म अाश म

In India which type of forest among the following occupies the largest area

भारत म िनिलखत म स िकस कार क वन सबस बड़ा फल आािदत करत ह

1 Sub-tropical Dry Evergreen Forest उप उकिटबधीय श सदाबहार वन

2 Mountain Wet Temperate Forest पवतीय आ शीतो वन

3 Tropical Moist Deciduous Forest उकिटबधीय आ पणपाती वन

4 Tropical Wet Evergreen Forest उकिटबधीय आ सदाबहार वन

Correct Answer -

Tropical Moist Deciduous Forest उकिटबधीय आ पणपाती वन

61)

62)

63)

64)

What is the proportion of lsquoJuvenile Populationrsquo (0-14 years) in India as per 2011Census

2011 की जनगणना क अनसार भारत म जवनाइल पॉपलशन यानी िकशोर जनस या (0-14 वष) का अनपात ाह

1 3076 of total population कल जनस या का 3076

2 2764 of total population कल जनस या का 2764

3 2933 of total population कल जनस या का 2933

4 3354 of total population कल जनस या का 3354

Correct Answer -

3076 of total population कल जनस या का 3076

What is the Belfast famous for बलफा िकसक िलए मशर ह

1 Belt of cotton textile industry कपास व उोग क

2 Ship-building industry जहाज िनमाण उोग

3 Agricultural machinery किष उपकरण

4 Aero planes manufacturing वाययान िनमाण

Correct Answer -

Ship-building industry जहाज िनमाण उोग

What is the most important occupation in tropical monsoon lands

उकिटबधीय मॉनसन भिम म सबस महपण वसाय ा ह

1 Mining खनन

2 Cattle rearing मवशी पालन

3 Agriculture किष

4 Nomadic herding नोमािडक जड़ी-बिटया

Correct Answer -

Agriculture किष

What is the most important characteristics of the islands (Indian) located in the Arabian sea

अरब सागर म थत ीपो (भारतीय) की सबस महपण िवशषता ा ह

ी ो

65)

66)

67)

1 There are all of coral origins सभी कोरल मल क ह

2 There are all very small in size य सभी आकार म बत छोट ह

3 They have a very dry climate इनकी जलवाय बत श ह

4 They are extended parts of the mainland व महाीप क िवारत िह ह

Correct Answer -

There are all of coral origins सभी कोरल मल क ह

What do the basalt layers of the Deccan indicate डन की बसा परत ा इिगत करती ह

1 All of the above उपरो सभी

2 Huge volcanic eruptions in the distant past दरथ अतीत म िवशाल ालामखीय िवोट

3 The immense erosional activity of the rivers निदयो की िवशाल रण गितिविध

4 The influence of weathering मौसम का भाव

Correct Answer -

Huge volcanic eruptions in the distant past दरथ अतीत म िवशाल ालामखीय िवोट

In the structure of planet Earth below the mantle the core is mainly made up of_____

पी ह की सरचना म मटल क नीच कोर म प स______ स िनिमत होती ह

1 aluminium एमीिनयम

2 silicon िसिलकॉन

3 chromium ोिमयम

4 iron लोहा

Correct Answer -

iron लोहा

One of the major Mid Oceanic Ridge is found in मख म-महासागर चोिटयो म स एक ______ म पायाजाता ह

1 Mid Pacific Ocean म शात महासागर

2 Mid Atlantic Ocean म अटलािटक महासागर

3 Mid Indian Ocean म भारतीय महासागर

4 Mid Arctic Ocean म आक िटक महासागर

68)

69)

70)

71)

Correct Answer -

Mid Atlantic Ocean म अटलािटक महासागर

Magma that reaches the Earthrsquos surface and then solidifies is called________

मा जो पी की सतह तक पचती ह और िफर ठोस हो जाती ह ________कहलाती ह

1 quartz ाटज

2 lava लावा

3 granite नाइट

4 silicates िसिलकट

Correct Answer -

lava लावा

Isotherms are the lines of equal_______ समताप रखाए समान _______की रखाए होती ह

1 pressure दाब

2 temperature तापमान

3 rainfall वषा

4 height ऊचाई

Correct Answer -

temperature तापमान

Mark the correct sequence of passes in the Western Ghats from north to south

पिमी घाटो म उर स दिण तक दर क सही अनम को िचित कर

1 Thalghat Palghat Bhorghat थलगघाट पालघाट भोरघाट

2 Thalghat Bhorghat Palghat थलघाट भोरघाट पालघाट

3 Bhorghat Thalghat Palghat भोरघाट थलघाट पालघाट

4 Palghat Bhorghat Thalghat पालघाट भोरघाट थलघाट

Correct Answer -

Thalghat Bhorghat Palghat थलघाट भोरघाट पालघाट

Which of the following does not have influence over the climate in India

ि ि ि ी ी

72)

73)

िनिलखत म स िकसका भाव भारत की जलवाय पर नही पड़ता ह

1 Ocean currents सागर की लहर

2 Nearness to equator भम रखा स िनकटता

3 Monsoons मानसन

4 Presence of Indian ocean भारतीय महासागर की उपथित

Correct Answer -

Ocean currents सागर की लहर

Which of the following cloud types has the characteristics like vertical tall narrow and puffy

िनिलखत म स िकस कार क मघो म लबवत लबी सकीण और थलता जसी िवशषताए ह

1 Cumulonimbus तफानी मघ

2 Cumulus मघ पज

3 Cirrocumulus पाभ कपासी मघ

4 Nimbostratus वषारी मघ

Correct Answer -

Cumulus मघ पज

Which of the following statement is INCORRECT about Crude Birth Rate

िनिलखत स कौन सा कथन अशोिधत ज दर क बार म सही नही ह

1 It cannot be used for comparing fertility level between two countries with different population characteristics

इसका उपयोग िविभ जनसा िवशषताओ वाल दो दशो क बीच जनन र की तलना क िलए नही िकया जा सकता ह

2 It is a standardized measure of fertility

यह जनन मता का मानकीकत उपाय ह

3 It is effected by the age-sex composition of the population

यह आबादी की आय-िलग सरचना स भािवत होता ह

4 It is expressed per 1000 population in a given geographical unit

यह िकसी दी गई भौगोिलक इकाई म ित 1000 जनसा पर िकया जाता ह

Correct Answer -

It is a standardized measure of fertility

यह जनन मता का मानकीकत उपाय ह

74)

75)

76)

77)

Which of the following state in India experienced negative decadal growth rate during 2001 to 2011census

भारत म िनिलखत म स िकस रा म वष 2001 स 2011 की जनगणना क दौरान नकाराक िगरावट दर ई

1 Tripura िपरा

2 Nagaland नागालड

3 Haryana हरयाणा

4 Odisha ओिडसा

Correct Answer -

Nagaland नागालड

Which of the following is NOT a characteristic of peninsular rivers

िनिलखत म स कौन सी िवशषता ायीपीय निदयो म नही होती ह

1 Flow through shallow valleys उथल घािटयो क माम स वाह

2 Seasonal flow मौसमी वाह

3 Little erosional activity थोड़ी कटावदार गितिविध

4 Meandering tendency often shifting their beds घमावदार वि अर अपन तटो को थानातरत करना

Correct Answer -

Meandering tendency often shifting their beds घमावदार वि अर अपन तटो को थानातरत करना

Which of the following gases in the atmosphere absorbs heat from the Sunrsquos radiation and the Earthssurface

वायमडल म िनिलखत म स कौन सी गस सय क िविकरण और पी की सतह स ऊा को अवशोिषत करती ह

1 Neon िनयॉन

2 Carbon dioxide काबन डाइऑाइड

3 Argon आगन

4 Nitrogen नाइट ोजन

Correct Answer -

Carbon dioxide काबन डाइऑाइड

Which of the following kind of settlement pattern is found at the confluence of rivers

ि ि ि ि ो

78)

79)

80)

िनिलखत म स िकस कार का वथापन पटन निदयो क सगम पर पाया जाता ह

1 Triangular Paern िकोणीय पटन

2 Circular or Semi-Circular Paern परप या अध-परप पटन

3 Nebular Paern नबलर पटन

4 Star ndashShaped Paern ार-आकार का पटन

Correct Answer -

Triangular Paern िकोणीय पटन

Which one was not the objective of the Biosphere Reserve Projects launched by the UNESCO

यनो ारा श की गई सरित जवमडल परयोजनाओ का उ इनम स कौन सा नही था

1 To promote teaching and research िशण और अनसधान को बढ़ावा दना

2 To make agriculture sustainable किष को दीघकािलक बनाना

3 To conserve ecosystems पारथितक त को सरित करना

4 To conserve genetic diversity for a longtime लब समय तक अनवािशक िविवधता को सरित करना

Correct Answer -

To make agriculture sustainable किष को दीघकािलक बनाना

Which region of the Earth surface receives the highest amount of insulation

पी सतह का कौन सा तापावरोधन की उतम माा ा करता ह

1 Land mass थलखड

2 Savannah region सवाना

3 Water bodies जल िनकाय

4 Tropical desert उकिटबधीय रिगान

Correct Answer -

Tropical desert उकिटबधीय रिगान

Which one of the following is not a biodiversity hotspot

िनिलखत म स कौन सा जव िविवधता का म जगह नही ह

1 Eastern Himalaya पव िहमालय

2 Eastern Ghats पव घाट

81)

82)

83)

3 Indo-Myanmar भारत-ामार

4 Westerm Ghats पिमी घाट

Correct Answer -

Eastern Ghats पव घाट

Which one of the following is NOT a part of the World Network of Biosphere Reserves based on theUNESCO Man and Biosphere Programme

यनो मन और बायोीयर कायम क आधार पर िनिलखत म स कौन बायोीयर रजव क िव नटवक कािहा नही ह

1 Gulf of Mannar मार की खाड़ी

2 Seshachalam शषाचलम

3 Sunderban सदरबन

4 Nilgiri नीलिगर

Correct Answer -

Seshachalam शषाचलम

Which one of the following is an example of ldquodesert vegetationrdquo

िनिलखत म स कौन मथलीय वनित का एक उदाहरण ह

1 Mosses and lichens दलदल और शवाल

2 Temperate grassland समशीतो घास क मदान

3 Coniferous forest शकधारी वन

4 Acacia and cactus एकािसया और कस

Correct Answer -

Acacia and cactus एकािसया और कस

Which one of the following reflects more sunlight िनिलखत म स कौन सा सय की रोशनी को अिधकपरावितत करता ह

1 Paddy crop land धान फसल भिम

2 Land covered with fresh snow ताजा बफ स आािदत भिम

3 Sand desert रतीली रिगान

4 Prairie land यरी भिम

84)

85)

86)

87)

Correct Answer -

Land covered with fresh snow ताजा बफ स आािदत भिम

Which layer of the atmosphere is in contact with the surface of the earthrsquos oceans

वायमडल की कौन सी परत पी क महासागरो की सतह क सपक म ह

1 Stratosphere समताप मडल

2 Mesosphere म मडल

3 Hydrosphere जलमडल

4 Troposphere ोभ मडल

Correct Answer -

Troposphere ोभ मडल

Mediterranean Sea is a border of which of the following countries भम सागर िनिलखत दशो म सिकसकी सीमा ह

1 None of these इनम स कोई नही

2 Iraq इराक

3 Lebanon लबनान

4 Jordan जॉडन

Correct Answer -

Lebanon लबनान

Benguela ocean currents are found along which coast बगएला महासागर धाराए िकस तट क साथ पायीजाती ह

1 East Coast of South America दिण अमरका क पव तट

2 East Coast of Africa अीका क पव तट

3 West Coast of South America दिण अमरका क पिमी तट

4 West Coast of Africa अीका क पिमी तट

Correct Answer -

West Coast of Africa अीका क पिमी तट

88)

89)

90)

Due to tension a block of land on one side being pushed up or upthrown relative to the downthrown blockis referred as

तनाव क कारण नीच फ क ए खड क साप भिम का एक खड एक ओर स ऊपर धकला जाता ह या ऊपर की ओरफ का जाता ह यह _____ क प म सदिभत ह

1 Thrust fault प श

2 Normal fault सामा श

3 Reverse fault म श

4 Strike slip fault नितलब सपण श

Correct Answer -

Normal fault सामा श

Inter-tropical doldrums is a zone of ______ अतर-उकिटबधीय डोलड ______ का एक ह

1 Frontolysis टोलायिसस

2 Convergence अिभसरण

3 Inter-tropical divergence zone अतर-उकिटबधीय िवचलन

4 Local wind थानीय वाय

Correct Answer -

Convergence अिभसरण

The Horse Latitudes are regions located at about _____ north and south of the equator

हॉस अाश भम रखा क उर और दिण म लगभग _____ पर थत ह

1 30ndash60 degree Latitude 30-60 िडी अाश

2 0ndash5 degree Latitude 0-5 िडी अाश

3 30 degree Latitude 30 िडी अाश

4 60ndash90 degree Latitude 60-90 िडी अाश

Correct Answer -

30 degree Latitude 30 िडी अाश

Generally evaporation is high over which part of the Earth

आम तौर पर पी क िकस भाग पर वाीकरण अिधक होता ह

1 Equatorial maritime भमवत समीय ी ी

91)

92)

2 Equatorial continental भमवत महाीपीय

3 Polar maritime वीय समीय

4 Polar continental वीय महाीपीय

Correct Answer -

Equatorial maritime भमवत समीय

A very high temperature during summer in north western India leads to what type of climaticcondition in south

उर पिमी भारत म गम क दौरान बत अिधक तापमान होन क कारण दिण म िकस कार की जलवाय थितउ करता ह

1 Depression over arabian sea अरब सागर पर अवनमन

2 Failure monsoon मानसन िवफलता

3 Successful monsoon मानसन सफलता

4 Cyclones चवात

Correct Answer -

Successful monsoon मानसन सफलता

Lightning and thunder are the resultant effect when तिड़त और गजन परणामी भाव ह जब

1 Two massive clouds hit powerfully each other first lightning is produced and later sound is produced

दो बड़ बादल एक दसर स शशाली ढग स टकरात ह पहल आकाशीय िवदयत उ होता ह और बाद म िन उहोती ह

2 Two massive clouds come into contact with the powerful wind collision this results into first sound and thenlightning

दो बड़ बादल शशाली पवन सघ क सपक म आत ह इसका परणामप पहल िन और िफर आकाशीय िवदयतउ होता ह

3 None of the above उपरो म स कोई भी नही

4 A high density cloud contains positively and negatively charged electric ions and when this interacts light andsound are simultaneously produced

एक उ घन बादल म धनाक और ऋणाक आविशत िवदयत आयन होत ह और जब यह परर भाव डालत ह तोकाश और िन एक साथ उािदत होती ह

Correct Answer -

A high density cloud contains positively and negatively charged electric ions and when this interacts light andsound are simultaneously produced

औ ि ि ो औ ो

93)

94)

95)

एक उ घन बादल म धनाक और ऋणाक आविशत िवदयत आयन होत ह और जब यह परर भाव डालत ह तोकाश और िन एक साथ उािदत होती ह

Doon Valley is able to grow rice because दन घाटी चावल उगान म सम ह ोिक

1 Other crops cannot be grown वहा अ फसलो को उगाया नही जा सकता ह

2 People in the valley are rice eaters घाटी म लोग चावल खान वाल ह

3 There is a huge export demand of rice वहा चावल की भारी िनयात माग ह

4 It has warm summer and snow melt waters for irrigation

वहा गिमया गम होती ह िसचाई क िलए बफ का िपघला आ पानी होता ह

Correct Answer -

It has warm summer and snow melt waters for irrigation

वहा गिमया गम होती ह िसचाई क िलए बफ का िपघला आ पानी होता ह

CANCELLED

In the geological time scale the Mesozoic Era DOES NOT contains which of the following periods

भगभय समय पमान पर मजीवी यग म िन कालो म स कौन नही ह

1 Triassic ट ाइऐिसक

2 Jurassic जरिसक

3 Cretaceous चाकमय

4 Carboniferous काबनी

Correct Answer -

Carboniferous काबनी

96)

1 P-3 Q-4 R-2 S-1

2 P-3 Q-4 R-1 S-2

3 P-3 Q-4 R-1 S-2

4 P-4 Q-3 R-2 S-1

Correct Answer -

P-4 Q-3 R-2 S-1

1 P-3 Q-1 R-4 S-2

2 P-3 Q-4 R-1 S-2

3 P-3 Q-2 R-4 S-1

97)

98)

4 P-2 Q-1 R-4 S-3

Correct Answer -

P-3 Q-4 R-1 S-2

1 P-3 Q-1 R-4 S-2

2 P-2 Q-3 R-4 S-1

3 P-2 Q-1 R-3 S-4

4 P-4 Q-2 R-1 S-3

Correct Answer -

P-3 Q-1 R-4 S-2

99)

100)

1 P-3 Q-2 R-4 S-1

2 P-1 Q-2 R-3 S-4

3 P-2 Q-3 R-1 S-4

4 P-4 Q-3 R-2 S-1

Correct Answer -

P-2 Q-3 R-1 S-4

ldquoHuman geography is the study of changing relationship between the unresting man and the unstableearthrdquo was defined by

lsquolsquoमानव भगोल ाकल आदमी और अथर पी क बीच सबध परवतन का अयन हrdquo ______ ारा परभािषत िकया गयाथा

1 J Brunches ज चस

2 EC Semple ईसी सल

3 HJ Mackinder एच ज मिकदर

4 PV Blache पीवी च

Correct Answer -

EC Semple ईसी सल

Sedimentary rocks are finally and ultimately derived from the____________

अवसादी चान अततः ________ स ा की जाती ह

1 action of earth movements पी की गितिविधयो

2 marine deposit समी िनप

3 weathering of metamorphic rocks पातरत चानो क अपय

4 weathering of igneous rocks आय चानो क अपय

Correct Answer -

weathering of igneous rocks आय चानो क अपय

Page 10: High School Teacher Eligibility Test- BOARD PROFESSIONAL ...peb.mp.gov.in/results/RESULT_18/HST_RES18/Final_anwser_key/HST… · M a ndl a / मंड ल ... Under the Madhya Pradesh

2)

3)

4)

Correct Answer -

36

Find the average औसत ात कर

25 35 45 55 65 amp 75

1 48

2 50

3 47

4 49

Correct Answer -

50

The below series uses a sequence of alphabets and numbers Identify the incorrect combination

नीच दी गई खला अरो एव साओ का अनम योग करती ह गलत सयोजन पहचान

(i) FT85DF77ER

(ii) FT85DF77ER

(iii) FT85DE77ER

(iv) FT85DF77ER

1 ii

2 iv

3 i

4 iii

Correct Answer -

iii

A and B together can complete a piece of work in 10 days and B alone can complete the same work in 20days In how many days can A alone complete the work

A और B एक साथ काम क एक भाग को 10 िदनो म समा त करत ह और B अकल उसी काम को 20 िदनो म समा त करता हिकतन िदनो म A अकल उस काम को समा त कर सकता ह

1 20 days 20 िदन

2 30 days 30 िदन

3 10 days 10 िदन

4 25 days 25 िदन

5)

6)

Correct Answer -

20 days 20 िदन

Which one of the following four addresses is NOT EXACTLY same as the one given below

िनिलखत िदए गए चार पतो म स नीच िदए गए पत क ठीक समान कौन सा एक नही ह

Ground Floor 59 Goulburn St

Sydney NSW 2000 Australia

+61 (02) 8987 3700

(i) Ground Floor 59 Goulburn St

Sydney NSW 2000 Australia

+61 (02) 8987 3700

(ii) Ground Floor 59 Goulburn St

Sydney NSW 2000 Australia

+61 (02) 8987 3700

(iii) Ground Floor 59 Govlburn St

Sqdney NSW 2000 Australia

+61 (02) 8987 3700

(iv) Ground Floor 59 Goulburn St

Sydney NSW 2000 Australia

+61 (02) 8987 3700

1 ii

2 iv

3 i

4 iii

Correct Answer -

iii

In a group of 75 people 32 of them like cold drink 56 of them like general water and each person likesat least one of the two drinks How many people like both

75 लोगो क समह म उनम स 32 को को िड क पसद ह उनम स 56 को सामा जल पसद ह और क को दो म स कम स कम एक पय पसद ह िकतन लोगो को दोनो पसद ह

1 11

2 13

7)

1)

2)

3 12

4 14

Correct Answer -

13

If A = 1 FAT = 27 then FAITH =

यिद A = 1 FAT = 27 तो FAITH =

1 41

2 40

3 42

4 44

Correct Answer -

44

Topic- PEDAGOGY

CANCELLED

Basic education became a major initiative because of the efforts of

ाथिमक िशा िन न क यास स एक मह वपण कायम बन गई

1 Dr Radhakrishnan डॉ राधाक णन

2 Dr Rajendra Prasad डॉ राज साद

3 Mahatma Gandhi महा मा गाधी

4 Rabindranath Tagore रबी नाथ टगोर

Correct Answer -

Mahatma Gandhi महा मा गाधी

The ldquoclause of phraserdquo is an unit of perception of लॉज ऑफ ज िन न क धारणा क एक इकाई ह

1 Image छिव

2 Concept अवधारणा

3 Language भाषा

4 Thought िवचार

Correct Answer -

3)

4)

5)

6)

Language भाषा

Which method is most suitable to study communication process among students

छाो क बीच सचार िया का अयन करन क िलए कौन सी िविध सबस उपय ह

1 Case Study मामल का अयन (कस टडी)

2 Systematic Observation वथत अवलोकन

3 Experimental Method योगाक िविध

4 Introspection आ-िनरीण (इट ोस शन)

Correct Answer -

Systematic Observation वथत अवलोकन

Genes in a human being are located in मानवो म जीन इनम थत होता ह

1 cytoplasm कोिशका (साइटो ला म)

2 ribosomes राइबोसोम

3 cell membranes कोिशका िझी

4 chromosomes गणस (ोमोसोम)

Correct Answer -

chromosomes गणस (ोमोसोम)

With smaller classes teachers are much more able to ____________

छोटी काओ क साथ िशक ____________ म अिधक सम होत ह

1 Go slow while teaching िशण क दौरान धीमी गित स जान

2 Narrate more personal experiences अिधक गत अनभवो को बतान

3 Adapt instruction to individual differences among students छाो क बीच गत मतभदो क िलए अनकल िनदश दन म

4 Make use of the extra space for extra-curricular activities पातर गितिविधयो क िलए अितर जगह का उपयोग करन

Correct Answer -

Adapt instruction to individual differences among students छाो क बीच गत मतभदो क िलए अनकल िनदश दन म

While engaging in a task the child gets bored This is a sign of

एक काम म होन पर बा ऊब जाता ह यह िन का सकत ह

1 the task requiring a professional approach काय को ावसाियक िकोण की आवकता ह

7)

8)

9)

2 the task becoming mechanically repetitive काय यािक प स दोहराव वाला ह

3 the child not being intelligent बा बमान नही ह

4 the child being incapable of learning बा सीखन म असमथ ह

Correct Answer -

the task becoming mechanically repetitive काय यािक प स दोहराव वाला ह

Educational Psychologists are more concerned with the learning in __________

शिणक मनोवािनक __________ म अिधगम क साथ अिधक िचितत होत ह

1 Formal environment औपचारक वातावरण

2 Informal environment अनौपचारक वातावरण

3 Physical environment भौितक वातावरण

4 Social environment सामािजक वातावरण

Correct Answer -

Formal environment औपचारक वातावरण

Special needs education is the type of education िवशष ज़रतो वाली िशा वह िशा होती ह जो

1 Given to person with disabilities अम य को दी जाती ह

2 Given to people from remote areas दर थ ो क लोगो को दी जाती ह

3 Provided to intelligent people बमान लोगो को दी जाती ह

4 Established by colonial masters औपिनविशक मखयाओ ारा थािपत की गई ह

Correct Answer -

Given to person with disabilities अम य को दी जाती ह

The Stanford-Binet scale of intelligence was first published in the year

ब क नफोड-िबनट कल को इस वष म पहली बार कािशत िकया गया था

1 1916

2 1903

3 1908

4 1900

Correct Answer -

1916

10)

11)

12)

13)

The term lsquofictional finalismrsquo was propounded by

पद lsquoकत योजनवादrsquo (िफ शनल फाइनिल म) इनक ारा ितपािदत िकया गया था

1 Skinner नर

2 Freud ायड

3 Adler एडलर

4 Pavlov पावलोव

Correct Answer -

Adler एडलर

Rational Emotive Behavior Therapy was propounded by

तक सगत भावनाक वहार थरपी िन क ारा ितपािदत की गई थी

1 Carl Jung काल यग

2 Carl Rogers काल रोजस

3 Aaron Beck आरोन बक

4 Albert Ellis अट एिलस

Correct Answer -

Albert Ellis अट एिलस

Which of the following indicates the quality of education in a school

िनिलखत म स या एक कल म िशा की गणव ता का सकतक ह

1 Text-books and Teaching-learning material पा-प तक तथा िशण व सीखन स सबिधत सामी

2 Infrastructural facilities at the school कल म आधारभत ढाच स सबिधत सिवधाए

3 Student achievement level िवािथयो का उपल तर

4 Classroom systems का की यव था

Correct Answer -

Student achievement level िवािथयो का उपल तर

Which of the following teachers can be identified with authoritarian teaching style

िनिलखत म स कौन सा िशक अिधकारवादी िशण शली क साथ पहचाना जा सकता ह

1 Laissez-faire teacher अब ध िशक

14)

15)

2 Democratic teacher लोकतीय िशक

3 Indifferent teacher िन प िशक

4 Direct instruction teacher िनदश िशक

Correct Answer -

Direct instruction teacher िनदश िशक

Who was the pioneer of classical conditioning

िचरितित ानकलन ( लािसकल कडीशिनग) क वतक कौन थ

1 Skinner नर

2 Pavlov पावलोव

3 Kohler कोहलर

4 Freud ायड

Correct Answer -

Pavlov पावलोव

Which of the following are true with reference to short term memory

1 Limited capacity

2 Brief storage of information

3 Unlimited capacity

4 Duration of storage less than twenty seconds

अ पकािलक मित क सदभ म िन न म स या स य ह

1 सीिमत मता

2 सचना का सि त भडारण

3 असीिमत मता

4 बीस सकड स कम भडारण की अविध

1 3 and 4 3 और 4

2 2 and 4 2 और 4

3 13 and 4 13 और 4

4 12 and 4 12 और 4

Correct Answer -

12 and 4 12 और 4

1)

2)

3)

4)

Topic- GEOGRAPHY

Which of the following ranges of population size is used to define Class-III city by Indian Census

भारतीय जनगणना ारा वग-III शहर को परभािषत करन क िलए िनिलखत म स िकस रज क जनसा आकारका उपयोग िकया जाता ह

1 20 000 to 49999 20 000 स 49999

2 30000 to 59999 30000 स 59999

3 24000 to 54999 24000 स 54999

4 50000 to 99999 50000 स 99999

Correct Answer -

20 000 to 49999 20 000 स 49999

Carbonaceous rocks which produce coal and oil belong to the category of rocks called_______

कोयल और तल का उादन करन वाली काबनय चान ______ नामक चानो की णी स सबिधत ह

1 metamorphic पातरत

2 sedimentary अवसादी

3 inorganic अजिवक

4 igneous आय

Correct Answer -

sedimentary अवसादी

The ruhr-complex is a major industrial centre in र-परसर िन का एक मख औोिगक क ह

1 North America उरी अमरका

2 Russia स

3 Germany जमनी

4 Europe यरोप

Correct Answer -

Germany जमनी

The term lsquoRegurrsquo refers to श lsquoरगरrsquo ______ स सबिधत ह

1 Deltaic alluvial soils डा जलोढ़ िमी

ि ी

5)

6)

2 Laterite soils लटराइट िमी

3 Red and yellow soils लाल और पीली िमी

4 Black cotton soils काली कपास िमी

Correct Answer -

Black cotton soils काली कपास िमी

Read the given statements and answer which of the following options isare correct

1 Lower the pressure greater the atmospheric disturbance

2 Air move from higher to low pressure

िदए गए कथन को पढ़ और उर द िक िन म स कौन सास िवक सही ह

1 िजतना दाब कम होगा वायमडलीय बाधाए उतनी अिधक होगी

2 वाय उ स िन दाब की ओर गित करती ह

1 Both Statements 1 and 2 are correct दोनो कथन 1 और 2 सही ह

2 Both Statements 1 and 2 are wrong दोनो कथन 1 और 2 गलत ह

3 Statement 1 is wrong and only Statement 2 is correct कथन 1 गलत ह और कवल कथन 2 सही ह

4 Statement 1 is correct and Statement 2 is wrong कथन 1 सही ह और कथन 2 गलत ह

Correct Answer -

Both Statements 1 and 2 are correct दोनो कथन 1 और 2 सही ह

CANCELLED

1 4 1 2 and 3 4 1 2 और 3

2 4 1 3 and 2 4 1 3 और 2

3 1 4 2 and 3 1 4 2 और 3

4 1 4 3 and 2 1 4 3 और 2

Correct Answer -

7)

8)

9)

1 4 3 and 2 1 4 3 और 2

CANCELLED

Karl Pearsonrsquos correlation co-efficient is काल िपयसन का सहसबध गणाक ह

1 Arithmec mean समार मा

2 Geometric mean गणोर मा

3 None of these इनम स कोई नही

4 Harmonic mean हराक मा

Correct Answer -

Geometric mean गणोर मा

CANCELLED

ldquoEach day is more or less the same the morning is clear and bright with a sea breeze as the Sun climbshigh in the sky heat mounts up dark clouds form then rain comes with thunder lighting But rain is soonoverrdquo Which of the following regions is described in the above passage

ldquoक िदन समान स अिधक या कम होता ह सम की हवा क साथ और उल सबह होती ह जस सयआकाश म ऊचा चढ़ता ह गम बढ़ जाती ह काल बादल बनत ह िफर िबजली क साथ बारश आती ह लिकनबारश जी ख हो जाती हlsquorsquo उपरो पा म िनिलखत म स िकन ो का वणन िकया गया ह

1 Equatorial भमरखीय

2 Equatorial भमरखीय

3 Savannah सवाना

4 Mediterranean आातरक (भमसागरीय)

5 Mediterranean आातरक (भमसागरीय)

6 Monsoon मानसन

7 Monsoon मानसन

Correct Answer -

Equatorial भमरखीय

Equatorial भमरखीय

CANCELLED

In which epoch of the geological history of the Earth dinosaurs reached their largest size

पी क भगभय इितहास क िकस यग म डायनासोर अपन सबस बड़ आकार तक पचि

10)

11)

1 Triassic ट ायिसक

2 Jurassic जरािसक

3 Cretaceous ीटशस

4 Permian पिमयन

Correct Answer -

Jurassic जरािसक

CANCELLED

A spring tide would occur in which of the following conditions

िनिलखत म स िकन थितयो म ार-भाटा आता ह

1 When the Sun Moon and Earth are in a straight line

जब सय चमा और पी एक सीधी रखा म होत ह

2 When the Moon and Earth are in right angle to each other

जब चमा और पी एक-दसर क दािहन कोण म होत ह

3 When the Earth and Moon are in right angle to the Sun

जब पी और चमा सय क दािहन कोण म होत ह

4 When the Sun and Moon are in right angle to each other

जब सय और चमा एक-दसर क दािहन कोण म होत ह

Correct Answer -

When the Sun Moon and Earth are in a straight line

जब सय चमा और पी एक सीधी रखा म होत ह

CANCELLED

An observe on the Earthrsquos surface always sees the same face of the moon because

एक पयवक को पी की सतह स हमशा चाद का एक ही फलक िदखाई दता ह ोिक

1 Its path of revolution around the earth is the same as that of the earth around the Sun

इसका पी क चारो ओर परमण का माग सय क चारो ओर पी क समान ही ह

2 Its period of revolution around the Earth is the same as its period of rotation around its own axis

इसकी पी क चारो ओर परमण की अविध उसकी अपनी धरी क चारो ओर घणन की अविध क समान ह

3 Its period of rotation is the same as that of the Earth इसकी घणन की अविध पी क समान ह

ी ि ी ी

12)

13)

14)

4 Its direct of rotation is the same as that of Earth घणन की िदशा पी क समान ही ह

Correct Answer -

Its period of revolution around the Earth is the same as its period of rotation around its own axis

इसकी पी क चारो ओर परमण की अविध उसकी अपनी धरी क चारो ओर घणन की अविध क समान ह

CANCELLED

The pebbles that are faceted by the sand-blasting and shaped polished by the wind abrasions are known as

पवन अपघषन ारा पॉिलश रत-िवोिटत और साच म ढला ककड़ ___________ क प म जाना जाता ह

1 Dreikanter िकोणक

2 Pediments िकोिनका

3 Inselberg इलबग

4 Dunes टीबा

Correct Answer -

Dreikanter िकोणक

CANCELLED

Astronomical unit is the average distance between खगोलीय इकाई ______ क बीच की औसत दरी ह

1 Earth and Mars पी और मगल

2 Earth and mercury पी और बध

3 Earth and moon पी और चमा

4 Earth and Sun पी और सय

Correct Answer -

Earth and Sun पी और सय

During cold weather season in the northern plains there will be an inflow of cyclonic disturbancesfrom the _________ directions

शीत मौसम क दौरान उरी मदानी इलाको म _________ िदशाओ स चवात सबधी गड़बड़ी का अतवाह होगा

1 East and Northwest पव और उरपिम

2 East and Northeast पव और पवर

3 West and East पिम और पव

ि औ ि

15)

16)

17)

4 West and Northwest पिम और उरपिम

Correct Answer -

West and Northwest पिम और उरपिम

During an earth quake the velocity of the body waves will________ along with the increase in densityof the material it is passing through

भकप क दौरान लहरो क ऊपरी भाग का वग घन म व क साथ-साथ ________ जो इसस गजरन वाली वको आग बढाएगी

1 not change नही बदलगा

2 increase initially and then decrease शआत म बढ़गा और िफर घटगा

3 increase बढ़गा

4 decrease घटगा

Correct Answer -

increase बढ़गा

The Clouded Leopard National park is situated in which of the following states

िनिलखत म िकस रा म धिमल तदआ रा ीय उान (ाउडड लपड नशनल पाक ) थत ह

1 Tripura िपरा

2 Uttar Pradesh उर दश

3 Assam असम

4 Mizoram िमजोरम

Correct Answer -

Tripura िपरा

Usually the land surfaces are heated more quickly than the water surfaces because _____________

आम तौर पर जल सतहो की तलना म भिम सतह अिधक तजी स गम होती ह ोिक _____________ ह

1 the specific heat of water is higher than land पानी की िविश ऊा भिम स अिधक

2 the specific heat of water is lesser than land पानी की िविश ऊा भिम स कम होती

3 the latent heat of water is higher than the land पानी की अतिनिहत ऊा भिम स अिधक

4 the land reflects more heat radiation than water भिम पानी की तलना म अिधक ऊा क िविकरण को पराविततकरती

18)

19)

20)

21)

Correct Answer -

the specific heat of water is higher than land पानी की िविश ऊा भिम स अिधक

The longest shore-line is along the state of सबस लबी समतटीय रखा िन रा क साथ ह

1 Maharashtra महारा

2 Orissa उड़ीसा

3 Kerala करल

4 Gujarat गजरात

Correct Answer -

Gujarat गजरात

The position when the Earth is farthest from the Sun is known as

जब पी सय स सबस दर होती ह तो उस थित को िन नाम स जाना जाता ह

1 Perihelion उपसौर

2 Vernal Equinox बसत िवषव

3 Aphelion अपसौर

4 Autumnal Equinox शराल िवषव

Correct Answer -

Aphelion अपसौर

The seasonal reversal of winds is the typical characteristic of

हवाओ का मौसमी परवतन ______ की सामा िवशषता ह

1 Mediterranean climates only कवल भमसागरीय जलवाय

2 All of the above climates उपय सभी मौसम

3 Monsoon climate only कवल मानसन जलवाय

4 Equatorial climate only कवल भमरखीय जलवाय

Correct Answer -

Monsoon climate only कवल मानसन जलवाय

In _________ rocks the minerals will occurs in beds or layers

ो ि ो ो

22)

23)

24)

______ चानो म खिनज तल या परतो म होत ह

1 metamorphic कायातरत

2 igneous and metamorphic आय और कायातरत

3 igneous आय

4 sedimentary अवसादी

Correct Answer -

sedimentary अवसादी

Black soil is ideal for the cultivation of cotton as कपास की खती क िलए काली िमी आदश ह ोिक

1 Its colour is black यह काली होती ह

2 It is found on plateau regions यह पठार ो म पायी जाती ह

3 It is made up of lava यह लावा स बनी होती ह

4 It can retain moisture यह नमी को बरकरार रख सकती ह

Correct Answer -

It can retain moisture यह नमी को बरकरार रख सकती ह

The National Survey and Mapping Organization of the country works under the Department of___________

दश का रा ीय सवण और मानिचण सगठन ___________ िवभाग क अतगत काय करता ह

1 Space अतर

2 Science and Technology िवान और तकनीक

3 Culture सित

4 Tourism पयटन

Correct Answer -

Science and Technology िवान और तकनीक

Palk strait separates India from पाक जलडमम भारत स _____ को अलग करता ह

1 Pakistan पािकान

2 Andaman Island अडमान ीप

3 China चीन

25)

26)

27)

4 Sri Lanka ीलका

Correct Answer -

Sri Lanka ीलका

Which among the following state is the major producer of Bauxite in India

िनिलखत म स कौन सा रा भारत म बॉाइट का मख उादक ह

1 Madhya Pradesh मदश

2 Rajasthan राजथान

3 Goa गोवा

4 Orissa उड़ीसा

Correct Answer -

Orissa उड़ीसा

Which of the following states DOES NOT share border with Chhattisgarh

िनिलखत म स कौन सा रा छीसगढ़ क साथ सीमा साझा नही करता ह

1 Telangana तलगाना

2 Uttar Pradesh उर दश

3 Bihar िबहार

4 Andhra Pradesh आ दश

Correct Answer -

Bihar िबहार

Which of the following statements is INCORRECT with respect to parallels of latitudes

अाश क समानातरो क सबध म िन निलखत म स कौन सा कथन गलत ह

1 A line joining places of equal latitude is known as parallel of largest

समान अाश क थानो को जोड़न वाली रखा को िवशालतम क समानातर क प म जाना जाता ह

2 They stat from equator and run parallels to it

व भम रखा स ारभ होत ह और इसक समानातर चलत ह

3 All parallels are equal in length सभी समातर लबाई म समान ह

4 All parallels are drawn as circles on the globe ोब पर सभी समानातर वो क प म खीच जात ह

28)

29)

30)

31)

Correct Answer -

All parallels are equal in length सभी समातर लबाई म समान ह

Which of the following Indian states is also known as a lsquoLand of Red river and Blue Hillsrsquo

िनिलखत म स िकस भारतीय रा को लाल नदी और नीली पहािड़यो की भिम क नाम स जाना जाता ह

1 Uttarkhand उराखड

2 Assam असम

3 Meghalaya मघालय

4 Arunachal Pradesh अणाचल दश

Correct Answer -

Assam असम

In spatial analysis of settlement Rn = 215 indicates which type of settlement arrangement

िनपटान क थािनक िवषण म Rn = 215 यह इिगत करता ह िक िकस कार की िनपटान वथा ह

1 Uniform यिनफॉम

2 Semi-Clustered समी- ल टर

3 Clustered ल टर

4 Random रडम

Correct Answer -

Uniform यिनफॉम

Who are known as the lsquoYellow Peoplersquo lsquoयलो पीपलrsquo क प म कौन जाना जाता ह

1 Mongoloids मोगोलोइडस

2 Nigroids नीोइडस

3 Australoids ऑ लॉइडस

4 Caucasoids कॉकसोइडस

Correct Answer -

Mongoloids मोगोलोइडस

ि ि ो ौ ि

32)

33)

34)

Who publishes the topographical map of India भारत क थलाकितक मानिच को कौन कािशत करता ह

1 Geographical Survey of India भारत का भौगोिलक सवण

2 Government of India भारत सरकार

3 Geological Survey of India भारत क भगभय सवण

4 Survey of India भारत का सवण

Correct Answer -

Survey of India भारत का सवण

Who among the following claimed geography to be the lsquoEcology of Manrsquo

िनिलखत म स िकसन भगोल को मन का पारथितकी कहा ह

1 Alfred Hener अड हटनर

2 Vidal-de la Blache वाइडल-िड लॉ ॉश

3 Oo Schluter ओटो टर

4 Harlan Barrow हरलन बारो

Correct Answer -

Harlan Barrow हरलन बारो

Who among the following is regarded as the founder of humanistic approach in geography

िनिलखत म स िकस भगोल म मानवतावादी िकोण का सथापक माना जाता ह

1 William Bunge िविलयम बग

2 Yi-Fu-Tuan यी-फ- यान

3 Brain JL Berry न जएल बरी

4 Richard Peet रचड पीट

Correct Answer -

Yi-Fu-Tuan यी-फ- यान

Who prepared Lorenz curve लोरज व िकसन तयार िकया

1 Geddes गडस

2 None of these इनम स कोई नही

3 Griffith Taylor ििफथ टलर

35)

36)

37)

4 Max U Lorenz म य लोरज

Correct Answer -

Max U Lorenz म य लोरज

Gulf Streams are the currents of which of the following oceans

खाड़ी की धाराए िनिलखत महासागरो म स िकसकी धाराए ह

1 North Atlantic Ocean उरी अटलािटक महासागर

2 North Pacific Ocean उरी शात महासागर

3 Arabian Sea अरब सागर

4 South Pacific Ocean दिण शात महासागर

Correct Answer -

North Atlantic Ocean उरी अटलािटक महासागर

Disintegration wearing away and removal of rock material is generally referred as

िशला पदाथ (रॉक सामी) का टटना िमटना और हटना आमतौर पर ________ क प म सदिभत िकया जाता ह

1 Shattering िवसकारक

2 Denudation अनाादन

3 Fault श

4 Decomposition िवयोजन

Correct Answer -

Denudation अनाादन

Variations in the length of day time and night from season to season are due to

मौसम स मौसम परवतन पर िदन क समय और रात क समय की अविध म िभताए िन कारण स होती ह

1 The Earthrsquos revolution round the Sun in an elliptical manner पी का दीघवाकार तरीक स सय क चारो घणन

2 The Earthrsquos rotation on its axis पी का इसकी धरी पर घणन

3 Revolution of the Earth on a tilted axis नत अ पर पी का घणन

4 Latitudinal position of the place थान की अाश थित

Correct Answer -

Revolution of the Earth on a tilted axis नत अ पर पी का घणन

38)

39)

40)

Point out the correct sequence of mountain ranges from north to south

उर स दिण तक पवत खलाओ क सही अनम को इिगत कर

1 Great Himalaya Middle Himalaya Outer Himalaya Trans Himalaya

महान िहमालय म िहमालय बा िहमालय परा िहमालय

2 Middle Himalaya Great Himalaya Trans Himalaya Outer Himalaya

म िहमालय महान िहमालय परा िहमालय बा िहमालय

3 Outer Himalaya Middle Himalaya Great Himalaya Trans Himalaya

बा िहमालय म िहमालय महान िहमालय परा िहमालय

4 Trans Himalaya Great Himalaya Middle Himalaya Outer Himalaya

परा िहमालय महान िहमालय म िहमालय बा िहमालय

Correct Answer -

Trans Himalaya Great Himalaya Middle Himalaya Outer Himalaya

परा िहमालय महान िहमालय म िहमालय बा िहमालय

Sunrsquos halo is produced by the refraction of light in सय का भामडल ______ म काश क अपवतन ाराउ होता ह

1 Ice crystals in Cirrro-Cumulus clouds पाभ-कपास मघो क बफ िल

2 Ice crystal in Cirrus clouds पाभमघो क बफ िल

3 Dust particles in Stratus clouds री मघो क धल कण

4 Water vapour in Stratus clouds री मघो क जल वा

Correct Answer -

Ice crystal in Cirrus clouds पाभमघो क बफ िल

Read the given statements and answer which of the following options isare correct

(1) The minerals present in the rocks exposed to atmosphere are not subjected to alteration

(2) Oxidation is one of the processes of chemical weathering

िदए गए कथनो को पढ़ और उर द िक िन म स कौन सास िवक सही ह

(1) वायमल स अनावत शल म उपथत खिनज परवतन क अधीन नही होता ह

(2) ऑीकरण रासायिनक अपय की ियाओ म स एक ह

ो ो

41)

42)

1 Both statements are wrong दोनो कथन गलत ह

2 Both statements are correct दोनो कथन सही ह

3 First statement is wrong and second statement is correct पहला कथन गलत ह और दसरा कथन सही ह

4 First statement is correct and second statement is wrong पहला कथन सही ह और दसरा कथन गलत ह

Correct Answer -

First statement is wrong and second statement is correct पहला कथन गलत ह और दसरा कथन सही ह

Read the given statements and answer which of the following options isare correct

1 Sunrsquos short waves enter the earth partially heating the atmosphere

2 Heated earth surface from the sun produces broader waves which interacts and heats the atmosphere

िदए गए कथन को पढ़ और उर द िक िन म स कौन सास िवक सही ह

1 सय की छोटी तरग पी म आिशक प स वश करती ह और वायमडल को ऊत करती ह

2 सय स ऊत पी की सतह िवारत तरग उ करती ह जो परर भाव डालती ह और वायमडल कोऊत करती ह

1 Both Statements 1 and 2 are correct दोनो कथन 1 और 2 सही ह

2 Both Statements 1 and 2 are wrong दोनो कथन 1 और 2 गलत ह

3 Statement 1 is wrong and only Statement 2 is correct कथन 1 गलत ह और कवल कथन 2 सही ह

4 Only statement 1 is correct कवल कथन 1 सही ह

Correct Answer -

Both Statements 1 and 2 are correct दोनो कथन 1 और 2 सही ह

Read the given statements and answer which of the following options isare correct

(1)The rocks that get changed due to heat and pressure are termed as metamorphic rocks

(2)Slate is one such type of metamorphic rock

िदए गए कथनो को पढ़ और उर द िक िन म स कौन सास िवक सही ह

(1) शल जो ऊा और दाब क कारण परवितत हो जात ह उ कायातरक शलो क प म जाना जाता ह

(2) ट एक तरह का कायातरक शल ह

1 Both statements are wrong दोनो कथन गलत ह

2 Both statements are correct दोनो कथन सही ह

3 First statement is wrong and second statement is correct पहला कथन गलत ह और दसरा कथन सही ह

ी औ

43)

44)

4 First statement is correct and second statement is wrong पहला कथन सही ह और दसरा कथन गलत ह

Correct Answer -

Both statements are correct दोनो कथन सही ह

Read the given statements and answer which of the following options isare correct

1 Higher temperature anomaly is observed in the northern hemisphere

2 Differential heating is absent in Northern Hemisphere

िदए गए कथनो को पढ़ और उर द िक िन म स कौन सास िवक सही ह

1 उरी गोलाध म उ तापमान िवसगित पायी जाती ह

2 उरी गोलाध म अतर ऊन अनपथत होती ह

1 Both Statements 1 and 2 are correct दोनो कथन 1 और 2 सही ह

2 Both Statements 1 and 2 are wrong दोनो कथन 1 और 2 गलत ह

3 Statement 1 is wrong and Statement 2 is correct कथन 1 गलत ह और कथन 2 सही ह

4 Statement 1 is correct and Statement 2 is wrong कथन 1 सही ह और कथन 2 गलत ह

Correct Answer -

Statement 1 is correct and Statement 2 is wrong कथन 1 सही ह और कथन 2 गलत ह

Read the given statements and answer which of the following options isare correct

(1) Plutonic rocks are intrusive type of igneous rocks

(2) It cools very slowly because the surrounding rock serves as insulation around the intrusion of magma

िदए गए कथनो को पढ़ और उर द िक िन म स कौन सास िवक सही ह

(1) िवतलीय शल अतवधी कार क आश शल ह

(2) यह बत धीर-धीर ठडा होता ह ोिक आस-पास क शल मा क अतवधन क चारो ओर रोधन क प म कायकरत ह

1 Both statements are wrong दोनो कथन गलत ह

2 Both statements are correct दोनो कथन सही ह

3 First statement is wrong and second statement is correct पहला कथन गलत ह और दसरा कथन सही ह

4 First statement is correct and second statement is wrong पहला कथन सही ह और दसरा कथन गलत ह

Correct Answer -

Both statements are correct दोनो कथन सही ह

45)

46)

47)

48)

The dust and ash material hurled from the volcanoes are termed as

ालामखी स िनकलन वाली धल और राख सामी को _______ क प म कहा जाता ह

1 Pyroclasc पाइरोाक

2 Hyperclastic हाइपराक

3 Hepiroclastic हिपरोाक

4 Cirroclastic िसरोाक

Correct Answer -

Pyroclasc पाइरोाक

The vertical difference in elevation between a low tide and high tide is referred as

कम ार और उ ार क बीच ऊचाई म लबवत अतर _____ स सदिभत होता ह

1 Tidal slope ारीय ढलान

2 Tidal elevation ारीय उयन

3 Tidal range ारीय परास

4 Tidal height ारीय ऊचाई

Correct Answer -

Tidal range ारीय परास

The maximum biodiversity is found in which of the following regions िनिलखत ो म स अिधकतमजव िविवधता िकसम पायी जाती ह

1 Amazon Basin अमज़न बिसन

2 East Indies ई इडीज

3 Congo Basin कागो बिसन

4 West indies व इडीज

Correct Answer -

Amazon Basin अमज़न बिसन

The cultivation of rice crop produces_______ चावल की फसल की खती ______ का उादन करती ह

1 SO2

49)

50)

51)

2 CH4

3 CFCs

4 CO2

Correct Answer -

CH4

The pressure system with higher pressure at the centre is called__________

क म उ दबाव वाली दबाव णाली को _______ कहा जाता ह

1 front अ

2 depression अवनमन

3 cyclone चवात

4 anti-cyclone ितचवात

Correct Answer -

anti-cyclone ितचवात

The Himalayan region is poor in mineral resources because िहमालयी खिनज ससाधनो म समनही ह ोिक

1 The displacement of rock strata has disturbed the arrangement of rocks and made it complex

शलीय परत क िवथापन न चानो की वथा को अवथत कर िदया ह और इस जिटल बना िदया ह

2 The climate conditions are not suitable for exploitation of minerals

जलवाय की थित खिनजो क दोहन क िलए उपय नही ह

3 The terrain makes explanation of minerals difficult and very costly due to transportation difficulties

भ-भाग परवहन की किठनाइयो क कारण खिनजो का दोहन मल और बत महगा बना दता ह

4 It is made up of crystalline rocks यह िलीय चानो स बना ह

Correct Answer -

The displacement of rock strata has disturbed the arrangement of rocks and made it complex

शलीय परत क िवथापन न चानो की वथा को अवथत कर िदया ह और इस जिटल बना िदया ह

The process through which the moisture is added to the atmosphere by vegetation is termed as

वह िया िजसक माम स वनित ारा वातावरण म नमी िमलायी जाती ह _______ क प म जानी जाती ह

52)

53)

54)

1 Condensation सघनन

2 Evapotranspiration वान-उजन

3 Radiation िविकरण

4 Precipitation वषण

Correct Answer -

Evapotranspiration वान-उजन

The process through which the terrestrial heat is transferred to air by direct contact is termed as

वह िया िजसम सपक ारा थलीय ऊा वाय म थानातरत हो जाती ह ______ क प म जानी जाती ह

1 Conduction चालन

2 Convection सवहन

3 Insolation आतपन

4 Radiation िविकरण

Correct Answer -

Conduction चालन

The largest area under mangroves is in which of the following statesunion territory

मोव क अतगत िनिलखत राोसघ शािसत दशो म स सबस बड़ा कौन सा ह

1 Andaman and Nicobar अमान और िनकोबार

2 Andhra Pradesh आ दश

3 West Bengal पिम बगाल

4 Gujarat गजरात

Correct Answer -

West Bengal पिम बगाल

The longitudinal transverse and surface waves in an earthquake originate from

भकप म दशातर अनथ और सतह तरग यहा उ होती ह

1 The focus on the surface of the Earth पी क सतह पर क -िबद म

2 The focus within the body of the Earth पी क भीतर क -िबद म

3 The epicenter within the body of the Earth पी क भीतर उपरक म

55)

56)

57)

4 The epicenter on the surface of the Earth पी क सतह पर उपरक म

Correct Answer -

The focus within the body of the Earth पी क भीतर क -िबद म

The down slope movement of material due to gravity is called______

गाकषण क कारण पदाथ की अनढाल गित को ______ कहा जाता ह

1 mass movement पदाथ सचलन

2 deposition िनप

3 erosion रण

4 volcanic movement ालामखीय सचलन

Correct Answer -

mass movement पदाथ सचलन

Shimla is cooler than Amritsar although both are on the same latitude This is because

िशमला म अमतसर स अिधक ठड ह हालािक दोनो समान अाश पर ह ऐसा ह ोिक

1 Shimla is at a greater height above sea level than Amritsar अमतसर की तलना म िशमला सम तल स अिधकऊचाई पर ह

2 Shimla is further north िशमला उर की ओर ह

3 Shimla is farther from the equator िशमला भम रखा स आग ह

4 Their longitudes differ उनकी दशातर रखाए िभ ह

Correct Answer -

Shimla is at a greater height above sea level than Amritsar अमतसर की तलना म िशमला सम तल स अिधकऊचाई पर ह

lsquoTempo of Urbanizationrsquo measures which of the following

lsquoशहरीकरण का टपोrsquo िनिलखत म स कौन सा उपाय ह

1 Speed of urbanizaon शहरीकरण की गित

2 None of the above इनम स कोई नही

3 Inequality of urbanizaon शहरीकरण की असमानता

4 Current level of urbanizaon शहरीकरण का वतमान र

Correct Answer -

58)

59)

60)

Speed of urbanizaon शहरीकरण की गित

Out of the following options choose the INCORRECT statement

िनिलखत िवको म स गलत कथन का चयन कर

1 The clear tracts in the equatorial region recover rapidly भम रखा म भभाग तजी स ठीक हो जात ह

2 The stable communities include a redwood forest a pine forest at high elevations

थर समदायो म एक रडवड वन उ ऊचाई पर एक दवदार वन शािमल ह

3 Any ecosystem moves towards maximum biomass and stability to survive

कोई भी पारथितकी त जीिवत रहन क िलए अिधकतम जवसहित और थरता की तरफ असर होता ह

4 Tropical rain forests near equator are stable ecosystems

भम रखा क पास उकिटबधीय वषा वन थर पारथितक त ह

Correct Answer -

The clear tracts in the equatorial region recover rapidly भम रखा म भभाग तजी स ठीक हो जात ह

Seasonal contrasts are maximum in मौसमी िवषमता अिधकतम ह

1 Mid latitudes म अाश म

2 Low attitudes िन अाश म

3 High latitudes उ अाश म

4 Subtropics उपोकिटबधीय म

Correct Answer -

Mid latitudes म अाश म

In India which type of forest among the following occupies the largest area

भारत म िनिलखत म स िकस कार क वन सबस बड़ा फल आािदत करत ह

1 Sub-tropical Dry Evergreen Forest उप उकिटबधीय श सदाबहार वन

2 Mountain Wet Temperate Forest पवतीय आ शीतो वन

3 Tropical Moist Deciduous Forest उकिटबधीय आ पणपाती वन

4 Tropical Wet Evergreen Forest उकिटबधीय आ सदाबहार वन

Correct Answer -

Tropical Moist Deciduous Forest उकिटबधीय आ पणपाती वन

61)

62)

63)

64)

What is the proportion of lsquoJuvenile Populationrsquo (0-14 years) in India as per 2011Census

2011 की जनगणना क अनसार भारत म जवनाइल पॉपलशन यानी िकशोर जनस या (0-14 वष) का अनपात ाह

1 3076 of total population कल जनस या का 3076

2 2764 of total population कल जनस या का 2764

3 2933 of total population कल जनस या का 2933

4 3354 of total population कल जनस या का 3354

Correct Answer -

3076 of total population कल जनस या का 3076

What is the Belfast famous for बलफा िकसक िलए मशर ह

1 Belt of cotton textile industry कपास व उोग क

2 Ship-building industry जहाज िनमाण उोग

3 Agricultural machinery किष उपकरण

4 Aero planes manufacturing वाययान िनमाण

Correct Answer -

Ship-building industry जहाज िनमाण उोग

What is the most important occupation in tropical monsoon lands

उकिटबधीय मॉनसन भिम म सबस महपण वसाय ा ह

1 Mining खनन

2 Cattle rearing मवशी पालन

3 Agriculture किष

4 Nomadic herding नोमािडक जड़ी-बिटया

Correct Answer -

Agriculture किष

What is the most important characteristics of the islands (Indian) located in the Arabian sea

अरब सागर म थत ीपो (भारतीय) की सबस महपण िवशषता ा ह

ी ो

65)

66)

67)

1 There are all of coral origins सभी कोरल मल क ह

2 There are all very small in size य सभी आकार म बत छोट ह

3 They have a very dry climate इनकी जलवाय बत श ह

4 They are extended parts of the mainland व महाीप क िवारत िह ह

Correct Answer -

There are all of coral origins सभी कोरल मल क ह

What do the basalt layers of the Deccan indicate डन की बसा परत ा इिगत करती ह

1 All of the above उपरो सभी

2 Huge volcanic eruptions in the distant past दरथ अतीत म िवशाल ालामखीय िवोट

3 The immense erosional activity of the rivers निदयो की िवशाल रण गितिविध

4 The influence of weathering मौसम का भाव

Correct Answer -

Huge volcanic eruptions in the distant past दरथ अतीत म िवशाल ालामखीय िवोट

In the structure of planet Earth below the mantle the core is mainly made up of_____

पी ह की सरचना म मटल क नीच कोर म प स______ स िनिमत होती ह

1 aluminium एमीिनयम

2 silicon िसिलकॉन

3 chromium ोिमयम

4 iron लोहा

Correct Answer -

iron लोहा

One of the major Mid Oceanic Ridge is found in मख म-महासागर चोिटयो म स एक ______ म पायाजाता ह

1 Mid Pacific Ocean म शात महासागर

2 Mid Atlantic Ocean म अटलािटक महासागर

3 Mid Indian Ocean म भारतीय महासागर

4 Mid Arctic Ocean म आक िटक महासागर

68)

69)

70)

71)

Correct Answer -

Mid Atlantic Ocean म अटलािटक महासागर

Magma that reaches the Earthrsquos surface and then solidifies is called________

मा जो पी की सतह तक पचती ह और िफर ठोस हो जाती ह ________कहलाती ह

1 quartz ाटज

2 lava लावा

3 granite नाइट

4 silicates िसिलकट

Correct Answer -

lava लावा

Isotherms are the lines of equal_______ समताप रखाए समान _______की रखाए होती ह

1 pressure दाब

2 temperature तापमान

3 rainfall वषा

4 height ऊचाई

Correct Answer -

temperature तापमान

Mark the correct sequence of passes in the Western Ghats from north to south

पिमी घाटो म उर स दिण तक दर क सही अनम को िचित कर

1 Thalghat Palghat Bhorghat थलगघाट पालघाट भोरघाट

2 Thalghat Bhorghat Palghat थलघाट भोरघाट पालघाट

3 Bhorghat Thalghat Palghat भोरघाट थलघाट पालघाट

4 Palghat Bhorghat Thalghat पालघाट भोरघाट थलघाट

Correct Answer -

Thalghat Bhorghat Palghat थलघाट भोरघाट पालघाट

Which of the following does not have influence over the climate in India

ि ि ि ी ी

72)

73)

िनिलखत म स िकसका भाव भारत की जलवाय पर नही पड़ता ह

1 Ocean currents सागर की लहर

2 Nearness to equator भम रखा स िनकटता

3 Monsoons मानसन

4 Presence of Indian ocean भारतीय महासागर की उपथित

Correct Answer -

Ocean currents सागर की लहर

Which of the following cloud types has the characteristics like vertical tall narrow and puffy

िनिलखत म स िकस कार क मघो म लबवत लबी सकीण और थलता जसी िवशषताए ह

1 Cumulonimbus तफानी मघ

2 Cumulus मघ पज

3 Cirrocumulus पाभ कपासी मघ

4 Nimbostratus वषारी मघ

Correct Answer -

Cumulus मघ पज

Which of the following statement is INCORRECT about Crude Birth Rate

िनिलखत स कौन सा कथन अशोिधत ज दर क बार म सही नही ह

1 It cannot be used for comparing fertility level between two countries with different population characteristics

इसका उपयोग िविभ जनसा िवशषताओ वाल दो दशो क बीच जनन र की तलना क िलए नही िकया जा सकता ह

2 It is a standardized measure of fertility

यह जनन मता का मानकीकत उपाय ह

3 It is effected by the age-sex composition of the population

यह आबादी की आय-िलग सरचना स भािवत होता ह

4 It is expressed per 1000 population in a given geographical unit

यह िकसी दी गई भौगोिलक इकाई म ित 1000 जनसा पर िकया जाता ह

Correct Answer -

It is a standardized measure of fertility

यह जनन मता का मानकीकत उपाय ह

74)

75)

76)

77)

Which of the following state in India experienced negative decadal growth rate during 2001 to 2011census

भारत म िनिलखत म स िकस रा म वष 2001 स 2011 की जनगणना क दौरान नकाराक िगरावट दर ई

1 Tripura िपरा

2 Nagaland नागालड

3 Haryana हरयाणा

4 Odisha ओिडसा

Correct Answer -

Nagaland नागालड

Which of the following is NOT a characteristic of peninsular rivers

िनिलखत म स कौन सी िवशषता ायीपीय निदयो म नही होती ह

1 Flow through shallow valleys उथल घािटयो क माम स वाह

2 Seasonal flow मौसमी वाह

3 Little erosional activity थोड़ी कटावदार गितिविध

4 Meandering tendency often shifting their beds घमावदार वि अर अपन तटो को थानातरत करना

Correct Answer -

Meandering tendency often shifting their beds घमावदार वि अर अपन तटो को थानातरत करना

Which of the following gases in the atmosphere absorbs heat from the Sunrsquos radiation and the Earthssurface

वायमडल म िनिलखत म स कौन सी गस सय क िविकरण और पी की सतह स ऊा को अवशोिषत करती ह

1 Neon िनयॉन

2 Carbon dioxide काबन डाइऑाइड

3 Argon आगन

4 Nitrogen नाइट ोजन

Correct Answer -

Carbon dioxide काबन डाइऑाइड

Which of the following kind of settlement pattern is found at the confluence of rivers

ि ि ि ि ो

78)

79)

80)

िनिलखत म स िकस कार का वथापन पटन निदयो क सगम पर पाया जाता ह

1 Triangular Paern िकोणीय पटन

2 Circular or Semi-Circular Paern परप या अध-परप पटन

3 Nebular Paern नबलर पटन

4 Star ndashShaped Paern ार-आकार का पटन

Correct Answer -

Triangular Paern िकोणीय पटन

Which one was not the objective of the Biosphere Reserve Projects launched by the UNESCO

यनो ारा श की गई सरित जवमडल परयोजनाओ का उ इनम स कौन सा नही था

1 To promote teaching and research िशण और अनसधान को बढ़ावा दना

2 To make agriculture sustainable किष को दीघकािलक बनाना

3 To conserve ecosystems पारथितक त को सरित करना

4 To conserve genetic diversity for a longtime लब समय तक अनवािशक िविवधता को सरित करना

Correct Answer -

To make agriculture sustainable किष को दीघकािलक बनाना

Which region of the Earth surface receives the highest amount of insulation

पी सतह का कौन सा तापावरोधन की उतम माा ा करता ह

1 Land mass थलखड

2 Savannah region सवाना

3 Water bodies जल िनकाय

4 Tropical desert उकिटबधीय रिगान

Correct Answer -

Tropical desert उकिटबधीय रिगान

Which one of the following is not a biodiversity hotspot

िनिलखत म स कौन सा जव िविवधता का म जगह नही ह

1 Eastern Himalaya पव िहमालय

2 Eastern Ghats पव घाट

81)

82)

83)

3 Indo-Myanmar भारत-ामार

4 Westerm Ghats पिमी घाट

Correct Answer -

Eastern Ghats पव घाट

Which one of the following is NOT a part of the World Network of Biosphere Reserves based on theUNESCO Man and Biosphere Programme

यनो मन और बायोीयर कायम क आधार पर िनिलखत म स कौन बायोीयर रजव क िव नटवक कािहा नही ह

1 Gulf of Mannar मार की खाड़ी

2 Seshachalam शषाचलम

3 Sunderban सदरबन

4 Nilgiri नीलिगर

Correct Answer -

Seshachalam शषाचलम

Which one of the following is an example of ldquodesert vegetationrdquo

िनिलखत म स कौन मथलीय वनित का एक उदाहरण ह

1 Mosses and lichens दलदल और शवाल

2 Temperate grassland समशीतो घास क मदान

3 Coniferous forest शकधारी वन

4 Acacia and cactus एकािसया और कस

Correct Answer -

Acacia and cactus एकािसया और कस

Which one of the following reflects more sunlight िनिलखत म स कौन सा सय की रोशनी को अिधकपरावितत करता ह

1 Paddy crop land धान फसल भिम

2 Land covered with fresh snow ताजा बफ स आािदत भिम

3 Sand desert रतीली रिगान

4 Prairie land यरी भिम

84)

85)

86)

87)

Correct Answer -

Land covered with fresh snow ताजा बफ स आािदत भिम

Which layer of the atmosphere is in contact with the surface of the earthrsquos oceans

वायमडल की कौन सी परत पी क महासागरो की सतह क सपक म ह

1 Stratosphere समताप मडल

2 Mesosphere म मडल

3 Hydrosphere जलमडल

4 Troposphere ोभ मडल

Correct Answer -

Troposphere ोभ मडल

Mediterranean Sea is a border of which of the following countries भम सागर िनिलखत दशो म सिकसकी सीमा ह

1 None of these इनम स कोई नही

2 Iraq इराक

3 Lebanon लबनान

4 Jordan जॉडन

Correct Answer -

Lebanon लबनान

Benguela ocean currents are found along which coast बगएला महासागर धाराए िकस तट क साथ पायीजाती ह

1 East Coast of South America दिण अमरका क पव तट

2 East Coast of Africa अीका क पव तट

3 West Coast of South America दिण अमरका क पिमी तट

4 West Coast of Africa अीका क पिमी तट

Correct Answer -

West Coast of Africa अीका क पिमी तट

88)

89)

90)

Due to tension a block of land on one side being pushed up or upthrown relative to the downthrown blockis referred as

तनाव क कारण नीच फ क ए खड क साप भिम का एक खड एक ओर स ऊपर धकला जाता ह या ऊपर की ओरफ का जाता ह यह _____ क प म सदिभत ह

1 Thrust fault प श

2 Normal fault सामा श

3 Reverse fault म श

4 Strike slip fault नितलब सपण श

Correct Answer -

Normal fault सामा श

Inter-tropical doldrums is a zone of ______ अतर-उकिटबधीय डोलड ______ का एक ह

1 Frontolysis टोलायिसस

2 Convergence अिभसरण

3 Inter-tropical divergence zone अतर-उकिटबधीय िवचलन

4 Local wind थानीय वाय

Correct Answer -

Convergence अिभसरण

The Horse Latitudes are regions located at about _____ north and south of the equator

हॉस अाश भम रखा क उर और दिण म लगभग _____ पर थत ह

1 30ndash60 degree Latitude 30-60 िडी अाश

2 0ndash5 degree Latitude 0-5 िडी अाश

3 30 degree Latitude 30 िडी अाश

4 60ndash90 degree Latitude 60-90 िडी अाश

Correct Answer -

30 degree Latitude 30 िडी अाश

Generally evaporation is high over which part of the Earth

आम तौर पर पी क िकस भाग पर वाीकरण अिधक होता ह

1 Equatorial maritime भमवत समीय ी ी

91)

92)

2 Equatorial continental भमवत महाीपीय

3 Polar maritime वीय समीय

4 Polar continental वीय महाीपीय

Correct Answer -

Equatorial maritime भमवत समीय

A very high temperature during summer in north western India leads to what type of climaticcondition in south

उर पिमी भारत म गम क दौरान बत अिधक तापमान होन क कारण दिण म िकस कार की जलवाय थितउ करता ह

1 Depression over arabian sea अरब सागर पर अवनमन

2 Failure monsoon मानसन िवफलता

3 Successful monsoon मानसन सफलता

4 Cyclones चवात

Correct Answer -

Successful monsoon मानसन सफलता

Lightning and thunder are the resultant effect when तिड़त और गजन परणामी भाव ह जब

1 Two massive clouds hit powerfully each other first lightning is produced and later sound is produced

दो बड़ बादल एक दसर स शशाली ढग स टकरात ह पहल आकाशीय िवदयत उ होता ह और बाद म िन उहोती ह

2 Two massive clouds come into contact with the powerful wind collision this results into first sound and thenlightning

दो बड़ बादल शशाली पवन सघ क सपक म आत ह इसका परणामप पहल िन और िफर आकाशीय िवदयतउ होता ह

3 None of the above उपरो म स कोई भी नही

4 A high density cloud contains positively and negatively charged electric ions and when this interacts light andsound are simultaneously produced

एक उ घन बादल म धनाक और ऋणाक आविशत िवदयत आयन होत ह और जब यह परर भाव डालत ह तोकाश और िन एक साथ उािदत होती ह

Correct Answer -

A high density cloud contains positively and negatively charged electric ions and when this interacts light andsound are simultaneously produced

औ ि ि ो औ ो

93)

94)

95)

एक उ घन बादल म धनाक और ऋणाक आविशत िवदयत आयन होत ह और जब यह परर भाव डालत ह तोकाश और िन एक साथ उािदत होती ह

Doon Valley is able to grow rice because दन घाटी चावल उगान म सम ह ोिक

1 Other crops cannot be grown वहा अ फसलो को उगाया नही जा सकता ह

2 People in the valley are rice eaters घाटी म लोग चावल खान वाल ह

3 There is a huge export demand of rice वहा चावल की भारी िनयात माग ह

4 It has warm summer and snow melt waters for irrigation

वहा गिमया गम होती ह िसचाई क िलए बफ का िपघला आ पानी होता ह

Correct Answer -

It has warm summer and snow melt waters for irrigation

वहा गिमया गम होती ह िसचाई क िलए बफ का िपघला आ पानी होता ह

CANCELLED

In the geological time scale the Mesozoic Era DOES NOT contains which of the following periods

भगभय समय पमान पर मजीवी यग म िन कालो म स कौन नही ह

1 Triassic ट ाइऐिसक

2 Jurassic जरिसक

3 Cretaceous चाकमय

4 Carboniferous काबनी

Correct Answer -

Carboniferous काबनी

96)

1 P-3 Q-4 R-2 S-1

2 P-3 Q-4 R-1 S-2

3 P-3 Q-4 R-1 S-2

4 P-4 Q-3 R-2 S-1

Correct Answer -

P-4 Q-3 R-2 S-1

1 P-3 Q-1 R-4 S-2

2 P-3 Q-4 R-1 S-2

3 P-3 Q-2 R-4 S-1

97)

98)

4 P-2 Q-1 R-4 S-3

Correct Answer -

P-3 Q-4 R-1 S-2

1 P-3 Q-1 R-4 S-2

2 P-2 Q-3 R-4 S-1

3 P-2 Q-1 R-3 S-4

4 P-4 Q-2 R-1 S-3

Correct Answer -

P-3 Q-1 R-4 S-2

99)

100)

1 P-3 Q-2 R-4 S-1

2 P-1 Q-2 R-3 S-4

3 P-2 Q-3 R-1 S-4

4 P-4 Q-3 R-2 S-1

Correct Answer -

P-2 Q-3 R-1 S-4

ldquoHuman geography is the study of changing relationship between the unresting man and the unstableearthrdquo was defined by

lsquolsquoमानव भगोल ाकल आदमी और अथर पी क बीच सबध परवतन का अयन हrdquo ______ ारा परभािषत िकया गयाथा

1 J Brunches ज चस

2 EC Semple ईसी सल

3 HJ Mackinder एच ज मिकदर

4 PV Blache पीवी च

Correct Answer -

EC Semple ईसी सल

Sedimentary rocks are finally and ultimately derived from the____________

अवसादी चान अततः ________ स ा की जाती ह

1 action of earth movements पी की गितिविधयो

2 marine deposit समी िनप

3 weathering of metamorphic rocks पातरत चानो क अपय

4 weathering of igneous rocks आय चानो क अपय

Correct Answer -

weathering of igneous rocks आय चानो क अपय

Page 11: High School Teacher Eligibility Test- BOARD PROFESSIONAL ...peb.mp.gov.in/results/RESULT_18/HST_RES18/Final_anwser_key/HST… · M a ndl a / मंड ल ... Under the Madhya Pradesh

5)

6)

Correct Answer -

20 days 20 िदन

Which one of the following four addresses is NOT EXACTLY same as the one given below

िनिलखत िदए गए चार पतो म स नीच िदए गए पत क ठीक समान कौन सा एक नही ह

Ground Floor 59 Goulburn St

Sydney NSW 2000 Australia

+61 (02) 8987 3700

(i) Ground Floor 59 Goulburn St

Sydney NSW 2000 Australia

+61 (02) 8987 3700

(ii) Ground Floor 59 Goulburn St

Sydney NSW 2000 Australia

+61 (02) 8987 3700

(iii) Ground Floor 59 Govlburn St

Sqdney NSW 2000 Australia

+61 (02) 8987 3700

(iv) Ground Floor 59 Goulburn St

Sydney NSW 2000 Australia

+61 (02) 8987 3700

1 ii

2 iv

3 i

4 iii

Correct Answer -

iii

In a group of 75 people 32 of them like cold drink 56 of them like general water and each person likesat least one of the two drinks How many people like both

75 लोगो क समह म उनम स 32 को को िड क पसद ह उनम स 56 को सामा जल पसद ह और क को दो म स कम स कम एक पय पसद ह िकतन लोगो को दोनो पसद ह

1 11

2 13

7)

1)

2)

3 12

4 14

Correct Answer -

13

If A = 1 FAT = 27 then FAITH =

यिद A = 1 FAT = 27 तो FAITH =

1 41

2 40

3 42

4 44

Correct Answer -

44

Topic- PEDAGOGY

CANCELLED

Basic education became a major initiative because of the efforts of

ाथिमक िशा िन न क यास स एक मह वपण कायम बन गई

1 Dr Radhakrishnan डॉ राधाक णन

2 Dr Rajendra Prasad डॉ राज साद

3 Mahatma Gandhi महा मा गाधी

4 Rabindranath Tagore रबी नाथ टगोर

Correct Answer -

Mahatma Gandhi महा मा गाधी

The ldquoclause of phraserdquo is an unit of perception of लॉज ऑफ ज िन न क धारणा क एक इकाई ह

1 Image छिव

2 Concept अवधारणा

3 Language भाषा

4 Thought िवचार

Correct Answer -

3)

4)

5)

6)

Language भाषा

Which method is most suitable to study communication process among students

छाो क बीच सचार िया का अयन करन क िलए कौन सी िविध सबस उपय ह

1 Case Study मामल का अयन (कस टडी)

2 Systematic Observation वथत अवलोकन

3 Experimental Method योगाक िविध

4 Introspection आ-िनरीण (इट ोस शन)

Correct Answer -

Systematic Observation वथत अवलोकन

Genes in a human being are located in मानवो म जीन इनम थत होता ह

1 cytoplasm कोिशका (साइटो ला म)

2 ribosomes राइबोसोम

3 cell membranes कोिशका िझी

4 chromosomes गणस (ोमोसोम)

Correct Answer -

chromosomes गणस (ोमोसोम)

With smaller classes teachers are much more able to ____________

छोटी काओ क साथ िशक ____________ म अिधक सम होत ह

1 Go slow while teaching िशण क दौरान धीमी गित स जान

2 Narrate more personal experiences अिधक गत अनभवो को बतान

3 Adapt instruction to individual differences among students छाो क बीच गत मतभदो क िलए अनकल िनदश दन म

4 Make use of the extra space for extra-curricular activities पातर गितिविधयो क िलए अितर जगह का उपयोग करन

Correct Answer -

Adapt instruction to individual differences among students छाो क बीच गत मतभदो क िलए अनकल िनदश दन म

While engaging in a task the child gets bored This is a sign of

एक काम म होन पर बा ऊब जाता ह यह िन का सकत ह

1 the task requiring a professional approach काय को ावसाियक िकोण की आवकता ह

7)

8)

9)

2 the task becoming mechanically repetitive काय यािक प स दोहराव वाला ह

3 the child not being intelligent बा बमान नही ह

4 the child being incapable of learning बा सीखन म असमथ ह

Correct Answer -

the task becoming mechanically repetitive काय यािक प स दोहराव वाला ह

Educational Psychologists are more concerned with the learning in __________

शिणक मनोवािनक __________ म अिधगम क साथ अिधक िचितत होत ह

1 Formal environment औपचारक वातावरण

2 Informal environment अनौपचारक वातावरण

3 Physical environment भौितक वातावरण

4 Social environment सामािजक वातावरण

Correct Answer -

Formal environment औपचारक वातावरण

Special needs education is the type of education िवशष ज़रतो वाली िशा वह िशा होती ह जो

1 Given to person with disabilities अम य को दी जाती ह

2 Given to people from remote areas दर थ ो क लोगो को दी जाती ह

3 Provided to intelligent people बमान लोगो को दी जाती ह

4 Established by colonial masters औपिनविशक मखयाओ ारा थािपत की गई ह

Correct Answer -

Given to person with disabilities अम य को दी जाती ह

The Stanford-Binet scale of intelligence was first published in the year

ब क नफोड-िबनट कल को इस वष म पहली बार कािशत िकया गया था

1 1916

2 1903

3 1908

4 1900

Correct Answer -

1916

10)

11)

12)

13)

The term lsquofictional finalismrsquo was propounded by

पद lsquoकत योजनवादrsquo (िफ शनल फाइनिल म) इनक ारा ितपािदत िकया गया था

1 Skinner नर

2 Freud ायड

3 Adler एडलर

4 Pavlov पावलोव

Correct Answer -

Adler एडलर

Rational Emotive Behavior Therapy was propounded by

तक सगत भावनाक वहार थरपी िन क ारा ितपािदत की गई थी

1 Carl Jung काल यग

2 Carl Rogers काल रोजस

3 Aaron Beck आरोन बक

4 Albert Ellis अट एिलस

Correct Answer -

Albert Ellis अट एिलस

Which of the following indicates the quality of education in a school

िनिलखत म स या एक कल म िशा की गणव ता का सकतक ह

1 Text-books and Teaching-learning material पा-प तक तथा िशण व सीखन स सबिधत सामी

2 Infrastructural facilities at the school कल म आधारभत ढाच स सबिधत सिवधाए

3 Student achievement level िवािथयो का उपल तर

4 Classroom systems का की यव था

Correct Answer -

Student achievement level िवािथयो का उपल तर

Which of the following teachers can be identified with authoritarian teaching style

िनिलखत म स कौन सा िशक अिधकारवादी िशण शली क साथ पहचाना जा सकता ह

1 Laissez-faire teacher अब ध िशक

14)

15)

2 Democratic teacher लोकतीय िशक

3 Indifferent teacher िन प िशक

4 Direct instruction teacher िनदश िशक

Correct Answer -

Direct instruction teacher िनदश िशक

Who was the pioneer of classical conditioning

िचरितित ानकलन ( लािसकल कडीशिनग) क वतक कौन थ

1 Skinner नर

2 Pavlov पावलोव

3 Kohler कोहलर

4 Freud ायड

Correct Answer -

Pavlov पावलोव

Which of the following are true with reference to short term memory

1 Limited capacity

2 Brief storage of information

3 Unlimited capacity

4 Duration of storage less than twenty seconds

अ पकािलक मित क सदभ म िन न म स या स य ह

1 सीिमत मता

2 सचना का सि त भडारण

3 असीिमत मता

4 बीस सकड स कम भडारण की अविध

1 3 and 4 3 और 4

2 2 and 4 2 और 4

3 13 and 4 13 और 4

4 12 and 4 12 और 4

Correct Answer -

12 and 4 12 और 4

1)

2)

3)

4)

Topic- GEOGRAPHY

Which of the following ranges of population size is used to define Class-III city by Indian Census

भारतीय जनगणना ारा वग-III शहर को परभािषत करन क िलए िनिलखत म स िकस रज क जनसा आकारका उपयोग िकया जाता ह

1 20 000 to 49999 20 000 स 49999

2 30000 to 59999 30000 स 59999

3 24000 to 54999 24000 स 54999

4 50000 to 99999 50000 स 99999

Correct Answer -

20 000 to 49999 20 000 स 49999

Carbonaceous rocks which produce coal and oil belong to the category of rocks called_______

कोयल और तल का उादन करन वाली काबनय चान ______ नामक चानो की णी स सबिधत ह

1 metamorphic पातरत

2 sedimentary अवसादी

3 inorganic अजिवक

4 igneous आय

Correct Answer -

sedimentary अवसादी

The ruhr-complex is a major industrial centre in र-परसर िन का एक मख औोिगक क ह

1 North America उरी अमरका

2 Russia स

3 Germany जमनी

4 Europe यरोप

Correct Answer -

Germany जमनी

The term lsquoRegurrsquo refers to श lsquoरगरrsquo ______ स सबिधत ह

1 Deltaic alluvial soils डा जलोढ़ िमी

ि ी

5)

6)

2 Laterite soils लटराइट िमी

3 Red and yellow soils लाल और पीली िमी

4 Black cotton soils काली कपास िमी

Correct Answer -

Black cotton soils काली कपास िमी

Read the given statements and answer which of the following options isare correct

1 Lower the pressure greater the atmospheric disturbance

2 Air move from higher to low pressure

िदए गए कथन को पढ़ और उर द िक िन म स कौन सास िवक सही ह

1 िजतना दाब कम होगा वायमडलीय बाधाए उतनी अिधक होगी

2 वाय उ स िन दाब की ओर गित करती ह

1 Both Statements 1 and 2 are correct दोनो कथन 1 और 2 सही ह

2 Both Statements 1 and 2 are wrong दोनो कथन 1 और 2 गलत ह

3 Statement 1 is wrong and only Statement 2 is correct कथन 1 गलत ह और कवल कथन 2 सही ह

4 Statement 1 is correct and Statement 2 is wrong कथन 1 सही ह और कथन 2 गलत ह

Correct Answer -

Both Statements 1 and 2 are correct दोनो कथन 1 और 2 सही ह

CANCELLED

1 4 1 2 and 3 4 1 2 और 3

2 4 1 3 and 2 4 1 3 और 2

3 1 4 2 and 3 1 4 2 और 3

4 1 4 3 and 2 1 4 3 और 2

Correct Answer -

7)

8)

9)

1 4 3 and 2 1 4 3 और 2

CANCELLED

Karl Pearsonrsquos correlation co-efficient is काल िपयसन का सहसबध गणाक ह

1 Arithmec mean समार मा

2 Geometric mean गणोर मा

3 None of these इनम स कोई नही

4 Harmonic mean हराक मा

Correct Answer -

Geometric mean गणोर मा

CANCELLED

ldquoEach day is more or less the same the morning is clear and bright with a sea breeze as the Sun climbshigh in the sky heat mounts up dark clouds form then rain comes with thunder lighting But rain is soonoverrdquo Which of the following regions is described in the above passage

ldquoक िदन समान स अिधक या कम होता ह सम की हवा क साथ और उल सबह होती ह जस सयआकाश म ऊचा चढ़ता ह गम बढ़ जाती ह काल बादल बनत ह िफर िबजली क साथ बारश आती ह लिकनबारश जी ख हो जाती हlsquorsquo उपरो पा म िनिलखत म स िकन ो का वणन िकया गया ह

1 Equatorial भमरखीय

2 Equatorial भमरखीय

3 Savannah सवाना

4 Mediterranean आातरक (भमसागरीय)

5 Mediterranean आातरक (भमसागरीय)

6 Monsoon मानसन

7 Monsoon मानसन

Correct Answer -

Equatorial भमरखीय

Equatorial भमरखीय

CANCELLED

In which epoch of the geological history of the Earth dinosaurs reached their largest size

पी क भगभय इितहास क िकस यग म डायनासोर अपन सबस बड़ आकार तक पचि

10)

11)

1 Triassic ट ायिसक

2 Jurassic जरािसक

3 Cretaceous ीटशस

4 Permian पिमयन

Correct Answer -

Jurassic जरािसक

CANCELLED

A spring tide would occur in which of the following conditions

िनिलखत म स िकन थितयो म ार-भाटा आता ह

1 When the Sun Moon and Earth are in a straight line

जब सय चमा और पी एक सीधी रखा म होत ह

2 When the Moon and Earth are in right angle to each other

जब चमा और पी एक-दसर क दािहन कोण म होत ह

3 When the Earth and Moon are in right angle to the Sun

जब पी और चमा सय क दािहन कोण म होत ह

4 When the Sun and Moon are in right angle to each other

जब सय और चमा एक-दसर क दािहन कोण म होत ह

Correct Answer -

When the Sun Moon and Earth are in a straight line

जब सय चमा और पी एक सीधी रखा म होत ह

CANCELLED

An observe on the Earthrsquos surface always sees the same face of the moon because

एक पयवक को पी की सतह स हमशा चाद का एक ही फलक िदखाई दता ह ोिक

1 Its path of revolution around the earth is the same as that of the earth around the Sun

इसका पी क चारो ओर परमण का माग सय क चारो ओर पी क समान ही ह

2 Its period of revolution around the Earth is the same as its period of rotation around its own axis

इसकी पी क चारो ओर परमण की अविध उसकी अपनी धरी क चारो ओर घणन की अविध क समान ह

3 Its period of rotation is the same as that of the Earth इसकी घणन की अविध पी क समान ह

ी ि ी ी

12)

13)

14)

4 Its direct of rotation is the same as that of Earth घणन की िदशा पी क समान ही ह

Correct Answer -

Its period of revolution around the Earth is the same as its period of rotation around its own axis

इसकी पी क चारो ओर परमण की अविध उसकी अपनी धरी क चारो ओर घणन की अविध क समान ह

CANCELLED

The pebbles that are faceted by the sand-blasting and shaped polished by the wind abrasions are known as

पवन अपघषन ारा पॉिलश रत-िवोिटत और साच म ढला ककड़ ___________ क प म जाना जाता ह

1 Dreikanter िकोणक

2 Pediments िकोिनका

3 Inselberg इलबग

4 Dunes टीबा

Correct Answer -

Dreikanter िकोणक

CANCELLED

Astronomical unit is the average distance between खगोलीय इकाई ______ क बीच की औसत दरी ह

1 Earth and Mars पी और मगल

2 Earth and mercury पी और बध

3 Earth and moon पी और चमा

4 Earth and Sun पी और सय

Correct Answer -

Earth and Sun पी और सय

During cold weather season in the northern plains there will be an inflow of cyclonic disturbancesfrom the _________ directions

शीत मौसम क दौरान उरी मदानी इलाको म _________ िदशाओ स चवात सबधी गड़बड़ी का अतवाह होगा

1 East and Northwest पव और उरपिम

2 East and Northeast पव और पवर

3 West and East पिम और पव

ि औ ि

15)

16)

17)

4 West and Northwest पिम और उरपिम

Correct Answer -

West and Northwest पिम और उरपिम

During an earth quake the velocity of the body waves will________ along with the increase in densityof the material it is passing through

भकप क दौरान लहरो क ऊपरी भाग का वग घन म व क साथ-साथ ________ जो इसस गजरन वाली वको आग बढाएगी

1 not change नही बदलगा

2 increase initially and then decrease शआत म बढ़गा और िफर घटगा

3 increase बढ़गा

4 decrease घटगा

Correct Answer -

increase बढ़गा

The Clouded Leopard National park is situated in which of the following states

िनिलखत म िकस रा म धिमल तदआ रा ीय उान (ाउडड लपड नशनल पाक ) थत ह

1 Tripura िपरा

2 Uttar Pradesh उर दश

3 Assam असम

4 Mizoram िमजोरम

Correct Answer -

Tripura िपरा

Usually the land surfaces are heated more quickly than the water surfaces because _____________

आम तौर पर जल सतहो की तलना म भिम सतह अिधक तजी स गम होती ह ोिक _____________ ह

1 the specific heat of water is higher than land पानी की िविश ऊा भिम स अिधक

2 the specific heat of water is lesser than land पानी की िविश ऊा भिम स कम होती

3 the latent heat of water is higher than the land पानी की अतिनिहत ऊा भिम स अिधक

4 the land reflects more heat radiation than water भिम पानी की तलना म अिधक ऊा क िविकरण को पराविततकरती

18)

19)

20)

21)

Correct Answer -

the specific heat of water is higher than land पानी की िविश ऊा भिम स अिधक

The longest shore-line is along the state of सबस लबी समतटीय रखा िन रा क साथ ह

1 Maharashtra महारा

2 Orissa उड़ीसा

3 Kerala करल

4 Gujarat गजरात

Correct Answer -

Gujarat गजरात

The position when the Earth is farthest from the Sun is known as

जब पी सय स सबस दर होती ह तो उस थित को िन नाम स जाना जाता ह

1 Perihelion उपसौर

2 Vernal Equinox बसत िवषव

3 Aphelion अपसौर

4 Autumnal Equinox शराल िवषव

Correct Answer -

Aphelion अपसौर

The seasonal reversal of winds is the typical characteristic of

हवाओ का मौसमी परवतन ______ की सामा िवशषता ह

1 Mediterranean climates only कवल भमसागरीय जलवाय

2 All of the above climates उपय सभी मौसम

3 Monsoon climate only कवल मानसन जलवाय

4 Equatorial climate only कवल भमरखीय जलवाय

Correct Answer -

Monsoon climate only कवल मानसन जलवाय

In _________ rocks the minerals will occurs in beds or layers

ो ि ो ो

22)

23)

24)

______ चानो म खिनज तल या परतो म होत ह

1 metamorphic कायातरत

2 igneous and metamorphic आय और कायातरत

3 igneous आय

4 sedimentary अवसादी

Correct Answer -

sedimentary अवसादी

Black soil is ideal for the cultivation of cotton as कपास की खती क िलए काली िमी आदश ह ोिक

1 Its colour is black यह काली होती ह

2 It is found on plateau regions यह पठार ो म पायी जाती ह

3 It is made up of lava यह लावा स बनी होती ह

4 It can retain moisture यह नमी को बरकरार रख सकती ह

Correct Answer -

It can retain moisture यह नमी को बरकरार रख सकती ह

The National Survey and Mapping Organization of the country works under the Department of___________

दश का रा ीय सवण और मानिचण सगठन ___________ िवभाग क अतगत काय करता ह

1 Space अतर

2 Science and Technology िवान और तकनीक

3 Culture सित

4 Tourism पयटन

Correct Answer -

Science and Technology िवान और तकनीक

Palk strait separates India from पाक जलडमम भारत स _____ को अलग करता ह

1 Pakistan पािकान

2 Andaman Island अडमान ीप

3 China चीन

25)

26)

27)

4 Sri Lanka ीलका

Correct Answer -

Sri Lanka ीलका

Which among the following state is the major producer of Bauxite in India

िनिलखत म स कौन सा रा भारत म बॉाइट का मख उादक ह

1 Madhya Pradesh मदश

2 Rajasthan राजथान

3 Goa गोवा

4 Orissa उड़ीसा

Correct Answer -

Orissa उड़ीसा

Which of the following states DOES NOT share border with Chhattisgarh

िनिलखत म स कौन सा रा छीसगढ़ क साथ सीमा साझा नही करता ह

1 Telangana तलगाना

2 Uttar Pradesh उर दश

3 Bihar िबहार

4 Andhra Pradesh आ दश

Correct Answer -

Bihar िबहार

Which of the following statements is INCORRECT with respect to parallels of latitudes

अाश क समानातरो क सबध म िन निलखत म स कौन सा कथन गलत ह

1 A line joining places of equal latitude is known as parallel of largest

समान अाश क थानो को जोड़न वाली रखा को िवशालतम क समानातर क प म जाना जाता ह

2 They stat from equator and run parallels to it

व भम रखा स ारभ होत ह और इसक समानातर चलत ह

3 All parallels are equal in length सभी समातर लबाई म समान ह

4 All parallels are drawn as circles on the globe ोब पर सभी समानातर वो क प म खीच जात ह

28)

29)

30)

31)

Correct Answer -

All parallels are equal in length सभी समातर लबाई म समान ह

Which of the following Indian states is also known as a lsquoLand of Red river and Blue Hillsrsquo

िनिलखत म स िकस भारतीय रा को लाल नदी और नीली पहािड़यो की भिम क नाम स जाना जाता ह

1 Uttarkhand उराखड

2 Assam असम

3 Meghalaya मघालय

4 Arunachal Pradesh अणाचल दश

Correct Answer -

Assam असम

In spatial analysis of settlement Rn = 215 indicates which type of settlement arrangement

िनपटान क थािनक िवषण म Rn = 215 यह इिगत करता ह िक िकस कार की िनपटान वथा ह

1 Uniform यिनफॉम

2 Semi-Clustered समी- ल टर

3 Clustered ल टर

4 Random रडम

Correct Answer -

Uniform यिनफॉम

Who are known as the lsquoYellow Peoplersquo lsquoयलो पीपलrsquo क प म कौन जाना जाता ह

1 Mongoloids मोगोलोइडस

2 Nigroids नीोइडस

3 Australoids ऑ लॉइडस

4 Caucasoids कॉकसोइडस

Correct Answer -

Mongoloids मोगोलोइडस

ि ि ो ौ ि

32)

33)

34)

Who publishes the topographical map of India भारत क थलाकितक मानिच को कौन कािशत करता ह

1 Geographical Survey of India भारत का भौगोिलक सवण

2 Government of India भारत सरकार

3 Geological Survey of India भारत क भगभय सवण

4 Survey of India भारत का सवण

Correct Answer -

Survey of India भारत का सवण

Who among the following claimed geography to be the lsquoEcology of Manrsquo

िनिलखत म स िकसन भगोल को मन का पारथितकी कहा ह

1 Alfred Hener अड हटनर

2 Vidal-de la Blache वाइडल-िड लॉ ॉश

3 Oo Schluter ओटो टर

4 Harlan Barrow हरलन बारो

Correct Answer -

Harlan Barrow हरलन बारो

Who among the following is regarded as the founder of humanistic approach in geography

िनिलखत म स िकस भगोल म मानवतावादी िकोण का सथापक माना जाता ह

1 William Bunge िविलयम बग

2 Yi-Fu-Tuan यी-फ- यान

3 Brain JL Berry न जएल बरी

4 Richard Peet रचड पीट

Correct Answer -

Yi-Fu-Tuan यी-फ- यान

Who prepared Lorenz curve लोरज व िकसन तयार िकया

1 Geddes गडस

2 None of these इनम स कोई नही

3 Griffith Taylor ििफथ टलर

35)

36)

37)

4 Max U Lorenz म य लोरज

Correct Answer -

Max U Lorenz म य लोरज

Gulf Streams are the currents of which of the following oceans

खाड़ी की धाराए िनिलखत महासागरो म स िकसकी धाराए ह

1 North Atlantic Ocean उरी अटलािटक महासागर

2 North Pacific Ocean उरी शात महासागर

3 Arabian Sea अरब सागर

4 South Pacific Ocean दिण शात महासागर

Correct Answer -

North Atlantic Ocean उरी अटलािटक महासागर

Disintegration wearing away and removal of rock material is generally referred as

िशला पदाथ (रॉक सामी) का टटना िमटना और हटना आमतौर पर ________ क प म सदिभत िकया जाता ह

1 Shattering िवसकारक

2 Denudation अनाादन

3 Fault श

4 Decomposition िवयोजन

Correct Answer -

Denudation अनाादन

Variations in the length of day time and night from season to season are due to

मौसम स मौसम परवतन पर िदन क समय और रात क समय की अविध म िभताए िन कारण स होती ह

1 The Earthrsquos revolution round the Sun in an elliptical manner पी का दीघवाकार तरीक स सय क चारो घणन

2 The Earthrsquos rotation on its axis पी का इसकी धरी पर घणन

3 Revolution of the Earth on a tilted axis नत अ पर पी का घणन

4 Latitudinal position of the place थान की अाश थित

Correct Answer -

Revolution of the Earth on a tilted axis नत अ पर पी का घणन

38)

39)

40)

Point out the correct sequence of mountain ranges from north to south

उर स दिण तक पवत खलाओ क सही अनम को इिगत कर

1 Great Himalaya Middle Himalaya Outer Himalaya Trans Himalaya

महान िहमालय म िहमालय बा िहमालय परा िहमालय

2 Middle Himalaya Great Himalaya Trans Himalaya Outer Himalaya

म िहमालय महान िहमालय परा िहमालय बा िहमालय

3 Outer Himalaya Middle Himalaya Great Himalaya Trans Himalaya

बा िहमालय म िहमालय महान िहमालय परा िहमालय

4 Trans Himalaya Great Himalaya Middle Himalaya Outer Himalaya

परा िहमालय महान िहमालय म िहमालय बा िहमालय

Correct Answer -

Trans Himalaya Great Himalaya Middle Himalaya Outer Himalaya

परा िहमालय महान िहमालय म िहमालय बा िहमालय

Sunrsquos halo is produced by the refraction of light in सय का भामडल ______ म काश क अपवतन ाराउ होता ह

1 Ice crystals in Cirrro-Cumulus clouds पाभ-कपास मघो क बफ िल

2 Ice crystal in Cirrus clouds पाभमघो क बफ िल

3 Dust particles in Stratus clouds री मघो क धल कण

4 Water vapour in Stratus clouds री मघो क जल वा

Correct Answer -

Ice crystal in Cirrus clouds पाभमघो क बफ िल

Read the given statements and answer which of the following options isare correct

(1) The minerals present in the rocks exposed to atmosphere are not subjected to alteration

(2) Oxidation is one of the processes of chemical weathering

िदए गए कथनो को पढ़ और उर द िक िन म स कौन सास िवक सही ह

(1) वायमल स अनावत शल म उपथत खिनज परवतन क अधीन नही होता ह

(2) ऑीकरण रासायिनक अपय की ियाओ म स एक ह

ो ो

41)

42)

1 Both statements are wrong दोनो कथन गलत ह

2 Both statements are correct दोनो कथन सही ह

3 First statement is wrong and second statement is correct पहला कथन गलत ह और दसरा कथन सही ह

4 First statement is correct and second statement is wrong पहला कथन सही ह और दसरा कथन गलत ह

Correct Answer -

First statement is wrong and second statement is correct पहला कथन गलत ह और दसरा कथन सही ह

Read the given statements and answer which of the following options isare correct

1 Sunrsquos short waves enter the earth partially heating the atmosphere

2 Heated earth surface from the sun produces broader waves which interacts and heats the atmosphere

िदए गए कथन को पढ़ और उर द िक िन म स कौन सास िवक सही ह

1 सय की छोटी तरग पी म आिशक प स वश करती ह और वायमडल को ऊत करती ह

2 सय स ऊत पी की सतह िवारत तरग उ करती ह जो परर भाव डालती ह और वायमडल कोऊत करती ह

1 Both Statements 1 and 2 are correct दोनो कथन 1 और 2 सही ह

2 Both Statements 1 and 2 are wrong दोनो कथन 1 और 2 गलत ह

3 Statement 1 is wrong and only Statement 2 is correct कथन 1 गलत ह और कवल कथन 2 सही ह

4 Only statement 1 is correct कवल कथन 1 सही ह

Correct Answer -

Both Statements 1 and 2 are correct दोनो कथन 1 और 2 सही ह

Read the given statements and answer which of the following options isare correct

(1)The rocks that get changed due to heat and pressure are termed as metamorphic rocks

(2)Slate is one such type of metamorphic rock

िदए गए कथनो को पढ़ और उर द िक िन म स कौन सास िवक सही ह

(1) शल जो ऊा और दाब क कारण परवितत हो जात ह उ कायातरक शलो क प म जाना जाता ह

(2) ट एक तरह का कायातरक शल ह

1 Both statements are wrong दोनो कथन गलत ह

2 Both statements are correct दोनो कथन सही ह

3 First statement is wrong and second statement is correct पहला कथन गलत ह और दसरा कथन सही ह

ी औ

43)

44)

4 First statement is correct and second statement is wrong पहला कथन सही ह और दसरा कथन गलत ह

Correct Answer -

Both statements are correct दोनो कथन सही ह

Read the given statements and answer which of the following options isare correct

1 Higher temperature anomaly is observed in the northern hemisphere

2 Differential heating is absent in Northern Hemisphere

िदए गए कथनो को पढ़ और उर द िक िन म स कौन सास िवक सही ह

1 उरी गोलाध म उ तापमान िवसगित पायी जाती ह

2 उरी गोलाध म अतर ऊन अनपथत होती ह

1 Both Statements 1 and 2 are correct दोनो कथन 1 और 2 सही ह

2 Both Statements 1 and 2 are wrong दोनो कथन 1 और 2 गलत ह

3 Statement 1 is wrong and Statement 2 is correct कथन 1 गलत ह और कथन 2 सही ह

4 Statement 1 is correct and Statement 2 is wrong कथन 1 सही ह और कथन 2 गलत ह

Correct Answer -

Statement 1 is correct and Statement 2 is wrong कथन 1 सही ह और कथन 2 गलत ह

Read the given statements and answer which of the following options isare correct

(1) Plutonic rocks are intrusive type of igneous rocks

(2) It cools very slowly because the surrounding rock serves as insulation around the intrusion of magma

िदए गए कथनो को पढ़ और उर द िक िन म स कौन सास िवक सही ह

(1) िवतलीय शल अतवधी कार क आश शल ह

(2) यह बत धीर-धीर ठडा होता ह ोिक आस-पास क शल मा क अतवधन क चारो ओर रोधन क प म कायकरत ह

1 Both statements are wrong दोनो कथन गलत ह

2 Both statements are correct दोनो कथन सही ह

3 First statement is wrong and second statement is correct पहला कथन गलत ह और दसरा कथन सही ह

4 First statement is correct and second statement is wrong पहला कथन सही ह और दसरा कथन गलत ह

Correct Answer -

Both statements are correct दोनो कथन सही ह

45)

46)

47)

48)

The dust and ash material hurled from the volcanoes are termed as

ालामखी स िनकलन वाली धल और राख सामी को _______ क प म कहा जाता ह

1 Pyroclasc पाइरोाक

2 Hyperclastic हाइपराक

3 Hepiroclastic हिपरोाक

4 Cirroclastic िसरोाक

Correct Answer -

Pyroclasc पाइरोाक

The vertical difference in elevation between a low tide and high tide is referred as

कम ार और उ ार क बीच ऊचाई म लबवत अतर _____ स सदिभत होता ह

1 Tidal slope ारीय ढलान

2 Tidal elevation ारीय उयन

3 Tidal range ारीय परास

4 Tidal height ारीय ऊचाई

Correct Answer -

Tidal range ारीय परास

The maximum biodiversity is found in which of the following regions िनिलखत ो म स अिधकतमजव िविवधता िकसम पायी जाती ह

1 Amazon Basin अमज़न बिसन

2 East Indies ई इडीज

3 Congo Basin कागो बिसन

4 West indies व इडीज

Correct Answer -

Amazon Basin अमज़न बिसन

The cultivation of rice crop produces_______ चावल की फसल की खती ______ का उादन करती ह

1 SO2

49)

50)

51)

2 CH4

3 CFCs

4 CO2

Correct Answer -

CH4

The pressure system with higher pressure at the centre is called__________

क म उ दबाव वाली दबाव णाली को _______ कहा जाता ह

1 front अ

2 depression अवनमन

3 cyclone चवात

4 anti-cyclone ितचवात

Correct Answer -

anti-cyclone ितचवात

The Himalayan region is poor in mineral resources because िहमालयी खिनज ससाधनो म समनही ह ोिक

1 The displacement of rock strata has disturbed the arrangement of rocks and made it complex

शलीय परत क िवथापन न चानो की वथा को अवथत कर िदया ह और इस जिटल बना िदया ह

2 The climate conditions are not suitable for exploitation of minerals

जलवाय की थित खिनजो क दोहन क िलए उपय नही ह

3 The terrain makes explanation of minerals difficult and very costly due to transportation difficulties

भ-भाग परवहन की किठनाइयो क कारण खिनजो का दोहन मल और बत महगा बना दता ह

4 It is made up of crystalline rocks यह िलीय चानो स बना ह

Correct Answer -

The displacement of rock strata has disturbed the arrangement of rocks and made it complex

शलीय परत क िवथापन न चानो की वथा को अवथत कर िदया ह और इस जिटल बना िदया ह

The process through which the moisture is added to the atmosphere by vegetation is termed as

वह िया िजसक माम स वनित ारा वातावरण म नमी िमलायी जाती ह _______ क प म जानी जाती ह

52)

53)

54)

1 Condensation सघनन

2 Evapotranspiration वान-उजन

3 Radiation िविकरण

4 Precipitation वषण

Correct Answer -

Evapotranspiration वान-उजन

The process through which the terrestrial heat is transferred to air by direct contact is termed as

वह िया िजसम सपक ारा थलीय ऊा वाय म थानातरत हो जाती ह ______ क प म जानी जाती ह

1 Conduction चालन

2 Convection सवहन

3 Insolation आतपन

4 Radiation िविकरण

Correct Answer -

Conduction चालन

The largest area under mangroves is in which of the following statesunion territory

मोव क अतगत िनिलखत राोसघ शािसत दशो म स सबस बड़ा कौन सा ह

1 Andaman and Nicobar अमान और िनकोबार

2 Andhra Pradesh आ दश

3 West Bengal पिम बगाल

4 Gujarat गजरात

Correct Answer -

West Bengal पिम बगाल

The longitudinal transverse and surface waves in an earthquake originate from

भकप म दशातर अनथ और सतह तरग यहा उ होती ह

1 The focus on the surface of the Earth पी क सतह पर क -िबद म

2 The focus within the body of the Earth पी क भीतर क -िबद म

3 The epicenter within the body of the Earth पी क भीतर उपरक म

55)

56)

57)

4 The epicenter on the surface of the Earth पी क सतह पर उपरक म

Correct Answer -

The focus within the body of the Earth पी क भीतर क -िबद म

The down slope movement of material due to gravity is called______

गाकषण क कारण पदाथ की अनढाल गित को ______ कहा जाता ह

1 mass movement पदाथ सचलन

2 deposition िनप

3 erosion रण

4 volcanic movement ालामखीय सचलन

Correct Answer -

mass movement पदाथ सचलन

Shimla is cooler than Amritsar although both are on the same latitude This is because

िशमला म अमतसर स अिधक ठड ह हालािक दोनो समान अाश पर ह ऐसा ह ोिक

1 Shimla is at a greater height above sea level than Amritsar अमतसर की तलना म िशमला सम तल स अिधकऊचाई पर ह

2 Shimla is further north िशमला उर की ओर ह

3 Shimla is farther from the equator िशमला भम रखा स आग ह

4 Their longitudes differ उनकी दशातर रखाए िभ ह

Correct Answer -

Shimla is at a greater height above sea level than Amritsar अमतसर की तलना म िशमला सम तल स अिधकऊचाई पर ह

lsquoTempo of Urbanizationrsquo measures which of the following

lsquoशहरीकरण का टपोrsquo िनिलखत म स कौन सा उपाय ह

1 Speed of urbanizaon शहरीकरण की गित

2 None of the above इनम स कोई नही

3 Inequality of urbanizaon शहरीकरण की असमानता

4 Current level of urbanizaon शहरीकरण का वतमान र

Correct Answer -

58)

59)

60)

Speed of urbanizaon शहरीकरण की गित

Out of the following options choose the INCORRECT statement

िनिलखत िवको म स गलत कथन का चयन कर

1 The clear tracts in the equatorial region recover rapidly भम रखा म भभाग तजी स ठीक हो जात ह

2 The stable communities include a redwood forest a pine forest at high elevations

थर समदायो म एक रडवड वन उ ऊचाई पर एक दवदार वन शािमल ह

3 Any ecosystem moves towards maximum biomass and stability to survive

कोई भी पारथितकी त जीिवत रहन क िलए अिधकतम जवसहित और थरता की तरफ असर होता ह

4 Tropical rain forests near equator are stable ecosystems

भम रखा क पास उकिटबधीय वषा वन थर पारथितक त ह

Correct Answer -

The clear tracts in the equatorial region recover rapidly भम रखा म भभाग तजी स ठीक हो जात ह

Seasonal contrasts are maximum in मौसमी िवषमता अिधकतम ह

1 Mid latitudes म अाश म

2 Low attitudes िन अाश म

3 High latitudes उ अाश म

4 Subtropics उपोकिटबधीय म

Correct Answer -

Mid latitudes म अाश म

In India which type of forest among the following occupies the largest area

भारत म िनिलखत म स िकस कार क वन सबस बड़ा फल आािदत करत ह

1 Sub-tropical Dry Evergreen Forest उप उकिटबधीय श सदाबहार वन

2 Mountain Wet Temperate Forest पवतीय आ शीतो वन

3 Tropical Moist Deciduous Forest उकिटबधीय आ पणपाती वन

4 Tropical Wet Evergreen Forest उकिटबधीय आ सदाबहार वन

Correct Answer -

Tropical Moist Deciduous Forest उकिटबधीय आ पणपाती वन

61)

62)

63)

64)

What is the proportion of lsquoJuvenile Populationrsquo (0-14 years) in India as per 2011Census

2011 की जनगणना क अनसार भारत म जवनाइल पॉपलशन यानी िकशोर जनस या (0-14 वष) का अनपात ाह

1 3076 of total population कल जनस या का 3076

2 2764 of total population कल जनस या का 2764

3 2933 of total population कल जनस या का 2933

4 3354 of total population कल जनस या का 3354

Correct Answer -

3076 of total population कल जनस या का 3076

What is the Belfast famous for बलफा िकसक िलए मशर ह

1 Belt of cotton textile industry कपास व उोग क

2 Ship-building industry जहाज िनमाण उोग

3 Agricultural machinery किष उपकरण

4 Aero planes manufacturing वाययान िनमाण

Correct Answer -

Ship-building industry जहाज िनमाण उोग

What is the most important occupation in tropical monsoon lands

उकिटबधीय मॉनसन भिम म सबस महपण वसाय ा ह

1 Mining खनन

2 Cattle rearing मवशी पालन

3 Agriculture किष

4 Nomadic herding नोमािडक जड़ी-बिटया

Correct Answer -

Agriculture किष

What is the most important characteristics of the islands (Indian) located in the Arabian sea

अरब सागर म थत ीपो (भारतीय) की सबस महपण िवशषता ा ह

ी ो

65)

66)

67)

1 There are all of coral origins सभी कोरल मल क ह

2 There are all very small in size य सभी आकार म बत छोट ह

3 They have a very dry climate इनकी जलवाय बत श ह

4 They are extended parts of the mainland व महाीप क िवारत िह ह

Correct Answer -

There are all of coral origins सभी कोरल मल क ह

What do the basalt layers of the Deccan indicate डन की बसा परत ा इिगत करती ह

1 All of the above उपरो सभी

2 Huge volcanic eruptions in the distant past दरथ अतीत म िवशाल ालामखीय िवोट

3 The immense erosional activity of the rivers निदयो की िवशाल रण गितिविध

4 The influence of weathering मौसम का भाव

Correct Answer -

Huge volcanic eruptions in the distant past दरथ अतीत म िवशाल ालामखीय िवोट

In the structure of planet Earth below the mantle the core is mainly made up of_____

पी ह की सरचना म मटल क नीच कोर म प स______ स िनिमत होती ह

1 aluminium एमीिनयम

2 silicon िसिलकॉन

3 chromium ोिमयम

4 iron लोहा

Correct Answer -

iron लोहा

One of the major Mid Oceanic Ridge is found in मख म-महासागर चोिटयो म स एक ______ म पायाजाता ह

1 Mid Pacific Ocean म शात महासागर

2 Mid Atlantic Ocean म अटलािटक महासागर

3 Mid Indian Ocean म भारतीय महासागर

4 Mid Arctic Ocean म आक िटक महासागर

68)

69)

70)

71)

Correct Answer -

Mid Atlantic Ocean म अटलािटक महासागर

Magma that reaches the Earthrsquos surface and then solidifies is called________

मा जो पी की सतह तक पचती ह और िफर ठोस हो जाती ह ________कहलाती ह

1 quartz ाटज

2 lava लावा

3 granite नाइट

4 silicates िसिलकट

Correct Answer -

lava लावा

Isotherms are the lines of equal_______ समताप रखाए समान _______की रखाए होती ह

1 pressure दाब

2 temperature तापमान

3 rainfall वषा

4 height ऊचाई

Correct Answer -

temperature तापमान

Mark the correct sequence of passes in the Western Ghats from north to south

पिमी घाटो म उर स दिण तक दर क सही अनम को िचित कर

1 Thalghat Palghat Bhorghat थलगघाट पालघाट भोरघाट

2 Thalghat Bhorghat Palghat थलघाट भोरघाट पालघाट

3 Bhorghat Thalghat Palghat भोरघाट थलघाट पालघाट

4 Palghat Bhorghat Thalghat पालघाट भोरघाट थलघाट

Correct Answer -

Thalghat Bhorghat Palghat थलघाट भोरघाट पालघाट

Which of the following does not have influence over the climate in India

ि ि ि ी ी

72)

73)

िनिलखत म स िकसका भाव भारत की जलवाय पर नही पड़ता ह

1 Ocean currents सागर की लहर

2 Nearness to equator भम रखा स िनकटता

3 Monsoons मानसन

4 Presence of Indian ocean भारतीय महासागर की उपथित

Correct Answer -

Ocean currents सागर की लहर

Which of the following cloud types has the characteristics like vertical tall narrow and puffy

िनिलखत म स िकस कार क मघो म लबवत लबी सकीण और थलता जसी िवशषताए ह

1 Cumulonimbus तफानी मघ

2 Cumulus मघ पज

3 Cirrocumulus पाभ कपासी मघ

4 Nimbostratus वषारी मघ

Correct Answer -

Cumulus मघ पज

Which of the following statement is INCORRECT about Crude Birth Rate

िनिलखत स कौन सा कथन अशोिधत ज दर क बार म सही नही ह

1 It cannot be used for comparing fertility level between two countries with different population characteristics

इसका उपयोग िविभ जनसा िवशषताओ वाल दो दशो क बीच जनन र की तलना क िलए नही िकया जा सकता ह

2 It is a standardized measure of fertility

यह जनन मता का मानकीकत उपाय ह

3 It is effected by the age-sex composition of the population

यह आबादी की आय-िलग सरचना स भािवत होता ह

4 It is expressed per 1000 population in a given geographical unit

यह िकसी दी गई भौगोिलक इकाई म ित 1000 जनसा पर िकया जाता ह

Correct Answer -

It is a standardized measure of fertility

यह जनन मता का मानकीकत उपाय ह

74)

75)

76)

77)

Which of the following state in India experienced negative decadal growth rate during 2001 to 2011census

भारत म िनिलखत म स िकस रा म वष 2001 स 2011 की जनगणना क दौरान नकाराक िगरावट दर ई

1 Tripura िपरा

2 Nagaland नागालड

3 Haryana हरयाणा

4 Odisha ओिडसा

Correct Answer -

Nagaland नागालड

Which of the following is NOT a characteristic of peninsular rivers

िनिलखत म स कौन सी िवशषता ायीपीय निदयो म नही होती ह

1 Flow through shallow valleys उथल घािटयो क माम स वाह

2 Seasonal flow मौसमी वाह

3 Little erosional activity थोड़ी कटावदार गितिविध

4 Meandering tendency often shifting their beds घमावदार वि अर अपन तटो को थानातरत करना

Correct Answer -

Meandering tendency often shifting their beds घमावदार वि अर अपन तटो को थानातरत करना

Which of the following gases in the atmosphere absorbs heat from the Sunrsquos radiation and the Earthssurface

वायमडल म िनिलखत म स कौन सी गस सय क िविकरण और पी की सतह स ऊा को अवशोिषत करती ह

1 Neon िनयॉन

2 Carbon dioxide काबन डाइऑाइड

3 Argon आगन

4 Nitrogen नाइट ोजन

Correct Answer -

Carbon dioxide काबन डाइऑाइड

Which of the following kind of settlement pattern is found at the confluence of rivers

ि ि ि ि ो

78)

79)

80)

िनिलखत म स िकस कार का वथापन पटन निदयो क सगम पर पाया जाता ह

1 Triangular Paern िकोणीय पटन

2 Circular or Semi-Circular Paern परप या अध-परप पटन

3 Nebular Paern नबलर पटन

4 Star ndashShaped Paern ार-आकार का पटन

Correct Answer -

Triangular Paern िकोणीय पटन

Which one was not the objective of the Biosphere Reserve Projects launched by the UNESCO

यनो ारा श की गई सरित जवमडल परयोजनाओ का उ इनम स कौन सा नही था

1 To promote teaching and research िशण और अनसधान को बढ़ावा दना

2 To make agriculture sustainable किष को दीघकािलक बनाना

3 To conserve ecosystems पारथितक त को सरित करना

4 To conserve genetic diversity for a longtime लब समय तक अनवािशक िविवधता को सरित करना

Correct Answer -

To make agriculture sustainable किष को दीघकािलक बनाना

Which region of the Earth surface receives the highest amount of insulation

पी सतह का कौन सा तापावरोधन की उतम माा ा करता ह

1 Land mass थलखड

2 Savannah region सवाना

3 Water bodies जल िनकाय

4 Tropical desert उकिटबधीय रिगान

Correct Answer -

Tropical desert उकिटबधीय रिगान

Which one of the following is not a biodiversity hotspot

िनिलखत म स कौन सा जव िविवधता का म जगह नही ह

1 Eastern Himalaya पव िहमालय

2 Eastern Ghats पव घाट

81)

82)

83)

3 Indo-Myanmar भारत-ामार

4 Westerm Ghats पिमी घाट

Correct Answer -

Eastern Ghats पव घाट

Which one of the following is NOT a part of the World Network of Biosphere Reserves based on theUNESCO Man and Biosphere Programme

यनो मन और बायोीयर कायम क आधार पर िनिलखत म स कौन बायोीयर रजव क िव नटवक कािहा नही ह

1 Gulf of Mannar मार की खाड़ी

2 Seshachalam शषाचलम

3 Sunderban सदरबन

4 Nilgiri नीलिगर

Correct Answer -

Seshachalam शषाचलम

Which one of the following is an example of ldquodesert vegetationrdquo

िनिलखत म स कौन मथलीय वनित का एक उदाहरण ह

1 Mosses and lichens दलदल और शवाल

2 Temperate grassland समशीतो घास क मदान

3 Coniferous forest शकधारी वन

4 Acacia and cactus एकािसया और कस

Correct Answer -

Acacia and cactus एकािसया और कस

Which one of the following reflects more sunlight िनिलखत म स कौन सा सय की रोशनी को अिधकपरावितत करता ह

1 Paddy crop land धान फसल भिम

2 Land covered with fresh snow ताजा बफ स आािदत भिम

3 Sand desert रतीली रिगान

4 Prairie land यरी भिम

84)

85)

86)

87)

Correct Answer -

Land covered with fresh snow ताजा बफ स आािदत भिम

Which layer of the atmosphere is in contact with the surface of the earthrsquos oceans

वायमडल की कौन सी परत पी क महासागरो की सतह क सपक म ह

1 Stratosphere समताप मडल

2 Mesosphere म मडल

3 Hydrosphere जलमडल

4 Troposphere ोभ मडल

Correct Answer -

Troposphere ोभ मडल

Mediterranean Sea is a border of which of the following countries भम सागर िनिलखत दशो म सिकसकी सीमा ह

1 None of these इनम स कोई नही

2 Iraq इराक

3 Lebanon लबनान

4 Jordan जॉडन

Correct Answer -

Lebanon लबनान

Benguela ocean currents are found along which coast बगएला महासागर धाराए िकस तट क साथ पायीजाती ह

1 East Coast of South America दिण अमरका क पव तट

2 East Coast of Africa अीका क पव तट

3 West Coast of South America दिण अमरका क पिमी तट

4 West Coast of Africa अीका क पिमी तट

Correct Answer -

West Coast of Africa अीका क पिमी तट

88)

89)

90)

Due to tension a block of land on one side being pushed up or upthrown relative to the downthrown blockis referred as

तनाव क कारण नीच फ क ए खड क साप भिम का एक खड एक ओर स ऊपर धकला जाता ह या ऊपर की ओरफ का जाता ह यह _____ क प म सदिभत ह

1 Thrust fault प श

2 Normal fault सामा श

3 Reverse fault म श

4 Strike slip fault नितलब सपण श

Correct Answer -

Normal fault सामा श

Inter-tropical doldrums is a zone of ______ अतर-उकिटबधीय डोलड ______ का एक ह

1 Frontolysis टोलायिसस

2 Convergence अिभसरण

3 Inter-tropical divergence zone अतर-उकिटबधीय िवचलन

4 Local wind थानीय वाय

Correct Answer -

Convergence अिभसरण

The Horse Latitudes are regions located at about _____ north and south of the equator

हॉस अाश भम रखा क उर और दिण म लगभग _____ पर थत ह

1 30ndash60 degree Latitude 30-60 िडी अाश

2 0ndash5 degree Latitude 0-5 िडी अाश

3 30 degree Latitude 30 िडी अाश

4 60ndash90 degree Latitude 60-90 िडी अाश

Correct Answer -

30 degree Latitude 30 िडी अाश

Generally evaporation is high over which part of the Earth

आम तौर पर पी क िकस भाग पर वाीकरण अिधक होता ह

1 Equatorial maritime भमवत समीय ी ी

91)

92)

2 Equatorial continental भमवत महाीपीय

3 Polar maritime वीय समीय

4 Polar continental वीय महाीपीय

Correct Answer -

Equatorial maritime भमवत समीय

A very high temperature during summer in north western India leads to what type of climaticcondition in south

उर पिमी भारत म गम क दौरान बत अिधक तापमान होन क कारण दिण म िकस कार की जलवाय थितउ करता ह

1 Depression over arabian sea अरब सागर पर अवनमन

2 Failure monsoon मानसन िवफलता

3 Successful monsoon मानसन सफलता

4 Cyclones चवात

Correct Answer -

Successful monsoon मानसन सफलता

Lightning and thunder are the resultant effect when तिड़त और गजन परणामी भाव ह जब

1 Two massive clouds hit powerfully each other first lightning is produced and later sound is produced

दो बड़ बादल एक दसर स शशाली ढग स टकरात ह पहल आकाशीय िवदयत उ होता ह और बाद म िन उहोती ह

2 Two massive clouds come into contact with the powerful wind collision this results into first sound and thenlightning

दो बड़ बादल शशाली पवन सघ क सपक म आत ह इसका परणामप पहल िन और िफर आकाशीय िवदयतउ होता ह

3 None of the above उपरो म स कोई भी नही

4 A high density cloud contains positively and negatively charged electric ions and when this interacts light andsound are simultaneously produced

एक उ घन बादल म धनाक और ऋणाक आविशत िवदयत आयन होत ह और जब यह परर भाव डालत ह तोकाश और िन एक साथ उािदत होती ह

Correct Answer -

A high density cloud contains positively and negatively charged electric ions and when this interacts light andsound are simultaneously produced

औ ि ि ो औ ो

93)

94)

95)

एक उ घन बादल म धनाक और ऋणाक आविशत िवदयत आयन होत ह और जब यह परर भाव डालत ह तोकाश और िन एक साथ उािदत होती ह

Doon Valley is able to grow rice because दन घाटी चावल उगान म सम ह ोिक

1 Other crops cannot be grown वहा अ फसलो को उगाया नही जा सकता ह

2 People in the valley are rice eaters घाटी म लोग चावल खान वाल ह

3 There is a huge export demand of rice वहा चावल की भारी िनयात माग ह

4 It has warm summer and snow melt waters for irrigation

वहा गिमया गम होती ह िसचाई क िलए बफ का िपघला आ पानी होता ह

Correct Answer -

It has warm summer and snow melt waters for irrigation

वहा गिमया गम होती ह िसचाई क िलए बफ का िपघला आ पानी होता ह

CANCELLED

In the geological time scale the Mesozoic Era DOES NOT contains which of the following periods

भगभय समय पमान पर मजीवी यग म िन कालो म स कौन नही ह

1 Triassic ट ाइऐिसक

2 Jurassic जरिसक

3 Cretaceous चाकमय

4 Carboniferous काबनी

Correct Answer -

Carboniferous काबनी

96)

1 P-3 Q-4 R-2 S-1

2 P-3 Q-4 R-1 S-2

3 P-3 Q-4 R-1 S-2

4 P-4 Q-3 R-2 S-1

Correct Answer -

P-4 Q-3 R-2 S-1

1 P-3 Q-1 R-4 S-2

2 P-3 Q-4 R-1 S-2

3 P-3 Q-2 R-4 S-1

97)

98)

4 P-2 Q-1 R-4 S-3

Correct Answer -

P-3 Q-4 R-1 S-2

1 P-3 Q-1 R-4 S-2

2 P-2 Q-3 R-4 S-1

3 P-2 Q-1 R-3 S-4

4 P-4 Q-2 R-1 S-3

Correct Answer -

P-3 Q-1 R-4 S-2

99)

100)

1 P-3 Q-2 R-4 S-1

2 P-1 Q-2 R-3 S-4

3 P-2 Q-3 R-1 S-4

4 P-4 Q-3 R-2 S-1

Correct Answer -

P-2 Q-3 R-1 S-4

ldquoHuman geography is the study of changing relationship between the unresting man and the unstableearthrdquo was defined by

lsquolsquoमानव भगोल ाकल आदमी और अथर पी क बीच सबध परवतन का अयन हrdquo ______ ारा परभािषत िकया गयाथा

1 J Brunches ज चस

2 EC Semple ईसी सल

3 HJ Mackinder एच ज मिकदर

4 PV Blache पीवी च

Correct Answer -

EC Semple ईसी सल

Sedimentary rocks are finally and ultimately derived from the____________

अवसादी चान अततः ________ स ा की जाती ह

1 action of earth movements पी की गितिविधयो

2 marine deposit समी िनप

3 weathering of metamorphic rocks पातरत चानो क अपय

4 weathering of igneous rocks आय चानो क अपय

Correct Answer -

weathering of igneous rocks आय चानो क अपय

Page 12: High School Teacher Eligibility Test- BOARD PROFESSIONAL ...peb.mp.gov.in/results/RESULT_18/HST_RES18/Final_anwser_key/HST… · M a ndl a / मंड ल ... Under the Madhya Pradesh

7)

1)

2)

3 12

4 14

Correct Answer -

13

If A = 1 FAT = 27 then FAITH =

यिद A = 1 FAT = 27 तो FAITH =

1 41

2 40

3 42

4 44

Correct Answer -

44

Topic- PEDAGOGY

CANCELLED

Basic education became a major initiative because of the efforts of

ाथिमक िशा िन न क यास स एक मह वपण कायम बन गई

1 Dr Radhakrishnan डॉ राधाक णन

2 Dr Rajendra Prasad डॉ राज साद

3 Mahatma Gandhi महा मा गाधी

4 Rabindranath Tagore रबी नाथ टगोर

Correct Answer -

Mahatma Gandhi महा मा गाधी

The ldquoclause of phraserdquo is an unit of perception of लॉज ऑफ ज िन न क धारणा क एक इकाई ह

1 Image छिव

2 Concept अवधारणा

3 Language भाषा

4 Thought िवचार

Correct Answer -

3)

4)

5)

6)

Language भाषा

Which method is most suitable to study communication process among students

छाो क बीच सचार िया का अयन करन क िलए कौन सी िविध सबस उपय ह

1 Case Study मामल का अयन (कस टडी)

2 Systematic Observation वथत अवलोकन

3 Experimental Method योगाक िविध

4 Introspection आ-िनरीण (इट ोस शन)

Correct Answer -

Systematic Observation वथत अवलोकन

Genes in a human being are located in मानवो म जीन इनम थत होता ह

1 cytoplasm कोिशका (साइटो ला म)

2 ribosomes राइबोसोम

3 cell membranes कोिशका िझी

4 chromosomes गणस (ोमोसोम)

Correct Answer -

chromosomes गणस (ोमोसोम)

With smaller classes teachers are much more able to ____________

छोटी काओ क साथ िशक ____________ म अिधक सम होत ह

1 Go slow while teaching िशण क दौरान धीमी गित स जान

2 Narrate more personal experiences अिधक गत अनभवो को बतान

3 Adapt instruction to individual differences among students छाो क बीच गत मतभदो क िलए अनकल िनदश दन म

4 Make use of the extra space for extra-curricular activities पातर गितिविधयो क िलए अितर जगह का उपयोग करन

Correct Answer -

Adapt instruction to individual differences among students छाो क बीच गत मतभदो क िलए अनकल िनदश दन म

While engaging in a task the child gets bored This is a sign of

एक काम म होन पर बा ऊब जाता ह यह िन का सकत ह

1 the task requiring a professional approach काय को ावसाियक िकोण की आवकता ह

7)

8)

9)

2 the task becoming mechanically repetitive काय यािक प स दोहराव वाला ह

3 the child not being intelligent बा बमान नही ह

4 the child being incapable of learning बा सीखन म असमथ ह

Correct Answer -

the task becoming mechanically repetitive काय यािक प स दोहराव वाला ह

Educational Psychologists are more concerned with the learning in __________

शिणक मनोवािनक __________ म अिधगम क साथ अिधक िचितत होत ह

1 Formal environment औपचारक वातावरण

2 Informal environment अनौपचारक वातावरण

3 Physical environment भौितक वातावरण

4 Social environment सामािजक वातावरण

Correct Answer -

Formal environment औपचारक वातावरण

Special needs education is the type of education िवशष ज़रतो वाली िशा वह िशा होती ह जो

1 Given to person with disabilities अम य को दी जाती ह

2 Given to people from remote areas दर थ ो क लोगो को दी जाती ह

3 Provided to intelligent people बमान लोगो को दी जाती ह

4 Established by colonial masters औपिनविशक मखयाओ ारा थािपत की गई ह

Correct Answer -

Given to person with disabilities अम य को दी जाती ह

The Stanford-Binet scale of intelligence was first published in the year

ब क नफोड-िबनट कल को इस वष म पहली बार कािशत िकया गया था

1 1916

2 1903

3 1908

4 1900

Correct Answer -

1916

10)

11)

12)

13)

The term lsquofictional finalismrsquo was propounded by

पद lsquoकत योजनवादrsquo (िफ शनल फाइनिल म) इनक ारा ितपािदत िकया गया था

1 Skinner नर

2 Freud ायड

3 Adler एडलर

4 Pavlov पावलोव

Correct Answer -

Adler एडलर

Rational Emotive Behavior Therapy was propounded by

तक सगत भावनाक वहार थरपी िन क ारा ितपािदत की गई थी

1 Carl Jung काल यग

2 Carl Rogers काल रोजस

3 Aaron Beck आरोन बक

4 Albert Ellis अट एिलस

Correct Answer -

Albert Ellis अट एिलस

Which of the following indicates the quality of education in a school

िनिलखत म स या एक कल म िशा की गणव ता का सकतक ह

1 Text-books and Teaching-learning material पा-प तक तथा िशण व सीखन स सबिधत सामी

2 Infrastructural facilities at the school कल म आधारभत ढाच स सबिधत सिवधाए

3 Student achievement level िवािथयो का उपल तर

4 Classroom systems का की यव था

Correct Answer -

Student achievement level िवािथयो का उपल तर

Which of the following teachers can be identified with authoritarian teaching style

िनिलखत म स कौन सा िशक अिधकारवादी िशण शली क साथ पहचाना जा सकता ह

1 Laissez-faire teacher अब ध िशक

14)

15)

2 Democratic teacher लोकतीय िशक

3 Indifferent teacher िन प िशक

4 Direct instruction teacher िनदश िशक

Correct Answer -

Direct instruction teacher िनदश िशक

Who was the pioneer of classical conditioning

िचरितित ानकलन ( लािसकल कडीशिनग) क वतक कौन थ

1 Skinner नर

2 Pavlov पावलोव

3 Kohler कोहलर

4 Freud ायड

Correct Answer -

Pavlov पावलोव

Which of the following are true with reference to short term memory

1 Limited capacity

2 Brief storage of information

3 Unlimited capacity

4 Duration of storage less than twenty seconds

अ पकािलक मित क सदभ म िन न म स या स य ह

1 सीिमत मता

2 सचना का सि त भडारण

3 असीिमत मता

4 बीस सकड स कम भडारण की अविध

1 3 and 4 3 और 4

2 2 and 4 2 और 4

3 13 and 4 13 और 4

4 12 and 4 12 और 4

Correct Answer -

12 and 4 12 और 4

1)

2)

3)

4)

Topic- GEOGRAPHY

Which of the following ranges of population size is used to define Class-III city by Indian Census

भारतीय जनगणना ारा वग-III शहर को परभािषत करन क िलए िनिलखत म स िकस रज क जनसा आकारका उपयोग िकया जाता ह

1 20 000 to 49999 20 000 स 49999

2 30000 to 59999 30000 स 59999

3 24000 to 54999 24000 स 54999

4 50000 to 99999 50000 स 99999

Correct Answer -

20 000 to 49999 20 000 स 49999

Carbonaceous rocks which produce coal and oil belong to the category of rocks called_______

कोयल और तल का उादन करन वाली काबनय चान ______ नामक चानो की णी स सबिधत ह

1 metamorphic पातरत

2 sedimentary अवसादी

3 inorganic अजिवक

4 igneous आय

Correct Answer -

sedimentary अवसादी

The ruhr-complex is a major industrial centre in र-परसर िन का एक मख औोिगक क ह

1 North America उरी अमरका

2 Russia स

3 Germany जमनी

4 Europe यरोप

Correct Answer -

Germany जमनी

The term lsquoRegurrsquo refers to श lsquoरगरrsquo ______ स सबिधत ह

1 Deltaic alluvial soils डा जलोढ़ िमी

ि ी

5)

6)

2 Laterite soils लटराइट िमी

3 Red and yellow soils लाल और पीली िमी

4 Black cotton soils काली कपास िमी

Correct Answer -

Black cotton soils काली कपास िमी

Read the given statements and answer which of the following options isare correct

1 Lower the pressure greater the atmospheric disturbance

2 Air move from higher to low pressure

िदए गए कथन को पढ़ और उर द िक िन म स कौन सास िवक सही ह

1 िजतना दाब कम होगा वायमडलीय बाधाए उतनी अिधक होगी

2 वाय उ स िन दाब की ओर गित करती ह

1 Both Statements 1 and 2 are correct दोनो कथन 1 और 2 सही ह

2 Both Statements 1 and 2 are wrong दोनो कथन 1 और 2 गलत ह

3 Statement 1 is wrong and only Statement 2 is correct कथन 1 गलत ह और कवल कथन 2 सही ह

4 Statement 1 is correct and Statement 2 is wrong कथन 1 सही ह और कथन 2 गलत ह

Correct Answer -

Both Statements 1 and 2 are correct दोनो कथन 1 और 2 सही ह

CANCELLED

1 4 1 2 and 3 4 1 2 और 3

2 4 1 3 and 2 4 1 3 और 2

3 1 4 2 and 3 1 4 2 और 3

4 1 4 3 and 2 1 4 3 और 2

Correct Answer -

7)

8)

9)

1 4 3 and 2 1 4 3 और 2

CANCELLED

Karl Pearsonrsquos correlation co-efficient is काल िपयसन का सहसबध गणाक ह

1 Arithmec mean समार मा

2 Geometric mean गणोर मा

3 None of these इनम स कोई नही

4 Harmonic mean हराक मा

Correct Answer -

Geometric mean गणोर मा

CANCELLED

ldquoEach day is more or less the same the morning is clear and bright with a sea breeze as the Sun climbshigh in the sky heat mounts up dark clouds form then rain comes with thunder lighting But rain is soonoverrdquo Which of the following regions is described in the above passage

ldquoक िदन समान स अिधक या कम होता ह सम की हवा क साथ और उल सबह होती ह जस सयआकाश म ऊचा चढ़ता ह गम बढ़ जाती ह काल बादल बनत ह िफर िबजली क साथ बारश आती ह लिकनबारश जी ख हो जाती हlsquorsquo उपरो पा म िनिलखत म स िकन ो का वणन िकया गया ह

1 Equatorial भमरखीय

2 Equatorial भमरखीय

3 Savannah सवाना

4 Mediterranean आातरक (भमसागरीय)

5 Mediterranean आातरक (भमसागरीय)

6 Monsoon मानसन

7 Monsoon मानसन

Correct Answer -

Equatorial भमरखीय

Equatorial भमरखीय

CANCELLED

In which epoch of the geological history of the Earth dinosaurs reached their largest size

पी क भगभय इितहास क िकस यग म डायनासोर अपन सबस बड़ आकार तक पचि

10)

11)

1 Triassic ट ायिसक

2 Jurassic जरािसक

3 Cretaceous ीटशस

4 Permian पिमयन

Correct Answer -

Jurassic जरािसक

CANCELLED

A spring tide would occur in which of the following conditions

िनिलखत म स िकन थितयो म ार-भाटा आता ह

1 When the Sun Moon and Earth are in a straight line

जब सय चमा और पी एक सीधी रखा म होत ह

2 When the Moon and Earth are in right angle to each other

जब चमा और पी एक-दसर क दािहन कोण म होत ह

3 When the Earth and Moon are in right angle to the Sun

जब पी और चमा सय क दािहन कोण म होत ह

4 When the Sun and Moon are in right angle to each other

जब सय और चमा एक-दसर क दािहन कोण म होत ह

Correct Answer -

When the Sun Moon and Earth are in a straight line

जब सय चमा और पी एक सीधी रखा म होत ह

CANCELLED

An observe on the Earthrsquos surface always sees the same face of the moon because

एक पयवक को पी की सतह स हमशा चाद का एक ही फलक िदखाई दता ह ोिक

1 Its path of revolution around the earth is the same as that of the earth around the Sun

इसका पी क चारो ओर परमण का माग सय क चारो ओर पी क समान ही ह

2 Its period of revolution around the Earth is the same as its period of rotation around its own axis

इसकी पी क चारो ओर परमण की अविध उसकी अपनी धरी क चारो ओर घणन की अविध क समान ह

3 Its period of rotation is the same as that of the Earth इसकी घणन की अविध पी क समान ह

ी ि ी ी

12)

13)

14)

4 Its direct of rotation is the same as that of Earth घणन की िदशा पी क समान ही ह

Correct Answer -

Its period of revolution around the Earth is the same as its period of rotation around its own axis

इसकी पी क चारो ओर परमण की अविध उसकी अपनी धरी क चारो ओर घणन की अविध क समान ह

CANCELLED

The pebbles that are faceted by the sand-blasting and shaped polished by the wind abrasions are known as

पवन अपघषन ारा पॉिलश रत-िवोिटत और साच म ढला ककड़ ___________ क प म जाना जाता ह

1 Dreikanter िकोणक

2 Pediments िकोिनका

3 Inselberg इलबग

4 Dunes टीबा

Correct Answer -

Dreikanter िकोणक

CANCELLED

Astronomical unit is the average distance between खगोलीय इकाई ______ क बीच की औसत दरी ह

1 Earth and Mars पी और मगल

2 Earth and mercury पी और बध

3 Earth and moon पी और चमा

4 Earth and Sun पी और सय

Correct Answer -

Earth and Sun पी और सय

During cold weather season in the northern plains there will be an inflow of cyclonic disturbancesfrom the _________ directions

शीत मौसम क दौरान उरी मदानी इलाको म _________ िदशाओ स चवात सबधी गड़बड़ी का अतवाह होगा

1 East and Northwest पव और उरपिम

2 East and Northeast पव और पवर

3 West and East पिम और पव

ि औ ि

15)

16)

17)

4 West and Northwest पिम और उरपिम

Correct Answer -

West and Northwest पिम और उरपिम

During an earth quake the velocity of the body waves will________ along with the increase in densityof the material it is passing through

भकप क दौरान लहरो क ऊपरी भाग का वग घन म व क साथ-साथ ________ जो इसस गजरन वाली वको आग बढाएगी

1 not change नही बदलगा

2 increase initially and then decrease शआत म बढ़गा और िफर घटगा

3 increase बढ़गा

4 decrease घटगा

Correct Answer -

increase बढ़गा

The Clouded Leopard National park is situated in which of the following states

िनिलखत म िकस रा म धिमल तदआ रा ीय उान (ाउडड लपड नशनल पाक ) थत ह

1 Tripura िपरा

2 Uttar Pradesh उर दश

3 Assam असम

4 Mizoram िमजोरम

Correct Answer -

Tripura िपरा

Usually the land surfaces are heated more quickly than the water surfaces because _____________

आम तौर पर जल सतहो की तलना म भिम सतह अिधक तजी स गम होती ह ोिक _____________ ह

1 the specific heat of water is higher than land पानी की िविश ऊा भिम स अिधक

2 the specific heat of water is lesser than land पानी की िविश ऊा भिम स कम होती

3 the latent heat of water is higher than the land पानी की अतिनिहत ऊा भिम स अिधक

4 the land reflects more heat radiation than water भिम पानी की तलना म अिधक ऊा क िविकरण को पराविततकरती

18)

19)

20)

21)

Correct Answer -

the specific heat of water is higher than land पानी की िविश ऊा भिम स अिधक

The longest shore-line is along the state of सबस लबी समतटीय रखा िन रा क साथ ह

1 Maharashtra महारा

2 Orissa उड़ीसा

3 Kerala करल

4 Gujarat गजरात

Correct Answer -

Gujarat गजरात

The position when the Earth is farthest from the Sun is known as

जब पी सय स सबस दर होती ह तो उस थित को िन नाम स जाना जाता ह

1 Perihelion उपसौर

2 Vernal Equinox बसत िवषव

3 Aphelion अपसौर

4 Autumnal Equinox शराल िवषव

Correct Answer -

Aphelion अपसौर

The seasonal reversal of winds is the typical characteristic of

हवाओ का मौसमी परवतन ______ की सामा िवशषता ह

1 Mediterranean climates only कवल भमसागरीय जलवाय

2 All of the above climates उपय सभी मौसम

3 Monsoon climate only कवल मानसन जलवाय

4 Equatorial climate only कवल भमरखीय जलवाय

Correct Answer -

Monsoon climate only कवल मानसन जलवाय

In _________ rocks the minerals will occurs in beds or layers

ो ि ो ो

22)

23)

24)

______ चानो म खिनज तल या परतो म होत ह

1 metamorphic कायातरत

2 igneous and metamorphic आय और कायातरत

3 igneous आय

4 sedimentary अवसादी

Correct Answer -

sedimentary अवसादी

Black soil is ideal for the cultivation of cotton as कपास की खती क िलए काली िमी आदश ह ोिक

1 Its colour is black यह काली होती ह

2 It is found on plateau regions यह पठार ो म पायी जाती ह

3 It is made up of lava यह लावा स बनी होती ह

4 It can retain moisture यह नमी को बरकरार रख सकती ह

Correct Answer -

It can retain moisture यह नमी को बरकरार रख सकती ह

The National Survey and Mapping Organization of the country works under the Department of___________

दश का रा ीय सवण और मानिचण सगठन ___________ िवभाग क अतगत काय करता ह

1 Space अतर

2 Science and Technology िवान और तकनीक

3 Culture सित

4 Tourism पयटन

Correct Answer -

Science and Technology िवान और तकनीक

Palk strait separates India from पाक जलडमम भारत स _____ को अलग करता ह

1 Pakistan पािकान

2 Andaman Island अडमान ीप

3 China चीन

25)

26)

27)

4 Sri Lanka ीलका

Correct Answer -

Sri Lanka ीलका

Which among the following state is the major producer of Bauxite in India

िनिलखत म स कौन सा रा भारत म बॉाइट का मख उादक ह

1 Madhya Pradesh मदश

2 Rajasthan राजथान

3 Goa गोवा

4 Orissa उड़ीसा

Correct Answer -

Orissa उड़ीसा

Which of the following states DOES NOT share border with Chhattisgarh

िनिलखत म स कौन सा रा छीसगढ़ क साथ सीमा साझा नही करता ह

1 Telangana तलगाना

2 Uttar Pradesh उर दश

3 Bihar िबहार

4 Andhra Pradesh आ दश

Correct Answer -

Bihar िबहार

Which of the following statements is INCORRECT with respect to parallels of latitudes

अाश क समानातरो क सबध म िन निलखत म स कौन सा कथन गलत ह

1 A line joining places of equal latitude is known as parallel of largest

समान अाश क थानो को जोड़न वाली रखा को िवशालतम क समानातर क प म जाना जाता ह

2 They stat from equator and run parallels to it

व भम रखा स ारभ होत ह और इसक समानातर चलत ह

3 All parallels are equal in length सभी समातर लबाई म समान ह

4 All parallels are drawn as circles on the globe ोब पर सभी समानातर वो क प म खीच जात ह

28)

29)

30)

31)

Correct Answer -

All parallels are equal in length सभी समातर लबाई म समान ह

Which of the following Indian states is also known as a lsquoLand of Red river and Blue Hillsrsquo

िनिलखत म स िकस भारतीय रा को लाल नदी और नीली पहािड़यो की भिम क नाम स जाना जाता ह

1 Uttarkhand उराखड

2 Assam असम

3 Meghalaya मघालय

4 Arunachal Pradesh अणाचल दश

Correct Answer -

Assam असम

In spatial analysis of settlement Rn = 215 indicates which type of settlement arrangement

िनपटान क थािनक िवषण म Rn = 215 यह इिगत करता ह िक िकस कार की िनपटान वथा ह

1 Uniform यिनफॉम

2 Semi-Clustered समी- ल टर

3 Clustered ल टर

4 Random रडम

Correct Answer -

Uniform यिनफॉम

Who are known as the lsquoYellow Peoplersquo lsquoयलो पीपलrsquo क प म कौन जाना जाता ह

1 Mongoloids मोगोलोइडस

2 Nigroids नीोइडस

3 Australoids ऑ लॉइडस

4 Caucasoids कॉकसोइडस

Correct Answer -

Mongoloids मोगोलोइडस

ि ि ो ौ ि

32)

33)

34)

Who publishes the topographical map of India भारत क थलाकितक मानिच को कौन कािशत करता ह

1 Geographical Survey of India भारत का भौगोिलक सवण

2 Government of India भारत सरकार

3 Geological Survey of India भारत क भगभय सवण

4 Survey of India भारत का सवण

Correct Answer -

Survey of India भारत का सवण

Who among the following claimed geography to be the lsquoEcology of Manrsquo

िनिलखत म स िकसन भगोल को मन का पारथितकी कहा ह

1 Alfred Hener अड हटनर

2 Vidal-de la Blache वाइडल-िड लॉ ॉश

3 Oo Schluter ओटो टर

4 Harlan Barrow हरलन बारो

Correct Answer -

Harlan Barrow हरलन बारो

Who among the following is regarded as the founder of humanistic approach in geography

िनिलखत म स िकस भगोल म मानवतावादी िकोण का सथापक माना जाता ह

1 William Bunge िविलयम बग

2 Yi-Fu-Tuan यी-फ- यान

3 Brain JL Berry न जएल बरी

4 Richard Peet रचड पीट

Correct Answer -

Yi-Fu-Tuan यी-फ- यान

Who prepared Lorenz curve लोरज व िकसन तयार िकया

1 Geddes गडस

2 None of these इनम स कोई नही

3 Griffith Taylor ििफथ टलर

35)

36)

37)

4 Max U Lorenz म य लोरज

Correct Answer -

Max U Lorenz म य लोरज

Gulf Streams are the currents of which of the following oceans

खाड़ी की धाराए िनिलखत महासागरो म स िकसकी धाराए ह

1 North Atlantic Ocean उरी अटलािटक महासागर

2 North Pacific Ocean उरी शात महासागर

3 Arabian Sea अरब सागर

4 South Pacific Ocean दिण शात महासागर

Correct Answer -

North Atlantic Ocean उरी अटलािटक महासागर

Disintegration wearing away and removal of rock material is generally referred as

िशला पदाथ (रॉक सामी) का टटना िमटना और हटना आमतौर पर ________ क प म सदिभत िकया जाता ह

1 Shattering िवसकारक

2 Denudation अनाादन

3 Fault श

4 Decomposition िवयोजन

Correct Answer -

Denudation अनाादन

Variations in the length of day time and night from season to season are due to

मौसम स मौसम परवतन पर िदन क समय और रात क समय की अविध म िभताए िन कारण स होती ह

1 The Earthrsquos revolution round the Sun in an elliptical manner पी का दीघवाकार तरीक स सय क चारो घणन

2 The Earthrsquos rotation on its axis पी का इसकी धरी पर घणन

3 Revolution of the Earth on a tilted axis नत अ पर पी का घणन

4 Latitudinal position of the place थान की अाश थित

Correct Answer -

Revolution of the Earth on a tilted axis नत अ पर पी का घणन

38)

39)

40)

Point out the correct sequence of mountain ranges from north to south

उर स दिण तक पवत खलाओ क सही अनम को इिगत कर

1 Great Himalaya Middle Himalaya Outer Himalaya Trans Himalaya

महान िहमालय म िहमालय बा िहमालय परा िहमालय

2 Middle Himalaya Great Himalaya Trans Himalaya Outer Himalaya

म िहमालय महान िहमालय परा िहमालय बा िहमालय

3 Outer Himalaya Middle Himalaya Great Himalaya Trans Himalaya

बा िहमालय म िहमालय महान िहमालय परा िहमालय

4 Trans Himalaya Great Himalaya Middle Himalaya Outer Himalaya

परा िहमालय महान िहमालय म िहमालय बा िहमालय

Correct Answer -

Trans Himalaya Great Himalaya Middle Himalaya Outer Himalaya

परा िहमालय महान िहमालय म िहमालय बा िहमालय

Sunrsquos halo is produced by the refraction of light in सय का भामडल ______ म काश क अपवतन ाराउ होता ह

1 Ice crystals in Cirrro-Cumulus clouds पाभ-कपास मघो क बफ िल

2 Ice crystal in Cirrus clouds पाभमघो क बफ िल

3 Dust particles in Stratus clouds री मघो क धल कण

4 Water vapour in Stratus clouds री मघो क जल वा

Correct Answer -

Ice crystal in Cirrus clouds पाभमघो क बफ िल

Read the given statements and answer which of the following options isare correct

(1) The minerals present in the rocks exposed to atmosphere are not subjected to alteration

(2) Oxidation is one of the processes of chemical weathering

िदए गए कथनो को पढ़ और उर द िक िन म स कौन सास िवक सही ह

(1) वायमल स अनावत शल म उपथत खिनज परवतन क अधीन नही होता ह

(2) ऑीकरण रासायिनक अपय की ियाओ म स एक ह

ो ो

41)

42)

1 Both statements are wrong दोनो कथन गलत ह

2 Both statements are correct दोनो कथन सही ह

3 First statement is wrong and second statement is correct पहला कथन गलत ह और दसरा कथन सही ह

4 First statement is correct and second statement is wrong पहला कथन सही ह और दसरा कथन गलत ह

Correct Answer -

First statement is wrong and second statement is correct पहला कथन गलत ह और दसरा कथन सही ह

Read the given statements and answer which of the following options isare correct

1 Sunrsquos short waves enter the earth partially heating the atmosphere

2 Heated earth surface from the sun produces broader waves which interacts and heats the atmosphere

िदए गए कथन को पढ़ और उर द िक िन म स कौन सास िवक सही ह

1 सय की छोटी तरग पी म आिशक प स वश करती ह और वायमडल को ऊत करती ह

2 सय स ऊत पी की सतह िवारत तरग उ करती ह जो परर भाव डालती ह और वायमडल कोऊत करती ह

1 Both Statements 1 and 2 are correct दोनो कथन 1 और 2 सही ह

2 Both Statements 1 and 2 are wrong दोनो कथन 1 और 2 गलत ह

3 Statement 1 is wrong and only Statement 2 is correct कथन 1 गलत ह और कवल कथन 2 सही ह

4 Only statement 1 is correct कवल कथन 1 सही ह

Correct Answer -

Both Statements 1 and 2 are correct दोनो कथन 1 और 2 सही ह

Read the given statements and answer which of the following options isare correct

(1)The rocks that get changed due to heat and pressure are termed as metamorphic rocks

(2)Slate is one such type of metamorphic rock

िदए गए कथनो को पढ़ और उर द िक िन म स कौन सास िवक सही ह

(1) शल जो ऊा और दाब क कारण परवितत हो जात ह उ कायातरक शलो क प म जाना जाता ह

(2) ट एक तरह का कायातरक शल ह

1 Both statements are wrong दोनो कथन गलत ह

2 Both statements are correct दोनो कथन सही ह

3 First statement is wrong and second statement is correct पहला कथन गलत ह और दसरा कथन सही ह

ी औ

43)

44)

4 First statement is correct and second statement is wrong पहला कथन सही ह और दसरा कथन गलत ह

Correct Answer -

Both statements are correct दोनो कथन सही ह

Read the given statements and answer which of the following options isare correct

1 Higher temperature anomaly is observed in the northern hemisphere

2 Differential heating is absent in Northern Hemisphere

िदए गए कथनो को पढ़ और उर द िक िन म स कौन सास िवक सही ह

1 उरी गोलाध म उ तापमान िवसगित पायी जाती ह

2 उरी गोलाध म अतर ऊन अनपथत होती ह

1 Both Statements 1 and 2 are correct दोनो कथन 1 और 2 सही ह

2 Both Statements 1 and 2 are wrong दोनो कथन 1 और 2 गलत ह

3 Statement 1 is wrong and Statement 2 is correct कथन 1 गलत ह और कथन 2 सही ह

4 Statement 1 is correct and Statement 2 is wrong कथन 1 सही ह और कथन 2 गलत ह

Correct Answer -

Statement 1 is correct and Statement 2 is wrong कथन 1 सही ह और कथन 2 गलत ह

Read the given statements and answer which of the following options isare correct

(1) Plutonic rocks are intrusive type of igneous rocks

(2) It cools very slowly because the surrounding rock serves as insulation around the intrusion of magma

िदए गए कथनो को पढ़ और उर द िक िन म स कौन सास िवक सही ह

(1) िवतलीय शल अतवधी कार क आश शल ह

(2) यह बत धीर-धीर ठडा होता ह ोिक आस-पास क शल मा क अतवधन क चारो ओर रोधन क प म कायकरत ह

1 Both statements are wrong दोनो कथन गलत ह

2 Both statements are correct दोनो कथन सही ह

3 First statement is wrong and second statement is correct पहला कथन गलत ह और दसरा कथन सही ह

4 First statement is correct and second statement is wrong पहला कथन सही ह और दसरा कथन गलत ह

Correct Answer -

Both statements are correct दोनो कथन सही ह

45)

46)

47)

48)

The dust and ash material hurled from the volcanoes are termed as

ालामखी स िनकलन वाली धल और राख सामी को _______ क प म कहा जाता ह

1 Pyroclasc पाइरोाक

2 Hyperclastic हाइपराक

3 Hepiroclastic हिपरोाक

4 Cirroclastic िसरोाक

Correct Answer -

Pyroclasc पाइरोाक

The vertical difference in elevation between a low tide and high tide is referred as

कम ार और उ ार क बीच ऊचाई म लबवत अतर _____ स सदिभत होता ह

1 Tidal slope ारीय ढलान

2 Tidal elevation ारीय उयन

3 Tidal range ारीय परास

4 Tidal height ारीय ऊचाई

Correct Answer -

Tidal range ारीय परास

The maximum biodiversity is found in which of the following regions िनिलखत ो म स अिधकतमजव िविवधता िकसम पायी जाती ह

1 Amazon Basin अमज़न बिसन

2 East Indies ई इडीज

3 Congo Basin कागो बिसन

4 West indies व इडीज

Correct Answer -

Amazon Basin अमज़न बिसन

The cultivation of rice crop produces_______ चावल की फसल की खती ______ का उादन करती ह

1 SO2

49)

50)

51)

2 CH4

3 CFCs

4 CO2

Correct Answer -

CH4

The pressure system with higher pressure at the centre is called__________

क म उ दबाव वाली दबाव णाली को _______ कहा जाता ह

1 front अ

2 depression अवनमन

3 cyclone चवात

4 anti-cyclone ितचवात

Correct Answer -

anti-cyclone ितचवात

The Himalayan region is poor in mineral resources because िहमालयी खिनज ससाधनो म समनही ह ोिक

1 The displacement of rock strata has disturbed the arrangement of rocks and made it complex

शलीय परत क िवथापन न चानो की वथा को अवथत कर िदया ह और इस जिटल बना िदया ह

2 The climate conditions are not suitable for exploitation of minerals

जलवाय की थित खिनजो क दोहन क िलए उपय नही ह

3 The terrain makes explanation of minerals difficult and very costly due to transportation difficulties

भ-भाग परवहन की किठनाइयो क कारण खिनजो का दोहन मल और बत महगा बना दता ह

4 It is made up of crystalline rocks यह िलीय चानो स बना ह

Correct Answer -

The displacement of rock strata has disturbed the arrangement of rocks and made it complex

शलीय परत क िवथापन न चानो की वथा को अवथत कर िदया ह और इस जिटल बना िदया ह

The process through which the moisture is added to the atmosphere by vegetation is termed as

वह िया िजसक माम स वनित ारा वातावरण म नमी िमलायी जाती ह _______ क प म जानी जाती ह

52)

53)

54)

1 Condensation सघनन

2 Evapotranspiration वान-उजन

3 Radiation िविकरण

4 Precipitation वषण

Correct Answer -

Evapotranspiration वान-उजन

The process through which the terrestrial heat is transferred to air by direct contact is termed as

वह िया िजसम सपक ारा थलीय ऊा वाय म थानातरत हो जाती ह ______ क प म जानी जाती ह

1 Conduction चालन

2 Convection सवहन

3 Insolation आतपन

4 Radiation िविकरण

Correct Answer -

Conduction चालन

The largest area under mangroves is in which of the following statesunion territory

मोव क अतगत िनिलखत राोसघ शािसत दशो म स सबस बड़ा कौन सा ह

1 Andaman and Nicobar अमान और िनकोबार

2 Andhra Pradesh आ दश

3 West Bengal पिम बगाल

4 Gujarat गजरात

Correct Answer -

West Bengal पिम बगाल

The longitudinal transverse and surface waves in an earthquake originate from

भकप म दशातर अनथ और सतह तरग यहा उ होती ह

1 The focus on the surface of the Earth पी क सतह पर क -िबद म

2 The focus within the body of the Earth पी क भीतर क -िबद म

3 The epicenter within the body of the Earth पी क भीतर उपरक म

55)

56)

57)

4 The epicenter on the surface of the Earth पी क सतह पर उपरक म

Correct Answer -

The focus within the body of the Earth पी क भीतर क -िबद म

The down slope movement of material due to gravity is called______

गाकषण क कारण पदाथ की अनढाल गित को ______ कहा जाता ह

1 mass movement पदाथ सचलन

2 deposition िनप

3 erosion रण

4 volcanic movement ालामखीय सचलन

Correct Answer -

mass movement पदाथ सचलन

Shimla is cooler than Amritsar although both are on the same latitude This is because

िशमला म अमतसर स अिधक ठड ह हालािक दोनो समान अाश पर ह ऐसा ह ोिक

1 Shimla is at a greater height above sea level than Amritsar अमतसर की तलना म िशमला सम तल स अिधकऊचाई पर ह

2 Shimla is further north िशमला उर की ओर ह

3 Shimla is farther from the equator िशमला भम रखा स आग ह

4 Their longitudes differ उनकी दशातर रखाए िभ ह

Correct Answer -

Shimla is at a greater height above sea level than Amritsar अमतसर की तलना म िशमला सम तल स अिधकऊचाई पर ह

lsquoTempo of Urbanizationrsquo measures which of the following

lsquoशहरीकरण का टपोrsquo िनिलखत म स कौन सा उपाय ह

1 Speed of urbanizaon शहरीकरण की गित

2 None of the above इनम स कोई नही

3 Inequality of urbanizaon शहरीकरण की असमानता

4 Current level of urbanizaon शहरीकरण का वतमान र

Correct Answer -

58)

59)

60)

Speed of urbanizaon शहरीकरण की गित

Out of the following options choose the INCORRECT statement

िनिलखत िवको म स गलत कथन का चयन कर

1 The clear tracts in the equatorial region recover rapidly भम रखा म भभाग तजी स ठीक हो जात ह

2 The stable communities include a redwood forest a pine forest at high elevations

थर समदायो म एक रडवड वन उ ऊचाई पर एक दवदार वन शािमल ह

3 Any ecosystem moves towards maximum biomass and stability to survive

कोई भी पारथितकी त जीिवत रहन क िलए अिधकतम जवसहित और थरता की तरफ असर होता ह

4 Tropical rain forests near equator are stable ecosystems

भम रखा क पास उकिटबधीय वषा वन थर पारथितक त ह

Correct Answer -

The clear tracts in the equatorial region recover rapidly भम रखा म भभाग तजी स ठीक हो जात ह

Seasonal contrasts are maximum in मौसमी िवषमता अिधकतम ह

1 Mid latitudes म अाश म

2 Low attitudes िन अाश म

3 High latitudes उ अाश म

4 Subtropics उपोकिटबधीय म

Correct Answer -

Mid latitudes म अाश म

In India which type of forest among the following occupies the largest area

भारत म िनिलखत म स िकस कार क वन सबस बड़ा फल आािदत करत ह

1 Sub-tropical Dry Evergreen Forest उप उकिटबधीय श सदाबहार वन

2 Mountain Wet Temperate Forest पवतीय आ शीतो वन

3 Tropical Moist Deciduous Forest उकिटबधीय आ पणपाती वन

4 Tropical Wet Evergreen Forest उकिटबधीय आ सदाबहार वन

Correct Answer -

Tropical Moist Deciduous Forest उकिटबधीय आ पणपाती वन

61)

62)

63)

64)

What is the proportion of lsquoJuvenile Populationrsquo (0-14 years) in India as per 2011Census

2011 की जनगणना क अनसार भारत म जवनाइल पॉपलशन यानी िकशोर जनस या (0-14 वष) का अनपात ाह

1 3076 of total population कल जनस या का 3076

2 2764 of total population कल जनस या का 2764

3 2933 of total population कल जनस या का 2933

4 3354 of total population कल जनस या का 3354

Correct Answer -

3076 of total population कल जनस या का 3076

What is the Belfast famous for बलफा िकसक िलए मशर ह

1 Belt of cotton textile industry कपास व उोग क

2 Ship-building industry जहाज िनमाण उोग

3 Agricultural machinery किष उपकरण

4 Aero planes manufacturing वाययान िनमाण

Correct Answer -

Ship-building industry जहाज िनमाण उोग

What is the most important occupation in tropical monsoon lands

उकिटबधीय मॉनसन भिम म सबस महपण वसाय ा ह

1 Mining खनन

2 Cattle rearing मवशी पालन

3 Agriculture किष

4 Nomadic herding नोमािडक जड़ी-बिटया

Correct Answer -

Agriculture किष

What is the most important characteristics of the islands (Indian) located in the Arabian sea

अरब सागर म थत ीपो (भारतीय) की सबस महपण िवशषता ा ह

ी ो

65)

66)

67)

1 There are all of coral origins सभी कोरल मल क ह

2 There are all very small in size य सभी आकार म बत छोट ह

3 They have a very dry climate इनकी जलवाय बत श ह

4 They are extended parts of the mainland व महाीप क िवारत िह ह

Correct Answer -

There are all of coral origins सभी कोरल मल क ह

What do the basalt layers of the Deccan indicate डन की बसा परत ा इिगत करती ह

1 All of the above उपरो सभी

2 Huge volcanic eruptions in the distant past दरथ अतीत म िवशाल ालामखीय िवोट

3 The immense erosional activity of the rivers निदयो की िवशाल रण गितिविध

4 The influence of weathering मौसम का भाव

Correct Answer -

Huge volcanic eruptions in the distant past दरथ अतीत म िवशाल ालामखीय िवोट

In the structure of planet Earth below the mantle the core is mainly made up of_____

पी ह की सरचना म मटल क नीच कोर म प स______ स िनिमत होती ह

1 aluminium एमीिनयम

2 silicon िसिलकॉन

3 chromium ोिमयम

4 iron लोहा

Correct Answer -

iron लोहा

One of the major Mid Oceanic Ridge is found in मख म-महासागर चोिटयो म स एक ______ म पायाजाता ह

1 Mid Pacific Ocean म शात महासागर

2 Mid Atlantic Ocean म अटलािटक महासागर

3 Mid Indian Ocean म भारतीय महासागर

4 Mid Arctic Ocean म आक िटक महासागर

68)

69)

70)

71)

Correct Answer -

Mid Atlantic Ocean म अटलािटक महासागर

Magma that reaches the Earthrsquos surface and then solidifies is called________

मा जो पी की सतह तक पचती ह और िफर ठोस हो जाती ह ________कहलाती ह

1 quartz ाटज

2 lava लावा

3 granite नाइट

4 silicates िसिलकट

Correct Answer -

lava लावा

Isotherms are the lines of equal_______ समताप रखाए समान _______की रखाए होती ह

1 pressure दाब

2 temperature तापमान

3 rainfall वषा

4 height ऊचाई

Correct Answer -

temperature तापमान

Mark the correct sequence of passes in the Western Ghats from north to south

पिमी घाटो म उर स दिण तक दर क सही अनम को िचित कर

1 Thalghat Palghat Bhorghat थलगघाट पालघाट भोरघाट

2 Thalghat Bhorghat Palghat थलघाट भोरघाट पालघाट

3 Bhorghat Thalghat Palghat भोरघाट थलघाट पालघाट

4 Palghat Bhorghat Thalghat पालघाट भोरघाट थलघाट

Correct Answer -

Thalghat Bhorghat Palghat थलघाट भोरघाट पालघाट

Which of the following does not have influence over the climate in India

ि ि ि ी ी

72)

73)

िनिलखत म स िकसका भाव भारत की जलवाय पर नही पड़ता ह

1 Ocean currents सागर की लहर

2 Nearness to equator भम रखा स िनकटता

3 Monsoons मानसन

4 Presence of Indian ocean भारतीय महासागर की उपथित

Correct Answer -

Ocean currents सागर की लहर

Which of the following cloud types has the characteristics like vertical tall narrow and puffy

िनिलखत म स िकस कार क मघो म लबवत लबी सकीण और थलता जसी िवशषताए ह

1 Cumulonimbus तफानी मघ

2 Cumulus मघ पज

3 Cirrocumulus पाभ कपासी मघ

4 Nimbostratus वषारी मघ

Correct Answer -

Cumulus मघ पज

Which of the following statement is INCORRECT about Crude Birth Rate

िनिलखत स कौन सा कथन अशोिधत ज दर क बार म सही नही ह

1 It cannot be used for comparing fertility level between two countries with different population characteristics

इसका उपयोग िविभ जनसा िवशषताओ वाल दो दशो क बीच जनन र की तलना क िलए नही िकया जा सकता ह

2 It is a standardized measure of fertility

यह जनन मता का मानकीकत उपाय ह

3 It is effected by the age-sex composition of the population

यह आबादी की आय-िलग सरचना स भािवत होता ह

4 It is expressed per 1000 population in a given geographical unit

यह िकसी दी गई भौगोिलक इकाई म ित 1000 जनसा पर िकया जाता ह

Correct Answer -

It is a standardized measure of fertility

यह जनन मता का मानकीकत उपाय ह

74)

75)

76)

77)

Which of the following state in India experienced negative decadal growth rate during 2001 to 2011census

भारत म िनिलखत म स िकस रा म वष 2001 स 2011 की जनगणना क दौरान नकाराक िगरावट दर ई

1 Tripura िपरा

2 Nagaland नागालड

3 Haryana हरयाणा

4 Odisha ओिडसा

Correct Answer -

Nagaland नागालड

Which of the following is NOT a characteristic of peninsular rivers

िनिलखत म स कौन सी िवशषता ायीपीय निदयो म नही होती ह

1 Flow through shallow valleys उथल घािटयो क माम स वाह

2 Seasonal flow मौसमी वाह

3 Little erosional activity थोड़ी कटावदार गितिविध

4 Meandering tendency often shifting their beds घमावदार वि अर अपन तटो को थानातरत करना

Correct Answer -

Meandering tendency often shifting their beds घमावदार वि अर अपन तटो को थानातरत करना

Which of the following gases in the atmosphere absorbs heat from the Sunrsquos radiation and the Earthssurface

वायमडल म िनिलखत म स कौन सी गस सय क िविकरण और पी की सतह स ऊा को अवशोिषत करती ह

1 Neon िनयॉन

2 Carbon dioxide काबन डाइऑाइड

3 Argon आगन

4 Nitrogen नाइट ोजन

Correct Answer -

Carbon dioxide काबन डाइऑाइड

Which of the following kind of settlement pattern is found at the confluence of rivers

ि ि ि ि ो

78)

79)

80)

िनिलखत म स िकस कार का वथापन पटन निदयो क सगम पर पाया जाता ह

1 Triangular Paern िकोणीय पटन

2 Circular or Semi-Circular Paern परप या अध-परप पटन

3 Nebular Paern नबलर पटन

4 Star ndashShaped Paern ार-आकार का पटन

Correct Answer -

Triangular Paern िकोणीय पटन

Which one was not the objective of the Biosphere Reserve Projects launched by the UNESCO

यनो ारा श की गई सरित जवमडल परयोजनाओ का उ इनम स कौन सा नही था

1 To promote teaching and research िशण और अनसधान को बढ़ावा दना

2 To make agriculture sustainable किष को दीघकािलक बनाना

3 To conserve ecosystems पारथितक त को सरित करना

4 To conserve genetic diversity for a longtime लब समय तक अनवािशक िविवधता को सरित करना

Correct Answer -

To make agriculture sustainable किष को दीघकािलक बनाना

Which region of the Earth surface receives the highest amount of insulation

पी सतह का कौन सा तापावरोधन की उतम माा ा करता ह

1 Land mass थलखड

2 Savannah region सवाना

3 Water bodies जल िनकाय

4 Tropical desert उकिटबधीय रिगान

Correct Answer -

Tropical desert उकिटबधीय रिगान

Which one of the following is not a biodiversity hotspot

िनिलखत म स कौन सा जव िविवधता का म जगह नही ह

1 Eastern Himalaya पव िहमालय

2 Eastern Ghats पव घाट

81)

82)

83)

3 Indo-Myanmar भारत-ामार

4 Westerm Ghats पिमी घाट

Correct Answer -

Eastern Ghats पव घाट

Which one of the following is NOT a part of the World Network of Biosphere Reserves based on theUNESCO Man and Biosphere Programme

यनो मन और बायोीयर कायम क आधार पर िनिलखत म स कौन बायोीयर रजव क िव नटवक कािहा नही ह

1 Gulf of Mannar मार की खाड़ी

2 Seshachalam शषाचलम

3 Sunderban सदरबन

4 Nilgiri नीलिगर

Correct Answer -

Seshachalam शषाचलम

Which one of the following is an example of ldquodesert vegetationrdquo

िनिलखत म स कौन मथलीय वनित का एक उदाहरण ह

1 Mosses and lichens दलदल और शवाल

2 Temperate grassland समशीतो घास क मदान

3 Coniferous forest शकधारी वन

4 Acacia and cactus एकािसया और कस

Correct Answer -

Acacia and cactus एकािसया और कस

Which one of the following reflects more sunlight िनिलखत म स कौन सा सय की रोशनी को अिधकपरावितत करता ह

1 Paddy crop land धान फसल भिम

2 Land covered with fresh snow ताजा बफ स आािदत भिम

3 Sand desert रतीली रिगान

4 Prairie land यरी भिम

84)

85)

86)

87)

Correct Answer -

Land covered with fresh snow ताजा बफ स आािदत भिम

Which layer of the atmosphere is in contact with the surface of the earthrsquos oceans

वायमडल की कौन सी परत पी क महासागरो की सतह क सपक म ह

1 Stratosphere समताप मडल

2 Mesosphere म मडल

3 Hydrosphere जलमडल

4 Troposphere ोभ मडल

Correct Answer -

Troposphere ोभ मडल

Mediterranean Sea is a border of which of the following countries भम सागर िनिलखत दशो म सिकसकी सीमा ह

1 None of these इनम स कोई नही

2 Iraq इराक

3 Lebanon लबनान

4 Jordan जॉडन

Correct Answer -

Lebanon लबनान

Benguela ocean currents are found along which coast बगएला महासागर धाराए िकस तट क साथ पायीजाती ह

1 East Coast of South America दिण अमरका क पव तट

2 East Coast of Africa अीका क पव तट

3 West Coast of South America दिण अमरका क पिमी तट

4 West Coast of Africa अीका क पिमी तट

Correct Answer -

West Coast of Africa अीका क पिमी तट

88)

89)

90)

Due to tension a block of land on one side being pushed up or upthrown relative to the downthrown blockis referred as

तनाव क कारण नीच फ क ए खड क साप भिम का एक खड एक ओर स ऊपर धकला जाता ह या ऊपर की ओरफ का जाता ह यह _____ क प म सदिभत ह

1 Thrust fault प श

2 Normal fault सामा श

3 Reverse fault म श

4 Strike slip fault नितलब सपण श

Correct Answer -

Normal fault सामा श

Inter-tropical doldrums is a zone of ______ अतर-उकिटबधीय डोलड ______ का एक ह

1 Frontolysis टोलायिसस

2 Convergence अिभसरण

3 Inter-tropical divergence zone अतर-उकिटबधीय िवचलन

4 Local wind थानीय वाय

Correct Answer -

Convergence अिभसरण

The Horse Latitudes are regions located at about _____ north and south of the equator

हॉस अाश भम रखा क उर और दिण म लगभग _____ पर थत ह

1 30ndash60 degree Latitude 30-60 िडी अाश

2 0ndash5 degree Latitude 0-5 िडी अाश

3 30 degree Latitude 30 िडी अाश

4 60ndash90 degree Latitude 60-90 िडी अाश

Correct Answer -

30 degree Latitude 30 िडी अाश

Generally evaporation is high over which part of the Earth

आम तौर पर पी क िकस भाग पर वाीकरण अिधक होता ह

1 Equatorial maritime भमवत समीय ी ी

91)

92)

2 Equatorial continental भमवत महाीपीय

3 Polar maritime वीय समीय

4 Polar continental वीय महाीपीय

Correct Answer -

Equatorial maritime भमवत समीय

A very high temperature during summer in north western India leads to what type of climaticcondition in south

उर पिमी भारत म गम क दौरान बत अिधक तापमान होन क कारण दिण म िकस कार की जलवाय थितउ करता ह

1 Depression over arabian sea अरब सागर पर अवनमन

2 Failure monsoon मानसन िवफलता

3 Successful monsoon मानसन सफलता

4 Cyclones चवात

Correct Answer -

Successful monsoon मानसन सफलता

Lightning and thunder are the resultant effect when तिड़त और गजन परणामी भाव ह जब

1 Two massive clouds hit powerfully each other first lightning is produced and later sound is produced

दो बड़ बादल एक दसर स शशाली ढग स टकरात ह पहल आकाशीय िवदयत उ होता ह और बाद म िन उहोती ह

2 Two massive clouds come into contact with the powerful wind collision this results into first sound and thenlightning

दो बड़ बादल शशाली पवन सघ क सपक म आत ह इसका परणामप पहल िन और िफर आकाशीय िवदयतउ होता ह

3 None of the above उपरो म स कोई भी नही

4 A high density cloud contains positively and negatively charged electric ions and when this interacts light andsound are simultaneously produced

एक उ घन बादल म धनाक और ऋणाक आविशत िवदयत आयन होत ह और जब यह परर भाव डालत ह तोकाश और िन एक साथ उािदत होती ह

Correct Answer -

A high density cloud contains positively and negatively charged electric ions and when this interacts light andsound are simultaneously produced

औ ि ि ो औ ो

93)

94)

95)

एक उ घन बादल म धनाक और ऋणाक आविशत िवदयत आयन होत ह और जब यह परर भाव डालत ह तोकाश और िन एक साथ उािदत होती ह

Doon Valley is able to grow rice because दन घाटी चावल उगान म सम ह ोिक

1 Other crops cannot be grown वहा अ फसलो को उगाया नही जा सकता ह

2 People in the valley are rice eaters घाटी म लोग चावल खान वाल ह

3 There is a huge export demand of rice वहा चावल की भारी िनयात माग ह

4 It has warm summer and snow melt waters for irrigation

वहा गिमया गम होती ह िसचाई क िलए बफ का िपघला आ पानी होता ह

Correct Answer -

It has warm summer and snow melt waters for irrigation

वहा गिमया गम होती ह िसचाई क िलए बफ का िपघला आ पानी होता ह

CANCELLED

In the geological time scale the Mesozoic Era DOES NOT contains which of the following periods

भगभय समय पमान पर मजीवी यग म िन कालो म स कौन नही ह

1 Triassic ट ाइऐिसक

2 Jurassic जरिसक

3 Cretaceous चाकमय

4 Carboniferous काबनी

Correct Answer -

Carboniferous काबनी

96)

1 P-3 Q-4 R-2 S-1

2 P-3 Q-4 R-1 S-2

3 P-3 Q-4 R-1 S-2

4 P-4 Q-3 R-2 S-1

Correct Answer -

P-4 Q-3 R-2 S-1

1 P-3 Q-1 R-4 S-2

2 P-3 Q-4 R-1 S-2

3 P-3 Q-2 R-4 S-1

97)

98)

4 P-2 Q-1 R-4 S-3

Correct Answer -

P-3 Q-4 R-1 S-2

1 P-3 Q-1 R-4 S-2

2 P-2 Q-3 R-4 S-1

3 P-2 Q-1 R-3 S-4

4 P-4 Q-2 R-1 S-3

Correct Answer -

P-3 Q-1 R-4 S-2

99)

100)

1 P-3 Q-2 R-4 S-1

2 P-1 Q-2 R-3 S-4

3 P-2 Q-3 R-1 S-4

4 P-4 Q-3 R-2 S-1

Correct Answer -

P-2 Q-3 R-1 S-4

ldquoHuman geography is the study of changing relationship between the unresting man and the unstableearthrdquo was defined by

lsquolsquoमानव भगोल ाकल आदमी और अथर पी क बीच सबध परवतन का अयन हrdquo ______ ारा परभािषत िकया गयाथा

1 J Brunches ज चस

2 EC Semple ईसी सल

3 HJ Mackinder एच ज मिकदर

4 PV Blache पीवी च

Correct Answer -

EC Semple ईसी सल

Sedimentary rocks are finally and ultimately derived from the____________

अवसादी चान अततः ________ स ा की जाती ह

1 action of earth movements पी की गितिविधयो

2 marine deposit समी िनप

3 weathering of metamorphic rocks पातरत चानो क अपय

4 weathering of igneous rocks आय चानो क अपय

Correct Answer -

weathering of igneous rocks आय चानो क अपय

Page 13: High School Teacher Eligibility Test- BOARD PROFESSIONAL ...peb.mp.gov.in/results/RESULT_18/HST_RES18/Final_anwser_key/HST… · M a ndl a / मंड ल ... Under the Madhya Pradesh

3)

4)

5)

6)

Language भाषा

Which method is most suitable to study communication process among students

छाो क बीच सचार िया का अयन करन क िलए कौन सी िविध सबस उपय ह

1 Case Study मामल का अयन (कस टडी)

2 Systematic Observation वथत अवलोकन

3 Experimental Method योगाक िविध

4 Introspection आ-िनरीण (इट ोस शन)

Correct Answer -

Systematic Observation वथत अवलोकन

Genes in a human being are located in मानवो म जीन इनम थत होता ह

1 cytoplasm कोिशका (साइटो ला म)

2 ribosomes राइबोसोम

3 cell membranes कोिशका िझी

4 chromosomes गणस (ोमोसोम)

Correct Answer -

chromosomes गणस (ोमोसोम)

With smaller classes teachers are much more able to ____________

छोटी काओ क साथ िशक ____________ म अिधक सम होत ह

1 Go slow while teaching िशण क दौरान धीमी गित स जान

2 Narrate more personal experiences अिधक गत अनभवो को बतान

3 Adapt instruction to individual differences among students छाो क बीच गत मतभदो क िलए अनकल िनदश दन म

4 Make use of the extra space for extra-curricular activities पातर गितिविधयो क िलए अितर जगह का उपयोग करन

Correct Answer -

Adapt instruction to individual differences among students छाो क बीच गत मतभदो क िलए अनकल िनदश दन म

While engaging in a task the child gets bored This is a sign of

एक काम म होन पर बा ऊब जाता ह यह िन का सकत ह

1 the task requiring a professional approach काय को ावसाियक िकोण की आवकता ह

7)

8)

9)

2 the task becoming mechanically repetitive काय यािक प स दोहराव वाला ह

3 the child not being intelligent बा बमान नही ह

4 the child being incapable of learning बा सीखन म असमथ ह

Correct Answer -

the task becoming mechanically repetitive काय यािक प स दोहराव वाला ह

Educational Psychologists are more concerned with the learning in __________

शिणक मनोवािनक __________ म अिधगम क साथ अिधक िचितत होत ह

1 Formal environment औपचारक वातावरण

2 Informal environment अनौपचारक वातावरण

3 Physical environment भौितक वातावरण

4 Social environment सामािजक वातावरण

Correct Answer -

Formal environment औपचारक वातावरण

Special needs education is the type of education िवशष ज़रतो वाली िशा वह िशा होती ह जो

1 Given to person with disabilities अम य को दी जाती ह

2 Given to people from remote areas दर थ ो क लोगो को दी जाती ह

3 Provided to intelligent people बमान लोगो को दी जाती ह

4 Established by colonial masters औपिनविशक मखयाओ ारा थािपत की गई ह

Correct Answer -

Given to person with disabilities अम य को दी जाती ह

The Stanford-Binet scale of intelligence was first published in the year

ब क नफोड-िबनट कल को इस वष म पहली बार कािशत िकया गया था

1 1916

2 1903

3 1908

4 1900

Correct Answer -

1916

10)

11)

12)

13)

The term lsquofictional finalismrsquo was propounded by

पद lsquoकत योजनवादrsquo (िफ शनल फाइनिल म) इनक ारा ितपािदत िकया गया था

1 Skinner नर

2 Freud ायड

3 Adler एडलर

4 Pavlov पावलोव

Correct Answer -

Adler एडलर

Rational Emotive Behavior Therapy was propounded by

तक सगत भावनाक वहार थरपी िन क ारा ितपािदत की गई थी

1 Carl Jung काल यग

2 Carl Rogers काल रोजस

3 Aaron Beck आरोन बक

4 Albert Ellis अट एिलस

Correct Answer -

Albert Ellis अट एिलस

Which of the following indicates the quality of education in a school

िनिलखत म स या एक कल म िशा की गणव ता का सकतक ह

1 Text-books and Teaching-learning material पा-प तक तथा िशण व सीखन स सबिधत सामी

2 Infrastructural facilities at the school कल म आधारभत ढाच स सबिधत सिवधाए

3 Student achievement level िवािथयो का उपल तर

4 Classroom systems का की यव था

Correct Answer -

Student achievement level िवािथयो का उपल तर

Which of the following teachers can be identified with authoritarian teaching style

िनिलखत म स कौन सा िशक अिधकारवादी िशण शली क साथ पहचाना जा सकता ह

1 Laissez-faire teacher अब ध िशक

14)

15)

2 Democratic teacher लोकतीय िशक

3 Indifferent teacher िन प िशक

4 Direct instruction teacher िनदश िशक

Correct Answer -

Direct instruction teacher िनदश िशक

Who was the pioneer of classical conditioning

िचरितित ानकलन ( लािसकल कडीशिनग) क वतक कौन थ

1 Skinner नर

2 Pavlov पावलोव

3 Kohler कोहलर

4 Freud ायड

Correct Answer -

Pavlov पावलोव

Which of the following are true with reference to short term memory

1 Limited capacity

2 Brief storage of information

3 Unlimited capacity

4 Duration of storage less than twenty seconds

अ पकािलक मित क सदभ म िन न म स या स य ह

1 सीिमत मता

2 सचना का सि त भडारण

3 असीिमत मता

4 बीस सकड स कम भडारण की अविध

1 3 and 4 3 और 4

2 2 and 4 2 और 4

3 13 and 4 13 और 4

4 12 and 4 12 और 4

Correct Answer -

12 and 4 12 और 4

1)

2)

3)

4)

Topic- GEOGRAPHY

Which of the following ranges of population size is used to define Class-III city by Indian Census

भारतीय जनगणना ारा वग-III शहर को परभािषत करन क िलए िनिलखत म स िकस रज क जनसा आकारका उपयोग िकया जाता ह

1 20 000 to 49999 20 000 स 49999

2 30000 to 59999 30000 स 59999

3 24000 to 54999 24000 स 54999

4 50000 to 99999 50000 स 99999

Correct Answer -

20 000 to 49999 20 000 स 49999

Carbonaceous rocks which produce coal and oil belong to the category of rocks called_______

कोयल और तल का उादन करन वाली काबनय चान ______ नामक चानो की णी स सबिधत ह

1 metamorphic पातरत

2 sedimentary अवसादी

3 inorganic अजिवक

4 igneous आय

Correct Answer -

sedimentary अवसादी

The ruhr-complex is a major industrial centre in र-परसर िन का एक मख औोिगक क ह

1 North America उरी अमरका

2 Russia स

3 Germany जमनी

4 Europe यरोप

Correct Answer -

Germany जमनी

The term lsquoRegurrsquo refers to श lsquoरगरrsquo ______ स सबिधत ह

1 Deltaic alluvial soils डा जलोढ़ िमी

ि ी

5)

6)

2 Laterite soils लटराइट िमी

3 Red and yellow soils लाल और पीली िमी

4 Black cotton soils काली कपास िमी

Correct Answer -

Black cotton soils काली कपास िमी

Read the given statements and answer which of the following options isare correct

1 Lower the pressure greater the atmospheric disturbance

2 Air move from higher to low pressure

िदए गए कथन को पढ़ और उर द िक िन म स कौन सास िवक सही ह

1 िजतना दाब कम होगा वायमडलीय बाधाए उतनी अिधक होगी

2 वाय उ स िन दाब की ओर गित करती ह

1 Both Statements 1 and 2 are correct दोनो कथन 1 और 2 सही ह

2 Both Statements 1 and 2 are wrong दोनो कथन 1 और 2 गलत ह

3 Statement 1 is wrong and only Statement 2 is correct कथन 1 गलत ह और कवल कथन 2 सही ह

4 Statement 1 is correct and Statement 2 is wrong कथन 1 सही ह और कथन 2 गलत ह

Correct Answer -

Both Statements 1 and 2 are correct दोनो कथन 1 और 2 सही ह

CANCELLED

1 4 1 2 and 3 4 1 2 और 3

2 4 1 3 and 2 4 1 3 और 2

3 1 4 2 and 3 1 4 2 और 3

4 1 4 3 and 2 1 4 3 और 2

Correct Answer -

7)

8)

9)

1 4 3 and 2 1 4 3 और 2

CANCELLED

Karl Pearsonrsquos correlation co-efficient is काल िपयसन का सहसबध गणाक ह

1 Arithmec mean समार मा

2 Geometric mean गणोर मा

3 None of these इनम स कोई नही

4 Harmonic mean हराक मा

Correct Answer -

Geometric mean गणोर मा

CANCELLED

ldquoEach day is more or less the same the morning is clear and bright with a sea breeze as the Sun climbshigh in the sky heat mounts up dark clouds form then rain comes with thunder lighting But rain is soonoverrdquo Which of the following regions is described in the above passage

ldquoक िदन समान स अिधक या कम होता ह सम की हवा क साथ और उल सबह होती ह जस सयआकाश म ऊचा चढ़ता ह गम बढ़ जाती ह काल बादल बनत ह िफर िबजली क साथ बारश आती ह लिकनबारश जी ख हो जाती हlsquorsquo उपरो पा म िनिलखत म स िकन ो का वणन िकया गया ह

1 Equatorial भमरखीय

2 Equatorial भमरखीय

3 Savannah सवाना

4 Mediterranean आातरक (भमसागरीय)

5 Mediterranean आातरक (भमसागरीय)

6 Monsoon मानसन

7 Monsoon मानसन

Correct Answer -

Equatorial भमरखीय

Equatorial भमरखीय

CANCELLED

In which epoch of the geological history of the Earth dinosaurs reached their largest size

पी क भगभय इितहास क िकस यग म डायनासोर अपन सबस बड़ आकार तक पचि

10)

11)

1 Triassic ट ायिसक

2 Jurassic जरािसक

3 Cretaceous ीटशस

4 Permian पिमयन

Correct Answer -

Jurassic जरािसक

CANCELLED

A spring tide would occur in which of the following conditions

िनिलखत म स िकन थितयो म ार-भाटा आता ह

1 When the Sun Moon and Earth are in a straight line

जब सय चमा और पी एक सीधी रखा म होत ह

2 When the Moon and Earth are in right angle to each other

जब चमा और पी एक-दसर क दािहन कोण म होत ह

3 When the Earth and Moon are in right angle to the Sun

जब पी और चमा सय क दािहन कोण म होत ह

4 When the Sun and Moon are in right angle to each other

जब सय और चमा एक-दसर क दािहन कोण म होत ह

Correct Answer -

When the Sun Moon and Earth are in a straight line

जब सय चमा और पी एक सीधी रखा म होत ह

CANCELLED

An observe on the Earthrsquos surface always sees the same face of the moon because

एक पयवक को पी की सतह स हमशा चाद का एक ही फलक िदखाई दता ह ोिक

1 Its path of revolution around the earth is the same as that of the earth around the Sun

इसका पी क चारो ओर परमण का माग सय क चारो ओर पी क समान ही ह

2 Its period of revolution around the Earth is the same as its period of rotation around its own axis

इसकी पी क चारो ओर परमण की अविध उसकी अपनी धरी क चारो ओर घणन की अविध क समान ह

3 Its period of rotation is the same as that of the Earth इसकी घणन की अविध पी क समान ह

ी ि ी ी

12)

13)

14)

4 Its direct of rotation is the same as that of Earth घणन की िदशा पी क समान ही ह

Correct Answer -

Its period of revolution around the Earth is the same as its period of rotation around its own axis

इसकी पी क चारो ओर परमण की अविध उसकी अपनी धरी क चारो ओर घणन की अविध क समान ह

CANCELLED

The pebbles that are faceted by the sand-blasting and shaped polished by the wind abrasions are known as

पवन अपघषन ारा पॉिलश रत-िवोिटत और साच म ढला ककड़ ___________ क प म जाना जाता ह

1 Dreikanter िकोणक

2 Pediments िकोिनका

3 Inselberg इलबग

4 Dunes टीबा

Correct Answer -

Dreikanter िकोणक

CANCELLED

Astronomical unit is the average distance between खगोलीय इकाई ______ क बीच की औसत दरी ह

1 Earth and Mars पी और मगल

2 Earth and mercury पी और बध

3 Earth and moon पी और चमा

4 Earth and Sun पी और सय

Correct Answer -

Earth and Sun पी और सय

During cold weather season in the northern plains there will be an inflow of cyclonic disturbancesfrom the _________ directions

शीत मौसम क दौरान उरी मदानी इलाको म _________ िदशाओ स चवात सबधी गड़बड़ी का अतवाह होगा

1 East and Northwest पव और उरपिम

2 East and Northeast पव और पवर

3 West and East पिम और पव

ि औ ि

15)

16)

17)

4 West and Northwest पिम और उरपिम

Correct Answer -

West and Northwest पिम और उरपिम

During an earth quake the velocity of the body waves will________ along with the increase in densityof the material it is passing through

भकप क दौरान लहरो क ऊपरी भाग का वग घन म व क साथ-साथ ________ जो इसस गजरन वाली वको आग बढाएगी

1 not change नही बदलगा

2 increase initially and then decrease शआत म बढ़गा और िफर घटगा

3 increase बढ़गा

4 decrease घटगा

Correct Answer -

increase बढ़गा

The Clouded Leopard National park is situated in which of the following states

िनिलखत म िकस रा म धिमल तदआ रा ीय उान (ाउडड लपड नशनल पाक ) थत ह

1 Tripura िपरा

2 Uttar Pradesh उर दश

3 Assam असम

4 Mizoram िमजोरम

Correct Answer -

Tripura िपरा

Usually the land surfaces are heated more quickly than the water surfaces because _____________

आम तौर पर जल सतहो की तलना म भिम सतह अिधक तजी स गम होती ह ोिक _____________ ह

1 the specific heat of water is higher than land पानी की िविश ऊा भिम स अिधक

2 the specific heat of water is lesser than land पानी की िविश ऊा भिम स कम होती

3 the latent heat of water is higher than the land पानी की अतिनिहत ऊा भिम स अिधक

4 the land reflects more heat radiation than water भिम पानी की तलना म अिधक ऊा क िविकरण को पराविततकरती

18)

19)

20)

21)

Correct Answer -

the specific heat of water is higher than land पानी की िविश ऊा भिम स अिधक

The longest shore-line is along the state of सबस लबी समतटीय रखा िन रा क साथ ह

1 Maharashtra महारा

2 Orissa उड़ीसा

3 Kerala करल

4 Gujarat गजरात

Correct Answer -

Gujarat गजरात

The position when the Earth is farthest from the Sun is known as

जब पी सय स सबस दर होती ह तो उस थित को िन नाम स जाना जाता ह

1 Perihelion उपसौर

2 Vernal Equinox बसत िवषव

3 Aphelion अपसौर

4 Autumnal Equinox शराल िवषव

Correct Answer -

Aphelion अपसौर

The seasonal reversal of winds is the typical characteristic of

हवाओ का मौसमी परवतन ______ की सामा िवशषता ह

1 Mediterranean climates only कवल भमसागरीय जलवाय

2 All of the above climates उपय सभी मौसम

3 Monsoon climate only कवल मानसन जलवाय

4 Equatorial climate only कवल भमरखीय जलवाय

Correct Answer -

Monsoon climate only कवल मानसन जलवाय

In _________ rocks the minerals will occurs in beds or layers

ो ि ो ो

22)

23)

24)

______ चानो म खिनज तल या परतो म होत ह

1 metamorphic कायातरत

2 igneous and metamorphic आय और कायातरत

3 igneous आय

4 sedimentary अवसादी

Correct Answer -

sedimentary अवसादी

Black soil is ideal for the cultivation of cotton as कपास की खती क िलए काली िमी आदश ह ोिक

1 Its colour is black यह काली होती ह

2 It is found on plateau regions यह पठार ो म पायी जाती ह

3 It is made up of lava यह लावा स बनी होती ह

4 It can retain moisture यह नमी को बरकरार रख सकती ह

Correct Answer -

It can retain moisture यह नमी को बरकरार रख सकती ह

The National Survey and Mapping Organization of the country works under the Department of___________

दश का रा ीय सवण और मानिचण सगठन ___________ िवभाग क अतगत काय करता ह

1 Space अतर

2 Science and Technology िवान और तकनीक

3 Culture सित

4 Tourism पयटन

Correct Answer -

Science and Technology िवान और तकनीक

Palk strait separates India from पाक जलडमम भारत स _____ को अलग करता ह

1 Pakistan पािकान

2 Andaman Island अडमान ीप

3 China चीन

25)

26)

27)

4 Sri Lanka ीलका

Correct Answer -

Sri Lanka ीलका

Which among the following state is the major producer of Bauxite in India

िनिलखत म स कौन सा रा भारत म बॉाइट का मख उादक ह

1 Madhya Pradesh मदश

2 Rajasthan राजथान

3 Goa गोवा

4 Orissa उड़ीसा

Correct Answer -

Orissa उड़ीसा

Which of the following states DOES NOT share border with Chhattisgarh

िनिलखत म स कौन सा रा छीसगढ़ क साथ सीमा साझा नही करता ह

1 Telangana तलगाना

2 Uttar Pradesh उर दश

3 Bihar िबहार

4 Andhra Pradesh आ दश

Correct Answer -

Bihar िबहार

Which of the following statements is INCORRECT with respect to parallels of latitudes

अाश क समानातरो क सबध म िन निलखत म स कौन सा कथन गलत ह

1 A line joining places of equal latitude is known as parallel of largest

समान अाश क थानो को जोड़न वाली रखा को िवशालतम क समानातर क प म जाना जाता ह

2 They stat from equator and run parallels to it

व भम रखा स ारभ होत ह और इसक समानातर चलत ह

3 All parallels are equal in length सभी समातर लबाई म समान ह

4 All parallels are drawn as circles on the globe ोब पर सभी समानातर वो क प म खीच जात ह

28)

29)

30)

31)

Correct Answer -

All parallels are equal in length सभी समातर लबाई म समान ह

Which of the following Indian states is also known as a lsquoLand of Red river and Blue Hillsrsquo

िनिलखत म स िकस भारतीय रा को लाल नदी और नीली पहािड़यो की भिम क नाम स जाना जाता ह

1 Uttarkhand उराखड

2 Assam असम

3 Meghalaya मघालय

4 Arunachal Pradesh अणाचल दश

Correct Answer -

Assam असम

In spatial analysis of settlement Rn = 215 indicates which type of settlement arrangement

िनपटान क थािनक िवषण म Rn = 215 यह इिगत करता ह िक िकस कार की िनपटान वथा ह

1 Uniform यिनफॉम

2 Semi-Clustered समी- ल टर

3 Clustered ल टर

4 Random रडम

Correct Answer -

Uniform यिनफॉम

Who are known as the lsquoYellow Peoplersquo lsquoयलो पीपलrsquo क प म कौन जाना जाता ह

1 Mongoloids मोगोलोइडस

2 Nigroids नीोइडस

3 Australoids ऑ लॉइडस

4 Caucasoids कॉकसोइडस

Correct Answer -

Mongoloids मोगोलोइडस

ि ि ो ौ ि

32)

33)

34)

Who publishes the topographical map of India भारत क थलाकितक मानिच को कौन कािशत करता ह

1 Geographical Survey of India भारत का भौगोिलक सवण

2 Government of India भारत सरकार

3 Geological Survey of India भारत क भगभय सवण

4 Survey of India भारत का सवण

Correct Answer -

Survey of India भारत का सवण

Who among the following claimed geography to be the lsquoEcology of Manrsquo

िनिलखत म स िकसन भगोल को मन का पारथितकी कहा ह

1 Alfred Hener अड हटनर

2 Vidal-de la Blache वाइडल-िड लॉ ॉश

3 Oo Schluter ओटो टर

4 Harlan Barrow हरलन बारो

Correct Answer -

Harlan Barrow हरलन बारो

Who among the following is regarded as the founder of humanistic approach in geography

िनिलखत म स िकस भगोल म मानवतावादी िकोण का सथापक माना जाता ह

1 William Bunge िविलयम बग

2 Yi-Fu-Tuan यी-फ- यान

3 Brain JL Berry न जएल बरी

4 Richard Peet रचड पीट

Correct Answer -

Yi-Fu-Tuan यी-फ- यान

Who prepared Lorenz curve लोरज व िकसन तयार िकया

1 Geddes गडस

2 None of these इनम स कोई नही

3 Griffith Taylor ििफथ टलर

35)

36)

37)

4 Max U Lorenz म य लोरज

Correct Answer -

Max U Lorenz म य लोरज

Gulf Streams are the currents of which of the following oceans

खाड़ी की धाराए िनिलखत महासागरो म स िकसकी धाराए ह

1 North Atlantic Ocean उरी अटलािटक महासागर

2 North Pacific Ocean उरी शात महासागर

3 Arabian Sea अरब सागर

4 South Pacific Ocean दिण शात महासागर

Correct Answer -

North Atlantic Ocean उरी अटलािटक महासागर

Disintegration wearing away and removal of rock material is generally referred as

िशला पदाथ (रॉक सामी) का टटना िमटना और हटना आमतौर पर ________ क प म सदिभत िकया जाता ह

1 Shattering िवसकारक

2 Denudation अनाादन

3 Fault श

4 Decomposition िवयोजन

Correct Answer -

Denudation अनाादन

Variations in the length of day time and night from season to season are due to

मौसम स मौसम परवतन पर िदन क समय और रात क समय की अविध म िभताए िन कारण स होती ह

1 The Earthrsquos revolution round the Sun in an elliptical manner पी का दीघवाकार तरीक स सय क चारो घणन

2 The Earthrsquos rotation on its axis पी का इसकी धरी पर घणन

3 Revolution of the Earth on a tilted axis नत अ पर पी का घणन

4 Latitudinal position of the place थान की अाश थित

Correct Answer -

Revolution of the Earth on a tilted axis नत अ पर पी का घणन

38)

39)

40)

Point out the correct sequence of mountain ranges from north to south

उर स दिण तक पवत खलाओ क सही अनम को इिगत कर

1 Great Himalaya Middle Himalaya Outer Himalaya Trans Himalaya

महान िहमालय म िहमालय बा िहमालय परा िहमालय

2 Middle Himalaya Great Himalaya Trans Himalaya Outer Himalaya

म िहमालय महान िहमालय परा िहमालय बा िहमालय

3 Outer Himalaya Middle Himalaya Great Himalaya Trans Himalaya

बा िहमालय म िहमालय महान िहमालय परा िहमालय

4 Trans Himalaya Great Himalaya Middle Himalaya Outer Himalaya

परा िहमालय महान िहमालय म िहमालय बा िहमालय

Correct Answer -

Trans Himalaya Great Himalaya Middle Himalaya Outer Himalaya

परा िहमालय महान िहमालय म िहमालय बा िहमालय

Sunrsquos halo is produced by the refraction of light in सय का भामडल ______ म काश क अपवतन ाराउ होता ह

1 Ice crystals in Cirrro-Cumulus clouds पाभ-कपास मघो क बफ िल

2 Ice crystal in Cirrus clouds पाभमघो क बफ िल

3 Dust particles in Stratus clouds री मघो क धल कण

4 Water vapour in Stratus clouds री मघो क जल वा

Correct Answer -

Ice crystal in Cirrus clouds पाभमघो क बफ िल

Read the given statements and answer which of the following options isare correct

(1) The minerals present in the rocks exposed to atmosphere are not subjected to alteration

(2) Oxidation is one of the processes of chemical weathering

िदए गए कथनो को पढ़ और उर द िक िन म स कौन सास िवक सही ह

(1) वायमल स अनावत शल म उपथत खिनज परवतन क अधीन नही होता ह

(2) ऑीकरण रासायिनक अपय की ियाओ म स एक ह

ो ो

41)

42)

1 Both statements are wrong दोनो कथन गलत ह

2 Both statements are correct दोनो कथन सही ह

3 First statement is wrong and second statement is correct पहला कथन गलत ह और दसरा कथन सही ह

4 First statement is correct and second statement is wrong पहला कथन सही ह और दसरा कथन गलत ह

Correct Answer -

First statement is wrong and second statement is correct पहला कथन गलत ह और दसरा कथन सही ह

Read the given statements and answer which of the following options isare correct

1 Sunrsquos short waves enter the earth partially heating the atmosphere

2 Heated earth surface from the sun produces broader waves which interacts and heats the atmosphere

िदए गए कथन को पढ़ और उर द िक िन म स कौन सास िवक सही ह

1 सय की छोटी तरग पी म आिशक प स वश करती ह और वायमडल को ऊत करती ह

2 सय स ऊत पी की सतह िवारत तरग उ करती ह जो परर भाव डालती ह और वायमडल कोऊत करती ह

1 Both Statements 1 and 2 are correct दोनो कथन 1 और 2 सही ह

2 Both Statements 1 and 2 are wrong दोनो कथन 1 और 2 गलत ह

3 Statement 1 is wrong and only Statement 2 is correct कथन 1 गलत ह और कवल कथन 2 सही ह

4 Only statement 1 is correct कवल कथन 1 सही ह

Correct Answer -

Both Statements 1 and 2 are correct दोनो कथन 1 और 2 सही ह

Read the given statements and answer which of the following options isare correct

(1)The rocks that get changed due to heat and pressure are termed as metamorphic rocks

(2)Slate is one such type of metamorphic rock

िदए गए कथनो को पढ़ और उर द िक िन म स कौन सास िवक सही ह

(1) शल जो ऊा और दाब क कारण परवितत हो जात ह उ कायातरक शलो क प म जाना जाता ह

(2) ट एक तरह का कायातरक शल ह

1 Both statements are wrong दोनो कथन गलत ह

2 Both statements are correct दोनो कथन सही ह

3 First statement is wrong and second statement is correct पहला कथन गलत ह और दसरा कथन सही ह

ी औ

43)

44)

4 First statement is correct and second statement is wrong पहला कथन सही ह और दसरा कथन गलत ह

Correct Answer -

Both statements are correct दोनो कथन सही ह

Read the given statements and answer which of the following options isare correct

1 Higher temperature anomaly is observed in the northern hemisphere

2 Differential heating is absent in Northern Hemisphere

िदए गए कथनो को पढ़ और उर द िक िन म स कौन सास िवक सही ह

1 उरी गोलाध म उ तापमान िवसगित पायी जाती ह

2 उरी गोलाध म अतर ऊन अनपथत होती ह

1 Both Statements 1 and 2 are correct दोनो कथन 1 और 2 सही ह

2 Both Statements 1 and 2 are wrong दोनो कथन 1 और 2 गलत ह

3 Statement 1 is wrong and Statement 2 is correct कथन 1 गलत ह और कथन 2 सही ह

4 Statement 1 is correct and Statement 2 is wrong कथन 1 सही ह और कथन 2 गलत ह

Correct Answer -

Statement 1 is correct and Statement 2 is wrong कथन 1 सही ह और कथन 2 गलत ह

Read the given statements and answer which of the following options isare correct

(1) Plutonic rocks are intrusive type of igneous rocks

(2) It cools very slowly because the surrounding rock serves as insulation around the intrusion of magma

िदए गए कथनो को पढ़ और उर द िक िन म स कौन सास िवक सही ह

(1) िवतलीय शल अतवधी कार क आश शल ह

(2) यह बत धीर-धीर ठडा होता ह ोिक आस-पास क शल मा क अतवधन क चारो ओर रोधन क प म कायकरत ह

1 Both statements are wrong दोनो कथन गलत ह

2 Both statements are correct दोनो कथन सही ह

3 First statement is wrong and second statement is correct पहला कथन गलत ह और दसरा कथन सही ह

4 First statement is correct and second statement is wrong पहला कथन सही ह और दसरा कथन गलत ह

Correct Answer -

Both statements are correct दोनो कथन सही ह

45)

46)

47)

48)

The dust and ash material hurled from the volcanoes are termed as

ालामखी स िनकलन वाली धल और राख सामी को _______ क प म कहा जाता ह

1 Pyroclasc पाइरोाक

2 Hyperclastic हाइपराक

3 Hepiroclastic हिपरोाक

4 Cirroclastic िसरोाक

Correct Answer -

Pyroclasc पाइरोाक

The vertical difference in elevation between a low tide and high tide is referred as

कम ार और उ ार क बीच ऊचाई म लबवत अतर _____ स सदिभत होता ह

1 Tidal slope ारीय ढलान

2 Tidal elevation ारीय उयन

3 Tidal range ारीय परास

4 Tidal height ारीय ऊचाई

Correct Answer -

Tidal range ारीय परास

The maximum biodiversity is found in which of the following regions िनिलखत ो म स अिधकतमजव िविवधता िकसम पायी जाती ह

1 Amazon Basin अमज़न बिसन

2 East Indies ई इडीज

3 Congo Basin कागो बिसन

4 West indies व इडीज

Correct Answer -

Amazon Basin अमज़न बिसन

The cultivation of rice crop produces_______ चावल की फसल की खती ______ का उादन करती ह

1 SO2

49)

50)

51)

2 CH4

3 CFCs

4 CO2

Correct Answer -

CH4

The pressure system with higher pressure at the centre is called__________

क म उ दबाव वाली दबाव णाली को _______ कहा जाता ह

1 front अ

2 depression अवनमन

3 cyclone चवात

4 anti-cyclone ितचवात

Correct Answer -

anti-cyclone ितचवात

The Himalayan region is poor in mineral resources because िहमालयी खिनज ससाधनो म समनही ह ोिक

1 The displacement of rock strata has disturbed the arrangement of rocks and made it complex

शलीय परत क िवथापन न चानो की वथा को अवथत कर िदया ह और इस जिटल बना िदया ह

2 The climate conditions are not suitable for exploitation of minerals

जलवाय की थित खिनजो क दोहन क िलए उपय नही ह

3 The terrain makes explanation of minerals difficult and very costly due to transportation difficulties

भ-भाग परवहन की किठनाइयो क कारण खिनजो का दोहन मल और बत महगा बना दता ह

4 It is made up of crystalline rocks यह िलीय चानो स बना ह

Correct Answer -

The displacement of rock strata has disturbed the arrangement of rocks and made it complex

शलीय परत क िवथापन न चानो की वथा को अवथत कर िदया ह और इस जिटल बना िदया ह

The process through which the moisture is added to the atmosphere by vegetation is termed as

वह िया िजसक माम स वनित ारा वातावरण म नमी िमलायी जाती ह _______ क प म जानी जाती ह

52)

53)

54)

1 Condensation सघनन

2 Evapotranspiration वान-उजन

3 Radiation िविकरण

4 Precipitation वषण

Correct Answer -

Evapotranspiration वान-उजन

The process through which the terrestrial heat is transferred to air by direct contact is termed as

वह िया िजसम सपक ारा थलीय ऊा वाय म थानातरत हो जाती ह ______ क प म जानी जाती ह

1 Conduction चालन

2 Convection सवहन

3 Insolation आतपन

4 Radiation िविकरण

Correct Answer -

Conduction चालन

The largest area under mangroves is in which of the following statesunion territory

मोव क अतगत िनिलखत राोसघ शािसत दशो म स सबस बड़ा कौन सा ह

1 Andaman and Nicobar अमान और िनकोबार

2 Andhra Pradesh आ दश

3 West Bengal पिम बगाल

4 Gujarat गजरात

Correct Answer -

West Bengal पिम बगाल

The longitudinal transverse and surface waves in an earthquake originate from

भकप म दशातर अनथ और सतह तरग यहा उ होती ह

1 The focus on the surface of the Earth पी क सतह पर क -िबद म

2 The focus within the body of the Earth पी क भीतर क -िबद म

3 The epicenter within the body of the Earth पी क भीतर उपरक म

55)

56)

57)

4 The epicenter on the surface of the Earth पी क सतह पर उपरक म

Correct Answer -

The focus within the body of the Earth पी क भीतर क -िबद म

The down slope movement of material due to gravity is called______

गाकषण क कारण पदाथ की अनढाल गित को ______ कहा जाता ह

1 mass movement पदाथ सचलन

2 deposition िनप

3 erosion रण

4 volcanic movement ालामखीय सचलन

Correct Answer -

mass movement पदाथ सचलन

Shimla is cooler than Amritsar although both are on the same latitude This is because

िशमला म अमतसर स अिधक ठड ह हालािक दोनो समान अाश पर ह ऐसा ह ोिक

1 Shimla is at a greater height above sea level than Amritsar अमतसर की तलना म िशमला सम तल स अिधकऊचाई पर ह

2 Shimla is further north िशमला उर की ओर ह

3 Shimla is farther from the equator िशमला भम रखा स आग ह

4 Their longitudes differ उनकी दशातर रखाए िभ ह

Correct Answer -

Shimla is at a greater height above sea level than Amritsar अमतसर की तलना म िशमला सम तल स अिधकऊचाई पर ह

lsquoTempo of Urbanizationrsquo measures which of the following

lsquoशहरीकरण का टपोrsquo िनिलखत म स कौन सा उपाय ह

1 Speed of urbanizaon शहरीकरण की गित

2 None of the above इनम स कोई नही

3 Inequality of urbanizaon शहरीकरण की असमानता

4 Current level of urbanizaon शहरीकरण का वतमान र

Correct Answer -

58)

59)

60)

Speed of urbanizaon शहरीकरण की गित

Out of the following options choose the INCORRECT statement

िनिलखत िवको म स गलत कथन का चयन कर

1 The clear tracts in the equatorial region recover rapidly भम रखा म भभाग तजी स ठीक हो जात ह

2 The stable communities include a redwood forest a pine forest at high elevations

थर समदायो म एक रडवड वन उ ऊचाई पर एक दवदार वन शािमल ह

3 Any ecosystem moves towards maximum biomass and stability to survive

कोई भी पारथितकी त जीिवत रहन क िलए अिधकतम जवसहित और थरता की तरफ असर होता ह

4 Tropical rain forests near equator are stable ecosystems

भम रखा क पास उकिटबधीय वषा वन थर पारथितक त ह

Correct Answer -

The clear tracts in the equatorial region recover rapidly भम रखा म भभाग तजी स ठीक हो जात ह

Seasonal contrasts are maximum in मौसमी िवषमता अिधकतम ह

1 Mid latitudes म अाश म

2 Low attitudes िन अाश म

3 High latitudes उ अाश म

4 Subtropics उपोकिटबधीय म

Correct Answer -

Mid latitudes म अाश म

In India which type of forest among the following occupies the largest area

भारत म िनिलखत म स िकस कार क वन सबस बड़ा फल आािदत करत ह

1 Sub-tropical Dry Evergreen Forest उप उकिटबधीय श सदाबहार वन

2 Mountain Wet Temperate Forest पवतीय आ शीतो वन

3 Tropical Moist Deciduous Forest उकिटबधीय आ पणपाती वन

4 Tropical Wet Evergreen Forest उकिटबधीय आ सदाबहार वन

Correct Answer -

Tropical Moist Deciduous Forest उकिटबधीय आ पणपाती वन

61)

62)

63)

64)

What is the proportion of lsquoJuvenile Populationrsquo (0-14 years) in India as per 2011Census

2011 की जनगणना क अनसार भारत म जवनाइल पॉपलशन यानी िकशोर जनस या (0-14 वष) का अनपात ाह

1 3076 of total population कल जनस या का 3076

2 2764 of total population कल जनस या का 2764

3 2933 of total population कल जनस या का 2933

4 3354 of total population कल जनस या का 3354

Correct Answer -

3076 of total population कल जनस या का 3076

What is the Belfast famous for बलफा िकसक िलए मशर ह

1 Belt of cotton textile industry कपास व उोग क

2 Ship-building industry जहाज िनमाण उोग

3 Agricultural machinery किष उपकरण

4 Aero planes manufacturing वाययान िनमाण

Correct Answer -

Ship-building industry जहाज िनमाण उोग

What is the most important occupation in tropical monsoon lands

उकिटबधीय मॉनसन भिम म सबस महपण वसाय ा ह

1 Mining खनन

2 Cattle rearing मवशी पालन

3 Agriculture किष

4 Nomadic herding नोमािडक जड़ी-बिटया

Correct Answer -

Agriculture किष

What is the most important characteristics of the islands (Indian) located in the Arabian sea

अरब सागर म थत ीपो (भारतीय) की सबस महपण िवशषता ा ह

ी ो

65)

66)

67)

1 There are all of coral origins सभी कोरल मल क ह

2 There are all very small in size य सभी आकार म बत छोट ह

3 They have a very dry climate इनकी जलवाय बत श ह

4 They are extended parts of the mainland व महाीप क िवारत िह ह

Correct Answer -

There are all of coral origins सभी कोरल मल क ह

What do the basalt layers of the Deccan indicate डन की बसा परत ा इिगत करती ह

1 All of the above उपरो सभी

2 Huge volcanic eruptions in the distant past दरथ अतीत म िवशाल ालामखीय िवोट

3 The immense erosional activity of the rivers निदयो की िवशाल रण गितिविध

4 The influence of weathering मौसम का भाव

Correct Answer -

Huge volcanic eruptions in the distant past दरथ अतीत म िवशाल ालामखीय िवोट

In the structure of planet Earth below the mantle the core is mainly made up of_____

पी ह की सरचना म मटल क नीच कोर म प स______ स िनिमत होती ह

1 aluminium एमीिनयम

2 silicon िसिलकॉन

3 chromium ोिमयम

4 iron लोहा

Correct Answer -

iron लोहा

One of the major Mid Oceanic Ridge is found in मख म-महासागर चोिटयो म स एक ______ म पायाजाता ह

1 Mid Pacific Ocean म शात महासागर

2 Mid Atlantic Ocean म अटलािटक महासागर

3 Mid Indian Ocean म भारतीय महासागर

4 Mid Arctic Ocean म आक िटक महासागर

68)

69)

70)

71)

Correct Answer -

Mid Atlantic Ocean म अटलािटक महासागर

Magma that reaches the Earthrsquos surface and then solidifies is called________

मा जो पी की सतह तक पचती ह और िफर ठोस हो जाती ह ________कहलाती ह

1 quartz ाटज

2 lava लावा

3 granite नाइट

4 silicates िसिलकट

Correct Answer -

lava लावा

Isotherms are the lines of equal_______ समताप रखाए समान _______की रखाए होती ह

1 pressure दाब

2 temperature तापमान

3 rainfall वषा

4 height ऊचाई

Correct Answer -

temperature तापमान

Mark the correct sequence of passes in the Western Ghats from north to south

पिमी घाटो म उर स दिण तक दर क सही अनम को िचित कर

1 Thalghat Palghat Bhorghat थलगघाट पालघाट भोरघाट

2 Thalghat Bhorghat Palghat थलघाट भोरघाट पालघाट

3 Bhorghat Thalghat Palghat भोरघाट थलघाट पालघाट

4 Palghat Bhorghat Thalghat पालघाट भोरघाट थलघाट

Correct Answer -

Thalghat Bhorghat Palghat थलघाट भोरघाट पालघाट

Which of the following does not have influence over the climate in India

ि ि ि ी ी

72)

73)

िनिलखत म स िकसका भाव भारत की जलवाय पर नही पड़ता ह

1 Ocean currents सागर की लहर

2 Nearness to equator भम रखा स िनकटता

3 Monsoons मानसन

4 Presence of Indian ocean भारतीय महासागर की उपथित

Correct Answer -

Ocean currents सागर की लहर

Which of the following cloud types has the characteristics like vertical tall narrow and puffy

िनिलखत म स िकस कार क मघो म लबवत लबी सकीण और थलता जसी िवशषताए ह

1 Cumulonimbus तफानी मघ

2 Cumulus मघ पज

3 Cirrocumulus पाभ कपासी मघ

4 Nimbostratus वषारी मघ

Correct Answer -

Cumulus मघ पज

Which of the following statement is INCORRECT about Crude Birth Rate

िनिलखत स कौन सा कथन अशोिधत ज दर क बार म सही नही ह

1 It cannot be used for comparing fertility level between two countries with different population characteristics

इसका उपयोग िविभ जनसा िवशषताओ वाल दो दशो क बीच जनन र की तलना क िलए नही िकया जा सकता ह

2 It is a standardized measure of fertility

यह जनन मता का मानकीकत उपाय ह

3 It is effected by the age-sex composition of the population

यह आबादी की आय-िलग सरचना स भािवत होता ह

4 It is expressed per 1000 population in a given geographical unit

यह िकसी दी गई भौगोिलक इकाई म ित 1000 जनसा पर िकया जाता ह

Correct Answer -

It is a standardized measure of fertility

यह जनन मता का मानकीकत उपाय ह

74)

75)

76)

77)

Which of the following state in India experienced negative decadal growth rate during 2001 to 2011census

भारत म िनिलखत म स िकस रा म वष 2001 स 2011 की जनगणना क दौरान नकाराक िगरावट दर ई

1 Tripura िपरा

2 Nagaland नागालड

3 Haryana हरयाणा

4 Odisha ओिडसा

Correct Answer -

Nagaland नागालड

Which of the following is NOT a characteristic of peninsular rivers

िनिलखत म स कौन सी िवशषता ायीपीय निदयो म नही होती ह

1 Flow through shallow valleys उथल घािटयो क माम स वाह

2 Seasonal flow मौसमी वाह

3 Little erosional activity थोड़ी कटावदार गितिविध

4 Meandering tendency often shifting their beds घमावदार वि अर अपन तटो को थानातरत करना

Correct Answer -

Meandering tendency often shifting their beds घमावदार वि अर अपन तटो को थानातरत करना

Which of the following gases in the atmosphere absorbs heat from the Sunrsquos radiation and the Earthssurface

वायमडल म िनिलखत म स कौन सी गस सय क िविकरण और पी की सतह स ऊा को अवशोिषत करती ह

1 Neon िनयॉन

2 Carbon dioxide काबन डाइऑाइड

3 Argon आगन

4 Nitrogen नाइट ोजन

Correct Answer -

Carbon dioxide काबन डाइऑाइड

Which of the following kind of settlement pattern is found at the confluence of rivers

ि ि ि ि ो

78)

79)

80)

िनिलखत म स िकस कार का वथापन पटन निदयो क सगम पर पाया जाता ह

1 Triangular Paern िकोणीय पटन

2 Circular or Semi-Circular Paern परप या अध-परप पटन

3 Nebular Paern नबलर पटन

4 Star ndashShaped Paern ार-आकार का पटन

Correct Answer -

Triangular Paern िकोणीय पटन

Which one was not the objective of the Biosphere Reserve Projects launched by the UNESCO

यनो ारा श की गई सरित जवमडल परयोजनाओ का उ इनम स कौन सा नही था

1 To promote teaching and research िशण और अनसधान को बढ़ावा दना

2 To make agriculture sustainable किष को दीघकािलक बनाना

3 To conserve ecosystems पारथितक त को सरित करना

4 To conserve genetic diversity for a longtime लब समय तक अनवािशक िविवधता को सरित करना

Correct Answer -

To make agriculture sustainable किष को दीघकािलक बनाना

Which region of the Earth surface receives the highest amount of insulation

पी सतह का कौन सा तापावरोधन की उतम माा ा करता ह

1 Land mass थलखड

2 Savannah region सवाना

3 Water bodies जल िनकाय

4 Tropical desert उकिटबधीय रिगान

Correct Answer -

Tropical desert उकिटबधीय रिगान

Which one of the following is not a biodiversity hotspot

िनिलखत म स कौन सा जव िविवधता का म जगह नही ह

1 Eastern Himalaya पव िहमालय

2 Eastern Ghats पव घाट

81)

82)

83)

3 Indo-Myanmar भारत-ामार

4 Westerm Ghats पिमी घाट

Correct Answer -

Eastern Ghats पव घाट

Which one of the following is NOT a part of the World Network of Biosphere Reserves based on theUNESCO Man and Biosphere Programme

यनो मन और बायोीयर कायम क आधार पर िनिलखत म स कौन बायोीयर रजव क िव नटवक कािहा नही ह

1 Gulf of Mannar मार की खाड़ी

2 Seshachalam शषाचलम

3 Sunderban सदरबन

4 Nilgiri नीलिगर

Correct Answer -

Seshachalam शषाचलम

Which one of the following is an example of ldquodesert vegetationrdquo

िनिलखत म स कौन मथलीय वनित का एक उदाहरण ह

1 Mosses and lichens दलदल और शवाल

2 Temperate grassland समशीतो घास क मदान

3 Coniferous forest शकधारी वन

4 Acacia and cactus एकािसया और कस

Correct Answer -

Acacia and cactus एकािसया और कस

Which one of the following reflects more sunlight िनिलखत म स कौन सा सय की रोशनी को अिधकपरावितत करता ह

1 Paddy crop land धान फसल भिम

2 Land covered with fresh snow ताजा बफ स आािदत भिम

3 Sand desert रतीली रिगान

4 Prairie land यरी भिम

84)

85)

86)

87)

Correct Answer -

Land covered with fresh snow ताजा बफ स आािदत भिम

Which layer of the atmosphere is in contact with the surface of the earthrsquos oceans

वायमडल की कौन सी परत पी क महासागरो की सतह क सपक म ह

1 Stratosphere समताप मडल

2 Mesosphere म मडल

3 Hydrosphere जलमडल

4 Troposphere ोभ मडल

Correct Answer -

Troposphere ोभ मडल

Mediterranean Sea is a border of which of the following countries भम सागर िनिलखत दशो म सिकसकी सीमा ह

1 None of these इनम स कोई नही

2 Iraq इराक

3 Lebanon लबनान

4 Jordan जॉडन

Correct Answer -

Lebanon लबनान

Benguela ocean currents are found along which coast बगएला महासागर धाराए िकस तट क साथ पायीजाती ह

1 East Coast of South America दिण अमरका क पव तट

2 East Coast of Africa अीका क पव तट

3 West Coast of South America दिण अमरका क पिमी तट

4 West Coast of Africa अीका क पिमी तट

Correct Answer -

West Coast of Africa अीका क पिमी तट

88)

89)

90)

Due to tension a block of land on one side being pushed up or upthrown relative to the downthrown blockis referred as

तनाव क कारण नीच फ क ए खड क साप भिम का एक खड एक ओर स ऊपर धकला जाता ह या ऊपर की ओरफ का जाता ह यह _____ क प म सदिभत ह

1 Thrust fault प श

2 Normal fault सामा श

3 Reverse fault म श

4 Strike slip fault नितलब सपण श

Correct Answer -

Normal fault सामा श

Inter-tropical doldrums is a zone of ______ अतर-उकिटबधीय डोलड ______ का एक ह

1 Frontolysis टोलायिसस

2 Convergence अिभसरण

3 Inter-tropical divergence zone अतर-उकिटबधीय िवचलन

4 Local wind थानीय वाय

Correct Answer -

Convergence अिभसरण

The Horse Latitudes are regions located at about _____ north and south of the equator

हॉस अाश भम रखा क उर और दिण म लगभग _____ पर थत ह

1 30ndash60 degree Latitude 30-60 िडी अाश

2 0ndash5 degree Latitude 0-5 िडी अाश

3 30 degree Latitude 30 िडी अाश

4 60ndash90 degree Latitude 60-90 िडी अाश

Correct Answer -

30 degree Latitude 30 िडी अाश

Generally evaporation is high over which part of the Earth

आम तौर पर पी क िकस भाग पर वाीकरण अिधक होता ह

1 Equatorial maritime भमवत समीय ी ी

91)

92)

2 Equatorial continental भमवत महाीपीय

3 Polar maritime वीय समीय

4 Polar continental वीय महाीपीय

Correct Answer -

Equatorial maritime भमवत समीय

A very high temperature during summer in north western India leads to what type of climaticcondition in south

उर पिमी भारत म गम क दौरान बत अिधक तापमान होन क कारण दिण म िकस कार की जलवाय थितउ करता ह

1 Depression over arabian sea अरब सागर पर अवनमन

2 Failure monsoon मानसन िवफलता

3 Successful monsoon मानसन सफलता

4 Cyclones चवात

Correct Answer -

Successful monsoon मानसन सफलता

Lightning and thunder are the resultant effect when तिड़त और गजन परणामी भाव ह जब

1 Two massive clouds hit powerfully each other first lightning is produced and later sound is produced

दो बड़ बादल एक दसर स शशाली ढग स टकरात ह पहल आकाशीय िवदयत उ होता ह और बाद म िन उहोती ह

2 Two massive clouds come into contact with the powerful wind collision this results into first sound and thenlightning

दो बड़ बादल शशाली पवन सघ क सपक म आत ह इसका परणामप पहल िन और िफर आकाशीय िवदयतउ होता ह

3 None of the above उपरो म स कोई भी नही

4 A high density cloud contains positively and negatively charged electric ions and when this interacts light andsound are simultaneously produced

एक उ घन बादल म धनाक और ऋणाक आविशत िवदयत आयन होत ह और जब यह परर भाव डालत ह तोकाश और िन एक साथ उािदत होती ह

Correct Answer -

A high density cloud contains positively and negatively charged electric ions and when this interacts light andsound are simultaneously produced

औ ि ि ो औ ो

93)

94)

95)

एक उ घन बादल म धनाक और ऋणाक आविशत िवदयत आयन होत ह और जब यह परर भाव डालत ह तोकाश और िन एक साथ उािदत होती ह

Doon Valley is able to grow rice because दन घाटी चावल उगान म सम ह ोिक

1 Other crops cannot be grown वहा अ फसलो को उगाया नही जा सकता ह

2 People in the valley are rice eaters घाटी म लोग चावल खान वाल ह

3 There is a huge export demand of rice वहा चावल की भारी िनयात माग ह

4 It has warm summer and snow melt waters for irrigation

वहा गिमया गम होती ह िसचाई क िलए बफ का िपघला आ पानी होता ह

Correct Answer -

It has warm summer and snow melt waters for irrigation

वहा गिमया गम होती ह िसचाई क िलए बफ का िपघला आ पानी होता ह

CANCELLED

In the geological time scale the Mesozoic Era DOES NOT contains which of the following periods

भगभय समय पमान पर मजीवी यग म िन कालो म स कौन नही ह

1 Triassic ट ाइऐिसक

2 Jurassic जरिसक

3 Cretaceous चाकमय

4 Carboniferous काबनी

Correct Answer -

Carboniferous काबनी

96)

1 P-3 Q-4 R-2 S-1

2 P-3 Q-4 R-1 S-2

3 P-3 Q-4 R-1 S-2

4 P-4 Q-3 R-2 S-1

Correct Answer -

P-4 Q-3 R-2 S-1

1 P-3 Q-1 R-4 S-2

2 P-3 Q-4 R-1 S-2

3 P-3 Q-2 R-4 S-1

97)

98)

4 P-2 Q-1 R-4 S-3

Correct Answer -

P-3 Q-4 R-1 S-2

1 P-3 Q-1 R-4 S-2

2 P-2 Q-3 R-4 S-1

3 P-2 Q-1 R-3 S-4

4 P-4 Q-2 R-1 S-3

Correct Answer -

P-3 Q-1 R-4 S-2

99)

100)

1 P-3 Q-2 R-4 S-1

2 P-1 Q-2 R-3 S-4

3 P-2 Q-3 R-1 S-4

4 P-4 Q-3 R-2 S-1

Correct Answer -

P-2 Q-3 R-1 S-4

ldquoHuman geography is the study of changing relationship between the unresting man and the unstableearthrdquo was defined by

lsquolsquoमानव भगोल ाकल आदमी और अथर पी क बीच सबध परवतन का अयन हrdquo ______ ारा परभािषत िकया गयाथा

1 J Brunches ज चस

2 EC Semple ईसी सल

3 HJ Mackinder एच ज मिकदर

4 PV Blache पीवी च

Correct Answer -

EC Semple ईसी सल

Sedimentary rocks are finally and ultimately derived from the____________

अवसादी चान अततः ________ स ा की जाती ह

1 action of earth movements पी की गितिविधयो

2 marine deposit समी िनप

3 weathering of metamorphic rocks पातरत चानो क अपय

4 weathering of igneous rocks आय चानो क अपय

Correct Answer -

weathering of igneous rocks आय चानो क अपय

Page 14: High School Teacher Eligibility Test- BOARD PROFESSIONAL ...peb.mp.gov.in/results/RESULT_18/HST_RES18/Final_anwser_key/HST… · M a ndl a / मंड ल ... Under the Madhya Pradesh

7)

8)

9)

2 the task becoming mechanically repetitive काय यािक प स दोहराव वाला ह

3 the child not being intelligent बा बमान नही ह

4 the child being incapable of learning बा सीखन म असमथ ह

Correct Answer -

the task becoming mechanically repetitive काय यािक प स दोहराव वाला ह

Educational Psychologists are more concerned with the learning in __________

शिणक मनोवािनक __________ म अिधगम क साथ अिधक िचितत होत ह

1 Formal environment औपचारक वातावरण

2 Informal environment अनौपचारक वातावरण

3 Physical environment भौितक वातावरण

4 Social environment सामािजक वातावरण

Correct Answer -

Formal environment औपचारक वातावरण

Special needs education is the type of education िवशष ज़रतो वाली िशा वह िशा होती ह जो

1 Given to person with disabilities अम य को दी जाती ह

2 Given to people from remote areas दर थ ो क लोगो को दी जाती ह

3 Provided to intelligent people बमान लोगो को दी जाती ह

4 Established by colonial masters औपिनविशक मखयाओ ारा थािपत की गई ह

Correct Answer -

Given to person with disabilities अम य को दी जाती ह

The Stanford-Binet scale of intelligence was first published in the year

ब क नफोड-िबनट कल को इस वष म पहली बार कािशत िकया गया था

1 1916

2 1903

3 1908

4 1900

Correct Answer -

1916

10)

11)

12)

13)

The term lsquofictional finalismrsquo was propounded by

पद lsquoकत योजनवादrsquo (िफ शनल फाइनिल म) इनक ारा ितपािदत िकया गया था

1 Skinner नर

2 Freud ायड

3 Adler एडलर

4 Pavlov पावलोव

Correct Answer -

Adler एडलर

Rational Emotive Behavior Therapy was propounded by

तक सगत भावनाक वहार थरपी िन क ारा ितपािदत की गई थी

1 Carl Jung काल यग

2 Carl Rogers काल रोजस

3 Aaron Beck आरोन बक

4 Albert Ellis अट एिलस

Correct Answer -

Albert Ellis अट एिलस

Which of the following indicates the quality of education in a school

िनिलखत म स या एक कल म िशा की गणव ता का सकतक ह

1 Text-books and Teaching-learning material पा-प तक तथा िशण व सीखन स सबिधत सामी

2 Infrastructural facilities at the school कल म आधारभत ढाच स सबिधत सिवधाए

3 Student achievement level िवािथयो का उपल तर

4 Classroom systems का की यव था

Correct Answer -

Student achievement level िवािथयो का उपल तर

Which of the following teachers can be identified with authoritarian teaching style

िनिलखत म स कौन सा िशक अिधकारवादी िशण शली क साथ पहचाना जा सकता ह

1 Laissez-faire teacher अब ध िशक

14)

15)

2 Democratic teacher लोकतीय िशक

3 Indifferent teacher िन प िशक

4 Direct instruction teacher िनदश िशक

Correct Answer -

Direct instruction teacher िनदश िशक

Who was the pioneer of classical conditioning

िचरितित ानकलन ( लािसकल कडीशिनग) क वतक कौन थ

1 Skinner नर

2 Pavlov पावलोव

3 Kohler कोहलर

4 Freud ायड

Correct Answer -

Pavlov पावलोव

Which of the following are true with reference to short term memory

1 Limited capacity

2 Brief storage of information

3 Unlimited capacity

4 Duration of storage less than twenty seconds

अ पकािलक मित क सदभ म िन न म स या स य ह

1 सीिमत मता

2 सचना का सि त भडारण

3 असीिमत मता

4 बीस सकड स कम भडारण की अविध

1 3 and 4 3 और 4

2 2 and 4 2 और 4

3 13 and 4 13 और 4

4 12 and 4 12 और 4

Correct Answer -

12 and 4 12 और 4

1)

2)

3)

4)

Topic- GEOGRAPHY

Which of the following ranges of population size is used to define Class-III city by Indian Census

भारतीय जनगणना ारा वग-III शहर को परभािषत करन क िलए िनिलखत म स िकस रज क जनसा आकारका उपयोग िकया जाता ह

1 20 000 to 49999 20 000 स 49999

2 30000 to 59999 30000 स 59999

3 24000 to 54999 24000 स 54999

4 50000 to 99999 50000 स 99999

Correct Answer -

20 000 to 49999 20 000 स 49999

Carbonaceous rocks which produce coal and oil belong to the category of rocks called_______

कोयल और तल का उादन करन वाली काबनय चान ______ नामक चानो की णी स सबिधत ह

1 metamorphic पातरत

2 sedimentary अवसादी

3 inorganic अजिवक

4 igneous आय

Correct Answer -

sedimentary अवसादी

The ruhr-complex is a major industrial centre in र-परसर िन का एक मख औोिगक क ह

1 North America उरी अमरका

2 Russia स

3 Germany जमनी

4 Europe यरोप

Correct Answer -

Germany जमनी

The term lsquoRegurrsquo refers to श lsquoरगरrsquo ______ स सबिधत ह

1 Deltaic alluvial soils डा जलोढ़ िमी

ि ी

5)

6)

2 Laterite soils लटराइट िमी

3 Red and yellow soils लाल और पीली िमी

4 Black cotton soils काली कपास िमी

Correct Answer -

Black cotton soils काली कपास िमी

Read the given statements and answer which of the following options isare correct

1 Lower the pressure greater the atmospheric disturbance

2 Air move from higher to low pressure

िदए गए कथन को पढ़ और उर द िक िन म स कौन सास िवक सही ह

1 िजतना दाब कम होगा वायमडलीय बाधाए उतनी अिधक होगी

2 वाय उ स िन दाब की ओर गित करती ह

1 Both Statements 1 and 2 are correct दोनो कथन 1 और 2 सही ह

2 Both Statements 1 and 2 are wrong दोनो कथन 1 और 2 गलत ह

3 Statement 1 is wrong and only Statement 2 is correct कथन 1 गलत ह और कवल कथन 2 सही ह

4 Statement 1 is correct and Statement 2 is wrong कथन 1 सही ह और कथन 2 गलत ह

Correct Answer -

Both Statements 1 and 2 are correct दोनो कथन 1 और 2 सही ह

CANCELLED

1 4 1 2 and 3 4 1 2 और 3

2 4 1 3 and 2 4 1 3 और 2

3 1 4 2 and 3 1 4 2 और 3

4 1 4 3 and 2 1 4 3 और 2

Correct Answer -

7)

8)

9)

1 4 3 and 2 1 4 3 और 2

CANCELLED

Karl Pearsonrsquos correlation co-efficient is काल िपयसन का सहसबध गणाक ह

1 Arithmec mean समार मा

2 Geometric mean गणोर मा

3 None of these इनम स कोई नही

4 Harmonic mean हराक मा

Correct Answer -

Geometric mean गणोर मा

CANCELLED

ldquoEach day is more or less the same the morning is clear and bright with a sea breeze as the Sun climbshigh in the sky heat mounts up dark clouds form then rain comes with thunder lighting But rain is soonoverrdquo Which of the following regions is described in the above passage

ldquoक िदन समान स अिधक या कम होता ह सम की हवा क साथ और उल सबह होती ह जस सयआकाश म ऊचा चढ़ता ह गम बढ़ जाती ह काल बादल बनत ह िफर िबजली क साथ बारश आती ह लिकनबारश जी ख हो जाती हlsquorsquo उपरो पा म िनिलखत म स िकन ो का वणन िकया गया ह

1 Equatorial भमरखीय

2 Equatorial भमरखीय

3 Savannah सवाना

4 Mediterranean आातरक (भमसागरीय)

5 Mediterranean आातरक (भमसागरीय)

6 Monsoon मानसन

7 Monsoon मानसन

Correct Answer -

Equatorial भमरखीय

Equatorial भमरखीय

CANCELLED

In which epoch of the geological history of the Earth dinosaurs reached their largest size

पी क भगभय इितहास क िकस यग म डायनासोर अपन सबस बड़ आकार तक पचि

10)

11)

1 Triassic ट ायिसक

2 Jurassic जरािसक

3 Cretaceous ीटशस

4 Permian पिमयन

Correct Answer -

Jurassic जरािसक

CANCELLED

A spring tide would occur in which of the following conditions

िनिलखत म स िकन थितयो म ार-भाटा आता ह

1 When the Sun Moon and Earth are in a straight line

जब सय चमा और पी एक सीधी रखा म होत ह

2 When the Moon and Earth are in right angle to each other

जब चमा और पी एक-दसर क दािहन कोण म होत ह

3 When the Earth and Moon are in right angle to the Sun

जब पी और चमा सय क दािहन कोण म होत ह

4 When the Sun and Moon are in right angle to each other

जब सय और चमा एक-दसर क दािहन कोण म होत ह

Correct Answer -

When the Sun Moon and Earth are in a straight line

जब सय चमा और पी एक सीधी रखा म होत ह

CANCELLED

An observe on the Earthrsquos surface always sees the same face of the moon because

एक पयवक को पी की सतह स हमशा चाद का एक ही फलक िदखाई दता ह ोिक

1 Its path of revolution around the earth is the same as that of the earth around the Sun

इसका पी क चारो ओर परमण का माग सय क चारो ओर पी क समान ही ह

2 Its period of revolution around the Earth is the same as its period of rotation around its own axis

इसकी पी क चारो ओर परमण की अविध उसकी अपनी धरी क चारो ओर घणन की अविध क समान ह

3 Its period of rotation is the same as that of the Earth इसकी घणन की अविध पी क समान ह

ी ि ी ी

12)

13)

14)

4 Its direct of rotation is the same as that of Earth घणन की िदशा पी क समान ही ह

Correct Answer -

Its period of revolution around the Earth is the same as its period of rotation around its own axis

इसकी पी क चारो ओर परमण की अविध उसकी अपनी धरी क चारो ओर घणन की अविध क समान ह

CANCELLED

The pebbles that are faceted by the sand-blasting and shaped polished by the wind abrasions are known as

पवन अपघषन ारा पॉिलश रत-िवोिटत और साच म ढला ककड़ ___________ क प म जाना जाता ह

1 Dreikanter िकोणक

2 Pediments िकोिनका

3 Inselberg इलबग

4 Dunes टीबा

Correct Answer -

Dreikanter िकोणक

CANCELLED

Astronomical unit is the average distance between खगोलीय इकाई ______ क बीच की औसत दरी ह

1 Earth and Mars पी और मगल

2 Earth and mercury पी और बध

3 Earth and moon पी और चमा

4 Earth and Sun पी और सय

Correct Answer -

Earth and Sun पी और सय

During cold weather season in the northern plains there will be an inflow of cyclonic disturbancesfrom the _________ directions

शीत मौसम क दौरान उरी मदानी इलाको म _________ िदशाओ स चवात सबधी गड़बड़ी का अतवाह होगा

1 East and Northwest पव और उरपिम

2 East and Northeast पव और पवर

3 West and East पिम और पव

ि औ ि

15)

16)

17)

4 West and Northwest पिम और उरपिम

Correct Answer -

West and Northwest पिम और उरपिम

During an earth quake the velocity of the body waves will________ along with the increase in densityof the material it is passing through

भकप क दौरान लहरो क ऊपरी भाग का वग घन म व क साथ-साथ ________ जो इसस गजरन वाली वको आग बढाएगी

1 not change नही बदलगा

2 increase initially and then decrease शआत म बढ़गा और िफर घटगा

3 increase बढ़गा

4 decrease घटगा

Correct Answer -

increase बढ़गा

The Clouded Leopard National park is situated in which of the following states

िनिलखत म िकस रा म धिमल तदआ रा ीय उान (ाउडड लपड नशनल पाक ) थत ह

1 Tripura िपरा

2 Uttar Pradesh उर दश

3 Assam असम

4 Mizoram िमजोरम

Correct Answer -

Tripura िपरा

Usually the land surfaces are heated more quickly than the water surfaces because _____________

आम तौर पर जल सतहो की तलना म भिम सतह अिधक तजी स गम होती ह ोिक _____________ ह

1 the specific heat of water is higher than land पानी की िविश ऊा भिम स अिधक

2 the specific heat of water is lesser than land पानी की िविश ऊा भिम स कम होती

3 the latent heat of water is higher than the land पानी की अतिनिहत ऊा भिम स अिधक

4 the land reflects more heat radiation than water भिम पानी की तलना म अिधक ऊा क िविकरण को पराविततकरती

18)

19)

20)

21)

Correct Answer -

the specific heat of water is higher than land पानी की िविश ऊा भिम स अिधक

The longest shore-line is along the state of सबस लबी समतटीय रखा िन रा क साथ ह

1 Maharashtra महारा

2 Orissa उड़ीसा

3 Kerala करल

4 Gujarat गजरात

Correct Answer -

Gujarat गजरात

The position when the Earth is farthest from the Sun is known as

जब पी सय स सबस दर होती ह तो उस थित को िन नाम स जाना जाता ह

1 Perihelion उपसौर

2 Vernal Equinox बसत िवषव

3 Aphelion अपसौर

4 Autumnal Equinox शराल िवषव

Correct Answer -

Aphelion अपसौर

The seasonal reversal of winds is the typical characteristic of

हवाओ का मौसमी परवतन ______ की सामा िवशषता ह

1 Mediterranean climates only कवल भमसागरीय जलवाय

2 All of the above climates उपय सभी मौसम

3 Monsoon climate only कवल मानसन जलवाय

4 Equatorial climate only कवल भमरखीय जलवाय

Correct Answer -

Monsoon climate only कवल मानसन जलवाय

In _________ rocks the minerals will occurs in beds or layers

ो ि ो ो

22)

23)

24)

______ चानो म खिनज तल या परतो म होत ह

1 metamorphic कायातरत

2 igneous and metamorphic आय और कायातरत

3 igneous आय

4 sedimentary अवसादी

Correct Answer -

sedimentary अवसादी

Black soil is ideal for the cultivation of cotton as कपास की खती क िलए काली िमी आदश ह ोिक

1 Its colour is black यह काली होती ह

2 It is found on plateau regions यह पठार ो म पायी जाती ह

3 It is made up of lava यह लावा स बनी होती ह

4 It can retain moisture यह नमी को बरकरार रख सकती ह

Correct Answer -

It can retain moisture यह नमी को बरकरार रख सकती ह

The National Survey and Mapping Organization of the country works under the Department of___________

दश का रा ीय सवण और मानिचण सगठन ___________ िवभाग क अतगत काय करता ह

1 Space अतर

2 Science and Technology िवान और तकनीक

3 Culture सित

4 Tourism पयटन

Correct Answer -

Science and Technology िवान और तकनीक

Palk strait separates India from पाक जलडमम भारत स _____ को अलग करता ह

1 Pakistan पािकान

2 Andaman Island अडमान ीप

3 China चीन

25)

26)

27)

4 Sri Lanka ीलका

Correct Answer -

Sri Lanka ीलका

Which among the following state is the major producer of Bauxite in India

िनिलखत म स कौन सा रा भारत म बॉाइट का मख उादक ह

1 Madhya Pradesh मदश

2 Rajasthan राजथान

3 Goa गोवा

4 Orissa उड़ीसा

Correct Answer -

Orissa उड़ीसा

Which of the following states DOES NOT share border with Chhattisgarh

िनिलखत म स कौन सा रा छीसगढ़ क साथ सीमा साझा नही करता ह

1 Telangana तलगाना

2 Uttar Pradesh उर दश

3 Bihar िबहार

4 Andhra Pradesh आ दश

Correct Answer -

Bihar िबहार

Which of the following statements is INCORRECT with respect to parallels of latitudes

अाश क समानातरो क सबध म िन निलखत म स कौन सा कथन गलत ह

1 A line joining places of equal latitude is known as parallel of largest

समान अाश क थानो को जोड़न वाली रखा को िवशालतम क समानातर क प म जाना जाता ह

2 They stat from equator and run parallels to it

व भम रखा स ारभ होत ह और इसक समानातर चलत ह

3 All parallels are equal in length सभी समातर लबाई म समान ह

4 All parallels are drawn as circles on the globe ोब पर सभी समानातर वो क प म खीच जात ह

28)

29)

30)

31)

Correct Answer -

All parallels are equal in length सभी समातर लबाई म समान ह

Which of the following Indian states is also known as a lsquoLand of Red river and Blue Hillsrsquo

िनिलखत म स िकस भारतीय रा को लाल नदी और नीली पहािड़यो की भिम क नाम स जाना जाता ह

1 Uttarkhand उराखड

2 Assam असम

3 Meghalaya मघालय

4 Arunachal Pradesh अणाचल दश

Correct Answer -

Assam असम

In spatial analysis of settlement Rn = 215 indicates which type of settlement arrangement

िनपटान क थािनक िवषण म Rn = 215 यह इिगत करता ह िक िकस कार की िनपटान वथा ह

1 Uniform यिनफॉम

2 Semi-Clustered समी- ल टर

3 Clustered ल टर

4 Random रडम

Correct Answer -

Uniform यिनफॉम

Who are known as the lsquoYellow Peoplersquo lsquoयलो पीपलrsquo क प म कौन जाना जाता ह

1 Mongoloids मोगोलोइडस

2 Nigroids नीोइडस

3 Australoids ऑ लॉइडस

4 Caucasoids कॉकसोइडस

Correct Answer -

Mongoloids मोगोलोइडस

ि ि ो ौ ि

32)

33)

34)

Who publishes the topographical map of India भारत क थलाकितक मानिच को कौन कािशत करता ह

1 Geographical Survey of India भारत का भौगोिलक सवण

2 Government of India भारत सरकार

3 Geological Survey of India भारत क भगभय सवण

4 Survey of India भारत का सवण

Correct Answer -

Survey of India भारत का सवण

Who among the following claimed geography to be the lsquoEcology of Manrsquo

िनिलखत म स िकसन भगोल को मन का पारथितकी कहा ह

1 Alfred Hener अड हटनर

2 Vidal-de la Blache वाइडल-िड लॉ ॉश

3 Oo Schluter ओटो टर

4 Harlan Barrow हरलन बारो

Correct Answer -

Harlan Barrow हरलन बारो

Who among the following is regarded as the founder of humanistic approach in geography

िनिलखत म स िकस भगोल म मानवतावादी िकोण का सथापक माना जाता ह

1 William Bunge िविलयम बग

2 Yi-Fu-Tuan यी-फ- यान

3 Brain JL Berry न जएल बरी

4 Richard Peet रचड पीट

Correct Answer -

Yi-Fu-Tuan यी-फ- यान

Who prepared Lorenz curve लोरज व िकसन तयार िकया

1 Geddes गडस

2 None of these इनम स कोई नही

3 Griffith Taylor ििफथ टलर

35)

36)

37)

4 Max U Lorenz म य लोरज

Correct Answer -

Max U Lorenz म य लोरज

Gulf Streams are the currents of which of the following oceans

खाड़ी की धाराए िनिलखत महासागरो म स िकसकी धाराए ह

1 North Atlantic Ocean उरी अटलािटक महासागर

2 North Pacific Ocean उरी शात महासागर

3 Arabian Sea अरब सागर

4 South Pacific Ocean दिण शात महासागर

Correct Answer -

North Atlantic Ocean उरी अटलािटक महासागर

Disintegration wearing away and removal of rock material is generally referred as

िशला पदाथ (रॉक सामी) का टटना िमटना और हटना आमतौर पर ________ क प म सदिभत िकया जाता ह

1 Shattering िवसकारक

2 Denudation अनाादन

3 Fault श

4 Decomposition िवयोजन

Correct Answer -

Denudation अनाादन

Variations in the length of day time and night from season to season are due to

मौसम स मौसम परवतन पर िदन क समय और रात क समय की अविध म िभताए िन कारण स होती ह

1 The Earthrsquos revolution round the Sun in an elliptical manner पी का दीघवाकार तरीक स सय क चारो घणन

2 The Earthrsquos rotation on its axis पी का इसकी धरी पर घणन

3 Revolution of the Earth on a tilted axis नत अ पर पी का घणन

4 Latitudinal position of the place थान की अाश थित

Correct Answer -

Revolution of the Earth on a tilted axis नत अ पर पी का घणन

38)

39)

40)

Point out the correct sequence of mountain ranges from north to south

उर स दिण तक पवत खलाओ क सही अनम को इिगत कर

1 Great Himalaya Middle Himalaya Outer Himalaya Trans Himalaya

महान िहमालय म िहमालय बा िहमालय परा िहमालय

2 Middle Himalaya Great Himalaya Trans Himalaya Outer Himalaya

म िहमालय महान िहमालय परा िहमालय बा िहमालय

3 Outer Himalaya Middle Himalaya Great Himalaya Trans Himalaya

बा िहमालय म िहमालय महान िहमालय परा िहमालय

4 Trans Himalaya Great Himalaya Middle Himalaya Outer Himalaya

परा िहमालय महान िहमालय म िहमालय बा िहमालय

Correct Answer -

Trans Himalaya Great Himalaya Middle Himalaya Outer Himalaya

परा िहमालय महान िहमालय म िहमालय बा िहमालय

Sunrsquos halo is produced by the refraction of light in सय का भामडल ______ म काश क अपवतन ाराउ होता ह

1 Ice crystals in Cirrro-Cumulus clouds पाभ-कपास मघो क बफ िल

2 Ice crystal in Cirrus clouds पाभमघो क बफ िल

3 Dust particles in Stratus clouds री मघो क धल कण

4 Water vapour in Stratus clouds री मघो क जल वा

Correct Answer -

Ice crystal in Cirrus clouds पाभमघो क बफ िल

Read the given statements and answer which of the following options isare correct

(1) The minerals present in the rocks exposed to atmosphere are not subjected to alteration

(2) Oxidation is one of the processes of chemical weathering

िदए गए कथनो को पढ़ और उर द िक िन म स कौन सास िवक सही ह

(1) वायमल स अनावत शल म उपथत खिनज परवतन क अधीन नही होता ह

(2) ऑीकरण रासायिनक अपय की ियाओ म स एक ह

ो ो

41)

42)

1 Both statements are wrong दोनो कथन गलत ह

2 Both statements are correct दोनो कथन सही ह

3 First statement is wrong and second statement is correct पहला कथन गलत ह और दसरा कथन सही ह

4 First statement is correct and second statement is wrong पहला कथन सही ह और दसरा कथन गलत ह

Correct Answer -

First statement is wrong and second statement is correct पहला कथन गलत ह और दसरा कथन सही ह

Read the given statements and answer which of the following options isare correct

1 Sunrsquos short waves enter the earth partially heating the atmosphere

2 Heated earth surface from the sun produces broader waves which interacts and heats the atmosphere

िदए गए कथन को पढ़ और उर द िक िन म स कौन सास िवक सही ह

1 सय की छोटी तरग पी म आिशक प स वश करती ह और वायमडल को ऊत करती ह

2 सय स ऊत पी की सतह िवारत तरग उ करती ह जो परर भाव डालती ह और वायमडल कोऊत करती ह

1 Both Statements 1 and 2 are correct दोनो कथन 1 और 2 सही ह

2 Both Statements 1 and 2 are wrong दोनो कथन 1 और 2 गलत ह

3 Statement 1 is wrong and only Statement 2 is correct कथन 1 गलत ह और कवल कथन 2 सही ह

4 Only statement 1 is correct कवल कथन 1 सही ह

Correct Answer -

Both Statements 1 and 2 are correct दोनो कथन 1 और 2 सही ह

Read the given statements and answer which of the following options isare correct

(1)The rocks that get changed due to heat and pressure are termed as metamorphic rocks

(2)Slate is one such type of metamorphic rock

िदए गए कथनो को पढ़ और उर द िक िन म स कौन सास िवक सही ह

(1) शल जो ऊा और दाब क कारण परवितत हो जात ह उ कायातरक शलो क प म जाना जाता ह

(2) ट एक तरह का कायातरक शल ह

1 Both statements are wrong दोनो कथन गलत ह

2 Both statements are correct दोनो कथन सही ह

3 First statement is wrong and second statement is correct पहला कथन गलत ह और दसरा कथन सही ह

ी औ

43)

44)

4 First statement is correct and second statement is wrong पहला कथन सही ह और दसरा कथन गलत ह

Correct Answer -

Both statements are correct दोनो कथन सही ह

Read the given statements and answer which of the following options isare correct

1 Higher temperature anomaly is observed in the northern hemisphere

2 Differential heating is absent in Northern Hemisphere

िदए गए कथनो को पढ़ और उर द िक िन म स कौन सास िवक सही ह

1 उरी गोलाध म उ तापमान िवसगित पायी जाती ह

2 उरी गोलाध म अतर ऊन अनपथत होती ह

1 Both Statements 1 and 2 are correct दोनो कथन 1 और 2 सही ह

2 Both Statements 1 and 2 are wrong दोनो कथन 1 और 2 गलत ह

3 Statement 1 is wrong and Statement 2 is correct कथन 1 गलत ह और कथन 2 सही ह

4 Statement 1 is correct and Statement 2 is wrong कथन 1 सही ह और कथन 2 गलत ह

Correct Answer -

Statement 1 is correct and Statement 2 is wrong कथन 1 सही ह और कथन 2 गलत ह

Read the given statements and answer which of the following options isare correct

(1) Plutonic rocks are intrusive type of igneous rocks

(2) It cools very slowly because the surrounding rock serves as insulation around the intrusion of magma

िदए गए कथनो को पढ़ और उर द िक िन म स कौन सास िवक सही ह

(1) िवतलीय शल अतवधी कार क आश शल ह

(2) यह बत धीर-धीर ठडा होता ह ोिक आस-पास क शल मा क अतवधन क चारो ओर रोधन क प म कायकरत ह

1 Both statements are wrong दोनो कथन गलत ह

2 Both statements are correct दोनो कथन सही ह

3 First statement is wrong and second statement is correct पहला कथन गलत ह और दसरा कथन सही ह

4 First statement is correct and second statement is wrong पहला कथन सही ह और दसरा कथन गलत ह

Correct Answer -

Both statements are correct दोनो कथन सही ह

45)

46)

47)

48)

The dust and ash material hurled from the volcanoes are termed as

ालामखी स िनकलन वाली धल और राख सामी को _______ क प म कहा जाता ह

1 Pyroclasc पाइरोाक

2 Hyperclastic हाइपराक

3 Hepiroclastic हिपरोाक

4 Cirroclastic िसरोाक

Correct Answer -

Pyroclasc पाइरोाक

The vertical difference in elevation between a low tide and high tide is referred as

कम ार और उ ार क बीच ऊचाई म लबवत अतर _____ स सदिभत होता ह

1 Tidal slope ारीय ढलान

2 Tidal elevation ारीय उयन

3 Tidal range ारीय परास

4 Tidal height ारीय ऊचाई

Correct Answer -

Tidal range ारीय परास

The maximum biodiversity is found in which of the following regions िनिलखत ो म स अिधकतमजव िविवधता िकसम पायी जाती ह

1 Amazon Basin अमज़न बिसन

2 East Indies ई इडीज

3 Congo Basin कागो बिसन

4 West indies व इडीज

Correct Answer -

Amazon Basin अमज़न बिसन

The cultivation of rice crop produces_______ चावल की फसल की खती ______ का उादन करती ह

1 SO2

49)

50)

51)

2 CH4

3 CFCs

4 CO2

Correct Answer -

CH4

The pressure system with higher pressure at the centre is called__________

क म उ दबाव वाली दबाव णाली को _______ कहा जाता ह

1 front अ

2 depression अवनमन

3 cyclone चवात

4 anti-cyclone ितचवात

Correct Answer -

anti-cyclone ितचवात

The Himalayan region is poor in mineral resources because िहमालयी खिनज ससाधनो म समनही ह ोिक

1 The displacement of rock strata has disturbed the arrangement of rocks and made it complex

शलीय परत क िवथापन न चानो की वथा को अवथत कर िदया ह और इस जिटल बना िदया ह

2 The climate conditions are not suitable for exploitation of minerals

जलवाय की थित खिनजो क दोहन क िलए उपय नही ह

3 The terrain makes explanation of minerals difficult and very costly due to transportation difficulties

भ-भाग परवहन की किठनाइयो क कारण खिनजो का दोहन मल और बत महगा बना दता ह

4 It is made up of crystalline rocks यह िलीय चानो स बना ह

Correct Answer -

The displacement of rock strata has disturbed the arrangement of rocks and made it complex

शलीय परत क िवथापन न चानो की वथा को अवथत कर िदया ह और इस जिटल बना िदया ह

The process through which the moisture is added to the atmosphere by vegetation is termed as

वह िया िजसक माम स वनित ारा वातावरण म नमी िमलायी जाती ह _______ क प म जानी जाती ह

52)

53)

54)

1 Condensation सघनन

2 Evapotranspiration वान-उजन

3 Radiation िविकरण

4 Precipitation वषण

Correct Answer -

Evapotranspiration वान-उजन

The process through which the terrestrial heat is transferred to air by direct contact is termed as

वह िया िजसम सपक ारा थलीय ऊा वाय म थानातरत हो जाती ह ______ क प म जानी जाती ह

1 Conduction चालन

2 Convection सवहन

3 Insolation आतपन

4 Radiation िविकरण

Correct Answer -

Conduction चालन

The largest area under mangroves is in which of the following statesunion territory

मोव क अतगत िनिलखत राोसघ शािसत दशो म स सबस बड़ा कौन सा ह

1 Andaman and Nicobar अमान और िनकोबार

2 Andhra Pradesh आ दश

3 West Bengal पिम बगाल

4 Gujarat गजरात

Correct Answer -

West Bengal पिम बगाल

The longitudinal transverse and surface waves in an earthquake originate from

भकप म दशातर अनथ और सतह तरग यहा उ होती ह

1 The focus on the surface of the Earth पी क सतह पर क -िबद म

2 The focus within the body of the Earth पी क भीतर क -िबद म

3 The epicenter within the body of the Earth पी क भीतर उपरक म

55)

56)

57)

4 The epicenter on the surface of the Earth पी क सतह पर उपरक म

Correct Answer -

The focus within the body of the Earth पी क भीतर क -िबद म

The down slope movement of material due to gravity is called______

गाकषण क कारण पदाथ की अनढाल गित को ______ कहा जाता ह

1 mass movement पदाथ सचलन

2 deposition िनप

3 erosion रण

4 volcanic movement ालामखीय सचलन

Correct Answer -

mass movement पदाथ सचलन

Shimla is cooler than Amritsar although both are on the same latitude This is because

िशमला म अमतसर स अिधक ठड ह हालािक दोनो समान अाश पर ह ऐसा ह ोिक

1 Shimla is at a greater height above sea level than Amritsar अमतसर की तलना म िशमला सम तल स अिधकऊचाई पर ह

2 Shimla is further north िशमला उर की ओर ह

3 Shimla is farther from the equator िशमला भम रखा स आग ह

4 Their longitudes differ उनकी दशातर रखाए िभ ह

Correct Answer -

Shimla is at a greater height above sea level than Amritsar अमतसर की तलना म िशमला सम तल स अिधकऊचाई पर ह

lsquoTempo of Urbanizationrsquo measures which of the following

lsquoशहरीकरण का टपोrsquo िनिलखत म स कौन सा उपाय ह

1 Speed of urbanizaon शहरीकरण की गित

2 None of the above इनम स कोई नही

3 Inequality of urbanizaon शहरीकरण की असमानता

4 Current level of urbanizaon शहरीकरण का वतमान र

Correct Answer -

58)

59)

60)

Speed of urbanizaon शहरीकरण की गित

Out of the following options choose the INCORRECT statement

िनिलखत िवको म स गलत कथन का चयन कर

1 The clear tracts in the equatorial region recover rapidly भम रखा म भभाग तजी स ठीक हो जात ह

2 The stable communities include a redwood forest a pine forest at high elevations

थर समदायो म एक रडवड वन उ ऊचाई पर एक दवदार वन शािमल ह

3 Any ecosystem moves towards maximum biomass and stability to survive

कोई भी पारथितकी त जीिवत रहन क िलए अिधकतम जवसहित और थरता की तरफ असर होता ह

4 Tropical rain forests near equator are stable ecosystems

भम रखा क पास उकिटबधीय वषा वन थर पारथितक त ह

Correct Answer -

The clear tracts in the equatorial region recover rapidly भम रखा म भभाग तजी स ठीक हो जात ह

Seasonal contrasts are maximum in मौसमी िवषमता अिधकतम ह

1 Mid latitudes म अाश म

2 Low attitudes िन अाश म

3 High latitudes उ अाश म

4 Subtropics उपोकिटबधीय म

Correct Answer -

Mid latitudes म अाश म

In India which type of forest among the following occupies the largest area

भारत म िनिलखत म स िकस कार क वन सबस बड़ा फल आािदत करत ह

1 Sub-tropical Dry Evergreen Forest उप उकिटबधीय श सदाबहार वन

2 Mountain Wet Temperate Forest पवतीय आ शीतो वन

3 Tropical Moist Deciduous Forest उकिटबधीय आ पणपाती वन

4 Tropical Wet Evergreen Forest उकिटबधीय आ सदाबहार वन

Correct Answer -

Tropical Moist Deciduous Forest उकिटबधीय आ पणपाती वन

61)

62)

63)

64)

What is the proportion of lsquoJuvenile Populationrsquo (0-14 years) in India as per 2011Census

2011 की जनगणना क अनसार भारत म जवनाइल पॉपलशन यानी िकशोर जनस या (0-14 वष) का अनपात ाह

1 3076 of total population कल जनस या का 3076

2 2764 of total population कल जनस या का 2764

3 2933 of total population कल जनस या का 2933

4 3354 of total population कल जनस या का 3354

Correct Answer -

3076 of total population कल जनस या का 3076

What is the Belfast famous for बलफा िकसक िलए मशर ह

1 Belt of cotton textile industry कपास व उोग क

2 Ship-building industry जहाज िनमाण उोग

3 Agricultural machinery किष उपकरण

4 Aero planes manufacturing वाययान िनमाण

Correct Answer -

Ship-building industry जहाज िनमाण उोग

What is the most important occupation in tropical monsoon lands

उकिटबधीय मॉनसन भिम म सबस महपण वसाय ा ह

1 Mining खनन

2 Cattle rearing मवशी पालन

3 Agriculture किष

4 Nomadic herding नोमािडक जड़ी-बिटया

Correct Answer -

Agriculture किष

What is the most important characteristics of the islands (Indian) located in the Arabian sea

अरब सागर म थत ीपो (भारतीय) की सबस महपण िवशषता ा ह

ी ो

65)

66)

67)

1 There are all of coral origins सभी कोरल मल क ह

2 There are all very small in size य सभी आकार म बत छोट ह

3 They have a very dry climate इनकी जलवाय बत श ह

4 They are extended parts of the mainland व महाीप क िवारत िह ह

Correct Answer -

There are all of coral origins सभी कोरल मल क ह

What do the basalt layers of the Deccan indicate डन की बसा परत ा इिगत करती ह

1 All of the above उपरो सभी

2 Huge volcanic eruptions in the distant past दरथ अतीत म िवशाल ालामखीय िवोट

3 The immense erosional activity of the rivers निदयो की िवशाल रण गितिविध

4 The influence of weathering मौसम का भाव

Correct Answer -

Huge volcanic eruptions in the distant past दरथ अतीत म िवशाल ालामखीय िवोट

In the structure of planet Earth below the mantle the core is mainly made up of_____

पी ह की सरचना म मटल क नीच कोर म प स______ स िनिमत होती ह

1 aluminium एमीिनयम

2 silicon िसिलकॉन

3 chromium ोिमयम

4 iron लोहा

Correct Answer -

iron लोहा

One of the major Mid Oceanic Ridge is found in मख म-महासागर चोिटयो म स एक ______ म पायाजाता ह

1 Mid Pacific Ocean म शात महासागर

2 Mid Atlantic Ocean म अटलािटक महासागर

3 Mid Indian Ocean म भारतीय महासागर

4 Mid Arctic Ocean म आक िटक महासागर

68)

69)

70)

71)

Correct Answer -

Mid Atlantic Ocean म अटलािटक महासागर

Magma that reaches the Earthrsquos surface and then solidifies is called________

मा जो पी की सतह तक पचती ह और िफर ठोस हो जाती ह ________कहलाती ह

1 quartz ाटज

2 lava लावा

3 granite नाइट

4 silicates िसिलकट

Correct Answer -

lava लावा

Isotherms are the lines of equal_______ समताप रखाए समान _______की रखाए होती ह

1 pressure दाब

2 temperature तापमान

3 rainfall वषा

4 height ऊचाई

Correct Answer -

temperature तापमान

Mark the correct sequence of passes in the Western Ghats from north to south

पिमी घाटो म उर स दिण तक दर क सही अनम को िचित कर

1 Thalghat Palghat Bhorghat थलगघाट पालघाट भोरघाट

2 Thalghat Bhorghat Palghat थलघाट भोरघाट पालघाट

3 Bhorghat Thalghat Palghat भोरघाट थलघाट पालघाट

4 Palghat Bhorghat Thalghat पालघाट भोरघाट थलघाट

Correct Answer -

Thalghat Bhorghat Palghat थलघाट भोरघाट पालघाट

Which of the following does not have influence over the climate in India

ि ि ि ी ी

72)

73)

िनिलखत म स िकसका भाव भारत की जलवाय पर नही पड़ता ह

1 Ocean currents सागर की लहर

2 Nearness to equator भम रखा स िनकटता

3 Monsoons मानसन

4 Presence of Indian ocean भारतीय महासागर की उपथित

Correct Answer -

Ocean currents सागर की लहर

Which of the following cloud types has the characteristics like vertical tall narrow and puffy

िनिलखत म स िकस कार क मघो म लबवत लबी सकीण और थलता जसी िवशषताए ह

1 Cumulonimbus तफानी मघ

2 Cumulus मघ पज

3 Cirrocumulus पाभ कपासी मघ

4 Nimbostratus वषारी मघ

Correct Answer -

Cumulus मघ पज

Which of the following statement is INCORRECT about Crude Birth Rate

िनिलखत स कौन सा कथन अशोिधत ज दर क बार म सही नही ह

1 It cannot be used for comparing fertility level between two countries with different population characteristics

इसका उपयोग िविभ जनसा िवशषताओ वाल दो दशो क बीच जनन र की तलना क िलए नही िकया जा सकता ह

2 It is a standardized measure of fertility

यह जनन मता का मानकीकत उपाय ह

3 It is effected by the age-sex composition of the population

यह आबादी की आय-िलग सरचना स भािवत होता ह

4 It is expressed per 1000 population in a given geographical unit

यह िकसी दी गई भौगोिलक इकाई म ित 1000 जनसा पर िकया जाता ह

Correct Answer -

It is a standardized measure of fertility

यह जनन मता का मानकीकत उपाय ह

74)

75)

76)

77)

Which of the following state in India experienced negative decadal growth rate during 2001 to 2011census

भारत म िनिलखत म स िकस रा म वष 2001 स 2011 की जनगणना क दौरान नकाराक िगरावट दर ई

1 Tripura िपरा

2 Nagaland नागालड

3 Haryana हरयाणा

4 Odisha ओिडसा

Correct Answer -

Nagaland नागालड

Which of the following is NOT a characteristic of peninsular rivers

िनिलखत म स कौन सी िवशषता ायीपीय निदयो म नही होती ह

1 Flow through shallow valleys उथल घािटयो क माम स वाह

2 Seasonal flow मौसमी वाह

3 Little erosional activity थोड़ी कटावदार गितिविध

4 Meandering tendency often shifting their beds घमावदार वि अर अपन तटो को थानातरत करना

Correct Answer -

Meandering tendency often shifting their beds घमावदार वि अर अपन तटो को थानातरत करना

Which of the following gases in the atmosphere absorbs heat from the Sunrsquos radiation and the Earthssurface

वायमडल म िनिलखत म स कौन सी गस सय क िविकरण और पी की सतह स ऊा को अवशोिषत करती ह

1 Neon िनयॉन

2 Carbon dioxide काबन डाइऑाइड

3 Argon आगन

4 Nitrogen नाइट ोजन

Correct Answer -

Carbon dioxide काबन डाइऑाइड

Which of the following kind of settlement pattern is found at the confluence of rivers

ि ि ि ि ो

78)

79)

80)

िनिलखत म स िकस कार का वथापन पटन निदयो क सगम पर पाया जाता ह

1 Triangular Paern िकोणीय पटन

2 Circular or Semi-Circular Paern परप या अध-परप पटन

3 Nebular Paern नबलर पटन

4 Star ndashShaped Paern ार-आकार का पटन

Correct Answer -

Triangular Paern िकोणीय पटन

Which one was not the objective of the Biosphere Reserve Projects launched by the UNESCO

यनो ारा श की गई सरित जवमडल परयोजनाओ का उ इनम स कौन सा नही था

1 To promote teaching and research िशण और अनसधान को बढ़ावा दना

2 To make agriculture sustainable किष को दीघकािलक बनाना

3 To conserve ecosystems पारथितक त को सरित करना

4 To conserve genetic diversity for a longtime लब समय तक अनवािशक िविवधता को सरित करना

Correct Answer -

To make agriculture sustainable किष को दीघकािलक बनाना

Which region of the Earth surface receives the highest amount of insulation

पी सतह का कौन सा तापावरोधन की उतम माा ा करता ह

1 Land mass थलखड

2 Savannah region सवाना

3 Water bodies जल िनकाय

4 Tropical desert उकिटबधीय रिगान

Correct Answer -

Tropical desert उकिटबधीय रिगान

Which one of the following is not a biodiversity hotspot

िनिलखत म स कौन सा जव िविवधता का म जगह नही ह

1 Eastern Himalaya पव िहमालय

2 Eastern Ghats पव घाट

81)

82)

83)

3 Indo-Myanmar भारत-ामार

4 Westerm Ghats पिमी घाट

Correct Answer -

Eastern Ghats पव घाट

Which one of the following is NOT a part of the World Network of Biosphere Reserves based on theUNESCO Man and Biosphere Programme

यनो मन और बायोीयर कायम क आधार पर िनिलखत म स कौन बायोीयर रजव क िव नटवक कािहा नही ह

1 Gulf of Mannar मार की खाड़ी

2 Seshachalam शषाचलम

3 Sunderban सदरबन

4 Nilgiri नीलिगर

Correct Answer -

Seshachalam शषाचलम

Which one of the following is an example of ldquodesert vegetationrdquo

िनिलखत म स कौन मथलीय वनित का एक उदाहरण ह

1 Mosses and lichens दलदल और शवाल

2 Temperate grassland समशीतो घास क मदान

3 Coniferous forest शकधारी वन

4 Acacia and cactus एकािसया और कस

Correct Answer -

Acacia and cactus एकािसया और कस

Which one of the following reflects more sunlight िनिलखत म स कौन सा सय की रोशनी को अिधकपरावितत करता ह

1 Paddy crop land धान फसल भिम

2 Land covered with fresh snow ताजा बफ स आािदत भिम

3 Sand desert रतीली रिगान

4 Prairie land यरी भिम

84)

85)

86)

87)

Correct Answer -

Land covered with fresh snow ताजा बफ स आािदत भिम

Which layer of the atmosphere is in contact with the surface of the earthrsquos oceans

वायमडल की कौन सी परत पी क महासागरो की सतह क सपक म ह

1 Stratosphere समताप मडल

2 Mesosphere म मडल

3 Hydrosphere जलमडल

4 Troposphere ोभ मडल

Correct Answer -

Troposphere ोभ मडल

Mediterranean Sea is a border of which of the following countries भम सागर िनिलखत दशो म सिकसकी सीमा ह

1 None of these इनम स कोई नही

2 Iraq इराक

3 Lebanon लबनान

4 Jordan जॉडन

Correct Answer -

Lebanon लबनान

Benguela ocean currents are found along which coast बगएला महासागर धाराए िकस तट क साथ पायीजाती ह

1 East Coast of South America दिण अमरका क पव तट

2 East Coast of Africa अीका क पव तट

3 West Coast of South America दिण अमरका क पिमी तट

4 West Coast of Africa अीका क पिमी तट

Correct Answer -

West Coast of Africa अीका क पिमी तट

88)

89)

90)

Due to tension a block of land on one side being pushed up or upthrown relative to the downthrown blockis referred as

तनाव क कारण नीच फ क ए खड क साप भिम का एक खड एक ओर स ऊपर धकला जाता ह या ऊपर की ओरफ का जाता ह यह _____ क प म सदिभत ह

1 Thrust fault प श

2 Normal fault सामा श

3 Reverse fault म श

4 Strike slip fault नितलब सपण श

Correct Answer -

Normal fault सामा श

Inter-tropical doldrums is a zone of ______ अतर-उकिटबधीय डोलड ______ का एक ह

1 Frontolysis टोलायिसस

2 Convergence अिभसरण

3 Inter-tropical divergence zone अतर-उकिटबधीय िवचलन

4 Local wind थानीय वाय

Correct Answer -

Convergence अिभसरण

The Horse Latitudes are regions located at about _____ north and south of the equator

हॉस अाश भम रखा क उर और दिण म लगभग _____ पर थत ह

1 30ndash60 degree Latitude 30-60 िडी अाश

2 0ndash5 degree Latitude 0-5 िडी अाश

3 30 degree Latitude 30 िडी अाश

4 60ndash90 degree Latitude 60-90 िडी अाश

Correct Answer -

30 degree Latitude 30 िडी अाश

Generally evaporation is high over which part of the Earth

आम तौर पर पी क िकस भाग पर वाीकरण अिधक होता ह

1 Equatorial maritime भमवत समीय ी ी

91)

92)

2 Equatorial continental भमवत महाीपीय

3 Polar maritime वीय समीय

4 Polar continental वीय महाीपीय

Correct Answer -

Equatorial maritime भमवत समीय

A very high temperature during summer in north western India leads to what type of climaticcondition in south

उर पिमी भारत म गम क दौरान बत अिधक तापमान होन क कारण दिण म िकस कार की जलवाय थितउ करता ह

1 Depression over arabian sea अरब सागर पर अवनमन

2 Failure monsoon मानसन िवफलता

3 Successful monsoon मानसन सफलता

4 Cyclones चवात

Correct Answer -

Successful monsoon मानसन सफलता

Lightning and thunder are the resultant effect when तिड़त और गजन परणामी भाव ह जब

1 Two massive clouds hit powerfully each other first lightning is produced and later sound is produced

दो बड़ बादल एक दसर स शशाली ढग स टकरात ह पहल आकाशीय िवदयत उ होता ह और बाद म िन उहोती ह

2 Two massive clouds come into contact with the powerful wind collision this results into first sound and thenlightning

दो बड़ बादल शशाली पवन सघ क सपक म आत ह इसका परणामप पहल िन और िफर आकाशीय िवदयतउ होता ह

3 None of the above उपरो म स कोई भी नही

4 A high density cloud contains positively and negatively charged electric ions and when this interacts light andsound are simultaneously produced

एक उ घन बादल म धनाक और ऋणाक आविशत िवदयत आयन होत ह और जब यह परर भाव डालत ह तोकाश और िन एक साथ उािदत होती ह

Correct Answer -

A high density cloud contains positively and negatively charged electric ions and when this interacts light andsound are simultaneously produced

औ ि ि ो औ ो

93)

94)

95)

एक उ घन बादल म धनाक और ऋणाक आविशत िवदयत आयन होत ह और जब यह परर भाव डालत ह तोकाश और िन एक साथ उािदत होती ह

Doon Valley is able to grow rice because दन घाटी चावल उगान म सम ह ोिक

1 Other crops cannot be grown वहा अ फसलो को उगाया नही जा सकता ह

2 People in the valley are rice eaters घाटी म लोग चावल खान वाल ह

3 There is a huge export demand of rice वहा चावल की भारी िनयात माग ह

4 It has warm summer and snow melt waters for irrigation

वहा गिमया गम होती ह िसचाई क िलए बफ का िपघला आ पानी होता ह

Correct Answer -

It has warm summer and snow melt waters for irrigation

वहा गिमया गम होती ह िसचाई क िलए बफ का िपघला आ पानी होता ह

CANCELLED

In the geological time scale the Mesozoic Era DOES NOT contains which of the following periods

भगभय समय पमान पर मजीवी यग म िन कालो म स कौन नही ह

1 Triassic ट ाइऐिसक

2 Jurassic जरिसक

3 Cretaceous चाकमय

4 Carboniferous काबनी

Correct Answer -

Carboniferous काबनी

96)

1 P-3 Q-4 R-2 S-1

2 P-3 Q-4 R-1 S-2

3 P-3 Q-4 R-1 S-2

4 P-4 Q-3 R-2 S-1

Correct Answer -

P-4 Q-3 R-2 S-1

1 P-3 Q-1 R-4 S-2

2 P-3 Q-4 R-1 S-2

3 P-3 Q-2 R-4 S-1

97)

98)

4 P-2 Q-1 R-4 S-3

Correct Answer -

P-3 Q-4 R-1 S-2

1 P-3 Q-1 R-4 S-2

2 P-2 Q-3 R-4 S-1

3 P-2 Q-1 R-3 S-4

4 P-4 Q-2 R-1 S-3

Correct Answer -

P-3 Q-1 R-4 S-2

99)

100)

1 P-3 Q-2 R-4 S-1

2 P-1 Q-2 R-3 S-4

3 P-2 Q-3 R-1 S-4

4 P-4 Q-3 R-2 S-1

Correct Answer -

P-2 Q-3 R-1 S-4

ldquoHuman geography is the study of changing relationship between the unresting man and the unstableearthrdquo was defined by

lsquolsquoमानव भगोल ाकल आदमी और अथर पी क बीच सबध परवतन का अयन हrdquo ______ ारा परभािषत िकया गयाथा

1 J Brunches ज चस

2 EC Semple ईसी सल

3 HJ Mackinder एच ज मिकदर

4 PV Blache पीवी च

Correct Answer -

EC Semple ईसी सल

Sedimentary rocks are finally and ultimately derived from the____________

अवसादी चान अततः ________ स ा की जाती ह

1 action of earth movements पी की गितिविधयो

2 marine deposit समी िनप

3 weathering of metamorphic rocks पातरत चानो क अपय

4 weathering of igneous rocks आय चानो क अपय

Correct Answer -

weathering of igneous rocks आय चानो क अपय

Page 15: High School Teacher Eligibility Test- BOARD PROFESSIONAL ...peb.mp.gov.in/results/RESULT_18/HST_RES18/Final_anwser_key/HST… · M a ndl a / मंड ल ... Under the Madhya Pradesh

10)

11)

12)

13)

The term lsquofictional finalismrsquo was propounded by

पद lsquoकत योजनवादrsquo (िफ शनल फाइनिल म) इनक ारा ितपािदत िकया गया था

1 Skinner नर

2 Freud ायड

3 Adler एडलर

4 Pavlov पावलोव

Correct Answer -

Adler एडलर

Rational Emotive Behavior Therapy was propounded by

तक सगत भावनाक वहार थरपी िन क ारा ितपािदत की गई थी

1 Carl Jung काल यग

2 Carl Rogers काल रोजस

3 Aaron Beck आरोन बक

4 Albert Ellis अट एिलस

Correct Answer -

Albert Ellis अट एिलस

Which of the following indicates the quality of education in a school

िनिलखत म स या एक कल म िशा की गणव ता का सकतक ह

1 Text-books and Teaching-learning material पा-प तक तथा िशण व सीखन स सबिधत सामी

2 Infrastructural facilities at the school कल म आधारभत ढाच स सबिधत सिवधाए

3 Student achievement level िवािथयो का उपल तर

4 Classroom systems का की यव था

Correct Answer -

Student achievement level िवािथयो का उपल तर

Which of the following teachers can be identified with authoritarian teaching style

िनिलखत म स कौन सा िशक अिधकारवादी िशण शली क साथ पहचाना जा सकता ह

1 Laissez-faire teacher अब ध िशक

14)

15)

2 Democratic teacher लोकतीय िशक

3 Indifferent teacher िन प िशक

4 Direct instruction teacher िनदश िशक

Correct Answer -

Direct instruction teacher िनदश िशक

Who was the pioneer of classical conditioning

िचरितित ानकलन ( लािसकल कडीशिनग) क वतक कौन थ

1 Skinner नर

2 Pavlov पावलोव

3 Kohler कोहलर

4 Freud ायड

Correct Answer -

Pavlov पावलोव

Which of the following are true with reference to short term memory

1 Limited capacity

2 Brief storage of information

3 Unlimited capacity

4 Duration of storage less than twenty seconds

अ पकािलक मित क सदभ म िन न म स या स य ह

1 सीिमत मता

2 सचना का सि त भडारण

3 असीिमत मता

4 बीस सकड स कम भडारण की अविध

1 3 and 4 3 और 4

2 2 and 4 2 और 4

3 13 and 4 13 और 4

4 12 and 4 12 और 4

Correct Answer -

12 and 4 12 और 4

1)

2)

3)

4)

Topic- GEOGRAPHY

Which of the following ranges of population size is used to define Class-III city by Indian Census

भारतीय जनगणना ारा वग-III शहर को परभािषत करन क िलए िनिलखत म स िकस रज क जनसा आकारका उपयोग िकया जाता ह

1 20 000 to 49999 20 000 स 49999

2 30000 to 59999 30000 स 59999

3 24000 to 54999 24000 स 54999

4 50000 to 99999 50000 स 99999

Correct Answer -

20 000 to 49999 20 000 स 49999

Carbonaceous rocks which produce coal and oil belong to the category of rocks called_______

कोयल और तल का उादन करन वाली काबनय चान ______ नामक चानो की णी स सबिधत ह

1 metamorphic पातरत

2 sedimentary अवसादी

3 inorganic अजिवक

4 igneous आय

Correct Answer -

sedimentary अवसादी

The ruhr-complex is a major industrial centre in र-परसर िन का एक मख औोिगक क ह

1 North America उरी अमरका

2 Russia स

3 Germany जमनी

4 Europe यरोप

Correct Answer -

Germany जमनी

The term lsquoRegurrsquo refers to श lsquoरगरrsquo ______ स सबिधत ह

1 Deltaic alluvial soils डा जलोढ़ िमी

ि ी

5)

6)

2 Laterite soils लटराइट िमी

3 Red and yellow soils लाल और पीली िमी

4 Black cotton soils काली कपास िमी

Correct Answer -

Black cotton soils काली कपास िमी

Read the given statements and answer which of the following options isare correct

1 Lower the pressure greater the atmospheric disturbance

2 Air move from higher to low pressure

िदए गए कथन को पढ़ और उर द िक िन म स कौन सास िवक सही ह

1 िजतना दाब कम होगा वायमडलीय बाधाए उतनी अिधक होगी

2 वाय उ स िन दाब की ओर गित करती ह

1 Both Statements 1 and 2 are correct दोनो कथन 1 और 2 सही ह

2 Both Statements 1 and 2 are wrong दोनो कथन 1 और 2 गलत ह

3 Statement 1 is wrong and only Statement 2 is correct कथन 1 गलत ह और कवल कथन 2 सही ह

4 Statement 1 is correct and Statement 2 is wrong कथन 1 सही ह और कथन 2 गलत ह

Correct Answer -

Both Statements 1 and 2 are correct दोनो कथन 1 और 2 सही ह

CANCELLED

1 4 1 2 and 3 4 1 2 और 3

2 4 1 3 and 2 4 1 3 और 2

3 1 4 2 and 3 1 4 2 और 3

4 1 4 3 and 2 1 4 3 और 2

Correct Answer -

7)

8)

9)

1 4 3 and 2 1 4 3 और 2

CANCELLED

Karl Pearsonrsquos correlation co-efficient is काल िपयसन का सहसबध गणाक ह

1 Arithmec mean समार मा

2 Geometric mean गणोर मा

3 None of these इनम स कोई नही

4 Harmonic mean हराक मा

Correct Answer -

Geometric mean गणोर मा

CANCELLED

ldquoEach day is more or less the same the morning is clear and bright with a sea breeze as the Sun climbshigh in the sky heat mounts up dark clouds form then rain comes with thunder lighting But rain is soonoverrdquo Which of the following regions is described in the above passage

ldquoक िदन समान स अिधक या कम होता ह सम की हवा क साथ और उल सबह होती ह जस सयआकाश म ऊचा चढ़ता ह गम बढ़ जाती ह काल बादल बनत ह िफर िबजली क साथ बारश आती ह लिकनबारश जी ख हो जाती हlsquorsquo उपरो पा म िनिलखत म स िकन ो का वणन िकया गया ह

1 Equatorial भमरखीय

2 Equatorial भमरखीय

3 Savannah सवाना

4 Mediterranean आातरक (भमसागरीय)

5 Mediterranean आातरक (भमसागरीय)

6 Monsoon मानसन

7 Monsoon मानसन

Correct Answer -

Equatorial भमरखीय

Equatorial भमरखीय

CANCELLED

In which epoch of the geological history of the Earth dinosaurs reached their largest size

पी क भगभय इितहास क िकस यग म डायनासोर अपन सबस बड़ आकार तक पचि

10)

11)

1 Triassic ट ायिसक

2 Jurassic जरािसक

3 Cretaceous ीटशस

4 Permian पिमयन

Correct Answer -

Jurassic जरािसक

CANCELLED

A spring tide would occur in which of the following conditions

िनिलखत म स िकन थितयो म ार-भाटा आता ह

1 When the Sun Moon and Earth are in a straight line

जब सय चमा और पी एक सीधी रखा म होत ह

2 When the Moon and Earth are in right angle to each other

जब चमा और पी एक-दसर क दािहन कोण म होत ह

3 When the Earth and Moon are in right angle to the Sun

जब पी और चमा सय क दािहन कोण म होत ह

4 When the Sun and Moon are in right angle to each other

जब सय और चमा एक-दसर क दािहन कोण म होत ह

Correct Answer -

When the Sun Moon and Earth are in a straight line

जब सय चमा और पी एक सीधी रखा म होत ह

CANCELLED

An observe on the Earthrsquos surface always sees the same face of the moon because

एक पयवक को पी की सतह स हमशा चाद का एक ही फलक िदखाई दता ह ोिक

1 Its path of revolution around the earth is the same as that of the earth around the Sun

इसका पी क चारो ओर परमण का माग सय क चारो ओर पी क समान ही ह

2 Its period of revolution around the Earth is the same as its period of rotation around its own axis

इसकी पी क चारो ओर परमण की अविध उसकी अपनी धरी क चारो ओर घणन की अविध क समान ह

3 Its period of rotation is the same as that of the Earth इसकी घणन की अविध पी क समान ह

ी ि ी ी

12)

13)

14)

4 Its direct of rotation is the same as that of Earth घणन की िदशा पी क समान ही ह

Correct Answer -

Its period of revolution around the Earth is the same as its period of rotation around its own axis

इसकी पी क चारो ओर परमण की अविध उसकी अपनी धरी क चारो ओर घणन की अविध क समान ह

CANCELLED

The pebbles that are faceted by the sand-blasting and shaped polished by the wind abrasions are known as

पवन अपघषन ारा पॉिलश रत-िवोिटत और साच म ढला ककड़ ___________ क प म जाना जाता ह

1 Dreikanter िकोणक

2 Pediments िकोिनका

3 Inselberg इलबग

4 Dunes टीबा

Correct Answer -

Dreikanter िकोणक

CANCELLED

Astronomical unit is the average distance between खगोलीय इकाई ______ क बीच की औसत दरी ह

1 Earth and Mars पी और मगल

2 Earth and mercury पी और बध

3 Earth and moon पी और चमा

4 Earth and Sun पी और सय

Correct Answer -

Earth and Sun पी और सय

During cold weather season in the northern plains there will be an inflow of cyclonic disturbancesfrom the _________ directions

शीत मौसम क दौरान उरी मदानी इलाको म _________ िदशाओ स चवात सबधी गड़बड़ी का अतवाह होगा

1 East and Northwest पव और उरपिम

2 East and Northeast पव और पवर

3 West and East पिम और पव

ि औ ि

15)

16)

17)

4 West and Northwest पिम और उरपिम

Correct Answer -

West and Northwest पिम और उरपिम

During an earth quake the velocity of the body waves will________ along with the increase in densityof the material it is passing through

भकप क दौरान लहरो क ऊपरी भाग का वग घन म व क साथ-साथ ________ जो इसस गजरन वाली वको आग बढाएगी

1 not change नही बदलगा

2 increase initially and then decrease शआत म बढ़गा और िफर घटगा

3 increase बढ़गा

4 decrease घटगा

Correct Answer -

increase बढ़गा

The Clouded Leopard National park is situated in which of the following states

िनिलखत म िकस रा म धिमल तदआ रा ीय उान (ाउडड लपड नशनल पाक ) थत ह

1 Tripura िपरा

2 Uttar Pradesh उर दश

3 Assam असम

4 Mizoram िमजोरम

Correct Answer -

Tripura िपरा

Usually the land surfaces are heated more quickly than the water surfaces because _____________

आम तौर पर जल सतहो की तलना म भिम सतह अिधक तजी स गम होती ह ोिक _____________ ह

1 the specific heat of water is higher than land पानी की िविश ऊा भिम स अिधक

2 the specific heat of water is lesser than land पानी की िविश ऊा भिम स कम होती

3 the latent heat of water is higher than the land पानी की अतिनिहत ऊा भिम स अिधक

4 the land reflects more heat radiation than water भिम पानी की तलना म अिधक ऊा क िविकरण को पराविततकरती

18)

19)

20)

21)

Correct Answer -

the specific heat of water is higher than land पानी की िविश ऊा भिम स अिधक

The longest shore-line is along the state of सबस लबी समतटीय रखा िन रा क साथ ह

1 Maharashtra महारा

2 Orissa उड़ीसा

3 Kerala करल

4 Gujarat गजरात

Correct Answer -

Gujarat गजरात

The position when the Earth is farthest from the Sun is known as

जब पी सय स सबस दर होती ह तो उस थित को िन नाम स जाना जाता ह

1 Perihelion उपसौर

2 Vernal Equinox बसत िवषव

3 Aphelion अपसौर

4 Autumnal Equinox शराल िवषव

Correct Answer -

Aphelion अपसौर

The seasonal reversal of winds is the typical characteristic of

हवाओ का मौसमी परवतन ______ की सामा िवशषता ह

1 Mediterranean climates only कवल भमसागरीय जलवाय

2 All of the above climates उपय सभी मौसम

3 Monsoon climate only कवल मानसन जलवाय

4 Equatorial climate only कवल भमरखीय जलवाय

Correct Answer -

Monsoon climate only कवल मानसन जलवाय

In _________ rocks the minerals will occurs in beds or layers

ो ि ो ो

22)

23)

24)

______ चानो म खिनज तल या परतो म होत ह

1 metamorphic कायातरत

2 igneous and metamorphic आय और कायातरत

3 igneous आय

4 sedimentary अवसादी

Correct Answer -

sedimentary अवसादी

Black soil is ideal for the cultivation of cotton as कपास की खती क िलए काली िमी आदश ह ोिक

1 Its colour is black यह काली होती ह

2 It is found on plateau regions यह पठार ो म पायी जाती ह

3 It is made up of lava यह लावा स बनी होती ह

4 It can retain moisture यह नमी को बरकरार रख सकती ह

Correct Answer -

It can retain moisture यह नमी को बरकरार रख सकती ह

The National Survey and Mapping Organization of the country works under the Department of___________

दश का रा ीय सवण और मानिचण सगठन ___________ िवभाग क अतगत काय करता ह

1 Space अतर

2 Science and Technology िवान और तकनीक

3 Culture सित

4 Tourism पयटन

Correct Answer -

Science and Technology िवान और तकनीक

Palk strait separates India from पाक जलडमम भारत स _____ को अलग करता ह

1 Pakistan पािकान

2 Andaman Island अडमान ीप

3 China चीन

25)

26)

27)

4 Sri Lanka ीलका

Correct Answer -

Sri Lanka ीलका

Which among the following state is the major producer of Bauxite in India

िनिलखत म स कौन सा रा भारत म बॉाइट का मख उादक ह

1 Madhya Pradesh मदश

2 Rajasthan राजथान

3 Goa गोवा

4 Orissa उड़ीसा

Correct Answer -

Orissa उड़ीसा

Which of the following states DOES NOT share border with Chhattisgarh

िनिलखत म स कौन सा रा छीसगढ़ क साथ सीमा साझा नही करता ह

1 Telangana तलगाना

2 Uttar Pradesh उर दश

3 Bihar िबहार

4 Andhra Pradesh आ दश

Correct Answer -

Bihar िबहार

Which of the following statements is INCORRECT with respect to parallels of latitudes

अाश क समानातरो क सबध म िन निलखत म स कौन सा कथन गलत ह

1 A line joining places of equal latitude is known as parallel of largest

समान अाश क थानो को जोड़न वाली रखा को िवशालतम क समानातर क प म जाना जाता ह

2 They stat from equator and run parallels to it

व भम रखा स ारभ होत ह और इसक समानातर चलत ह

3 All parallels are equal in length सभी समातर लबाई म समान ह

4 All parallels are drawn as circles on the globe ोब पर सभी समानातर वो क प म खीच जात ह

28)

29)

30)

31)

Correct Answer -

All parallels are equal in length सभी समातर लबाई म समान ह

Which of the following Indian states is also known as a lsquoLand of Red river and Blue Hillsrsquo

िनिलखत म स िकस भारतीय रा को लाल नदी और नीली पहािड़यो की भिम क नाम स जाना जाता ह

1 Uttarkhand उराखड

2 Assam असम

3 Meghalaya मघालय

4 Arunachal Pradesh अणाचल दश

Correct Answer -

Assam असम

In spatial analysis of settlement Rn = 215 indicates which type of settlement arrangement

िनपटान क थािनक िवषण म Rn = 215 यह इिगत करता ह िक िकस कार की िनपटान वथा ह

1 Uniform यिनफॉम

2 Semi-Clustered समी- ल टर

3 Clustered ल टर

4 Random रडम

Correct Answer -

Uniform यिनफॉम

Who are known as the lsquoYellow Peoplersquo lsquoयलो पीपलrsquo क प म कौन जाना जाता ह

1 Mongoloids मोगोलोइडस

2 Nigroids नीोइडस

3 Australoids ऑ लॉइडस

4 Caucasoids कॉकसोइडस

Correct Answer -

Mongoloids मोगोलोइडस

ि ि ो ौ ि

32)

33)

34)

Who publishes the topographical map of India भारत क थलाकितक मानिच को कौन कािशत करता ह

1 Geographical Survey of India भारत का भौगोिलक सवण

2 Government of India भारत सरकार

3 Geological Survey of India भारत क भगभय सवण

4 Survey of India भारत का सवण

Correct Answer -

Survey of India भारत का सवण

Who among the following claimed geography to be the lsquoEcology of Manrsquo

िनिलखत म स िकसन भगोल को मन का पारथितकी कहा ह

1 Alfred Hener अड हटनर

2 Vidal-de la Blache वाइडल-िड लॉ ॉश

3 Oo Schluter ओटो टर

4 Harlan Barrow हरलन बारो

Correct Answer -

Harlan Barrow हरलन बारो

Who among the following is regarded as the founder of humanistic approach in geography

िनिलखत म स िकस भगोल म मानवतावादी िकोण का सथापक माना जाता ह

1 William Bunge िविलयम बग

2 Yi-Fu-Tuan यी-फ- यान

3 Brain JL Berry न जएल बरी

4 Richard Peet रचड पीट

Correct Answer -

Yi-Fu-Tuan यी-फ- यान

Who prepared Lorenz curve लोरज व िकसन तयार िकया

1 Geddes गडस

2 None of these इनम स कोई नही

3 Griffith Taylor ििफथ टलर

35)

36)

37)

4 Max U Lorenz म य लोरज

Correct Answer -

Max U Lorenz म य लोरज

Gulf Streams are the currents of which of the following oceans

खाड़ी की धाराए िनिलखत महासागरो म स िकसकी धाराए ह

1 North Atlantic Ocean उरी अटलािटक महासागर

2 North Pacific Ocean उरी शात महासागर

3 Arabian Sea अरब सागर

4 South Pacific Ocean दिण शात महासागर

Correct Answer -

North Atlantic Ocean उरी अटलािटक महासागर

Disintegration wearing away and removal of rock material is generally referred as

िशला पदाथ (रॉक सामी) का टटना िमटना और हटना आमतौर पर ________ क प म सदिभत िकया जाता ह

1 Shattering िवसकारक

2 Denudation अनाादन

3 Fault श

4 Decomposition िवयोजन

Correct Answer -

Denudation अनाादन

Variations in the length of day time and night from season to season are due to

मौसम स मौसम परवतन पर िदन क समय और रात क समय की अविध म िभताए िन कारण स होती ह

1 The Earthrsquos revolution round the Sun in an elliptical manner पी का दीघवाकार तरीक स सय क चारो घणन

2 The Earthrsquos rotation on its axis पी का इसकी धरी पर घणन

3 Revolution of the Earth on a tilted axis नत अ पर पी का घणन

4 Latitudinal position of the place थान की अाश थित

Correct Answer -

Revolution of the Earth on a tilted axis नत अ पर पी का घणन

38)

39)

40)

Point out the correct sequence of mountain ranges from north to south

उर स दिण तक पवत खलाओ क सही अनम को इिगत कर

1 Great Himalaya Middle Himalaya Outer Himalaya Trans Himalaya

महान िहमालय म िहमालय बा िहमालय परा िहमालय

2 Middle Himalaya Great Himalaya Trans Himalaya Outer Himalaya

म िहमालय महान िहमालय परा िहमालय बा िहमालय

3 Outer Himalaya Middle Himalaya Great Himalaya Trans Himalaya

बा िहमालय म िहमालय महान िहमालय परा िहमालय

4 Trans Himalaya Great Himalaya Middle Himalaya Outer Himalaya

परा िहमालय महान िहमालय म िहमालय बा िहमालय

Correct Answer -

Trans Himalaya Great Himalaya Middle Himalaya Outer Himalaya

परा िहमालय महान िहमालय म िहमालय बा िहमालय

Sunrsquos halo is produced by the refraction of light in सय का भामडल ______ म काश क अपवतन ाराउ होता ह

1 Ice crystals in Cirrro-Cumulus clouds पाभ-कपास मघो क बफ िल

2 Ice crystal in Cirrus clouds पाभमघो क बफ िल

3 Dust particles in Stratus clouds री मघो क धल कण

4 Water vapour in Stratus clouds री मघो क जल वा

Correct Answer -

Ice crystal in Cirrus clouds पाभमघो क बफ िल

Read the given statements and answer which of the following options isare correct

(1) The minerals present in the rocks exposed to atmosphere are not subjected to alteration

(2) Oxidation is one of the processes of chemical weathering

िदए गए कथनो को पढ़ और उर द िक िन म स कौन सास िवक सही ह

(1) वायमल स अनावत शल म उपथत खिनज परवतन क अधीन नही होता ह

(2) ऑीकरण रासायिनक अपय की ियाओ म स एक ह

ो ो

41)

42)

1 Both statements are wrong दोनो कथन गलत ह

2 Both statements are correct दोनो कथन सही ह

3 First statement is wrong and second statement is correct पहला कथन गलत ह और दसरा कथन सही ह

4 First statement is correct and second statement is wrong पहला कथन सही ह और दसरा कथन गलत ह

Correct Answer -

First statement is wrong and second statement is correct पहला कथन गलत ह और दसरा कथन सही ह

Read the given statements and answer which of the following options isare correct

1 Sunrsquos short waves enter the earth partially heating the atmosphere

2 Heated earth surface from the sun produces broader waves which interacts and heats the atmosphere

िदए गए कथन को पढ़ और उर द िक िन म स कौन सास िवक सही ह

1 सय की छोटी तरग पी म आिशक प स वश करती ह और वायमडल को ऊत करती ह

2 सय स ऊत पी की सतह िवारत तरग उ करती ह जो परर भाव डालती ह और वायमडल कोऊत करती ह

1 Both Statements 1 and 2 are correct दोनो कथन 1 और 2 सही ह

2 Both Statements 1 and 2 are wrong दोनो कथन 1 और 2 गलत ह

3 Statement 1 is wrong and only Statement 2 is correct कथन 1 गलत ह और कवल कथन 2 सही ह

4 Only statement 1 is correct कवल कथन 1 सही ह

Correct Answer -

Both Statements 1 and 2 are correct दोनो कथन 1 और 2 सही ह

Read the given statements and answer which of the following options isare correct

(1)The rocks that get changed due to heat and pressure are termed as metamorphic rocks

(2)Slate is one such type of metamorphic rock

िदए गए कथनो को पढ़ और उर द िक िन म स कौन सास िवक सही ह

(1) शल जो ऊा और दाब क कारण परवितत हो जात ह उ कायातरक शलो क प म जाना जाता ह

(2) ट एक तरह का कायातरक शल ह

1 Both statements are wrong दोनो कथन गलत ह

2 Both statements are correct दोनो कथन सही ह

3 First statement is wrong and second statement is correct पहला कथन गलत ह और दसरा कथन सही ह

ी औ

43)

44)

4 First statement is correct and second statement is wrong पहला कथन सही ह और दसरा कथन गलत ह

Correct Answer -

Both statements are correct दोनो कथन सही ह

Read the given statements and answer which of the following options isare correct

1 Higher temperature anomaly is observed in the northern hemisphere

2 Differential heating is absent in Northern Hemisphere

िदए गए कथनो को पढ़ और उर द िक िन म स कौन सास िवक सही ह

1 उरी गोलाध म उ तापमान िवसगित पायी जाती ह

2 उरी गोलाध म अतर ऊन अनपथत होती ह

1 Both Statements 1 and 2 are correct दोनो कथन 1 और 2 सही ह

2 Both Statements 1 and 2 are wrong दोनो कथन 1 और 2 गलत ह

3 Statement 1 is wrong and Statement 2 is correct कथन 1 गलत ह और कथन 2 सही ह

4 Statement 1 is correct and Statement 2 is wrong कथन 1 सही ह और कथन 2 गलत ह

Correct Answer -

Statement 1 is correct and Statement 2 is wrong कथन 1 सही ह और कथन 2 गलत ह

Read the given statements and answer which of the following options isare correct

(1) Plutonic rocks are intrusive type of igneous rocks

(2) It cools very slowly because the surrounding rock serves as insulation around the intrusion of magma

िदए गए कथनो को पढ़ और उर द िक िन म स कौन सास िवक सही ह

(1) िवतलीय शल अतवधी कार क आश शल ह

(2) यह बत धीर-धीर ठडा होता ह ोिक आस-पास क शल मा क अतवधन क चारो ओर रोधन क प म कायकरत ह

1 Both statements are wrong दोनो कथन गलत ह

2 Both statements are correct दोनो कथन सही ह

3 First statement is wrong and second statement is correct पहला कथन गलत ह और दसरा कथन सही ह

4 First statement is correct and second statement is wrong पहला कथन सही ह और दसरा कथन गलत ह

Correct Answer -

Both statements are correct दोनो कथन सही ह

45)

46)

47)

48)

The dust and ash material hurled from the volcanoes are termed as

ालामखी स िनकलन वाली धल और राख सामी को _______ क प म कहा जाता ह

1 Pyroclasc पाइरोाक

2 Hyperclastic हाइपराक

3 Hepiroclastic हिपरोाक

4 Cirroclastic िसरोाक

Correct Answer -

Pyroclasc पाइरोाक

The vertical difference in elevation between a low tide and high tide is referred as

कम ार और उ ार क बीच ऊचाई म लबवत अतर _____ स सदिभत होता ह

1 Tidal slope ारीय ढलान

2 Tidal elevation ारीय उयन

3 Tidal range ारीय परास

4 Tidal height ारीय ऊचाई

Correct Answer -

Tidal range ारीय परास

The maximum biodiversity is found in which of the following regions िनिलखत ो म स अिधकतमजव िविवधता िकसम पायी जाती ह

1 Amazon Basin अमज़न बिसन

2 East Indies ई इडीज

3 Congo Basin कागो बिसन

4 West indies व इडीज

Correct Answer -

Amazon Basin अमज़न बिसन

The cultivation of rice crop produces_______ चावल की फसल की खती ______ का उादन करती ह

1 SO2

49)

50)

51)

2 CH4

3 CFCs

4 CO2

Correct Answer -

CH4

The pressure system with higher pressure at the centre is called__________

क म उ दबाव वाली दबाव णाली को _______ कहा जाता ह

1 front अ

2 depression अवनमन

3 cyclone चवात

4 anti-cyclone ितचवात

Correct Answer -

anti-cyclone ितचवात

The Himalayan region is poor in mineral resources because िहमालयी खिनज ससाधनो म समनही ह ोिक

1 The displacement of rock strata has disturbed the arrangement of rocks and made it complex

शलीय परत क िवथापन न चानो की वथा को अवथत कर िदया ह और इस जिटल बना िदया ह

2 The climate conditions are not suitable for exploitation of minerals

जलवाय की थित खिनजो क दोहन क िलए उपय नही ह

3 The terrain makes explanation of minerals difficult and very costly due to transportation difficulties

भ-भाग परवहन की किठनाइयो क कारण खिनजो का दोहन मल और बत महगा बना दता ह

4 It is made up of crystalline rocks यह िलीय चानो स बना ह

Correct Answer -

The displacement of rock strata has disturbed the arrangement of rocks and made it complex

शलीय परत क िवथापन न चानो की वथा को अवथत कर िदया ह और इस जिटल बना िदया ह

The process through which the moisture is added to the atmosphere by vegetation is termed as

वह िया िजसक माम स वनित ारा वातावरण म नमी िमलायी जाती ह _______ क प म जानी जाती ह

52)

53)

54)

1 Condensation सघनन

2 Evapotranspiration वान-उजन

3 Radiation िविकरण

4 Precipitation वषण

Correct Answer -

Evapotranspiration वान-उजन

The process through which the terrestrial heat is transferred to air by direct contact is termed as

वह िया िजसम सपक ारा थलीय ऊा वाय म थानातरत हो जाती ह ______ क प म जानी जाती ह

1 Conduction चालन

2 Convection सवहन

3 Insolation आतपन

4 Radiation िविकरण

Correct Answer -

Conduction चालन

The largest area under mangroves is in which of the following statesunion territory

मोव क अतगत िनिलखत राोसघ शािसत दशो म स सबस बड़ा कौन सा ह

1 Andaman and Nicobar अमान और िनकोबार

2 Andhra Pradesh आ दश

3 West Bengal पिम बगाल

4 Gujarat गजरात

Correct Answer -

West Bengal पिम बगाल

The longitudinal transverse and surface waves in an earthquake originate from

भकप म दशातर अनथ और सतह तरग यहा उ होती ह

1 The focus on the surface of the Earth पी क सतह पर क -िबद म

2 The focus within the body of the Earth पी क भीतर क -िबद म

3 The epicenter within the body of the Earth पी क भीतर उपरक म

55)

56)

57)

4 The epicenter on the surface of the Earth पी क सतह पर उपरक म

Correct Answer -

The focus within the body of the Earth पी क भीतर क -िबद म

The down slope movement of material due to gravity is called______

गाकषण क कारण पदाथ की अनढाल गित को ______ कहा जाता ह

1 mass movement पदाथ सचलन

2 deposition िनप

3 erosion रण

4 volcanic movement ालामखीय सचलन

Correct Answer -

mass movement पदाथ सचलन

Shimla is cooler than Amritsar although both are on the same latitude This is because

िशमला म अमतसर स अिधक ठड ह हालािक दोनो समान अाश पर ह ऐसा ह ोिक

1 Shimla is at a greater height above sea level than Amritsar अमतसर की तलना म िशमला सम तल स अिधकऊचाई पर ह

2 Shimla is further north िशमला उर की ओर ह

3 Shimla is farther from the equator िशमला भम रखा स आग ह

4 Their longitudes differ उनकी दशातर रखाए िभ ह

Correct Answer -

Shimla is at a greater height above sea level than Amritsar अमतसर की तलना म िशमला सम तल स अिधकऊचाई पर ह

lsquoTempo of Urbanizationrsquo measures which of the following

lsquoशहरीकरण का टपोrsquo िनिलखत म स कौन सा उपाय ह

1 Speed of urbanizaon शहरीकरण की गित

2 None of the above इनम स कोई नही

3 Inequality of urbanizaon शहरीकरण की असमानता

4 Current level of urbanizaon शहरीकरण का वतमान र

Correct Answer -

58)

59)

60)

Speed of urbanizaon शहरीकरण की गित

Out of the following options choose the INCORRECT statement

िनिलखत िवको म स गलत कथन का चयन कर

1 The clear tracts in the equatorial region recover rapidly भम रखा म भभाग तजी स ठीक हो जात ह

2 The stable communities include a redwood forest a pine forest at high elevations

थर समदायो म एक रडवड वन उ ऊचाई पर एक दवदार वन शािमल ह

3 Any ecosystem moves towards maximum biomass and stability to survive

कोई भी पारथितकी त जीिवत रहन क िलए अिधकतम जवसहित और थरता की तरफ असर होता ह

4 Tropical rain forests near equator are stable ecosystems

भम रखा क पास उकिटबधीय वषा वन थर पारथितक त ह

Correct Answer -

The clear tracts in the equatorial region recover rapidly भम रखा म भभाग तजी स ठीक हो जात ह

Seasonal contrasts are maximum in मौसमी िवषमता अिधकतम ह

1 Mid latitudes म अाश म

2 Low attitudes िन अाश म

3 High latitudes उ अाश म

4 Subtropics उपोकिटबधीय म

Correct Answer -

Mid latitudes म अाश म

In India which type of forest among the following occupies the largest area

भारत म िनिलखत म स िकस कार क वन सबस बड़ा फल आािदत करत ह

1 Sub-tropical Dry Evergreen Forest उप उकिटबधीय श सदाबहार वन

2 Mountain Wet Temperate Forest पवतीय आ शीतो वन

3 Tropical Moist Deciduous Forest उकिटबधीय आ पणपाती वन

4 Tropical Wet Evergreen Forest उकिटबधीय आ सदाबहार वन

Correct Answer -

Tropical Moist Deciduous Forest उकिटबधीय आ पणपाती वन

61)

62)

63)

64)

What is the proportion of lsquoJuvenile Populationrsquo (0-14 years) in India as per 2011Census

2011 की जनगणना क अनसार भारत म जवनाइल पॉपलशन यानी िकशोर जनस या (0-14 वष) का अनपात ाह

1 3076 of total population कल जनस या का 3076

2 2764 of total population कल जनस या का 2764

3 2933 of total population कल जनस या का 2933

4 3354 of total population कल जनस या का 3354

Correct Answer -

3076 of total population कल जनस या का 3076

What is the Belfast famous for बलफा िकसक िलए मशर ह

1 Belt of cotton textile industry कपास व उोग क

2 Ship-building industry जहाज िनमाण उोग

3 Agricultural machinery किष उपकरण

4 Aero planes manufacturing वाययान िनमाण

Correct Answer -

Ship-building industry जहाज िनमाण उोग

What is the most important occupation in tropical monsoon lands

उकिटबधीय मॉनसन भिम म सबस महपण वसाय ा ह

1 Mining खनन

2 Cattle rearing मवशी पालन

3 Agriculture किष

4 Nomadic herding नोमािडक जड़ी-बिटया

Correct Answer -

Agriculture किष

What is the most important characteristics of the islands (Indian) located in the Arabian sea

अरब सागर म थत ीपो (भारतीय) की सबस महपण िवशषता ा ह

ी ो

65)

66)

67)

1 There are all of coral origins सभी कोरल मल क ह

2 There are all very small in size य सभी आकार म बत छोट ह

3 They have a very dry climate इनकी जलवाय बत श ह

4 They are extended parts of the mainland व महाीप क िवारत िह ह

Correct Answer -

There are all of coral origins सभी कोरल मल क ह

What do the basalt layers of the Deccan indicate डन की बसा परत ा इिगत करती ह

1 All of the above उपरो सभी

2 Huge volcanic eruptions in the distant past दरथ अतीत म िवशाल ालामखीय िवोट

3 The immense erosional activity of the rivers निदयो की िवशाल रण गितिविध

4 The influence of weathering मौसम का भाव

Correct Answer -

Huge volcanic eruptions in the distant past दरथ अतीत म िवशाल ालामखीय िवोट

In the structure of planet Earth below the mantle the core is mainly made up of_____

पी ह की सरचना म मटल क नीच कोर म प स______ स िनिमत होती ह

1 aluminium एमीिनयम

2 silicon िसिलकॉन

3 chromium ोिमयम

4 iron लोहा

Correct Answer -

iron लोहा

One of the major Mid Oceanic Ridge is found in मख म-महासागर चोिटयो म स एक ______ म पायाजाता ह

1 Mid Pacific Ocean म शात महासागर

2 Mid Atlantic Ocean म अटलािटक महासागर

3 Mid Indian Ocean म भारतीय महासागर

4 Mid Arctic Ocean म आक िटक महासागर

68)

69)

70)

71)

Correct Answer -

Mid Atlantic Ocean म अटलािटक महासागर

Magma that reaches the Earthrsquos surface and then solidifies is called________

मा जो पी की सतह तक पचती ह और िफर ठोस हो जाती ह ________कहलाती ह

1 quartz ाटज

2 lava लावा

3 granite नाइट

4 silicates िसिलकट

Correct Answer -

lava लावा

Isotherms are the lines of equal_______ समताप रखाए समान _______की रखाए होती ह

1 pressure दाब

2 temperature तापमान

3 rainfall वषा

4 height ऊचाई

Correct Answer -

temperature तापमान

Mark the correct sequence of passes in the Western Ghats from north to south

पिमी घाटो म उर स दिण तक दर क सही अनम को िचित कर

1 Thalghat Palghat Bhorghat थलगघाट पालघाट भोरघाट

2 Thalghat Bhorghat Palghat थलघाट भोरघाट पालघाट

3 Bhorghat Thalghat Palghat भोरघाट थलघाट पालघाट

4 Palghat Bhorghat Thalghat पालघाट भोरघाट थलघाट

Correct Answer -

Thalghat Bhorghat Palghat थलघाट भोरघाट पालघाट

Which of the following does not have influence over the climate in India

ि ि ि ी ी

72)

73)

िनिलखत म स िकसका भाव भारत की जलवाय पर नही पड़ता ह

1 Ocean currents सागर की लहर

2 Nearness to equator भम रखा स िनकटता

3 Monsoons मानसन

4 Presence of Indian ocean भारतीय महासागर की उपथित

Correct Answer -

Ocean currents सागर की लहर

Which of the following cloud types has the characteristics like vertical tall narrow and puffy

िनिलखत म स िकस कार क मघो म लबवत लबी सकीण और थलता जसी िवशषताए ह

1 Cumulonimbus तफानी मघ

2 Cumulus मघ पज

3 Cirrocumulus पाभ कपासी मघ

4 Nimbostratus वषारी मघ

Correct Answer -

Cumulus मघ पज

Which of the following statement is INCORRECT about Crude Birth Rate

िनिलखत स कौन सा कथन अशोिधत ज दर क बार म सही नही ह

1 It cannot be used for comparing fertility level between two countries with different population characteristics

इसका उपयोग िविभ जनसा िवशषताओ वाल दो दशो क बीच जनन र की तलना क िलए नही िकया जा सकता ह

2 It is a standardized measure of fertility

यह जनन मता का मानकीकत उपाय ह

3 It is effected by the age-sex composition of the population

यह आबादी की आय-िलग सरचना स भािवत होता ह

4 It is expressed per 1000 population in a given geographical unit

यह िकसी दी गई भौगोिलक इकाई म ित 1000 जनसा पर िकया जाता ह

Correct Answer -

It is a standardized measure of fertility

यह जनन मता का मानकीकत उपाय ह

74)

75)

76)

77)

Which of the following state in India experienced negative decadal growth rate during 2001 to 2011census

भारत म िनिलखत म स िकस रा म वष 2001 स 2011 की जनगणना क दौरान नकाराक िगरावट दर ई

1 Tripura िपरा

2 Nagaland नागालड

3 Haryana हरयाणा

4 Odisha ओिडसा

Correct Answer -

Nagaland नागालड

Which of the following is NOT a characteristic of peninsular rivers

िनिलखत म स कौन सी िवशषता ायीपीय निदयो म नही होती ह

1 Flow through shallow valleys उथल घािटयो क माम स वाह

2 Seasonal flow मौसमी वाह

3 Little erosional activity थोड़ी कटावदार गितिविध

4 Meandering tendency often shifting their beds घमावदार वि अर अपन तटो को थानातरत करना

Correct Answer -

Meandering tendency often shifting their beds घमावदार वि अर अपन तटो को थानातरत करना

Which of the following gases in the atmosphere absorbs heat from the Sunrsquos radiation and the Earthssurface

वायमडल म िनिलखत म स कौन सी गस सय क िविकरण और पी की सतह स ऊा को अवशोिषत करती ह

1 Neon िनयॉन

2 Carbon dioxide काबन डाइऑाइड

3 Argon आगन

4 Nitrogen नाइट ोजन

Correct Answer -

Carbon dioxide काबन डाइऑाइड

Which of the following kind of settlement pattern is found at the confluence of rivers

ि ि ि ि ो

78)

79)

80)

िनिलखत म स िकस कार का वथापन पटन निदयो क सगम पर पाया जाता ह

1 Triangular Paern िकोणीय पटन

2 Circular or Semi-Circular Paern परप या अध-परप पटन

3 Nebular Paern नबलर पटन

4 Star ndashShaped Paern ार-आकार का पटन

Correct Answer -

Triangular Paern िकोणीय पटन

Which one was not the objective of the Biosphere Reserve Projects launched by the UNESCO

यनो ारा श की गई सरित जवमडल परयोजनाओ का उ इनम स कौन सा नही था

1 To promote teaching and research िशण और अनसधान को बढ़ावा दना

2 To make agriculture sustainable किष को दीघकािलक बनाना

3 To conserve ecosystems पारथितक त को सरित करना

4 To conserve genetic diversity for a longtime लब समय तक अनवािशक िविवधता को सरित करना

Correct Answer -

To make agriculture sustainable किष को दीघकािलक बनाना

Which region of the Earth surface receives the highest amount of insulation

पी सतह का कौन सा तापावरोधन की उतम माा ा करता ह

1 Land mass थलखड

2 Savannah region सवाना

3 Water bodies जल िनकाय

4 Tropical desert उकिटबधीय रिगान

Correct Answer -

Tropical desert उकिटबधीय रिगान

Which one of the following is not a biodiversity hotspot

िनिलखत म स कौन सा जव िविवधता का म जगह नही ह

1 Eastern Himalaya पव िहमालय

2 Eastern Ghats पव घाट

81)

82)

83)

3 Indo-Myanmar भारत-ामार

4 Westerm Ghats पिमी घाट

Correct Answer -

Eastern Ghats पव घाट

Which one of the following is NOT a part of the World Network of Biosphere Reserves based on theUNESCO Man and Biosphere Programme

यनो मन और बायोीयर कायम क आधार पर िनिलखत म स कौन बायोीयर रजव क िव नटवक कािहा नही ह

1 Gulf of Mannar मार की खाड़ी

2 Seshachalam शषाचलम

3 Sunderban सदरबन

4 Nilgiri नीलिगर

Correct Answer -

Seshachalam शषाचलम

Which one of the following is an example of ldquodesert vegetationrdquo

िनिलखत म स कौन मथलीय वनित का एक उदाहरण ह

1 Mosses and lichens दलदल और शवाल

2 Temperate grassland समशीतो घास क मदान

3 Coniferous forest शकधारी वन

4 Acacia and cactus एकािसया और कस

Correct Answer -

Acacia and cactus एकािसया और कस

Which one of the following reflects more sunlight िनिलखत म स कौन सा सय की रोशनी को अिधकपरावितत करता ह

1 Paddy crop land धान फसल भिम

2 Land covered with fresh snow ताजा बफ स आािदत भिम

3 Sand desert रतीली रिगान

4 Prairie land यरी भिम

84)

85)

86)

87)

Correct Answer -

Land covered with fresh snow ताजा बफ स आािदत भिम

Which layer of the atmosphere is in contact with the surface of the earthrsquos oceans

वायमडल की कौन सी परत पी क महासागरो की सतह क सपक म ह

1 Stratosphere समताप मडल

2 Mesosphere म मडल

3 Hydrosphere जलमडल

4 Troposphere ोभ मडल

Correct Answer -

Troposphere ोभ मडल

Mediterranean Sea is a border of which of the following countries भम सागर िनिलखत दशो म सिकसकी सीमा ह

1 None of these इनम स कोई नही

2 Iraq इराक

3 Lebanon लबनान

4 Jordan जॉडन

Correct Answer -

Lebanon लबनान

Benguela ocean currents are found along which coast बगएला महासागर धाराए िकस तट क साथ पायीजाती ह

1 East Coast of South America दिण अमरका क पव तट

2 East Coast of Africa अीका क पव तट

3 West Coast of South America दिण अमरका क पिमी तट

4 West Coast of Africa अीका क पिमी तट

Correct Answer -

West Coast of Africa अीका क पिमी तट

88)

89)

90)

Due to tension a block of land on one side being pushed up or upthrown relative to the downthrown blockis referred as

तनाव क कारण नीच फ क ए खड क साप भिम का एक खड एक ओर स ऊपर धकला जाता ह या ऊपर की ओरफ का जाता ह यह _____ क प म सदिभत ह

1 Thrust fault प श

2 Normal fault सामा श

3 Reverse fault म श

4 Strike slip fault नितलब सपण श

Correct Answer -

Normal fault सामा श

Inter-tropical doldrums is a zone of ______ अतर-उकिटबधीय डोलड ______ का एक ह

1 Frontolysis टोलायिसस

2 Convergence अिभसरण

3 Inter-tropical divergence zone अतर-उकिटबधीय िवचलन

4 Local wind थानीय वाय

Correct Answer -

Convergence अिभसरण

The Horse Latitudes are regions located at about _____ north and south of the equator

हॉस अाश भम रखा क उर और दिण म लगभग _____ पर थत ह

1 30ndash60 degree Latitude 30-60 िडी अाश

2 0ndash5 degree Latitude 0-5 िडी अाश

3 30 degree Latitude 30 िडी अाश

4 60ndash90 degree Latitude 60-90 िडी अाश

Correct Answer -

30 degree Latitude 30 िडी अाश

Generally evaporation is high over which part of the Earth

आम तौर पर पी क िकस भाग पर वाीकरण अिधक होता ह

1 Equatorial maritime भमवत समीय ी ी

91)

92)

2 Equatorial continental भमवत महाीपीय

3 Polar maritime वीय समीय

4 Polar continental वीय महाीपीय

Correct Answer -

Equatorial maritime भमवत समीय

A very high temperature during summer in north western India leads to what type of climaticcondition in south

उर पिमी भारत म गम क दौरान बत अिधक तापमान होन क कारण दिण म िकस कार की जलवाय थितउ करता ह

1 Depression over arabian sea अरब सागर पर अवनमन

2 Failure monsoon मानसन िवफलता

3 Successful monsoon मानसन सफलता

4 Cyclones चवात

Correct Answer -

Successful monsoon मानसन सफलता

Lightning and thunder are the resultant effect when तिड़त और गजन परणामी भाव ह जब

1 Two massive clouds hit powerfully each other first lightning is produced and later sound is produced

दो बड़ बादल एक दसर स शशाली ढग स टकरात ह पहल आकाशीय िवदयत उ होता ह और बाद म िन उहोती ह

2 Two massive clouds come into contact with the powerful wind collision this results into first sound and thenlightning

दो बड़ बादल शशाली पवन सघ क सपक म आत ह इसका परणामप पहल िन और िफर आकाशीय िवदयतउ होता ह

3 None of the above उपरो म स कोई भी नही

4 A high density cloud contains positively and negatively charged electric ions and when this interacts light andsound are simultaneously produced

एक उ घन बादल म धनाक और ऋणाक आविशत िवदयत आयन होत ह और जब यह परर भाव डालत ह तोकाश और िन एक साथ उािदत होती ह

Correct Answer -

A high density cloud contains positively and negatively charged electric ions and when this interacts light andsound are simultaneously produced

औ ि ि ो औ ो

93)

94)

95)

एक उ घन बादल म धनाक और ऋणाक आविशत िवदयत आयन होत ह और जब यह परर भाव डालत ह तोकाश और िन एक साथ उािदत होती ह

Doon Valley is able to grow rice because दन घाटी चावल उगान म सम ह ोिक

1 Other crops cannot be grown वहा अ फसलो को उगाया नही जा सकता ह

2 People in the valley are rice eaters घाटी म लोग चावल खान वाल ह

3 There is a huge export demand of rice वहा चावल की भारी िनयात माग ह

4 It has warm summer and snow melt waters for irrigation

वहा गिमया गम होती ह िसचाई क िलए बफ का िपघला आ पानी होता ह

Correct Answer -

It has warm summer and snow melt waters for irrigation

वहा गिमया गम होती ह िसचाई क िलए बफ का िपघला आ पानी होता ह

CANCELLED

In the geological time scale the Mesozoic Era DOES NOT contains which of the following periods

भगभय समय पमान पर मजीवी यग म िन कालो म स कौन नही ह

1 Triassic ट ाइऐिसक

2 Jurassic जरिसक

3 Cretaceous चाकमय

4 Carboniferous काबनी

Correct Answer -

Carboniferous काबनी

96)

1 P-3 Q-4 R-2 S-1

2 P-3 Q-4 R-1 S-2

3 P-3 Q-4 R-1 S-2

4 P-4 Q-3 R-2 S-1

Correct Answer -

P-4 Q-3 R-2 S-1

1 P-3 Q-1 R-4 S-2

2 P-3 Q-4 R-1 S-2

3 P-3 Q-2 R-4 S-1

97)

98)

4 P-2 Q-1 R-4 S-3

Correct Answer -

P-3 Q-4 R-1 S-2

1 P-3 Q-1 R-4 S-2

2 P-2 Q-3 R-4 S-1

3 P-2 Q-1 R-3 S-4

4 P-4 Q-2 R-1 S-3

Correct Answer -

P-3 Q-1 R-4 S-2

99)

100)

1 P-3 Q-2 R-4 S-1

2 P-1 Q-2 R-3 S-4

3 P-2 Q-3 R-1 S-4

4 P-4 Q-3 R-2 S-1

Correct Answer -

P-2 Q-3 R-1 S-4

ldquoHuman geography is the study of changing relationship between the unresting man and the unstableearthrdquo was defined by

lsquolsquoमानव भगोल ाकल आदमी और अथर पी क बीच सबध परवतन का अयन हrdquo ______ ारा परभािषत िकया गयाथा

1 J Brunches ज चस

2 EC Semple ईसी सल

3 HJ Mackinder एच ज मिकदर

4 PV Blache पीवी च

Correct Answer -

EC Semple ईसी सल

Sedimentary rocks are finally and ultimately derived from the____________

अवसादी चान अततः ________ स ा की जाती ह

1 action of earth movements पी की गितिविधयो

2 marine deposit समी िनप

3 weathering of metamorphic rocks पातरत चानो क अपय

4 weathering of igneous rocks आय चानो क अपय

Correct Answer -

weathering of igneous rocks आय चानो क अपय

Page 16: High School Teacher Eligibility Test- BOARD PROFESSIONAL ...peb.mp.gov.in/results/RESULT_18/HST_RES18/Final_anwser_key/HST… · M a ndl a / मंड ल ... Under the Madhya Pradesh

14)

15)

2 Democratic teacher लोकतीय िशक

3 Indifferent teacher िन प िशक

4 Direct instruction teacher िनदश िशक

Correct Answer -

Direct instruction teacher िनदश िशक

Who was the pioneer of classical conditioning

िचरितित ानकलन ( लािसकल कडीशिनग) क वतक कौन थ

1 Skinner नर

2 Pavlov पावलोव

3 Kohler कोहलर

4 Freud ायड

Correct Answer -

Pavlov पावलोव

Which of the following are true with reference to short term memory

1 Limited capacity

2 Brief storage of information

3 Unlimited capacity

4 Duration of storage less than twenty seconds

अ पकािलक मित क सदभ म िन न म स या स य ह

1 सीिमत मता

2 सचना का सि त भडारण

3 असीिमत मता

4 बीस सकड स कम भडारण की अविध

1 3 and 4 3 और 4

2 2 and 4 2 और 4

3 13 and 4 13 और 4

4 12 and 4 12 और 4

Correct Answer -

12 and 4 12 और 4

1)

2)

3)

4)

Topic- GEOGRAPHY

Which of the following ranges of population size is used to define Class-III city by Indian Census

भारतीय जनगणना ारा वग-III शहर को परभािषत करन क िलए िनिलखत म स िकस रज क जनसा आकारका उपयोग िकया जाता ह

1 20 000 to 49999 20 000 स 49999

2 30000 to 59999 30000 स 59999

3 24000 to 54999 24000 स 54999

4 50000 to 99999 50000 स 99999

Correct Answer -

20 000 to 49999 20 000 स 49999

Carbonaceous rocks which produce coal and oil belong to the category of rocks called_______

कोयल और तल का उादन करन वाली काबनय चान ______ नामक चानो की णी स सबिधत ह

1 metamorphic पातरत

2 sedimentary अवसादी

3 inorganic अजिवक

4 igneous आय

Correct Answer -

sedimentary अवसादी

The ruhr-complex is a major industrial centre in र-परसर िन का एक मख औोिगक क ह

1 North America उरी अमरका

2 Russia स

3 Germany जमनी

4 Europe यरोप

Correct Answer -

Germany जमनी

The term lsquoRegurrsquo refers to श lsquoरगरrsquo ______ स सबिधत ह

1 Deltaic alluvial soils डा जलोढ़ िमी

ि ी

5)

6)

2 Laterite soils लटराइट िमी

3 Red and yellow soils लाल और पीली िमी

4 Black cotton soils काली कपास िमी

Correct Answer -

Black cotton soils काली कपास िमी

Read the given statements and answer which of the following options isare correct

1 Lower the pressure greater the atmospheric disturbance

2 Air move from higher to low pressure

िदए गए कथन को पढ़ और उर द िक िन म स कौन सास िवक सही ह

1 िजतना दाब कम होगा वायमडलीय बाधाए उतनी अिधक होगी

2 वाय उ स िन दाब की ओर गित करती ह

1 Both Statements 1 and 2 are correct दोनो कथन 1 और 2 सही ह

2 Both Statements 1 and 2 are wrong दोनो कथन 1 और 2 गलत ह

3 Statement 1 is wrong and only Statement 2 is correct कथन 1 गलत ह और कवल कथन 2 सही ह

4 Statement 1 is correct and Statement 2 is wrong कथन 1 सही ह और कथन 2 गलत ह

Correct Answer -

Both Statements 1 and 2 are correct दोनो कथन 1 और 2 सही ह

CANCELLED

1 4 1 2 and 3 4 1 2 और 3

2 4 1 3 and 2 4 1 3 और 2

3 1 4 2 and 3 1 4 2 और 3

4 1 4 3 and 2 1 4 3 और 2

Correct Answer -

7)

8)

9)

1 4 3 and 2 1 4 3 और 2

CANCELLED

Karl Pearsonrsquos correlation co-efficient is काल िपयसन का सहसबध गणाक ह

1 Arithmec mean समार मा

2 Geometric mean गणोर मा

3 None of these इनम स कोई नही

4 Harmonic mean हराक मा

Correct Answer -

Geometric mean गणोर मा

CANCELLED

ldquoEach day is more or less the same the morning is clear and bright with a sea breeze as the Sun climbshigh in the sky heat mounts up dark clouds form then rain comes with thunder lighting But rain is soonoverrdquo Which of the following regions is described in the above passage

ldquoक िदन समान स अिधक या कम होता ह सम की हवा क साथ और उल सबह होती ह जस सयआकाश म ऊचा चढ़ता ह गम बढ़ जाती ह काल बादल बनत ह िफर िबजली क साथ बारश आती ह लिकनबारश जी ख हो जाती हlsquorsquo उपरो पा म िनिलखत म स िकन ो का वणन िकया गया ह

1 Equatorial भमरखीय

2 Equatorial भमरखीय

3 Savannah सवाना

4 Mediterranean आातरक (भमसागरीय)

5 Mediterranean आातरक (भमसागरीय)

6 Monsoon मानसन

7 Monsoon मानसन

Correct Answer -

Equatorial भमरखीय

Equatorial भमरखीय

CANCELLED

In which epoch of the geological history of the Earth dinosaurs reached their largest size

पी क भगभय इितहास क िकस यग म डायनासोर अपन सबस बड़ आकार तक पचि

10)

11)

1 Triassic ट ायिसक

2 Jurassic जरािसक

3 Cretaceous ीटशस

4 Permian पिमयन

Correct Answer -

Jurassic जरािसक

CANCELLED

A spring tide would occur in which of the following conditions

िनिलखत म स िकन थितयो म ार-भाटा आता ह

1 When the Sun Moon and Earth are in a straight line

जब सय चमा और पी एक सीधी रखा म होत ह

2 When the Moon and Earth are in right angle to each other

जब चमा और पी एक-दसर क दािहन कोण म होत ह

3 When the Earth and Moon are in right angle to the Sun

जब पी और चमा सय क दािहन कोण म होत ह

4 When the Sun and Moon are in right angle to each other

जब सय और चमा एक-दसर क दािहन कोण म होत ह

Correct Answer -

When the Sun Moon and Earth are in a straight line

जब सय चमा और पी एक सीधी रखा म होत ह

CANCELLED

An observe on the Earthrsquos surface always sees the same face of the moon because

एक पयवक को पी की सतह स हमशा चाद का एक ही फलक िदखाई दता ह ोिक

1 Its path of revolution around the earth is the same as that of the earth around the Sun

इसका पी क चारो ओर परमण का माग सय क चारो ओर पी क समान ही ह

2 Its period of revolution around the Earth is the same as its period of rotation around its own axis

इसकी पी क चारो ओर परमण की अविध उसकी अपनी धरी क चारो ओर घणन की अविध क समान ह

3 Its period of rotation is the same as that of the Earth इसकी घणन की अविध पी क समान ह

ी ि ी ी

12)

13)

14)

4 Its direct of rotation is the same as that of Earth घणन की िदशा पी क समान ही ह

Correct Answer -

Its period of revolution around the Earth is the same as its period of rotation around its own axis

इसकी पी क चारो ओर परमण की अविध उसकी अपनी धरी क चारो ओर घणन की अविध क समान ह

CANCELLED

The pebbles that are faceted by the sand-blasting and shaped polished by the wind abrasions are known as

पवन अपघषन ारा पॉिलश रत-िवोिटत और साच म ढला ककड़ ___________ क प म जाना जाता ह

1 Dreikanter िकोणक

2 Pediments िकोिनका

3 Inselberg इलबग

4 Dunes टीबा

Correct Answer -

Dreikanter िकोणक

CANCELLED

Astronomical unit is the average distance between खगोलीय इकाई ______ क बीच की औसत दरी ह

1 Earth and Mars पी और मगल

2 Earth and mercury पी और बध

3 Earth and moon पी और चमा

4 Earth and Sun पी और सय

Correct Answer -

Earth and Sun पी और सय

During cold weather season in the northern plains there will be an inflow of cyclonic disturbancesfrom the _________ directions

शीत मौसम क दौरान उरी मदानी इलाको म _________ िदशाओ स चवात सबधी गड़बड़ी का अतवाह होगा

1 East and Northwest पव और उरपिम

2 East and Northeast पव और पवर

3 West and East पिम और पव

ि औ ि

15)

16)

17)

4 West and Northwest पिम और उरपिम

Correct Answer -

West and Northwest पिम और उरपिम

During an earth quake the velocity of the body waves will________ along with the increase in densityof the material it is passing through

भकप क दौरान लहरो क ऊपरी भाग का वग घन म व क साथ-साथ ________ जो इसस गजरन वाली वको आग बढाएगी

1 not change नही बदलगा

2 increase initially and then decrease शआत म बढ़गा और िफर घटगा

3 increase बढ़गा

4 decrease घटगा

Correct Answer -

increase बढ़गा

The Clouded Leopard National park is situated in which of the following states

िनिलखत म िकस रा म धिमल तदआ रा ीय उान (ाउडड लपड नशनल पाक ) थत ह

1 Tripura िपरा

2 Uttar Pradesh उर दश

3 Assam असम

4 Mizoram िमजोरम

Correct Answer -

Tripura िपरा

Usually the land surfaces are heated more quickly than the water surfaces because _____________

आम तौर पर जल सतहो की तलना म भिम सतह अिधक तजी स गम होती ह ोिक _____________ ह

1 the specific heat of water is higher than land पानी की िविश ऊा भिम स अिधक

2 the specific heat of water is lesser than land पानी की िविश ऊा भिम स कम होती

3 the latent heat of water is higher than the land पानी की अतिनिहत ऊा भिम स अिधक

4 the land reflects more heat radiation than water भिम पानी की तलना म अिधक ऊा क िविकरण को पराविततकरती

18)

19)

20)

21)

Correct Answer -

the specific heat of water is higher than land पानी की िविश ऊा भिम स अिधक

The longest shore-line is along the state of सबस लबी समतटीय रखा िन रा क साथ ह

1 Maharashtra महारा

2 Orissa उड़ीसा

3 Kerala करल

4 Gujarat गजरात

Correct Answer -

Gujarat गजरात

The position when the Earth is farthest from the Sun is known as

जब पी सय स सबस दर होती ह तो उस थित को िन नाम स जाना जाता ह

1 Perihelion उपसौर

2 Vernal Equinox बसत िवषव

3 Aphelion अपसौर

4 Autumnal Equinox शराल िवषव

Correct Answer -

Aphelion अपसौर

The seasonal reversal of winds is the typical characteristic of

हवाओ का मौसमी परवतन ______ की सामा िवशषता ह

1 Mediterranean climates only कवल भमसागरीय जलवाय

2 All of the above climates उपय सभी मौसम

3 Monsoon climate only कवल मानसन जलवाय

4 Equatorial climate only कवल भमरखीय जलवाय

Correct Answer -

Monsoon climate only कवल मानसन जलवाय

In _________ rocks the minerals will occurs in beds or layers

ो ि ो ो

22)

23)

24)

______ चानो म खिनज तल या परतो म होत ह

1 metamorphic कायातरत

2 igneous and metamorphic आय और कायातरत

3 igneous आय

4 sedimentary अवसादी

Correct Answer -

sedimentary अवसादी

Black soil is ideal for the cultivation of cotton as कपास की खती क िलए काली िमी आदश ह ोिक

1 Its colour is black यह काली होती ह

2 It is found on plateau regions यह पठार ो म पायी जाती ह

3 It is made up of lava यह लावा स बनी होती ह

4 It can retain moisture यह नमी को बरकरार रख सकती ह

Correct Answer -

It can retain moisture यह नमी को बरकरार रख सकती ह

The National Survey and Mapping Organization of the country works under the Department of___________

दश का रा ीय सवण और मानिचण सगठन ___________ िवभाग क अतगत काय करता ह

1 Space अतर

2 Science and Technology िवान और तकनीक

3 Culture सित

4 Tourism पयटन

Correct Answer -

Science and Technology िवान और तकनीक

Palk strait separates India from पाक जलडमम भारत स _____ को अलग करता ह

1 Pakistan पािकान

2 Andaman Island अडमान ीप

3 China चीन

25)

26)

27)

4 Sri Lanka ीलका

Correct Answer -

Sri Lanka ीलका

Which among the following state is the major producer of Bauxite in India

िनिलखत म स कौन सा रा भारत म बॉाइट का मख उादक ह

1 Madhya Pradesh मदश

2 Rajasthan राजथान

3 Goa गोवा

4 Orissa उड़ीसा

Correct Answer -

Orissa उड़ीसा

Which of the following states DOES NOT share border with Chhattisgarh

िनिलखत म स कौन सा रा छीसगढ़ क साथ सीमा साझा नही करता ह

1 Telangana तलगाना

2 Uttar Pradesh उर दश

3 Bihar िबहार

4 Andhra Pradesh आ दश

Correct Answer -

Bihar िबहार

Which of the following statements is INCORRECT with respect to parallels of latitudes

अाश क समानातरो क सबध म िन निलखत म स कौन सा कथन गलत ह

1 A line joining places of equal latitude is known as parallel of largest

समान अाश क थानो को जोड़न वाली रखा को िवशालतम क समानातर क प म जाना जाता ह

2 They stat from equator and run parallels to it

व भम रखा स ारभ होत ह और इसक समानातर चलत ह

3 All parallels are equal in length सभी समातर लबाई म समान ह

4 All parallels are drawn as circles on the globe ोब पर सभी समानातर वो क प म खीच जात ह

28)

29)

30)

31)

Correct Answer -

All parallels are equal in length सभी समातर लबाई म समान ह

Which of the following Indian states is also known as a lsquoLand of Red river and Blue Hillsrsquo

िनिलखत म स िकस भारतीय रा को लाल नदी और नीली पहािड़यो की भिम क नाम स जाना जाता ह

1 Uttarkhand उराखड

2 Assam असम

3 Meghalaya मघालय

4 Arunachal Pradesh अणाचल दश

Correct Answer -

Assam असम

In spatial analysis of settlement Rn = 215 indicates which type of settlement arrangement

िनपटान क थािनक िवषण म Rn = 215 यह इिगत करता ह िक िकस कार की िनपटान वथा ह

1 Uniform यिनफॉम

2 Semi-Clustered समी- ल टर

3 Clustered ल टर

4 Random रडम

Correct Answer -

Uniform यिनफॉम

Who are known as the lsquoYellow Peoplersquo lsquoयलो पीपलrsquo क प म कौन जाना जाता ह

1 Mongoloids मोगोलोइडस

2 Nigroids नीोइडस

3 Australoids ऑ लॉइडस

4 Caucasoids कॉकसोइडस

Correct Answer -

Mongoloids मोगोलोइडस

ि ि ो ौ ि

32)

33)

34)

Who publishes the topographical map of India भारत क थलाकितक मानिच को कौन कािशत करता ह

1 Geographical Survey of India भारत का भौगोिलक सवण

2 Government of India भारत सरकार

3 Geological Survey of India भारत क भगभय सवण

4 Survey of India भारत का सवण

Correct Answer -

Survey of India भारत का सवण

Who among the following claimed geography to be the lsquoEcology of Manrsquo

िनिलखत म स िकसन भगोल को मन का पारथितकी कहा ह

1 Alfred Hener अड हटनर

2 Vidal-de la Blache वाइडल-िड लॉ ॉश

3 Oo Schluter ओटो टर

4 Harlan Barrow हरलन बारो

Correct Answer -

Harlan Barrow हरलन बारो

Who among the following is regarded as the founder of humanistic approach in geography

िनिलखत म स िकस भगोल म मानवतावादी िकोण का सथापक माना जाता ह

1 William Bunge िविलयम बग

2 Yi-Fu-Tuan यी-फ- यान

3 Brain JL Berry न जएल बरी

4 Richard Peet रचड पीट

Correct Answer -

Yi-Fu-Tuan यी-फ- यान

Who prepared Lorenz curve लोरज व िकसन तयार िकया

1 Geddes गडस

2 None of these इनम स कोई नही

3 Griffith Taylor ििफथ टलर

35)

36)

37)

4 Max U Lorenz म य लोरज

Correct Answer -

Max U Lorenz म य लोरज

Gulf Streams are the currents of which of the following oceans

खाड़ी की धाराए िनिलखत महासागरो म स िकसकी धाराए ह

1 North Atlantic Ocean उरी अटलािटक महासागर

2 North Pacific Ocean उरी शात महासागर

3 Arabian Sea अरब सागर

4 South Pacific Ocean दिण शात महासागर

Correct Answer -

North Atlantic Ocean उरी अटलािटक महासागर

Disintegration wearing away and removal of rock material is generally referred as

िशला पदाथ (रॉक सामी) का टटना िमटना और हटना आमतौर पर ________ क प म सदिभत िकया जाता ह

1 Shattering िवसकारक

2 Denudation अनाादन

3 Fault श

4 Decomposition िवयोजन

Correct Answer -

Denudation अनाादन

Variations in the length of day time and night from season to season are due to

मौसम स मौसम परवतन पर िदन क समय और रात क समय की अविध म िभताए िन कारण स होती ह

1 The Earthrsquos revolution round the Sun in an elliptical manner पी का दीघवाकार तरीक स सय क चारो घणन

2 The Earthrsquos rotation on its axis पी का इसकी धरी पर घणन

3 Revolution of the Earth on a tilted axis नत अ पर पी का घणन

4 Latitudinal position of the place थान की अाश थित

Correct Answer -

Revolution of the Earth on a tilted axis नत अ पर पी का घणन

38)

39)

40)

Point out the correct sequence of mountain ranges from north to south

उर स दिण तक पवत खलाओ क सही अनम को इिगत कर

1 Great Himalaya Middle Himalaya Outer Himalaya Trans Himalaya

महान िहमालय म िहमालय बा िहमालय परा िहमालय

2 Middle Himalaya Great Himalaya Trans Himalaya Outer Himalaya

म िहमालय महान िहमालय परा िहमालय बा िहमालय

3 Outer Himalaya Middle Himalaya Great Himalaya Trans Himalaya

बा िहमालय म िहमालय महान िहमालय परा िहमालय

4 Trans Himalaya Great Himalaya Middle Himalaya Outer Himalaya

परा िहमालय महान िहमालय म िहमालय बा िहमालय

Correct Answer -

Trans Himalaya Great Himalaya Middle Himalaya Outer Himalaya

परा िहमालय महान िहमालय म िहमालय बा िहमालय

Sunrsquos halo is produced by the refraction of light in सय का भामडल ______ म काश क अपवतन ाराउ होता ह

1 Ice crystals in Cirrro-Cumulus clouds पाभ-कपास मघो क बफ िल

2 Ice crystal in Cirrus clouds पाभमघो क बफ िल

3 Dust particles in Stratus clouds री मघो क धल कण

4 Water vapour in Stratus clouds री मघो क जल वा

Correct Answer -

Ice crystal in Cirrus clouds पाभमघो क बफ िल

Read the given statements and answer which of the following options isare correct

(1) The minerals present in the rocks exposed to atmosphere are not subjected to alteration

(2) Oxidation is one of the processes of chemical weathering

िदए गए कथनो को पढ़ और उर द िक िन म स कौन सास िवक सही ह

(1) वायमल स अनावत शल म उपथत खिनज परवतन क अधीन नही होता ह

(2) ऑीकरण रासायिनक अपय की ियाओ म स एक ह

ो ो

41)

42)

1 Both statements are wrong दोनो कथन गलत ह

2 Both statements are correct दोनो कथन सही ह

3 First statement is wrong and second statement is correct पहला कथन गलत ह और दसरा कथन सही ह

4 First statement is correct and second statement is wrong पहला कथन सही ह और दसरा कथन गलत ह

Correct Answer -

First statement is wrong and second statement is correct पहला कथन गलत ह और दसरा कथन सही ह

Read the given statements and answer which of the following options isare correct

1 Sunrsquos short waves enter the earth partially heating the atmosphere

2 Heated earth surface from the sun produces broader waves which interacts and heats the atmosphere

िदए गए कथन को पढ़ और उर द िक िन म स कौन सास िवक सही ह

1 सय की छोटी तरग पी म आिशक प स वश करती ह और वायमडल को ऊत करती ह

2 सय स ऊत पी की सतह िवारत तरग उ करती ह जो परर भाव डालती ह और वायमडल कोऊत करती ह

1 Both Statements 1 and 2 are correct दोनो कथन 1 और 2 सही ह

2 Both Statements 1 and 2 are wrong दोनो कथन 1 और 2 गलत ह

3 Statement 1 is wrong and only Statement 2 is correct कथन 1 गलत ह और कवल कथन 2 सही ह

4 Only statement 1 is correct कवल कथन 1 सही ह

Correct Answer -

Both Statements 1 and 2 are correct दोनो कथन 1 और 2 सही ह

Read the given statements and answer which of the following options isare correct

(1)The rocks that get changed due to heat and pressure are termed as metamorphic rocks

(2)Slate is one such type of metamorphic rock

िदए गए कथनो को पढ़ और उर द िक िन म स कौन सास िवक सही ह

(1) शल जो ऊा और दाब क कारण परवितत हो जात ह उ कायातरक शलो क प म जाना जाता ह

(2) ट एक तरह का कायातरक शल ह

1 Both statements are wrong दोनो कथन गलत ह

2 Both statements are correct दोनो कथन सही ह

3 First statement is wrong and second statement is correct पहला कथन गलत ह और दसरा कथन सही ह

ी औ

43)

44)

4 First statement is correct and second statement is wrong पहला कथन सही ह और दसरा कथन गलत ह

Correct Answer -

Both statements are correct दोनो कथन सही ह

Read the given statements and answer which of the following options isare correct

1 Higher temperature anomaly is observed in the northern hemisphere

2 Differential heating is absent in Northern Hemisphere

िदए गए कथनो को पढ़ और उर द िक िन म स कौन सास िवक सही ह

1 उरी गोलाध म उ तापमान िवसगित पायी जाती ह

2 उरी गोलाध म अतर ऊन अनपथत होती ह

1 Both Statements 1 and 2 are correct दोनो कथन 1 और 2 सही ह

2 Both Statements 1 and 2 are wrong दोनो कथन 1 और 2 गलत ह

3 Statement 1 is wrong and Statement 2 is correct कथन 1 गलत ह और कथन 2 सही ह

4 Statement 1 is correct and Statement 2 is wrong कथन 1 सही ह और कथन 2 गलत ह

Correct Answer -

Statement 1 is correct and Statement 2 is wrong कथन 1 सही ह और कथन 2 गलत ह

Read the given statements and answer which of the following options isare correct

(1) Plutonic rocks are intrusive type of igneous rocks

(2) It cools very slowly because the surrounding rock serves as insulation around the intrusion of magma

िदए गए कथनो को पढ़ और उर द िक िन म स कौन सास िवक सही ह

(1) िवतलीय शल अतवधी कार क आश शल ह

(2) यह बत धीर-धीर ठडा होता ह ोिक आस-पास क शल मा क अतवधन क चारो ओर रोधन क प म कायकरत ह

1 Both statements are wrong दोनो कथन गलत ह

2 Both statements are correct दोनो कथन सही ह

3 First statement is wrong and second statement is correct पहला कथन गलत ह और दसरा कथन सही ह

4 First statement is correct and second statement is wrong पहला कथन सही ह और दसरा कथन गलत ह

Correct Answer -

Both statements are correct दोनो कथन सही ह

45)

46)

47)

48)

The dust and ash material hurled from the volcanoes are termed as

ालामखी स िनकलन वाली धल और राख सामी को _______ क प म कहा जाता ह

1 Pyroclasc पाइरोाक

2 Hyperclastic हाइपराक

3 Hepiroclastic हिपरोाक

4 Cirroclastic िसरोाक

Correct Answer -

Pyroclasc पाइरोाक

The vertical difference in elevation between a low tide and high tide is referred as

कम ार और उ ार क बीच ऊचाई म लबवत अतर _____ स सदिभत होता ह

1 Tidal slope ारीय ढलान

2 Tidal elevation ारीय उयन

3 Tidal range ारीय परास

4 Tidal height ारीय ऊचाई

Correct Answer -

Tidal range ारीय परास

The maximum biodiversity is found in which of the following regions िनिलखत ो म स अिधकतमजव िविवधता िकसम पायी जाती ह

1 Amazon Basin अमज़न बिसन

2 East Indies ई इडीज

3 Congo Basin कागो बिसन

4 West indies व इडीज

Correct Answer -

Amazon Basin अमज़न बिसन

The cultivation of rice crop produces_______ चावल की फसल की खती ______ का उादन करती ह

1 SO2

49)

50)

51)

2 CH4

3 CFCs

4 CO2

Correct Answer -

CH4

The pressure system with higher pressure at the centre is called__________

क म उ दबाव वाली दबाव णाली को _______ कहा जाता ह

1 front अ

2 depression अवनमन

3 cyclone चवात

4 anti-cyclone ितचवात

Correct Answer -

anti-cyclone ितचवात

The Himalayan region is poor in mineral resources because िहमालयी खिनज ससाधनो म समनही ह ोिक

1 The displacement of rock strata has disturbed the arrangement of rocks and made it complex

शलीय परत क िवथापन न चानो की वथा को अवथत कर िदया ह और इस जिटल बना िदया ह

2 The climate conditions are not suitable for exploitation of minerals

जलवाय की थित खिनजो क दोहन क िलए उपय नही ह

3 The terrain makes explanation of minerals difficult and very costly due to transportation difficulties

भ-भाग परवहन की किठनाइयो क कारण खिनजो का दोहन मल और बत महगा बना दता ह

4 It is made up of crystalline rocks यह िलीय चानो स बना ह

Correct Answer -

The displacement of rock strata has disturbed the arrangement of rocks and made it complex

शलीय परत क िवथापन न चानो की वथा को अवथत कर िदया ह और इस जिटल बना िदया ह

The process through which the moisture is added to the atmosphere by vegetation is termed as

वह िया िजसक माम स वनित ारा वातावरण म नमी िमलायी जाती ह _______ क प म जानी जाती ह

52)

53)

54)

1 Condensation सघनन

2 Evapotranspiration वान-उजन

3 Radiation िविकरण

4 Precipitation वषण

Correct Answer -

Evapotranspiration वान-उजन

The process through which the terrestrial heat is transferred to air by direct contact is termed as

वह िया िजसम सपक ारा थलीय ऊा वाय म थानातरत हो जाती ह ______ क प म जानी जाती ह

1 Conduction चालन

2 Convection सवहन

3 Insolation आतपन

4 Radiation िविकरण

Correct Answer -

Conduction चालन

The largest area under mangroves is in which of the following statesunion territory

मोव क अतगत िनिलखत राोसघ शािसत दशो म स सबस बड़ा कौन सा ह

1 Andaman and Nicobar अमान और िनकोबार

2 Andhra Pradesh आ दश

3 West Bengal पिम बगाल

4 Gujarat गजरात

Correct Answer -

West Bengal पिम बगाल

The longitudinal transverse and surface waves in an earthquake originate from

भकप म दशातर अनथ और सतह तरग यहा उ होती ह

1 The focus on the surface of the Earth पी क सतह पर क -िबद म

2 The focus within the body of the Earth पी क भीतर क -िबद म

3 The epicenter within the body of the Earth पी क भीतर उपरक म

55)

56)

57)

4 The epicenter on the surface of the Earth पी क सतह पर उपरक म

Correct Answer -

The focus within the body of the Earth पी क भीतर क -िबद म

The down slope movement of material due to gravity is called______

गाकषण क कारण पदाथ की अनढाल गित को ______ कहा जाता ह

1 mass movement पदाथ सचलन

2 deposition िनप

3 erosion रण

4 volcanic movement ालामखीय सचलन

Correct Answer -

mass movement पदाथ सचलन

Shimla is cooler than Amritsar although both are on the same latitude This is because

िशमला म अमतसर स अिधक ठड ह हालािक दोनो समान अाश पर ह ऐसा ह ोिक

1 Shimla is at a greater height above sea level than Amritsar अमतसर की तलना म िशमला सम तल स अिधकऊचाई पर ह

2 Shimla is further north िशमला उर की ओर ह

3 Shimla is farther from the equator िशमला भम रखा स आग ह

4 Their longitudes differ उनकी दशातर रखाए िभ ह

Correct Answer -

Shimla is at a greater height above sea level than Amritsar अमतसर की तलना म िशमला सम तल स अिधकऊचाई पर ह

lsquoTempo of Urbanizationrsquo measures which of the following

lsquoशहरीकरण का टपोrsquo िनिलखत म स कौन सा उपाय ह

1 Speed of urbanizaon शहरीकरण की गित

2 None of the above इनम स कोई नही

3 Inequality of urbanizaon शहरीकरण की असमानता

4 Current level of urbanizaon शहरीकरण का वतमान र

Correct Answer -

58)

59)

60)

Speed of urbanizaon शहरीकरण की गित

Out of the following options choose the INCORRECT statement

िनिलखत िवको म स गलत कथन का चयन कर

1 The clear tracts in the equatorial region recover rapidly भम रखा म भभाग तजी स ठीक हो जात ह

2 The stable communities include a redwood forest a pine forest at high elevations

थर समदायो म एक रडवड वन उ ऊचाई पर एक दवदार वन शािमल ह

3 Any ecosystem moves towards maximum biomass and stability to survive

कोई भी पारथितकी त जीिवत रहन क िलए अिधकतम जवसहित और थरता की तरफ असर होता ह

4 Tropical rain forests near equator are stable ecosystems

भम रखा क पास उकिटबधीय वषा वन थर पारथितक त ह

Correct Answer -

The clear tracts in the equatorial region recover rapidly भम रखा म भभाग तजी स ठीक हो जात ह

Seasonal contrasts are maximum in मौसमी िवषमता अिधकतम ह

1 Mid latitudes म अाश म

2 Low attitudes िन अाश म

3 High latitudes उ अाश म

4 Subtropics उपोकिटबधीय म

Correct Answer -

Mid latitudes म अाश म

In India which type of forest among the following occupies the largest area

भारत म िनिलखत म स िकस कार क वन सबस बड़ा फल आािदत करत ह

1 Sub-tropical Dry Evergreen Forest उप उकिटबधीय श सदाबहार वन

2 Mountain Wet Temperate Forest पवतीय आ शीतो वन

3 Tropical Moist Deciduous Forest उकिटबधीय आ पणपाती वन

4 Tropical Wet Evergreen Forest उकिटबधीय आ सदाबहार वन

Correct Answer -

Tropical Moist Deciduous Forest उकिटबधीय आ पणपाती वन

61)

62)

63)

64)

What is the proportion of lsquoJuvenile Populationrsquo (0-14 years) in India as per 2011Census

2011 की जनगणना क अनसार भारत म जवनाइल पॉपलशन यानी िकशोर जनस या (0-14 वष) का अनपात ाह

1 3076 of total population कल जनस या का 3076

2 2764 of total population कल जनस या का 2764

3 2933 of total population कल जनस या का 2933

4 3354 of total population कल जनस या का 3354

Correct Answer -

3076 of total population कल जनस या का 3076

What is the Belfast famous for बलफा िकसक िलए मशर ह

1 Belt of cotton textile industry कपास व उोग क

2 Ship-building industry जहाज िनमाण उोग

3 Agricultural machinery किष उपकरण

4 Aero planes manufacturing वाययान िनमाण

Correct Answer -

Ship-building industry जहाज िनमाण उोग

What is the most important occupation in tropical monsoon lands

उकिटबधीय मॉनसन भिम म सबस महपण वसाय ा ह

1 Mining खनन

2 Cattle rearing मवशी पालन

3 Agriculture किष

4 Nomadic herding नोमािडक जड़ी-बिटया

Correct Answer -

Agriculture किष

What is the most important characteristics of the islands (Indian) located in the Arabian sea

अरब सागर म थत ीपो (भारतीय) की सबस महपण िवशषता ा ह

ी ो

65)

66)

67)

1 There are all of coral origins सभी कोरल मल क ह

2 There are all very small in size य सभी आकार म बत छोट ह

3 They have a very dry climate इनकी जलवाय बत श ह

4 They are extended parts of the mainland व महाीप क िवारत िह ह

Correct Answer -

There are all of coral origins सभी कोरल मल क ह

What do the basalt layers of the Deccan indicate डन की बसा परत ा इिगत करती ह

1 All of the above उपरो सभी

2 Huge volcanic eruptions in the distant past दरथ अतीत म िवशाल ालामखीय िवोट

3 The immense erosional activity of the rivers निदयो की िवशाल रण गितिविध

4 The influence of weathering मौसम का भाव

Correct Answer -

Huge volcanic eruptions in the distant past दरथ अतीत म िवशाल ालामखीय िवोट

In the structure of planet Earth below the mantle the core is mainly made up of_____

पी ह की सरचना म मटल क नीच कोर म प स______ स िनिमत होती ह

1 aluminium एमीिनयम

2 silicon िसिलकॉन

3 chromium ोिमयम

4 iron लोहा

Correct Answer -

iron लोहा

One of the major Mid Oceanic Ridge is found in मख म-महासागर चोिटयो म स एक ______ म पायाजाता ह

1 Mid Pacific Ocean म शात महासागर

2 Mid Atlantic Ocean म अटलािटक महासागर

3 Mid Indian Ocean म भारतीय महासागर

4 Mid Arctic Ocean म आक िटक महासागर

68)

69)

70)

71)

Correct Answer -

Mid Atlantic Ocean म अटलािटक महासागर

Magma that reaches the Earthrsquos surface and then solidifies is called________

मा जो पी की सतह तक पचती ह और िफर ठोस हो जाती ह ________कहलाती ह

1 quartz ाटज

2 lava लावा

3 granite नाइट

4 silicates िसिलकट

Correct Answer -

lava लावा

Isotherms are the lines of equal_______ समताप रखाए समान _______की रखाए होती ह

1 pressure दाब

2 temperature तापमान

3 rainfall वषा

4 height ऊचाई

Correct Answer -

temperature तापमान

Mark the correct sequence of passes in the Western Ghats from north to south

पिमी घाटो म उर स दिण तक दर क सही अनम को िचित कर

1 Thalghat Palghat Bhorghat थलगघाट पालघाट भोरघाट

2 Thalghat Bhorghat Palghat थलघाट भोरघाट पालघाट

3 Bhorghat Thalghat Palghat भोरघाट थलघाट पालघाट

4 Palghat Bhorghat Thalghat पालघाट भोरघाट थलघाट

Correct Answer -

Thalghat Bhorghat Palghat थलघाट भोरघाट पालघाट

Which of the following does not have influence over the climate in India

ि ि ि ी ी

72)

73)

िनिलखत म स िकसका भाव भारत की जलवाय पर नही पड़ता ह

1 Ocean currents सागर की लहर

2 Nearness to equator भम रखा स िनकटता

3 Monsoons मानसन

4 Presence of Indian ocean भारतीय महासागर की उपथित

Correct Answer -

Ocean currents सागर की लहर

Which of the following cloud types has the characteristics like vertical tall narrow and puffy

िनिलखत म स िकस कार क मघो म लबवत लबी सकीण और थलता जसी िवशषताए ह

1 Cumulonimbus तफानी मघ

2 Cumulus मघ पज

3 Cirrocumulus पाभ कपासी मघ

4 Nimbostratus वषारी मघ

Correct Answer -

Cumulus मघ पज

Which of the following statement is INCORRECT about Crude Birth Rate

िनिलखत स कौन सा कथन अशोिधत ज दर क बार म सही नही ह

1 It cannot be used for comparing fertility level between two countries with different population characteristics

इसका उपयोग िविभ जनसा िवशषताओ वाल दो दशो क बीच जनन र की तलना क िलए नही िकया जा सकता ह

2 It is a standardized measure of fertility

यह जनन मता का मानकीकत उपाय ह

3 It is effected by the age-sex composition of the population

यह आबादी की आय-िलग सरचना स भािवत होता ह

4 It is expressed per 1000 population in a given geographical unit

यह िकसी दी गई भौगोिलक इकाई म ित 1000 जनसा पर िकया जाता ह

Correct Answer -

It is a standardized measure of fertility

यह जनन मता का मानकीकत उपाय ह

74)

75)

76)

77)

Which of the following state in India experienced negative decadal growth rate during 2001 to 2011census

भारत म िनिलखत म स िकस रा म वष 2001 स 2011 की जनगणना क दौरान नकाराक िगरावट दर ई

1 Tripura िपरा

2 Nagaland नागालड

3 Haryana हरयाणा

4 Odisha ओिडसा

Correct Answer -

Nagaland नागालड

Which of the following is NOT a characteristic of peninsular rivers

िनिलखत म स कौन सी िवशषता ायीपीय निदयो म नही होती ह

1 Flow through shallow valleys उथल घािटयो क माम स वाह

2 Seasonal flow मौसमी वाह

3 Little erosional activity थोड़ी कटावदार गितिविध

4 Meandering tendency often shifting their beds घमावदार वि अर अपन तटो को थानातरत करना

Correct Answer -

Meandering tendency often shifting their beds घमावदार वि अर अपन तटो को थानातरत करना

Which of the following gases in the atmosphere absorbs heat from the Sunrsquos radiation and the Earthssurface

वायमडल म िनिलखत म स कौन सी गस सय क िविकरण और पी की सतह स ऊा को अवशोिषत करती ह

1 Neon िनयॉन

2 Carbon dioxide काबन डाइऑाइड

3 Argon आगन

4 Nitrogen नाइट ोजन

Correct Answer -

Carbon dioxide काबन डाइऑाइड

Which of the following kind of settlement pattern is found at the confluence of rivers

ि ि ि ि ो

78)

79)

80)

िनिलखत म स िकस कार का वथापन पटन निदयो क सगम पर पाया जाता ह

1 Triangular Paern िकोणीय पटन

2 Circular or Semi-Circular Paern परप या अध-परप पटन

3 Nebular Paern नबलर पटन

4 Star ndashShaped Paern ार-आकार का पटन

Correct Answer -

Triangular Paern िकोणीय पटन

Which one was not the objective of the Biosphere Reserve Projects launched by the UNESCO

यनो ारा श की गई सरित जवमडल परयोजनाओ का उ इनम स कौन सा नही था

1 To promote teaching and research िशण और अनसधान को बढ़ावा दना

2 To make agriculture sustainable किष को दीघकािलक बनाना

3 To conserve ecosystems पारथितक त को सरित करना

4 To conserve genetic diversity for a longtime लब समय तक अनवािशक िविवधता को सरित करना

Correct Answer -

To make agriculture sustainable किष को दीघकािलक बनाना

Which region of the Earth surface receives the highest amount of insulation

पी सतह का कौन सा तापावरोधन की उतम माा ा करता ह

1 Land mass थलखड

2 Savannah region सवाना

3 Water bodies जल िनकाय

4 Tropical desert उकिटबधीय रिगान

Correct Answer -

Tropical desert उकिटबधीय रिगान

Which one of the following is not a biodiversity hotspot

िनिलखत म स कौन सा जव िविवधता का म जगह नही ह

1 Eastern Himalaya पव िहमालय

2 Eastern Ghats पव घाट

81)

82)

83)

3 Indo-Myanmar भारत-ामार

4 Westerm Ghats पिमी घाट

Correct Answer -

Eastern Ghats पव घाट

Which one of the following is NOT a part of the World Network of Biosphere Reserves based on theUNESCO Man and Biosphere Programme

यनो मन और बायोीयर कायम क आधार पर िनिलखत म स कौन बायोीयर रजव क िव नटवक कािहा नही ह

1 Gulf of Mannar मार की खाड़ी

2 Seshachalam शषाचलम

3 Sunderban सदरबन

4 Nilgiri नीलिगर

Correct Answer -

Seshachalam शषाचलम

Which one of the following is an example of ldquodesert vegetationrdquo

िनिलखत म स कौन मथलीय वनित का एक उदाहरण ह

1 Mosses and lichens दलदल और शवाल

2 Temperate grassland समशीतो घास क मदान

3 Coniferous forest शकधारी वन

4 Acacia and cactus एकािसया और कस

Correct Answer -

Acacia and cactus एकािसया और कस

Which one of the following reflects more sunlight िनिलखत म स कौन सा सय की रोशनी को अिधकपरावितत करता ह

1 Paddy crop land धान फसल भिम

2 Land covered with fresh snow ताजा बफ स आािदत भिम

3 Sand desert रतीली रिगान

4 Prairie land यरी भिम

84)

85)

86)

87)

Correct Answer -

Land covered with fresh snow ताजा बफ स आािदत भिम

Which layer of the atmosphere is in contact with the surface of the earthrsquos oceans

वायमडल की कौन सी परत पी क महासागरो की सतह क सपक म ह

1 Stratosphere समताप मडल

2 Mesosphere म मडल

3 Hydrosphere जलमडल

4 Troposphere ोभ मडल

Correct Answer -

Troposphere ोभ मडल

Mediterranean Sea is a border of which of the following countries भम सागर िनिलखत दशो म सिकसकी सीमा ह

1 None of these इनम स कोई नही

2 Iraq इराक

3 Lebanon लबनान

4 Jordan जॉडन

Correct Answer -

Lebanon लबनान

Benguela ocean currents are found along which coast बगएला महासागर धाराए िकस तट क साथ पायीजाती ह

1 East Coast of South America दिण अमरका क पव तट

2 East Coast of Africa अीका क पव तट

3 West Coast of South America दिण अमरका क पिमी तट

4 West Coast of Africa अीका क पिमी तट

Correct Answer -

West Coast of Africa अीका क पिमी तट

88)

89)

90)

Due to tension a block of land on one side being pushed up or upthrown relative to the downthrown blockis referred as

तनाव क कारण नीच फ क ए खड क साप भिम का एक खड एक ओर स ऊपर धकला जाता ह या ऊपर की ओरफ का जाता ह यह _____ क प म सदिभत ह

1 Thrust fault प श

2 Normal fault सामा श

3 Reverse fault म श

4 Strike slip fault नितलब सपण श

Correct Answer -

Normal fault सामा श

Inter-tropical doldrums is a zone of ______ अतर-उकिटबधीय डोलड ______ का एक ह

1 Frontolysis टोलायिसस

2 Convergence अिभसरण

3 Inter-tropical divergence zone अतर-उकिटबधीय िवचलन

4 Local wind थानीय वाय

Correct Answer -

Convergence अिभसरण

The Horse Latitudes are regions located at about _____ north and south of the equator

हॉस अाश भम रखा क उर और दिण म लगभग _____ पर थत ह

1 30ndash60 degree Latitude 30-60 िडी अाश

2 0ndash5 degree Latitude 0-5 िडी अाश

3 30 degree Latitude 30 िडी अाश

4 60ndash90 degree Latitude 60-90 िडी अाश

Correct Answer -

30 degree Latitude 30 िडी अाश

Generally evaporation is high over which part of the Earth

आम तौर पर पी क िकस भाग पर वाीकरण अिधक होता ह

1 Equatorial maritime भमवत समीय ी ी

91)

92)

2 Equatorial continental भमवत महाीपीय

3 Polar maritime वीय समीय

4 Polar continental वीय महाीपीय

Correct Answer -

Equatorial maritime भमवत समीय

A very high temperature during summer in north western India leads to what type of climaticcondition in south

उर पिमी भारत म गम क दौरान बत अिधक तापमान होन क कारण दिण म िकस कार की जलवाय थितउ करता ह

1 Depression over arabian sea अरब सागर पर अवनमन

2 Failure monsoon मानसन िवफलता

3 Successful monsoon मानसन सफलता

4 Cyclones चवात

Correct Answer -

Successful monsoon मानसन सफलता

Lightning and thunder are the resultant effect when तिड़त और गजन परणामी भाव ह जब

1 Two massive clouds hit powerfully each other first lightning is produced and later sound is produced

दो बड़ बादल एक दसर स शशाली ढग स टकरात ह पहल आकाशीय िवदयत उ होता ह और बाद म िन उहोती ह

2 Two massive clouds come into contact with the powerful wind collision this results into first sound and thenlightning

दो बड़ बादल शशाली पवन सघ क सपक म आत ह इसका परणामप पहल िन और िफर आकाशीय िवदयतउ होता ह

3 None of the above उपरो म स कोई भी नही

4 A high density cloud contains positively and negatively charged electric ions and when this interacts light andsound are simultaneously produced

एक उ घन बादल म धनाक और ऋणाक आविशत िवदयत आयन होत ह और जब यह परर भाव डालत ह तोकाश और िन एक साथ उािदत होती ह

Correct Answer -

A high density cloud contains positively and negatively charged electric ions and when this interacts light andsound are simultaneously produced

औ ि ि ो औ ो

93)

94)

95)

एक उ घन बादल म धनाक और ऋणाक आविशत िवदयत आयन होत ह और जब यह परर भाव डालत ह तोकाश और िन एक साथ उािदत होती ह

Doon Valley is able to grow rice because दन घाटी चावल उगान म सम ह ोिक

1 Other crops cannot be grown वहा अ फसलो को उगाया नही जा सकता ह

2 People in the valley are rice eaters घाटी म लोग चावल खान वाल ह

3 There is a huge export demand of rice वहा चावल की भारी िनयात माग ह

4 It has warm summer and snow melt waters for irrigation

वहा गिमया गम होती ह िसचाई क िलए बफ का िपघला आ पानी होता ह

Correct Answer -

It has warm summer and snow melt waters for irrigation

वहा गिमया गम होती ह िसचाई क िलए बफ का िपघला आ पानी होता ह

CANCELLED

In the geological time scale the Mesozoic Era DOES NOT contains which of the following periods

भगभय समय पमान पर मजीवी यग म िन कालो म स कौन नही ह

1 Triassic ट ाइऐिसक

2 Jurassic जरिसक

3 Cretaceous चाकमय

4 Carboniferous काबनी

Correct Answer -

Carboniferous काबनी

96)

1 P-3 Q-4 R-2 S-1

2 P-3 Q-4 R-1 S-2

3 P-3 Q-4 R-1 S-2

4 P-4 Q-3 R-2 S-1

Correct Answer -

P-4 Q-3 R-2 S-1

1 P-3 Q-1 R-4 S-2

2 P-3 Q-4 R-1 S-2

3 P-3 Q-2 R-4 S-1

97)

98)

4 P-2 Q-1 R-4 S-3

Correct Answer -

P-3 Q-4 R-1 S-2

1 P-3 Q-1 R-4 S-2

2 P-2 Q-3 R-4 S-1

3 P-2 Q-1 R-3 S-4

4 P-4 Q-2 R-1 S-3

Correct Answer -

P-3 Q-1 R-4 S-2

99)

100)

1 P-3 Q-2 R-4 S-1

2 P-1 Q-2 R-3 S-4

3 P-2 Q-3 R-1 S-4

4 P-4 Q-3 R-2 S-1

Correct Answer -

P-2 Q-3 R-1 S-4

ldquoHuman geography is the study of changing relationship between the unresting man and the unstableearthrdquo was defined by

lsquolsquoमानव भगोल ाकल आदमी और अथर पी क बीच सबध परवतन का अयन हrdquo ______ ारा परभािषत िकया गयाथा

1 J Brunches ज चस

2 EC Semple ईसी सल

3 HJ Mackinder एच ज मिकदर

4 PV Blache पीवी च

Correct Answer -

EC Semple ईसी सल

Sedimentary rocks are finally and ultimately derived from the____________

अवसादी चान अततः ________ स ा की जाती ह

1 action of earth movements पी की गितिविधयो

2 marine deposit समी िनप

3 weathering of metamorphic rocks पातरत चानो क अपय

4 weathering of igneous rocks आय चानो क अपय

Correct Answer -

weathering of igneous rocks आय चानो क अपय

Page 17: High School Teacher Eligibility Test- BOARD PROFESSIONAL ...peb.mp.gov.in/results/RESULT_18/HST_RES18/Final_anwser_key/HST… · M a ndl a / मंड ल ... Under the Madhya Pradesh

1)

2)

3)

4)

Topic- GEOGRAPHY

Which of the following ranges of population size is used to define Class-III city by Indian Census

भारतीय जनगणना ारा वग-III शहर को परभािषत करन क िलए िनिलखत म स िकस रज क जनसा आकारका उपयोग िकया जाता ह

1 20 000 to 49999 20 000 स 49999

2 30000 to 59999 30000 स 59999

3 24000 to 54999 24000 स 54999

4 50000 to 99999 50000 स 99999

Correct Answer -

20 000 to 49999 20 000 स 49999

Carbonaceous rocks which produce coal and oil belong to the category of rocks called_______

कोयल और तल का उादन करन वाली काबनय चान ______ नामक चानो की णी स सबिधत ह

1 metamorphic पातरत

2 sedimentary अवसादी

3 inorganic अजिवक

4 igneous आय

Correct Answer -

sedimentary अवसादी

The ruhr-complex is a major industrial centre in र-परसर िन का एक मख औोिगक क ह

1 North America उरी अमरका

2 Russia स

3 Germany जमनी

4 Europe यरोप

Correct Answer -

Germany जमनी

The term lsquoRegurrsquo refers to श lsquoरगरrsquo ______ स सबिधत ह

1 Deltaic alluvial soils डा जलोढ़ िमी

ि ी

5)

6)

2 Laterite soils लटराइट िमी

3 Red and yellow soils लाल और पीली िमी

4 Black cotton soils काली कपास िमी

Correct Answer -

Black cotton soils काली कपास िमी

Read the given statements and answer which of the following options isare correct

1 Lower the pressure greater the atmospheric disturbance

2 Air move from higher to low pressure

िदए गए कथन को पढ़ और उर द िक िन म स कौन सास िवक सही ह

1 िजतना दाब कम होगा वायमडलीय बाधाए उतनी अिधक होगी

2 वाय उ स िन दाब की ओर गित करती ह

1 Both Statements 1 and 2 are correct दोनो कथन 1 और 2 सही ह

2 Both Statements 1 and 2 are wrong दोनो कथन 1 और 2 गलत ह

3 Statement 1 is wrong and only Statement 2 is correct कथन 1 गलत ह और कवल कथन 2 सही ह

4 Statement 1 is correct and Statement 2 is wrong कथन 1 सही ह और कथन 2 गलत ह

Correct Answer -

Both Statements 1 and 2 are correct दोनो कथन 1 और 2 सही ह

CANCELLED

1 4 1 2 and 3 4 1 2 और 3

2 4 1 3 and 2 4 1 3 और 2

3 1 4 2 and 3 1 4 2 और 3

4 1 4 3 and 2 1 4 3 और 2

Correct Answer -

7)

8)

9)

1 4 3 and 2 1 4 3 और 2

CANCELLED

Karl Pearsonrsquos correlation co-efficient is काल िपयसन का सहसबध गणाक ह

1 Arithmec mean समार मा

2 Geometric mean गणोर मा

3 None of these इनम स कोई नही

4 Harmonic mean हराक मा

Correct Answer -

Geometric mean गणोर मा

CANCELLED

ldquoEach day is more or less the same the morning is clear and bright with a sea breeze as the Sun climbshigh in the sky heat mounts up dark clouds form then rain comes with thunder lighting But rain is soonoverrdquo Which of the following regions is described in the above passage

ldquoक िदन समान स अिधक या कम होता ह सम की हवा क साथ और उल सबह होती ह जस सयआकाश म ऊचा चढ़ता ह गम बढ़ जाती ह काल बादल बनत ह िफर िबजली क साथ बारश आती ह लिकनबारश जी ख हो जाती हlsquorsquo उपरो पा म िनिलखत म स िकन ो का वणन िकया गया ह

1 Equatorial भमरखीय

2 Equatorial भमरखीय

3 Savannah सवाना

4 Mediterranean आातरक (भमसागरीय)

5 Mediterranean आातरक (भमसागरीय)

6 Monsoon मानसन

7 Monsoon मानसन

Correct Answer -

Equatorial भमरखीय

Equatorial भमरखीय

CANCELLED

In which epoch of the geological history of the Earth dinosaurs reached their largest size

पी क भगभय इितहास क िकस यग म डायनासोर अपन सबस बड़ आकार तक पचि

10)

11)

1 Triassic ट ायिसक

2 Jurassic जरािसक

3 Cretaceous ीटशस

4 Permian पिमयन

Correct Answer -

Jurassic जरािसक

CANCELLED

A spring tide would occur in which of the following conditions

िनिलखत म स िकन थितयो म ार-भाटा आता ह

1 When the Sun Moon and Earth are in a straight line

जब सय चमा और पी एक सीधी रखा म होत ह

2 When the Moon and Earth are in right angle to each other

जब चमा और पी एक-दसर क दािहन कोण म होत ह

3 When the Earth and Moon are in right angle to the Sun

जब पी और चमा सय क दािहन कोण म होत ह

4 When the Sun and Moon are in right angle to each other

जब सय और चमा एक-दसर क दािहन कोण म होत ह

Correct Answer -

When the Sun Moon and Earth are in a straight line

जब सय चमा और पी एक सीधी रखा म होत ह

CANCELLED

An observe on the Earthrsquos surface always sees the same face of the moon because

एक पयवक को पी की सतह स हमशा चाद का एक ही फलक िदखाई दता ह ोिक

1 Its path of revolution around the earth is the same as that of the earth around the Sun

इसका पी क चारो ओर परमण का माग सय क चारो ओर पी क समान ही ह

2 Its period of revolution around the Earth is the same as its period of rotation around its own axis

इसकी पी क चारो ओर परमण की अविध उसकी अपनी धरी क चारो ओर घणन की अविध क समान ह

3 Its period of rotation is the same as that of the Earth इसकी घणन की अविध पी क समान ह

ी ि ी ी

12)

13)

14)

4 Its direct of rotation is the same as that of Earth घणन की िदशा पी क समान ही ह

Correct Answer -

Its period of revolution around the Earth is the same as its period of rotation around its own axis

इसकी पी क चारो ओर परमण की अविध उसकी अपनी धरी क चारो ओर घणन की अविध क समान ह

CANCELLED

The pebbles that are faceted by the sand-blasting and shaped polished by the wind abrasions are known as

पवन अपघषन ारा पॉिलश रत-िवोिटत और साच म ढला ककड़ ___________ क प म जाना जाता ह

1 Dreikanter िकोणक

2 Pediments िकोिनका

3 Inselberg इलबग

4 Dunes टीबा

Correct Answer -

Dreikanter िकोणक

CANCELLED

Astronomical unit is the average distance between खगोलीय इकाई ______ क बीच की औसत दरी ह

1 Earth and Mars पी और मगल

2 Earth and mercury पी और बध

3 Earth and moon पी और चमा

4 Earth and Sun पी और सय

Correct Answer -

Earth and Sun पी और सय

During cold weather season in the northern plains there will be an inflow of cyclonic disturbancesfrom the _________ directions

शीत मौसम क दौरान उरी मदानी इलाको म _________ िदशाओ स चवात सबधी गड़बड़ी का अतवाह होगा

1 East and Northwest पव और उरपिम

2 East and Northeast पव और पवर

3 West and East पिम और पव

ि औ ि

15)

16)

17)

4 West and Northwest पिम और उरपिम

Correct Answer -

West and Northwest पिम और उरपिम

During an earth quake the velocity of the body waves will________ along with the increase in densityof the material it is passing through

भकप क दौरान लहरो क ऊपरी भाग का वग घन म व क साथ-साथ ________ जो इसस गजरन वाली वको आग बढाएगी

1 not change नही बदलगा

2 increase initially and then decrease शआत म बढ़गा और िफर घटगा

3 increase बढ़गा

4 decrease घटगा

Correct Answer -

increase बढ़गा

The Clouded Leopard National park is situated in which of the following states

िनिलखत म िकस रा म धिमल तदआ रा ीय उान (ाउडड लपड नशनल पाक ) थत ह

1 Tripura िपरा

2 Uttar Pradesh उर दश

3 Assam असम

4 Mizoram िमजोरम

Correct Answer -

Tripura िपरा

Usually the land surfaces are heated more quickly than the water surfaces because _____________

आम तौर पर जल सतहो की तलना म भिम सतह अिधक तजी स गम होती ह ोिक _____________ ह

1 the specific heat of water is higher than land पानी की िविश ऊा भिम स अिधक

2 the specific heat of water is lesser than land पानी की िविश ऊा भिम स कम होती

3 the latent heat of water is higher than the land पानी की अतिनिहत ऊा भिम स अिधक

4 the land reflects more heat radiation than water भिम पानी की तलना म अिधक ऊा क िविकरण को पराविततकरती

18)

19)

20)

21)

Correct Answer -

the specific heat of water is higher than land पानी की िविश ऊा भिम स अिधक

The longest shore-line is along the state of सबस लबी समतटीय रखा िन रा क साथ ह

1 Maharashtra महारा

2 Orissa उड़ीसा

3 Kerala करल

4 Gujarat गजरात

Correct Answer -

Gujarat गजरात

The position when the Earth is farthest from the Sun is known as

जब पी सय स सबस दर होती ह तो उस थित को िन नाम स जाना जाता ह

1 Perihelion उपसौर

2 Vernal Equinox बसत िवषव

3 Aphelion अपसौर

4 Autumnal Equinox शराल िवषव

Correct Answer -

Aphelion अपसौर

The seasonal reversal of winds is the typical characteristic of

हवाओ का मौसमी परवतन ______ की सामा िवशषता ह

1 Mediterranean climates only कवल भमसागरीय जलवाय

2 All of the above climates उपय सभी मौसम

3 Monsoon climate only कवल मानसन जलवाय

4 Equatorial climate only कवल भमरखीय जलवाय

Correct Answer -

Monsoon climate only कवल मानसन जलवाय

In _________ rocks the minerals will occurs in beds or layers

ो ि ो ो

22)

23)

24)

______ चानो म खिनज तल या परतो म होत ह

1 metamorphic कायातरत

2 igneous and metamorphic आय और कायातरत

3 igneous आय

4 sedimentary अवसादी

Correct Answer -

sedimentary अवसादी

Black soil is ideal for the cultivation of cotton as कपास की खती क िलए काली िमी आदश ह ोिक

1 Its colour is black यह काली होती ह

2 It is found on plateau regions यह पठार ो म पायी जाती ह

3 It is made up of lava यह लावा स बनी होती ह

4 It can retain moisture यह नमी को बरकरार रख सकती ह

Correct Answer -

It can retain moisture यह नमी को बरकरार रख सकती ह

The National Survey and Mapping Organization of the country works under the Department of___________

दश का रा ीय सवण और मानिचण सगठन ___________ िवभाग क अतगत काय करता ह

1 Space अतर

2 Science and Technology िवान और तकनीक

3 Culture सित

4 Tourism पयटन

Correct Answer -

Science and Technology िवान और तकनीक

Palk strait separates India from पाक जलडमम भारत स _____ को अलग करता ह

1 Pakistan पािकान

2 Andaman Island अडमान ीप

3 China चीन

25)

26)

27)

4 Sri Lanka ीलका

Correct Answer -

Sri Lanka ीलका

Which among the following state is the major producer of Bauxite in India

िनिलखत म स कौन सा रा भारत म बॉाइट का मख उादक ह

1 Madhya Pradesh मदश

2 Rajasthan राजथान

3 Goa गोवा

4 Orissa उड़ीसा

Correct Answer -

Orissa उड़ीसा

Which of the following states DOES NOT share border with Chhattisgarh

िनिलखत म स कौन सा रा छीसगढ़ क साथ सीमा साझा नही करता ह

1 Telangana तलगाना

2 Uttar Pradesh उर दश

3 Bihar िबहार

4 Andhra Pradesh आ दश

Correct Answer -

Bihar िबहार

Which of the following statements is INCORRECT with respect to parallels of latitudes

अाश क समानातरो क सबध म िन निलखत म स कौन सा कथन गलत ह

1 A line joining places of equal latitude is known as parallel of largest

समान अाश क थानो को जोड़न वाली रखा को िवशालतम क समानातर क प म जाना जाता ह

2 They stat from equator and run parallels to it

व भम रखा स ारभ होत ह और इसक समानातर चलत ह

3 All parallels are equal in length सभी समातर लबाई म समान ह

4 All parallels are drawn as circles on the globe ोब पर सभी समानातर वो क प म खीच जात ह

28)

29)

30)

31)

Correct Answer -

All parallels are equal in length सभी समातर लबाई म समान ह

Which of the following Indian states is also known as a lsquoLand of Red river and Blue Hillsrsquo

िनिलखत म स िकस भारतीय रा को लाल नदी और नीली पहािड़यो की भिम क नाम स जाना जाता ह

1 Uttarkhand उराखड

2 Assam असम

3 Meghalaya मघालय

4 Arunachal Pradesh अणाचल दश

Correct Answer -

Assam असम

In spatial analysis of settlement Rn = 215 indicates which type of settlement arrangement

िनपटान क थािनक िवषण म Rn = 215 यह इिगत करता ह िक िकस कार की िनपटान वथा ह

1 Uniform यिनफॉम

2 Semi-Clustered समी- ल टर

3 Clustered ल टर

4 Random रडम

Correct Answer -

Uniform यिनफॉम

Who are known as the lsquoYellow Peoplersquo lsquoयलो पीपलrsquo क प म कौन जाना जाता ह

1 Mongoloids मोगोलोइडस

2 Nigroids नीोइडस

3 Australoids ऑ लॉइडस

4 Caucasoids कॉकसोइडस

Correct Answer -

Mongoloids मोगोलोइडस

ि ि ो ौ ि

32)

33)

34)

Who publishes the topographical map of India भारत क थलाकितक मानिच को कौन कािशत करता ह

1 Geographical Survey of India भारत का भौगोिलक सवण

2 Government of India भारत सरकार

3 Geological Survey of India भारत क भगभय सवण

4 Survey of India भारत का सवण

Correct Answer -

Survey of India भारत का सवण

Who among the following claimed geography to be the lsquoEcology of Manrsquo

िनिलखत म स िकसन भगोल को मन का पारथितकी कहा ह

1 Alfred Hener अड हटनर

2 Vidal-de la Blache वाइडल-िड लॉ ॉश

3 Oo Schluter ओटो टर

4 Harlan Barrow हरलन बारो

Correct Answer -

Harlan Barrow हरलन बारो

Who among the following is regarded as the founder of humanistic approach in geography

िनिलखत म स िकस भगोल म मानवतावादी िकोण का सथापक माना जाता ह

1 William Bunge िविलयम बग

2 Yi-Fu-Tuan यी-फ- यान

3 Brain JL Berry न जएल बरी

4 Richard Peet रचड पीट

Correct Answer -

Yi-Fu-Tuan यी-फ- यान

Who prepared Lorenz curve लोरज व िकसन तयार िकया

1 Geddes गडस

2 None of these इनम स कोई नही

3 Griffith Taylor ििफथ टलर

35)

36)

37)

4 Max U Lorenz म य लोरज

Correct Answer -

Max U Lorenz म य लोरज

Gulf Streams are the currents of which of the following oceans

खाड़ी की धाराए िनिलखत महासागरो म स िकसकी धाराए ह

1 North Atlantic Ocean उरी अटलािटक महासागर

2 North Pacific Ocean उरी शात महासागर

3 Arabian Sea अरब सागर

4 South Pacific Ocean दिण शात महासागर

Correct Answer -

North Atlantic Ocean उरी अटलािटक महासागर

Disintegration wearing away and removal of rock material is generally referred as

िशला पदाथ (रॉक सामी) का टटना िमटना और हटना आमतौर पर ________ क प म सदिभत िकया जाता ह

1 Shattering िवसकारक

2 Denudation अनाादन

3 Fault श

4 Decomposition िवयोजन

Correct Answer -

Denudation अनाादन

Variations in the length of day time and night from season to season are due to

मौसम स मौसम परवतन पर िदन क समय और रात क समय की अविध म िभताए िन कारण स होती ह

1 The Earthrsquos revolution round the Sun in an elliptical manner पी का दीघवाकार तरीक स सय क चारो घणन

2 The Earthrsquos rotation on its axis पी का इसकी धरी पर घणन

3 Revolution of the Earth on a tilted axis नत अ पर पी का घणन

4 Latitudinal position of the place थान की अाश थित

Correct Answer -

Revolution of the Earth on a tilted axis नत अ पर पी का घणन

38)

39)

40)

Point out the correct sequence of mountain ranges from north to south

उर स दिण तक पवत खलाओ क सही अनम को इिगत कर

1 Great Himalaya Middle Himalaya Outer Himalaya Trans Himalaya

महान िहमालय म िहमालय बा िहमालय परा िहमालय

2 Middle Himalaya Great Himalaya Trans Himalaya Outer Himalaya

म िहमालय महान िहमालय परा िहमालय बा िहमालय

3 Outer Himalaya Middle Himalaya Great Himalaya Trans Himalaya

बा िहमालय म िहमालय महान िहमालय परा िहमालय

4 Trans Himalaya Great Himalaya Middle Himalaya Outer Himalaya

परा िहमालय महान िहमालय म िहमालय बा िहमालय

Correct Answer -

Trans Himalaya Great Himalaya Middle Himalaya Outer Himalaya

परा िहमालय महान िहमालय म िहमालय बा िहमालय

Sunrsquos halo is produced by the refraction of light in सय का भामडल ______ म काश क अपवतन ाराउ होता ह

1 Ice crystals in Cirrro-Cumulus clouds पाभ-कपास मघो क बफ िल

2 Ice crystal in Cirrus clouds पाभमघो क बफ िल

3 Dust particles in Stratus clouds री मघो क धल कण

4 Water vapour in Stratus clouds री मघो क जल वा

Correct Answer -

Ice crystal in Cirrus clouds पाभमघो क बफ िल

Read the given statements and answer which of the following options isare correct

(1) The minerals present in the rocks exposed to atmosphere are not subjected to alteration

(2) Oxidation is one of the processes of chemical weathering

िदए गए कथनो को पढ़ और उर द िक िन म स कौन सास िवक सही ह

(1) वायमल स अनावत शल म उपथत खिनज परवतन क अधीन नही होता ह

(2) ऑीकरण रासायिनक अपय की ियाओ म स एक ह

ो ो

41)

42)

1 Both statements are wrong दोनो कथन गलत ह

2 Both statements are correct दोनो कथन सही ह

3 First statement is wrong and second statement is correct पहला कथन गलत ह और दसरा कथन सही ह

4 First statement is correct and second statement is wrong पहला कथन सही ह और दसरा कथन गलत ह

Correct Answer -

First statement is wrong and second statement is correct पहला कथन गलत ह और दसरा कथन सही ह

Read the given statements and answer which of the following options isare correct

1 Sunrsquos short waves enter the earth partially heating the atmosphere

2 Heated earth surface from the sun produces broader waves which interacts and heats the atmosphere

िदए गए कथन को पढ़ और उर द िक िन म स कौन सास िवक सही ह

1 सय की छोटी तरग पी म आिशक प स वश करती ह और वायमडल को ऊत करती ह

2 सय स ऊत पी की सतह िवारत तरग उ करती ह जो परर भाव डालती ह और वायमडल कोऊत करती ह

1 Both Statements 1 and 2 are correct दोनो कथन 1 और 2 सही ह

2 Both Statements 1 and 2 are wrong दोनो कथन 1 और 2 गलत ह

3 Statement 1 is wrong and only Statement 2 is correct कथन 1 गलत ह और कवल कथन 2 सही ह

4 Only statement 1 is correct कवल कथन 1 सही ह

Correct Answer -

Both Statements 1 and 2 are correct दोनो कथन 1 और 2 सही ह

Read the given statements and answer which of the following options isare correct

(1)The rocks that get changed due to heat and pressure are termed as metamorphic rocks

(2)Slate is one such type of metamorphic rock

िदए गए कथनो को पढ़ और उर द िक िन म स कौन सास िवक सही ह

(1) शल जो ऊा और दाब क कारण परवितत हो जात ह उ कायातरक शलो क प म जाना जाता ह

(2) ट एक तरह का कायातरक शल ह

1 Both statements are wrong दोनो कथन गलत ह

2 Both statements are correct दोनो कथन सही ह

3 First statement is wrong and second statement is correct पहला कथन गलत ह और दसरा कथन सही ह

ी औ

43)

44)

4 First statement is correct and second statement is wrong पहला कथन सही ह और दसरा कथन गलत ह

Correct Answer -

Both statements are correct दोनो कथन सही ह

Read the given statements and answer which of the following options isare correct

1 Higher temperature anomaly is observed in the northern hemisphere

2 Differential heating is absent in Northern Hemisphere

िदए गए कथनो को पढ़ और उर द िक िन म स कौन सास िवक सही ह

1 उरी गोलाध म उ तापमान िवसगित पायी जाती ह

2 उरी गोलाध म अतर ऊन अनपथत होती ह

1 Both Statements 1 and 2 are correct दोनो कथन 1 और 2 सही ह

2 Both Statements 1 and 2 are wrong दोनो कथन 1 और 2 गलत ह

3 Statement 1 is wrong and Statement 2 is correct कथन 1 गलत ह और कथन 2 सही ह

4 Statement 1 is correct and Statement 2 is wrong कथन 1 सही ह और कथन 2 गलत ह

Correct Answer -

Statement 1 is correct and Statement 2 is wrong कथन 1 सही ह और कथन 2 गलत ह

Read the given statements and answer which of the following options isare correct

(1) Plutonic rocks are intrusive type of igneous rocks

(2) It cools very slowly because the surrounding rock serves as insulation around the intrusion of magma

िदए गए कथनो को पढ़ और उर द िक िन म स कौन सास िवक सही ह

(1) िवतलीय शल अतवधी कार क आश शल ह

(2) यह बत धीर-धीर ठडा होता ह ोिक आस-पास क शल मा क अतवधन क चारो ओर रोधन क प म कायकरत ह

1 Both statements are wrong दोनो कथन गलत ह

2 Both statements are correct दोनो कथन सही ह

3 First statement is wrong and second statement is correct पहला कथन गलत ह और दसरा कथन सही ह

4 First statement is correct and second statement is wrong पहला कथन सही ह और दसरा कथन गलत ह

Correct Answer -

Both statements are correct दोनो कथन सही ह

45)

46)

47)

48)

The dust and ash material hurled from the volcanoes are termed as

ालामखी स िनकलन वाली धल और राख सामी को _______ क प म कहा जाता ह

1 Pyroclasc पाइरोाक

2 Hyperclastic हाइपराक

3 Hepiroclastic हिपरोाक

4 Cirroclastic िसरोाक

Correct Answer -

Pyroclasc पाइरोाक

The vertical difference in elevation between a low tide and high tide is referred as

कम ार और उ ार क बीच ऊचाई म लबवत अतर _____ स सदिभत होता ह

1 Tidal slope ारीय ढलान

2 Tidal elevation ारीय उयन

3 Tidal range ारीय परास

4 Tidal height ारीय ऊचाई

Correct Answer -

Tidal range ारीय परास

The maximum biodiversity is found in which of the following regions िनिलखत ो म स अिधकतमजव िविवधता िकसम पायी जाती ह

1 Amazon Basin अमज़न बिसन

2 East Indies ई इडीज

3 Congo Basin कागो बिसन

4 West indies व इडीज

Correct Answer -

Amazon Basin अमज़न बिसन

The cultivation of rice crop produces_______ चावल की फसल की खती ______ का उादन करती ह

1 SO2

49)

50)

51)

2 CH4

3 CFCs

4 CO2

Correct Answer -

CH4

The pressure system with higher pressure at the centre is called__________

क म उ दबाव वाली दबाव णाली को _______ कहा जाता ह

1 front अ

2 depression अवनमन

3 cyclone चवात

4 anti-cyclone ितचवात

Correct Answer -

anti-cyclone ितचवात

The Himalayan region is poor in mineral resources because िहमालयी खिनज ससाधनो म समनही ह ोिक

1 The displacement of rock strata has disturbed the arrangement of rocks and made it complex

शलीय परत क िवथापन न चानो की वथा को अवथत कर िदया ह और इस जिटल बना िदया ह

2 The climate conditions are not suitable for exploitation of minerals

जलवाय की थित खिनजो क दोहन क िलए उपय नही ह

3 The terrain makes explanation of minerals difficult and very costly due to transportation difficulties

भ-भाग परवहन की किठनाइयो क कारण खिनजो का दोहन मल और बत महगा बना दता ह

4 It is made up of crystalline rocks यह िलीय चानो स बना ह

Correct Answer -

The displacement of rock strata has disturbed the arrangement of rocks and made it complex

शलीय परत क िवथापन न चानो की वथा को अवथत कर िदया ह और इस जिटल बना िदया ह

The process through which the moisture is added to the atmosphere by vegetation is termed as

वह िया िजसक माम स वनित ारा वातावरण म नमी िमलायी जाती ह _______ क प म जानी जाती ह

52)

53)

54)

1 Condensation सघनन

2 Evapotranspiration वान-उजन

3 Radiation िविकरण

4 Precipitation वषण

Correct Answer -

Evapotranspiration वान-उजन

The process through which the terrestrial heat is transferred to air by direct contact is termed as

वह िया िजसम सपक ारा थलीय ऊा वाय म थानातरत हो जाती ह ______ क प म जानी जाती ह

1 Conduction चालन

2 Convection सवहन

3 Insolation आतपन

4 Radiation िविकरण

Correct Answer -

Conduction चालन

The largest area under mangroves is in which of the following statesunion territory

मोव क अतगत िनिलखत राोसघ शािसत दशो म स सबस बड़ा कौन सा ह

1 Andaman and Nicobar अमान और िनकोबार

2 Andhra Pradesh आ दश

3 West Bengal पिम बगाल

4 Gujarat गजरात

Correct Answer -

West Bengal पिम बगाल

The longitudinal transverse and surface waves in an earthquake originate from

भकप म दशातर अनथ और सतह तरग यहा उ होती ह

1 The focus on the surface of the Earth पी क सतह पर क -िबद म

2 The focus within the body of the Earth पी क भीतर क -िबद म

3 The epicenter within the body of the Earth पी क भीतर उपरक म

55)

56)

57)

4 The epicenter on the surface of the Earth पी क सतह पर उपरक म

Correct Answer -

The focus within the body of the Earth पी क भीतर क -िबद म

The down slope movement of material due to gravity is called______

गाकषण क कारण पदाथ की अनढाल गित को ______ कहा जाता ह

1 mass movement पदाथ सचलन

2 deposition िनप

3 erosion रण

4 volcanic movement ालामखीय सचलन

Correct Answer -

mass movement पदाथ सचलन

Shimla is cooler than Amritsar although both are on the same latitude This is because

िशमला म अमतसर स अिधक ठड ह हालािक दोनो समान अाश पर ह ऐसा ह ोिक

1 Shimla is at a greater height above sea level than Amritsar अमतसर की तलना म िशमला सम तल स अिधकऊचाई पर ह

2 Shimla is further north िशमला उर की ओर ह

3 Shimla is farther from the equator िशमला भम रखा स आग ह

4 Their longitudes differ उनकी दशातर रखाए िभ ह

Correct Answer -

Shimla is at a greater height above sea level than Amritsar अमतसर की तलना म िशमला सम तल स अिधकऊचाई पर ह

lsquoTempo of Urbanizationrsquo measures which of the following

lsquoशहरीकरण का टपोrsquo िनिलखत म स कौन सा उपाय ह

1 Speed of urbanizaon शहरीकरण की गित

2 None of the above इनम स कोई नही

3 Inequality of urbanizaon शहरीकरण की असमानता

4 Current level of urbanizaon शहरीकरण का वतमान र

Correct Answer -

58)

59)

60)

Speed of urbanizaon शहरीकरण की गित

Out of the following options choose the INCORRECT statement

िनिलखत िवको म स गलत कथन का चयन कर

1 The clear tracts in the equatorial region recover rapidly भम रखा म भभाग तजी स ठीक हो जात ह

2 The stable communities include a redwood forest a pine forest at high elevations

थर समदायो म एक रडवड वन उ ऊचाई पर एक दवदार वन शािमल ह

3 Any ecosystem moves towards maximum biomass and stability to survive

कोई भी पारथितकी त जीिवत रहन क िलए अिधकतम जवसहित और थरता की तरफ असर होता ह

4 Tropical rain forests near equator are stable ecosystems

भम रखा क पास उकिटबधीय वषा वन थर पारथितक त ह

Correct Answer -

The clear tracts in the equatorial region recover rapidly भम रखा म भभाग तजी स ठीक हो जात ह

Seasonal contrasts are maximum in मौसमी िवषमता अिधकतम ह

1 Mid latitudes म अाश म

2 Low attitudes िन अाश म

3 High latitudes उ अाश म

4 Subtropics उपोकिटबधीय म

Correct Answer -

Mid latitudes म अाश म

In India which type of forest among the following occupies the largest area

भारत म िनिलखत म स िकस कार क वन सबस बड़ा फल आािदत करत ह

1 Sub-tropical Dry Evergreen Forest उप उकिटबधीय श सदाबहार वन

2 Mountain Wet Temperate Forest पवतीय आ शीतो वन

3 Tropical Moist Deciduous Forest उकिटबधीय आ पणपाती वन

4 Tropical Wet Evergreen Forest उकिटबधीय आ सदाबहार वन

Correct Answer -

Tropical Moist Deciduous Forest उकिटबधीय आ पणपाती वन

61)

62)

63)

64)

What is the proportion of lsquoJuvenile Populationrsquo (0-14 years) in India as per 2011Census

2011 की जनगणना क अनसार भारत म जवनाइल पॉपलशन यानी िकशोर जनस या (0-14 वष) का अनपात ाह

1 3076 of total population कल जनस या का 3076

2 2764 of total population कल जनस या का 2764

3 2933 of total population कल जनस या का 2933

4 3354 of total population कल जनस या का 3354

Correct Answer -

3076 of total population कल जनस या का 3076

What is the Belfast famous for बलफा िकसक िलए मशर ह

1 Belt of cotton textile industry कपास व उोग क

2 Ship-building industry जहाज िनमाण उोग

3 Agricultural machinery किष उपकरण

4 Aero planes manufacturing वाययान िनमाण

Correct Answer -

Ship-building industry जहाज िनमाण उोग

What is the most important occupation in tropical monsoon lands

उकिटबधीय मॉनसन भिम म सबस महपण वसाय ा ह

1 Mining खनन

2 Cattle rearing मवशी पालन

3 Agriculture किष

4 Nomadic herding नोमािडक जड़ी-बिटया

Correct Answer -

Agriculture किष

What is the most important characteristics of the islands (Indian) located in the Arabian sea

अरब सागर म थत ीपो (भारतीय) की सबस महपण िवशषता ा ह

ी ो

65)

66)

67)

1 There are all of coral origins सभी कोरल मल क ह

2 There are all very small in size य सभी आकार म बत छोट ह

3 They have a very dry climate इनकी जलवाय बत श ह

4 They are extended parts of the mainland व महाीप क िवारत िह ह

Correct Answer -

There are all of coral origins सभी कोरल मल क ह

What do the basalt layers of the Deccan indicate डन की बसा परत ा इिगत करती ह

1 All of the above उपरो सभी

2 Huge volcanic eruptions in the distant past दरथ अतीत म िवशाल ालामखीय िवोट

3 The immense erosional activity of the rivers निदयो की िवशाल रण गितिविध

4 The influence of weathering मौसम का भाव

Correct Answer -

Huge volcanic eruptions in the distant past दरथ अतीत म िवशाल ालामखीय िवोट

In the structure of planet Earth below the mantle the core is mainly made up of_____

पी ह की सरचना म मटल क नीच कोर म प स______ स िनिमत होती ह

1 aluminium एमीिनयम

2 silicon िसिलकॉन

3 chromium ोिमयम

4 iron लोहा

Correct Answer -

iron लोहा

One of the major Mid Oceanic Ridge is found in मख म-महासागर चोिटयो म स एक ______ म पायाजाता ह

1 Mid Pacific Ocean म शात महासागर

2 Mid Atlantic Ocean म अटलािटक महासागर

3 Mid Indian Ocean म भारतीय महासागर

4 Mid Arctic Ocean म आक िटक महासागर

68)

69)

70)

71)

Correct Answer -

Mid Atlantic Ocean म अटलािटक महासागर

Magma that reaches the Earthrsquos surface and then solidifies is called________

मा जो पी की सतह तक पचती ह और िफर ठोस हो जाती ह ________कहलाती ह

1 quartz ाटज

2 lava लावा

3 granite नाइट

4 silicates िसिलकट

Correct Answer -

lava लावा

Isotherms are the lines of equal_______ समताप रखाए समान _______की रखाए होती ह

1 pressure दाब

2 temperature तापमान

3 rainfall वषा

4 height ऊचाई

Correct Answer -

temperature तापमान

Mark the correct sequence of passes in the Western Ghats from north to south

पिमी घाटो म उर स दिण तक दर क सही अनम को िचित कर

1 Thalghat Palghat Bhorghat थलगघाट पालघाट भोरघाट

2 Thalghat Bhorghat Palghat थलघाट भोरघाट पालघाट

3 Bhorghat Thalghat Palghat भोरघाट थलघाट पालघाट

4 Palghat Bhorghat Thalghat पालघाट भोरघाट थलघाट

Correct Answer -

Thalghat Bhorghat Palghat थलघाट भोरघाट पालघाट

Which of the following does not have influence over the climate in India

ि ि ि ी ी

72)

73)

िनिलखत म स िकसका भाव भारत की जलवाय पर नही पड़ता ह

1 Ocean currents सागर की लहर

2 Nearness to equator भम रखा स िनकटता

3 Monsoons मानसन

4 Presence of Indian ocean भारतीय महासागर की उपथित

Correct Answer -

Ocean currents सागर की लहर

Which of the following cloud types has the characteristics like vertical tall narrow and puffy

िनिलखत म स िकस कार क मघो म लबवत लबी सकीण और थलता जसी िवशषताए ह

1 Cumulonimbus तफानी मघ

2 Cumulus मघ पज

3 Cirrocumulus पाभ कपासी मघ

4 Nimbostratus वषारी मघ

Correct Answer -

Cumulus मघ पज

Which of the following statement is INCORRECT about Crude Birth Rate

िनिलखत स कौन सा कथन अशोिधत ज दर क बार म सही नही ह

1 It cannot be used for comparing fertility level between two countries with different population characteristics

इसका उपयोग िविभ जनसा िवशषताओ वाल दो दशो क बीच जनन र की तलना क िलए नही िकया जा सकता ह

2 It is a standardized measure of fertility

यह जनन मता का मानकीकत उपाय ह

3 It is effected by the age-sex composition of the population

यह आबादी की आय-िलग सरचना स भािवत होता ह

4 It is expressed per 1000 population in a given geographical unit

यह िकसी दी गई भौगोिलक इकाई म ित 1000 जनसा पर िकया जाता ह

Correct Answer -

It is a standardized measure of fertility

यह जनन मता का मानकीकत उपाय ह

74)

75)

76)

77)

Which of the following state in India experienced negative decadal growth rate during 2001 to 2011census

भारत म िनिलखत म स िकस रा म वष 2001 स 2011 की जनगणना क दौरान नकाराक िगरावट दर ई

1 Tripura िपरा

2 Nagaland नागालड

3 Haryana हरयाणा

4 Odisha ओिडसा

Correct Answer -

Nagaland नागालड

Which of the following is NOT a characteristic of peninsular rivers

िनिलखत म स कौन सी िवशषता ायीपीय निदयो म नही होती ह

1 Flow through shallow valleys उथल घािटयो क माम स वाह

2 Seasonal flow मौसमी वाह

3 Little erosional activity थोड़ी कटावदार गितिविध

4 Meandering tendency often shifting their beds घमावदार वि अर अपन तटो को थानातरत करना

Correct Answer -

Meandering tendency often shifting their beds घमावदार वि अर अपन तटो को थानातरत करना

Which of the following gases in the atmosphere absorbs heat from the Sunrsquos radiation and the Earthssurface

वायमडल म िनिलखत म स कौन सी गस सय क िविकरण और पी की सतह स ऊा को अवशोिषत करती ह

1 Neon िनयॉन

2 Carbon dioxide काबन डाइऑाइड

3 Argon आगन

4 Nitrogen नाइट ोजन

Correct Answer -

Carbon dioxide काबन डाइऑाइड

Which of the following kind of settlement pattern is found at the confluence of rivers

ि ि ि ि ो

78)

79)

80)

िनिलखत म स िकस कार का वथापन पटन निदयो क सगम पर पाया जाता ह

1 Triangular Paern िकोणीय पटन

2 Circular or Semi-Circular Paern परप या अध-परप पटन

3 Nebular Paern नबलर पटन

4 Star ndashShaped Paern ार-आकार का पटन

Correct Answer -

Triangular Paern िकोणीय पटन

Which one was not the objective of the Biosphere Reserve Projects launched by the UNESCO

यनो ारा श की गई सरित जवमडल परयोजनाओ का उ इनम स कौन सा नही था

1 To promote teaching and research िशण और अनसधान को बढ़ावा दना

2 To make agriculture sustainable किष को दीघकािलक बनाना

3 To conserve ecosystems पारथितक त को सरित करना

4 To conserve genetic diversity for a longtime लब समय तक अनवािशक िविवधता को सरित करना

Correct Answer -

To make agriculture sustainable किष को दीघकािलक बनाना

Which region of the Earth surface receives the highest amount of insulation

पी सतह का कौन सा तापावरोधन की उतम माा ा करता ह

1 Land mass थलखड

2 Savannah region सवाना

3 Water bodies जल िनकाय

4 Tropical desert उकिटबधीय रिगान

Correct Answer -

Tropical desert उकिटबधीय रिगान

Which one of the following is not a biodiversity hotspot

िनिलखत म स कौन सा जव िविवधता का म जगह नही ह

1 Eastern Himalaya पव िहमालय

2 Eastern Ghats पव घाट

81)

82)

83)

3 Indo-Myanmar भारत-ामार

4 Westerm Ghats पिमी घाट

Correct Answer -

Eastern Ghats पव घाट

Which one of the following is NOT a part of the World Network of Biosphere Reserves based on theUNESCO Man and Biosphere Programme

यनो मन और बायोीयर कायम क आधार पर िनिलखत म स कौन बायोीयर रजव क िव नटवक कािहा नही ह

1 Gulf of Mannar मार की खाड़ी

2 Seshachalam शषाचलम

3 Sunderban सदरबन

4 Nilgiri नीलिगर

Correct Answer -

Seshachalam शषाचलम

Which one of the following is an example of ldquodesert vegetationrdquo

िनिलखत म स कौन मथलीय वनित का एक उदाहरण ह

1 Mosses and lichens दलदल और शवाल

2 Temperate grassland समशीतो घास क मदान

3 Coniferous forest शकधारी वन

4 Acacia and cactus एकािसया और कस

Correct Answer -

Acacia and cactus एकािसया और कस

Which one of the following reflects more sunlight िनिलखत म स कौन सा सय की रोशनी को अिधकपरावितत करता ह

1 Paddy crop land धान फसल भिम

2 Land covered with fresh snow ताजा बफ स आािदत भिम

3 Sand desert रतीली रिगान

4 Prairie land यरी भिम

84)

85)

86)

87)

Correct Answer -

Land covered with fresh snow ताजा बफ स आािदत भिम

Which layer of the atmosphere is in contact with the surface of the earthrsquos oceans

वायमडल की कौन सी परत पी क महासागरो की सतह क सपक म ह

1 Stratosphere समताप मडल

2 Mesosphere म मडल

3 Hydrosphere जलमडल

4 Troposphere ोभ मडल

Correct Answer -

Troposphere ोभ मडल

Mediterranean Sea is a border of which of the following countries भम सागर िनिलखत दशो म सिकसकी सीमा ह

1 None of these इनम स कोई नही

2 Iraq इराक

3 Lebanon लबनान

4 Jordan जॉडन

Correct Answer -

Lebanon लबनान

Benguela ocean currents are found along which coast बगएला महासागर धाराए िकस तट क साथ पायीजाती ह

1 East Coast of South America दिण अमरका क पव तट

2 East Coast of Africa अीका क पव तट

3 West Coast of South America दिण अमरका क पिमी तट

4 West Coast of Africa अीका क पिमी तट

Correct Answer -

West Coast of Africa अीका क पिमी तट

88)

89)

90)

Due to tension a block of land on one side being pushed up or upthrown relative to the downthrown blockis referred as

तनाव क कारण नीच फ क ए खड क साप भिम का एक खड एक ओर स ऊपर धकला जाता ह या ऊपर की ओरफ का जाता ह यह _____ क प म सदिभत ह

1 Thrust fault प श

2 Normal fault सामा श

3 Reverse fault म श

4 Strike slip fault नितलब सपण श

Correct Answer -

Normal fault सामा श

Inter-tropical doldrums is a zone of ______ अतर-उकिटबधीय डोलड ______ का एक ह

1 Frontolysis टोलायिसस

2 Convergence अिभसरण

3 Inter-tropical divergence zone अतर-उकिटबधीय िवचलन

4 Local wind थानीय वाय

Correct Answer -

Convergence अिभसरण

The Horse Latitudes are regions located at about _____ north and south of the equator

हॉस अाश भम रखा क उर और दिण म लगभग _____ पर थत ह

1 30ndash60 degree Latitude 30-60 िडी अाश

2 0ndash5 degree Latitude 0-5 िडी अाश

3 30 degree Latitude 30 िडी अाश

4 60ndash90 degree Latitude 60-90 िडी अाश

Correct Answer -

30 degree Latitude 30 िडी अाश

Generally evaporation is high over which part of the Earth

आम तौर पर पी क िकस भाग पर वाीकरण अिधक होता ह

1 Equatorial maritime भमवत समीय ी ी

91)

92)

2 Equatorial continental भमवत महाीपीय

3 Polar maritime वीय समीय

4 Polar continental वीय महाीपीय

Correct Answer -

Equatorial maritime भमवत समीय

A very high temperature during summer in north western India leads to what type of climaticcondition in south

उर पिमी भारत म गम क दौरान बत अिधक तापमान होन क कारण दिण म िकस कार की जलवाय थितउ करता ह

1 Depression over arabian sea अरब सागर पर अवनमन

2 Failure monsoon मानसन िवफलता

3 Successful monsoon मानसन सफलता

4 Cyclones चवात

Correct Answer -

Successful monsoon मानसन सफलता

Lightning and thunder are the resultant effect when तिड़त और गजन परणामी भाव ह जब

1 Two massive clouds hit powerfully each other first lightning is produced and later sound is produced

दो बड़ बादल एक दसर स शशाली ढग स टकरात ह पहल आकाशीय िवदयत उ होता ह और बाद म िन उहोती ह

2 Two massive clouds come into contact with the powerful wind collision this results into first sound and thenlightning

दो बड़ बादल शशाली पवन सघ क सपक म आत ह इसका परणामप पहल िन और िफर आकाशीय िवदयतउ होता ह

3 None of the above उपरो म स कोई भी नही

4 A high density cloud contains positively and negatively charged electric ions and when this interacts light andsound are simultaneously produced

एक उ घन बादल म धनाक और ऋणाक आविशत िवदयत आयन होत ह और जब यह परर भाव डालत ह तोकाश और िन एक साथ उािदत होती ह

Correct Answer -

A high density cloud contains positively and negatively charged electric ions and when this interacts light andsound are simultaneously produced

औ ि ि ो औ ो

93)

94)

95)

एक उ घन बादल म धनाक और ऋणाक आविशत िवदयत आयन होत ह और जब यह परर भाव डालत ह तोकाश और िन एक साथ उािदत होती ह

Doon Valley is able to grow rice because दन घाटी चावल उगान म सम ह ोिक

1 Other crops cannot be grown वहा अ फसलो को उगाया नही जा सकता ह

2 People in the valley are rice eaters घाटी म लोग चावल खान वाल ह

3 There is a huge export demand of rice वहा चावल की भारी िनयात माग ह

4 It has warm summer and snow melt waters for irrigation

वहा गिमया गम होती ह िसचाई क िलए बफ का िपघला आ पानी होता ह

Correct Answer -

It has warm summer and snow melt waters for irrigation

वहा गिमया गम होती ह िसचाई क िलए बफ का िपघला आ पानी होता ह

CANCELLED

In the geological time scale the Mesozoic Era DOES NOT contains which of the following periods

भगभय समय पमान पर मजीवी यग म िन कालो म स कौन नही ह

1 Triassic ट ाइऐिसक

2 Jurassic जरिसक

3 Cretaceous चाकमय

4 Carboniferous काबनी

Correct Answer -

Carboniferous काबनी

96)

1 P-3 Q-4 R-2 S-1

2 P-3 Q-4 R-1 S-2

3 P-3 Q-4 R-1 S-2

4 P-4 Q-3 R-2 S-1

Correct Answer -

P-4 Q-3 R-2 S-1

1 P-3 Q-1 R-4 S-2

2 P-3 Q-4 R-1 S-2

3 P-3 Q-2 R-4 S-1

97)

98)

4 P-2 Q-1 R-4 S-3

Correct Answer -

P-3 Q-4 R-1 S-2

1 P-3 Q-1 R-4 S-2

2 P-2 Q-3 R-4 S-1

3 P-2 Q-1 R-3 S-4

4 P-4 Q-2 R-1 S-3

Correct Answer -

P-3 Q-1 R-4 S-2

99)

100)

1 P-3 Q-2 R-4 S-1

2 P-1 Q-2 R-3 S-4

3 P-2 Q-3 R-1 S-4

4 P-4 Q-3 R-2 S-1

Correct Answer -

P-2 Q-3 R-1 S-4

ldquoHuman geography is the study of changing relationship between the unresting man and the unstableearthrdquo was defined by

lsquolsquoमानव भगोल ाकल आदमी और अथर पी क बीच सबध परवतन का अयन हrdquo ______ ारा परभािषत िकया गयाथा

1 J Brunches ज चस

2 EC Semple ईसी सल

3 HJ Mackinder एच ज मिकदर

4 PV Blache पीवी च

Correct Answer -

EC Semple ईसी सल

Sedimentary rocks are finally and ultimately derived from the____________

अवसादी चान अततः ________ स ा की जाती ह

1 action of earth movements पी की गितिविधयो

2 marine deposit समी िनप

3 weathering of metamorphic rocks पातरत चानो क अपय

4 weathering of igneous rocks आय चानो क अपय

Correct Answer -

weathering of igneous rocks आय चानो क अपय

Page 18: High School Teacher Eligibility Test- BOARD PROFESSIONAL ...peb.mp.gov.in/results/RESULT_18/HST_RES18/Final_anwser_key/HST… · M a ndl a / मंड ल ... Under the Madhya Pradesh

5)

6)

2 Laterite soils लटराइट िमी

3 Red and yellow soils लाल और पीली िमी

4 Black cotton soils काली कपास िमी

Correct Answer -

Black cotton soils काली कपास िमी

Read the given statements and answer which of the following options isare correct

1 Lower the pressure greater the atmospheric disturbance

2 Air move from higher to low pressure

िदए गए कथन को पढ़ और उर द िक िन म स कौन सास िवक सही ह

1 िजतना दाब कम होगा वायमडलीय बाधाए उतनी अिधक होगी

2 वाय उ स िन दाब की ओर गित करती ह

1 Both Statements 1 and 2 are correct दोनो कथन 1 और 2 सही ह

2 Both Statements 1 and 2 are wrong दोनो कथन 1 और 2 गलत ह

3 Statement 1 is wrong and only Statement 2 is correct कथन 1 गलत ह और कवल कथन 2 सही ह

4 Statement 1 is correct and Statement 2 is wrong कथन 1 सही ह और कथन 2 गलत ह

Correct Answer -

Both Statements 1 and 2 are correct दोनो कथन 1 और 2 सही ह

CANCELLED

1 4 1 2 and 3 4 1 2 और 3

2 4 1 3 and 2 4 1 3 और 2

3 1 4 2 and 3 1 4 2 और 3

4 1 4 3 and 2 1 4 3 और 2

Correct Answer -

7)

8)

9)

1 4 3 and 2 1 4 3 और 2

CANCELLED

Karl Pearsonrsquos correlation co-efficient is काल िपयसन का सहसबध गणाक ह

1 Arithmec mean समार मा

2 Geometric mean गणोर मा

3 None of these इनम स कोई नही

4 Harmonic mean हराक मा

Correct Answer -

Geometric mean गणोर मा

CANCELLED

ldquoEach day is more or less the same the morning is clear and bright with a sea breeze as the Sun climbshigh in the sky heat mounts up dark clouds form then rain comes with thunder lighting But rain is soonoverrdquo Which of the following regions is described in the above passage

ldquoक िदन समान स अिधक या कम होता ह सम की हवा क साथ और उल सबह होती ह जस सयआकाश म ऊचा चढ़ता ह गम बढ़ जाती ह काल बादल बनत ह िफर िबजली क साथ बारश आती ह लिकनबारश जी ख हो जाती हlsquorsquo उपरो पा म िनिलखत म स िकन ो का वणन िकया गया ह

1 Equatorial भमरखीय

2 Equatorial भमरखीय

3 Savannah सवाना

4 Mediterranean आातरक (भमसागरीय)

5 Mediterranean आातरक (भमसागरीय)

6 Monsoon मानसन

7 Monsoon मानसन

Correct Answer -

Equatorial भमरखीय

Equatorial भमरखीय

CANCELLED

In which epoch of the geological history of the Earth dinosaurs reached their largest size

पी क भगभय इितहास क िकस यग म डायनासोर अपन सबस बड़ आकार तक पचि

10)

11)

1 Triassic ट ायिसक

2 Jurassic जरािसक

3 Cretaceous ीटशस

4 Permian पिमयन

Correct Answer -

Jurassic जरािसक

CANCELLED

A spring tide would occur in which of the following conditions

िनिलखत म स िकन थितयो म ार-भाटा आता ह

1 When the Sun Moon and Earth are in a straight line

जब सय चमा और पी एक सीधी रखा म होत ह

2 When the Moon and Earth are in right angle to each other

जब चमा और पी एक-दसर क दािहन कोण म होत ह

3 When the Earth and Moon are in right angle to the Sun

जब पी और चमा सय क दािहन कोण म होत ह

4 When the Sun and Moon are in right angle to each other

जब सय और चमा एक-दसर क दािहन कोण म होत ह

Correct Answer -

When the Sun Moon and Earth are in a straight line

जब सय चमा और पी एक सीधी रखा म होत ह

CANCELLED

An observe on the Earthrsquos surface always sees the same face of the moon because

एक पयवक को पी की सतह स हमशा चाद का एक ही फलक िदखाई दता ह ोिक

1 Its path of revolution around the earth is the same as that of the earth around the Sun

इसका पी क चारो ओर परमण का माग सय क चारो ओर पी क समान ही ह

2 Its period of revolution around the Earth is the same as its period of rotation around its own axis

इसकी पी क चारो ओर परमण की अविध उसकी अपनी धरी क चारो ओर घणन की अविध क समान ह

3 Its period of rotation is the same as that of the Earth इसकी घणन की अविध पी क समान ह

ी ि ी ी

12)

13)

14)

4 Its direct of rotation is the same as that of Earth घणन की िदशा पी क समान ही ह

Correct Answer -

Its period of revolution around the Earth is the same as its period of rotation around its own axis

इसकी पी क चारो ओर परमण की अविध उसकी अपनी धरी क चारो ओर घणन की अविध क समान ह

CANCELLED

The pebbles that are faceted by the sand-blasting and shaped polished by the wind abrasions are known as

पवन अपघषन ारा पॉिलश रत-िवोिटत और साच म ढला ककड़ ___________ क प म जाना जाता ह

1 Dreikanter िकोणक

2 Pediments िकोिनका

3 Inselberg इलबग

4 Dunes टीबा

Correct Answer -

Dreikanter िकोणक

CANCELLED

Astronomical unit is the average distance between खगोलीय इकाई ______ क बीच की औसत दरी ह

1 Earth and Mars पी और मगल

2 Earth and mercury पी और बध

3 Earth and moon पी और चमा

4 Earth and Sun पी और सय

Correct Answer -

Earth and Sun पी और सय

During cold weather season in the northern plains there will be an inflow of cyclonic disturbancesfrom the _________ directions

शीत मौसम क दौरान उरी मदानी इलाको म _________ िदशाओ स चवात सबधी गड़बड़ी का अतवाह होगा

1 East and Northwest पव और उरपिम

2 East and Northeast पव और पवर

3 West and East पिम और पव

ि औ ि

15)

16)

17)

4 West and Northwest पिम और उरपिम

Correct Answer -

West and Northwest पिम और उरपिम

During an earth quake the velocity of the body waves will________ along with the increase in densityof the material it is passing through

भकप क दौरान लहरो क ऊपरी भाग का वग घन म व क साथ-साथ ________ जो इसस गजरन वाली वको आग बढाएगी

1 not change नही बदलगा

2 increase initially and then decrease शआत म बढ़गा और िफर घटगा

3 increase बढ़गा

4 decrease घटगा

Correct Answer -

increase बढ़गा

The Clouded Leopard National park is situated in which of the following states

िनिलखत म िकस रा म धिमल तदआ रा ीय उान (ाउडड लपड नशनल पाक ) थत ह

1 Tripura िपरा

2 Uttar Pradesh उर दश

3 Assam असम

4 Mizoram िमजोरम

Correct Answer -

Tripura िपरा

Usually the land surfaces are heated more quickly than the water surfaces because _____________

आम तौर पर जल सतहो की तलना म भिम सतह अिधक तजी स गम होती ह ोिक _____________ ह

1 the specific heat of water is higher than land पानी की िविश ऊा भिम स अिधक

2 the specific heat of water is lesser than land पानी की िविश ऊा भिम स कम होती

3 the latent heat of water is higher than the land पानी की अतिनिहत ऊा भिम स अिधक

4 the land reflects more heat radiation than water भिम पानी की तलना म अिधक ऊा क िविकरण को पराविततकरती

18)

19)

20)

21)

Correct Answer -

the specific heat of water is higher than land पानी की िविश ऊा भिम स अिधक

The longest shore-line is along the state of सबस लबी समतटीय रखा िन रा क साथ ह

1 Maharashtra महारा

2 Orissa उड़ीसा

3 Kerala करल

4 Gujarat गजरात

Correct Answer -

Gujarat गजरात

The position when the Earth is farthest from the Sun is known as

जब पी सय स सबस दर होती ह तो उस थित को िन नाम स जाना जाता ह

1 Perihelion उपसौर

2 Vernal Equinox बसत िवषव

3 Aphelion अपसौर

4 Autumnal Equinox शराल िवषव

Correct Answer -

Aphelion अपसौर

The seasonal reversal of winds is the typical characteristic of

हवाओ का मौसमी परवतन ______ की सामा िवशषता ह

1 Mediterranean climates only कवल भमसागरीय जलवाय

2 All of the above climates उपय सभी मौसम

3 Monsoon climate only कवल मानसन जलवाय

4 Equatorial climate only कवल भमरखीय जलवाय

Correct Answer -

Monsoon climate only कवल मानसन जलवाय

In _________ rocks the minerals will occurs in beds or layers

ो ि ो ो

22)

23)

24)

______ चानो म खिनज तल या परतो म होत ह

1 metamorphic कायातरत

2 igneous and metamorphic आय और कायातरत

3 igneous आय

4 sedimentary अवसादी

Correct Answer -

sedimentary अवसादी

Black soil is ideal for the cultivation of cotton as कपास की खती क िलए काली िमी आदश ह ोिक

1 Its colour is black यह काली होती ह

2 It is found on plateau regions यह पठार ो म पायी जाती ह

3 It is made up of lava यह लावा स बनी होती ह

4 It can retain moisture यह नमी को बरकरार रख सकती ह

Correct Answer -

It can retain moisture यह नमी को बरकरार रख सकती ह

The National Survey and Mapping Organization of the country works under the Department of___________

दश का रा ीय सवण और मानिचण सगठन ___________ िवभाग क अतगत काय करता ह

1 Space अतर

2 Science and Technology िवान और तकनीक

3 Culture सित

4 Tourism पयटन

Correct Answer -

Science and Technology िवान और तकनीक

Palk strait separates India from पाक जलडमम भारत स _____ को अलग करता ह

1 Pakistan पािकान

2 Andaman Island अडमान ीप

3 China चीन

25)

26)

27)

4 Sri Lanka ीलका

Correct Answer -

Sri Lanka ीलका

Which among the following state is the major producer of Bauxite in India

िनिलखत म स कौन सा रा भारत म बॉाइट का मख उादक ह

1 Madhya Pradesh मदश

2 Rajasthan राजथान

3 Goa गोवा

4 Orissa उड़ीसा

Correct Answer -

Orissa उड़ीसा

Which of the following states DOES NOT share border with Chhattisgarh

िनिलखत म स कौन सा रा छीसगढ़ क साथ सीमा साझा नही करता ह

1 Telangana तलगाना

2 Uttar Pradesh उर दश

3 Bihar िबहार

4 Andhra Pradesh आ दश

Correct Answer -

Bihar िबहार

Which of the following statements is INCORRECT with respect to parallels of latitudes

अाश क समानातरो क सबध म िन निलखत म स कौन सा कथन गलत ह

1 A line joining places of equal latitude is known as parallel of largest

समान अाश क थानो को जोड़न वाली रखा को िवशालतम क समानातर क प म जाना जाता ह

2 They stat from equator and run parallels to it

व भम रखा स ारभ होत ह और इसक समानातर चलत ह

3 All parallels are equal in length सभी समातर लबाई म समान ह

4 All parallels are drawn as circles on the globe ोब पर सभी समानातर वो क प म खीच जात ह

28)

29)

30)

31)

Correct Answer -

All parallels are equal in length सभी समातर लबाई म समान ह

Which of the following Indian states is also known as a lsquoLand of Red river and Blue Hillsrsquo

िनिलखत म स िकस भारतीय रा को लाल नदी और नीली पहािड़यो की भिम क नाम स जाना जाता ह

1 Uttarkhand उराखड

2 Assam असम

3 Meghalaya मघालय

4 Arunachal Pradesh अणाचल दश

Correct Answer -

Assam असम

In spatial analysis of settlement Rn = 215 indicates which type of settlement arrangement

िनपटान क थािनक िवषण म Rn = 215 यह इिगत करता ह िक िकस कार की िनपटान वथा ह

1 Uniform यिनफॉम

2 Semi-Clustered समी- ल टर

3 Clustered ल टर

4 Random रडम

Correct Answer -

Uniform यिनफॉम

Who are known as the lsquoYellow Peoplersquo lsquoयलो पीपलrsquo क प म कौन जाना जाता ह

1 Mongoloids मोगोलोइडस

2 Nigroids नीोइडस

3 Australoids ऑ लॉइडस

4 Caucasoids कॉकसोइडस

Correct Answer -

Mongoloids मोगोलोइडस

ि ि ो ौ ि

32)

33)

34)

Who publishes the topographical map of India भारत क थलाकितक मानिच को कौन कािशत करता ह

1 Geographical Survey of India भारत का भौगोिलक सवण

2 Government of India भारत सरकार

3 Geological Survey of India भारत क भगभय सवण

4 Survey of India भारत का सवण

Correct Answer -

Survey of India भारत का सवण

Who among the following claimed geography to be the lsquoEcology of Manrsquo

िनिलखत म स िकसन भगोल को मन का पारथितकी कहा ह

1 Alfred Hener अड हटनर

2 Vidal-de la Blache वाइडल-िड लॉ ॉश

3 Oo Schluter ओटो टर

4 Harlan Barrow हरलन बारो

Correct Answer -

Harlan Barrow हरलन बारो

Who among the following is regarded as the founder of humanistic approach in geography

िनिलखत म स िकस भगोल म मानवतावादी िकोण का सथापक माना जाता ह

1 William Bunge िविलयम बग

2 Yi-Fu-Tuan यी-फ- यान

3 Brain JL Berry न जएल बरी

4 Richard Peet रचड पीट

Correct Answer -

Yi-Fu-Tuan यी-फ- यान

Who prepared Lorenz curve लोरज व िकसन तयार िकया

1 Geddes गडस

2 None of these इनम स कोई नही

3 Griffith Taylor ििफथ टलर

35)

36)

37)

4 Max U Lorenz म य लोरज

Correct Answer -

Max U Lorenz म य लोरज

Gulf Streams are the currents of which of the following oceans

खाड़ी की धाराए िनिलखत महासागरो म स िकसकी धाराए ह

1 North Atlantic Ocean उरी अटलािटक महासागर

2 North Pacific Ocean उरी शात महासागर

3 Arabian Sea अरब सागर

4 South Pacific Ocean दिण शात महासागर

Correct Answer -

North Atlantic Ocean उरी अटलािटक महासागर

Disintegration wearing away and removal of rock material is generally referred as

िशला पदाथ (रॉक सामी) का टटना िमटना और हटना आमतौर पर ________ क प म सदिभत िकया जाता ह

1 Shattering िवसकारक

2 Denudation अनाादन

3 Fault श

4 Decomposition िवयोजन

Correct Answer -

Denudation अनाादन

Variations in the length of day time and night from season to season are due to

मौसम स मौसम परवतन पर िदन क समय और रात क समय की अविध म िभताए िन कारण स होती ह

1 The Earthrsquos revolution round the Sun in an elliptical manner पी का दीघवाकार तरीक स सय क चारो घणन

2 The Earthrsquos rotation on its axis पी का इसकी धरी पर घणन

3 Revolution of the Earth on a tilted axis नत अ पर पी का घणन

4 Latitudinal position of the place थान की अाश थित

Correct Answer -

Revolution of the Earth on a tilted axis नत अ पर पी का घणन

38)

39)

40)

Point out the correct sequence of mountain ranges from north to south

उर स दिण तक पवत खलाओ क सही अनम को इिगत कर

1 Great Himalaya Middle Himalaya Outer Himalaya Trans Himalaya

महान िहमालय म िहमालय बा िहमालय परा िहमालय

2 Middle Himalaya Great Himalaya Trans Himalaya Outer Himalaya

म िहमालय महान िहमालय परा िहमालय बा िहमालय

3 Outer Himalaya Middle Himalaya Great Himalaya Trans Himalaya

बा िहमालय म िहमालय महान िहमालय परा िहमालय

4 Trans Himalaya Great Himalaya Middle Himalaya Outer Himalaya

परा िहमालय महान िहमालय म िहमालय बा िहमालय

Correct Answer -

Trans Himalaya Great Himalaya Middle Himalaya Outer Himalaya

परा िहमालय महान िहमालय म िहमालय बा िहमालय

Sunrsquos halo is produced by the refraction of light in सय का भामडल ______ म काश क अपवतन ाराउ होता ह

1 Ice crystals in Cirrro-Cumulus clouds पाभ-कपास मघो क बफ िल

2 Ice crystal in Cirrus clouds पाभमघो क बफ िल

3 Dust particles in Stratus clouds री मघो क धल कण

4 Water vapour in Stratus clouds री मघो क जल वा

Correct Answer -

Ice crystal in Cirrus clouds पाभमघो क बफ िल

Read the given statements and answer which of the following options isare correct

(1) The minerals present in the rocks exposed to atmosphere are not subjected to alteration

(2) Oxidation is one of the processes of chemical weathering

िदए गए कथनो को पढ़ और उर द िक िन म स कौन सास िवक सही ह

(1) वायमल स अनावत शल म उपथत खिनज परवतन क अधीन नही होता ह

(2) ऑीकरण रासायिनक अपय की ियाओ म स एक ह

ो ो

41)

42)

1 Both statements are wrong दोनो कथन गलत ह

2 Both statements are correct दोनो कथन सही ह

3 First statement is wrong and second statement is correct पहला कथन गलत ह और दसरा कथन सही ह

4 First statement is correct and second statement is wrong पहला कथन सही ह और दसरा कथन गलत ह

Correct Answer -

First statement is wrong and second statement is correct पहला कथन गलत ह और दसरा कथन सही ह

Read the given statements and answer which of the following options isare correct

1 Sunrsquos short waves enter the earth partially heating the atmosphere

2 Heated earth surface from the sun produces broader waves which interacts and heats the atmosphere

िदए गए कथन को पढ़ और उर द िक िन म स कौन सास िवक सही ह

1 सय की छोटी तरग पी म आिशक प स वश करती ह और वायमडल को ऊत करती ह

2 सय स ऊत पी की सतह िवारत तरग उ करती ह जो परर भाव डालती ह और वायमडल कोऊत करती ह

1 Both Statements 1 and 2 are correct दोनो कथन 1 और 2 सही ह

2 Both Statements 1 and 2 are wrong दोनो कथन 1 और 2 गलत ह

3 Statement 1 is wrong and only Statement 2 is correct कथन 1 गलत ह और कवल कथन 2 सही ह

4 Only statement 1 is correct कवल कथन 1 सही ह

Correct Answer -

Both Statements 1 and 2 are correct दोनो कथन 1 और 2 सही ह

Read the given statements and answer which of the following options isare correct

(1)The rocks that get changed due to heat and pressure are termed as metamorphic rocks

(2)Slate is one such type of metamorphic rock

िदए गए कथनो को पढ़ और उर द िक िन म स कौन सास िवक सही ह

(1) शल जो ऊा और दाब क कारण परवितत हो जात ह उ कायातरक शलो क प म जाना जाता ह

(2) ट एक तरह का कायातरक शल ह

1 Both statements are wrong दोनो कथन गलत ह

2 Both statements are correct दोनो कथन सही ह

3 First statement is wrong and second statement is correct पहला कथन गलत ह और दसरा कथन सही ह

ी औ

43)

44)

4 First statement is correct and second statement is wrong पहला कथन सही ह और दसरा कथन गलत ह

Correct Answer -

Both statements are correct दोनो कथन सही ह

Read the given statements and answer which of the following options isare correct

1 Higher temperature anomaly is observed in the northern hemisphere

2 Differential heating is absent in Northern Hemisphere

िदए गए कथनो को पढ़ और उर द िक िन म स कौन सास िवक सही ह

1 उरी गोलाध म उ तापमान िवसगित पायी जाती ह

2 उरी गोलाध म अतर ऊन अनपथत होती ह

1 Both Statements 1 and 2 are correct दोनो कथन 1 और 2 सही ह

2 Both Statements 1 and 2 are wrong दोनो कथन 1 और 2 गलत ह

3 Statement 1 is wrong and Statement 2 is correct कथन 1 गलत ह और कथन 2 सही ह

4 Statement 1 is correct and Statement 2 is wrong कथन 1 सही ह और कथन 2 गलत ह

Correct Answer -

Statement 1 is correct and Statement 2 is wrong कथन 1 सही ह और कथन 2 गलत ह

Read the given statements and answer which of the following options isare correct

(1) Plutonic rocks are intrusive type of igneous rocks

(2) It cools very slowly because the surrounding rock serves as insulation around the intrusion of magma

िदए गए कथनो को पढ़ और उर द िक िन म स कौन सास िवक सही ह

(1) िवतलीय शल अतवधी कार क आश शल ह

(2) यह बत धीर-धीर ठडा होता ह ोिक आस-पास क शल मा क अतवधन क चारो ओर रोधन क प म कायकरत ह

1 Both statements are wrong दोनो कथन गलत ह

2 Both statements are correct दोनो कथन सही ह

3 First statement is wrong and second statement is correct पहला कथन गलत ह और दसरा कथन सही ह

4 First statement is correct and second statement is wrong पहला कथन सही ह और दसरा कथन गलत ह

Correct Answer -

Both statements are correct दोनो कथन सही ह

45)

46)

47)

48)

The dust and ash material hurled from the volcanoes are termed as

ालामखी स िनकलन वाली धल और राख सामी को _______ क प म कहा जाता ह

1 Pyroclasc पाइरोाक

2 Hyperclastic हाइपराक

3 Hepiroclastic हिपरोाक

4 Cirroclastic िसरोाक

Correct Answer -

Pyroclasc पाइरोाक

The vertical difference in elevation between a low tide and high tide is referred as

कम ार और उ ार क बीच ऊचाई म लबवत अतर _____ स सदिभत होता ह

1 Tidal slope ारीय ढलान

2 Tidal elevation ारीय उयन

3 Tidal range ारीय परास

4 Tidal height ारीय ऊचाई

Correct Answer -

Tidal range ारीय परास

The maximum biodiversity is found in which of the following regions िनिलखत ो म स अिधकतमजव िविवधता िकसम पायी जाती ह

1 Amazon Basin अमज़न बिसन

2 East Indies ई इडीज

3 Congo Basin कागो बिसन

4 West indies व इडीज

Correct Answer -

Amazon Basin अमज़न बिसन

The cultivation of rice crop produces_______ चावल की फसल की खती ______ का उादन करती ह

1 SO2

49)

50)

51)

2 CH4

3 CFCs

4 CO2

Correct Answer -

CH4

The pressure system with higher pressure at the centre is called__________

क म उ दबाव वाली दबाव णाली को _______ कहा जाता ह

1 front अ

2 depression अवनमन

3 cyclone चवात

4 anti-cyclone ितचवात

Correct Answer -

anti-cyclone ितचवात

The Himalayan region is poor in mineral resources because िहमालयी खिनज ससाधनो म समनही ह ोिक

1 The displacement of rock strata has disturbed the arrangement of rocks and made it complex

शलीय परत क िवथापन न चानो की वथा को अवथत कर िदया ह और इस जिटल बना िदया ह

2 The climate conditions are not suitable for exploitation of minerals

जलवाय की थित खिनजो क दोहन क िलए उपय नही ह

3 The terrain makes explanation of minerals difficult and very costly due to transportation difficulties

भ-भाग परवहन की किठनाइयो क कारण खिनजो का दोहन मल और बत महगा बना दता ह

4 It is made up of crystalline rocks यह िलीय चानो स बना ह

Correct Answer -

The displacement of rock strata has disturbed the arrangement of rocks and made it complex

शलीय परत क िवथापन न चानो की वथा को अवथत कर िदया ह और इस जिटल बना िदया ह

The process through which the moisture is added to the atmosphere by vegetation is termed as

वह िया िजसक माम स वनित ारा वातावरण म नमी िमलायी जाती ह _______ क प म जानी जाती ह

52)

53)

54)

1 Condensation सघनन

2 Evapotranspiration वान-उजन

3 Radiation िविकरण

4 Precipitation वषण

Correct Answer -

Evapotranspiration वान-उजन

The process through which the terrestrial heat is transferred to air by direct contact is termed as

वह िया िजसम सपक ारा थलीय ऊा वाय म थानातरत हो जाती ह ______ क प म जानी जाती ह

1 Conduction चालन

2 Convection सवहन

3 Insolation आतपन

4 Radiation िविकरण

Correct Answer -

Conduction चालन

The largest area under mangroves is in which of the following statesunion territory

मोव क अतगत िनिलखत राोसघ शािसत दशो म स सबस बड़ा कौन सा ह

1 Andaman and Nicobar अमान और िनकोबार

2 Andhra Pradesh आ दश

3 West Bengal पिम बगाल

4 Gujarat गजरात

Correct Answer -

West Bengal पिम बगाल

The longitudinal transverse and surface waves in an earthquake originate from

भकप म दशातर अनथ और सतह तरग यहा उ होती ह

1 The focus on the surface of the Earth पी क सतह पर क -िबद म

2 The focus within the body of the Earth पी क भीतर क -िबद म

3 The epicenter within the body of the Earth पी क भीतर उपरक म

55)

56)

57)

4 The epicenter on the surface of the Earth पी क सतह पर उपरक म

Correct Answer -

The focus within the body of the Earth पी क भीतर क -िबद म

The down slope movement of material due to gravity is called______

गाकषण क कारण पदाथ की अनढाल गित को ______ कहा जाता ह

1 mass movement पदाथ सचलन

2 deposition िनप

3 erosion रण

4 volcanic movement ालामखीय सचलन

Correct Answer -

mass movement पदाथ सचलन

Shimla is cooler than Amritsar although both are on the same latitude This is because

िशमला म अमतसर स अिधक ठड ह हालािक दोनो समान अाश पर ह ऐसा ह ोिक

1 Shimla is at a greater height above sea level than Amritsar अमतसर की तलना म िशमला सम तल स अिधकऊचाई पर ह

2 Shimla is further north िशमला उर की ओर ह

3 Shimla is farther from the equator िशमला भम रखा स आग ह

4 Their longitudes differ उनकी दशातर रखाए िभ ह

Correct Answer -

Shimla is at a greater height above sea level than Amritsar अमतसर की तलना म िशमला सम तल स अिधकऊचाई पर ह

lsquoTempo of Urbanizationrsquo measures which of the following

lsquoशहरीकरण का टपोrsquo िनिलखत म स कौन सा उपाय ह

1 Speed of urbanizaon शहरीकरण की गित

2 None of the above इनम स कोई नही

3 Inequality of urbanizaon शहरीकरण की असमानता

4 Current level of urbanizaon शहरीकरण का वतमान र

Correct Answer -

58)

59)

60)

Speed of urbanizaon शहरीकरण की गित

Out of the following options choose the INCORRECT statement

िनिलखत िवको म स गलत कथन का चयन कर

1 The clear tracts in the equatorial region recover rapidly भम रखा म भभाग तजी स ठीक हो जात ह

2 The stable communities include a redwood forest a pine forest at high elevations

थर समदायो म एक रडवड वन उ ऊचाई पर एक दवदार वन शािमल ह

3 Any ecosystem moves towards maximum biomass and stability to survive

कोई भी पारथितकी त जीिवत रहन क िलए अिधकतम जवसहित और थरता की तरफ असर होता ह

4 Tropical rain forests near equator are stable ecosystems

भम रखा क पास उकिटबधीय वषा वन थर पारथितक त ह

Correct Answer -

The clear tracts in the equatorial region recover rapidly भम रखा म भभाग तजी स ठीक हो जात ह

Seasonal contrasts are maximum in मौसमी िवषमता अिधकतम ह

1 Mid latitudes म अाश म

2 Low attitudes िन अाश म

3 High latitudes उ अाश म

4 Subtropics उपोकिटबधीय म

Correct Answer -

Mid latitudes म अाश म

In India which type of forest among the following occupies the largest area

भारत म िनिलखत म स िकस कार क वन सबस बड़ा फल आािदत करत ह

1 Sub-tropical Dry Evergreen Forest उप उकिटबधीय श सदाबहार वन

2 Mountain Wet Temperate Forest पवतीय आ शीतो वन

3 Tropical Moist Deciduous Forest उकिटबधीय आ पणपाती वन

4 Tropical Wet Evergreen Forest उकिटबधीय आ सदाबहार वन

Correct Answer -

Tropical Moist Deciduous Forest उकिटबधीय आ पणपाती वन

61)

62)

63)

64)

What is the proportion of lsquoJuvenile Populationrsquo (0-14 years) in India as per 2011Census

2011 की जनगणना क अनसार भारत म जवनाइल पॉपलशन यानी िकशोर जनस या (0-14 वष) का अनपात ाह

1 3076 of total population कल जनस या का 3076

2 2764 of total population कल जनस या का 2764

3 2933 of total population कल जनस या का 2933

4 3354 of total population कल जनस या का 3354

Correct Answer -

3076 of total population कल जनस या का 3076

What is the Belfast famous for बलफा िकसक िलए मशर ह

1 Belt of cotton textile industry कपास व उोग क

2 Ship-building industry जहाज िनमाण उोग

3 Agricultural machinery किष उपकरण

4 Aero planes manufacturing वाययान िनमाण

Correct Answer -

Ship-building industry जहाज िनमाण उोग

What is the most important occupation in tropical monsoon lands

उकिटबधीय मॉनसन भिम म सबस महपण वसाय ा ह

1 Mining खनन

2 Cattle rearing मवशी पालन

3 Agriculture किष

4 Nomadic herding नोमािडक जड़ी-बिटया

Correct Answer -

Agriculture किष

What is the most important characteristics of the islands (Indian) located in the Arabian sea

अरब सागर म थत ीपो (भारतीय) की सबस महपण िवशषता ा ह

ी ो

65)

66)

67)

1 There are all of coral origins सभी कोरल मल क ह

2 There are all very small in size य सभी आकार म बत छोट ह

3 They have a very dry climate इनकी जलवाय बत श ह

4 They are extended parts of the mainland व महाीप क िवारत िह ह

Correct Answer -

There are all of coral origins सभी कोरल मल क ह

What do the basalt layers of the Deccan indicate डन की बसा परत ा इिगत करती ह

1 All of the above उपरो सभी

2 Huge volcanic eruptions in the distant past दरथ अतीत म िवशाल ालामखीय िवोट

3 The immense erosional activity of the rivers निदयो की िवशाल रण गितिविध

4 The influence of weathering मौसम का भाव

Correct Answer -

Huge volcanic eruptions in the distant past दरथ अतीत म िवशाल ालामखीय िवोट

In the structure of planet Earth below the mantle the core is mainly made up of_____

पी ह की सरचना म मटल क नीच कोर म प स______ स िनिमत होती ह

1 aluminium एमीिनयम

2 silicon िसिलकॉन

3 chromium ोिमयम

4 iron लोहा

Correct Answer -

iron लोहा

One of the major Mid Oceanic Ridge is found in मख म-महासागर चोिटयो म स एक ______ म पायाजाता ह

1 Mid Pacific Ocean म शात महासागर

2 Mid Atlantic Ocean म अटलािटक महासागर

3 Mid Indian Ocean म भारतीय महासागर

4 Mid Arctic Ocean म आक िटक महासागर

68)

69)

70)

71)

Correct Answer -

Mid Atlantic Ocean म अटलािटक महासागर

Magma that reaches the Earthrsquos surface and then solidifies is called________

मा जो पी की सतह तक पचती ह और िफर ठोस हो जाती ह ________कहलाती ह

1 quartz ाटज

2 lava लावा

3 granite नाइट

4 silicates िसिलकट

Correct Answer -

lava लावा

Isotherms are the lines of equal_______ समताप रखाए समान _______की रखाए होती ह

1 pressure दाब

2 temperature तापमान

3 rainfall वषा

4 height ऊचाई

Correct Answer -

temperature तापमान

Mark the correct sequence of passes in the Western Ghats from north to south

पिमी घाटो म उर स दिण तक दर क सही अनम को िचित कर

1 Thalghat Palghat Bhorghat थलगघाट पालघाट भोरघाट

2 Thalghat Bhorghat Palghat थलघाट भोरघाट पालघाट

3 Bhorghat Thalghat Palghat भोरघाट थलघाट पालघाट

4 Palghat Bhorghat Thalghat पालघाट भोरघाट थलघाट

Correct Answer -

Thalghat Bhorghat Palghat थलघाट भोरघाट पालघाट

Which of the following does not have influence over the climate in India

ि ि ि ी ी

72)

73)

िनिलखत म स िकसका भाव भारत की जलवाय पर नही पड़ता ह

1 Ocean currents सागर की लहर

2 Nearness to equator भम रखा स िनकटता

3 Monsoons मानसन

4 Presence of Indian ocean भारतीय महासागर की उपथित

Correct Answer -

Ocean currents सागर की लहर

Which of the following cloud types has the characteristics like vertical tall narrow and puffy

िनिलखत म स िकस कार क मघो म लबवत लबी सकीण और थलता जसी िवशषताए ह

1 Cumulonimbus तफानी मघ

2 Cumulus मघ पज

3 Cirrocumulus पाभ कपासी मघ

4 Nimbostratus वषारी मघ

Correct Answer -

Cumulus मघ पज

Which of the following statement is INCORRECT about Crude Birth Rate

िनिलखत स कौन सा कथन अशोिधत ज दर क बार म सही नही ह

1 It cannot be used for comparing fertility level between two countries with different population characteristics

इसका उपयोग िविभ जनसा िवशषताओ वाल दो दशो क बीच जनन र की तलना क िलए नही िकया जा सकता ह

2 It is a standardized measure of fertility

यह जनन मता का मानकीकत उपाय ह

3 It is effected by the age-sex composition of the population

यह आबादी की आय-िलग सरचना स भािवत होता ह

4 It is expressed per 1000 population in a given geographical unit

यह िकसी दी गई भौगोिलक इकाई म ित 1000 जनसा पर िकया जाता ह

Correct Answer -

It is a standardized measure of fertility

यह जनन मता का मानकीकत उपाय ह

74)

75)

76)

77)

Which of the following state in India experienced negative decadal growth rate during 2001 to 2011census

भारत म िनिलखत म स िकस रा म वष 2001 स 2011 की जनगणना क दौरान नकाराक िगरावट दर ई

1 Tripura िपरा

2 Nagaland नागालड

3 Haryana हरयाणा

4 Odisha ओिडसा

Correct Answer -

Nagaland नागालड

Which of the following is NOT a characteristic of peninsular rivers

िनिलखत म स कौन सी िवशषता ायीपीय निदयो म नही होती ह

1 Flow through shallow valleys उथल घािटयो क माम स वाह

2 Seasonal flow मौसमी वाह

3 Little erosional activity थोड़ी कटावदार गितिविध

4 Meandering tendency often shifting their beds घमावदार वि अर अपन तटो को थानातरत करना

Correct Answer -

Meandering tendency often shifting their beds घमावदार वि अर अपन तटो को थानातरत करना

Which of the following gases in the atmosphere absorbs heat from the Sunrsquos radiation and the Earthssurface

वायमडल म िनिलखत म स कौन सी गस सय क िविकरण और पी की सतह स ऊा को अवशोिषत करती ह

1 Neon िनयॉन

2 Carbon dioxide काबन डाइऑाइड

3 Argon आगन

4 Nitrogen नाइट ोजन

Correct Answer -

Carbon dioxide काबन डाइऑाइड

Which of the following kind of settlement pattern is found at the confluence of rivers

ि ि ि ि ो

78)

79)

80)

िनिलखत म स िकस कार का वथापन पटन निदयो क सगम पर पाया जाता ह

1 Triangular Paern िकोणीय पटन

2 Circular or Semi-Circular Paern परप या अध-परप पटन

3 Nebular Paern नबलर पटन

4 Star ndashShaped Paern ार-आकार का पटन

Correct Answer -

Triangular Paern िकोणीय पटन

Which one was not the objective of the Biosphere Reserve Projects launched by the UNESCO

यनो ारा श की गई सरित जवमडल परयोजनाओ का उ इनम स कौन सा नही था

1 To promote teaching and research िशण और अनसधान को बढ़ावा दना

2 To make agriculture sustainable किष को दीघकािलक बनाना

3 To conserve ecosystems पारथितक त को सरित करना

4 To conserve genetic diversity for a longtime लब समय तक अनवािशक िविवधता को सरित करना

Correct Answer -

To make agriculture sustainable किष को दीघकािलक बनाना

Which region of the Earth surface receives the highest amount of insulation

पी सतह का कौन सा तापावरोधन की उतम माा ा करता ह

1 Land mass थलखड

2 Savannah region सवाना

3 Water bodies जल िनकाय

4 Tropical desert उकिटबधीय रिगान

Correct Answer -

Tropical desert उकिटबधीय रिगान

Which one of the following is not a biodiversity hotspot

िनिलखत म स कौन सा जव िविवधता का म जगह नही ह

1 Eastern Himalaya पव िहमालय

2 Eastern Ghats पव घाट

81)

82)

83)

3 Indo-Myanmar भारत-ामार

4 Westerm Ghats पिमी घाट

Correct Answer -

Eastern Ghats पव घाट

Which one of the following is NOT a part of the World Network of Biosphere Reserves based on theUNESCO Man and Biosphere Programme

यनो मन और बायोीयर कायम क आधार पर िनिलखत म स कौन बायोीयर रजव क िव नटवक कािहा नही ह

1 Gulf of Mannar मार की खाड़ी

2 Seshachalam शषाचलम

3 Sunderban सदरबन

4 Nilgiri नीलिगर

Correct Answer -

Seshachalam शषाचलम

Which one of the following is an example of ldquodesert vegetationrdquo

िनिलखत म स कौन मथलीय वनित का एक उदाहरण ह

1 Mosses and lichens दलदल और शवाल

2 Temperate grassland समशीतो घास क मदान

3 Coniferous forest शकधारी वन

4 Acacia and cactus एकािसया और कस

Correct Answer -

Acacia and cactus एकािसया और कस

Which one of the following reflects more sunlight िनिलखत म स कौन सा सय की रोशनी को अिधकपरावितत करता ह

1 Paddy crop land धान फसल भिम

2 Land covered with fresh snow ताजा बफ स आािदत भिम

3 Sand desert रतीली रिगान

4 Prairie land यरी भिम

84)

85)

86)

87)

Correct Answer -

Land covered with fresh snow ताजा बफ स आािदत भिम

Which layer of the atmosphere is in contact with the surface of the earthrsquos oceans

वायमडल की कौन सी परत पी क महासागरो की सतह क सपक म ह

1 Stratosphere समताप मडल

2 Mesosphere म मडल

3 Hydrosphere जलमडल

4 Troposphere ोभ मडल

Correct Answer -

Troposphere ोभ मडल

Mediterranean Sea is a border of which of the following countries भम सागर िनिलखत दशो म सिकसकी सीमा ह

1 None of these इनम स कोई नही

2 Iraq इराक

3 Lebanon लबनान

4 Jordan जॉडन

Correct Answer -

Lebanon लबनान

Benguela ocean currents are found along which coast बगएला महासागर धाराए िकस तट क साथ पायीजाती ह

1 East Coast of South America दिण अमरका क पव तट

2 East Coast of Africa अीका क पव तट

3 West Coast of South America दिण अमरका क पिमी तट

4 West Coast of Africa अीका क पिमी तट

Correct Answer -

West Coast of Africa अीका क पिमी तट

88)

89)

90)

Due to tension a block of land on one side being pushed up or upthrown relative to the downthrown blockis referred as

तनाव क कारण नीच फ क ए खड क साप भिम का एक खड एक ओर स ऊपर धकला जाता ह या ऊपर की ओरफ का जाता ह यह _____ क प म सदिभत ह

1 Thrust fault प श

2 Normal fault सामा श

3 Reverse fault म श

4 Strike slip fault नितलब सपण श

Correct Answer -

Normal fault सामा श

Inter-tropical doldrums is a zone of ______ अतर-उकिटबधीय डोलड ______ का एक ह

1 Frontolysis टोलायिसस

2 Convergence अिभसरण

3 Inter-tropical divergence zone अतर-उकिटबधीय िवचलन

4 Local wind थानीय वाय

Correct Answer -

Convergence अिभसरण

The Horse Latitudes are regions located at about _____ north and south of the equator

हॉस अाश भम रखा क उर और दिण म लगभग _____ पर थत ह

1 30ndash60 degree Latitude 30-60 िडी अाश

2 0ndash5 degree Latitude 0-5 िडी अाश

3 30 degree Latitude 30 िडी अाश

4 60ndash90 degree Latitude 60-90 िडी अाश

Correct Answer -

30 degree Latitude 30 िडी अाश

Generally evaporation is high over which part of the Earth

आम तौर पर पी क िकस भाग पर वाीकरण अिधक होता ह

1 Equatorial maritime भमवत समीय ी ी

91)

92)

2 Equatorial continental भमवत महाीपीय

3 Polar maritime वीय समीय

4 Polar continental वीय महाीपीय

Correct Answer -

Equatorial maritime भमवत समीय

A very high temperature during summer in north western India leads to what type of climaticcondition in south

उर पिमी भारत म गम क दौरान बत अिधक तापमान होन क कारण दिण म िकस कार की जलवाय थितउ करता ह

1 Depression over arabian sea अरब सागर पर अवनमन

2 Failure monsoon मानसन िवफलता

3 Successful monsoon मानसन सफलता

4 Cyclones चवात

Correct Answer -

Successful monsoon मानसन सफलता

Lightning and thunder are the resultant effect when तिड़त और गजन परणामी भाव ह जब

1 Two massive clouds hit powerfully each other first lightning is produced and later sound is produced

दो बड़ बादल एक दसर स शशाली ढग स टकरात ह पहल आकाशीय िवदयत उ होता ह और बाद म िन उहोती ह

2 Two massive clouds come into contact with the powerful wind collision this results into first sound and thenlightning

दो बड़ बादल शशाली पवन सघ क सपक म आत ह इसका परणामप पहल िन और िफर आकाशीय िवदयतउ होता ह

3 None of the above उपरो म स कोई भी नही

4 A high density cloud contains positively and negatively charged electric ions and when this interacts light andsound are simultaneously produced

एक उ घन बादल म धनाक और ऋणाक आविशत िवदयत आयन होत ह और जब यह परर भाव डालत ह तोकाश और िन एक साथ उािदत होती ह

Correct Answer -

A high density cloud contains positively and negatively charged electric ions and when this interacts light andsound are simultaneously produced

औ ि ि ो औ ो

93)

94)

95)

एक उ घन बादल म धनाक और ऋणाक आविशत िवदयत आयन होत ह और जब यह परर भाव डालत ह तोकाश और िन एक साथ उािदत होती ह

Doon Valley is able to grow rice because दन घाटी चावल उगान म सम ह ोिक

1 Other crops cannot be grown वहा अ फसलो को उगाया नही जा सकता ह

2 People in the valley are rice eaters घाटी म लोग चावल खान वाल ह

3 There is a huge export demand of rice वहा चावल की भारी िनयात माग ह

4 It has warm summer and snow melt waters for irrigation

वहा गिमया गम होती ह िसचाई क िलए बफ का िपघला आ पानी होता ह

Correct Answer -

It has warm summer and snow melt waters for irrigation

वहा गिमया गम होती ह िसचाई क िलए बफ का िपघला आ पानी होता ह

CANCELLED

In the geological time scale the Mesozoic Era DOES NOT contains which of the following periods

भगभय समय पमान पर मजीवी यग म िन कालो म स कौन नही ह

1 Triassic ट ाइऐिसक

2 Jurassic जरिसक

3 Cretaceous चाकमय

4 Carboniferous काबनी

Correct Answer -

Carboniferous काबनी

96)

1 P-3 Q-4 R-2 S-1

2 P-3 Q-4 R-1 S-2

3 P-3 Q-4 R-1 S-2

4 P-4 Q-3 R-2 S-1

Correct Answer -

P-4 Q-3 R-2 S-1

1 P-3 Q-1 R-4 S-2

2 P-3 Q-4 R-1 S-2

3 P-3 Q-2 R-4 S-1

97)

98)

4 P-2 Q-1 R-4 S-3

Correct Answer -

P-3 Q-4 R-1 S-2

1 P-3 Q-1 R-4 S-2

2 P-2 Q-3 R-4 S-1

3 P-2 Q-1 R-3 S-4

4 P-4 Q-2 R-1 S-3

Correct Answer -

P-3 Q-1 R-4 S-2

99)

100)

1 P-3 Q-2 R-4 S-1

2 P-1 Q-2 R-3 S-4

3 P-2 Q-3 R-1 S-4

4 P-4 Q-3 R-2 S-1

Correct Answer -

P-2 Q-3 R-1 S-4

ldquoHuman geography is the study of changing relationship between the unresting man and the unstableearthrdquo was defined by

lsquolsquoमानव भगोल ाकल आदमी और अथर पी क बीच सबध परवतन का अयन हrdquo ______ ारा परभािषत िकया गयाथा

1 J Brunches ज चस

2 EC Semple ईसी सल

3 HJ Mackinder एच ज मिकदर

4 PV Blache पीवी च

Correct Answer -

EC Semple ईसी सल

Sedimentary rocks are finally and ultimately derived from the____________

अवसादी चान अततः ________ स ा की जाती ह

1 action of earth movements पी की गितिविधयो

2 marine deposit समी िनप

3 weathering of metamorphic rocks पातरत चानो क अपय

4 weathering of igneous rocks आय चानो क अपय

Correct Answer -

weathering of igneous rocks आय चानो क अपय

Page 19: High School Teacher Eligibility Test- BOARD PROFESSIONAL ...peb.mp.gov.in/results/RESULT_18/HST_RES18/Final_anwser_key/HST… · M a ndl a / मंड ल ... Under the Madhya Pradesh

7)

8)

9)

1 4 3 and 2 1 4 3 और 2

CANCELLED

Karl Pearsonrsquos correlation co-efficient is काल िपयसन का सहसबध गणाक ह

1 Arithmec mean समार मा

2 Geometric mean गणोर मा

3 None of these इनम स कोई नही

4 Harmonic mean हराक मा

Correct Answer -

Geometric mean गणोर मा

CANCELLED

ldquoEach day is more or less the same the morning is clear and bright with a sea breeze as the Sun climbshigh in the sky heat mounts up dark clouds form then rain comes with thunder lighting But rain is soonoverrdquo Which of the following regions is described in the above passage

ldquoक िदन समान स अिधक या कम होता ह सम की हवा क साथ और उल सबह होती ह जस सयआकाश म ऊचा चढ़ता ह गम बढ़ जाती ह काल बादल बनत ह िफर िबजली क साथ बारश आती ह लिकनबारश जी ख हो जाती हlsquorsquo उपरो पा म िनिलखत म स िकन ो का वणन िकया गया ह

1 Equatorial भमरखीय

2 Equatorial भमरखीय

3 Savannah सवाना

4 Mediterranean आातरक (भमसागरीय)

5 Mediterranean आातरक (भमसागरीय)

6 Monsoon मानसन

7 Monsoon मानसन

Correct Answer -

Equatorial भमरखीय

Equatorial भमरखीय

CANCELLED

In which epoch of the geological history of the Earth dinosaurs reached their largest size

पी क भगभय इितहास क िकस यग म डायनासोर अपन सबस बड़ आकार तक पचि

10)

11)

1 Triassic ट ायिसक

2 Jurassic जरािसक

3 Cretaceous ीटशस

4 Permian पिमयन

Correct Answer -

Jurassic जरािसक

CANCELLED

A spring tide would occur in which of the following conditions

िनिलखत म स िकन थितयो म ार-भाटा आता ह

1 When the Sun Moon and Earth are in a straight line

जब सय चमा और पी एक सीधी रखा म होत ह

2 When the Moon and Earth are in right angle to each other

जब चमा और पी एक-दसर क दािहन कोण म होत ह

3 When the Earth and Moon are in right angle to the Sun

जब पी और चमा सय क दािहन कोण म होत ह

4 When the Sun and Moon are in right angle to each other

जब सय और चमा एक-दसर क दािहन कोण म होत ह

Correct Answer -

When the Sun Moon and Earth are in a straight line

जब सय चमा और पी एक सीधी रखा म होत ह

CANCELLED

An observe on the Earthrsquos surface always sees the same face of the moon because

एक पयवक को पी की सतह स हमशा चाद का एक ही फलक िदखाई दता ह ोिक

1 Its path of revolution around the earth is the same as that of the earth around the Sun

इसका पी क चारो ओर परमण का माग सय क चारो ओर पी क समान ही ह

2 Its period of revolution around the Earth is the same as its period of rotation around its own axis

इसकी पी क चारो ओर परमण की अविध उसकी अपनी धरी क चारो ओर घणन की अविध क समान ह

3 Its period of rotation is the same as that of the Earth इसकी घणन की अविध पी क समान ह

ी ि ी ी

12)

13)

14)

4 Its direct of rotation is the same as that of Earth घणन की िदशा पी क समान ही ह

Correct Answer -

Its period of revolution around the Earth is the same as its period of rotation around its own axis

इसकी पी क चारो ओर परमण की अविध उसकी अपनी धरी क चारो ओर घणन की अविध क समान ह

CANCELLED

The pebbles that are faceted by the sand-blasting and shaped polished by the wind abrasions are known as

पवन अपघषन ारा पॉिलश रत-िवोिटत और साच म ढला ककड़ ___________ क प म जाना जाता ह

1 Dreikanter िकोणक

2 Pediments िकोिनका

3 Inselberg इलबग

4 Dunes टीबा

Correct Answer -

Dreikanter िकोणक

CANCELLED

Astronomical unit is the average distance between खगोलीय इकाई ______ क बीच की औसत दरी ह

1 Earth and Mars पी और मगल

2 Earth and mercury पी और बध

3 Earth and moon पी और चमा

4 Earth and Sun पी और सय

Correct Answer -

Earth and Sun पी और सय

During cold weather season in the northern plains there will be an inflow of cyclonic disturbancesfrom the _________ directions

शीत मौसम क दौरान उरी मदानी इलाको म _________ िदशाओ स चवात सबधी गड़बड़ी का अतवाह होगा

1 East and Northwest पव और उरपिम

2 East and Northeast पव और पवर

3 West and East पिम और पव

ि औ ि

15)

16)

17)

4 West and Northwest पिम और उरपिम

Correct Answer -

West and Northwest पिम और उरपिम

During an earth quake the velocity of the body waves will________ along with the increase in densityof the material it is passing through

भकप क दौरान लहरो क ऊपरी भाग का वग घन म व क साथ-साथ ________ जो इसस गजरन वाली वको आग बढाएगी

1 not change नही बदलगा

2 increase initially and then decrease शआत म बढ़गा और िफर घटगा

3 increase बढ़गा

4 decrease घटगा

Correct Answer -

increase बढ़गा

The Clouded Leopard National park is situated in which of the following states

िनिलखत म िकस रा म धिमल तदआ रा ीय उान (ाउडड लपड नशनल पाक ) थत ह

1 Tripura िपरा

2 Uttar Pradesh उर दश

3 Assam असम

4 Mizoram िमजोरम

Correct Answer -

Tripura िपरा

Usually the land surfaces are heated more quickly than the water surfaces because _____________

आम तौर पर जल सतहो की तलना म भिम सतह अिधक तजी स गम होती ह ोिक _____________ ह

1 the specific heat of water is higher than land पानी की िविश ऊा भिम स अिधक

2 the specific heat of water is lesser than land पानी की िविश ऊा भिम स कम होती

3 the latent heat of water is higher than the land पानी की अतिनिहत ऊा भिम स अिधक

4 the land reflects more heat radiation than water भिम पानी की तलना म अिधक ऊा क िविकरण को पराविततकरती

18)

19)

20)

21)

Correct Answer -

the specific heat of water is higher than land पानी की िविश ऊा भिम स अिधक

The longest shore-line is along the state of सबस लबी समतटीय रखा िन रा क साथ ह

1 Maharashtra महारा

2 Orissa उड़ीसा

3 Kerala करल

4 Gujarat गजरात

Correct Answer -

Gujarat गजरात

The position when the Earth is farthest from the Sun is known as

जब पी सय स सबस दर होती ह तो उस थित को िन नाम स जाना जाता ह

1 Perihelion उपसौर

2 Vernal Equinox बसत िवषव

3 Aphelion अपसौर

4 Autumnal Equinox शराल िवषव

Correct Answer -

Aphelion अपसौर

The seasonal reversal of winds is the typical characteristic of

हवाओ का मौसमी परवतन ______ की सामा िवशषता ह

1 Mediterranean climates only कवल भमसागरीय जलवाय

2 All of the above climates उपय सभी मौसम

3 Monsoon climate only कवल मानसन जलवाय

4 Equatorial climate only कवल भमरखीय जलवाय

Correct Answer -

Monsoon climate only कवल मानसन जलवाय

In _________ rocks the minerals will occurs in beds or layers

ो ि ो ो

22)

23)

24)

______ चानो म खिनज तल या परतो म होत ह

1 metamorphic कायातरत

2 igneous and metamorphic आय और कायातरत

3 igneous आय

4 sedimentary अवसादी

Correct Answer -

sedimentary अवसादी

Black soil is ideal for the cultivation of cotton as कपास की खती क िलए काली िमी आदश ह ोिक

1 Its colour is black यह काली होती ह

2 It is found on plateau regions यह पठार ो म पायी जाती ह

3 It is made up of lava यह लावा स बनी होती ह

4 It can retain moisture यह नमी को बरकरार रख सकती ह

Correct Answer -

It can retain moisture यह नमी को बरकरार रख सकती ह

The National Survey and Mapping Organization of the country works under the Department of___________

दश का रा ीय सवण और मानिचण सगठन ___________ िवभाग क अतगत काय करता ह

1 Space अतर

2 Science and Technology िवान और तकनीक

3 Culture सित

4 Tourism पयटन

Correct Answer -

Science and Technology िवान और तकनीक

Palk strait separates India from पाक जलडमम भारत स _____ को अलग करता ह

1 Pakistan पािकान

2 Andaman Island अडमान ीप

3 China चीन

25)

26)

27)

4 Sri Lanka ीलका

Correct Answer -

Sri Lanka ीलका

Which among the following state is the major producer of Bauxite in India

िनिलखत म स कौन सा रा भारत म बॉाइट का मख उादक ह

1 Madhya Pradesh मदश

2 Rajasthan राजथान

3 Goa गोवा

4 Orissa उड़ीसा

Correct Answer -

Orissa उड़ीसा

Which of the following states DOES NOT share border with Chhattisgarh

िनिलखत म स कौन सा रा छीसगढ़ क साथ सीमा साझा नही करता ह

1 Telangana तलगाना

2 Uttar Pradesh उर दश

3 Bihar िबहार

4 Andhra Pradesh आ दश

Correct Answer -

Bihar िबहार

Which of the following statements is INCORRECT with respect to parallels of latitudes

अाश क समानातरो क सबध म िन निलखत म स कौन सा कथन गलत ह

1 A line joining places of equal latitude is known as parallel of largest

समान अाश क थानो को जोड़न वाली रखा को िवशालतम क समानातर क प म जाना जाता ह

2 They stat from equator and run parallels to it

व भम रखा स ारभ होत ह और इसक समानातर चलत ह

3 All parallels are equal in length सभी समातर लबाई म समान ह

4 All parallels are drawn as circles on the globe ोब पर सभी समानातर वो क प म खीच जात ह

28)

29)

30)

31)

Correct Answer -

All parallels are equal in length सभी समातर लबाई म समान ह

Which of the following Indian states is also known as a lsquoLand of Red river and Blue Hillsrsquo

िनिलखत म स िकस भारतीय रा को लाल नदी और नीली पहािड़यो की भिम क नाम स जाना जाता ह

1 Uttarkhand उराखड

2 Assam असम

3 Meghalaya मघालय

4 Arunachal Pradesh अणाचल दश

Correct Answer -

Assam असम

In spatial analysis of settlement Rn = 215 indicates which type of settlement arrangement

िनपटान क थािनक िवषण म Rn = 215 यह इिगत करता ह िक िकस कार की िनपटान वथा ह

1 Uniform यिनफॉम

2 Semi-Clustered समी- ल टर

3 Clustered ल टर

4 Random रडम

Correct Answer -

Uniform यिनफॉम

Who are known as the lsquoYellow Peoplersquo lsquoयलो पीपलrsquo क प म कौन जाना जाता ह

1 Mongoloids मोगोलोइडस

2 Nigroids नीोइडस

3 Australoids ऑ लॉइडस

4 Caucasoids कॉकसोइडस

Correct Answer -

Mongoloids मोगोलोइडस

ि ि ो ौ ि

32)

33)

34)

Who publishes the topographical map of India भारत क थलाकितक मानिच को कौन कािशत करता ह

1 Geographical Survey of India भारत का भौगोिलक सवण

2 Government of India भारत सरकार

3 Geological Survey of India भारत क भगभय सवण

4 Survey of India भारत का सवण

Correct Answer -

Survey of India भारत का सवण

Who among the following claimed geography to be the lsquoEcology of Manrsquo

िनिलखत म स िकसन भगोल को मन का पारथितकी कहा ह

1 Alfred Hener अड हटनर

2 Vidal-de la Blache वाइडल-िड लॉ ॉश

3 Oo Schluter ओटो टर

4 Harlan Barrow हरलन बारो

Correct Answer -

Harlan Barrow हरलन बारो

Who among the following is regarded as the founder of humanistic approach in geography

िनिलखत म स िकस भगोल म मानवतावादी िकोण का सथापक माना जाता ह

1 William Bunge िविलयम बग

2 Yi-Fu-Tuan यी-फ- यान

3 Brain JL Berry न जएल बरी

4 Richard Peet रचड पीट

Correct Answer -

Yi-Fu-Tuan यी-फ- यान

Who prepared Lorenz curve लोरज व िकसन तयार िकया

1 Geddes गडस

2 None of these इनम स कोई नही

3 Griffith Taylor ििफथ टलर

35)

36)

37)

4 Max U Lorenz म य लोरज

Correct Answer -

Max U Lorenz म य लोरज

Gulf Streams are the currents of which of the following oceans

खाड़ी की धाराए िनिलखत महासागरो म स िकसकी धाराए ह

1 North Atlantic Ocean उरी अटलािटक महासागर

2 North Pacific Ocean उरी शात महासागर

3 Arabian Sea अरब सागर

4 South Pacific Ocean दिण शात महासागर

Correct Answer -

North Atlantic Ocean उरी अटलािटक महासागर

Disintegration wearing away and removal of rock material is generally referred as

िशला पदाथ (रॉक सामी) का टटना िमटना और हटना आमतौर पर ________ क प म सदिभत िकया जाता ह

1 Shattering िवसकारक

2 Denudation अनाादन

3 Fault श

4 Decomposition िवयोजन

Correct Answer -

Denudation अनाादन

Variations in the length of day time and night from season to season are due to

मौसम स मौसम परवतन पर िदन क समय और रात क समय की अविध म िभताए िन कारण स होती ह

1 The Earthrsquos revolution round the Sun in an elliptical manner पी का दीघवाकार तरीक स सय क चारो घणन

2 The Earthrsquos rotation on its axis पी का इसकी धरी पर घणन

3 Revolution of the Earth on a tilted axis नत अ पर पी का घणन

4 Latitudinal position of the place थान की अाश थित

Correct Answer -

Revolution of the Earth on a tilted axis नत अ पर पी का घणन

38)

39)

40)

Point out the correct sequence of mountain ranges from north to south

उर स दिण तक पवत खलाओ क सही अनम को इिगत कर

1 Great Himalaya Middle Himalaya Outer Himalaya Trans Himalaya

महान िहमालय म िहमालय बा िहमालय परा िहमालय

2 Middle Himalaya Great Himalaya Trans Himalaya Outer Himalaya

म िहमालय महान िहमालय परा िहमालय बा िहमालय

3 Outer Himalaya Middle Himalaya Great Himalaya Trans Himalaya

बा िहमालय म िहमालय महान िहमालय परा िहमालय

4 Trans Himalaya Great Himalaya Middle Himalaya Outer Himalaya

परा िहमालय महान िहमालय म िहमालय बा िहमालय

Correct Answer -

Trans Himalaya Great Himalaya Middle Himalaya Outer Himalaya

परा िहमालय महान िहमालय म िहमालय बा िहमालय

Sunrsquos halo is produced by the refraction of light in सय का भामडल ______ म काश क अपवतन ाराउ होता ह

1 Ice crystals in Cirrro-Cumulus clouds पाभ-कपास मघो क बफ िल

2 Ice crystal in Cirrus clouds पाभमघो क बफ िल

3 Dust particles in Stratus clouds री मघो क धल कण

4 Water vapour in Stratus clouds री मघो क जल वा

Correct Answer -

Ice crystal in Cirrus clouds पाभमघो क बफ िल

Read the given statements and answer which of the following options isare correct

(1) The minerals present in the rocks exposed to atmosphere are not subjected to alteration

(2) Oxidation is one of the processes of chemical weathering

िदए गए कथनो को पढ़ और उर द िक िन म स कौन सास िवक सही ह

(1) वायमल स अनावत शल म उपथत खिनज परवतन क अधीन नही होता ह

(2) ऑीकरण रासायिनक अपय की ियाओ म स एक ह

ो ो

41)

42)

1 Both statements are wrong दोनो कथन गलत ह

2 Both statements are correct दोनो कथन सही ह

3 First statement is wrong and second statement is correct पहला कथन गलत ह और दसरा कथन सही ह

4 First statement is correct and second statement is wrong पहला कथन सही ह और दसरा कथन गलत ह

Correct Answer -

First statement is wrong and second statement is correct पहला कथन गलत ह और दसरा कथन सही ह

Read the given statements and answer which of the following options isare correct

1 Sunrsquos short waves enter the earth partially heating the atmosphere

2 Heated earth surface from the sun produces broader waves which interacts and heats the atmosphere

िदए गए कथन को पढ़ और उर द िक िन म स कौन सास िवक सही ह

1 सय की छोटी तरग पी म आिशक प स वश करती ह और वायमडल को ऊत करती ह

2 सय स ऊत पी की सतह िवारत तरग उ करती ह जो परर भाव डालती ह और वायमडल कोऊत करती ह

1 Both Statements 1 and 2 are correct दोनो कथन 1 और 2 सही ह

2 Both Statements 1 and 2 are wrong दोनो कथन 1 और 2 गलत ह

3 Statement 1 is wrong and only Statement 2 is correct कथन 1 गलत ह और कवल कथन 2 सही ह

4 Only statement 1 is correct कवल कथन 1 सही ह

Correct Answer -

Both Statements 1 and 2 are correct दोनो कथन 1 और 2 सही ह

Read the given statements and answer which of the following options isare correct

(1)The rocks that get changed due to heat and pressure are termed as metamorphic rocks

(2)Slate is one such type of metamorphic rock

िदए गए कथनो को पढ़ और उर द िक िन म स कौन सास िवक सही ह

(1) शल जो ऊा और दाब क कारण परवितत हो जात ह उ कायातरक शलो क प म जाना जाता ह

(2) ट एक तरह का कायातरक शल ह

1 Both statements are wrong दोनो कथन गलत ह

2 Both statements are correct दोनो कथन सही ह

3 First statement is wrong and second statement is correct पहला कथन गलत ह और दसरा कथन सही ह

ी औ

43)

44)

4 First statement is correct and second statement is wrong पहला कथन सही ह और दसरा कथन गलत ह

Correct Answer -

Both statements are correct दोनो कथन सही ह

Read the given statements and answer which of the following options isare correct

1 Higher temperature anomaly is observed in the northern hemisphere

2 Differential heating is absent in Northern Hemisphere

िदए गए कथनो को पढ़ और उर द िक िन म स कौन सास िवक सही ह

1 उरी गोलाध म उ तापमान िवसगित पायी जाती ह

2 उरी गोलाध म अतर ऊन अनपथत होती ह

1 Both Statements 1 and 2 are correct दोनो कथन 1 और 2 सही ह

2 Both Statements 1 and 2 are wrong दोनो कथन 1 और 2 गलत ह

3 Statement 1 is wrong and Statement 2 is correct कथन 1 गलत ह और कथन 2 सही ह

4 Statement 1 is correct and Statement 2 is wrong कथन 1 सही ह और कथन 2 गलत ह

Correct Answer -

Statement 1 is correct and Statement 2 is wrong कथन 1 सही ह और कथन 2 गलत ह

Read the given statements and answer which of the following options isare correct

(1) Plutonic rocks are intrusive type of igneous rocks

(2) It cools very slowly because the surrounding rock serves as insulation around the intrusion of magma

िदए गए कथनो को पढ़ और उर द िक िन म स कौन सास िवक सही ह

(1) िवतलीय शल अतवधी कार क आश शल ह

(2) यह बत धीर-धीर ठडा होता ह ोिक आस-पास क शल मा क अतवधन क चारो ओर रोधन क प म कायकरत ह

1 Both statements are wrong दोनो कथन गलत ह

2 Both statements are correct दोनो कथन सही ह

3 First statement is wrong and second statement is correct पहला कथन गलत ह और दसरा कथन सही ह

4 First statement is correct and second statement is wrong पहला कथन सही ह और दसरा कथन गलत ह

Correct Answer -

Both statements are correct दोनो कथन सही ह

45)

46)

47)

48)

The dust and ash material hurled from the volcanoes are termed as

ालामखी स िनकलन वाली धल और राख सामी को _______ क प म कहा जाता ह

1 Pyroclasc पाइरोाक

2 Hyperclastic हाइपराक

3 Hepiroclastic हिपरोाक

4 Cirroclastic िसरोाक

Correct Answer -

Pyroclasc पाइरोाक

The vertical difference in elevation between a low tide and high tide is referred as

कम ार और उ ार क बीच ऊचाई म लबवत अतर _____ स सदिभत होता ह

1 Tidal slope ारीय ढलान

2 Tidal elevation ारीय उयन

3 Tidal range ारीय परास

4 Tidal height ारीय ऊचाई

Correct Answer -

Tidal range ारीय परास

The maximum biodiversity is found in which of the following regions िनिलखत ो म स अिधकतमजव िविवधता िकसम पायी जाती ह

1 Amazon Basin अमज़न बिसन

2 East Indies ई इडीज

3 Congo Basin कागो बिसन

4 West indies व इडीज

Correct Answer -

Amazon Basin अमज़न बिसन

The cultivation of rice crop produces_______ चावल की फसल की खती ______ का उादन करती ह

1 SO2

49)

50)

51)

2 CH4

3 CFCs

4 CO2

Correct Answer -

CH4

The pressure system with higher pressure at the centre is called__________

क म उ दबाव वाली दबाव णाली को _______ कहा जाता ह

1 front अ

2 depression अवनमन

3 cyclone चवात

4 anti-cyclone ितचवात

Correct Answer -

anti-cyclone ितचवात

The Himalayan region is poor in mineral resources because िहमालयी खिनज ससाधनो म समनही ह ोिक

1 The displacement of rock strata has disturbed the arrangement of rocks and made it complex

शलीय परत क िवथापन न चानो की वथा को अवथत कर िदया ह और इस जिटल बना िदया ह

2 The climate conditions are not suitable for exploitation of minerals

जलवाय की थित खिनजो क दोहन क िलए उपय नही ह

3 The terrain makes explanation of minerals difficult and very costly due to transportation difficulties

भ-भाग परवहन की किठनाइयो क कारण खिनजो का दोहन मल और बत महगा बना दता ह

4 It is made up of crystalline rocks यह िलीय चानो स बना ह

Correct Answer -

The displacement of rock strata has disturbed the arrangement of rocks and made it complex

शलीय परत क िवथापन न चानो की वथा को अवथत कर िदया ह और इस जिटल बना िदया ह

The process through which the moisture is added to the atmosphere by vegetation is termed as

वह िया िजसक माम स वनित ारा वातावरण म नमी िमलायी जाती ह _______ क प म जानी जाती ह

52)

53)

54)

1 Condensation सघनन

2 Evapotranspiration वान-उजन

3 Radiation िविकरण

4 Precipitation वषण

Correct Answer -

Evapotranspiration वान-उजन

The process through which the terrestrial heat is transferred to air by direct contact is termed as

वह िया िजसम सपक ारा थलीय ऊा वाय म थानातरत हो जाती ह ______ क प म जानी जाती ह

1 Conduction चालन

2 Convection सवहन

3 Insolation आतपन

4 Radiation िविकरण

Correct Answer -

Conduction चालन

The largest area under mangroves is in which of the following statesunion territory

मोव क अतगत िनिलखत राोसघ शािसत दशो म स सबस बड़ा कौन सा ह

1 Andaman and Nicobar अमान और िनकोबार

2 Andhra Pradesh आ दश

3 West Bengal पिम बगाल

4 Gujarat गजरात

Correct Answer -

West Bengal पिम बगाल

The longitudinal transverse and surface waves in an earthquake originate from

भकप म दशातर अनथ और सतह तरग यहा उ होती ह

1 The focus on the surface of the Earth पी क सतह पर क -िबद म

2 The focus within the body of the Earth पी क भीतर क -िबद म

3 The epicenter within the body of the Earth पी क भीतर उपरक म

55)

56)

57)

4 The epicenter on the surface of the Earth पी क सतह पर उपरक म

Correct Answer -

The focus within the body of the Earth पी क भीतर क -िबद म

The down slope movement of material due to gravity is called______

गाकषण क कारण पदाथ की अनढाल गित को ______ कहा जाता ह

1 mass movement पदाथ सचलन

2 deposition िनप

3 erosion रण

4 volcanic movement ालामखीय सचलन

Correct Answer -

mass movement पदाथ सचलन

Shimla is cooler than Amritsar although both are on the same latitude This is because

िशमला म अमतसर स अिधक ठड ह हालािक दोनो समान अाश पर ह ऐसा ह ोिक

1 Shimla is at a greater height above sea level than Amritsar अमतसर की तलना म िशमला सम तल स अिधकऊचाई पर ह

2 Shimla is further north िशमला उर की ओर ह

3 Shimla is farther from the equator िशमला भम रखा स आग ह

4 Their longitudes differ उनकी दशातर रखाए िभ ह

Correct Answer -

Shimla is at a greater height above sea level than Amritsar अमतसर की तलना म िशमला सम तल स अिधकऊचाई पर ह

lsquoTempo of Urbanizationrsquo measures which of the following

lsquoशहरीकरण का टपोrsquo िनिलखत म स कौन सा उपाय ह

1 Speed of urbanizaon शहरीकरण की गित

2 None of the above इनम स कोई नही

3 Inequality of urbanizaon शहरीकरण की असमानता

4 Current level of urbanizaon शहरीकरण का वतमान र

Correct Answer -

58)

59)

60)

Speed of urbanizaon शहरीकरण की गित

Out of the following options choose the INCORRECT statement

िनिलखत िवको म स गलत कथन का चयन कर

1 The clear tracts in the equatorial region recover rapidly भम रखा म भभाग तजी स ठीक हो जात ह

2 The stable communities include a redwood forest a pine forest at high elevations

थर समदायो म एक रडवड वन उ ऊचाई पर एक दवदार वन शािमल ह

3 Any ecosystem moves towards maximum biomass and stability to survive

कोई भी पारथितकी त जीिवत रहन क िलए अिधकतम जवसहित और थरता की तरफ असर होता ह

4 Tropical rain forests near equator are stable ecosystems

भम रखा क पास उकिटबधीय वषा वन थर पारथितक त ह

Correct Answer -

The clear tracts in the equatorial region recover rapidly भम रखा म भभाग तजी स ठीक हो जात ह

Seasonal contrasts are maximum in मौसमी िवषमता अिधकतम ह

1 Mid latitudes म अाश म

2 Low attitudes िन अाश म

3 High latitudes उ अाश म

4 Subtropics उपोकिटबधीय म

Correct Answer -

Mid latitudes म अाश म

In India which type of forest among the following occupies the largest area

भारत म िनिलखत म स िकस कार क वन सबस बड़ा फल आािदत करत ह

1 Sub-tropical Dry Evergreen Forest उप उकिटबधीय श सदाबहार वन

2 Mountain Wet Temperate Forest पवतीय आ शीतो वन

3 Tropical Moist Deciduous Forest उकिटबधीय आ पणपाती वन

4 Tropical Wet Evergreen Forest उकिटबधीय आ सदाबहार वन

Correct Answer -

Tropical Moist Deciduous Forest उकिटबधीय आ पणपाती वन

61)

62)

63)

64)

What is the proportion of lsquoJuvenile Populationrsquo (0-14 years) in India as per 2011Census

2011 की जनगणना क अनसार भारत म जवनाइल पॉपलशन यानी िकशोर जनस या (0-14 वष) का अनपात ाह

1 3076 of total population कल जनस या का 3076

2 2764 of total population कल जनस या का 2764

3 2933 of total population कल जनस या का 2933

4 3354 of total population कल जनस या का 3354

Correct Answer -

3076 of total population कल जनस या का 3076

What is the Belfast famous for बलफा िकसक िलए मशर ह

1 Belt of cotton textile industry कपास व उोग क

2 Ship-building industry जहाज िनमाण उोग

3 Agricultural machinery किष उपकरण

4 Aero planes manufacturing वाययान िनमाण

Correct Answer -

Ship-building industry जहाज िनमाण उोग

What is the most important occupation in tropical monsoon lands

उकिटबधीय मॉनसन भिम म सबस महपण वसाय ा ह

1 Mining खनन

2 Cattle rearing मवशी पालन

3 Agriculture किष

4 Nomadic herding नोमािडक जड़ी-बिटया

Correct Answer -

Agriculture किष

What is the most important characteristics of the islands (Indian) located in the Arabian sea

अरब सागर म थत ीपो (भारतीय) की सबस महपण िवशषता ा ह

ी ो

65)

66)

67)

1 There are all of coral origins सभी कोरल मल क ह

2 There are all very small in size य सभी आकार म बत छोट ह

3 They have a very dry climate इनकी जलवाय बत श ह

4 They are extended parts of the mainland व महाीप क िवारत िह ह

Correct Answer -

There are all of coral origins सभी कोरल मल क ह

What do the basalt layers of the Deccan indicate डन की बसा परत ा इिगत करती ह

1 All of the above उपरो सभी

2 Huge volcanic eruptions in the distant past दरथ अतीत म िवशाल ालामखीय िवोट

3 The immense erosional activity of the rivers निदयो की िवशाल रण गितिविध

4 The influence of weathering मौसम का भाव

Correct Answer -

Huge volcanic eruptions in the distant past दरथ अतीत म िवशाल ालामखीय िवोट

In the structure of planet Earth below the mantle the core is mainly made up of_____

पी ह की सरचना म मटल क नीच कोर म प स______ स िनिमत होती ह

1 aluminium एमीिनयम

2 silicon िसिलकॉन

3 chromium ोिमयम

4 iron लोहा

Correct Answer -

iron लोहा

One of the major Mid Oceanic Ridge is found in मख म-महासागर चोिटयो म स एक ______ म पायाजाता ह

1 Mid Pacific Ocean म शात महासागर

2 Mid Atlantic Ocean म अटलािटक महासागर

3 Mid Indian Ocean म भारतीय महासागर

4 Mid Arctic Ocean म आक िटक महासागर

68)

69)

70)

71)

Correct Answer -

Mid Atlantic Ocean म अटलािटक महासागर

Magma that reaches the Earthrsquos surface and then solidifies is called________

मा जो पी की सतह तक पचती ह और िफर ठोस हो जाती ह ________कहलाती ह

1 quartz ाटज

2 lava लावा

3 granite नाइट

4 silicates िसिलकट

Correct Answer -

lava लावा

Isotherms are the lines of equal_______ समताप रखाए समान _______की रखाए होती ह

1 pressure दाब

2 temperature तापमान

3 rainfall वषा

4 height ऊचाई

Correct Answer -

temperature तापमान

Mark the correct sequence of passes in the Western Ghats from north to south

पिमी घाटो म उर स दिण तक दर क सही अनम को िचित कर

1 Thalghat Palghat Bhorghat थलगघाट पालघाट भोरघाट

2 Thalghat Bhorghat Palghat थलघाट भोरघाट पालघाट

3 Bhorghat Thalghat Palghat भोरघाट थलघाट पालघाट

4 Palghat Bhorghat Thalghat पालघाट भोरघाट थलघाट

Correct Answer -

Thalghat Bhorghat Palghat थलघाट भोरघाट पालघाट

Which of the following does not have influence over the climate in India

ि ि ि ी ी

72)

73)

िनिलखत म स िकसका भाव भारत की जलवाय पर नही पड़ता ह

1 Ocean currents सागर की लहर

2 Nearness to equator भम रखा स िनकटता

3 Monsoons मानसन

4 Presence of Indian ocean भारतीय महासागर की उपथित

Correct Answer -

Ocean currents सागर की लहर

Which of the following cloud types has the characteristics like vertical tall narrow and puffy

िनिलखत म स िकस कार क मघो म लबवत लबी सकीण और थलता जसी िवशषताए ह

1 Cumulonimbus तफानी मघ

2 Cumulus मघ पज

3 Cirrocumulus पाभ कपासी मघ

4 Nimbostratus वषारी मघ

Correct Answer -

Cumulus मघ पज

Which of the following statement is INCORRECT about Crude Birth Rate

िनिलखत स कौन सा कथन अशोिधत ज दर क बार म सही नही ह

1 It cannot be used for comparing fertility level between two countries with different population characteristics

इसका उपयोग िविभ जनसा िवशषताओ वाल दो दशो क बीच जनन र की तलना क िलए नही िकया जा सकता ह

2 It is a standardized measure of fertility

यह जनन मता का मानकीकत उपाय ह

3 It is effected by the age-sex composition of the population

यह आबादी की आय-िलग सरचना स भािवत होता ह

4 It is expressed per 1000 population in a given geographical unit

यह िकसी दी गई भौगोिलक इकाई म ित 1000 जनसा पर िकया जाता ह

Correct Answer -

It is a standardized measure of fertility

यह जनन मता का मानकीकत उपाय ह

74)

75)

76)

77)

Which of the following state in India experienced negative decadal growth rate during 2001 to 2011census

भारत म िनिलखत म स िकस रा म वष 2001 स 2011 की जनगणना क दौरान नकाराक िगरावट दर ई

1 Tripura िपरा

2 Nagaland नागालड

3 Haryana हरयाणा

4 Odisha ओिडसा

Correct Answer -

Nagaland नागालड

Which of the following is NOT a characteristic of peninsular rivers

िनिलखत म स कौन सी िवशषता ायीपीय निदयो म नही होती ह

1 Flow through shallow valleys उथल घािटयो क माम स वाह

2 Seasonal flow मौसमी वाह

3 Little erosional activity थोड़ी कटावदार गितिविध

4 Meandering tendency often shifting their beds घमावदार वि अर अपन तटो को थानातरत करना

Correct Answer -

Meandering tendency often shifting their beds घमावदार वि अर अपन तटो को थानातरत करना

Which of the following gases in the atmosphere absorbs heat from the Sunrsquos radiation and the Earthssurface

वायमडल म िनिलखत म स कौन सी गस सय क िविकरण और पी की सतह स ऊा को अवशोिषत करती ह

1 Neon िनयॉन

2 Carbon dioxide काबन डाइऑाइड

3 Argon आगन

4 Nitrogen नाइट ोजन

Correct Answer -

Carbon dioxide काबन डाइऑाइड

Which of the following kind of settlement pattern is found at the confluence of rivers

ि ि ि ि ो

78)

79)

80)

िनिलखत म स िकस कार का वथापन पटन निदयो क सगम पर पाया जाता ह

1 Triangular Paern िकोणीय पटन

2 Circular or Semi-Circular Paern परप या अध-परप पटन

3 Nebular Paern नबलर पटन

4 Star ndashShaped Paern ार-आकार का पटन

Correct Answer -

Triangular Paern िकोणीय पटन

Which one was not the objective of the Biosphere Reserve Projects launched by the UNESCO

यनो ारा श की गई सरित जवमडल परयोजनाओ का उ इनम स कौन सा नही था

1 To promote teaching and research िशण और अनसधान को बढ़ावा दना

2 To make agriculture sustainable किष को दीघकािलक बनाना

3 To conserve ecosystems पारथितक त को सरित करना

4 To conserve genetic diversity for a longtime लब समय तक अनवािशक िविवधता को सरित करना

Correct Answer -

To make agriculture sustainable किष को दीघकािलक बनाना

Which region of the Earth surface receives the highest amount of insulation

पी सतह का कौन सा तापावरोधन की उतम माा ा करता ह

1 Land mass थलखड

2 Savannah region सवाना

3 Water bodies जल िनकाय

4 Tropical desert उकिटबधीय रिगान

Correct Answer -

Tropical desert उकिटबधीय रिगान

Which one of the following is not a biodiversity hotspot

िनिलखत म स कौन सा जव िविवधता का म जगह नही ह

1 Eastern Himalaya पव िहमालय

2 Eastern Ghats पव घाट

81)

82)

83)

3 Indo-Myanmar भारत-ामार

4 Westerm Ghats पिमी घाट

Correct Answer -

Eastern Ghats पव घाट

Which one of the following is NOT a part of the World Network of Biosphere Reserves based on theUNESCO Man and Biosphere Programme

यनो मन और बायोीयर कायम क आधार पर िनिलखत म स कौन बायोीयर रजव क िव नटवक कािहा नही ह

1 Gulf of Mannar मार की खाड़ी

2 Seshachalam शषाचलम

3 Sunderban सदरबन

4 Nilgiri नीलिगर

Correct Answer -

Seshachalam शषाचलम

Which one of the following is an example of ldquodesert vegetationrdquo

िनिलखत म स कौन मथलीय वनित का एक उदाहरण ह

1 Mosses and lichens दलदल और शवाल

2 Temperate grassland समशीतो घास क मदान

3 Coniferous forest शकधारी वन

4 Acacia and cactus एकािसया और कस

Correct Answer -

Acacia and cactus एकािसया और कस

Which one of the following reflects more sunlight िनिलखत म स कौन सा सय की रोशनी को अिधकपरावितत करता ह

1 Paddy crop land धान फसल भिम

2 Land covered with fresh snow ताजा बफ स आािदत भिम

3 Sand desert रतीली रिगान

4 Prairie land यरी भिम

84)

85)

86)

87)

Correct Answer -

Land covered with fresh snow ताजा बफ स आािदत भिम

Which layer of the atmosphere is in contact with the surface of the earthrsquos oceans

वायमडल की कौन सी परत पी क महासागरो की सतह क सपक म ह

1 Stratosphere समताप मडल

2 Mesosphere म मडल

3 Hydrosphere जलमडल

4 Troposphere ोभ मडल

Correct Answer -

Troposphere ोभ मडल

Mediterranean Sea is a border of which of the following countries भम सागर िनिलखत दशो म सिकसकी सीमा ह

1 None of these इनम स कोई नही

2 Iraq इराक

3 Lebanon लबनान

4 Jordan जॉडन

Correct Answer -

Lebanon लबनान

Benguela ocean currents are found along which coast बगएला महासागर धाराए िकस तट क साथ पायीजाती ह

1 East Coast of South America दिण अमरका क पव तट

2 East Coast of Africa अीका क पव तट

3 West Coast of South America दिण अमरका क पिमी तट

4 West Coast of Africa अीका क पिमी तट

Correct Answer -

West Coast of Africa अीका क पिमी तट

88)

89)

90)

Due to tension a block of land on one side being pushed up or upthrown relative to the downthrown blockis referred as

तनाव क कारण नीच फ क ए खड क साप भिम का एक खड एक ओर स ऊपर धकला जाता ह या ऊपर की ओरफ का जाता ह यह _____ क प म सदिभत ह

1 Thrust fault प श

2 Normal fault सामा श

3 Reverse fault म श

4 Strike slip fault नितलब सपण श

Correct Answer -

Normal fault सामा श

Inter-tropical doldrums is a zone of ______ अतर-उकिटबधीय डोलड ______ का एक ह

1 Frontolysis टोलायिसस

2 Convergence अिभसरण

3 Inter-tropical divergence zone अतर-उकिटबधीय िवचलन

4 Local wind थानीय वाय

Correct Answer -

Convergence अिभसरण

The Horse Latitudes are regions located at about _____ north and south of the equator

हॉस अाश भम रखा क उर और दिण म लगभग _____ पर थत ह

1 30ndash60 degree Latitude 30-60 िडी अाश

2 0ndash5 degree Latitude 0-5 िडी अाश

3 30 degree Latitude 30 िडी अाश

4 60ndash90 degree Latitude 60-90 िडी अाश

Correct Answer -

30 degree Latitude 30 िडी अाश

Generally evaporation is high over which part of the Earth

आम तौर पर पी क िकस भाग पर वाीकरण अिधक होता ह

1 Equatorial maritime भमवत समीय ी ी

91)

92)

2 Equatorial continental भमवत महाीपीय

3 Polar maritime वीय समीय

4 Polar continental वीय महाीपीय

Correct Answer -

Equatorial maritime भमवत समीय

A very high temperature during summer in north western India leads to what type of climaticcondition in south

उर पिमी भारत म गम क दौरान बत अिधक तापमान होन क कारण दिण म िकस कार की जलवाय थितउ करता ह

1 Depression over arabian sea अरब सागर पर अवनमन

2 Failure monsoon मानसन िवफलता

3 Successful monsoon मानसन सफलता

4 Cyclones चवात

Correct Answer -

Successful monsoon मानसन सफलता

Lightning and thunder are the resultant effect when तिड़त और गजन परणामी भाव ह जब

1 Two massive clouds hit powerfully each other first lightning is produced and later sound is produced

दो बड़ बादल एक दसर स शशाली ढग स टकरात ह पहल आकाशीय िवदयत उ होता ह और बाद म िन उहोती ह

2 Two massive clouds come into contact with the powerful wind collision this results into first sound and thenlightning

दो बड़ बादल शशाली पवन सघ क सपक म आत ह इसका परणामप पहल िन और िफर आकाशीय िवदयतउ होता ह

3 None of the above उपरो म स कोई भी नही

4 A high density cloud contains positively and negatively charged electric ions and when this interacts light andsound are simultaneously produced

एक उ घन बादल म धनाक और ऋणाक आविशत िवदयत आयन होत ह और जब यह परर भाव डालत ह तोकाश और िन एक साथ उािदत होती ह

Correct Answer -

A high density cloud contains positively and negatively charged electric ions and when this interacts light andsound are simultaneously produced

औ ि ि ो औ ो

93)

94)

95)

एक उ घन बादल म धनाक और ऋणाक आविशत िवदयत आयन होत ह और जब यह परर भाव डालत ह तोकाश और िन एक साथ उािदत होती ह

Doon Valley is able to grow rice because दन घाटी चावल उगान म सम ह ोिक

1 Other crops cannot be grown वहा अ फसलो को उगाया नही जा सकता ह

2 People in the valley are rice eaters घाटी म लोग चावल खान वाल ह

3 There is a huge export demand of rice वहा चावल की भारी िनयात माग ह

4 It has warm summer and snow melt waters for irrigation

वहा गिमया गम होती ह िसचाई क िलए बफ का िपघला आ पानी होता ह

Correct Answer -

It has warm summer and snow melt waters for irrigation

वहा गिमया गम होती ह िसचाई क िलए बफ का िपघला आ पानी होता ह

CANCELLED

In the geological time scale the Mesozoic Era DOES NOT contains which of the following periods

भगभय समय पमान पर मजीवी यग म िन कालो म स कौन नही ह

1 Triassic ट ाइऐिसक

2 Jurassic जरिसक

3 Cretaceous चाकमय

4 Carboniferous काबनी

Correct Answer -

Carboniferous काबनी

96)

1 P-3 Q-4 R-2 S-1

2 P-3 Q-4 R-1 S-2

3 P-3 Q-4 R-1 S-2

4 P-4 Q-3 R-2 S-1

Correct Answer -

P-4 Q-3 R-2 S-1

1 P-3 Q-1 R-4 S-2

2 P-3 Q-4 R-1 S-2

3 P-3 Q-2 R-4 S-1

97)

98)

4 P-2 Q-1 R-4 S-3

Correct Answer -

P-3 Q-4 R-1 S-2

1 P-3 Q-1 R-4 S-2

2 P-2 Q-3 R-4 S-1

3 P-2 Q-1 R-3 S-4

4 P-4 Q-2 R-1 S-3

Correct Answer -

P-3 Q-1 R-4 S-2

99)

100)

1 P-3 Q-2 R-4 S-1

2 P-1 Q-2 R-3 S-4

3 P-2 Q-3 R-1 S-4

4 P-4 Q-3 R-2 S-1

Correct Answer -

P-2 Q-3 R-1 S-4

ldquoHuman geography is the study of changing relationship between the unresting man and the unstableearthrdquo was defined by

lsquolsquoमानव भगोल ाकल आदमी और अथर पी क बीच सबध परवतन का अयन हrdquo ______ ारा परभािषत िकया गयाथा

1 J Brunches ज चस

2 EC Semple ईसी सल

3 HJ Mackinder एच ज मिकदर

4 PV Blache पीवी च

Correct Answer -

EC Semple ईसी सल

Sedimentary rocks are finally and ultimately derived from the____________

अवसादी चान अततः ________ स ा की जाती ह

1 action of earth movements पी की गितिविधयो

2 marine deposit समी िनप

3 weathering of metamorphic rocks पातरत चानो क अपय

4 weathering of igneous rocks आय चानो क अपय

Correct Answer -

weathering of igneous rocks आय चानो क अपय

Page 20: High School Teacher Eligibility Test- BOARD PROFESSIONAL ...peb.mp.gov.in/results/RESULT_18/HST_RES18/Final_anwser_key/HST… · M a ndl a / मंड ल ... Under the Madhya Pradesh

10)

11)

1 Triassic ट ायिसक

2 Jurassic जरािसक

3 Cretaceous ीटशस

4 Permian पिमयन

Correct Answer -

Jurassic जरािसक

CANCELLED

A spring tide would occur in which of the following conditions

िनिलखत म स िकन थितयो म ार-भाटा आता ह

1 When the Sun Moon and Earth are in a straight line

जब सय चमा और पी एक सीधी रखा म होत ह

2 When the Moon and Earth are in right angle to each other

जब चमा और पी एक-दसर क दािहन कोण म होत ह

3 When the Earth and Moon are in right angle to the Sun

जब पी और चमा सय क दािहन कोण म होत ह

4 When the Sun and Moon are in right angle to each other

जब सय और चमा एक-दसर क दािहन कोण म होत ह

Correct Answer -

When the Sun Moon and Earth are in a straight line

जब सय चमा और पी एक सीधी रखा म होत ह

CANCELLED

An observe on the Earthrsquos surface always sees the same face of the moon because

एक पयवक को पी की सतह स हमशा चाद का एक ही फलक िदखाई दता ह ोिक

1 Its path of revolution around the earth is the same as that of the earth around the Sun

इसका पी क चारो ओर परमण का माग सय क चारो ओर पी क समान ही ह

2 Its period of revolution around the Earth is the same as its period of rotation around its own axis

इसकी पी क चारो ओर परमण की अविध उसकी अपनी धरी क चारो ओर घणन की अविध क समान ह

3 Its period of rotation is the same as that of the Earth इसकी घणन की अविध पी क समान ह

ी ि ी ी

12)

13)

14)

4 Its direct of rotation is the same as that of Earth घणन की िदशा पी क समान ही ह

Correct Answer -

Its period of revolution around the Earth is the same as its period of rotation around its own axis

इसकी पी क चारो ओर परमण की अविध उसकी अपनी धरी क चारो ओर घणन की अविध क समान ह

CANCELLED

The pebbles that are faceted by the sand-blasting and shaped polished by the wind abrasions are known as

पवन अपघषन ारा पॉिलश रत-िवोिटत और साच म ढला ककड़ ___________ क प म जाना जाता ह

1 Dreikanter िकोणक

2 Pediments िकोिनका

3 Inselberg इलबग

4 Dunes टीबा

Correct Answer -

Dreikanter िकोणक

CANCELLED

Astronomical unit is the average distance between खगोलीय इकाई ______ क बीच की औसत दरी ह

1 Earth and Mars पी और मगल

2 Earth and mercury पी और बध

3 Earth and moon पी और चमा

4 Earth and Sun पी और सय

Correct Answer -

Earth and Sun पी और सय

During cold weather season in the northern plains there will be an inflow of cyclonic disturbancesfrom the _________ directions

शीत मौसम क दौरान उरी मदानी इलाको म _________ िदशाओ स चवात सबधी गड़बड़ी का अतवाह होगा

1 East and Northwest पव और उरपिम

2 East and Northeast पव और पवर

3 West and East पिम और पव

ि औ ि

15)

16)

17)

4 West and Northwest पिम और उरपिम

Correct Answer -

West and Northwest पिम और उरपिम

During an earth quake the velocity of the body waves will________ along with the increase in densityof the material it is passing through

भकप क दौरान लहरो क ऊपरी भाग का वग घन म व क साथ-साथ ________ जो इसस गजरन वाली वको आग बढाएगी

1 not change नही बदलगा

2 increase initially and then decrease शआत म बढ़गा और िफर घटगा

3 increase बढ़गा

4 decrease घटगा

Correct Answer -

increase बढ़गा

The Clouded Leopard National park is situated in which of the following states

िनिलखत म िकस रा म धिमल तदआ रा ीय उान (ाउडड लपड नशनल पाक ) थत ह

1 Tripura िपरा

2 Uttar Pradesh उर दश

3 Assam असम

4 Mizoram िमजोरम

Correct Answer -

Tripura िपरा

Usually the land surfaces are heated more quickly than the water surfaces because _____________

आम तौर पर जल सतहो की तलना म भिम सतह अिधक तजी स गम होती ह ोिक _____________ ह

1 the specific heat of water is higher than land पानी की िविश ऊा भिम स अिधक

2 the specific heat of water is lesser than land पानी की िविश ऊा भिम स कम होती

3 the latent heat of water is higher than the land पानी की अतिनिहत ऊा भिम स अिधक

4 the land reflects more heat radiation than water भिम पानी की तलना म अिधक ऊा क िविकरण को पराविततकरती

18)

19)

20)

21)

Correct Answer -

the specific heat of water is higher than land पानी की िविश ऊा भिम स अिधक

The longest shore-line is along the state of सबस लबी समतटीय रखा िन रा क साथ ह

1 Maharashtra महारा

2 Orissa उड़ीसा

3 Kerala करल

4 Gujarat गजरात

Correct Answer -

Gujarat गजरात

The position when the Earth is farthest from the Sun is known as

जब पी सय स सबस दर होती ह तो उस थित को िन नाम स जाना जाता ह

1 Perihelion उपसौर

2 Vernal Equinox बसत िवषव

3 Aphelion अपसौर

4 Autumnal Equinox शराल िवषव

Correct Answer -

Aphelion अपसौर

The seasonal reversal of winds is the typical characteristic of

हवाओ का मौसमी परवतन ______ की सामा िवशषता ह

1 Mediterranean climates only कवल भमसागरीय जलवाय

2 All of the above climates उपय सभी मौसम

3 Monsoon climate only कवल मानसन जलवाय

4 Equatorial climate only कवल भमरखीय जलवाय

Correct Answer -

Monsoon climate only कवल मानसन जलवाय

In _________ rocks the minerals will occurs in beds or layers

ो ि ो ो

22)

23)

24)

______ चानो म खिनज तल या परतो म होत ह

1 metamorphic कायातरत

2 igneous and metamorphic आय और कायातरत

3 igneous आय

4 sedimentary अवसादी

Correct Answer -

sedimentary अवसादी

Black soil is ideal for the cultivation of cotton as कपास की खती क िलए काली िमी आदश ह ोिक

1 Its colour is black यह काली होती ह

2 It is found on plateau regions यह पठार ो म पायी जाती ह

3 It is made up of lava यह लावा स बनी होती ह

4 It can retain moisture यह नमी को बरकरार रख सकती ह

Correct Answer -

It can retain moisture यह नमी को बरकरार रख सकती ह

The National Survey and Mapping Organization of the country works under the Department of___________

दश का रा ीय सवण और मानिचण सगठन ___________ िवभाग क अतगत काय करता ह

1 Space अतर

2 Science and Technology िवान और तकनीक

3 Culture सित

4 Tourism पयटन

Correct Answer -

Science and Technology िवान और तकनीक

Palk strait separates India from पाक जलडमम भारत स _____ को अलग करता ह

1 Pakistan पािकान

2 Andaman Island अडमान ीप

3 China चीन

25)

26)

27)

4 Sri Lanka ीलका

Correct Answer -

Sri Lanka ीलका

Which among the following state is the major producer of Bauxite in India

िनिलखत म स कौन सा रा भारत म बॉाइट का मख उादक ह

1 Madhya Pradesh मदश

2 Rajasthan राजथान

3 Goa गोवा

4 Orissa उड़ीसा

Correct Answer -

Orissa उड़ीसा

Which of the following states DOES NOT share border with Chhattisgarh

िनिलखत म स कौन सा रा छीसगढ़ क साथ सीमा साझा नही करता ह

1 Telangana तलगाना

2 Uttar Pradesh उर दश

3 Bihar िबहार

4 Andhra Pradesh आ दश

Correct Answer -

Bihar िबहार

Which of the following statements is INCORRECT with respect to parallels of latitudes

अाश क समानातरो क सबध म िन निलखत म स कौन सा कथन गलत ह

1 A line joining places of equal latitude is known as parallel of largest

समान अाश क थानो को जोड़न वाली रखा को िवशालतम क समानातर क प म जाना जाता ह

2 They stat from equator and run parallels to it

व भम रखा स ारभ होत ह और इसक समानातर चलत ह

3 All parallels are equal in length सभी समातर लबाई म समान ह

4 All parallels are drawn as circles on the globe ोब पर सभी समानातर वो क प म खीच जात ह

28)

29)

30)

31)

Correct Answer -

All parallels are equal in length सभी समातर लबाई म समान ह

Which of the following Indian states is also known as a lsquoLand of Red river and Blue Hillsrsquo

िनिलखत म स िकस भारतीय रा को लाल नदी और नीली पहािड़यो की भिम क नाम स जाना जाता ह

1 Uttarkhand उराखड

2 Assam असम

3 Meghalaya मघालय

4 Arunachal Pradesh अणाचल दश

Correct Answer -

Assam असम

In spatial analysis of settlement Rn = 215 indicates which type of settlement arrangement

िनपटान क थािनक िवषण म Rn = 215 यह इिगत करता ह िक िकस कार की िनपटान वथा ह

1 Uniform यिनफॉम

2 Semi-Clustered समी- ल टर

3 Clustered ल टर

4 Random रडम

Correct Answer -

Uniform यिनफॉम

Who are known as the lsquoYellow Peoplersquo lsquoयलो पीपलrsquo क प म कौन जाना जाता ह

1 Mongoloids मोगोलोइडस

2 Nigroids नीोइडस

3 Australoids ऑ लॉइडस

4 Caucasoids कॉकसोइडस

Correct Answer -

Mongoloids मोगोलोइडस

ि ि ो ौ ि

32)

33)

34)

Who publishes the topographical map of India भारत क थलाकितक मानिच को कौन कािशत करता ह

1 Geographical Survey of India भारत का भौगोिलक सवण

2 Government of India भारत सरकार

3 Geological Survey of India भारत क भगभय सवण

4 Survey of India भारत का सवण

Correct Answer -

Survey of India भारत का सवण

Who among the following claimed geography to be the lsquoEcology of Manrsquo

िनिलखत म स िकसन भगोल को मन का पारथितकी कहा ह

1 Alfred Hener अड हटनर

2 Vidal-de la Blache वाइडल-िड लॉ ॉश

3 Oo Schluter ओटो टर

4 Harlan Barrow हरलन बारो

Correct Answer -

Harlan Barrow हरलन बारो

Who among the following is regarded as the founder of humanistic approach in geography

िनिलखत म स िकस भगोल म मानवतावादी िकोण का सथापक माना जाता ह

1 William Bunge िविलयम बग

2 Yi-Fu-Tuan यी-फ- यान

3 Brain JL Berry न जएल बरी

4 Richard Peet रचड पीट

Correct Answer -

Yi-Fu-Tuan यी-फ- यान

Who prepared Lorenz curve लोरज व िकसन तयार िकया

1 Geddes गडस

2 None of these इनम स कोई नही

3 Griffith Taylor ििफथ टलर

35)

36)

37)

4 Max U Lorenz म य लोरज

Correct Answer -

Max U Lorenz म य लोरज

Gulf Streams are the currents of which of the following oceans

खाड़ी की धाराए िनिलखत महासागरो म स िकसकी धाराए ह

1 North Atlantic Ocean उरी अटलािटक महासागर

2 North Pacific Ocean उरी शात महासागर

3 Arabian Sea अरब सागर

4 South Pacific Ocean दिण शात महासागर

Correct Answer -

North Atlantic Ocean उरी अटलािटक महासागर

Disintegration wearing away and removal of rock material is generally referred as

िशला पदाथ (रॉक सामी) का टटना िमटना और हटना आमतौर पर ________ क प म सदिभत िकया जाता ह

1 Shattering िवसकारक

2 Denudation अनाादन

3 Fault श

4 Decomposition िवयोजन

Correct Answer -

Denudation अनाादन

Variations in the length of day time and night from season to season are due to

मौसम स मौसम परवतन पर िदन क समय और रात क समय की अविध म िभताए िन कारण स होती ह

1 The Earthrsquos revolution round the Sun in an elliptical manner पी का दीघवाकार तरीक स सय क चारो घणन

2 The Earthrsquos rotation on its axis पी का इसकी धरी पर घणन

3 Revolution of the Earth on a tilted axis नत अ पर पी का घणन

4 Latitudinal position of the place थान की अाश थित

Correct Answer -

Revolution of the Earth on a tilted axis नत अ पर पी का घणन

38)

39)

40)

Point out the correct sequence of mountain ranges from north to south

उर स दिण तक पवत खलाओ क सही अनम को इिगत कर

1 Great Himalaya Middle Himalaya Outer Himalaya Trans Himalaya

महान िहमालय म िहमालय बा िहमालय परा िहमालय

2 Middle Himalaya Great Himalaya Trans Himalaya Outer Himalaya

म िहमालय महान िहमालय परा िहमालय बा िहमालय

3 Outer Himalaya Middle Himalaya Great Himalaya Trans Himalaya

बा िहमालय म िहमालय महान िहमालय परा िहमालय

4 Trans Himalaya Great Himalaya Middle Himalaya Outer Himalaya

परा िहमालय महान िहमालय म िहमालय बा िहमालय

Correct Answer -

Trans Himalaya Great Himalaya Middle Himalaya Outer Himalaya

परा िहमालय महान िहमालय म िहमालय बा िहमालय

Sunrsquos halo is produced by the refraction of light in सय का भामडल ______ म काश क अपवतन ाराउ होता ह

1 Ice crystals in Cirrro-Cumulus clouds पाभ-कपास मघो क बफ िल

2 Ice crystal in Cirrus clouds पाभमघो क बफ िल

3 Dust particles in Stratus clouds री मघो क धल कण

4 Water vapour in Stratus clouds री मघो क जल वा

Correct Answer -

Ice crystal in Cirrus clouds पाभमघो क बफ िल

Read the given statements and answer which of the following options isare correct

(1) The minerals present in the rocks exposed to atmosphere are not subjected to alteration

(2) Oxidation is one of the processes of chemical weathering

िदए गए कथनो को पढ़ और उर द िक िन म स कौन सास िवक सही ह

(1) वायमल स अनावत शल म उपथत खिनज परवतन क अधीन नही होता ह

(2) ऑीकरण रासायिनक अपय की ियाओ म स एक ह

ो ो

41)

42)

1 Both statements are wrong दोनो कथन गलत ह

2 Both statements are correct दोनो कथन सही ह

3 First statement is wrong and second statement is correct पहला कथन गलत ह और दसरा कथन सही ह

4 First statement is correct and second statement is wrong पहला कथन सही ह और दसरा कथन गलत ह

Correct Answer -

First statement is wrong and second statement is correct पहला कथन गलत ह और दसरा कथन सही ह

Read the given statements and answer which of the following options isare correct

1 Sunrsquos short waves enter the earth partially heating the atmosphere

2 Heated earth surface from the sun produces broader waves which interacts and heats the atmosphere

िदए गए कथन को पढ़ और उर द िक िन म स कौन सास िवक सही ह

1 सय की छोटी तरग पी म आिशक प स वश करती ह और वायमडल को ऊत करती ह

2 सय स ऊत पी की सतह िवारत तरग उ करती ह जो परर भाव डालती ह और वायमडल कोऊत करती ह

1 Both Statements 1 and 2 are correct दोनो कथन 1 और 2 सही ह

2 Both Statements 1 and 2 are wrong दोनो कथन 1 और 2 गलत ह

3 Statement 1 is wrong and only Statement 2 is correct कथन 1 गलत ह और कवल कथन 2 सही ह

4 Only statement 1 is correct कवल कथन 1 सही ह

Correct Answer -

Both Statements 1 and 2 are correct दोनो कथन 1 और 2 सही ह

Read the given statements and answer which of the following options isare correct

(1)The rocks that get changed due to heat and pressure are termed as metamorphic rocks

(2)Slate is one such type of metamorphic rock

िदए गए कथनो को पढ़ और उर द िक िन म स कौन सास िवक सही ह

(1) शल जो ऊा और दाब क कारण परवितत हो जात ह उ कायातरक शलो क प म जाना जाता ह

(2) ट एक तरह का कायातरक शल ह

1 Both statements are wrong दोनो कथन गलत ह

2 Both statements are correct दोनो कथन सही ह

3 First statement is wrong and second statement is correct पहला कथन गलत ह और दसरा कथन सही ह

ी औ

43)

44)

4 First statement is correct and second statement is wrong पहला कथन सही ह और दसरा कथन गलत ह

Correct Answer -

Both statements are correct दोनो कथन सही ह

Read the given statements and answer which of the following options isare correct

1 Higher temperature anomaly is observed in the northern hemisphere

2 Differential heating is absent in Northern Hemisphere

िदए गए कथनो को पढ़ और उर द िक िन म स कौन सास िवक सही ह

1 उरी गोलाध म उ तापमान िवसगित पायी जाती ह

2 उरी गोलाध म अतर ऊन अनपथत होती ह

1 Both Statements 1 and 2 are correct दोनो कथन 1 और 2 सही ह

2 Both Statements 1 and 2 are wrong दोनो कथन 1 और 2 गलत ह

3 Statement 1 is wrong and Statement 2 is correct कथन 1 गलत ह और कथन 2 सही ह

4 Statement 1 is correct and Statement 2 is wrong कथन 1 सही ह और कथन 2 गलत ह

Correct Answer -

Statement 1 is correct and Statement 2 is wrong कथन 1 सही ह और कथन 2 गलत ह

Read the given statements and answer which of the following options isare correct

(1) Plutonic rocks are intrusive type of igneous rocks

(2) It cools very slowly because the surrounding rock serves as insulation around the intrusion of magma

िदए गए कथनो को पढ़ और उर द िक िन म स कौन सास िवक सही ह

(1) िवतलीय शल अतवधी कार क आश शल ह

(2) यह बत धीर-धीर ठडा होता ह ोिक आस-पास क शल मा क अतवधन क चारो ओर रोधन क प म कायकरत ह

1 Both statements are wrong दोनो कथन गलत ह

2 Both statements are correct दोनो कथन सही ह

3 First statement is wrong and second statement is correct पहला कथन गलत ह और दसरा कथन सही ह

4 First statement is correct and second statement is wrong पहला कथन सही ह और दसरा कथन गलत ह

Correct Answer -

Both statements are correct दोनो कथन सही ह

45)

46)

47)

48)

The dust and ash material hurled from the volcanoes are termed as

ालामखी स िनकलन वाली धल और राख सामी को _______ क प म कहा जाता ह

1 Pyroclasc पाइरोाक

2 Hyperclastic हाइपराक

3 Hepiroclastic हिपरोाक

4 Cirroclastic िसरोाक

Correct Answer -

Pyroclasc पाइरोाक

The vertical difference in elevation between a low tide and high tide is referred as

कम ार और उ ार क बीच ऊचाई म लबवत अतर _____ स सदिभत होता ह

1 Tidal slope ारीय ढलान

2 Tidal elevation ारीय उयन

3 Tidal range ारीय परास

4 Tidal height ारीय ऊचाई

Correct Answer -

Tidal range ारीय परास

The maximum biodiversity is found in which of the following regions िनिलखत ो म स अिधकतमजव िविवधता िकसम पायी जाती ह

1 Amazon Basin अमज़न बिसन

2 East Indies ई इडीज

3 Congo Basin कागो बिसन

4 West indies व इडीज

Correct Answer -

Amazon Basin अमज़न बिसन

The cultivation of rice crop produces_______ चावल की फसल की खती ______ का उादन करती ह

1 SO2

49)

50)

51)

2 CH4

3 CFCs

4 CO2

Correct Answer -

CH4

The pressure system with higher pressure at the centre is called__________

क म उ दबाव वाली दबाव णाली को _______ कहा जाता ह

1 front अ

2 depression अवनमन

3 cyclone चवात

4 anti-cyclone ितचवात

Correct Answer -

anti-cyclone ितचवात

The Himalayan region is poor in mineral resources because िहमालयी खिनज ससाधनो म समनही ह ोिक

1 The displacement of rock strata has disturbed the arrangement of rocks and made it complex

शलीय परत क िवथापन न चानो की वथा को अवथत कर िदया ह और इस जिटल बना िदया ह

2 The climate conditions are not suitable for exploitation of minerals

जलवाय की थित खिनजो क दोहन क िलए उपय नही ह

3 The terrain makes explanation of minerals difficult and very costly due to transportation difficulties

भ-भाग परवहन की किठनाइयो क कारण खिनजो का दोहन मल और बत महगा बना दता ह

4 It is made up of crystalline rocks यह िलीय चानो स बना ह

Correct Answer -

The displacement of rock strata has disturbed the arrangement of rocks and made it complex

शलीय परत क िवथापन न चानो की वथा को अवथत कर िदया ह और इस जिटल बना िदया ह

The process through which the moisture is added to the atmosphere by vegetation is termed as

वह िया िजसक माम स वनित ारा वातावरण म नमी िमलायी जाती ह _______ क प म जानी जाती ह

52)

53)

54)

1 Condensation सघनन

2 Evapotranspiration वान-उजन

3 Radiation िविकरण

4 Precipitation वषण

Correct Answer -

Evapotranspiration वान-उजन

The process through which the terrestrial heat is transferred to air by direct contact is termed as

वह िया िजसम सपक ारा थलीय ऊा वाय म थानातरत हो जाती ह ______ क प म जानी जाती ह

1 Conduction चालन

2 Convection सवहन

3 Insolation आतपन

4 Radiation िविकरण

Correct Answer -

Conduction चालन

The largest area under mangroves is in which of the following statesunion territory

मोव क अतगत िनिलखत राोसघ शािसत दशो म स सबस बड़ा कौन सा ह

1 Andaman and Nicobar अमान और िनकोबार

2 Andhra Pradesh आ दश

3 West Bengal पिम बगाल

4 Gujarat गजरात

Correct Answer -

West Bengal पिम बगाल

The longitudinal transverse and surface waves in an earthquake originate from

भकप म दशातर अनथ और सतह तरग यहा उ होती ह

1 The focus on the surface of the Earth पी क सतह पर क -िबद म

2 The focus within the body of the Earth पी क भीतर क -िबद म

3 The epicenter within the body of the Earth पी क भीतर उपरक म

55)

56)

57)

4 The epicenter on the surface of the Earth पी क सतह पर उपरक म

Correct Answer -

The focus within the body of the Earth पी क भीतर क -िबद म

The down slope movement of material due to gravity is called______

गाकषण क कारण पदाथ की अनढाल गित को ______ कहा जाता ह

1 mass movement पदाथ सचलन

2 deposition िनप

3 erosion रण

4 volcanic movement ालामखीय सचलन

Correct Answer -

mass movement पदाथ सचलन

Shimla is cooler than Amritsar although both are on the same latitude This is because

िशमला म अमतसर स अिधक ठड ह हालािक दोनो समान अाश पर ह ऐसा ह ोिक

1 Shimla is at a greater height above sea level than Amritsar अमतसर की तलना म िशमला सम तल स अिधकऊचाई पर ह

2 Shimla is further north िशमला उर की ओर ह

3 Shimla is farther from the equator िशमला भम रखा स आग ह

4 Their longitudes differ उनकी दशातर रखाए िभ ह

Correct Answer -

Shimla is at a greater height above sea level than Amritsar अमतसर की तलना म िशमला सम तल स अिधकऊचाई पर ह

lsquoTempo of Urbanizationrsquo measures which of the following

lsquoशहरीकरण का टपोrsquo िनिलखत म स कौन सा उपाय ह

1 Speed of urbanizaon शहरीकरण की गित

2 None of the above इनम स कोई नही

3 Inequality of urbanizaon शहरीकरण की असमानता

4 Current level of urbanizaon शहरीकरण का वतमान र

Correct Answer -

58)

59)

60)

Speed of urbanizaon शहरीकरण की गित

Out of the following options choose the INCORRECT statement

िनिलखत िवको म स गलत कथन का चयन कर

1 The clear tracts in the equatorial region recover rapidly भम रखा म भभाग तजी स ठीक हो जात ह

2 The stable communities include a redwood forest a pine forest at high elevations

थर समदायो म एक रडवड वन उ ऊचाई पर एक दवदार वन शािमल ह

3 Any ecosystem moves towards maximum biomass and stability to survive

कोई भी पारथितकी त जीिवत रहन क िलए अिधकतम जवसहित और थरता की तरफ असर होता ह

4 Tropical rain forests near equator are stable ecosystems

भम रखा क पास उकिटबधीय वषा वन थर पारथितक त ह

Correct Answer -

The clear tracts in the equatorial region recover rapidly भम रखा म भभाग तजी स ठीक हो जात ह

Seasonal contrasts are maximum in मौसमी िवषमता अिधकतम ह

1 Mid latitudes म अाश म

2 Low attitudes िन अाश म

3 High latitudes उ अाश म

4 Subtropics उपोकिटबधीय म

Correct Answer -

Mid latitudes म अाश म

In India which type of forest among the following occupies the largest area

भारत म िनिलखत म स िकस कार क वन सबस बड़ा फल आािदत करत ह

1 Sub-tropical Dry Evergreen Forest उप उकिटबधीय श सदाबहार वन

2 Mountain Wet Temperate Forest पवतीय आ शीतो वन

3 Tropical Moist Deciduous Forest उकिटबधीय आ पणपाती वन

4 Tropical Wet Evergreen Forest उकिटबधीय आ सदाबहार वन

Correct Answer -

Tropical Moist Deciduous Forest उकिटबधीय आ पणपाती वन

61)

62)

63)

64)

What is the proportion of lsquoJuvenile Populationrsquo (0-14 years) in India as per 2011Census

2011 की जनगणना क अनसार भारत म जवनाइल पॉपलशन यानी िकशोर जनस या (0-14 वष) का अनपात ाह

1 3076 of total population कल जनस या का 3076

2 2764 of total population कल जनस या का 2764

3 2933 of total population कल जनस या का 2933

4 3354 of total population कल जनस या का 3354

Correct Answer -

3076 of total population कल जनस या का 3076

What is the Belfast famous for बलफा िकसक िलए मशर ह

1 Belt of cotton textile industry कपास व उोग क

2 Ship-building industry जहाज िनमाण उोग

3 Agricultural machinery किष उपकरण

4 Aero planes manufacturing वाययान िनमाण

Correct Answer -

Ship-building industry जहाज िनमाण उोग

What is the most important occupation in tropical monsoon lands

उकिटबधीय मॉनसन भिम म सबस महपण वसाय ा ह

1 Mining खनन

2 Cattle rearing मवशी पालन

3 Agriculture किष

4 Nomadic herding नोमािडक जड़ी-बिटया

Correct Answer -

Agriculture किष

What is the most important characteristics of the islands (Indian) located in the Arabian sea

अरब सागर म थत ीपो (भारतीय) की सबस महपण िवशषता ा ह

ी ो

65)

66)

67)

1 There are all of coral origins सभी कोरल मल क ह

2 There are all very small in size य सभी आकार म बत छोट ह

3 They have a very dry climate इनकी जलवाय बत श ह

4 They are extended parts of the mainland व महाीप क िवारत िह ह

Correct Answer -

There are all of coral origins सभी कोरल मल क ह

What do the basalt layers of the Deccan indicate डन की बसा परत ा इिगत करती ह

1 All of the above उपरो सभी

2 Huge volcanic eruptions in the distant past दरथ अतीत म िवशाल ालामखीय िवोट

3 The immense erosional activity of the rivers निदयो की िवशाल रण गितिविध

4 The influence of weathering मौसम का भाव

Correct Answer -

Huge volcanic eruptions in the distant past दरथ अतीत म िवशाल ालामखीय िवोट

In the structure of planet Earth below the mantle the core is mainly made up of_____

पी ह की सरचना म मटल क नीच कोर म प स______ स िनिमत होती ह

1 aluminium एमीिनयम

2 silicon िसिलकॉन

3 chromium ोिमयम

4 iron लोहा

Correct Answer -

iron लोहा

One of the major Mid Oceanic Ridge is found in मख म-महासागर चोिटयो म स एक ______ म पायाजाता ह

1 Mid Pacific Ocean म शात महासागर

2 Mid Atlantic Ocean म अटलािटक महासागर

3 Mid Indian Ocean म भारतीय महासागर

4 Mid Arctic Ocean म आक िटक महासागर

68)

69)

70)

71)

Correct Answer -

Mid Atlantic Ocean म अटलािटक महासागर

Magma that reaches the Earthrsquos surface and then solidifies is called________

मा जो पी की सतह तक पचती ह और िफर ठोस हो जाती ह ________कहलाती ह

1 quartz ाटज

2 lava लावा

3 granite नाइट

4 silicates िसिलकट

Correct Answer -

lava लावा

Isotherms are the lines of equal_______ समताप रखाए समान _______की रखाए होती ह

1 pressure दाब

2 temperature तापमान

3 rainfall वषा

4 height ऊचाई

Correct Answer -

temperature तापमान

Mark the correct sequence of passes in the Western Ghats from north to south

पिमी घाटो म उर स दिण तक दर क सही अनम को िचित कर

1 Thalghat Palghat Bhorghat थलगघाट पालघाट भोरघाट

2 Thalghat Bhorghat Palghat थलघाट भोरघाट पालघाट

3 Bhorghat Thalghat Palghat भोरघाट थलघाट पालघाट

4 Palghat Bhorghat Thalghat पालघाट भोरघाट थलघाट

Correct Answer -

Thalghat Bhorghat Palghat थलघाट भोरघाट पालघाट

Which of the following does not have influence over the climate in India

ि ि ि ी ी

72)

73)

िनिलखत म स िकसका भाव भारत की जलवाय पर नही पड़ता ह

1 Ocean currents सागर की लहर

2 Nearness to equator भम रखा स िनकटता

3 Monsoons मानसन

4 Presence of Indian ocean भारतीय महासागर की उपथित

Correct Answer -

Ocean currents सागर की लहर

Which of the following cloud types has the characteristics like vertical tall narrow and puffy

िनिलखत म स िकस कार क मघो म लबवत लबी सकीण और थलता जसी िवशषताए ह

1 Cumulonimbus तफानी मघ

2 Cumulus मघ पज

3 Cirrocumulus पाभ कपासी मघ

4 Nimbostratus वषारी मघ

Correct Answer -

Cumulus मघ पज

Which of the following statement is INCORRECT about Crude Birth Rate

िनिलखत स कौन सा कथन अशोिधत ज दर क बार म सही नही ह

1 It cannot be used for comparing fertility level between two countries with different population characteristics

इसका उपयोग िविभ जनसा िवशषताओ वाल दो दशो क बीच जनन र की तलना क िलए नही िकया जा सकता ह

2 It is a standardized measure of fertility

यह जनन मता का मानकीकत उपाय ह

3 It is effected by the age-sex composition of the population

यह आबादी की आय-िलग सरचना स भािवत होता ह

4 It is expressed per 1000 population in a given geographical unit

यह िकसी दी गई भौगोिलक इकाई म ित 1000 जनसा पर िकया जाता ह

Correct Answer -

It is a standardized measure of fertility

यह जनन मता का मानकीकत उपाय ह

74)

75)

76)

77)

Which of the following state in India experienced negative decadal growth rate during 2001 to 2011census

भारत म िनिलखत म स िकस रा म वष 2001 स 2011 की जनगणना क दौरान नकाराक िगरावट दर ई

1 Tripura िपरा

2 Nagaland नागालड

3 Haryana हरयाणा

4 Odisha ओिडसा

Correct Answer -

Nagaland नागालड

Which of the following is NOT a characteristic of peninsular rivers

िनिलखत म स कौन सी िवशषता ायीपीय निदयो म नही होती ह

1 Flow through shallow valleys उथल घािटयो क माम स वाह

2 Seasonal flow मौसमी वाह

3 Little erosional activity थोड़ी कटावदार गितिविध

4 Meandering tendency often shifting their beds घमावदार वि अर अपन तटो को थानातरत करना

Correct Answer -

Meandering tendency often shifting their beds घमावदार वि अर अपन तटो को थानातरत करना

Which of the following gases in the atmosphere absorbs heat from the Sunrsquos radiation and the Earthssurface

वायमडल म िनिलखत म स कौन सी गस सय क िविकरण और पी की सतह स ऊा को अवशोिषत करती ह

1 Neon िनयॉन

2 Carbon dioxide काबन डाइऑाइड

3 Argon आगन

4 Nitrogen नाइट ोजन

Correct Answer -

Carbon dioxide काबन डाइऑाइड

Which of the following kind of settlement pattern is found at the confluence of rivers

ि ि ि ि ो

78)

79)

80)

िनिलखत म स िकस कार का वथापन पटन निदयो क सगम पर पाया जाता ह

1 Triangular Paern िकोणीय पटन

2 Circular or Semi-Circular Paern परप या अध-परप पटन

3 Nebular Paern नबलर पटन

4 Star ndashShaped Paern ार-आकार का पटन

Correct Answer -

Triangular Paern िकोणीय पटन

Which one was not the objective of the Biosphere Reserve Projects launched by the UNESCO

यनो ारा श की गई सरित जवमडल परयोजनाओ का उ इनम स कौन सा नही था

1 To promote teaching and research िशण और अनसधान को बढ़ावा दना

2 To make agriculture sustainable किष को दीघकािलक बनाना

3 To conserve ecosystems पारथितक त को सरित करना

4 To conserve genetic diversity for a longtime लब समय तक अनवािशक िविवधता को सरित करना

Correct Answer -

To make agriculture sustainable किष को दीघकािलक बनाना

Which region of the Earth surface receives the highest amount of insulation

पी सतह का कौन सा तापावरोधन की उतम माा ा करता ह

1 Land mass थलखड

2 Savannah region सवाना

3 Water bodies जल िनकाय

4 Tropical desert उकिटबधीय रिगान

Correct Answer -

Tropical desert उकिटबधीय रिगान

Which one of the following is not a biodiversity hotspot

िनिलखत म स कौन सा जव िविवधता का म जगह नही ह

1 Eastern Himalaya पव िहमालय

2 Eastern Ghats पव घाट

81)

82)

83)

3 Indo-Myanmar भारत-ामार

4 Westerm Ghats पिमी घाट

Correct Answer -

Eastern Ghats पव घाट

Which one of the following is NOT a part of the World Network of Biosphere Reserves based on theUNESCO Man and Biosphere Programme

यनो मन और बायोीयर कायम क आधार पर िनिलखत म स कौन बायोीयर रजव क िव नटवक कािहा नही ह

1 Gulf of Mannar मार की खाड़ी

2 Seshachalam शषाचलम

3 Sunderban सदरबन

4 Nilgiri नीलिगर

Correct Answer -

Seshachalam शषाचलम

Which one of the following is an example of ldquodesert vegetationrdquo

िनिलखत म स कौन मथलीय वनित का एक उदाहरण ह

1 Mosses and lichens दलदल और शवाल

2 Temperate grassland समशीतो घास क मदान

3 Coniferous forest शकधारी वन

4 Acacia and cactus एकािसया और कस

Correct Answer -

Acacia and cactus एकािसया और कस

Which one of the following reflects more sunlight िनिलखत म स कौन सा सय की रोशनी को अिधकपरावितत करता ह

1 Paddy crop land धान फसल भिम

2 Land covered with fresh snow ताजा बफ स आािदत भिम

3 Sand desert रतीली रिगान

4 Prairie land यरी भिम

84)

85)

86)

87)

Correct Answer -

Land covered with fresh snow ताजा बफ स आािदत भिम

Which layer of the atmosphere is in contact with the surface of the earthrsquos oceans

वायमडल की कौन सी परत पी क महासागरो की सतह क सपक म ह

1 Stratosphere समताप मडल

2 Mesosphere म मडल

3 Hydrosphere जलमडल

4 Troposphere ोभ मडल

Correct Answer -

Troposphere ोभ मडल

Mediterranean Sea is a border of which of the following countries भम सागर िनिलखत दशो म सिकसकी सीमा ह

1 None of these इनम स कोई नही

2 Iraq इराक

3 Lebanon लबनान

4 Jordan जॉडन

Correct Answer -

Lebanon लबनान

Benguela ocean currents are found along which coast बगएला महासागर धाराए िकस तट क साथ पायीजाती ह

1 East Coast of South America दिण अमरका क पव तट

2 East Coast of Africa अीका क पव तट

3 West Coast of South America दिण अमरका क पिमी तट

4 West Coast of Africa अीका क पिमी तट

Correct Answer -

West Coast of Africa अीका क पिमी तट

88)

89)

90)

Due to tension a block of land on one side being pushed up or upthrown relative to the downthrown blockis referred as

तनाव क कारण नीच फ क ए खड क साप भिम का एक खड एक ओर स ऊपर धकला जाता ह या ऊपर की ओरफ का जाता ह यह _____ क प म सदिभत ह

1 Thrust fault प श

2 Normal fault सामा श

3 Reverse fault म श

4 Strike slip fault नितलब सपण श

Correct Answer -

Normal fault सामा श

Inter-tropical doldrums is a zone of ______ अतर-उकिटबधीय डोलड ______ का एक ह

1 Frontolysis टोलायिसस

2 Convergence अिभसरण

3 Inter-tropical divergence zone अतर-उकिटबधीय िवचलन

4 Local wind थानीय वाय

Correct Answer -

Convergence अिभसरण

The Horse Latitudes are regions located at about _____ north and south of the equator

हॉस अाश भम रखा क उर और दिण म लगभग _____ पर थत ह

1 30ndash60 degree Latitude 30-60 िडी अाश

2 0ndash5 degree Latitude 0-5 िडी अाश

3 30 degree Latitude 30 िडी अाश

4 60ndash90 degree Latitude 60-90 िडी अाश

Correct Answer -

30 degree Latitude 30 िडी अाश

Generally evaporation is high over which part of the Earth

आम तौर पर पी क िकस भाग पर वाीकरण अिधक होता ह

1 Equatorial maritime भमवत समीय ी ी

91)

92)

2 Equatorial continental भमवत महाीपीय

3 Polar maritime वीय समीय

4 Polar continental वीय महाीपीय

Correct Answer -

Equatorial maritime भमवत समीय

A very high temperature during summer in north western India leads to what type of climaticcondition in south

उर पिमी भारत म गम क दौरान बत अिधक तापमान होन क कारण दिण म िकस कार की जलवाय थितउ करता ह

1 Depression over arabian sea अरब सागर पर अवनमन

2 Failure monsoon मानसन िवफलता

3 Successful monsoon मानसन सफलता

4 Cyclones चवात

Correct Answer -

Successful monsoon मानसन सफलता

Lightning and thunder are the resultant effect when तिड़त और गजन परणामी भाव ह जब

1 Two massive clouds hit powerfully each other first lightning is produced and later sound is produced

दो बड़ बादल एक दसर स शशाली ढग स टकरात ह पहल आकाशीय िवदयत उ होता ह और बाद म िन उहोती ह

2 Two massive clouds come into contact with the powerful wind collision this results into first sound and thenlightning

दो बड़ बादल शशाली पवन सघ क सपक म आत ह इसका परणामप पहल िन और िफर आकाशीय िवदयतउ होता ह

3 None of the above उपरो म स कोई भी नही

4 A high density cloud contains positively and negatively charged electric ions and when this interacts light andsound are simultaneously produced

एक उ घन बादल म धनाक और ऋणाक आविशत िवदयत आयन होत ह और जब यह परर भाव डालत ह तोकाश और िन एक साथ उािदत होती ह

Correct Answer -

A high density cloud contains positively and negatively charged electric ions and when this interacts light andsound are simultaneously produced

औ ि ि ो औ ो

93)

94)

95)

एक उ घन बादल म धनाक और ऋणाक आविशत िवदयत आयन होत ह और जब यह परर भाव डालत ह तोकाश और िन एक साथ उािदत होती ह

Doon Valley is able to grow rice because दन घाटी चावल उगान म सम ह ोिक

1 Other crops cannot be grown वहा अ फसलो को उगाया नही जा सकता ह

2 People in the valley are rice eaters घाटी म लोग चावल खान वाल ह

3 There is a huge export demand of rice वहा चावल की भारी िनयात माग ह

4 It has warm summer and snow melt waters for irrigation

वहा गिमया गम होती ह िसचाई क िलए बफ का िपघला आ पानी होता ह

Correct Answer -

It has warm summer and snow melt waters for irrigation

वहा गिमया गम होती ह िसचाई क िलए बफ का िपघला आ पानी होता ह

CANCELLED

In the geological time scale the Mesozoic Era DOES NOT contains which of the following periods

भगभय समय पमान पर मजीवी यग म िन कालो म स कौन नही ह

1 Triassic ट ाइऐिसक

2 Jurassic जरिसक

3 Cretaceous चाकमय

4 Carboniferous काबनी

Correct Answer -

Carboniferous काबनी

96)

1 P-3 Q-4 R-2 S-1

2 P-3 Q-4 R-1 S-2

3 P-3 Q-4 R-1 S-2

4 P-4 Q-3 R-2 S-1

Correct Answer -

P-4 Q-3 R-2 S-1

1 P-3 Q-1 R-4 S-2

2 P-3 Q-4 R-1 S-2

3 P-3 Q-2 R-4 S-1

97)

98)

4 P-2 Q-1 R-4 S-3

Correct Answer -

P-3 Q-4 R-1 S-2

1 P-3 Q-1 R-4 S-2

2 P-2 Q-3 R-4 S-1

3 P-2 Q-1 R-3 S-4

4 P-4 Q-2 R-1 S-3

Correct Answer -

P-3 Q-1 R-4 S-2

99)

100)

1 P-3 Q-2 R-4 S-1

2 P-1 Q-2 R-3 S-4

3 P-2 Q-3 R-1 S-4

4 P-4 Q-3 R-2 S-1

Correct Answer -

P-2 Q-3 R-1 S-4

ldquoHuman geography is the study of changing relationship between the unresting man and the unstableearthrdquo was defined by

lsquolsquoमानव भगोल ाकल आदमी और अथर पी क बीच सबध परवतन का अयन हrdquo ______ ारा परभािषत िकया गयाथा

1 J Brunches ज चस

2 EC Semple ईसी सल

3 HJ Mackinder एच ज मिकदर

4 PV Blache पीवी च

Correct Answer -

EC Semple ईसी सल

Sedimentary rocks are finally and ultimately derived from the____________

अवसादी चान अततः ________ स ा की जाती ह

1 action of earth movements पी की गितिविधयो

2 marine deposit समी िनप

3 weathering of metamorphic rocks पातरत चानो क अपय

4 weathering of igneous rocks आय चानो क अपय

Correct Answer -

weathering of igneous rocks आय चानो क अपय

Page 21: High School Teacher Eligibility Test- BOARD PROFESSIONAL ...peb.mp.gov.in/results/RESULT_18/HST_RES18/Final_anwser_key/HST… · M a ndl a / मंड ल ... Under the Madhya Pradesh

12)

13)

14)

4 Its direct of rotation is the same as that of Earth घणन की िदशा पी क समान ही ह

Correct Answer -

Its period of revolution around the Earth is the same as its period of rotation around its own axis

इसकी पी क चारो ओर परमण की अविध उसकी अपनी धरी क चारो ओर घणन की अविध क समान ह

CANCELLED

The pebbles that are faceted by the sand-blasting and shaped polished by the wind abrasions are known as

पवन अपघषन ारा पॉिलश रत-िवोिटत और साच म ढला ककड़ ___________ क प म जाना जाता ह

1 Dreikanter िकोणक

2 Pediments िकोिनका

3 Inselberg इलबग

4 Dunes टीबा

Correct Answer -

Dreikanter िकोणक

CANCELLED

Astronomical unit is the average distance between खगोलीय इकाई ______ क बीच की औसत दरी ह

1 Earth and Mars पी और मगल

2 Earth and mercury पी और बध

3 Earth and moon पी और चमा

4 Earth and Sun पी और सय

Correct Answer -

Earth and Sun पी और सय

During cold weather season in the northern plains there will be an inflow of cyclonic disturbancesfrom the _________ directions

शीत मौसम क दौरान उरी मदानी इलाको म _________ िदशाओ स चवात सबधी गड़बड़ी का अतवाह होगा

1 East and Northwest पव और उरपिम

2 East and Northeast पव और पवर

3 West and East पिम और पव

ि औ ि

15)

16)

17)

4 West and Northwest पिम और उरपिम

Correct Answer -

West and Northwest पिम और उरपिम

During an earth quake the velocity of the body waves will________ along with the increase in densityof the material it is passing through

भकप क दौरान लहरो क ऊपरी भाग का वग घन म व क साथ-साथ ________ जो इसस गजरन वाली वको आग बढाएगी

1 not change नही बदलगा

2 increase initially and then decrease शआत म बढ़गा और िफर घटगा

3 increase बढ़गा

4 decrease घटगा

Correct Answer -

increase बढ़गा

The Clouded Leopard National park is situated in which of the following states

िनिलखत म िकस रा म धिमल तदआ रा ीय उान (ाउडड लपड नशनल पाक ) थत ह

1 Tripura िपरा

2 Uttar Pradesh उर दश

3 Assam असम

4 Mizoram िमजोरम

Correct Answer -

Tripura िपरा

Usually the land surfaces are heated more quickly than the water surfaces because _____________

आम तौर पर जल सतहो की तलना म भिम सतह अिधक तजी स गम होती ह ोिक _____________ ह

1 the specific heat of water is higher than land पानी की िविश ऊा भिम स अिधक

2 the specific heat of water is lesser than land पानी की िविश ऊा भिम स कम होती

3 the latent heat of water is higher than the land पानी की अतिनिहत ऊा भिम स अिधक

4 the land reflects more heat radiation than water भिम पानी की तलना म अिधक ऊा क िविकरण को पराविततकरती

18)

19)

20)

21)

Correct Answer -

the specific heat of water is higher than land पानी की िविश ऊा भिम स अिधक

The longest shore-line is along the state of सबस लबी समतटीय रखा िन रा क साथ ह

1 Maharashtra महारा

2 Orissa उड़ीसा

3 Kerala करल

4 Gujarat गजरात

Correct Answer -

Gujarat गजरात

The position when the Earth is farthest from the Sun is known as

जब पी सय स सबस दर होती ह तो उस थित को िन नाम स जाना जाता ह

1 Perihelion उपसौर

2 Vernal Equinox बसत िवषव

3 Aphelion अपसौर

4 Autumnal Equinox शराल िवषव

Correct Answer -

Aphelion अपसौर

The seasonal reversal of winds is the typical characteristic of

हवाओ का मौसमी परवतन ______ की सामा िवशषता ह

1 Mediterranean climates only कवल भमसागरीय जलवाय

2 All of the above climates उपय सभी मौसम

3 Monsoon climate only कवल मानसन जलवाय

4 Equatorial climate only कवल भमरखीय जलवाय

Correct Answer -

Monsoon climate only कवल मानसन जलवाय

In _________ rocks the minerals will occurs in beds or layers

ो ि ो ो

22)

23)

24)

______ चानो म खिनज तल या परतो म होत ह

1 metamorphic कायातरत

2 igneous and metamorphic आय और कायातरत

3 igneous आय

4 sedimentary अवसादी

Correct Answer -

sedimentary अवसादी

Black soil is ideal for the cultivation of cotton as कपास की खती क िलए काली िमी आदश ह ोिक

1 Its colour is black यह काली होती ह

2 It is found on plateau regions यह पठार ो म पायी जाती ह

3 It is made up of lava यह लावा स बनी होती ह

4 It can retain moisture यह नमी को बरकरार रख सकती ह

Correct Answer -

It can retain moisture यह नमी को बरकरार रख सकती ह

The National Survey and Mapping Organization of the country works under the Department of___________

दश का रा ीय सवण और मानिचण सगठन ___________ िवभाग क अतगत काय करता ह

1 Space अतर

2 Science and Technology िवान और तकनीक

3 Culture सित

4 Tourism पयटन

Correct Answer -

Science and Technology िवान और तकनीक

Palk strait separates India from पाक जलडमम भारत स _____ को अलग करता ह

1 Pakistan पािकान

2 Andaman Island अडमान ीप

3 China चीन

25)

26)

27)

4 Sri Lanka ीलका

Correct Answer -

Sri Lanka ीलका

Which among the following state is the major producer of Bauxite in India

िनिलखत म स कौन सा रा भारत म बॉाइट का मख उादक ह

1 Madhya Pradesh मदश

2 Rajasthan राजथान

3 Goa गोवा

4 Orissa उड़ीसा

Correct Answer -

Orissa उड़ीसा

Which of the following states DOES NOT share border with Chhattisgarh

िनिलखत म स कौन सा रा छीसगढ़ क साथ सीमा साझा नही करता ह

1 Telangana तलगाना

2 Uttar Pradesh उर दश

3 Bihar िबहार

4 Andhra Pradesh आ दश

Correct Answer -

Bihar िबहार

Which of the following statements is INCORRECT with respect to parallels of latitudes

अाश क समानातरो क सबध म िन निलखत म स कौन सा कथन गलत ह

1 A line joining places of equal latitude is known as parallel of largest

समान अाश क थानो को जोड़न वाली रखा को िवशालतम क समानातर क प म जाना जाता ह

2 They stat from equator and run parallels to it

व भम रखा स ारभ होत ह और इसक समानातर चलत ह

3 All parallels are equal in length सभी समातर लबाई म समान ह

4 All parallels are drawn as circles on the globe ोब पर सभी समानातर वो क प म खीच जात ह

28)

29)

30)

31)

Correct Answer -

All parallels are equal in length सभी समातर लबाई म समान ह

Which of the following Indian states is also known as a lsquoLand of Red river and Blue Hillsrsquo

िनिलखत म स िकस भारतीय रा को लाल नदी और नीली पहािड़यो की भिम क नाम स जाना जाता ह

1 Uttarkhand उराखड

2 Assam असम

3 Meghalaya मघालय

4 Arunachal Pradesh अणाचल दश

Correct Answer -

Assam असम

In spatial analysis of settlement Rn = 215 indicates which type of settlement arrangement

िनपटान क थािनक िवषण म Rn = 215 यह इिगत करता ह िक िकस कार की िनपटान वथा ह

1 Uniform यिनफॉम

2 Semi-Clustered समी- ल टर

3 Clustered ल टर

4 Random रडम

Correct Answer -

Uniform यिनफॉम

Who are known as the lsquoYellow Peoplersquo lsquoयलो पीपलrsquo क प म कौन जाना जाता ह

1 Mongoloids मोगोलोइडस

2 Nigroids नीोइडस

3 Australoids ऑ लॉइडस

4 Caucasoids कॉकसोइडस

Correct Answer -

Mongoloids मोगोलोइडस

ि ि ो ौ ि

32)

33)

34)

Who publishes the topographical map of India भारत क थलाकितक मानिच को कौन कािशत करता ह

1 Geographical Survey of India भारत का भौगोिलक सवण

2 Government of India भारत सरकार

3 Geological Survey of India भारत क भगभय सवण

4 Survey of India भारत का सवण

Correct Answer -

Survey of India भारत का सवण

Who among the following claimed geography to be the lsquoEcology of Manrsquo

िनिलखत म स िकसन भगोल को मन का पारथितकी कहा ह

1 Alfred Hener अड हटनर

2 Vidal-de la Blache वाइडल-िड लॉ ॉश

3 Oo Schluter ओटो टर

4 Harlan Barrow हरलन बारो

Correct Answer -

Harlan Barrow हरलन बारो

Who among the following is regarded as the founder of humanistic approach in geography

िनिलखत म स िकस भगोल म मानवतावादी िकोण का सथापक माना जाता ह

1 William Bunge िविलयम बग

2 Yi-Fu-Tuan यी-फ- यान

3 Brain JL Berry न जएल बरी

4 Richard Peet रचड पीट

Correct Answer -

Yi-Fu-Tuan यी-फ- यान

Who prepared Lorenz curve लोरज व िकसन तयार िकया

1 Geddes गडस

2 None of these इनम स कोई नही

3 Griffith Taylor ििफथ टलर

35)

36)

37)

4 Max U Lorenz म य लोरज

Correct Answer -

Max U Lorenz म य लोरज

Gulf Streams are the currents of which of the following oceans

खाड़ी की धाराए िनिलखत महासागरो म स िकसकी धाराए ह

1 North Atlantic Ocean उरी अटलािटक महासागर

2 North Pacific Ocean उरी शात महासागर

3 Arabian Sea अरब सागर

4 South Pacific Ocean दिण शात महासागर

Correct Answer -

North Atlantic Ocean उरी अटलािटक महासागर

Disintegration wearing away and removal of rock material is generally referred as

िशला पदाथ (रॉक सामी) का टटना िमटना और हटना आमतौर पर ________ क प म सदिभत िकया जाता ह

1 Shattering िवसकारक

2 Denudation अनाादन

3 Fault श

4 Decomposition िवयोजन

Correct Answer -

Denudation अनाादन

Variations in the length of day time and night from season to season are due to

मौसम स मौसम परवतन पर िदन क समय और रात क समय की अविध म िभताए िन कारण स होती ह

1 The Earthrsquos revolution round the Sun in an elliptical manner पी का दीघवाकार तरीक स सय क चारो घणन

2 The Earthrsquos rotation on its axis पी का इसकी धरी पर घणन

3 Revolution of the Earth on a tilted axis नत अ पर पी का घणन

4 Latitudinal position of the place थान की अाश थित

Correct Answer -

Revolution of the Earth on a tilted axis नत अ पर पी का घणन

38)

39)

40)

Point out the correct sequence of mountain ranges from north to south

उर स दिण तक पवत खलाओ क सही अनम को इिगत कर

1 Great Himalaya Middle Himalaya Outer Himalaya Trans Himalaya

महान िहमालय म िहमालय बा िहमालय परा िहमालय

2 Middle Himalaya Great Himalaya Trans Himalaya Outer Himalaya

म िहमालय महान िहमालय परा िहमालय बा िहमालय

3 Outer Himalaya Middle Himalaya Great Himalaya Trans Himalaya

बा िहमालय म िहमालय महान िहमालय परा िहमालय

4 Trans Himalaya Great Himalaya Middle Himalaya Outer Himalaya

परा िहमालय महान िहमालय म िहमालय बा िहमालय

Correct Answer -

Trans Himalaya Great Himalaya Middle Himalaya Outer Himalaya

परा िहमालय महान िहमालय म िहमालय बा िहमालय

Sunrsquos halo is produced by the refraction of light in सय का भामडल ______ म काश क अपवतन ाराउ होता ह

1 Ice crystals in Cirrro-Cumulus clouds पाभ-कपास मघो क बफ िल

2 Ice crystal in Cirrus clouds पाभमघो क बफ िल

3 Dust particles in Stratus clouds री मघो क धल कण

4 Water vapour in Stratus clouds री मघो क जल वा

Correct Answer -

Ice crystal in Cirrus clouds पाभमघो क बफ िल

Read the given statements and answer which of the following options isare correct

(1) The minerals present in the rocks exposed to atmosphere are not subjected to alteration

(2) Oxidation is one of the processes of chemical weathering

िदए गए कथनो को पढ़ और उर द िक िन म स कौन सास िवक सही ह

(1) वायमल स अनावत शल म उपथत खिनज परवतन क अधीन नही होता ह

(2) ऑीकरण रासायिनक अपय की ियाओ म स एक ह

ो ो

41)

42)

1 Both statements are wrong दोनो कथन गलत ह

2 Both statements are correct दोनो कथन सही ह

3 First statement is wrong and second statement is correct पहला कथन गलत ह और दसरा कथन सही ह

4 First statement is correct and second statement is wrong पहला कथन सही ह और दसरा कथन गलत ह

Correct Answer -

First statement is wrong and second statement is correct पहला कथन गलत ह और दसरा कथन सही ह

Read the given statements and answer which of the following options isare correct

1 Sunrsquos short waves enter the earth partially heating the atmosphere

2 Heated earth surface from the sun produces broader waves which interacts and heats the atmosphere

िदए गए कथन को पढ़ और उर द िक िन म स कौन सास िवक सही ह

1 सय की छोटी तरग पी म आिशक प स वश करती ह और वायमडल को ऊत करती ह

2 सय स ऊत पी की सतह िवारत तरग उ करती ह जो परर भाव डालती ह और वायमडल कोऊत करती ह

1 Both Statements 1 and 2 are correct दोनो कथन 1 और 2 सही ह

2 Both Statements 1 and 2 are wrong दोनो कथन 1 और 2 गलत ह

3 Statement 1 is wrong and only Statement 2 is correct कथन 1 गलत ह और कवल कथन 2 सही ह

4 Only statement 1 is correct कवल कथन 1 सही ह

Correct Answer -

Both Statements 1 and 2 are correct दोनो कथन 1 और 2 सही ह

Read the given statements and answer which of the following options isare correct

(1)The rocks that get changed due to heat and pressure are termed as metamorphic rocks

(2)Slate is one such type of metamorphic rock

िदए गए कथनो को पढ़ और उर द िक िन म स कौन सास िवक सही ह

(1) शल जो ऊा और दाब क कारण परवितत हो जात ह उ कायातरक शलो क प म जाना जाता ह

(2) ट एक तरह का कायातरक शल ह

1 Both statements are wrong दोनो कथन गलत ह

2 Both statements are correct दोनो कथन सही ह

3 First statement is wrong and second statement is correct पहला कथन गलत ह और दसरा कथन सही ह

ी औ

43)

44)

4 First statement is correct and second statement is wrong पहला कथन सही ह और दसरा कथन गलत ह

Correct Answer -

Both statements are correct दोनो कथन सही ह

Read the given statements and answer which of the following options isare correct

1 Higher temperature anomaly is observed in the northern hemisphere

2 Differential heating is absent in Northern Hemisphere

िदए गए कथनो को पढ़ और उर द िक िन म स कौन सास िवक सही ह

1 उरी गोलाध म उ तापमान िवसगित पायी जाती ह

2 उरी गोलाध म अतर ऊन अनपथत होती ह

1 Both Statements 1 and 2 are correct दोनो कथन 1 और 2 सही ह

2 Both Statements 1 and 2 are wrong दोनो कथन 1 और 2 गलत ह

3 Statement 1 is wrong and Statement 2 is correct कथन 1 गलत ह और कथन 2 सही ह

4 Statement 1 is correct and Statement 2 is wrong कथन 1 सही ह और कथन 2 गलत ह

Correct Answer -

Statement 1 is correct and Statement 2 is wrong कथन 1 सही ह और कथन 2 गलत ह

Read the given statements and answer which of the following options isare correct

(1) Plutonic rocks are intrusive type of igneous rocks

(2) It cools very slowly because the surrounding rock serves as insulation around the intrusion of magma

िदए गए कथनो को पढ़ और उर द िक िन म स कौन सास िवक सही ह

(1) िवतलीय शल अतवधी कार क आश शल ह

(2) यह बत धीर-धीर ठडा होता ह ोिक आस-पास क शल मा क अतवधन क चारो ओर रोधन क प म कायकरत ह

1 Both statements are wrong दोनो कथन गलत ह

2 Both statements are correct दोनो कथन सही ह

3 First statement is wrong and second statement is correct पहला कथन गलत ह और दसरा कथन सही ह

4 First statement is correct and second statement is wrong पहला कथन सही ह और दसरा कथन गलत ह

Correct Answer -

Both statements are correct दोनो कथन सही ह

45)

46)

47)

48)

The dust and ash material hurled from the volcanoes are termed as

ालामखी स िनकलन वाली धल और राख सामी को _______ क प म कहा जाता ह

1 Pyroclasc पाइरोाक

2 Hyperclastic हाइपराक

3 Hepiroclastic हिपरोाक

4 Cirroclastic िसरोाक

Correct Answer -

Pyroclasc पाइरोाक

The vertical difference in elevation between a low tide and high tide is referred as

कम ार और उ ार क बीच ऊचाई म लबवत अतर _____ स सदिभत होता ह

1 Tidal slope ारीय ढलान

2 Tidal elevation ारीय उयन

3 Tidal range ारीय परास

4 Tidal height ारीय ऊचाई

Correct Answer -

Tidal range ारीय परास

The maximum biodiversity is found in which of the following regions िनिलखत ो म स अिधकतमजव िविवधता िकसम पायी जाती ह

1 Amazon Basin अमज़न बिसन

2 East Indies ई इडीज

3 Congo Basin कागो बिसन

4 West indies व इडीज

Correct Answer -

Amazon Basin अमज़न बिसन

The cultivation of rice crop produces_______ चावल की फसल की खती ______ का उादन करती ह

1 SO2

49)

50)

51)

2 CH4

3 CFCs

4 CO2

Correct Answer -

CH4

The pressure system with higher pressure at the centre is called__________

क म उ दबाव वाली दबाव णाली को _______ कहा जाता ह

1 front अ

2 depression अवनमन

3 cyclone चवात

4 anti-cyclone ितचवात

Correct Answer -

anti-cyclone ितचवात

The Himalayan region is poor in mineral resources because िहमालयी खिनज ससाधनो म समनही ह ोिक

1 The displacement of rock strata has disturbed the arrangement of rocks and made it complex

शलीय परत क िवथापन न चानो की वथा को अवथत कर िदया ह और इस जिटल बना िदया ह

2 The climate conditions are not suitable for exploitation of minerals

जलवाय की थित खिनजो क दोहन क िलए उपय नही ह

3 The terrain makes explanation of minerals difficult and very costly due to transportation difficulties

भ-भाग परवहन की किठनाइयो क कारण खिनजो का दोहन मल और बत महगा बना दता ह

4 It is made up of crystalline rocks यह िलीय चानो स बना ह

Correct Answer -

The displacement of rock strata has disturbed the arrangement of rocks and made it complex

शलीय परत क िवथापन न चानो की वथा को अवथत कर िदया ह और इस जिटल बना िदया ह

The process through which the moisture is added to the atmosphere by vegetation is termed as

वह िया िजसक माम स वनित ारा वातावरण म नमी िमलायी जाती ह _______ क प म जानी जाती ह

52)

53)

54)

1 Condensation सघनन

2 Evapotranspiration वान-उजन

3 Radiation िविकरण

4 Precipitation वषण

Correct Answer -

Evapotranspiration वान-उजन

The process through which the terrestrial heat is transferred to air by direct contact is termed as

वह िया िजसम सपक ारा थलीय ऊा वाय म थानातरत हो जाती ह ______ क प म जानी जाती ह

1 Conduction चालन

2 Convection सवहन

3 Insolation आतपन

4 Radiation िविकरण

Correct Answer -

Conduction चालन

The largest area under mangroves is in which of the following statesunion territory

मोव क अतगत िनिलखत राोसघ शािसत दशो म स सबस बड़ा कौन सा ह

1 Andaman and Nicobar अमान और िनकोबार

2 Andhra Pradesh आ दश

3 West Bengal पिम बगाल

4 Gujarat गजरात

Correct Answer -

West Bengal पिम बगाल

The longitudinal transverse and surface waves in an earthquake originate from

भकप म दशातर अनथ और सतह तरग यहा उ होती ह

1 The focus on the surface of the Earth पी क सतह पर क -िबद म

2 The focus within the body of the Earth पी क भीतर क -िबद म

3 The epicenter within the body of the Earth पी क भीतर उपरक म

55)

56)

57)

4 The epicenter on the surface of the Earth पी क सतह पर उपरक म

Correct Answer -

The focus within the body of the Earth पी क भीतर क -िबद म

The down slope movement of material due to gravity is called______

गाकषण क कारण पदाथ की अनढाल गित को ______ कहा जाता ह

1 mass movement पदाथ सचलन

2 deposition िनप

3 erosion रण

4 volcanic movement ालामखीय सचलन

Correct Answer -

mass movement पदाथ सचलन

Shimla is cooler than Amritsar although both are on the same latitude This is because

िशमला म अमतसर स अिधक ठड ह हालािक दोनो समान अाश पर ह ऐसा ह ोिक

1 Shimla is at a greater height above sea level than Amritsar अमतसर की तलना म िशमला सम तल स अिधकऊचाई पर ह

2 Shimla is further north िशमला उर की ओर ह

3 Shimla is farther from the equator िशमला भम रखा स आग ह

4 Their longitudes differ उनकी दशातर रखाए िभ ह

Correct Answer -

Shimla is at a greater height above sea level than Amritsar अमतसर की तलना म िशमला सम तल स अिधकऊचाई पर ह

lsquoTempo of Urbanizationrsquo measures which of the following

lsquoशहरीकरण का टपोrsquo िनिलखत म स कौन सा उपाय ह

1 Speed of urbanizaon शहरीकरण की गित

2 None of the above इनम स कोई नही

3 Inequality of urbanizaon शहरीकरण की असमानता

4 Current level of urbanizaon शहरीकरण का वतमान र

Correct Answer -

58)

59)

60)

Speed of urbanizaon शहरीकरण की गित

Out of the following options choose the INCORRECT statement

िनिलखत िवको म स गलत कथन का चयन कर

1 The clear tracts in the equatorial region recover rapidly भम रखा म भभाग तजी स ठीक हो जात ह

2 The stable communities include a redwood forest a pine forest at high elevations

थर समदायो म एक रडवड वन उ ऊचाई पर एक दवदार वन शािमल ह

3 Any ecosystem moves towards maximum biomass and stability to survive

कोई भी पारथितकी त जीिवत रहन क िलए अिधकतम जवसहित और थरता की तरफ असर होता ह

4 Tropical rain forests near equator are stable ecosystems

भम रखा क पास उकिटबधीय वषा वन थर पारथितक त ह

Correct Answer -

The clear tracts in the equatorial region recover rapidly भम रखा म भभाग तजी स ठीक हो जात ह

Seasonal contrasts are maximum in मौसमी िवषमता अिधकतम ह

1 Mid latitudes म अाश म

2 Low attitudes िन अाश म

3 High latitudes उ अाश म

4 Subtropics उपोकिटबधीय म

Correct Answer -

Mid latitudes म अाश म

In India which type of forest among the following occupies the largest area

भारत म िनिलखत म स िकस कार क वन सबस बड़ा फल आािदत करत ह

1 Sub-tropical Dry Evergreen Forest उप उकिटबधीय श सदाबहार वन

2 Mountain Wet Temperate Forest पवतीय आ शीतो वन

3 Tropical Moist Deciduous Forest उकिटबधीय आ पणपाती वन

4 Tropical Wet Evergreen Forest उकिटबधीय आ सदाबहार वन

Correct Answer -

Tropical Moist Deciduous Forest उकिटबधीय आ पणपाती वन

61)

62)

63)

64)

What is the proportion of lsquoJuvenile Populationrsquo (0-14 years) in India as per 2011Census

2011 की जनगणना क अनसार भारत म जवनाइल पॉपलशन यानी िकशोर जनस या (0-14 वष) का अनपात ाह

1 3076 of total population कल जनस या का 3076

2 2764 of total population कल जनस या का 2764

3 2933 of total population कल जनस या का 2933

4 3354 of total population कल जनस या का 3354

Correct Answer -

3076 of total population कल जनस या का 3076

What is the Belfast famous for बलफा िकसक िलए मशर ह

1 Belt of cotton textile industry कपास व उोग क

2 Ship-building industry जहाज िनमाण उोग

3 Agricultural machinery किष उपकरण

4 Aero planes manufacturing वाययान िनमाण

Correct Answer -

Ship-building industry जहाज िनमाण उोग

What is the most important occupation in tropical monsoon lands

उकिटबधीय मॉनसन भिम म सबस महपण वसाय ा ह

1 Mining खनन

2 Cattle rearing मवशी पालन

3 Agriculture किष

4 Nomadic herding नोमािडक जड़ी-बिटया

Correct Answer -

Agriculture किष

What is the most important characteristics of the islands (Indian) located in the Arabian sea

अरब सागर म थत ीपो (भारतीय) की सबस महपण िवशषता ा ह

ी ो

65)

66)

67)

1 There are all of coral origins सभी कोरल मल क ह

2 There are all very small in size य सभी आकार म बत छोट ह

3 They have a very dry climate इनकी जलवाय बत श ह

4 They are extended parts of the mainland व महाीप क िवारत िह ह

Correct Answer -

There are all of coral origins सभी कोरल मल क ह

What do the basalt layers of the Deccan indicate डन की बसा परत ा इिगत करती ह

1 All of the above उपरो सभी

2 Huge volcanic eruptions in the distant past दरथ अतीत म िवशाल ालामखीय िवोट

3 The immense erosional activity of the rivers निदयो की िवशाल रण गितिविध

4 The influence of weathering मौसम का भाव

Correct Answer -

Huge volcanic eruptions in the distant past दरथ अतीत म िवशाल ालामखीय िवोट

In the structure of planet Earth below the mantle the core is mainly made up of_____

पी ह की सरचना म मटल क नीच कोर म प स______ स िनिमत होती ह

1 aluminium एमीिनयम

2 silicon िसिलकॉन

3 chromium ोिमयम

4 iron लोहा

Correct Answer -

iron लोहा

One of the major Mid Oceanic Ridge is found in मख म-महासागर चोिटयो म स एक ______ म पायाजाता ह

1 Mid Pacific Ocean म शात महासागर

2 Mid Atlantic Ocean म अटलािटक महासागर

3 Mid Indian Ocean म भारतीय महासागर

4 Mid Arctic Ocean म आक िटक महासागर

68)

69)

70)

71)

Correct Answer -

Mid Atlantic Ocean म अटलािटक महासागर

Magma that reaches the Earthrsquos surface and then solidifies is called________

मा जो पी की सतह तक पचती ह और िफर ठोस हो जाती ह ________कहलाती ह

1 quartz ाटज

2 lava लावा

3 granite नाइट

4 silicates िसिलकट

Correct Answer -

lava लावा

Isotherms are the lines of equal_______ समताप रखाए समान _______की रखाए होती ह

1 pressure दाब

2 temperature तापमान

3 rainfall वषा

4 height ऊचाई

Correct Answer -

temperature तापमान

Mark the correct sequence of passes in the Western Ghats from north to south

पिमी घाटो म उर स दिण तक दर क सही अनम को िचित कर

1 Thalghat Palghat Bhorghat थलगघाट पालघाट भोरघाट

2 Thalghat Bhorghat Palghat थलघाट भोरघाट पालघाट

3 Bhorghat Thalghat Palghat भोरघाट थलघाट पालघाट

4 Palghat Bhorghat Thalghat पालघाट भोरघाट थलघाट

Correct Answer -

Thalghat Bhorghat Palghat थलघाट भोरघाट पालघाट

Which of the following does not have influence over the climate in India

ि ि ि ी ी

72)

73)

िनिलखत म स िकसका भाव भारत की जलवाय पर नही पड़ता ह

1 Ocean currents सागर की लहर

2 Nearness to equator भम रखा स िनकटता

3 Monsoons मानसन

4 Presence of Indian ocean भारतीय महासागर की उपथित

Correct Answer -

Ocean currents सागर की लहर

Which of the following cloud types has the characteristics like vertical tall narrow and puffy

िनिलखत म स िकस कार क मघो म लबवत लबी सकीण और थलता जसी िवशषताए ह

1 Cumulonimbus तफानी मघ

2 Cumulus मघ पज

3 Cirrocumulus पाभ कपासी मघ

4 Nimbostratus वषारी मघ

Correct Answer -

Cumulus मघ पज

Which of the following statement is INCORRECT about Crude Birth Rate

िनिलखत स कौन सा कथन अशोिधत ज दर क बार म सही नही ह

1 It cannot be used for comparing fertility level between two countries with different population characteristics

इसका उपयोग िविभ जनसा िवशषताओ वाल दो दशो क बीच जनन र की तलना क िलए नही िकया जा सकता ह

2 It is a standardized measure of fertility

यह जनन मता का मानकीकत उपाय ह

3 It is effected by the age-sex composition of the population

यह आबादी की आय-िलग सरचना स भािवत होता ह

4 It is expressed per 1000 population in a given geographical unit

यह िकसी दी गई भौगोिलक इकाई म ित 1000 जनसा पर िकया जाता ह

Correct Answer -

It is a standardized measure of fertility

यह जनन मता का मानकीकत उपाय ह

74)

75)

76)

77)

Which of the following state in India experienced negative decadal growth rate during 2001 to 2011census

भारत म िनिलखत म स िकस रा म वष 2001 स 2011 की जनगणना क दौरान नकाराक िगरावट दर ई

1 Tripura िपरा

2 Nagaland नागालड

3 Haryana हरयाणा

4 Odisha ओिडसा

Correct Answer -

Nagaland नागालड

Which of the following is NOT a characteristic of peninsular rivers

िनिलखत म स कौन सी िवशषता ायीपीय निदयो म नही होती ह

1 Flow through shallow valleys उथल घािटयो क माम स वाह

2 Seasonal flow मौसमी वाह

3 Little erosional activity थोड़ी कटावदार गितिविध

4 Meandering tendency often shifting their beds घमावदार वि अर अपन तटो को थानातरत करना

Correct Answer -

Meandering tendency often shifting their beds घमावदार वि अर अपन तटो को थानातरत करना

Which of the following gases in the atmosphere absorbs heat from the Sunrsquos radiation and the Earthssurface

वायमडल म िनिलखत म स कौन सी गस सय क िविकरण और पी की सतह स ऊा को अवशोिषत करती ह

1 Neon िनयॉन

2 Carbon dioxide काबन डाइऑाइड

3 Argon आगन

4 Nitrogen नाइट ोजन

Correct Answer -

Carbon dioxide काबन डाइऑाइड

Which of the following kind of settlement pattern is found at the confluence of rivers

ि ि ि ि ो

78)

79)

80)

िनिलखत म स िकस कार का वथापन पटन निदयो क सगम पर पाया जाता ह

1 Triangular Paern िकोणीय पटन

2 Circular or Semi-Circular Paern परप या अध-परप पटन

3 Nebular Paern नबलर पटन

4 Star ndashShaped Paern ार-आकार का पटन

Correct Answer -

Triangular Paern िकोणीय पटन

Which one was not the objective of the Biosphere Reserve Projects launched by the UNESCO

यनो ारा श की गई सरित जवमडल परयोजनाओ का उ इनम स कौन सा नही था

1 To promote teaching and research िशण और अनसधान को बढ़ावा दना

2 To make agriculture sustainable किष को दीघकािलक बनाना

3 To conserve ecosystems पारथितक त को सरित करना

4 To conserve genetic diversity for a longtime लब समय तक अनवािशक िविवधता को सरित करना

Correct Answer -

To make agriculture sustainable किष को दीघकािलक बनाना

Which region of the Earth surface receives the highest amount of insulation

पी सतह का कौन सा तापावरोधन की उतम माा ा करता ह

1 Land mass थलखड

2 Savannah region सवाना

3 Water bodies जल िनकाय

4 Tropical desert उकिटबधीय रिगान

Correct Answer -

Tropical desert उकिटबधीय रिगान

Which one of the following is not a biodiversity hotspot

िनिलखत म स कौन सा जव िविवधता का म जगह नही ह

1 Eastern Himalaya पव िहमालय

2 Eastern Ghats पव घाट

81)

82)

83)

3 Indo-Myanmar भारत-ामार

4 Westerm Ghats पिमी घाट

Correct Answer -

Eastern Ghats पव घाट

Which one of the following is NOT a part of the World Network of Biosphere Reserves based on theUNESCO Man and Biosphere Programme

यनो मन और बायोीयर कायम क आधार पर िनिलखत म स कौन बायोीयर रजव क िव नटवक कािहा नही ह

1 Gulf of Mannar मार की खाड़ी

2 Seshachalam शषाचलम

3 Sunderban सदरबन

4 Nilgiri नीलिगर

Correct Answer -

Seshachalam शषाचलम

Which one of the following is an example of ldquodesert vegetationrdquo

िनिलखत म स कौन मथलीय वनित का एक उदाहरण ह

1 Mosses and lichens दलदल और शवाल

2 Temperate grassland समशीतो घास क मदान

3 Coniferous forest शकधारी वन

4 Acacia and cactus एकािसया और कस

Correct Answer -

Acacia and cactus एकािसया और कस

Which one of the following reflects more sunlight िनिलखत म स कौन सा सय की रोशनी को अिधकपरावितत करता ह

1 Paddy crop land धान फसल भिम

2 Land covered with fresh snow ताजा बफ स आािदत भिम

3 Sand desert रतीली रिगान

4 Prairie land यरी भिम

84)

85)

86)

87)

Correct Answer -

Land covered with fresh snow ताजा बफ स आािदत भिम

Which layer of the atmosphere is in contact with the surface of the earthrsquos oceans

वायमडल की कौन सी परत पी क महासागरो की सतह क सपक म ह

1 Stratosphere समताप मडल

2 Mesosphere म मडल

3 Hydrosphere जलमडल

4 Troposphere ोभ मडल

Correct Answer -

Troposphere ोभ मडल

Mediterranean Sea is a border of which of the following countries भम सागर िनिलखत दशो म सिकसकी सीमा ह

1 None of these इनम स कोई नही

2 Iraq इराक

3 Lebanon लबनान

4 Jordan जॉडन

Correct Answer -

Lebanon लबनान

Benguela ocean currents are found along which coast बगएला महासागर धाराए िकस तट क साथ पायीजाती ह

1 East Coast of South America दिण अमरका क पव तट

2 East Coast of Africa अीका क पव तट

3 West Coast of South America दिण अमरका क पिमी तट

4 West Coast of Africa अीका क पिमी तट

Correct Answer -

West Coast of Africa अीका क पिमी तट

88)

89)

90)

Due to tension a block of land on one side being pushed up or upthrown relative to the downthrown blockis referred as

तनाव क कारण नीच फ क ए खड क साप भिम का एक खड एक ओर स ऊपर धकला जाता ह या ऊपर की ओरफ का जाता ह यह _____ क प म सदिभत ह

1 Thrust fault प श

2 Normal fault सामा श

3 Reverse fault म श

4 Strike slip fault नितलब सपण श

Correct Answer -

Normal fault सामा श

Inter-tropical doldrums is a zone of ______ अतर-उकिटबधीय डोलड ______ का एक ह

1 Frontolysis टोलायिसस

2 Convergence अिभसरण

3 Inter-tropical divergence zone अतर-उकिटबधीय िवचलन

4 Local wind थानीय वाय

Correct Answer -

Convergence अिभसरण

The Horse Latitudes are regions located at about _____ north and south of the equator

हॉस अाश भम रखा क उर और दिण म लगभग _____ पर थत ह

1 30ndash60 degree Latitude 30-60 िडी अाश

2 0ndash5 degree Latitude 0-5 िडी अाश

3 30 degree Latitude 30 िडी अाश

4 60ndash90 degree Latitude 60-90 िडी अाश

Correct Answer -

30 degree Latitude 30 िडी अाश

Generally evaporation is high over which part of the Earth

आम तौर पर पी क िकस भाग पर वाीकरण अिधक होता ह

1 Equatorial maritime भमवत समीय ी ी

91)

92)

2 Equatorial continental भमवत महाीपीय

3 Polar maritime वीय समीय

4 Polar continental वीय महाीपीय

Correct Answer -

Equatorial maritime भमवत समीय

A very high temperature during summer in north western India leads to what type of climaticcondition in south

उर पिमी भारत म गम क दौरान बत अिधक तापमान होन क कारण दिण म िकस कार की जलवाय थितउ करता ह

1 Depression over arabian sea अरब सागर पर अवनमन

2 Failure monsoon मानसन िवफलता

3 Successful monsoon मानसन सफलता

4 Cyclones चवात

Correct Answer -

Successful monsoon मानसन सफलता

Lightning and thunder are the resultant effect when तिड़त और गजन परणामी भाव ह जब

1 Two massive clouds hit powerfully each other first lightning is produced and later sound is produced

दो बड़ बादल एक दसर स शशाली ढग स टकरात ह पहल आकाशीय िवदयत उ होता ह और बाद म िन उहोती ह

2 Two massive clouds come into contact with the powerful wind collision this results into first sound and thenlightning

दो बड़ बादल शशाली पवन सघ क सपक म आत ह इसका परणामप पहल िन और िफर आकाशीय िवदयतउ होता ह

3 None of the above उपरो म स कोई भी नही

4 A high density cloud contains positively and negatively charged electric ions and when this interacts light andsound are simultaneously produced

एक उ घन बादल म धनाक और ऋणाक आविशत िवदयत आयन होत ह और जब यह परर भाव डालत ह तोकाश और िन एक साथ उािदत होती ह

Correct Answer -

A high density cloud contains positively and negatively charged electric ions and when this interacts light andsound are simultaneously produced

औ ि ि ो औ ो

93)

94)

95)

एक उ घन बादल म धनाक और ऋणाक आविशत िवदयत आयन होत ह और जब यह परर भाव डालत ह तोकाश और िन एक साथ उािदत होती ह

Doon Valley is able to grow rice because दन घाटी चावल उगान म सम ह ोिक

1 Other crops cannot be grown वहा अ फसलो को उगाया नही जा सकता ह

2 People in the valley are rice eaters घाटी म लोग चावल खान वाल ह

3 There is a huge export demand of rice वहा चावल की भारी िनयात माग ह

4 It has warm summer and snow melt waters for irrigation

वहा गिमया गम होती ह िसचाई क िलए बफ का िपघला आ पानी होता ह

Correct Answer -

It has warm summer and snow melt waters for irrigation

वहा गिमया गम होती ह िसचाई क िलए बफ का िपघला आ पानी होता ह

CANCELLED

In the geological time scale the Mesozoic Era DOES NOT contains which of the following periods

भगभय समय पमान पर मजीवी यग म िन कालो म स कौन नही ह

1 Triassic ट ाइऐिसक

2 Jurassic जरिसक

3 Cretaceous चाकमय

4 Carboniferous काबनी

Correct Answer -

Carboniferous काबनी

96)

1 P-3 Q-4 R-2 S-1

2 P-3 Q-4 R-1 S-2

3 P-3 Q-4 R-1 S-2

4 P-4 Q-3 R-2 S-1

Correct Answer -

P-4 Q-3 R-2 S-1

1 P-3 Q-1 R-4 S-2

2 P-3 Q-4 R-1 S-2

3 P-3 Q-2 R-4 S-1

97)

98)

4 P-2 Q-1 R-4 S-3

Correct Answer -

P-3 Q-4 R-1 S-2

1 P-3 Q-1 R-4 S-2

2 P-2 Q-3 R-4 S-1

3 P-2 Q-1 R-3 S-4

4 P-4 Q-2 R-1 S-3

Correct Answer -

P-3 Q-1 R-4 S-2

99)

100)

1 P-3 Q-2 R-4 S-1

2 P-1 Q-2 R-3 S-4

3 P-2 Q-3 R-1 S-4

4 P-4 Q-3 R-2 S-1

Correct Answer -

P-2 Q-3 R-1 S-4

ldquoHuman geography is the study of changing relationship between the unresting man and the unstableearthrdquo was defined by

lsquolsquoमानव भगोल ाकल आदमी और अथर पी क बीच सबध परवतन का अयन हrdquo ______ ारा परभािषत िकया गयाथा

1 J Brunches ज चस

2 EC Semple ईसी सल

3 HJ Mackinder एच ज मिकदर

4 PV Blache पीवी च

Correct Answer -

EC Semple ईसी सल

Sedimentary rocks are finally and ultimately derived from the____________

अवसादी चान अततः ________ स ा की जाती ह

1 action of earth movements पी की गितिविधयो

2 marine deposit समी िनप

3 weathering of metamorphic rocks पातरत चानो क अपय

4 weathering of igneous rocks आय चानो क अपय

Correct Answer -

weathering of igneous rocks आय चानो क अपय

Page 22: High School Teacher Eligibility Test- BOARD PROFESSIONAL ...peb.mp.gov.in/results/RESULT_18/HST_RES18/Final_anwser_key/HST… · M a ndl a / मंड ल ... Under the Madhya Pradesh

15)

16)

17)

4 West and Northwest पिम और उरपिम

Correct Answer -

West and Northwest पिम और उरपिम

During an earth quake the velocity of the body waves will________ along with the increase in densityof the material it is passing through

भकप क दौरान लहरो क ऊपरी भाग का वग घन म व क साथ-साथ ________ जो इसस गजरन वाली वको आग बढाएगी

1 not change नही बदलगा

2 increase initially and then decrease शआत म बढ़गा और िफर घटगा

3 increase बढ़गा

4 decrease घटगा

Correct Answer -

increase बढ़गा

The Clouded Leopard National park is situated in which of the following states

िनिलखत म िकस रा म धिमल तदआ रा ीय उान (ाउडड लपड नशनल पाक ) थत ह

1 Tripura िपरा

2 Uttar Pradesh उर दश

3 Assam असम

4 Mizoram िमजोरम

Correct Answer -

Tripura िपरा

Usually the land surfaces are heated more quickly than the water surfaces because _____________

आम तौर पर जल सतहो की तलना म भिम सतह अिधक तजी स गम होती ह ोिक _____________ ह

1 the specific heat of water is higher than land पानी की िविश ऊा भिम स अिधक

2 the specific heat of water is lesser than land पानी की िविश ऊा भिम स कम होती

3 the latent heat of water is higher than the land पानी की अतिनिहत ऊा भिम स अिधक

4 the land reflects more heat radiation than water भिम पानी की तलना म अिधक ऊा क िविकरण को पराविततकरती

18)

19)

20)

21)

Correct Answer -

the specific heat of water is higher than land पानी की िविश ऊा भिम स अिधक

The longest shore-line is along the state of सबस लबी समतटीय रखा िन रा क साथ ह

1 Maharashtra महारा

2 Orissa उड़ीसा

3 Kerala करल

4 Gujarat गजरात

Correct Answer -

Gujarat गजरात

The position when the Earth is farthest from the Sun is known as

जब पी सय स सबस दर होती ह तो उस थित को िन नाम स जाना जाता ह

1 Perihelion उपसौर

2 Vernal Equinox बसत िवषव

3 Aphelion अपसौर

4 Autumnal Equinox शराल िवषव

Correct Answer -

Aphelion अपसौर

The seasonal reversal of winds is the typical characteristic of

हवाओ का मौसमी परवतन ______ की सामा िवशषता ह

1 Mediterranean climates only कवल भमसागरीय जलवाय

2 All of the above climates उपय सभी मौसम

3 Monsoon climate only कवल मानसन जलवाय

4 Equatorial climate only कवल भमरखीय जलवाय

Correct Answer -

Monsoon climate only कवल मानसन जलवाय

In _________ rocks the minerals will occurs in beds or layers

ो ि ो ो

22)

23)

24)

______ चानो म खिनज तल या परतो म होत ह

1 metamorphic कायातरत

2 igneous and metamorphic आय और कायातरत

3 igneous आय

4 sedimentary अवसादी

Correct Answer -

sedimentary अवसादी

Black soil is ideal for the cultivation of cotton as कपास की खती क िलए काली िमी आदश ह ोिक

1 Its colour is black यह काली होती ह

2 It is found on plateau regions यह पठार ो म पायी जाती ह

3 It is made up of lava यह लावा स बनी होती ह

4 It can retain moisture यह नमी को बरकरार रख सकती ह

Correct Answer -

It can retain moisture यह नमी को बरकरार रख सकती ह

The National Survey and Mapping Organization of the country works under the Department of___________

दश का रा ीय सवण और मानिचण सगठन ___________ िवभाग क अतगत काय करता ह

1 Space अतर

2 Science and Technology िवान और तकनीक

3 Culture सित

4 Tourism पयटन

Correct Answer -

Science and Technology िवान और तकनीक

Palk strait separates India from पाक जलडमम भारत स _____ को अलग करता ह

1 Pakistan पािकान

2 Andaman Island अडमान ीप

3 China चीन

25)

26)

27)

4 Sri Lanka ीलका

Correct Answer -

Sri Lanka ीलका

Which among the following state is the major producer of Bauxite in India

िनिलखत म स कौन सा रा भारत म बॉाइट का मख उादक ह

1 Madhya Pradesh मदश

2 Rajasthan राजथान

3 Goa गोवा

4 Orissa उड़ीसा

Correct Answer -

Orissa उड़ीसा

Which of the following states DOES NOT share border with Chhattisgarh

िनिलखत म स कौन सा रा छीसगढ़ क साथ सीमा साझा नही करता ह

1 Telangana तलगाना

2 Uttar Pradesh उर दश

3 Bihar िबहार

4 Andhra Pradesh आ दश

Correct Answer -

Bihar िबहार

Which of the following statements is INCORRECT with respect to parallels of latitudes

अाश क समानातरो क सबध म िन निलखत म स कौन सा कथन गलत ह

1 A line joining places of equal latitude is known as parallel of largest

समान अाश क थानो को जोड़न वाली रखा को िवशालतम क समानातर क प म जाना जाता ह

2 They stat from equator and run parallels to it

व भम रखा स ारभ होत ह और इसक समानातर चलत ह

3 All parallels are equal in length सभी समातर लबाई म समान ह

4 All parallels are drawn as circles on the globe ोब पर सभी समानातर वो क प म खीच जात ह

28)

29)

30)

31)

Correct Answer -

All parallels are equal in length सभी समातर लबाई म समान ह

Which of the following Indian states is also known as a lsquoLand of Red river and Blue Hillsrsquo

िनिलखत म स िकस भारतीय रा को लाल नदी और नीली पहािड़यो की भिम क नाम स जाना जाता ह

1 Uttarkhand उराखड

2 Assam असम

3 Meghalaya मघालय

4 Arunachal Pradesh अणाचल दश

Correct Answer -

Assam असम

In spatial analysis of settlement Rn = 215 indicates which type of settlement arrangement

िनपटान क थािनक िवषण म Rn = 215 यह इिगत करता ह िक िकस कार की िनपटान वथा ह

1 Uniform यिनफॉम

2 Semi-Clustered समी- ल टर

3 Clustered ल टर

4 Random रडम

Correct Answer -

Uniform यिनफॉम

Who are known as the lsquoYellow Peoplersquo lsquoयलो पीपलrsquo क प म कौन जाना जाता ह

1 Mongoloids मोगोलोइडस

2 Nigroids नीोइडस

3 Australoids ऑ लॉइडस

4 Caucasoids कॉकसोइडस

Correct Answer -

Mongoloids मोगोलोइडस

ि ि ो ौ ि

32)

33)

34)

Who publishes the topographical map of India भारत क थलाकितक मानिच को कौन कािशत करता ह

1 Geographical Survey of India भारत का भौगोिलक सवण

2 Government of India भारत सरकार

3 Geological Survey of India भारत क भगभय सवण

4 Survey of India भारत का सवण

Correct Answer -

Survey of India भारत का सवण

Who among the following claimed geography to be the lsquoEcology of Manrsquo

िनिलखत म स िकसन भगोल को मन का पारथितकी कहा ह

1 Alfred Hener अड हटनर

2 Vidal-de la Blache वाइडल-िड लॉ ॉश

3 Oo Schluter ओटो टर

4 Harlan Barrow हरलन बारो

Correct Answer -

Harlan Barrow हरलन बारो

Who among the following is regarded as the founder of humanistic approach in geography

िनिलखत म स िकस भगोल म मानवतावादी िकोण का सथापक माना जाता ह

1 William Bunge िविलयम बग

2 Yi-Fu-Tuan यी-फ- यान

3 Brain JL Berry न जएल बरी

4 Richard Peet रचड पीट

Correct Answer -

Yi-Fu-Tuan यी-फ- यान

Who prepared Lorenz curve लोरज व िकसन तयार िकया

1 Geddes गडस

2 None of these इनम स कोई नही

3 Griffith Taylor ििफथ टलर

35)

36)

37)

4 Max U Lorenz म य लोरज

Correct Answer -

Max U Lorenz म य लोरज

Gulf Streams are the currents of which of the following oceans

खाड़ी की धाराए िनिलखत महासागरो म स िकसकी धाराए ह

1 North Atlantic Ocean उरी अटलािटक महासागर

2 North Pacific Ocean उरी शात महासागर

3 Arabian Sea अरब सागर

4 South Pacific Ocean दिण शात महासागर

Correct Answer -

North Atlantic Ocean उरी अटलािटक महासागर

Disintegration wearing away and removal of rock material is generally referred as

िशला पदाथ (रॉक सामी) का टटना िमटना और हटना आमतौर पर ________ क प म सदिभत िकया जाता ह

1 Shattering िवसकारक

2 Denudation अनाादन

3 Fault श

4 Decomposition िवयोजन

Correct Answer -

Denudation अनाादन

Variations in the length of day time and night from season to season are due to

मौसम स मौसम परवतन पर िदन क समय और रात क समय की अविध म िभताए िन कारण स होती ह

1 The Earthrsquos revolution round the Sun in an elliptical manner पी का दीघवाकार तरीक स सय क चारो घणन

2 The Earthrsquos rotation on its axis पी का इसकी धरी पर घणन

3 Revolution of the Earth on a tilted axis नत अ पर पी का घणन

4 Latitudinal position of the place थान की अाश थित

Correct Answer -

Revolution of the Earth on a tilted axis नत अ पर पी का घणन

38)

39)

40)

Point out the correct sequence of mountain ranges from north to south

उर स दिण तक पवत खलाओ क सही अनम को इिगत कर

1 Great Himalaya Middle Himalaya Outer Himalaya Trans Himalaya

महान िहमालय म िहमालय बा िहमालय परा िहमालय

2 Middle Himalaya Great Himalaya Trans Himalaya Outer Himalaya

म िहमालय महान िहमालय परा िहमालय बा िहमालय

3 Outer Himalaya Middle Himalaya Great Himalaya Trans Himalaya

बा िहमालय म िहमालय महान िहमालय परा िहमालय

4 Trans Himalaya Great Himalaya Middle Himalaya Outer Himalaya

परा िहमालय महान िहमालय म िहमालय बा िहमालय

Correct Answer -

Trans Himalaya Great Himalaya Middle Himalaya Outer Himalaya

परा िहमालय महान िहमालय म िहमालय बा िहमालय

Sunrsquos halo is produced by the refraction of light in सय का भामडल ______ म काश क अपवतन ाराउ होता ह

1 Ice crystals in Cirrro-Cumulus clouds पाभ-कपास मघो क बफ िल

2 Ice crystal in Cirrus clouds पाभमघो क बफ िल

3 Dust particles in Stratus clouds री मघो क धल कण

4 Water vapour in Stratus clouds री मघो क जल वा

Correct Answer -

Ice crystal in Cirrus clouds पाभमघो क बफ िल

Read the given statements and answer which of the following options isare correct

(1) The minerals present in the rocks exposed to atmosphere are not subjected to alteration

(2) Oxidation is one of the processes of chemical weathering

िदए गए कथनो को पढ़ और उर द िक िन म स कौन सास िवक सही ह

(1) वायमल स अनावत शल म उपथत खिनज परवतन क अधीन नही होता ह

(2) ऑीकरण रासायिनक अपय की ियाओ म स एक ह

ो ो

41)

42)

1 Both statements are wrong दोनो कथन गलत ह

2 Both statements are correct दोनो कथन सही ह

3 First statement is wrong and second statement is correct पहला कथन गलत ह और दसरा कथन सही ह

4 First statement is correct and second statement is wrong पहला कथन सही ह और दसरा कथन गलत ह

Correct Answer -

First statement is wrong and second statement is correct पहला कथन गलत ह और दसरा कथन सही ह

Read the given statements and answer which of the following options isare correct

1 Sunrsquos short waves enter the earth partially heating the atmosphere

2 Heated earth surface from the sun produces broader waves which interacts and heats the atmosphere

िदए गए कथन को पढ़ और उर द िक िन म स कौन सास िवक सही ह

1 सय की छोटी तरग पी म आिशक प स वश करती ह और वायमडल को ऊत करती ह

2 सय स ऊत पी की सतह िवारत तरग उ करती ह जो परर भाव डालती ह और वायमडल कोऊत करती ह

1 Both Statements 1 and 2 are correct दोनो कथन 1 और 2 सही ह

2 Both Statements 1 and 2 are wrong दोनो कथन 1 और 2 गलत ह

3 Statement 1 is wrong and only Statement 2 is correct कथन 1 गलत ह और कवल कथन 2 सही ह

4 Only statement 1 is correct कवल कथन 1 सही ह

Correct Answer -

Both Statements 1 and 2 are correct दोनो कथन 1 और 2 सही ह

Read the given statements and answer which of the following options isare correct

(1)The rocks that get changed due to heat and pressure are termed as metamorphic rocks

(2)Slate is one such type of metamorphic rock

िदए गए कथनो को पढ़ और उर द िक िन म स कौन सास िवक सही ह

(1) शल जो ऊा और दाब क कारण परवितत हो जात ह उ कायातरक शलो क प म जाना जाता ह

(2) ट एक तरह का कायातरक शल ह

1 Both statements are wrong दोनो कथन गलत ह

2 Both statements are correct दोनो कथन सही ह

3 First statement is wrong and second statement is correct पहला कथन गलत ह और दसरा कथन सही ह

ी औ

43)

44)

4 First statement is correct and second statement is wrong पहला कथन सही ह और दसरा कथन गलत ह

Correct Answer -

Both statements are correct दोनो कथन सही ह

Read the given statements and answer which of the following options isare correct

1 Higher temperature anomaly is observed in the northern hemisphere

2 Differential heating is absent in Northern Hemisphere

िदए गए कथनो को पढ़ और उर द िक िन म स कौन सास िवक सही ह

1 उरी गोलाध म उ तापमान िवसगित पायी जाती ह

2 उरी गोलाध म अतर ऊन अनपथत होती ह

1 Both Statements 1 and 2 are correct दोनो कथन 1 और 2 सही ह

2 Both Statements 1 and 2 are wrong दोनो कथन 1 और 2 गलत ह

3 Statement 1 is wrong and Statement 2 is correct कथन 1 गलत ह और कथन 2 सही ह

4 Statement 1 is correct and Statement 2 is wrong कथन 1 सही ह और कथन 2 गलत ह

Correct Answer -

Statement 1 is correct and Statement 2 is wrong कथन 1 सही ह और कथन 2 गलत ह

Read the given statements and answer which of the following options isare correct

(1) Plutonic rocks are intrusive type of igneous rocks

(2) It cools very slowly because the surrounding rock serves as insulation around the intrusion of magma

िदए गए कथनो को पढ़ और उर द िक िन म स कौन सास िवक सही ह

(1) िवतलीय शल अतवधी कार क आश शल ह

(2) यह बत धीर-धीर ठडा होता ह ोिक आस-पास क शल मा क अतवधन क चारो ओर रोधन क प म कायकरत ह

1 Both statements are wrong दोनो कथन गलत ह

2 Both statements are correct दोनो कथन सही ह

3 First statement is wrong and second statement is correct पहला कथन गलत ह और दसरा कथन सही ह

4 First statement is correct and second statement is wrong पहला कथन सही ह और दसरा कथन गलत ह

Correct Answer -

Both statements are correct दोनो कथन सही ह

45)

46)

47)

48)

The dust and ash material hurled from the volcanoes are termed as

ालामखी स िनकलन वाली धल और राख सामी को _______ क प म कहा जाता ह

1 Pyroclasc पाइरोाक

2 Hyperclastic हाइपराक

3 Hepiroclastic हिपरोाक

4 Cirroclastic िसरोाक

Correct Answer -

Pyroclasc पाइरोाक

The vertical difference in elevation between a low tide and high tide is referred as

कम ार और उ ार क बीच ऊचाई म लबवत अतर _____ स सदिभत होता ह

1 Tidal slope ारीय ढलान

2 Tidal elevation ारीय उयन

3 Tidal range ारीय परास

4 Tidal height ारीय ऊचाई

Correct Answer -

Tidal range ारीय परास

The maximum biodiversity is found in which of the following regions िनिलखत ो म स अिधकतमजव िविवधता िकसम पायी जाती ह

1 Amazon Basin अमज़न बिसन

2 East Indies ई इडीज

3 Congo Basin कागो बिसन

4 West indies व इडीज

Correct Answer -

Amazon Basin अमज़न बिसन

The cultivation of rice crop produces_______ चावल की फसल की खती ______ का उादन करती ह

1 SO2

49)

50)

51)

2 CH4

3 CFCs

4 CO2

Correct Answer -

CH4

The pressure system with higher pressure at the centre is called__________

क म उ दबाव वाली दबाव णाली को _______ कहा जाता ह

1 front अ

2 depression अवनमन

3 cyclone चवात

4 anti-cyclone ितचवात

Correct Answer -

anti-cyclone ितचवात

The Himalayan region is poor in mineral resources because िहमालयी खिनज ससाधनो म समनही ह ोिक

1 The displacement of rock strata has disturbed the arrangement of rocks and made it complex

शलीय परत क िवथापन न चानो की वथा को अवथत कर िदया ह और इस जिटल बना िदया ह

2 The climate conditions are not suitable for exploitation of minerals

जलवाय की थित खिनजो क दोहन क िलए उपय नही ह

3 The terrain makes explanation of minerals difficult and very costly due to transportation difficulties

भ-भाग परवहन की किठनाइयो क कारण खिनजो का दोहन मल और बत महगा बना दता ह

4 It is made up of crystalline rocks यह िलीय चानो स बना ह

Correct Answer -

The displacement of rock strata has disturbed the arrangement of rocks and made it complex

शलीय परत क िवथापन न चानो की वथा को अवथत कर िदया ह और इस जिटल बना िदया ह

The process through which the moisture is added to the atmosphere by vegetation is termed as

वह िया िजसक माम स वनित ारा वातावरण म नमी िमलायी जाती ह _______ क प म जानी जाती ह

52)

53)

54)

1 Condensation सघनन

2 Evapotranspiration वान-उजन

3 Radiation िविकरण

4 Precipitation वषण

Correct Answer -

Evapotranspiration वान-उजन

The process through which the terrestrial heat is transferred to air by direct contact is termed as

वह िया िजसम सपक ारा थलीय ऊा वाय म थानातरत हो जाती ह ______ क प म जानी जाती ह

1 Conduction चालन

2 Convection सवहन

3 Insolation आतपन

4 Radiation िविकरण

Correct Answer -

Conduction चालन

The largest area under mangroves is in which of the following statesunion territory

मोव क अतगत िनिलखत राोसघ शािसत दशो म स सबस बड़ा कौन सा ह

1 Andaman and Nicobar अमान और िनकोबार

2 Andhra Pradesh आ दश

3 West Bengal पिम बगाल

4 Gujarat गजरात

Correct Answer -

West Bengal पिम बगाल

The longitudinal transverse and surface waves in an earthquake originate from

भकप म दशातर अनथ और सतह तरग यहा उ होती ह

1 The focus on the surface of the Earth पी क सतह पर क -िबद म

2 The focus within the body of the Earth पी क भीतर क -िबद म

3 The epicenter within the body of the Earth पी क भीतर उपरक म

55)

56)

57)

4 The epicenter on the surface of the Earth पी क सतह पर उपरक म

Correct Answer -

The focus within the body of the Earth पी क भीतर क -िबद म

The down slope movement of material due to gravity is called______

गाकषण क कारण पदाथ की अनढाल गित को ______ कहा जाता ह

1 mass movement पदाथ सचलन

2 deposition िनप

3 erosion रण

4 volcanic movement ालामखीय सचलन

Correct Answer -

mass movement पदाथ सचलन

Shimla is cooler than Amritsar although both are on the same latitude This is because

िशमला म अमतसर स अिधक ठड ह हालािक दोनो समान अाश पर ह ऐसा ह ोिक

1 Shimla is at a greater height above sea level than Amritsar अमतसर की तलना म िशमला सम तल स अिधकऊचाई पर ह

2 Shimla is further north िशमला उर की ओर ह

3 Shimla is farther from the equator िशमला भम रखा स आग ह

4 Their longitudes differ उनकी दशातर रखाए िभ ह

Correct Answer -

Shimla is at a greater height above sea level than Amritsar अमतसर की तलना म िशमला सम तल स अिधकऊचाई पर ह

lsquoTempo of Urbanizationrsquo measures which of the following

lsquoशहरीकरण का टपोrsquo िनिलखत म स कौन सा उपाय ह

1 Speed of urbanizaon शहरीकरण की गित

2 None of the above इनम स कोई नही

3 Inequality of urbanizaon शहरीकरण की असमानता

4 Current level of urbanizaon शहरीकरण का वतमान र

Correct Answer -

58)

59)

60)

Speed of urbanizaon शहरीकरण की गित

Out of the following options choose the INCORRECT statement

िनिलखत िवको म स गलत कथन का चयन कर

1 The clear tracts in the equatorial region recover rapidly भम रखा म भभाग तजी स ठीक हो जात ह

2 The stable communities include a redwood forest a pine forest at high elevations

थर समदायो म एक रडवड वन उ ऊचाई पर एक दवदार वन शािमल ह

3 Any ecosystem moves towards maximum biomass and stability to survive

कोई भी पारथितकी त जीिवत रहन क िलए अिधकतम जवसहित और थरता की तरफ असर होता ह

4 Tropical rain forests near equator are stable ecosystems

भम रखा क पास उकिटबधीय वषा वन थर पारथितक त ह

Correct Answer -

The clear tracts in the equatorial region recover rapidly भम रखा म भभाग तजी स ठीक हो जात ह

Seasonal contrasts are maximum in मौसमी िवषमता अिधकतम ह

1 Mid latitudes म अाश म

2 Low attitudes िन अाश म

3 High latitudes उ अाश म

4 Subtropics उपोकिटबधीय म

Correct Answer -

Mid latitudes म अाश म

In India which type of forest among the following occupies the largest area

भारत म िनिलखत म स िकस कार क वन सबस बड़ा फल आािदत करत ह

1 Sub-tropical Dry Evergreen Forest उप उकिटबधीय श सदाबहार वन

2 Mountain Wet Temperate Forest पवतीय आ शीतो वन

3 Tropical Moist Deciduous Forest उकिटबधीय आ पणपाती वन

4 Tropical Wet Evergreen Forest उकिटबधीय आ सदाबहार वन

Correct Answer -

Tropical Moist Deciduous Forest उकिटबधीय आ पणपाती वन

61)

62)

63)

64)

What is the proportion of lsquoJuvenile Populationrsquo (0-14 years) in India as per 2011Census

2011 की जनगणना क अनसार भारत म जवनाइल पॉपलशन यानी िकशोर जनस या (0-14 वष) का अनपात ाह

1 3076 of total population कल जनस या का 3076

2 2764 of total population कल जनस या का 2764

3 2933 of total population कल जनस या का 2933

4 3354 of total population कल जनस या का 3354

Correct Answer -

3076 of total population कल जनस या का 3076

What is the Belfast famous for बलफा िकसक िलए मशर ह

1 Belt of cotton textile industry कपास व उोग क

2 Ship-building industry जहाज िनमाण उोग

3 Agricultural machinery किष उपकरण

4 Aero planes manufacturing वाययान िनमाण

Correct Answer -

Ship-building industry जहाज िनमाण उोग

What is the most important occupation in tropical monsoon lands

उकिटबधीय मॉनसन भिम म सबस महपण वसाय ा ह

1 Mining खनन

2 Cattle rearing मवशी पालन

3 Agriculture किष

4 Nomadic herding नोमािडक जड़ी-बिटया

Correct Answer -

Agriculture किष

What is the most important characteristics of the islands (Indian) located in the Arabian sea

अरब सागर म थत ीपो (भारतीय) की सबस महपण िवशषता ा ह

ी ो

65)

66)

67)

1 There are all of coral origins सभी कोरल मल क ह

2 There are all very small in size य सभी आकार म बत छोट ह

3 They have a very dry climate इनकी जलवाय बत श ह

4 They are extended parts of the mainland व महाीप क िवारत िह ह

Correct Answer -

There are all of coral origins सभी कोरल मल क ह

What do the basalt layers of the Deccan indicate डन की बसा परत ा इिगत करती ह

1 All of the above उपरो सभी

2 Huge volcanic eruptions in the distant past दरथ अतीत म िवशाल ालामखीय िवोट

3 The immense erosional activity of the rivers निदयो की िवशाल रण गितिविध

4 The influence of weathering मौसम का भाव

Correct Answer -

Huge volcanic eruptions in the distant past दरथ अतीत म िवशाल ालामखीय िवोट

In the structure of planet Earth below the mantle the core is mainly made up of_____

पी ह की सरचना म मटल क नीच कोर म प स______ स िनिमत होती ह

1 aluminium एमीिनयम

2 silicon िसिलकॉन

3 chromium ोिमयम

4 iron लोहा

Correct Answer -

iron लोहा

One of the major Mid Oceanic Ridge is found in मख म-महासागर चोिटयो म स एक ______ म पायाजाता ह

1 Mid Pacific Ocean म शात महासागर

2 Mid Atlantic Ocean म अटलािटक महासागर

3 Mid Indian Ocean म भारतीय महासागर

4 Mid Arctic Ocean म आक िटक महासागर

68)

69)

70)

71)

Correct Answer -

Mid Atlantic Ocean म अटलािटक महासागर

Magma that reaches the Earthrsquos surface and then solidifies is called________

मा जो पी की सतह तक पचती ह और िफर ठोस हो जाती ह ________कहलाती ह

1 quartz ाटज

2 lava लावा

3 granite नाइट

4 silicates िसिलकट

Correct Answer -

lava लावा

Isotherms are the lines of equal_______ समताप रखाए समान _______की रखाए होती ह

1 pressure दाब

2 temperature तापमान

3 rainfall वषा

4 height ऊचाई

Correct Answer -

temperature तापमान

Mark the correct sequence of passes in the Western Ghats from north to south

पिमी घाटो म उर स दिण तक दर क सही अनम को िचित कर

1 Thalghat Palghat Bhorghat थलगघाट पालघाट भोरघाट

2 Thalghat Bhorghat Palghat थलघाट भोरघाट पालघाट

3 Bhorghat Thalghat Palghat भोरघाट थलघाट पालघाट

4 Palghat Bhorghat Thalghat पालघाट भोरघाट थलघाट

Correct Answer -

Thalghat Bhorghat Palghat थलघाट भोरघाट पालघाट

Which of the following does not have influence over the climate in India

ि ि ि ी ी

72)

73)

िनिलखत म स िकसका भाव भारत की जलवाय पर नही पड़ता ह

1 Ocean currents सागर की लहर

2 Nearness to equator भम रखा स िनकटता

3 Monsoons मानसन

4 Presence of Indian ocean भारतीय महासागर की उपथित

Correct Answer -

Ocean currents सागर की लहर

Which of the following cloud types has the characteristics like vertical tall narrow and puffy

िनिलखत म स िकस कार क मघो म लबवत लबी सकीण और थलता जसी िवशषताए ह

1 Cumulonimbus तफानी मघ

2 Cumulus मघ पज

3 Cirrocumulus पाभ कपासी मघ

4 Nimbostratus वषारी मघ

Correct Answer -

Cumulus मघ पज

Which of the following statement is INCORRECT about Crude Birth Rate

िनिलखत स कौन सा कथन अशोिधत ज दर क बार म सही नही ह

1 It cannot be used for comparing fertility level between two countries with different population characteristics

इसका उपयोग िविभ जनसा िवशषताओ वाल दो दशो क बीच जनन र की तलना क िलए नही िकया जा सकता ह

2 It is a standardized measure of fertility

यह जनन मता का मानकीकत उपाय ह

3 It is effected by the age-sex composition of the population

यह आबादी की आय-िलग सरचना स भािवत होता ह

4 It is expressed per 1000 population in a given geographical unit

यह िकसी दी गई भौगोिलक इकाई म ित 1000 जनसा पर िकया जाता ह

Correct Answer -

It is a standardized measure of fertility

यह जनन मता का मानकीकत उपाय ह

74)

75)

76)

77)

Which of the following state in India experienced negative decadal growth rate during 2001 to 2011census

भारत म िनिलखत म स िकस रा म वष 2001 स 2011 की जनगणना क दौरान नकाराक िगरावट दर ई

1 Tripura िपरा

2 Nagaland नागालड

3 Haryana हरयाणा

4 Odisha ओिडसा

Correct Answer -

Nagaland नागालड

Which of the following is NOT a characteristic of peninsular rivers

िनिलखत म स कौन सी िवशषता ायीपीय निदयो म नही होती ह

1 Flow through shallow valleys उथल घािटयो क माम स वाह

2 Seasonal flow मौसमी वाह

3 Little erosional activity थोड़ी कटावदार गितिविध

4 Meandering tendency often shifting their beds घमावदार वि अर अपन तटो को थानातरत करना

Correct Answer -

Meandering tendency often shifting their beds घमावदार वि अर अपन तटो को थानातरत करना

Which of the following gases in the atmosphere absorbs heat from the Sunrsquos radiation and the Earthssurface

वायमडल म िनिलखत म स कौन सी गस सय क िविकरण और पी की सतह स ऊा को अवशोिषत करती ह

1 Neon िनयॉन

2 Carbon dioxide काबन डाइऑाइड

3 Argon आगन

4 Nitrogen नाइट ोजन

Correct Answer -

Carbon dioxide काबन डाइऑाइड

Which of the following kind of settlement pattern is found at the confluence of rivers

ि ि ि ि ो

78)

79)

80)

िनिलखत म स िकस कार का वथापन पटन निदयो क सगम पर पाया जाता ह

1 Triangular Paern िकोणीय पटन

2 Circular or Semi-Circular Paern परप या अध-परप पटन

3 Nebular Paern नबलर पटन

4 Star ndashShaped Paern ार-आकार का पटन

Correct Answer -

Triangular Paern िकोणीय पटन

Which one was not the objective of the Biosphere Reserve Projects launched by the UNESCO

यनो ारा श की गई सरित जवमडल परयोजनाओ का उ इनम स कौन सा नही था

1 To promote teaching and research िशण और अनसधान को बढ़ावा दना

2 To make agriculture sustainable किष को दीघकािलक बनाना

3 To conserve ecosystems पारथितक त को सरित करना

4 To conserve genetic diversity for a longtime लब समय तक अनवािशक िविवधता को सरित करना

Correct Answer -

To make agriculture sustainable किष को दीघकािलक बनाना

Which region of the Earth surface receives the highest amount of insulation

पी सतह का कौन सा तापावरोधन की उतम माा ा करता ह

1 Land mass थलखड

2 Savannah region सवाना

3 Water bodies जल िनकाय

4 Tropical desert उकिटबधीय रिगान

Correct Answer -

Tropical desert उकिटबधीय रिगान

Which one of the following is not a biodiversity hotspot

िनिलखत म स कौन सा जव िविवधता का म जगह नही ह

1 Eastern Himalaya पव िहमालय

2 Eastern Ghats पव घाट

81)

82)

83)

3 Indo-Myanmar भारत-ामार

4 Westerm Ghats पिमी घाट

Correct Answer -

Eastern Ghats पव घाट

Which one of the following is NOT a part of the World Network of Biosphere Reserves based on theUNESCO Man and Biosphere Programme

यनो मन और बायोीयर कायम क आधार पर िनिलखत म स कौन बायोीयर रजव क िव नटवक कािहा नही ह

1 Gulf of Mannar मार की खाड़ी

2 Seshachalam शषाचलम

3 Sunderban सदरबन

4 Nilgiri नीलिगर

Correct Answer -

Seshachalam शषाचलम

Which one of the following is an example of ldquodesert vegetationrdquo

िनिलखत म स कौन मथलीय वनित का एक उदाहरण ह

1 Mosses and lichens दलदल और शवाल

2 Temperate grassland समशीतो घास क मदान

3 Coniferous forest शकधारी वन

4 Acacia and cactus एकािसया और कस

Correct Answer -

Acacia and cactus एकािसया और कस

Which one of the following reflects more sunlight िनिलखत म स कौन सा सय की रोशनी को अिधकपरावितत करता ह

1 Paddy crop land धान फसल भिम

2 Land covered with fresh snow ताजा बफ स आािदत भिम

3 Sand desert रतीली रिगान

4 Prairie land यरी भिम

84)

85)

86)

87)

Correct Answer -

Land covered with fresh snow ताजा बफ स आािदत भिम

Which layer of the atmosphere is in contact with the surface of the earthrsquos oceans

वायमडल की कौन सी परत पी क महासागरो की सतह क सपक म ह

1 Stratosphere समताप मडल

2 Mesosphere म मडल

3 Hydrosphere जलमडल

4 Troposphere ोभ मडल

Correct Answer -

Troposphere ोभ मडल

Mediterranean Sea is a border of which of the following countries भम सागर िनिलखत दशो म सिकसकी सीमा ह

1 None of these इनम स कोई नही

2 Iraq इराक

3 Lebanon लबनान

4 Jordan जॉडन

Correct Answer -

Lebanon लबनान

Benguela ocean currents are found along which coast बगएला महासागर धाराए िकस तट क साथ पायीजाती ह

1 East Coast of South America दिण अमरका क पव तट

2 East Coast of Africa अीका क पव तट

3 West Coast of South America दिण अमरका क पिमी तट

4 West Coast of Africa अीका क पिमी तट

Correct Answer -

West Coast of Africa अीका क पिमी तट

88)

89)

90)

Due to tension a block of land on one side being pushed up or upthrown relative to the downthrown blockis referred as

तनाव क कारण नीच फ क ए खड क साप भिम का एक खड एक ओर स ऊपर धकला जाता ह या ऊपर की ओरफ का जाता ह यह _____ क प म सदिभत ह

1 Thrust fault प श

2 Normal fault सामा श

3 Reverse fault म श

4 Strike slip fault नितलब सपण श

Correct Answer -

Normal fault सामा श

Inter-tropical doldrums is a zone of ______ अतर-उकिटबधीय डोलड ______ का एक ह

1 Frontolysis टोलायिसस

2 Convergence अिभसरण

3 Inter-tropical divergence zone अतर-उकिटबधीय िवचलन

4 Local wind थानीय वाय

Correct Answer -

Convergence अिभसरण

The Horse Latitudes are regions located at about _____ north and south of the equator

हॉस अाश भम रखा क उर और दिण म लगभग _____ पर थत ह

1 30ndash60 degree Latitude 30-60 िडी अाश

2 0ndash5 degree Latitude 0-5 िडी अाश

3 30 degree Latitude 30 िडी अाश

4 60ndash90 degree Latitude 60-90 िडी अाश

Correct Answer -

30 degree Latitude 30 िडी अाश

Generally evaporation is high over which part of the Earth

आम तौर पर पी क िकस भाग पर वाीकरण अिधक होता ह

1 Equatorial maritime भमवत समीय ी ी

91)

92)

2 Equatorial continental भमवत महाीपीय

3 Polar maritime वीय समीय

4 Polar continental वीय महाीपीय

Correct Answer -

Equatorial maritime भमवत समीय

A very high temperature during summer in north western India leads to what type of climaticcondition in south

उर पिमी भारत म गम क दौरान बत अिधक तापमान होन क कारण दिण म िकस कार की जलवाय थितउ करता ह

1 Depression over arabian sea अरब सागर पर अवनमन

2 Failure monsoon मानसन िवफलता

3 Successful monsoon मानसन सफलता

4 Cyclones चवात

Correct Answer -

Successful monsoon मानसन सफलता

Lightning and thunder are the resultant effect when तिड़त और गजन परणामी भाव ह जब

1 Two massive clouds hit powerfully each other first lightning is produced and later sound is produced

दो बड़ बादल एक दसर स शशाली ढग स टकरात ह पहल आकाशीय िवदयत उ होता ह और बाद म िन उहोती ह

2 Two massive clouds come into contact with the powerful wind collision this results into first sound and thenlightning

दो बड़ बादल शशाली पवन सघ क सपक म आत ह इसका परणामप पहल िन और िफर आकाशीय िवदयतउ होता ह

3 None of the above उपरो म स कोई भी नही

4 A high density cloud contains positively and negatively charged electric ions and when this interacts light andsound are simultaneously produced

एक उ घन बादल म धनाक और ऋणाक आविशत िवदयत आयन होत ह और जब यह परर भाव डालत ह तोकाश और िन एक साथ उािदत होती ह

Correct Answer -

A high density cloud contains positively and negatively charged electric ions and when this interacts light andsound are simultaneously produced

औ ि ि ो औ ो

93)

94)

95)

एक उ घन बादल म धनाक और ऋणाक आविशत िवदयत आयन होत ह और जब यह परर भाव डालत ह तोकाश और िन एक साथ उािदत होती ह

Doon Valley is able to grow rice because दन घाटी चावल उगान म सम ह ोिक

1 Other crops cannot be grown वहा अ फसलो को उगाया नही जा सकता ह

2 People in the valley are rice eaters घाटी म लोग चावल खान वाल ह

3 There is a huge export demand of rice वहा चावल की भारी िनयात माग ह

4 It has warm summer and snow melt waters for irrigation

वहा गिमया गम होती ह िसचाई क िलए बफ का िपघला आ पानी होता ह

Correct Answer -

It has warm summer and snow melt waters for irrigation

वहा गिमया गम होती ह िसचाई क िलए बफ का िपघला आ पानी होता ह

CANCELLED

In the geological time scale the Mesozoic Era DOES NOT contains which of the following periods

भगभय समय पमान पर मजीवी यग म िन कालो म स कौन नही ह

1 Triassic ट ाइऐिसक

2 Jurassic जरिसक

3 Cretaceous चाकमय

4 Carboniferous काबनी

Correct Answer -

Carboniferous काबनी

96)

1 P-3 Q-4 R-2 S-1

2 P-3 Q-4 R-1 S-2

3 P-3 Q-4 R-1 S-2

4 P-4 Q-3 R-2 S-1

Correct Answer -

P-4 Q-3 R-2 S-1

1 P-3 Q-1 R-4 S-2

2 P-3 Q-4 R-1 S-2

3 P-3 Q-2 R-4 S-1

97)

98)

4 P-2 Q-1 R-4 S-3

Correct Answer -

P-3 Q-4 R-1 S-2

1 P-3 Q-1 R-4 S-2

2 P-2 Q-3 R-4 S-1

3 P-2 Q-1 R-3 S-4

4 P-4 Q-2 R-1 S-3

Correct Answer -

P-3 Q-1 R-4 S-2

99)

100)

1 P-3 Q-2 R-4 S-1

2 P-1 Q-2 R-3 S-4

3 P-2 Q-3 R-1 S-4

4 P-4 Q-3 R-2 S-1

Correct Answer -

P-2 Q-3 R-1 S-4

ldquoHuman geography is the study of changing relationship between the unresting man and the unstableearthrdquo was defined by

lsquolsquoमानव भगोल ाकल आदमी और अथर पी क बीच सबध परवतन का अयन हrdquo ______ ारा परभािषत िकया गयाथा

1 J Brunches ज चस

2 EC Semple ईसी सल

3 HJ Mackinder एच ज मिकदर

4 PV Blache पीवी च

Correct Answer -

EC Semple ईसी सल

Sedimentary rocks are finally and ultimately derived from the____________

अवसादी चान अततः ________ स ा की जाती ह

1 action of earth movements पी की गितिविधयो

2 marine deposit समी िनप

3 weathering of metamorphic rocks पातरत चानो क अपय

4 weathering of igneous rocks आय चानो क अपय

Correct Answer -

weathering of igneous rocks आय चानो क अपय

Page 23: High School Teacher Eligibility Test- BOARD PROFESSIONAL ...peb.mp.gov.in/results/RESULT_18/HST_RES18/Final_anwser_key/HST… · M a ndl a / मंड ल ... Under the Madhya Pradesh

18)

19)

20)

21)

Correct Answer -

the specific heat of water is higher than land पानी की िविश ऊा भिम स अिधक

The longest shore-line is along the state of सबस लबी समतटीय रखा िन रा क साथ ह

1 Maharashtra महारा

2 Orissa उड़ीसा

3 Kerala करल

4 Gujarat गजरात

Correct Answer -

Gujarat गजरात

The position when the Earth is farthest from the Sun is known as

जब पी सय स सबस दर होती ह तो उस थित को िन नाम स जाना जाता ह

1 Perihelion उपसौर

2 Vernal Equinox बसत िवषव

3 Aphelion अपसौर

4 Autumnal Equinox शराल िवषव

Correct Answer -

Aphelion अपसौर

The seasonal reversal of winds is the typical characteristic of

हवाओ का मौसमी परवतन ______ की सामा िवशषता ह

1 Mediterranean climates only कवल भमसागरीय जलवाय

2 All of the above climates उपय सभी मौसम

3 Monsoon climate only कवल मानसन जलवाय

4 Equatorial climate only कवल भमरखीय जलवाय

Correct Answer -

Monsoon climate only कवल मानसन जलवाय

In _________ rocks the minerals will occurs in beds or layers

ो ि ो ो

22)

23)

24)

______ चानो म खिनज तल या परतो म होत ह

1 metamorphic कायातरत

2 igneous and metamorphic आय और कायातरत

3 igneous आय

4 sedimentary अवसादी

Correct Answer -

sedimentary अवसादी

Black soil is ideal for the cultivation of cotton as कपास की खती क िलए काली िमी आदश ह ोिक

1 Its colour is black यह काली होती ह

2 It is found on plateau regions यह पठार ो म पायी जाती ह

3 It is made up of lava यह लावा स बनी होती ह

4 It can retain moisture यह नमी को बरकरार रख सकती ह

Correct Answer -

It can retain moisture यह नमी को बरकरार रख सकती ह

The National Survey and Mapping Organization of the country works under the Department of___________

दश का रा ीय सवण और मानिचण सगठन ___________ िवभाग क अतगत काय करता ह

1 Space अतर

2 Science and Technology िवान और तकनीक

3 Culture सित

4 Tourism पयटन

Correct Answer -

Science and Technology िवान और तकनीक

Palk strait separates India from पाक जलडमम भारत स _____ को अलग करता ह

1 Pakistan पािकान

2 Andaman Island अडमान ीप

3 China चीन

25)

26)

27)

4 Sri Lanka ीलका

Correct Answer -

Sri Lanka ीलका

Which among the following state is the major producer of Bauxite in India

िनिलखत म स कौन सा रा भारत म बॉाइट का मख उादक ह

1 Madhya Pradesh मदश

2 Rajasthan राजथान

3 Goa गोवा

4 Orissa उड़ीसा

Correct Answer -

Orissa उड़ीसा

Which of the following states DOES NOT share border with Chhattisgarh

िनिलखत म स कौन सा रा छीसगढ़ क साथ सीमा साझा नही करता ह

1 Telangana तलगाना

2 Uttar Pradesh उर दश

3 Bihar िबहार

4 Andhra Pradesh आ दश

Correct Answer -

Bihar िबहार

Which of the following statements is INCORRECT with respect to parallels of latitudes

अाश क समानातरो क सबध म िन निलखत म स कौन सा कथन गलत ह

1 A line joining places of equal latitude is known as parallel of largest

समान अाश क थानो को जोड़न वाली रखा को िवशालतम क समानातर क प म जाना जाता ह

2 They stat from equator and run parallels to it

व भम रखा स ारभ होत ह और इसक समानातर चलत ह

3 All parallels are equal in length सभी समातर लबाई म समान ह

4 All parallels are drawn as circles on the globe ोब पर सभी समानातर वो क प म खीच जात ह

28)

29)

30)

31)

Correct Answer -

All parallels are equal in length सभी समातर लबाई म समान ह

Which of the following Indian states is also known as a lsquoLand of Red river and Blue Hillsrsquo

िनिलखत म स िकस भारतीय रा को लाल नदी और नीली पहािड़यो की भिम क नाम स जाना जाता ह

1 Uttarkhand उराखड

2 Assam असम

3 Meghalaya मघालय

4 Arunachal Pradesh अणाचल दश

Correct Answer -

Assam असम

In spatial analysis of settlement Rn = 215 indicates which type of settlement arrangement

िनपटान क थािनक िवषण म Rn = 215 यह इिगत करता ह िक िकस कार की िनपटान वथा ह

1 Uniform यिनफॉम

2 Semi-Clustered समी- ल टर

3 Clustered ल टर

4 Random रडम

Correct Answer -

Uniform यिनफॉम

Who are known as the lsquoYellow Peoplersquo lsquoयलो पीपलrsquo क प म कौन जाना जाता ह

1 Mongoloids मोगोलोइडस

2 Nigroids नीोइडस

3 Australoids ऑ लॉइडस

4 Caucasoids कॉकसोइडस

Correct Answer -

Mongoloids मोगोलोइडस

ि ि ो ौ ि

32)

33)

34)

Who publishes the topographical map of India भारत क थलाकितक मानिच को कौन कािशत करता ह

1 Geographical Survey of India भारत का भौगोिलक सवण

2 Government of India भारत सरकार

3 Geological Survey of India भारत क भगभय सवण

4 Survey of India भारत का सवण

Correct Answer -

Survey of India भारत का सवण

Who among the following claimed geography to be the lsquoEcology of Manrsquo

िनिलखत म स िकसन भगोल को मन का पारथितकी कहा ह

1 Alfred Hener अड हटनर

2 Vidal-de la Blache वाइडल-िड लॉ ॉश

3 Oo Schluter ओटो टर

4 Harlan Barrow हरलन बारो

Correct Answer -

Harlan Barrow हरलन बारो

Who among the following is regarded as the founder of humanistic approach in geography

िनिलखत म स िकस भगोल म मानवतावादी िकोण का सथापक माना जाता ह

1 William Bunge िविलयम बग

2 Yi-Fu-Tuan यी-फ- यान

3 Brain JL Berry न जएल बरी

4 Richard Peet रचड पीट

Correct Answer -

Yi-Fu-Tuan यी-फ- यान

Who prepared Lorenz curve लोरज व िकसन तयार िकया

1 Geddes गडस

2 None of these इनम स कोई नही

3 Griffith Taylor ििफथ टलर

35)

36)

37)

4 Max U Lorenz म य लोरज

Correct Answer -

Max U Lorenz म य लोरज

Gulf Streams are the currents of which of the following oceans

खाड़ी की धाराए िनिलखत महासागरो म स िकसकी धाराए ह

1 North Atlantic Ocean उरी अटलािटक महासागर

2 North Pacific Ocean उरी शात महासागर

3 Arabian Sea अरब सागर

4 South Pacific Ocean दिण शात महासागर

Correct Answer -

North Atlantic Ocean उरी अटलािटक महासागर

Disintegration wearing away and removal of rock material is generally referred as

िशला पदाथ (रॉक सामी) का टटना िमटना और हटना आमतौर पर ________ क प म सदिभत िकया जाता ह

1 Shattering िवसकारक

2 Denudation अनाादन

3 Fault श

4 Decomposition िवयोजन

Correct Answer -

Denudation अनाादन

Variations in the length of day time and night from season to season are due to

मौसम स मौसम परवतन पर िदन क समय और रात क समय की अविध म िभताए िन कारण स होती ह

1 The Earthrsquos revolution round the Sun in an elliptical manner पी का दीघवाकार तरीक स सय क चारो घणन

2 The Earthrsquos rotation on its axis पी का इसकी धरी पर घणन

3 Revolution of the Earth on a tilted axis नत अ पर पी का घणन

4 Latitudinal position of the place थान की अाश थित

Correct Answer -

Revolution of the Earth on a tilted axis नत अ पर पी का घणन

38)

39)

40)

Point out the correct sequence of mountain ranges from north to south

उर स दिण तक पवत खलाओ क सही अनम को इिगत कर

1 Great Himalaya Middle Himalaya Outer Himalaya Trans Himalaya

महान िहमालय म िहमालय बा िहमालय परा िहमालय

2 Middle Himalaya Great Himalaya Trans Himalaya Outer Himalaya

म िहमालय महान िहमालय परा िहमालय बा िहमालय

3 Outer Himalaya Middle Himalaya Great Himalaya Trans Himalaya

बा िहमालय म िहमालय महान िहमालय परा िहमालय

4 Trans Himalaya Great Himalaya Middle Himalaya Outer Himalaya

परा िहमालय महान िहमालय म िहमालय बा िहमालय

Correct Answer -

Trans Himalaya Great Himalaya Middle Himalaya Outer Himalaya

परा िहमालय महान िहमालय म िहमालय बा िहमालय

Sunrsquos halo is produced by the refraction of light in सय का भामडल ______ म काश क अपवतन ाराउ होता ह

1 Ice crystals in Cirrro-Cumulus clouds पाभ-कपास मघो क बफ िल

2 Ice crystal in Cirrus clouds पाभमघो क बफ िल

3 Dust particles in Stratus clouds री मघो क धल कण

4 Water vapour in Stratus clouds री मघो क जल वा

Correct Answer -

Ice crystal in Cirrus clouds पाभमघो क बफ िल

Read the given statements and answer which of the following options isare correct

(1) The minerals present in the rocks exposed to atmosphere are not subjected to alteration

(2) Oxidation is one of the processes of chemical weathering

िदए गए कथनो को पढ़ और उर द िक िन म स कौन सास िवक सही ह

(1) वायमल स अनावत शल म उपथत खिनज परवतन क अधीन नही होता ह

(2) ऑीकरण रासायिनक अपय की ियाओ म स एक ह

ो ो

41)

42)

1 Both statements are wrong दोनो कथन गलत ह

2 Both statements are correct दोनो कथन सही ह

3 First statement is wrong and second statement is correct पहला कथन गलत ह और दसरा कथन सही ह

4 First statement is correct and second statement is wrong पहला कथन सही ह और दसरा कथन गलत ह

Correct Answer -

First statement is wrong and second statement is correct पहला कथन गलत ह और दसरा कथन सही ह

Read the given statements and answer which of the following options isare correct

1 Sunrsquos short waves enter the earth partially heating the atmosphere

2 Heated earth surface from the sun produces broader waves which interacts and heats the atmosphere

िदए गए कथन को पढ़ और उर द िक िन म स कौन सास िवक सही ह

1 सय की छोटी तरग पी म आिशक प स वश करती ह और वायमडल को ऊत करती ह

2 सय स ऊत पी की सतह िवारत तरग उ करती ह जो परर भाव डालती ह और वायमडल कोऊत करती ह

1 Both Statements 1 and 2 are correct दोनो कथन 1 और 2 सही ह

2 Both Statements 1 and 2 are wrong दोनो कथन 1 और 2 गलत ह

3 Statement 1 is wrong and only Statement 2 is correct कथन 1 गलत ह और कवल कथन 2 सही ह

4 Only statement 1 is correct कवल कथन 1 सही ह

Correct Answer -

Both Statements 1 and 2 are correct दोनो कथन 1 और 2 सही ह

Read the given statements and answer which of the following options isare correct

(1)The rocks that get changed due to heat and pressure are termed as metamorphic rocks

(2)Slate is one such type of metamorphic rock

िदए गए कथनो को पढ़ और उर द िक िन म स कौन सास िवक सही ह

(1) शल जो ऊा और दाब क कारण परवितत हो जात ह उ कायातरक शलो क प म जाना जाता ह

(2) ट एक तरह का कायातरक शल ह

1 Both statements are wrong दोनो कथन गलत ह

2 Both statements are correct दोनो कथन सही ह

3 First statement is wrong and second statement is correct पहला कथन गलत ह और दसरा कथन सही ह

ी औ

43)

44)

4 First statement is correct and second statement is wrong पहला कथन सही ह और दसरा कथन गलत ह

Correct Answer -

Both statements are correct दोनो कथन सही ह

Read the given statements and answer which of the following options isare correct

1 Higher temperature anomaly is observed in the northern hemisphere

2 Differential heating is absent in Northern Hemisphere

िदए गए कथनो को पढ़ और उर द िक िन म स कौन सास िवक सही ह

1 उरी गोलाध म उ तापमान िवसगित पायी जाती ह

2 उरी गोलाध म अतर ऊन अनपथत होती ह

1 Both Statements 1 and 2 are correct दोनो कथन 1 और 2 सही ह

2 Both Statements 1 and 2 are wrong दोनो कथन 1 और 2 गलत ह

3 Statement 1 is wrong and Statement 2 is correct कथन 1 गलत ह और कथन 2 सही ह

4 Statement 1 is correct and Statement 2 is wrong कथन 1 सही ह और कथन 2 गलत ह

Correct Answer -

Statement 1 is correct and Statement 2 is wrong कथन 1 सही ह और कथन 2 गलत ह

Read the given statements and answer which of the following options isare correct

(1) Plutonic rocks are intrusive type of igneous rocks

(2) It cools very slowly because the surrounding rock serves as insulation around the intrusion of magma

िदए गए कथनो को पढ़ और उर द िक िन म स कौन सास िवक सही ह

(1) िवतलीय शल अतवधी कार क आश शल ह

(2) यह बत धीर-धीर ठडा होता ह ोिक आस-पास क शल मा क अतवधन क चारो ओर रोधन क प म कायकरत ह

1 Both statements are wrong दोनो कथन गलत ह

2 Both statements are correct दोनो कथन सही ह

3 First statement is wrong and second statement is correct पहला कथन गलत ह और दसरा कथन सही ह

4 First statement is correct and second statement is wrong पहला कथन सही ह और दसरा कथन गलत ह

Correct Answer -

Both statements are correct दोनो कथन सही ह

45)

46)

47)

48)

The dust and ash material hurled from the volcanoes are termed as

ालामखी स िनकलन वाली धल और राख सामी को _______ क प म कहा जाता ह

1 Pyroclasc पाइरोाक

2 Hyperclastic हाइपराक

3 Hepiroclastic हिपरोाक

4 Cirroclastic िसरोाक

Correct Answer -

Pyroclasc पाइरोाक

The vertical difference in elevation between a low tide and high tide is referred as

कम ार और उ ार क बीच ऊचाई म लबवत अतर _____ स सदिभत होता ह

1 Tidal slope ारीय ढलान

2 Tidal elevation ारीय उयन

3 Tidal range ारीय परास

4 Tidal height ारीय ऊचाई

Correct Answer -

Tidal range ारीय परास

The maximum biodiversity is found in which of the following regions िनिलखत ो म स अिधकतमजव िविवधता िकसम पायी जाती ह

1 Amazon Basin अमज़न बिसन

2 East Indies ई इडीज

3 Congo Basin कागो बिसन

4 West indies व इडीज

Correct Answer -

Amazon Basin अमज़न बिसन

The cultivation of rice crop produces_______ चावल की फसल की खती ______ का उादन करती ह

1 SO2

49)

50)

51)

2 CH4

3 CFCs

4 CO2

Correct Answer -

CH4

The pressure system with higher pressure at the centre is called__________

क म उ दबाव वाली दबाव णाली को _______ कहा जाता ह

1 front अ

2 depression अवनमन

3 cyclone चवात

4 anti-cyclone ितचवात

Correct Answer -

anti-cyclone ितचवात

The Himalayan region is poor in mineral resources because िहमालयी खिनज ससाधनो म समनही ह ोिक

1 The displacement of rock strata has disturbed the arrangement of rocks and made it complex

शलीय परत क िवथापन न चानो की वथा को अवथत कर िदया ह और इस जिटल बना िदया ह

2 The climate conditions are not suitable for exploitation of minerals

जलवाय की थित खिनजो क दोहन क िलए उपय नही ह

3 The terrain makes explanation of minerals difficult and very costly due to transportation difficulties

भ-भाग परवहन की किठनाइयो क कारण खिनजो का दोहन मल और बत महगा बना दता ह

4 It is made up of crystalline rocks यह िलीय चानो स बना ह

Correct Answer -

The displacement of rock strata has disturbed the arrangement of rocks and made it complex

शलीय परत क िवथापन न चानो की वथा को अवथत कर िदया ह और इस जिटल बना िदया ह

The process through which the moisture is added to the atmosphere by vegetation is termed as

वह िया िजसक माम स वनित ारा वातावरण म नमी िमलायी जाती ह _______ क प म जानी जाती ह

52)

53)

54)

1 Condensation सघनन

2 Evapotranspiration वान-उजन

3 Radiation िविकरण

4 Precipitation वषण

Correct Answer -

Evapotranspiration वान-उजन

The process through which the terrestrial heat is transferred to air by direct contact is termed as

वह िया िजसम सपक ारा थलीय ऊा वाय म थानातरत हो जाती ह ______ क प म जानी जाती ह

1 Conduction चालन

2 Convection सवहन

3 Insolation आतपन

4 Radiation िविकरण

Correct Answer -

Conduction चालन

The largest area under mangroves is in which of the following statesunion territory

मोव क अतगत िनिलखत राोसघ शािसत दशो म स सबस बड़ा कौन सा ह

1 Andaman and Nicobar अमान और िनकोबार

2 Andhra Pradesh आ दश

3 West Bengal पिम बगाल

4 Gujarat गजरात

Correct Answer -

West Bengal पिम बगाल

The longitudinal transverse and surface waves in an earthquake originate from

भकप म दशातर अनथ और सतह तरग यहा उ होती ह

1 The focus on the surface of the Earth पी क सतह पर क -िबद म

2 The focus within the body of the Earth पी क भीतर क -िबद म

3 The epicenter within the body of the Earth पी क भीतर उपरक म

55)

56)

57)

4 The epicenter on the surface of the Earth पी क सतह पर उपरक म

Correct Answer -

The focus within the body of the Earth पी क भीतर क -िबद म

The down slope movement of material due to gravity is called______

गाकषण क कारण पदाथ की अनढाल गित को ______ कहा जाता ह

1 mass movement पदाथ सचलन

2 deposition िनप

3 erosion रण

4 volcanic movement ालामखीय सचलन

Correct Answer -

mass movement पदाथ सचलन

Shimla is cooler than Amritsar although both are on the same latitude This is because

िशमला म अमतसर स अिधक ठड ह हालािक दोनो समान अाश पर ह ऐसा ह ोिक

1 Shimla is at a greater height above sea level than Amritsar अमतसर की तलना म िशमला सम तल स अिधकऊचाई पर ह

2 Shimla is further north िशमला उर की ओर ह

3 Shimla is farther from the equator िशमला भम रखा स आग ह

4 Their longitudes differ उनकी दशातर रखाए िभ ह

Correct Answer -

Shimla is at a greater height above sea level than Amritsar अमतसर की तलना म िशमला सम तल स अिधकऊचाई पर ह

lsquoTempo of Urbanizationrsquo measures which of the following

lsquoशहरीकरण का टपोrsquo िनिलखत म स कौन सा उपाय ह

1 Speed of urbanizaon शहरीकरण की गित

2 None of the above इनम स कोई नही

3 Inequality of urbanizaon शहरीकरण की असमानता

4 Current level of urbanizaon शहरीकरण का वतमान र

Correct Answer -

58)

59)

60)

Speed of urbanizaon शहरीकरण की गित

Out of the following options choose the INCORRECT statement

िनिलखत िवको म स गलत कथन का चयन कर

1 The clear tracts in the equatorial region recover rapidly भम रखा म भभाग तजी स ठीक हो जात ह

2 The stable communities include a redwood forest a pine forest at high elevations

थर समदायो म एक रडवड वन उ ऊचाई पर एक दवदार वन शािमल ह

3 Any ecosystem moves towards maximum biomass and stability to survive

कोई भी पारथितकी त जीिवत रहन क िलए अिधकतम जवसहित और थरता की तरफ असर होता ह

4 Tropical rain forests near equator are stable ecosystems

भम रखा क पास उकिटबधीय वषा वन थर पारथितक त ह

Correct Answer -

The clear tracts in the equatorial region recover rapidly भम रखा म भभाग तजी स ठीक हो जात ह

Seasonal contrasts are maximum in मौसमी िवषमता अिधकतम ह

1 Mid latitudes म अाश म

2 Low attitudes िन अाश म

3 High latitudes उ अाश म

4 Subtropics उपोकिटबधीय म

Correct Answer -

Mid latitudes म अाश म

In India which type of forest among the following occupies the largest area

भारत म िनिलखत म स िकस कार क वन सबस बड़ा फल आािदत करत ह

1 Sub-tropical Dry Evergreen Forest उप उकिटबधीय श सदाबहार वन

2 Mountain Wet Temperate Forest पवतीय आ शीतो वन

3 Tropical Moist Deciduous Forest उकिटबधीय आ पणपाती वन

4 Tropical Wet Evergreen Forest उकिटबधीय आ सदाबहार वन

Correct Answer -

Tropical Moist Deciduous Forest उकिटबधीय आ पणपाती वन

61)

62)

63)

64)

What is the proportion of lsquoJuvenile Populationrsquo (0-14 years) in India as per 2011Census

2011 की जनगणना क अनसार भारत म जवनाइल पॉपलशन यानी िकशोर जनस या (0-14 वष) का अनपात ाह

1 3076 of total population कल जनस या का 3076

2 2764 of total population कल जनस या का 2764

3 2933 of total population कल जनस या का 2933

4 3354 of total population कल जनस या का 3354

Correct Answer -

3076 of total population कल जनस या का 3076

What is the Belfast famous for बलफा िकसक िलए मशर ह

1 Belt of cotton textile industry कपास व उोग क

2 Ship-building industry जहाज िनमाण उोग

3 Agricultural machinery किष उपकरण

4 Aero planes manufacturing वाययान िनमाण

Correct Answer -

Ship-building industry जहाज िनमाण उोग

What is the most important occupation in tropical monsoon lands

उकिटबधीय मॉनसन भिम म सबस महपण वसाय ा ह

1 Mining खनन

2 Cattle rearing मवशी पालन

3 Agriculture किष

4 Nomadic herding नोमािडक जड़ी-बिटया

Correct Answer -

Agriculture किष

What is the most important characteristics of the islands (Indian) located in the Arabian sea

अरब सागर म थत ीपो (भारतीय) की सबस महपण िवशषता ा ह

ी ो

65)

66)

67)

1 There are all of coral origins सभी कोरल मल क ह

2 There are all very small in size य सभी आकार म बत छोट ह

3 They have a very dry climate इनकी जलवाय बत श ह

4 They are extended parts of the mainland व महाीप क िवारत िह ह

Correct Answer -

There are all of coral origins सभी कोरल मल क ह

What do the basalt layers of the Deccan indicate डन की बसा परत ा इिगत करती ह

1 All of the above उपरो सभी

2 Huge volcanic eruptions in the distant past दरथ अतीत म िवशाल ालामखीय िवोट

3 The immense erosional activity of the rivers निदयो की िवशाल रण गितिविध

4 The influence of weathering मौसम का भाव

Correct Answer -

Huge volcanic eruptions in the distant past दरथ अतीत म िवशाल ालामखीय िवोट

In the structure of planet Earth below the mantle the core is mainly made up of_____

पी ह की सरचना म मटल क नीच कोर म प स______ स िनिमत होती ह

1 aluminium एमीिनयम

2 silicon िसिलकॉन

3 chromium ोिमयम

4 iron लोहा

Correct Answer -

iron लोहा

One of the major Mid Oceanic Ridge is found in मख म-महासागर चोिटयो म स एक ______ म पायाजाता ह

1 Mid Pacific Ocean म शात महासागर

2 Mid Atlantic Ocean म अटलािटक महासागर

3 Mid Indian Ocean म भारतीय महासागर

4 Mid Arctic Ocean म आक िटक महासागर

68)

69)

70)

71)

Correct Answer -

Mid Atlantic Ocean म अटलािटक महासागर

Magma that reaches the Earthrsquos surface and then solidifies is called________

मा जो पी की सतह तक पचती ह और िफर ठोस हो जाती ह ________कहलाती ह

1 quartz ाटज

2 lava लावा

3 granite नाइट

4 silicates िसिलकट

Correct Answer -

lava लावा

Isotherms are the lines of equal_______ समताप रखाए समान _______की रखाए होती ह

1 pressure दाब

2 temperature तापमान

3 rainfall वषा

4 height ऊचाई

Correct Answer -

temperature तापमान

Mark the correct sequence of passes in the Western Ghats from north to south

पिमी घाटो म उर स दिण तक दर क सही अनम को िचित कर

1 Thalghat Palghat Bhorghat थलगघाट पालघाट भोरघाट

2 Thalghat Bhorghat Palghat थलघाट भोरघाट पालघाट

3 Bhorghat Thalghat Palghat भोरघाट थलघाट पालघाट

4 Palghat Bhorghat Thalghat पालघाट भोरघाट थलघाट

Correct Answer -

Thalghat Bhorghat Palghat थलघाट भोरघाट पालघाट

Which of the following does not have influence over the climate in India

ि ि ि ी ी

72)

73)

िनिलखत म स िकसका भाव भारत की जलवाय पर नही पड़ता ह

1 Ocean currents सागर की लहर

2 Nearness to equator भम रखा स िनकटता

3 Monsoons मानसन

4 Presence of Indian ocean भारतीय महासागर की उपथित

Correct Answer -

Ocean currents सागर की लहर

Which of the following cloud types has the characteristics like vertical tall narrow and puffy

िनिलखत म स िकस कार क मघो म लबवत लबी सकीण और थलता जसी िवशषताए ह

1 Cumulonimbus तफानी मघ

2 Cumulus मघ पज

3 Cirrocumulus पाभ कपासी मघ

4 Nimbostratus वषारी मघ

Correct Answer -

Cumulus मघ पज

Which of the following statement is INCORRECT about Crude Birth Rate

िनिलखत स कौन सा कथन अशोिधत ज दर क बार म सही नही ह

1 It cannot be used for comparing fertility level between two countries with different population characteristics

इसका उपयोग िविभ जनसा िवशषताओ वाल दो दशो क बीच जनन र की तलना क िलए नही िकया जा सकता ह

2 It is a standardized measure of fertility

यह जनन मता का मानकीकत उपाय ह

3 It is effected by the age-sex composition of the population

यह आबादी की आय-िलग सरचना स भािवत होता ह

4 It is expressed per 1000 population in a given geographical unit

यह िकसी दी गई भौगोिलक इकाई म ित 1000 जनसा पर िकया जाता ह

Correct Answer -

It is a standardized measure of fertility

यह जनन मता का मानकीकत उपाय ह

74)

75)

76)

77)

Which of the following state in India experienced negative decadal growth rate during 2001 to 2011census

भारत म िनिलखत म स िकस रा म वष 2001 स 2011 की जनगणना क दौरान नकाराक िगरावट दर ई

1 Tripura िपरा

2 Nagaland नागालड

3 Haryana हरयाणा

4 Odisha ओिडसा

Correct Answer -

Nagaland नागालड

Which of the following is NOT a characteristic of peninsular rivers

िनिलखत म स कौन सी िवशषता ायीपीय निदयो म नही होती ह

1 Flow through shallow valleys उथल घािटयो क माम स वाह

2 Seasonal flow मौसमी वाह

3 Little erosional activity थोड़ी कटावदार गितिविध

4 Meandering tendency often shifting their beds घमावदार वि अर अपन तटो को थानातरत करना

Correct Answer -

Meandering tendency often shifting their beds घमावदार वि अर अपन तटो को थानातरत करना

Which of the following gases in the atmosphere absorbs heat from the Sunrsquos radiation and the Earthssurface

वायमडल म िनिलखत म स कौन सी गस सय क िविकरण और पी की सतह स ऊा को अवशोिषत करती ह

1 Neon िनयॉन

2 Carbon dioxide काबन डाइऑाइड

3 Argon आगन

4 Nitrogen नाइट ोजन

Correct Answer -

Carbon dioxide काबन डाइऑाइड

Which of the following kind of settlement pattern is found at the confluence of rivers

ि ि ि ि ो

78)

79)

80)

िनिलखत म स िकस कार का वथापन पटन निदयो क सगम पर पाया जाता ह

1 Triangular Paern िकोणीय पटन

2 Circular or Semi-Circular Paern परप या अध-परप पटन

3 Nebular Paern नबलर पटन

4 Star ndashShaped Paern ार-आकार का पटन

Correct Answer -

Triangular Paern िकोणीय पटन

Which one was not the objective of the Biosphere Reserve Projects launched by the UNESCO

यनो ारा श की गई सरित जवमडल परयोजनाओ का उ इनम स कौन सा नही था

1 To promote teaching and research िशण और अनसधान को बढ़ावा दना

2 To make agriculture sustainable किष को दीघकािलक बनाना

3 To conserve ecosystems पारथितक त को सरित करना

4 To conserve genetic diversity for a longtime लब समय तक अनवािशक िविवधता को सरित करना

Correct Answer -

To make agriculture sustainable किष को दीघकािलक बनाना

Which region of the Earth surface receives the highest amount of insulation

पी सतह का कौन सा तापावरोधन की उतम माा ा करता ह

1 Land mass थलखड

2 Savannah region सवाना

3 Water bodies जल िनकाय

4 Tropical desert उकिटबधीय रिगान

Correct Answer -

Tropical desert उकिटबधीय रिगान

Which one of the following is not a biodiversity hotspot

िनिलखत म स कौन सा जव िविवधता का म जगह नही ह

1 Eastern Himalaya पव िहमालय

2 Eastern Ghats पव घाट

81)

82)

83)

3 Indo-Myanmar भारत-ामार

4 Westerm Ghats पिमी घाट

Correct Answer -

Eastern Ghats पव घाट

Which one of the following is NOT a part of the World Network of Biosphere Reserves based on theUNESCO Man and Biosphere Programme

यनो मन और बायोीयर कायम क आधार पर िनिलखत म स कौन बायोीयर रजव क िव नटवक कािहा नही ह

1 Gulf of Mannar मार की खाड़ी

2 Seshachalam शषाचलम

3 Sunderban सदरबन

4 Nilgiri नीलिगर

Correct Answer -

Seshachalam शषाचलम

Which one of the following is an example of ldquodesert vegetationrdquo

िनिलखत म स कौन मथलीय वनित का एक उदाहरण ह

1 Mosses and lichens दलदल और शवाल

2 Temperate grassland समशीतो घास क मदान

3 Coniferous forest शकधारी वन

4 Acacia and cactus एकािसया और कस

Correct Answer -

Acacia and cactus एकािसया और कस

Which one of the following reflects more sunlight िनिलखत म स कौन सा सय की रोशनी को अिधकपरावितत करता ह

1 Paddy crop land धान फसल भिम

2 Land covered with fresh snow ताजा बफ स आािदत भिम

3 Sand desert रतीली रिगान

4 Prairie land यरी भिम

84)

85)

86)

87)

Correct Answer -

Land covered with fresh snow ताजा बफ स आािदत भिम

Which layer of the atmosphere is in contact with the surface of the earthrsquos oceans

वायमडल की कौन सी परत पी क महासागरो की सतह क सपक म ह

1 Stratosphere समताप मडल

2 Mesosphere म मडल

3 Hydrosphere जलमडल

4 Troposphere ोभ मडल

Correct Answer -

Troposphere ोभ मडल

Mediterranean Sea is a border of which of the following countries भम सागर िनिलखत दशो म सिकसकी सीमा ह

1 None of these इनम स कोई नही

2 Iraq इराक

3 Lebanon लबनान

4 Jordan जॉडन

Correct Answer -

Lebanon लबनान

Benguela ocean currents are found along which coast बगएला महासागर धाराए िकस तट क साथ पायीजाती ह

1 East Coast of South America दिण अमरका क पव तट

2 East Coast of Africa अीका क पव तट

3 West Coast of South America दिण अमरका क पिमी तट

4 West Coast of Africa अीका क पिमी तट

Correct Answer -

West Coast of Africa अीका क पिमी तट

88)

89)

90)

Due to tension a block of land on one side being pushed up or upthrown relative to the downthrown blockis referred as

तनाव क कारण नीच फ क ए खड क साप भिम का एक खड एक ओर स ऊपर धकला जाता ह या ऊपर की ओरफ का जाता ह यह _____ क प म सदिभत ह

1 Thrust fault प श

2 Normal fault सामा श

3 Reverse fault म श

4 Strike slip fault नितलब सपण श

Correct Answer -

Normal fault सामा श

Inter-tropical doldrums is a zone of ______ अतर-उकिटबधीय डोलड ______ का एक ह

1 Frontolysis टोलायिसस

2 Convergence अिभसरण

3 Inter-tropical divergence zone अतर-उकिटबधीय िवचलन

4 Local wind थानीय वाय

Correct Answer -

Convergence अिभसरण

The Horse Latitudes are regions located at about _____ north and south of the equator

हॉस अाश भम रखा क उर और दिण म लगभग _____ पर थत ह

1 30ndash60 degree Latitude 30-60 िडी अाश

2 0ndash5 degree Latitude 0-5 िडी अाश

3 30 degree Latitude 30 िडी अाश

4 60ndash90 degree Latitude 60-90 िडी अाश

Correct Answer -

30 degree Latitude 30 िडी अाश

Generally evaporation is high over which part of the Earth

आम तौर पर पी क िकस भाग पर वाीकरण अिधक होता ह

1 Equatorial maritime भमवत समीय ी ी

91)

92)

2 Equatorial continental भमवत महाीपीय

3 Polar maritime वीय समीय

4 Polar continental वीय महाीपीय

Correct Answer -

Equatorial maritime भमवत समीय

A very high temperature during summer in north western India leads to what type of climaticcondition in south

उर पिमी भारत म गम क दौरान बत अिधक तापमान होन क कारण दिण म िकस कार की जलवाय थितउ करता ह

1 Depression over arabian sea अरब सागर पर अवनमन

2 Failure monsoon मानसन िवफलता

3 Successful monsoon मानसन सफलता

4 Cyclones चवात

Correct Answer -

Successful monsoon मानसन सफलता

Lightning and thunder are the resultant effect when तिड़त और गजन परणामी भाव ह जब

1 Two massive clouds hit powerfully each other first lightning is produced and later sound is produced

दो बड़ बादल एक दसर स शशाली ढग स टकरात ह पहल आकाशीय िवदयत उ होता ह और बाद म िन उहोती ह

2 Two massive clouds come into contact with the powerful wind collision this results into first sound and thenlightning

दो बड़ बादल शशाली पवन सघ क सपक म आत ह इसका परणामप पहल िन और िफर आकाशीय िवदयतउ होता ह

3 None of the above उपरो म स कोई भी नही

4 A high density cloud contains positively and negatively charged electric ions and when this interacts light andsound are simultaneously produced

एक उ घन बादल म धनाक और ऋणाक आविशत िवदयत आयन होत ह और जब यह परर भाव डालत ह तोकाश और िन एक साथ उािदत होती ह

Correct Answer -

A high density cloud contains positively and negatively charged electric ions and when this interacts light andsound are simultaneously produced

औ ि ि ो औ ो

93)

94)

95)

एक उ घन बादल म धनाक और ऋणाक आविशत िवदयत आयन होत ह और जब यह परर भाव डालत ह तोकाश और िन एक साथ उािदत होती ह

Doon Valley is able to grow rice because दन घाटी चावल उगान म सम ह ोिक

1 Other crops cannot be grown वहा अ फसलो को उगाया नही जा सकता ह

2 People in the valley are rice eaters घाटी म लोग चावल खान वाल ह

3 There is a huge export demand of rice वहा चावल की भारी िनयात माग ह

4 It has warm summer and snow melt waters for irrigation

वहा गिमया गम होती ह िसचाई क िलए बफ का िपघला आ पानी होता ह

Correct Answer -

It has warm summer and snow melt waters for irrigation

वहा गिमया गम होती ह िसचाई क िलए बफ का िपघला आ पानी होता ह

CANCELLED

In the geological time scale the Mesozoic Era DOES NOT contains which of the following periods

भगभय समय पमान पर मजीवी यग म िन कालो म स कौन नही ह

1 Triassic ट ाइऐिसक

2 Jurassic जरिसक

3 Cretaceous चाकमय

4 Carboniferous काबनी

Correct Answer -

Carboniferous काबनी

96)

1 P-3 Q-4 R-2 S-1

2 P-3 Q-4 R-1 S-2

3 P-3 Q-4 R-1 S-2

4 P-4 Q-3 R-2 S-1

Correct Answer -

P-4 Q-3 R-2 S-1

1 P-3 Q-1 R-4 S-2

2 P-3 Q-4 R-1 S-2

3 P-3 Q-2 R-4 S-1

97)

98)

4 P-2 Q-1 R-4 S-3

Correct Answer -

P-3 Q-4 R-1 S-2

1 P-3 Q-1 R-4 S-2

2 P-2 Q-3 R-4 S-1

3 P-2 Q-1 R-3 S-4

4 P-4 Q-2 R-1 S-3

Correct Answer -

P-3 Q-1 R-4 S-2

99)

100)

1 P-3 Q-2 R-4 S-1

2 P-1 Q-2 R-3 S-4

3 P-2 Q-3 R-1 S-4

4 P-4 Q-3 R-2 S-1

Correct Answer -

P-2 Q-3 R-1 S-4

ldquoHuman geography is the study of changing relationship between the unresting man and the unstableearthrdquo was defined by

lsquolsquoमानव भगोल ाकल आदमी और अथर पी क बीच सबध परवतन का अयन हrdquo ______ ारा परभािषत िकया गयाथा

1 J Brunches ज चस

2 EC Semple ईसी सल

3 HJ Mackinder एच ज मिकदर

4 PV Blache पीवी च

Correct Answer -

EC Semple ईसी सल

Sedimentary rocks are finally and ultimately derived from the____________

अवसादी चान अततः ________ स ा की जाती ह

1 action of earth movements पी की गितिविधयो

2 marine deposit समी िनप

3 weathering of metamorphic rocks पातरत चानो क अपय

4 weathering of igneous rocks आय चानो क अपय

Correct Answer -

weathering of igneous rocks आय चानो क अपय

Page 24: High School Teacher Eligibility Test- BOARD PROFESSIONAL ...peb.mp.gov.in/results/RESULT_18/HST_RES18/Final_anwser_key/HST… · M a ndl a / मंड ल ... Under the Madhya Pradesh

22)

23)

24)

______ चानो म खिनज तल या परतो म होत ह

1 metamorphic कायातरत

2 igneous and metamorphic आय और कायातरत

3 igneous आय

4 sedimentary अवसादी

Correct Answer -

sedimentary अवसादी

Black soil is ideal for the cultivation of cotton as कपास की खती क िलए काली िमी आदश ह ोिक

1 Its colour is black यह काली होती ह

2 It is found on plateau regions यह पठार ो म पायी जाती ह

3 It is made up of lava यह लावा स बनी होती ह

4 It can retain moisture यह नमी को बरकरार रख सकती ह

Correct Answer -

It can retain moisture यह नमी को बरकरार रख सकती ह

The National Survey and Mapping Organization of the country works under the Department of___________

दश का रा ीय सवण और मानिचण सगठन ___________ िवभाग क अतगत काय करता ह

1 Space अतर

2 Science and Technology िवान और तकनीक

3 Culture सित

4 Tourism पयटन

Correct Answer -

Science and Technology िवान और तकनीक

Palk strait separates India from पाक जलडमम भारत स _____ को अलग करता ह

1 Pakistan पािकान

2 Andaman Island अडमान ीप

3 China चीन

25)

26)

27)

4 Sri Lanka ीलका

Correct Answer -

Sri Lanka ीलका

Which among the following state is the major producer of Bauxite in India

िनिलखत म स कौन सा रा भारत म बॉाइट का मख उादक ह

1 Madhya Pradesh मदश

2 Rajasthan राजथान

3 Goa गोवा

4 Orissa उड़ीसा

Correct Answer -

Orissa उड़ीसा

Which of the following states DOES NOT share border with Chhattisgarh

िनिलखत म स कौन सा रा छीसगढ़ क साथ सीमा साझा नही करता ह

1 Telangana तलगाना

2 Uttar Pradesh उर दश

3 Bihar िबहार

4 Andhra Pradesh आ दश

Correct Answer -

Bihar िबहार

Which of the following statements is INCORRECT with respect to parallels of latitudes

अाश क समानातरो क सबध म िन निलखत म स कौन सा कथन गलत ह

1 A line joining places of equal latitude is known as parallel of largest

समान अाश क थानो को जोड़न वाली रखा को िवशालतम क समानातर क प म जाना जाता ह

2 They stat from equator and run parallels to it

व भम रखा स ारभ होत ह और इसक समानातर चलत ह

3 All parallels are equal in length सभी समातर लबाई म समान ह

4 All parallels are drawn as circles on the globe ोब पर सभी समानातर वो क प म खीच जात ह

28)

29)

30)

31)

Correct Answer -

All parallels are equal in length सभी समातर लबाई म समान ह

Which of the following Indian states is also known as a lsquoLand of Red river and Blue Hillsrsquo

िनिलखत म स िकस भारतीय रा को लाल नदी और नीली पहािड़यो की भिम क नाम स जाना जाता ह

1 Uttarkhand उराखड

2 Assam असम

3 Meghalaya मघालय

4 Arunachal Pradesh अणाचल दश

Correct Answer -

Assam असम

In spatial analysis of settlement Rn = 215 indicates which type of settlement arrangement

िनपटान क थािनक िवषण म Rn = 215 यह इिगत करता ह िक िकस कार की िनपटान वथा ह

1 Uniform यिनफॉम

2 Semi-Clustered समी- ल टर

3 Clustered ल टर

4 Random रडम

Correct Answer -

Uniform यिनफॉम

Who are known as the lsquoYellow Peoplersquo lsquoयलो पीपलrsquo क प म कौन जाना जाता ह

1 Mongoloids मोगोलोइडस

2 Nigroids नीोइडस

3 Australoids ऑ लॉइडस

4 Caucasoids कॉकसोइडस

Correct Answer -

Mongoloids मोगोलोइडस

ि ि ो ौ ि

32)

33)

34)

Who publishes the topographical map of India भारत क थलाकितक मानिच को कौन कािशत करता ह

1 Geographical Survey of India भारत का भौगोिलक सवण

2 Government of India भारत सरकार

3 Geological Survey of India भारत क भगभय सवण

4 Survey of India भारत का सवण

Correct Answer -

Survey of India भारत का सवण

Who among the following claimed geography to be the lsquoEcology of Manrsquo

िनिलखत म स िकसन भगोल को मन का पारथितकी कहा ह

1 Alfred Hener अड हटनर

2 Vidal-de la Blache वाइडल-िड लॉ ॉश

3 Oo Schluter ओटो टर

4 Harlan Barrow हरलन बारो

Correct Answer -

Harlan Barrow हरलन बारो

Who among the following is regarded as the founder of humanistic approach in geography

िनिलखत म स िकस भगोल म मानवतावादी िकोण का सथापक माना जाता ह

1 William Bunge िविलयम बग

2 Yi-Fu-Tuan यी-फ- यान

3 Brain JL Berry न जएल बरी

4 Richard Peet रचड पीट

Correct Answer -

Yi-Fu-Tuan यी-फ- यान

Who prepared Lorenz curve लोरज व िकसन तयार िकया

1 Geddes गडस

2 None of these इनम स कोई नही

3 Griffith Taylor ििफथ टलर

35)

36)

37)

4 Max U Lorenz म य लोरज

Correct Answer -

Max U Lorenz म य लोरज

Gulf Streams are the currents of which of the following oceans

खाड़ी की धाराए िनिलखत महासागरो म स िकसकी धाराए ह

1 North Atlantic Ocean उरी अटलािटक महासागर

2 North Pacific Ocean उरी शात महासागर

3 Arabian Sea अरब सागर

4 South Pacific Ocean दिण शात महासागर

Correct Answer -

North Atlantic Ocean उरी अटलािटक महासागर

Disintegration wearing away and removal of rock material is generally referred as

िशला पदाथ (रॉक सामी) का टटना िमटना और हटना आमतौर पर ________ क प म सदिभत िकया जाता ह

1 Shattering िवसकारक

2 Denudation अनाादन

3 Fault श

4 Decomposition िवयोजन

Correct Answer -

Denudation अनाादन

Variations in the length of day time and night from season to season are due to

मौसम स मौसम परवतन पर िदन क समय और रात क समय की अविध म िभताए िन कारण स होती ह

1 The Earthrsquos revolution round the Sun in an elliptical manner पी का दीघवाकार तरीक स सय क चारो घणन

2 The Earthrsquos rotation on its axis पी का इसकी धरी पर घणन

3 Revolution of the Earth on a tilted axis नत अ पर पी का घणन

4 Latitudinal position of the place थान की अाश थित

Correct Answer -

Revolution of the Earth on a tilted axis नत अ पर पी का घणन

38)

39)

40)

Point out the correct sequence of mountain ranges from north to south

उर स दिण तक पवत खलाओ क सही अनम को इिगत कर

1 Great Himalaya Middle Himalaya Outer Himalaya Trans Himalaya

महान िहमालय म िहमालय बा िहमालय परा िहमालय

2 Middle Himalaya Great Himalaya Trans Himalaya Outer Himalaya

म िहमालय महान िहमालय परा िहमालय बा िहमालय

3 Outer Himalaya Middle Himalaya Great Himalaya Trans Himalaya

बा िहमालय म िहमालय महान िहमालय परा िहमालय

4 Trans Himalaya Great Himalaya Middle Himalaya Outer Himalaya

परा िहमालय महान िहमालय म िहमालय बा िहमालय

Correct Answer -

Trans Himalaya Great Himalaya Middle Himalaya Outer Himalaya

परा िहमालय महान िहमालय म िहमालय बा िहमालय

Sunrsquos halo is produced by the refraction of light in सय का भामडल ______ म काश क अपवतन ाराउ होता ह

1 Ice crystals in Cirrro-Cumulus clouds पाभ-कपास मघो क बफ िल

2 Ice crystal in Cirrus clouds पाभमघो क बफ िल

3 Dust particles in Stratus clouds री मघो क धल कण

4 Water vapour in Stratus clouds री मघो क जल वा

Correct Answer -

Ice crystal in Cirrus clouds पाभमघो क बफ िल

Read the given statements and answer which of the following options isare correct

(1) The minerals present in the rocks exposed to atmosphere are not subjected to alteration

(2) Oxidation is one of the processes of chemical weathering

िदए गए कथनो को पढ़ और उर द िक िन म स कौन सास िवक सही ह

(1) वायमल स अनावत शल म उपथत खिनज परवतन क अधीन नही होता ह

(2) ऑीकरण रासायिनक अपय की ियाओ म स एक ह

ो ो

41)

42)

1 Both statements are wrong दोनो कथन गलत ह

2 Both statements are correct दोनो कथन सही ह

3 First statement is wrong and second statement is correct पहला कथन गलत ह और दसरा कथन सही ह

4 First statement is correct and second statement is wrong पहला कथन सही ह और दसरा कथन गलत ह

Correct Answer -

First statement is wrong and second statement is correct पहला कथन गलत ह और दसरा कथन सही ह

Read the given statements and answer which of the following options isare correct

1 Sunrsquos short waves enter the earth partially heating the atmosphere

2 Heated earth surface from the sun produces broader waves which interacts and heats the atmosphere

िदए गए कथन को पढ़ और उर द िक िन म स कौन सास िवक सही ह

1 सय की छोटी तरग पी म आिशक प स वश करती ह और वायमडल को ऊत करती ह

2 सय स ऊत पी की सतह िवारत तरग उ करती ह जो परर भाव डालती ह और वायमडल कोऊत करती ह

1 Both Statements 1 and 2 are correct दोनो कथन 1 और 2 सही ह

2 Both Statements 1 and 2 are wrong दोनो कथन 1 और 2 गलत ह

3 Statement 1 is wrong and only Statement 2 is correct कथन 1 गलत ह और कवल कथन 2 सही ह

4 Only statement 1 is correct कवल कथन 1 सही ह

Correct Answer -

Both Statements 1 and 2 are correct दोनो कथन 1 और 2 सही ह

Read the given statements and answer which of the following options isare correct

(1)The rocks that get changed due to heat and pressure are termed as metamorphic rocks

(2)Slate is one such type of metamorphic rock

िदए गए कथनो को पढ़ और उर द िक िन म स कौन सास िवक सही ह

(1) शल जो ऊा और दाब क कारण परवितत हो जात ह उ कायातरक शलो क प म जाना जाता ह

(2) ट एक तरह का कायातरक शल ह

1 Both statements are wrong दोनो कथन गलत ह

2 Both statements are correct दोनो कथन सही ह

3 First statement is wrong and second statement is correct पहला कथन गलत ह और दसरा कथन सही ह

ी औ

43)

44)

4 First statement is correct and second statement is wrong पहला कथन सही ह और दसरा कथन गलत ह

Correct Answer -

Both statements are correct दोनो कथन सही ह

Read the given statements and answer which of the following options isare correct

1 Higher temperature anomaly is observed in the northern hemisphere

2 Differential heating is absent in Northern Hemisphere

िदए गए कथनो को पढ़ और उर द िक िन म स कौन सास िवक सही ह

1 उरी गोलाध म उ तापमान िवसगित पायी जाती ह

2 उरी गोलाध म अतर ऊन अनपथत होती ह

1 Both Statements 1 and 2 are correct दोनो कथन 1 और 2 सही ह

2 Both Statements 1 and 2 are wrong दोनो कथन 1 और 2 गलत ह

3 Statement 1 is wrong and Statement 2 is correct कथन 1 गलत ह और कथन 2 सही ह

4 Statement 1 is correct and Statement 2 is wrong कथन 1 सही ह और कथन 2 गलत ह

Correct Answer -

Statement 1 is correct and Statement 2 is wrong कथन 1 सही ह और कथन 2 गलत ह

Read the given statements and answer which of the following options isare correct

(1) Plutonic rocks are intrusive type of igneous rocks

(2) It cools very slowly because the surrounding rock serves as insulation around the intrusion of magma

िदए गए कथनो को पढ़ और उर द िक िन म स कौन सास िवक सही ह

(1) िवतलीय शल अतवधी कार क आश शल ह

(2) यह बत धीर-धीर ठडा होता ह ोिक आस-पास क शल मा क अतवधन क चारो ओर रोधन क प म कायकरत ह

1 Both statements are wrong दोनो कथन गलत ह

2 Both statements are correct दोनो कथन सही ह

3 First statement is wrong and second statement is correct पहला कथन गलत ह और दसरा कथन सही ह

4 First statement is correct and second statement is wrong पहला कथन सही ह और दसरा कथन गलत ह

Correct Answer -

Both statements are correct दोनो कथन सही ह

45)

46)

47)

48)

The dust and ash material hurled from the volcanoes are termed as

ालामखी स िनकलन वाली धल और राख सामी को _______ क प म कहा जाता ह

1 Pyroclasc पाइरोाक

2 Hyperclastic हाइपराक

3 Hepiroclastic हिपरोाक

4 Cirroclastic िसरोाक

Correct Answer -

Pyroclasc पाइरोाक

The vertical difference in elevation between a low tide and high tide is referred as

कम ार और उ ार क बीच ऊचाई म लबवत अतर _____ स सदिभत होता ह

1 Tidal slope ारीय ढलान

2 Tidal elevation ारीय उयन

3 Tidal range ारीय परास

4 Tidal height ारीय ऊचाई

Correct Answer -

Tidal range ारीय परास

The maximum biodiversity is found in which of the following regions िनिलखत ो म स अिधकतमजव िविवधता िकसम पायी जाती ह

1 Amazon Basin अमज़न बिसन

2 East Indies ई इडीज

3 Congo Basin कागो बिसन

4 West indies व इडीज

Correct Answer -

Amazon Basin अमज़न बिसन

The cultivation of rice crop produces_______ चावल की फसल की खती ______ का उादन करती ह

1 SO2

49)

50)

51)

2 CH4

3 CFCs

4 CO2

Correct Answer -

CH4

The pressure system with higher pressure at the centre is called__________

क म उ दबाव वाली दबाव णाली को _______ कहा जाता ह

1 front अ

2 depression अवनमन

3 cyclone चवात

4 anti-cyclone ितचवात

Correct Answer -

anti-cyclone ितचवात

The Himalayan region is poor in mineral resources because िहमालयी खिनज ससाधनो म समनही ह ोिक

1 The displacement of rock strata has disturbed the arrangement of rocks and made it complex

शलीय परत क िवथापन न चानो की वथा को अवथत कर िदया ह और इस जिटल बना िदया ह

2 The climate conditions are not suitable for exploitation of minerals

जलवाय की थित खिनजो क दोहन क िलए उपय नही ह

3 The terrain makes explanation of minerals difficult and very costly due to transportation difficulties

भ-भाग परवहन की किठनाइयो क कारण खिनजो का दोहन मल और बत महगा बना दता ह

4 It is made up of crystalline rocks यह िलीय चानो स बना ह

Correct Answer -

The displacement of rock strata has disturbed the arrangement of rocks and made it complex

शलीय परत क िवथापन न चानो की वथा को अवथत कर िदया ह और इस जिटल बना िदया ह

The process through which the moisture is added to the atmosphere by vegetation is termed as

वह िया िजसक माम स वनित ारा वातावरण म नमी िमलायी जाती ह _______ क प म जानी जाती ह

52)

53)

54)

1 Condensation सघनन

2 Evapotranspiration वान-उजन

3 Radiation िविकरण

4 Precipitation वषण

Correct Answer -

Evapotranspiration वान-उजन

The process through which the terrestrial heat is transferred to air by direct contact is termed as

वह िया िजसम सपक ारा थलीय ऊा वाय म थानातरत हो जाती ह ______ क प म जानी जाती ह

1 Conduction चालन

2 Convection सवहन

3 Insolation आतपन

4 Radiation िविकरण

Correct Answer -

Conduction चालन

The largest area under mangroves is in which of the following statesunion territory

मोव क अतगत िनिलखत राोसघ शािसत दशो म स सबस बड़ा कौन सा ह

1 Andaman and Nicobar अमान और िनकोबार

2 Andhra Pradesh आ दश

3 West Bengal पिम बगाल

4 Gujarat गजरात

Correct Answer -

West Bengal पिम बगाल

The longitudinal transverse and surface waves in an earthquake originate from

भकप म दशातर अनथ और सतह तरग यहा उ होती ह

1 The focus on the surface of the Earth पी क सतह पर क -िबद म

2 The focus within the body of the Earth पी क भीतर क -िबद म

3 The epicenter within the body of the Earth पी क भीतर उपरक म

55)

56)

57)

4 The epicenter on the surface of the Earth पी क सतह पर उपरक म

Correct Answer -

The focus within the body of the Earth पी क भीतर क -िबद म

The down slope movement of material due to gravity is called______

गाकषण क कारण पदाथ की अनढाल गित को ______ कहा जाता ह

1 mass movement पदाथ सचलन

2 deposition िनप

3 erosion रण

4 volcanic movement ालामखीय सचलन

Correct Answer -

mass movement पदाथ सचलन

Shimla is cooler than Amritsar although both are on the same latitude This is because

िशमला म अमतसर स अिधक ठड ह हालािक दोनो समान अाश पर ह ऐसा ह ोिक

1 Shimla is at a greater height above sea level than Amritsar अमतसर की तलना म िशमला सम तल स अिधकऊचाई पर ह

2 Shimla is further north िशमला उर की ओर ह

3 Shimla is farther from the equator िशमला भम रखा स आग ह

4 Their longitudes differ उनकी दशातर रखाए िभ ह

Correct Answer -

Shimla is at a greater height above sea level than Amritsar अमतसर की तलना म िशमला सम तल स अिधकऊचाई पर ह

lsquoTempo of Urbanizationrsquo measures which of the following

lsquoशहरीकरण का टपोrsquo िनिलखत म स कौन सा उपाय ह

1 Speed of urbanizaon शहरीकरण की गित

2 None of the above इनम स कोई नही

3 Inequality of urbanizaon शहरीकरण की असमानता

4 Current level of urbanizaon शहरीकरण का वतमान र

Correct Answer -

58)

59)

60)

Speed of urbanizaon शहरीकरण की गित

Out of the following options choose the INCORRECT statement

िनिलखत िवको म स गलत कथन का चयन कर

1 The clear tracts in the equatorial region recover rapidly भम रखा म भभाग तजी स ठीक हो जात ह

2 The stable communities include a redwood forest a pine forest at high elevations

थर समदायो म एक रडवड वन उ ऊचाई पर एक दवदार वन शािमल ह

3 Any ecosystem moves towards maximum biomass and stability to survive

कोई भी पारथितकी त जीिवत रहन क िलए अिधकतम जवसहित और थरता की तरफ असर होता ह

4 Tropical rain forests near equator are stable ecosystems

भम रखा क पास उकिटबधीय वषा वन थर पारथितक त ह

Correct Answer -

The clear tracts in the equatorial region recover rapidly भम रखा म भभाग तजी स ठीक हो जात ह

Seasonal contrasts are maximum in मौसमी िवषमता अिधकतम ह

1 Mid latitudes म अाश म

2 Low attitudes िन अाश म

3 High latitudes उ अाश म

4 Subtropics उपोकिटबधीय म

Correct Answer -

Mid latitudes म अाश म

In India which type of forest among the following occupies the largest area

भारत म िनिलखत म स िकस कार क वन सबस बड़ा फल आािदत करत ह

1 Sub-tropical Dry Evergreen Forest उप उकिटबधीय श सदाबहार वन

2 Mountain Wet Temperate Forest पवतीय आ शीतो वन

3 Tropical Moist Deciduous Forest उकिटबधीय आ पणपाती वन

4 Tropical Wet Evergreen Forest उकिटबधीय आ सदाबहार वन

Correct Answer -

Tropical Moist Deciduous Forest उकिटबधीय आ पणपाती वन

61)

62)

63)

64)

What is the proportion of lsquoJuvenile Populationrsquo (0-14 years) in India as per 2011Census

2011 की जनगणना क अनसार भारत म जवनाइल पॉपलशन यानी िकशोर जनस या (0-14 वष) का अनपात ाह

1 3076 of total population कल जनस या का 3076

2 2764 of total population कल जनस या का 2764

3 2933 of total population कल जनस या का 2933

4 3354 of total population कल जनस या का 3354

Correct Answer -

3076 of total population कल जनस या का 3076

What is the Belfast famous for बलफा िकसक िलए मशर ह

1 Belt of cotton textile industry कपास व उोग क

2 Ship-building industry जहाज िनमाण उोग

3 Agricultural machinery किष उपकरण

4 Aero planes manufacturing वाययान िनमाण

Correct Answer -

Ship-building industry जहाज िनमाण उोग

What is the most important occupation in tropical monsoon lands

उकिटबधीय मॉनसन भिम म सबस महपण वसाय ा ह

1 Mining खनन

2 Cattle rearing मवशी पालन

3 Agriculture किष

4 Nomadic herding नोमािडक जड़ी-बिटया

Correct Answer -

Agriculture किष

What is the most important characteristics of the islands (Indian) located in the Arabian sea

अरब सागर म थत ीपो (भारतीय) की सबस महपण िवशषता ा ह

ी ो

65)

66)

67)

1 There are all of coral origins सभी कोरल मल क ह

2 There are all very small in size य सभी आकार म बत छोट ह

3 They have a very dry climate इनकी जलवाय बत श ह

4 They are extended parts of the mainland व महाीप क िवारत िह ह

Correct Answer -

There are all of coral origins सभी कोरल मल क ह

What do the basalt layers of the Deccan indicate डन की बसा परत ा इिगत करती ह

1 All of the above उपरो सभी

2 Huge volcanic eruptions in the distant past दरथ अतीत म िवशाल ालामखीय िवोट

3 The immense erosional activity of the rivers निदयो की िवशाल रण गितिविध

4 The influence of weathering मौसम का भाव

Correct Answer -

Huge volcanic eruptions in the distant past दरथ अतीत म िवशाल ालामखीय िवोट

In the structure of planet Earth below the mantle the core is mainly made up of_____

पी ह की सरचना म मटल क नीच कोर म प स______ स िनिमत होती ह

1 aluminium एमीिनयम

2 silicon िसिलकॉन

3 chromium ोिमयम

4 iron लोहा

Correct Answer -

iron लोहा

One of the major Mid Oceanic Ridge is found in मख म-महासागर चोिटयो म स एक ______ म पायाजाता ह

1 Mid Pacific Ocean म शात महासागर

2 Mid Atlantic Ocean म अटलािटक महासागर

3 Mid Indian Ocean म भारतीय महासागर

4 Mid Arctic Ocean म आक िटक महासागर

68)

69)

70)

71)

Correct Answer -

Mid Atlantic Ocean म अटलािटक महासागर

Magma that reaches the Earthrsquos surface and then solidifies is called________

मा जो पी की सतह तक पचती ह और िफर ठोस हो जाती ह ________कहलाती ह

1 quartz ाटज

2 lava लावा

3 granite नाइट

4 silicates िसिलकट

Correct Answer -

lava लावा

Isotherms are the lines of equal_______ समताप रखाए समान _______की रखाए होती ह

1 pressure दाब

2 temperature तापमान

3 rainfall वषा

4 height ऊचाई

Correct Answer -

temperature तापमान

Mark the correct sequence of passes in the Western Ghats from north to south

पिमी घाटो म उर स दिण तक दर क सही अनम को िचित कर

1 Thalghat Palghat Bhorghat थलगघाट पालघाट भोरघाट

2 Thalghat Bhorghat Palghat थलघाट भोरघाट पालघाट

3 Bhorghat Thalghat Palghat भोरघाट थलघाट पालघाट

4 Palghat Bhorghat Thalghat पालघाट भोरघाट थलघाट

Correct Answer -

Thalghat Bhorghat Palghat थलघाट भोरघाट पालघाट

Which of the following does not have influence over the climate in India

ि ि ि ी ी

72)

73)

िनिलखत म स िकसका भाव भारत की जलवाय पर नही पड़ता ह

1 Ocean currents सागर की लहर

2 Nearness to equator भम रखा स िनकटता

3 Monsoons मानसन

4 Presence of Indian ocean भारतीय महासागर की उपथित

Correct Answer -

Ocean currents सागर की लहर

Which of the following cloud types has the characteristics like vertical tall narrow and puffy

िनिलखत म स िकस कार क मघो म लबवत लबी सकीण और थलता जसी िवशषताए ह

1 Cumulonimbus तफानी मघ

2 Cumulus मघ पज

3 Cirrocumulus पाभ कपासी मघ

4 Nimbostratus वषारी मघ

Correct Answer -

Cumulus मघ पज

Which of the following statement is INCORRECT about Crude Birth Rate

िनिलखत स कौन सा कथन अशोिधत ज दर क बार म सही नही ह

1 It cannot be used for comparing fertility level between two countries with different population characteristics

इसका उपयोग िविभ जनसा िवशषताओ वाल दो दशो क बीच जनन र की तलना क िलए नही िकया जा सकता ह

2 It is a standardized measure of fertility

यह जनन मता का मानकीकत उपाय ह

3 It is effected by the age-sex composition of the population

यह आबादी की आय-िलग सरचना स भािवत होता ह

4 It is expressed per 1000 population in a given geographical unit

यह िकसी दी गई भौगोिलक इकाई म ित 1000 जनसा पर िकया जाता ह

Correct Answer -

It is a standardized measure of fertility

यह जनन मता का मानकीकत उपाय ह

74)

75)

76)

77)

Which of the following state in India experienced negative decadal growth rate during 2001 to 2011census

भारत म िनिलखत म स िकस रा म वष 2001 स 2011 की जनगणना क दौरान नकाराक िगरावट दर ई

1 Tripura िपरा

2 Nagaland नागालड

3 Haryana हरयाणा

4 Odisha ओिडसा

Correct Answer -

Nagaland नागालड

Which of the following is NOT a characteristic of peninsular rivers

िनिलखत म स कौन सी िवशषता ायीपीय निदयो म नही होती ह

1 Flow through shallow valleys उथल घािटयो क माम स वाह

2 Seasonal flow मौसमी वाह

3 Little erosional activity थोड़ी कटावदार गितिविध

4 Meandering tendency often shifting their beds घमावदार वि अर अपन तटो को थानातरत करना

Correct Answer -

Meandering tendency often shifting their beds घमावदार वि अर अपन तटो को थानातरत करना

Which of the following gases in the atmosphere absorbs heat from the Sunrsquos radiation and the Earthssurface

वायमडल म िनिलखत म स कौन सी गस सय क िविकरण और पी की सतह स ऊा को अवशोिषत करती ह

1 Neon िनयॉन

2 Carbon dioxide काबन डाइऑाइड

3 Argon आगन

4 Nitrogen नाइट ोजन

Correct Answer -

Carbon dioxide काबन डाइऑाइड

Which of the following kind of settlement pattern is found at the confluence of rivers

ि ि ि ि ो

78)

79)

80)

िनिलखत म स िकस कार का वथापन पटन निदयो क सगम पर पाया जाता ह

1 Triangular Paern िकोणीय पटन

2 Circular or Semi-Circular Paern परप या अध-परप पटन

3 Nebular Paern नबलर पटन

4 Star ndashShaped Paern ार-आकार का पटन

Correct Answer -

Triangular Paern िकोणीय पटन

Which one was not the objective of the Biosphere Reserve Projects launched by the UNESCO

यनो ारा श की गई सरित जवमडल परयोजनाओ का उ इनम स कौन सा नही था

1 To promote teaching and research िशण और अनसधान को बढ़ावा दना

2 To make agriculture sustainable किष को दीघकािलक बनाना

3 To conserve ecosystems पारथितक त को सरित करना

4 To conserve genetic diversity for a longtime लब समय तक अनवािशक िविवधता को सरित करना

Correct Answer -

To make agriculture sustainable किष को दीघकािलक बनाना

Which region of the Earth surface receives the highest amount of insulation

पी सतह का कौन सा तापावरोधन की उतम माा ा करता ह

1 Land mass थलखड

2 Savannah region सवाना

3 Water bodies जल िनकाय

4 Tropical desert उकिटबधीय रिगान

Correct Answer -

Tropical desert उकिटबधीय रिगान

Which one of the following is not a biodiversity hotspot

िनिलखत म स कौन सा जव िविवधता का म जगह नही ह

1 Eastern Himalaya पव िहमालय

2 Eastern Ghats पव घाट

81)

82)

83)

3 Indo-Myanmar भारत-ामार

4 Westerm Ghats पिमी घाट

Correct Answer -

Eastern Ghats पव घाट

Which one of the following is NOT a part of the World Network of Biosphere Reserves based on theUNESCO Man and Biosphere Programme

यनो मन और बायोीयर कायम क आधार पर िनिलखत म स कौन बायोीयर रजव क िव नटवक कािहा नही ह

1 Gulf of Mannar मार की खाड़ी

2 Seshachalam शषाचलम

3 Sunderban सदरबन

4 Nilgiri नीलिगर

Correct Answer -

Seshachalam शषाचलम

Which one of the following is an example of ldquodesert vegetationrdquo

िनिलखत म स कौन मथलीय वनित का एक उदाहरण ह

1 Mosses and lichens दलदल और शवाल

2 Temperate grassland समशीतो घास क मदान

3 Coniferous forest शकधारी वन

4 Acacia and cactus एकािसया और कस

Correct Answer -

Acacia and cactus एकािसया और कस

Which one of the following reflects more sunlight िनिलखत म स कौन सा सय की रोशनी को अिधकपरावितत करता ह

1 Paddy crop land धान फसल भिम

2 Land covered with fresh snow ताजा बफ स आािदत भिम

3 Sand desert रतीली रिगान

4 Prairie land यरी भिम

84)

85)

86)

87)

Correct Answer -

Land covered with fresh snow ताजा बफ स आािदत भिम

Which layer of the atmosphere is in contact with the surface of the earthrsquos oceans

वायमडल की कौन सी परत पी क महासागरो की सतह क सपक म ह

1 Stratosphere समताप मडल

2 Mesosphere म मडल

3 Hydrosphere जलमडल

4 Troposphere ोभ मडल

Correct Answer -

Troposphere ोभ मडल

Mediterranean Sea is a border of which of the following countries भम सागर िनिलखत दशो म सिकसकी सीमा ह

1 None of these इनम स कोई नही

2 Iraq इराक

3 Lebanon लबनान

4 Jordan जॉडन

Correct Answer -

Lebanon लबनान

Benguela ocean currents are found along which coast बगएला महासागर धाराए िकस तट क साथ पायीजाती ह

1 East Coast of South America दिण अमरका क पव तट

2 East Coast of Africa अीका क पव तट

3 West Coast of South America दिण अमरका क पिमी तट

4 West Coast of Africa अीका क पिमी तट

Correct Answer -

West Coast of Africa अीका क पिमी तट

88)

89)

90)

Due to tension a block of land on one side being pushed up or upthrown relative to the downthrown blockis referred as

तनाव क कारण नीच फ क ए खड क साप भिम का एक खड एक ओर स ऊपर धकला जाता ह या ऊपर की ओरफ का जाता ह यह _____ क प म सदिभत ह

1 Thrust fault प श

2 Normal fault सामा श

3 Reverse fault म श

4 Strike slip fault नितलब सपण श

Correct Answer -

Normal fault सामा श

Inter-tropical doldrums is a zone of ______ अतर-उकिटबधीय डोलड ______ का एक ह

1 Frontolysis टोलायिसस

2 Convergence अिभसरण

3 Inter-tropical divergence zone अतर-उकिटबधीय िवचलन

4 Local wind थानीय वाय

Correct Answer -

Convergence अिभसरण

The Horse Latitudes are regions located at about _____ north and south of the equator

हॉस अाश भम रखा क उर और दिण म लगभग _____ पर थत ह

1 30ndash60 degree Latitude 30-60 िडी अाश

2 0ndash5 degree Latitude 0-5 िडी अाश

3 30 degree Latitude 30 िडी अाश

4 60ndash90 degree Latitude 60-90 िडी अाश

Correct Answer -

30 degree Latitude 30 िडी अाश

Generally evaporation is high over which part of the Earth

आम तौर पर पी क िकस भाग पर वाीकरण अिधक होता ह

1 Equatorial maritime भमवत समीय ी ी

91)

92)

2 Equatorial continental भमवत महाीपीय

3 Polar maritime वीय समीय

4 Polar continental वीय महाीपीय

Correct Answer -

Equatorial maritime भमवत समीय

A very high temperature during summer in north western India leads to what type of climaticcondition in south

उर पिमी भारत म गम क दौरान बत अिधक तापमान होन क कारण दिण म िकस कार की जलवाय थितउ करता ह

1 Depression over arabian sea अरब सागर पर अवनमन

2 Failure monsoon मानसन िवफलता

3 Successful monsoon मानसन सफलता

4 Cyclones चवात

Correct Answer -

Successful monsoon मानसन सफलता

Lightning and thunder are the resultant effect when तिड़त और गजन परणामी भाव ह जब

1 Two massive clouds hit powerfully each other first lightning is produced and later sound is produced

दो बड़ बादल एक दसर स शशाली ढग स टकरात ह पहल आकाशीय िवदयत उ होता ह और बाद म िन उहोती ह

2 Two massive clouds come into contact with the powerful wind collision this results into first sound and thenlightning

दो बड़ बादल शशाली पवन सघ क सपक म आत ह इसका परणामप पहल िन और िफर आकाशीय िवदयतउ होता ह

3 None of the above उपरो म स कोई भी नही

4 A high density cloud contains positively and negatively charged electric ions and when this interacts light andsound are simultaneously produced

एक उ घन बादल म धनाक और ऋणाक आविशत िवदयत आयन होत ह और जब यह परर भाव डालत ह तोकाश और िन एक साथ उािदत होती ह

Correct Answer -

A high density cloud contains positively and negatively charged electric ions and when this interacts light andsound are simultaneously produced

औ ि ि ो औ ो

93)

94)

95)

एक उ घन बादल म धनाक और ऋणाक आविशत िवदयत आयन होत ह और जब यह परर भाव डालत ह तोकाश और िन एक साथ उािदत होती ह

Doon Valley is able to grow rice because दन घाटी चावल उगान म सम ह ोिक

1 Other crops cannot be grown वहा अ फसलो को उगाया नही जा सकता ह

2 People in the valley are rice eaters घाटी म लोग चावल खान वाल ह

3 There is a huge export demand of rice वहा चावल की भारी िनयात माग ह

4 It has warm summer and snow melt waters for irrigation

वहा गिमया गम होती ह िसचाई क िलए बफ का िपघला आ पानी होता ह

Correct Answer -

It has warm summer and snow melt waters for irrigation

वहा गिमया गम होती ह िसचाई क िलए बफ का िपघला आ पानी होता ह

CANCELLED

In the geological time scale the Mesozoic Era DOES NOT contains which of the following periods

भगभय समय पमान पर मजीवी यग म िन कालो म स कौन नही ह

1 Triassic ट ाइऐिसक

2 Jurassic जरिसक

3 Cretaceous चाकमय

4 Carboniferous काबनी

Correct Answer -

Carboniferous काबनी

96)

1 P-3 Q-4 R-2 S-1

2 P-3 Q-4 R-1 S-2

3 P-3 Q-4 R-1 S-2

4 P-4 Q-3 R-2 S-1

Correct Answer -

P-4 Q-3 R-2 S-1

1 P-3 Q-1 R-4 S-2

2 P-3 Q-4 R-1 S-2

3 P-3 Q-2 R-4 S-1

97)

98)

4 P-2 Q-1 R-4 S-3

Correct Answer -

P-3 Q-4 R-1 S-2

1 P-3 Q-1 R-4 S-2

2 P-2 Q-3 R-4 S-1

3 P-2 Q-1 R-3 S-4

4 P-4 Q-2 R-1 S-3

Correct Answer -

P-3 Q-1 R-4 S-2

99)

100)

1 P-3 Q-2 R-4 S-1

2 P-1 Q-2 R-3 S-4

3 P-2 Q-3 R-1 S-4

4 P-4 Q-3 R-2 S-1

Correct Answer -

P-2 Q-3 R-1 S-4

ldquoHuman geography is the study of changing relationship between the unresting man and the unstableearthrdquo was defined by

lsquolsquoमानव भगोल ाकल आदमी और अथर पी क बीच सबध परवतन का अयन हrdquo ______ ारा परभािषत िकया गयाथा

1 J Brunches ज चस

2 EC Semple ईसी सल

3 HJ Mackinder एच ज मिकदर

4 PV Blache पीवी च

Correct Answer -

EC Semple ईसी सल

Sedimentary rocks are finally and ultimately derived from the____________

अवसादी चान अततः ________ स ा की जाती ह

1 action of earth movements पी की गितिविधयो

2 marine deposit समी िनप

3 weathering of metamorphic rocks पातरत चानो क अपय

4 weathering of igneous rocks आय चानो क अपय

Correct Answer -

weathering of igneous rocks आय चानो क अपय

Page 25: High School Teacher Eligibility Test- BOARD PROFESSIONAL ...peb.mp.gov.in/results/RESULT_18/HST_RES18/Final_anwser_key/HST… · M a ndl a / मंड ल ... Under the Madhya Pradesh

25)

26)

27)

4 Sri Lanka ीलका

Correct Answer -

Sri Lanka ीलका

Which among the following state is the major producer of Bauxite in India

िनिलखत म स कौन सा रा भारत म बॉाइट का मख उादक ह

1 Madhya Pradesh मदश

2 Rajasthan राजथान

3 Goa गोवा

4 Orissa उड़ीसा

Correct Answer -

Orissa उड़ीसा

Which of the following states DOES NOT share border with Chhattisgarh

िनिलखत म स कौन सा रा छीसगढ़ क साथ सीमा साझा नही करता ह

1 Telangana तलगाना

2 Uttar Pradesh उर दश

3 Bihar िबहार

4 Andhra Pradesh आ दश

Correct Answer -

Bihar िबहार

Which of the following statements is INCORRECT with respect to parallels of latitudes

अाश क समानातरो क सबध म िन निलखत म स कौन सा कथन गलत ह

1 A line joining places of equal latitude is known as parallel of largest

समान अाश क थानो को जोड़न वाली रखा को िवशालतम क समानातर क प म जाना जाता ह

2 They stat from equator and run parallels to it

व भम रखा स ारभ होत ह और इसक समानातर चलत ह

3 All parallels are equal in length सभी समातर लबाई म समान ह

4 All parallels are drawn as circles on the globe ोब पर सभी समानातर वो क प म खीच जात ह

28)

29)

30)

31)

Correct Answer -

All parallels are equal in length सभी समातर लबाई म समान ह

Which of the following Indian states is also known as a lsquoLand of Red river and Blue Hillsrsquo

िनिलखत म स िकस भारतीय रा को लाल नदी और नीली पहािड़यो की भिम क नाम स जाना जाता ह

1 Uttarkhand उराखड

2 Assam असम

3 Meghalaya मघालय

4 Arunachal Pradesh अणाचल दश

Correct Answer -

Assam असम

In spatial analysis of settlement Rn = 215 indicates which type of settlement arrangement

िनपटान क थािनक िवषण म Rn = 215 यह इिगत करता ह िक िकस कार की िनपटान वथा ह

1 Uniform यिनफॉम

2 Semi-Clustered समी- ल टर

3 Clustered ल टर

4 Random रडम

Correct Answer -

Uniform यिनफॉम

Who are known as the lsquoYellow Peoplersquo lsquoयलो पीपलrsquo क प म कौन जाना जाता ह

1 Mongoloids मोगोलोइडस

2 Nigroids नीोइडस

3 Australoids ऑ लॉइडस

4 Caucasoids कॉकसोइडस

Correct Answer -

Mongoloids मोगोलोइडस

ि ि ो ौ ि

32)

33)

34)

Who publishes the topographical map of India भारत क थलाकितक मानिच को कौन कािशत करता ह

1 Geographical Survey of India भारत का भौगोिलक सवण

2 Government of India भारत सरकार

3 Geological Survey of India भारत क भगभय सवण

4 Survey of India भारत का सवण

Correct Answer -

Survey of India भारत का सवण

Who among the following claimed geography to be the lsquoEcology of Manrsquo

िनिलखत म स िकसन भगोल को मन का पारथितकी कहा ह

1 Alfred Hener अड हटनर

2 Vidal-de la Blache वाइडल-िड लॉ ॉश

3 Oo Schluter ओटो टर

4 Harlan Barrow हरलन बारो

Correct Answer -

Harlan Barrow हरलन बारो

Who among the following is regarded as the founder of humanistic approach in geography

िनिलखत म स िकस भगोल म मानवतावादी िकोण का सथापक माना जाता ह

1 William Bunge िविलयम बग

2 Yi-Fu-Tuan यी-फ- यान

3 Brain JL Berry न जएल बरी

4 Richard Peet रचड पीट

Correct Answer -

Yi-Fu-Tuan यी-फ- यान

Who prepared Lorenz curve लोरज व िकसन तयार िकया

1 Geddes गडस

2 None of these इनम स कोई नही

3 Griffith Taylor ििफथ टलर

35)

36)

37)

4 Max U Lorenz म य लोरज

Correct Answer -

Max U Lorenz म य लोरज

Gulf Streams are the currents of which of the following oceans

खाड़ी की धाराए िनिलखत महासागरो म स िकसकी धाराए ह

1 North Atlantic Ocean उरी अटलािटक महासागर

2 North Pacific Ocean उरी शात महासागर

3 Arabian Sea अरब सागर

4 South Pacific Ocean दिण शात महासागर

Correct Answer -

North Atlantic Ocean उरी अटलािटक महासागर

Disintegration wearing away and removal of rock material is generally referred as

िशला पदाथ (रॉक सामी) का टटना िमटना और हटना आमतौर पर ________ क प म सदिभत िकया जाता ह

1 Shattering िवसकारक

2 Denudation अनाादन

3 Fault श

4 Decomposition िवयोजन

Correct Answer -

Denudation अनाादन

Variations in the length of day time and night from season to season are due to

मौसम स मौसम परवतन पर िदन क समय और रात क समय की अविध म िभताए िन कारण स होती ह

1 The Earthrsquos revolution round the Sun in an elliptical manner पी का दीघवाकार तरीक स सय क चारो घणन

2 The Earthrsquos rotation on its axis पी का इसकी धरी पर घणन

3 Revolution of the Earth on a tilted axis नत अ पर पी का घणन

4 Latitudinal position of the place थान की अाश थित

Correct Answer -

Revolution of the Earth on a tilted axis नत अ पर पी का घणन

38)

39)

40)

Point out the correct sequence of mountain ranges from north to south

उर स दिण तक पवत खलाओ क सही अनम को इिगत कर

1 Great Himalaya Middle Himalaya Outer Himalaya Trans Himalaya

महान िहमालय म िहमालय बा िहमालय परा िहमालय

2 Middle Himalaya Great Himalaya Trans Himalaya Outer Himalaya

म िहमालय महान िहमालय परा िहमालय बा िहमालय

3 Outer Himalaya Middle Himalaya Great Himalaya Trans Himalaya

बा िहमालय म िहमालय महान िहमालय परा िहमालय

4 Trans Himalaya Great Himalaya Middle Himalaya Outer Himalaya

परा िहमालय महान िहमालय म िहमालय बा िहमालय

Correct Answer -

Trans Himalaya Great Himalaya Middle Himalaya Outer Himalaya

परा िहमालय महान िहमालय म िहमालय बा िहमालय

Sunrsquos halo is produced by the refraction of light in सय का भामडल ______ म काश क अपवतन ाराउ होता ह

1 Ice crystals in Cirrro-Cumulus clouds पाभ-कपास मघो क बफ िल

2 Ice crystal in Cirrus clouds पाभमघो क बफ िल

3 Dust particles in Stratus clouds री मघो क धल कण

4 Water vapour in Stratus clouds री मघो क जल वा

Correct Answer -

Ice crystal in Cirrus clouds पाभमघो क बफ िल

Read the given statements and answer which of the following options isare correct

(1) The minerals present in the rocks exposed to atmosphere are not subjected to alteration

(2) Oxidation is one of the processes of chemical weathering

िदए गए कथनो को पढ़ और उर द िक िन म स कौन सास िवक सही ह

(1) वायमल स अनावत शल म उपथत खिनज परवतन क अधीन नही होता ह

(2) ऑीकरण रासायिनक अपय की ियाओ म स एक ह

ो ो

41)

42)

1 Both statements are wrong दोनो कथन गलत ह

2 Both statements are correct दोनो कथन सही ह

3 First statement is wrong and second statement is correct पहला कथन गलत ह और दसरा कथन सही ह

4 First statement is correct and second statement is wrong पहला कथन सही ह और दसरा कथन गलत ह

Correct Answer -

First statement is wrong and second statement is correct पहला कथन गलत ह और दसरा कथन सही ह

Read the given statements and answer which of the following options isare correct

1 Sunrsquos short waves enter the earth partially heating the atmosphere

2 Heated earth surface from the sun produces broader waves which interacts and heats the atmosphere

िदए गए कथन को पढ़ और उर द िक िन म स कौन सास िवक सही ह

1 सय की छोटी तरग पी म आिशक प स वश करती ह और वायमडल को ऊत करती ह

2 सय स ऊत पी की सतह िवारत तरग उ करती ह जो परर भाव डालती ह और वायमडल कोऊत करती ह

1 Both Statements 1 and 2 are correct दोनो कथन 1 और 2 सही ह

2 Both Statements 1 and 2 are wrong दोनो कथन 1 और 2 गलत ह

3 Statement 1 is wrong and only Statement 2 is correct कथन 1 गलत ह और कवल कथन 2 सही ह

4 Only statement 1 is correct कवल कथन 1 सही ह

Correct Answer -

Both Statements 1 and 2 are correct दोनो कथन 1 और 2 सही ह

Read the given statements and answer which of the following options isare correct

(1)The rocks that get changed due to heat and pressure are termed as metamorphic rocks

(2)Slate is one such type of metamorphic rock

िदए गए कथनो को पढ़ और उर द िक िन म स कौन सास िवक सही ह

(1) शल जो ऊा और दाब क कारण परवितत हो जात ह उ कायातरक शलो क प म जाना जाता ह

(2) ट एक तरह का कायातरक शल ह

1 Both statements are wrong दोनो कथन गलत ह

2 Both statements are correct दोनो कथन सही ह

3 First statement is wrong and second statement is correct पहला कथन गलत ह और दसरा कथन सही ह

ी औ

43)

44)

4 First statement is correct and second statement is wrong पहला कथन सही ह और दसरा कथन गलत ह

Correct Answer -

Both statements are correct दोनो कथन सही ह

Read the given statements and answer which of the following options isare correct

1 Higher temperature anomaly is observed in the northern hemisphere

2 Differential heating is absent in Northern Hemisphere

िदए गए कथनो को पढ़ और उर द िक िन म स कौन सास िवक सही ह

1 उरी गोलाध म उ तापमान िवसगित पायी जाती ह

2 उरी गोलाध म अतर ऊन अनपथत होती ह

1 Both Statements 1 and 2 are correct दोनो कथन 1 और 2 सही ह

2 Both Statements 1 and 2 are wrong दोनो कथन 1 और 2 गलत ह

3 Statement 1 is wrong and Statement 2 is correct कथन 1 गलत ह और कथन 2 सही ह

4 Statement 1 is correct and Statement 2 is wrong कथन 1 सही ह और कथन 2 गलत ह

Correct Answer -

Statement 1 is correct and Statement 2 is wrong कथन 1 सही ह और कथन 2 गलत ह

Read the given statements and answer which of the following options isare correct

(1) Plutonic rocks are intrusive type of igneous rocks

(2) It cools very slowly because the surrounding rock serves as insulation around the intrusion of magma

िदए गए कथनो को पढ़ और उर द िक िन म स कौन सास िवक सही ह

(1) िवतलीय शल अतवधी कार क आश शल ह

(2) यह बत धीर-धीर ठडा होता ह ोिक आस-पास क शल मा क अतवधन क चारो ओर रोधन क प म कायकरत ह

1 Both statements are wrong दोनो कथन गलत ह

2 Both statements are correct दोनो कथन सही ह

3 First statement is wrong and second statement is correct पहला कथन गलत ह और दसरा कथन सही ह

4 First statement is correct and second statement is wrong पहला कथन सही ह और दसरा कथन गलत ह

Correct Answer -

Both statements are correct दोनो कथन सही ह

45)

46)

47)

48)

The dust and ash material hurled from the volcanoes are termed as

ालामखी स िनकलन वाली धल और राख सामी को _______ क प म कहा जाता ह

1 Pyroclasc पाइरोाक

2 Hyperclastic हाइपराक

3 Hepiroclastic हिपरोाक

4 Cirroclastic िसरोाक

Correct Answer -

Pyroclasc पाइरोाक

The vertical difference in elevation between a low tide and high tide is referred as

कम ार और उ ार क बीच ऊचाई म लबवत अतर _____ स सदिभत होता ह

1 Tidal slope ारीय ढलान

2 Tidal elevation ारीय उयन

3 Tidal range ारीय परास

4 Tidal height ारीय ऊचाई

Correct Answer -

Tidal range ारीय परास

The maximum biodiversity is found in which of the following regions िनिलखत ो म स अिधकतमजव िविवधता िकसम पायी जाती ह

1 Amazon Basin अमज़न बिसन

2 East Indies ई इडीज

3 Congo Basin कागो बिसन

4 West indies व इडीज

Correct Answer -

Amazon Basin अमज़न बिसन

The cultivation of rice crop produces_______ चावल की फसल की खती ______ का उादन करती ह

1 SO2

49)

50)

51)

2 CH4

3 CFCs

4 CO2

Correct Answer -

CH4

The pressure system with higher pressure at the centre is called__________

क म उ दबाव वाली दबाव णाली को _______ कहा जाता ह

1 front अ

2 depression अवनमन

3 cyclone चवात

4 anti-cyclone ितचवात

Correct Answer -

anti-cyclone ितचवात

The Himalayan region is poor in mineral resources because िहमालयी खिनज ससाधनो म समनही ह ोिक

1 The displacement of rock strata has disturbed the arrangement of rocks and made it complex

शलीय परत क िवथापन न चानो की वथा को अवथत कर िदया ह और इस जिटल बना िदया ह

2 The climate conditions are not suitable for exploitation of minerals

जलवाय की थित खिनजो क दोहन क िलए उपय नही ह

3 The terrain makes explanation of minerals difficult and very costly due to transportation difficulties

भ-भाग परवहन की किठनाइयो क कारण खिनजो का दोहन मल और बत महगा बना दता ह

4 It is made up of crystalline rocks यह िलीय चानो स बना ह

Correct Answer -

The displacement of rock strata has disturbed the arrangement of rocks and made it complex

शलीय परत क िवथापन न चानो की वथा को अवथत कर िदया ह और इस जिटल बना िदया ह

The process through which the moisture is added to the atmosphere by vegetation is termed as

वह िया िजसक माम स वनित ारा वातावरण म नमी िमलायी जाती ह _______ क प म जानी जाती ह

52)

53)

54)

1 Condensation सघनन

2 Evapotranspiration वान-उजन

3 Radiation िविकरण

4 Precipitation वषण

Correct Answer -

Evapotranspiration वान-उजन

The process through which the terrestrial heat is transferred to air by direct contact is termed as

वह िया िजसम सपक ारा थलीय ऊा वाय म थानातरत हो जाती ह ______ क प म जानी जाती ह

1 Conduction चालन

2 Convection सवहन

3 Insolation आतपन

4 Radiation िविकरण

Correct Answer -

Conduction चालन

The largest area under mangroves is in which of the following statesunion territory

मोव क अतगत िनिलखत राोसघ शािसत दशो म स सबस बड़ा कौन सा ह

1 Andaman and Nicobar अमान और िनकोबार

2 Andhra Pradesh आ दश

3 West Bengal पिम बगाल

4 Gujarat गजरात

Correct Answer -

West Bengal पिम बगाल

The longitudinal transverse and surface waves in an earthquake originate from

भकप म दशातर अनथ और सतह तरग यहा उ होती ह

1 The focus on the surface of the Earth पी क सतह पर क -िबद म

2 The focus within the body of the Earth पी क भीतर क -िबद म

3 The epicenter within the body of the Earth पी क भीतर उपरक म

55)

56)

57)

4 The epicenter on the surface of the Earth पी क सतह पर उपरक म

Correct Answer -

The focus within the body of the Earth पी क भीतर क -िबद म

The down slope movement of material due to gravity is called______

गाकषण क कारण पदाथ की अनढाल गित को ______ कहा जाता ह

1 mass movement पदाथ सचलन

2 deposition िनप

3 erosion रण

4 volcanic movement ालामखीय सचलन

Correct Answer -

mass movement पदाथ सचलन

Shimla is cooler than Amritsar although both are on the same latitude This is because

िशमला म अमतसर स अिधक ठड ह हालािक दोनो समान अाश पर ह ऐसा ह ोिक

1 Shimla is at a greater height above sea level than Amritsar अमतसर की तलना म िशमला सम तल स अिधकऊचाई पर ह

2 Shimla is further north िशमला उर की ओर ह

3 Shimla is farther from the equator िशमला भम रखा स आग ह

4 Their longitudes differ उनकी दशातर रखाए िभ ह

Correct Answer -

Shimla is at a greater height above sea level than Amritsar अमतसर की तलना म िशमला सम तल स अिधकऊचाई पर ह

lsquoTempo of Urbanizationrsquo measures which of the following

lsquoशहरीकरण का टपोrsquo िनिलखत म स कौन सा उपाय ह

1 Speed of urbanizaon शहरीकरण की गित

2 None of the above इनम स कोई नही

3 Inequality of urbanizaon शहरीकरण की असमानता

4 Current level of urbanizaon शहरीकरण का वतमान र

Correct Answer -

58)

59)

60)

Speed of urbanizaon शहरीकरण की गित

Out of the following options choose the INCORRECT statement

िनिलखत िवको म स गलत कथन का चयन कर

1 The clear tracts in the equatorial region recover rapidly भम रखा म भभाग तजी स ठीक हो जात ह

2 The stable communities include a redwood forest a pine forest at high elevations

थर समदायो म एक रडवड वन उ ऊचाई पर एक दवदार वन शािमल ह

3 Any ecosystem moves towards maximum biomass and stability to survive

कोई भी पारथितकी त जीिवत रहन क िलए अिधकतम जवसहित और थरता की तरफ असर होता ह

4 Tropical rain forests near equator are stable ecosystems

भम रखा क पास उकिटबधीय वषा वन थर पारथितक त ह

Correct Answer -

The clear tracts in the equatorial region recover rapidly भम रखा म भभाग तजी स ठीक हो जात ह

Seasonal contrasts are maximum in मौसमी िवषमता अिधकतम ह

1 Mid latitudes म अाश म

2 Low attitudes िन अाश म

3 High latitudes उ अाश म

4 Subtropics उपोकिटबधीय म

Correct Answer -

Mid latitudes म अाश म

In India which type of forest among the following occupies the largest area

भारत म िनिलखत म स िकस कार क वन सबस बड़ा फल आािदत करत ह

1 Sub-tropical Dry Evergreen Forest उप उकिटबधीय श सदाबहार वन

2 Mountain Wet Temperate Forest पवतीय आ शीतो वन

3 Tropical Moist Deciduous Forest उकिटबधीय आ पणपाती वन

4 Tropical Wet Evergreen Forest उकिटबधीय आ सदाबहार वन

Correct Answer -

Tropical Moist Deciduous Forest उकिटबधीय आ पणपाती वन

61)

62)

63)

64)

What is the proportion of lsquoJuvenile Populationrsquo (0-14 years) in India as per 2011Census

2011 की जनगणना क अनसार भारत म जवनाइल पॉपलशन यानी िकशोर जनस या (0-14 वष) का अनपात ाह

1 3076 of total population कल जनस या का 3076

2 2764 of total population कल जनस या का 2764

3 2933 of total population कल जनस या का 2933

4 3354 of total population कल जनस या का 3354

Correct Answer -

3076 of total population कल जनस या का 3076

What is the Belfast famous for बलफा िकसक िलए मशर ह

1 Belt of cotton textile industry कपास व उोग क

2 Ship-building industry जहाज िनमाण उोग

3 Agricultural machinery किष उपकरण

4 Aero planes manufacturing वाययान िनमाण

Correct Answer -

Ship-building industry जहाज िनमाण उोग

What is the most important occupation in tropical monsoon lands

उकिटबधीय मॉनसन भिम म सबस महपण वसाय ा ह

1 Mining खनन

2 Cattle rearing मवशी पालन

3 Agriculture किष

4 Nomadic herding नोमािडक जड़ी-बिटया

Correct Answer -

Agriculture किष

What is the most important characteristics of the islands (Indian) located in the Arabian sea

अरब सागर म थत ीपो (भारतीय) की सबस महपण िवशषता ा ह

ी ो

65)

66)

67)

1 There are all of coral origins सभी कोरल मल क ह

2 There are all very small in size य सभी आकार म बत छोट ह

3 They have a very dry climate इनकी जलवाय बत श ह

4 They are extended parts of the mainland व महाीप क िवारत िह ह

Correct Answer -

There are all of coral origins सभी कोरल मल क ह

What do the basalt layers of the Deccan indicate डन की बसा परत ा इिगत करती ह

1 All of the above उपरो सभी

2 Huge volcanic eruptions in the distant past दरथ अतीत म िवशाल ालामखीय िवोट

3 The immense erosional activity of the rivers निदयो की िवशाल रण गितिविध

4 The influence of weathering मौसम का भाव

Correct Answer -

Huge volcanic eruptions in the distant past दरथ अतीत म िवशाल ालामखीय िवोट

In the structure of planet Earth below the mantle the core is mainly made up of_____

पी ह की सरचना म मटल क नीच कोर म प स______ स िनिमत होती ह

1 aluminium एमीिनयम

2 silicon िसिलकॉन

3 chromium ोिमयम

4 iron लोहा

Correct Answer -

iron लोहा

One of the major Mid Oceanic Ridge is found in मख म-महासागर चोिटयो म स एक ______ म पायाजाता ह

1 Mid Pacific Ocean म शात महासागर

2 Mid Atlantic Ocean म अटलािटक महासागर

3 Mid Indian Ocean म भारतीय महासागर

4 Mid Arctic Ocean म आक िटक महासागर

68)

69)

70)

71)

Correct Answer -

Mid Atlantic Ocean म अटलािटक महासागर

Magma that reaches the Earthrsquos surface and then solidifies is called________

मा जो पी की सतह तक पचती ह और िफर ठोस हो जाती ह ________कहलाती ह

1 quartz ाटज

2 lava लावा

3 granite नाइट

4 silicates िसिलकट

Correct Answer -

lava लावा

Isotherms are the lines of equal_______ समताप रखाए समान _______की रखाए होती ह

1 pressure दाब

2 temperature तापमान

3 rainfall वषा

4 height ऊचाई

Correct Answer -

temperature तापमान

Mark the correct sequence of passes in the Western Ghats from north to south

पिमी घाटो म उर स दिण तक दर क सही अनम को िचित कर

1 Thalghat Palghat Bhorghat थलगघाट पालघाट भोरघाट

2 Thalghat Bhorghat Palghat थलघाट भोरघाट पालघाट

3 Bhorghat Thalghat Palghat भोरघाट थलघाट पालघाट

4 Palghat Bhorghat Thalghat पालघाट भोरघाट थलघाट

Correct Answer -

Thalghat Bhorghat Palghat थलघाट भोरघाट पालघाट

Which of the following does not have influence over the climate in India

ि ि ि ी ी

72)

73)

िनिलखत म स िकसका भाव भारत की जलवाय पर नही पड़ता ह

1 Ocean currents सागर की लहर

2 Nearness to equator भम रखा स िनकटता

3 Monsoons मानसन

4 Presence of Indian ocean भारतीय महासागर की उपथित

Correct Answer -

Ocean currents सागर की लहर

Which of the following cloud types has the characteristics like vertical tall narrow and puffy

िनिलखत म स िकस कार क मघो म लबवत लबी सकीण और थलता जसी िवशषताए ह

1 Cumulonimbus तफानी मघ

2 Cumulus मघ पज

3 Cirrocumulus पाभ कपासी मघ

4 Nimbostratus वषारी मघ

Correct Answer -

Cumulus मघ पज

Which of the following statement is INCORRECT about Crude Birth Rate

िनिलखत स कौन सा कथन अशोिधत ज दर क बार म सही नही ह

1 It cannot be used for comparing fertility level between two countries with different population characteristics

इसका उपयोग िविभ जनसा िवशषताओ वाल दो दशो क बीच जनन र की तलना क िलए नही िकया जा सकता ह

2 It is a standardized measure of fertility

यह जनन मता का मानकीकत उपाय ह

3 It is effected by the age-sex composition of the population

यह आबादी की आय-िलग सरचना स भािवत होता ह

4 It is expressed per 1000 population in a given geographical unit

यह िकसी दी गई भौगोिलक इकाई म ित 1000 जनसा पर िकया जाता ह

Correct Answer -

It is a standardized measure of fertility

यह जनन मता का मानकीकत उपाय ह

74)

75)

76)

77)

Which of the following state in India experienced negative decadal growth rate during 2001 to 2011census

भारत म िनिलखत म स िकस रा म वष 2001 स 2011 की जनगणना क दौरान नकाराक िगरावट दर ई

1 Tripura िपरा

2 Nagaland नागालड

3 Haryana हरयाणा

4 Odisha ओिडसा

Correct Answer -

Nagaland नागालड

Which of the following is NOT a characteristic of peninsular rivers

िनिलखत म स कौन सी िवशषता ायीपीय निदयो म नही होती ह

1 Flow through shallow valleys उथल घािटयो क माम स वाह

2 Seasonal flow मौसमी वाह

3 Little erosional activity थोड़ी कटावदार गितिविध

4 Meandering tendency often shifting their beds घमावदार वि अर अपन तटो को थानातरत करना

Correct Answer -

Meandering tendency often shifting their beds घमावदार वि अर अपन तटो को थानातरत करना

Which of the following gases in the atmosphere absorbs heat from the Sunrsquos radiation and the Earthssurface

वायमडल म िनिलखत म स कौन सी गस सय क िविकरण और पी की सतह स ऊा को अवशोिषत करती ह

1 Neon िनयॉन

2 Carbon dioxide काबन डाइऑाइड

3 Argon आगन

4 Nitrogen नाइट ोजन

Correct Answer -

Carbon dioxide काबन डाइऑाइड

Which of the following kind of settlement pattern is found at the confluence of rivers

ि ि ि ि ो

78)

79)

80)

िनिलखत म स िकस कार का वथापन पटन निदयो क सगम पर पाया जाता ह

1 Triangular Paern िकोणीय पटन

2 Circular or Semi-Circular Paern परप या अध-परप पटन

3 Nebular Paern नबलर पटन

4 Star ndashShaped Paern ार-आकार का पटन

Correct Answer -

Triangular Paern िकोणीय पटन

Which one was not the objective of the Biosphere Reserve Projects launched by the UNESCO

यनो ारा श की गई सरित जवमडल परयोजनाओ का उ इनम स कौन सा नही था

1 To promote teaching and research िशण और अनसधान को बढ़ावा दना

2 To make agriculture sustainable किष को दीघकािलक बनाना

3 To conserve ecosystems पारथितक त को सरित करना

4 To conserve genetic diversity for a longtime लब समय तक अनवािशक िविवधता को सरित करना

Correct Answer -

To make agriculture sustainable किष को दीघकािलक बनाना

Which region of the Earth surface receives the highest amount of insulation

पी सतह का कौन सा तापावरोधन की उतम माा ा करता ह

1 Land mass थलखड

2 Savannah region सवाना

3 Water bodies जल िनकाय

4 Tropical desert उकिटबधीय रिगान

Correct Answer -

Tropical desert उकिटबधीय रिगान

Which one of the following is not a biodiversity hotspot

िनिलखत म स कौन सा जव िविवधता का म जगह नही ह

1 Eastern Himalaya पव िहमालय

2 Eastern Ghats पव घाट

81)

82)

83)

3 Indo-Myanmar भारत-ामार

4 Westerm Ghats पिमी घाट

Correct Answer -

Eastern Ghats पव घाट

Which one of the following is NOT a part of the World Network of Biosphere Reserves based on theUNESCO Man and Biosphere Programme

यनो मन और बायोीयर कायम क आधार पर िनिलखत म स कौन बायोीयर रजव क िव नटवक कािहा नही ह

1 Gulf of Mannar मार की खाड़ी

2 Seshachalam शषाचलम

3 Sunderban सदरबन

4 Nilgiri नीलिगर

Correct Answer -

Seshachalam शषाचलम

Which one of the following is an example of ldquodesert vegetationrdquo

िनिलखत म स कौन मथलीय वनित का एक उदाहरण ह

1 Mosses and lichens दलदल और शवाल

2 Temperate grassland समशीतो घास क मदान

3 Coniferous forest शकधारी वन

4 Acacia and cactus एकािसया और कस

Correct Answer -

Acacia and cactus एकािसया और कस

Which one of the following reflects more sunlight िनिलखत म स कौन सा सय की रोशनी को अिधकपरावितत करता ह

1 Paddy crop land धान फसल भिम

2 Land covered with fresh snow ताजा बफ स आािदत भिम

3 Sand desert रतीली रिगान

4 Prairie land यरी भिम

84)

85)

86)

87)

Correct Answer -

Land covered with fresh snow ताजा बफ स आािदत भिम

Which layer of the atmosphere is in contact with the surface of the earthrsquos oceans

वायमडल की कौन सी परत पी क महासागरो की सतह क सपक म ह

1 Stratosphere समताप मडल

2 Mesosphere म मडल

3 Hydrosphere जलमडल

4 Troposphere ोभ मडल

Correct Answer -

Troposphere ोभ मडल

Mediterranean Sea is a border of which of the following countries भम सागर िनिलखत दशो म सिकसकी सीमा ह

1 None of these इनम स कोई नही

2 Iraq इराक

3 Lebanon लबनान

4 Jordan जॉडन

Correct Answer -

Lebanon लबनान

Benguela ocean currents are found along which coast बगएला महासागर धाराए िकस तट क साथ पायीजाती ह

1 East Coast of South America दिण अमरका क पव तट

2 East Coast of Africa अीका क पव तट

3 West Coast of South America दिण अमरका क पिमी तट

4 West Coast of Africa अीका क पिमी तट

Correct Answer -

West Coast of Africa अीका क पिमी तट

88)

89)

90)

Due to tension a block of land on one side being pushed up or upthrown relative to the downthrown blockis referred as

तनाव क कारण नीच फ क ए खड क साप भिम का एक खड एक ओर स ऊपर धकला जाता ह या ऊपर की ओरफ का जाता ह यह _____ क प म सदिभत ह

1 Thrust fault प श

2 Normal fault सामा श

3 Reverse fault म श

4 Strike slip fault नितलब सपण श

Correct Answer -

Normal fault सामा श

Inter-tropical doldrums is a zone of ______ अतर-उकिटबधीय डोलड ______ का एक ह

1 Frontolysis टोलायिसस

2 Convergence अिभसरण

3 Inter-tropical divergence zone अतर-उकिटबधीय िवचलन

4 Local wind थानीय वाय

Correct Answer -

Convergence अिभसरण

The Horse Latitudes are regions located at about _____ north and south of the equator

हॉस अाश भम रखा क उर और दिण म लगभग _____ पर थत ह

1 30ndash60 degree Latitude 30-60 िडी अाश

2 0ndash5 degree Latitude 0-5 िडी अाश

3 30 degree Latitude 30 िडी अाश

4 60ndash90 degree Latitude 60-90 िडी अाश

Correct Answer -

30 degree Latitude 30 िडी अाश

Generally evaporation is high over which part of the Earth

आम तौर पर पी क िकस भाग पर वाीकरण अिधक होता ह

1 Equatorial maritime भमवत समीय ी ी

91)

92)

2 Equatorial continental भमवत महाीपीय

3 Polar maritime वीय समीय

4 Polar continental वीय महाीपीय

Correct Answer -

Equatorial maritime भमवत समीय

A very high temperature during summer in north western India leads to what type of climaticcondition in south

उर पिमी भारत म गम क दौरान बत अिधक तापमान होन क कारण दिण म िकस कार की जलवाय थितउ करता ह

1 Depression over arabian sea अरब सागर पर अवनमन

2 Failure monsoon मानसन िवफलता

3 Successful monsoon मानसन सफलता

4 Cyclones चवात

Correct Answer -

Successful monsoon मानसन सफलता

Lightning and thunder are the resultant effect when तिड़त और गजन परणामी भाव ह जब

1 Two massive clouds hit powerfully each other first lightning is produced and later sound is produced

दो बड़ बादल एक दसर स शशाली ढग स टकरात ह पहल आकाशीय िवदयत उ होता ह और बाद म िन उहोती ह

2 Two massive clouds come into contact with the powerful wind collision this results into first sound and thenlightning

दो बड़ बादल शशाली पवन सघ क सपक म आत ह इसका परणामप पहल िन और िफर आकाशीय िवदयतउ होता ह

3 None of the above उपरो म स कोई भी नही

4 A high density cloud contains positively and negatively charged electric ions and when this interacts light andsound are simultaneously produced

एक उ घन बादल म धनाक और ऋणाक आविशत िवदयत आयन होत ह और जब यह परर भाव डालत ह तोकाश और िन एक साथ उािदत होती ह

Correct Answer -

A high density cloud contains positively and negatively charged electric ions and when this interacts light andsound are simultaneously produced

औ ि ि ो औ ो

93)

94)

95)

एक उ घन बादल म धनाक और ऋणाक आविशत िवदयत आयन होत ह और जब यह परर भाव डालत ह तोकाश और िन एक साथ उािदत होती ह

Doon Valley is able to grow rice because दन घाटी चावल उगान म सम ह ोिक

1 Other crops cannot be grown वहा अ फसलो को उगाया नही जा सकता ह

2 People in the valley are rice eaters घाटी म लोग चावल खान वाल ह

3 There is a huge export demand of rice वहा चावल की भारी िनयात माग ह

4 It has warm summer and snow melt waters for irrigation

वहा गिमया गम होती ह िसचाई क िलए बफ का िपघला आ पानी होता ह

Correct Answer -

It has warm summer and snow melt waters for irrigation

वहा गिमया गम होती ह िसचाई क िलए बफ का िपघला आ पानी होता ह

CANCELLED

In the geological time scale the Mesozoic Era DOES NOT contains which of the following periods

भगभय समय पमान पर मजीवी यग म िन कालो म स कौन नही ह

1 Triassic ट ाइऐिसक

2 Jurassic जरिसक

3 Cretaceous चाकमय

4 Carboniferous काबनी

Correct Answer -

Carboniferous काबनी

96)

1 P-3 Q-4 R-2 S-1

2 P-3 Q-4 R-1 S-2

3 P-3 Q-4 R-1 S-2

4 P-4 Q-3 R-2 S-1

Correct Answer -

P-4 Q-3 R-2 S-1

1 P-3 Q-1 R-4 S-2

2 P-3 Q-4 R-1 S-2

3 P-3 Q-2 R-4 S-1

97)

98)

4 P-2 Q-1 R-4 S-3

Correct Answer -

P-3 Q-4 R-1 S-2

1 P-3 Q-1 R-4 S-2

2 P-2 Q-3 R-4 S-1

3 P-2 Q-1 R-3 S-4

4 P-4 Q-2 R-1 S-3

Correct Answer -

P-3 Q-1 R-4 S-2

99)

100)

1 P-3 Q-2 R-4 S-1

2 P-1 Q-2 R-3 S-4

3 P-2 Q-3 R-1 S-4

4 P-4 Q-3 R-2 S-1

Correct Answer -

P-2 Q-3 R-1 S-4

ldquoHuman geography is the study of changing relationship between the unresting man and the unstableearthrdquo was defined by

lsquolsquoमानव भगोल ाकल आदमी और अथर पी क बीच सबध परवतन का अयन हrdquo ______ ारा परभािषत िकया गयाथा

1 J Brunches ज चस

2 EC Semple ईसी सल

3 HJ Mackinder एच ज मिकदर

4 PV Blache पीवी च

Correct Answer -

EC Semple ईसी सल

Sedimentary rocks are finally and ultimately derived from the____________

अवसादी चान अततः ________ स ा की जाती ह

1 action of earth movements पी की गितिविधयो

2 marine deposit समी िनप

3 weathering of metamorphic rocks पातरत चानो क अपय

4 weathering of igneous rocks आय चानो क अपय

Correct Answer -

weathering of igneous rocks आय चानो क अपय

Page 26: High School Teacher Eligibility Test- BOARD PROFESSIONAL ...peb.mp.gov.in/results/RESULT_18/HST_RES18/Final_anwser_key/HST… · M a ndl a / मंड ल ... Under the Madhya Pradesh

28)

29)

30)

31)

Correct Answer -

All parallels are equal in length सभी समातर लबाई म समान ह

Which of the following Indian states is also known as a lsquoLand of Red river and Blue Hillsrsquo

िनिलखत म स िकस भारतीय रा को लाल नदी और नीली पहािड़यो की भिम क नाम स जाना जाता ह

1 Uttarkhand उराखड

2 Assam असम

3 Meghalaya मघालय

4 Arunachal Pradesh अणाचल दश

Correct Answer -

Assam असम

In spatial analysis of settlement Rn = 215 indicates which type of settlement arrangement

िनपटान क थािनक िवषण म Rn = 215 यह इिगत करता ह िक िकस कार की िनपटान वथा ह

1 Uniform यिनफॉम

2 Semi-Clustered समी- ल टर

3 Clustered ल टर

4 Random रडम

Correct Answer -

Uniform यिनफॉम

Who are known as the lsquoYellow Peoplersquo lsquoयलो पीपलrsquo क प म कौन जाना जाता ह

1 Mongoloids मोगोलोइडस

2 Nigroids नीोइडस

3 Australoids ऑ लॉइडस

4 Caucasoids कॉकसोइडस

Correct Answer -

Mongoloids मोगोलोइडस

ि ि ो ौ ि

32)

33)

34)

Who publishes the topographical map of India भारत क थलाकितक मानिच को कौन कािशत करता ह

1 Geographical Survey of India भारत का भौगोिलक सवण

2 Government of India भारत सरकार

3 Geological Survey of India भारत क भगभय सवण

4 Survey of India भारत का सवण

Correct Answer -

Survey of India भारत का सवण

Who among the following claimed geography to be the lsquoEcology of Manrsquo

िनिलखत म स िकसन भगोल को मन का पारथितकी कहा ह

1 Alfred Hener अड हटनर

2 Vidal-de la Blache वाइडल-िड लॉ ॉश

3 Oo Schluter ओटो टर

4 Harlan Barrow हरलन बारो

Correct Answer -

Harlan Barrow हरलन बारो

Who among the following is regarded as the founder of humanistic approach in geography

िनिलखत म स िकस भगोल म मानवतावादी िकोण का सथापक माना जाता ह

1 William Bunge िविलयम बग

2 Yi-Fu-Tuan यी-फ- यान

3 Brain JL Berry न जएल बरी

4 Richard Peet रचड पीट

Correct Answer -

Yi-Fu-Tuan यी-फ- यान

Who prepared Lorenz curve लोरज व िकसन तयार िकया

1 Geddes गडस

2 None of these इनम स कोई नही

3 Griffith Taylor ििफथ टलर

35)

36)

37)

4 Max U Lorenz म य लोरज

Correct Answer -

Max U Lorenz म य लोरज

Gulf Streams are the currents of which of the following oceans

खाड़ी की धाराए िनिलखत महासागरो म स िकसकी धाराए ह

1 North Atlantic Ocean उरी अटलािटक महासागर

2 North Pacific Ocean उरी शात महासागर

3 Arabian Sea अरब सागर

4 South Pacific Ocean दिण शात महासागर

Correct Answer -

North Atlantic Ocean उरी अटलािटक महासागर

Disintegration wearing away and removal of rock material is generally referred as

िशला पदाथ (रॉक सामी) का टटना िमटना और हटना आमतौर पर ________ क प म सदिभत िकया जाता ह

1 Shattering िवसकारक

2 Denudation अनाादन

3 Fault श

4 Decomposition िवयोजन

Correct Answer -

Denudation अनाादन

Variations in the length of day time and night from season to season are due to

मौसम स मौसम परवतन पर िदन क समय और रात क समय की अविध म िभताए िन कारण स होती ह

1 The Earthrsquos revolution round the Sun in an elliptical manner पी का दीघवाकार तरीक स सय क चारो घणन

2 The Earthrsquos rotation on its axis पी का इसकी धरी पर घणन

3 Revolution of the Earth on a tilted axis नत अ पर पी का घणन

4 Latitudinal position of the place थान की अाश थित

Correct Answer -

Revolution of the Earth on a tilted axis नत अ पर पी का घणन

38)

39)

40)

Point out the correct sequence of mountain ranges from north to south

उर स दिण तक पवत खलाओ क सही अनम को इिगत कर

1 Great Himalaya Middle Himalaya Outer Himalaya Trans Himalaya

महान िहमालय म िहमालय बा िहमालय परा िहमालय

2 Middle Himalaya Great Himalaya Trans Himalaya Outer Himalaya

म िहमालय महान िहमालय परा िहमालय बा िहमालय

3 Outer Himalaya Middle Himalaya Great Himalaya Trans Himalaya

बा िहमालय म िहमालय महान िहमालय परा िहमालय

4 Trans Himalaya Great Himalaya Middle Himalaya Outer Himalaya

परा िहमालय महान िहमालय म िहमालय बा िहमालय

Correct Answer -

Trans Himalaya Great Himalaya Middle Himalaya Outer Himalaya

परा िहमालय महान िहमालय म िहमालय बा िहमालय

Sunrsquos halo is produced by the refraction of light in सय का भामडल ______ म काश क अपवतन ाराउ होता ह

1 Ice crystals in Cirrro-Cumulus clouds पाभ-कपास मघो क बफ िल

2 Ice crystal in Cirrus clouds पाभमघो क बफ िल

3 Dust particles in Stratus clouds री मघो क धल कण

4 Water vapour in Stratus clouds री मघो क जल वा

Correct Answer -

Ice crystal in Cirrus clouds पाभमघो क बफ िल

Read the given statements and answer which of the following options isare correct

(1) The minerals present in the rocks exposed to atmosphere are not subjected to alteration

(2) Oxidation is one of the processes of chemical weathering

िदए गए कथनो को पढ़ और उर द िक िन म स कौन सास िवक सही ह

(1) वायमल स अनावत शल म उपथत खिनज परवतन क अधीन नही होता ह

(2) ऑीकरण रासायिनक अपय की ियाओ म स एक ह

ो ो

41)

42)

1 Both statements are wrong दोनो कथन गलत ह

2 Both statements are correct दोनो कथन सही ह

3 First statement is wrong and second statement is correct पहला कथन गलत ह और दसरा कथन सही ह

4 First statement is correct and second statement is wrong पहला कथन सही ह और दसरा कथन गलत ह

Correct Answer -

First statement is wrong and second statement is correct पहला कथन गलत ह और दसरा कथन सही ह

Read the given statements and answer which of the following options isare correct

1 Sunrsquos short waves enter the earth partially heating the atmosphere

2 Heated earth surface from the sun produces broader waves which interacts and heats the atmosphere

िदए गए कथन को पढ़ और उर द िक िन म स कौन सास िवक सही ह

1 सय की छोटी तरग पी म आिशक प स वश करती ह और वायमडल को ऊत करती ह

2 सय स ऊत पी की सतह िवारत तरग उ करती ह जो परर भाव डालती ह और वायमडल कोऊत करती ह

1 Both Statements 1 and 2 are correct दोनो कथन 1 और 2 सही ह

2 Both Statements 1 and 2 are wrong दोनो कथन 1 और 2 गलत ह

3 Statement 1 is wrong and only Statement 2 is correct कथन 1 गलत ह और कवल कथन 2 सही ह

4 Only statement 1 is correct कवल कथन 1 सही ह

Correct Answer -

Both Statements 1 and 2 are correct दोनो कथन 1 और 2 सही ह

Read the given statements and answer which of the following options isare correct

(1)The rocks that get changed due to heat and pressure are termed as metamorphic rocks

(2)Slate is one such type of metamorphic rock

िदए गए कथनो को पढ़ और उर द िक िन म स कौन सास िवक सही ह

(1) शल जो ऊा और दाब क कारण परवितत हो जात ह उ कायातरक शलो क प म जाना जाता ह

(2) ट एक तरह का कायातरक शल ह

1 Both statements are wrong दोनो कथन गलत ह

2 Both statements are correct दोनो कथन सही ह

3 First statement is wrong and second statement is correct पहला कथन गलत ह और दसरा कथन सही ह

ी औ

43)

44)

4 First statement is correct and second statement is wrong पहला कथन सही ह और दसरा कथन गलत ह

Correct Answer -

Both statements are correct दोनो कथन सही ह

Read the given statements and answer which of the following options isare correct

1 Higher temperature anomaly is observed in the northern hemisphere

2 Differential heating is absent in Northern Hemisphere

िदए गए कथनो को पढ़ और उर द िक िन म स कौन सास िवक सही ह

1 उरी गोलाध म उ तापमान िवसगित पायी जाती ह

2 उरी गोलाध म अतर ऊन अनपथत होती ह

1 Both Statements 1 and 2 are correct दोनो कथन 1 और 2 सही ह

2 Both Statements 1 and 2 are wrong दोनो कथन 1 और 2 गलत ह

3 Statement 1 is wrong and Statement 2 is correct कथन 1 गलत ह और कथन 2 सही ह

4 Statement 1 is correct and Statement 2 is wrong कथन 1 सही ह और कथन 2 गलत ह

Correct Answer -

Statement 1 is correct and Statement 2 is wrong कथन 1 सही ह और कथन 2 गलत ह

Read the given statements and answer which of the following options isare correct

(1) Plutonic rocks are intrusive type of igneous rocks

(2) It cools very slowly because the surrounding rock serves as insulation around the intrusion of magma

िदए गए कथनो को पढ़ और उर द िक िन म स कौन सास िवक सही ह

(1) िवतलीय शल अतवधी कार क आश शल ह

(2) यह बत धीर-धीर ठडा होता ह ोिक आस-पास क शल मा क अतवधन क चारो ओर रोधन क प म कायकरत ह

1 Both statements are wrong दोनो कथन गलत ह

2 Both statements are correct दोनो कथन सही ह

3 First statement is wrong and second statement is correct पहला कथन गलत ह और दसरा कथन सही ह

4 First statement is correct and second statement is wrong पहला कथन सही ह और दसरा कथन गलत ह

Correct Answer -

Both statements are correct दोनो कथन सही ह

45)

46)

47)

48)

The dust and ash material hurled from the volcanoes are termed as

ालामखी स िनकलन वाली धल और राख सामी को _______ क प म कहा जाता ह

1 Pyroclasc पाइरोाक

2 Hyperclastic हाइपराक

3 Hepiroclastic हिपरोाक

4 Cirroclastic िसरोाक

Correct Answer -

Pyroclasc पाइरोाक

The vertical difference in elevation between a low tide and high tide is referred as

कम ार और उ ार क बीच ऊचाई म लबवत अतर _____ स सदिभत होता ह

1 Tidal slope ारीय ढलान

2 Tidal elevation ारीय उयन

3 Tidal range ारीय परास

4 Tidal height ारीय ऊचाई

Correct Answer -

Tidal range ारीय परास

The maximum biodiversity is found in which of the following regions िनिलखत ो म स अिधकतमजव िविवधता िकसम पायी जाती ह

1 Amazon Basin अमज़न बिसन

2 East Indies ई इडीज

3 Congo Basin कागो बिसन

4 West indies व इडीज

Correct Answer -

Amazon Basin अमज़न बिसन

The cultivation of rice crop produces_______ चावल की फसल की खती ______ का उादन करती ह

1 SO2

49)

50)

51)

2 CH4

3 CFCs

4 CO2

Correct Answer -

CH4

The pressure system with higher pressure at the centre is called__________

क म उ दबाव वाली दबाव णाली को _______ कहा जाता ह

1 front अ

2 depression अवनमन

3 cyclone चवात

4 anti-cyclone ितचवात

Correct Answer -

anti-cyclone ितचवात

The Himalayan region is poor in mineral resources because िहमालयी खिनज ससाधनो म समनही ह ोिक

1 The displacement of rock strata has disturbed the arrangement of rocks and made it complex

शलीय परत क िवथापन न चानो की वथा को अवथत कर िदया ह और इस जिटल बना िदया ह

2 The climate conditions are not suitable for exploitation of minerals

जलवाय की थित खिनजो क दोहन क िलए उपय नही ह

3 The terrain makes explanation of minerals difficult and very costly due to transportation difficulties

भ-भाग परवहन की किठनाइयो क कारण खिनजो का दोहन मल और बत महगा बना दता ह

4 It is made up of crystalline rocks यह िलीय चानो स बना ह

Correct Answer -

The displacement of rock strata has disturbed the arrangement of rocks and made it complex

शलीय परत क िवथापन न चानो की वथा को अवथत कर िदया ह और इस जिटल बना िदया ह

The process through which the moisture is added to the atmosphere by vegetation is termed as

वह िया िजसक माम स वनित ारा वातावरण म नमी िमलायी जाती ह _______ क प म जानी जाती ह

52)

53)

54)

1 Condensation सघनन

2 Evapotranspiration वान-उजन

3 Radiation िविकरण

4 Precipitation वषण

Correct Answer -

Evapotranspiration वान-उजन

The process through which the terrestrial heat is transferred to air by direct contact is termed as

वह िया िजसम सपक ारा थलीय ऊा वाय म थानातरत हो जाती ह ______ क प म जानी जाती ह

1 Conduction चालन

2 Convection सवहन

3 Insolation आतपन

4 Radiation िविकरण

Correct Answer -

Conduction चालन

The largest area under mangroves is in which of the following statesunion territory

मोव क अतगत िनिलखत राोसघ शािसत दशो म स सबस बड़ा कौन सा ह

1 Andaman and Nicobar अमान और िनकोबार

2 Andhra Pradesh आ दश

3 West Bengal पिम बगाल

4 Gujarat गजरात

Correct Answer -

West Bengal पिम बगाल

The longitudinal transverse and surface waves in an earthquake originate from

भकप म दशातर अनथ और सतह तरग यहा उ होती ह

1 The focus on the surface of the Earth पी क सतह पर क -िबद म

2 The focus within the body of the Earth पी क भीतर क -िबद म

3 The epicenter within the body of the Earth पी क भीतर उपरक म

55)

56)

57)

4 The epicenter on the surface of the Earth पी क सतह पर उपरक म

Correct Answer -

The focus within the body of the Earth पी क भीतर क -िबद म

The down slope movement of material due to gravity is called______

गाकषण क कारण पदाथ की अनढाल गित को ______ कहा जाता ह

1 mass movement पदाथ सचलन

2 deposition िनप

3 erosion रण

4 volcanic movement ालामखीय सचलन

Correct Answer -

mass movement पदाथ सचलन

Shimla is cooler than Amritsar although both are on the same latitude This is because

िशमला म अमतसर स अिधक ठड ह हालािक दोनो समान अाश पर ह ऐसा ह ोिक

1 Shimla is at a greater height above sea level than Amritsar अमतसर की तलना म िशमला सम तल स अिधकऊचाई पर ह

2 Shimla is further north िशमला उर की ओर ह

3 Shimla is farther from the equator िशमला भम रखा स आग ह

4 Their longitudes differ उनकी दशातर रखाए िभ ह

Correct Answer -

Shimla is at a greater height above sea level than Amritsar अमतसर की तलना म िशमला सम तल स अिधकऊचाई पर ह

lsquoTempo of Urbanizationrsquo measures which of the following

lsquoशहरीकरण का टपोrsquo िनिलखत म स कौन सा उपाय ह

1 Speed of urbanizaon शहरीकरण की गित

2 None of the above इनम स कोई नही

3 Inequality of urbanizaon शहरीकरण की असमानता

4 Current level of urbanizaon शहरीकरण का वतमान र

Correct Answer -

58)

59)

60)

Speed of urbanizaon शहरीकरण की गित

Out of the following options choose the INCORRECT statement

िनिलखत िवको म स गलत कथन का चयन कर

1 The clear tracts in the equatorial region recover rapidly भम रखा म भभाग तजी स ठीक हो जात ह

2 The stable communities include a redwood forest a pine forest at high elevations

थर समदायो म एक रडवड वन उ ऊचाई पर एक दवदार वन शािमल ह

3 Any ecosystem moves towards maximum biomass and stability to survive

कोई भी पारथितकी त जीिवत रहन क िलए अिधकतम जवसहित और थरता की तरफ असर होता ह

4 Tropical rain forests near equator are stable ecosystems

भम रखा क पास उकिटबधीय वषा वन थर पारथितक त ह

Correct Answer -

The clear tracts in the equatorial region recover rapidly भम रखा म भभाग तजी स ठीक हो जात ह

Seasonal contrasts are maximum in मौसमी िवषमता अिधकतम ह

1 Mid latitudes म अाश म

2 Low attitudes िन अाश म

3 High latitudes उ अाश म

4 Subtropics उपोकिटबधीय म

Correct Answer -

Mid latitudes म अाश म

In India which type of forest among the following occupies the largest area

भारत म िनिलखत म स िकस कार क वन सबस बड़ा फल आािदत करत ह

1 Sub-tropical Dry Evergreen Forest उप उकिटबधीय श सदाबहार वन

2 Mountain Wet Temperate Forest पवतीय आ शीतो वन

3 Tropical Moist Deciduous Forest उकिटबधीय आ पणपाती वन

4 Tropical Wet Evergreen Forest उकिटबधीय आ सदाबहार वन

Correct Answer -

Tropical Moist Deciduous Forest उकिटबधीय आ पणपाती वन

61)

62)

63)

64)

What is the proportion of lsquoJuvenile Populationrsquo (0-14 years) in India as per 2011Census

2011 की जनगणना क अनसार भारत म जवनाइल पॉपलशन यानी िकशोर जनस या (0-14 वष) का अनपात ाह

1 3076 of total population कल जनस या का 3076

2 2764 of total population कल जनस या का 2764

3 2933 of total population कल जनस या का 2933

4 3354 of total population कल जनस या का 3354

Correct Answer -

3076 of total population कल जनस या का 3076

What is the Belfast famous for बलफा िकसक िलए मशर ह

1 Belt of cotton textile industry कपास व उोग क

2 Ship-building industry जहाज िनमाण उोग

3 Agricultural machinery किष उपकरण

4 Aero planes manufacturing वाययान िनमाण

Correct Answer -

Ship-building industry जहाज िनमाण उोग

What is the most important occupation in tropical monsoon lands

उकिटबधीय मॉनसन भिम म सबस महपण वसाय ा ह

1 Mining खनन

2 Cattle rearing मवशी पालन

3 Agriculture किष

4 Nomadic herding नोमािडक जड़ी-बिटया

Correct Answer -

Agriculture किष

What is the most important characteristics of the islands (Indian) located in the Arabian sea

अरब सागर म थत ीपो (भारतीय) की सबस महपण िवशषता ा ह

ी ो

65)

66)

67)

1 There are all of coral origins सभी कोरल मल क ह

2 There are all very small in size य सभी आकार म बत छोट ह

3 They have a very dry climate इनकी जलवाय बत श ह

4 They are extended parts of the mainland व महाीप क िवारत िह ह

Correct Answer -

There are all of coral origins सभी कोरल मल क ह

What do the basalt layers of the Deccan indicate डन की बसा परत ा इिगत करती ह

1 All of the above उपरो सभी

2 Huge volcanic eruptions in the distant past दरथ अतीत म िवशाल ालामखीय िवोट

3 The immense erosional activity of the rivers निदयो की िवशाल रण गितिविध

4 The influence of weathering मौसम का भाव

Correct Answer -

Huge volcanic eruptions in the distant past दरथ अतीत म िवशाल ालामखीय िवोट

In the structure of planet Earth below the mantle the core is mainly made up of_____

पी ह की सरचना म मटल क नीच कोर म प स______ स िनिमत होती ह

1 aluminium एमीिनयम

2 silicon िसिलकॉन

3 chromium ोिमयम

4 iron लोहा

Correct Answer -

iron लोहा

One of the major Mid Oceanic Ridge is found in मख म-महासागर चोिटयो म स एक ______ म पायाजाता ह

1 Mid Pacific Ocean म शात महासागर

2 Mid Atlantic Ocean म अटलािटक महासागर

3 Mid Indian Ocean म भारतीय महासागर

4 Mid Arctic Ocean म आक िटक महासागर

68)

69)

70)

71)

Correct Answer -

Mid Atlantic Ocean म अटलािटक महासागर

Magma that reaches the Earthrsquos surface and then solidifies is called________

मा जो पी की सतह तक पचती ह और िफर ठोस हो जाती ह ________कहलाती ह

1 quartz ाटज

2 lava लावा

3 granite नाइट

4 silicates िसिलकट

Correct Answer -

lava लावा

Isotherms are the lines of equal_______ समताप रखाए समान _______की रखाए होती ह

1 pressure दाब

2 temperature तापमान

3 rainfall वषा

4 height ऊचाई

Correct Answer -

temperature तापमान

Mark the correct sequence of passes in the Western Ghats from north to south

पिमी घाटो म उर स दिण तक दर क सही अनम को िचित कर

1 Thalghat Palghat Bhorghat थलगघाट पालघाट भोरघाट

2 Thalghat Bhorghat Palghat थलघाट भोरघाट पालघाट

3 Bhorghat Thalghat Palghat भोरघाट थलघाट पालघाट

4 Palghat Bhorghat Thalghat पालघाट भोरघाट थलघाट

Correct Answer -

Thalghat Bhorghat Palghat थलघाट भोरघाट पालघाट

Which of the following does not have influence over the climate in India

ि ि ि ी ी

72)

73)

िनिलखत म स िकसका भाव भारत की जलवाय पर नही पड़ता ह

1 Ocean currents सागर की लहर

2 Nearness to equator भम रखा स िनकटता

3 Monsoons मानसन

4 Presence of Indian ocean भारतीय महासागर की उपथित

Correct Answer -

Ocean currents सागर की लहर

Which of the following cloud types has the characteristics like vertical tall narrow and puffy

िनिलखत म स िकस कार क मघो म लबवत लबी सकीण और थलता जसी िवशषताए ह

1 Cumulonimbus तफानी मघ

2 Cumulus मघ पज

3 Cirrocumulus पाभ कपासी मघ

4 Nimbostratus वषारी मघ

Correct Answer -

Cumulus मघ पज

Which of the following statement is INCORRECT about Crude Birth Rate

िनिलखत स कौन सा कथन अशोिधत ज दर क बार म सही नही ह

1 It cannot be used for comparing fertility level between two countries with different population characteristics

इसका उपयोग िविभ जनसा िवशषताओ वाल दो दशो क बीच जनन र की तलना क िलए नही िकया जा सकता ह

2 It is a standardized measure of fertility

यह जनन मता का मानकीकत उपाय ह

3 It is effected by the age-sex composition of the population

यह आबादी की आय-िलग सरचना स भािवत होता ह

4 It is expressed per 1000 population in a given geographical unit

यह िकसी दी गई भौगोिलक इकाई म ित 1000 जनसा पर िकया जाता ह

Correct Answer -

It is a standardized measure of fertility

यह जनन मता का मानकीकत उपाय ह

74)

75)

76)

77)

Which of the following state in India experienced negative decadal growth rate during 2001 to 2011census

भारत म िनिलखत म स िकस रा म वष 2001 स 2011 की जनगणना क दौरान नकाराक िगरावट दर ई

1 Tripura िपरा

2 Nagaland नागालड

3 Haryana हरयाणा

4 Odisha ओिडसा

Correct Answer -

Nagaland नागालड

Which of the following is NOT a characteristic of peninsular rivers

िनिलखत म स कौन सी िवशषता ायीपीय निदयो म नही होती ह

1 Flow through shallow valleys उथल घािटयो क माम स वाह

2 Seasonal flow मौसमी वाह

3 Little erosional activity थोड़ी कटावदार गितिविध

4 Meandering tendency often shifting their beds घमावदार वि अर अपन तटो को थानातरत करना

Correct Answer -

Meandering tendency often shifting their beds घमावदार वि अर अपन तटो को थानातरत करना

Which of the following gases in the atmosphere absorbs heat from the Sunrsquos radiation and the Earthssurface

वायमडल म िनिलखत म स कौन सी गस सय क िविकरण और पी की सतह स ऊा को अवशोिषत करती ह

1 Neon िनयॉन

2 Carbon dioxide काबन डाइऑाइड

3 Argon आगन

4 Nitrogen नाइट ोजन

Correct Answer -

Carbon dioxide काबन डाइऑाइड

Which of the following kind of settlement pattern is found at the confluence of rivers

ि ि ि ि ो

78)

79)

80)

िनिलखत म स िकस कार का वथापन पटन निदयो क सगम पर पाया जाता ह

1 Triangular Paern िकोणीय पटन

2 Circular or Semi-Circular Paern परप या अध-परप पटन

3 Nebular Paern नबलर पटन

4 Star ndashShaped Paern ार-आकार का पटन

Correct Answer -

Triangular Paern िकोणीय पटन

Which one was not the objective of the Biosphere Reserve Projects launched by the UNESCO

यनो ारा श की गई सरित जवमडल परयोजनाओ का उ इनम स कौन सा नही था

1 To promote teaching and research िशण और अनसधान को बढ़ावा दना

2 To make agriculture sustainable किष को दीघकािलक बनाना

3 To conserve ecosystems पारथितक त को सरित करना

4 To conserve genetic diversity for a longtime लब समय तक अनवािशक िविवधता को सरित करना

Correct Answer -

To make agriculture sustainable किष को दीघकािलक बनाना

Which region of the Earth surface receives the highest amount of insulation

पी सतह का कौन सा तापावरोधन की उतम माा ा करता ह

1 Land mass थलखड

2 Savannah region सवाना

3 Water bodies जल िनकाय

4 Tropical desert उकिटबधीय रिगान

Correct Answer -

Tropical desert उकिटबधीय रिगान

Which one of the following is not a biodiversity hotspot

िनिलखत म स कौन सा जव िविवधता का म जगह नही ह

1 Eastern Himalaya पव िहमालय

2 Eastern Ghats पव घाट

81)

82)

83)

3 Indo-Myanmar भारत-ामार

4 Westerm Ghats पिमी घाट

Correct Answer -

Eastern Ghats पव घाट

Which one of the following is NOT a part of the World Network of Biosphere Reserves based on theUNESCO Man and Biosphere Programme

यनो मन और बायोीयर कायम क आधार पर िनिलखत म स कौन बायोीयर रजव क िव नटवक कािहा नही ह

1 Gulf of Mannar मार की खाड़ी

2 Seshachalam शषाचलम

3 Sunderban सदरबन

4 Nilgiri नीलिगर

Correct Answer -

Seshachalam शषाचलम

Which one of the following is an example of ldquodesert vegetationrdquo

िनिलखत म स कौन मथलीय वनित का एक उदाहरण ह

1 Mosses and lichens दलदल और शवाल

2 Temperate grassland समशीतो घास क मदान

3 Coniferous forest शकधारी वन

4 Acacia and cactus एकािसया और कस

Correct Answer -

Acacia and cactus एकािसया और कस

Which one of the following reflects more sunlight िनिलखत म स कौन सा सय की रोशनी को अिधकपरावितत करता ह

1 Paddy crop land धान फसल भिम

2 Land covered with fresh snow ताजा बफ स आािदत भिम

3 Sand desert रतीली रिगान

4 Prairie land यरी भिम

84)

85)

86)

87)

Correct Answer -

Land covered with fresh snow ताजा बफ स आािदत भिम

Which layer of the atmosphere is in contact with the surface of the earthrsquos oceans

वायमडल की कौन सी परत पी क महासागरो की सतह क सपक म ह

1 Stratosphere समताप मडल

2 Mesosphere म मडल

3 Hydrosphere जलमडल

4 Troposphere ोभ मडल

Correct Answer -

Troposphere ोभ मडल

Mediterranean Sea is a border of which of the following countries भम सागर िनिलखत दशो म सिकसकी सीमा ह

1 None of these इनम स कोई नही

2 Iraq इराक

3 Lebanon लबनान

4 Jordan जॉडन

Correct Answer -

Lebanon लबनान

Benguela ocean currents are found along which coast बगएला महासागर धाराए िकस तट क साथ पायीजाती ह

1 East Coast of South America दिण अमरका क पव तट

2 East Coast of Africa अीका क पव तट

3 West Coast of South America दिण अमरका क पिमी तट

4 West Coast of Africa अीका क पिमी तट

Correct Answer -

West Coast of Africa अीका क पिमी तट

88)

89)

90)

Due to tension a block of land on one side being pushed up or upthrown relative to the downthrown blockis referred as

तनाव क कारण नीच फ क ए खड क साप भिम का एक खड एक ओर स ऊपर धकला जाता ह या ऊपर की ओरफ का जाता ह यह _____ क प म सदिभत ह

1 Thrust fault प श

2 Normal fault सामा श

3 Reverse fault म श

4 Strike slip fault नितलब सपण श

Correct Answer -

Normal fault सामा श

Inter-tropical doldrums is a zone of ______ अतर-उकिटबधीय डोलड ______ का एक ह

1 Frontolysis टोलायिसस

2 Convergence अिभसरण

3 Inter-tropical divergence zone अतर-उकिटबधीय िवचलन

4 Local wind थानीय वाय

Correct Answer -

Convergence अिभसरण

The Horse Latitudes are regions located at about _____ north and south of the equator

हॉस अाश भम रखा क उर और दिण म लगभग _____ पर थत ह

1 30ndash60 degree Latitude 30-60 िडी अाश

2 0ndash5 degree Latitude 0-5 िडी अाश

3 30 degree Latitude 30 िडी अाश

4 60ndash90 degree Latitude 60-90 िडी अाश

Correct Answer -

30 degree Latitude 30 िडी अाश

Generally evaporation is high over which part of the Earth

आम तौर पर पी क िकस भाग पर वाीकरण अिधक होता ह

1 Equatorial maritime भमवत समीय ी ी

91)

92)

2 Equatorial continental भमवत महाीपीय

3 Polar maritime वीय समीय

4 Polar continental वीय महाीपीय

Correct Answer -

Equatorial maritime भमवत समीय

A very high temperature during summer in north western India leads to what type of climaticcondition in south

उर पिमी भारत म गम क दौरान बत अिधक तापमान होन क कारण दिण म िकस कार की जलवाय थितउ करता ह

1 Depression over arabian sea अरब सागर पर अवनमन

2 Failure monsoon मानसन िवफलता

3 Successful monsoon मानसन सफलता

4 Cyclones चवात

Correct Answer -

Successful monsoon मानसन सफलता

Lightning and thunder are the resultant effect when तिड़त और गजन परणामी भाव ह जब

1 Two massive clouds hit powerfully each other first lightning is produced and later sound is produced

दो बड़ बादल एक दसर स शशाली ढग स टकरात ह पहल आकाशीय िवदयत उ होता ह और बाद म िन उहोती ह

2 Two massive clouds come into contact with the powerful wind collision this results into first sound and thenlightning

दो बड़ बादल शशाली पवन सघ क सपक म आत ह इसका परणामप पहल िन और िफर आकाशीय िवदयतउ होता ह

3 None of the above उपरो म स कोई भी नही

4 A high density cloud contains positively and negatively charged electric ions and when this interacts light andsound are simultaneously produced

एक उ घन बादल म धनाक और ऋणाक आविशत िवदयत आयन होत ह और जब यह परर भाव डालत ह तोकाश और िन एक साथ उािदत होती ह

Correct Answer -

A high density cloud contains positively and negatively charged electric ions and when this interacts light andsound are simultaneously produced

औ ि ि ो औ ो

93)

94)

95)

एक उ घन बादल म धनाक और ऋणाक आविशत िवदयत आयन होत ह और जब यह परर भाव डालत ह तोकाश और िन एक साथ उािदत होती ह

Doon Valley is able to grow rice because दन घाटी चावल उगान म सम ह ोिक

1 Other crops cannot be grown वहा अ फसलो को उगाया नही जा सकता ह

2 People in the valley are rice eaters घाटी म लोग चावल खान वाल ह

3 There is a huge export demand of rice वहा चावल की भारी िनयात माग ह

4 It has warm summer and snow melt waters for irrigation

वहा गिमया गम होती ह िसचाई क िलए बफ का िपघला आ पानी होता ह

Correct Answer -

It has warm summer and snow melt waters for irrigation

वहा गिमया गम होती ह िसचाई क िलए बफ का िपघला आ पानी होता ह

CANCELLED

In the geological time scale the Mesozoic Era DOES NOT contains which of the following periods

भगभय समय पमान पर मजीवी यग म िन कालो म स कौन नही ह

1 Triassic ट ाइऐिसक

2 Jurassic जरिसक

3 Cretaceous चाकमय

4 Carboniferous काबनी

Correct Answer -

Carboniferous काबनी

96)

1 P-3 Q-4 R-2 S-1

2 P-3 Q-4 R-1 S-2

3 P-3 Q-4 R-1 S-2

4 P-4 Q-3 R-2 S-1

Correct Answer -

P-4 Q-3 R-2 S-1

1 P-3 Q-1 R-4 S-2

2 P-3 Q-4 R-1 S-2

3 P-3 Q-2 R-4 S-1

97)

98)

4 P-2 Q-1 R-4 S-3

Correct Answer -

P-3 Q-4 R-1 S-2

1 P-3 Q-1 R-4 S-2

2 P-2 Q-3 R-4 S-1

3 P-2 Q-1 R-3 S-4

4 P-4 Q-2 R-1 S-3

Correct Answer -

P-3 Q-1 R-4 S-2

99)

100)

1 P-3 Q-2 R-4 S-1

2 P-1 Q-2 R-3 S-4

3 P-2 Q-3 R-1 S-4

4 P-4 Q-3 R-2 S-1

Correct Answer -

P-2 Q-3 R-1 S-4

ldquoHuman geography is the study of changing relationship between the unresting man and the unstableearthrdquo was defined by

lsquolsquoमानव भगोल ाकल आदमी और अथर पी क बीच सबध परवतन का अयन हrdquo ______ ारा परभािषत िकया गयाथा

1 J Brunches ज चस

2 EC Semple ईसी सल

3 HJ Mackinder एच ज मिकदर

4 PV Blache पीवी च

Correct Answer -

EC Semple ईसी सल

Sedimentary rocks are finally and ultimately derived from the____________

अवसादी चान अततः ________ स ा की जाती ह

1 action of earth movements पी की गितिविधयो

2 marine deposit समी िनप

3 weathering of metamorphic rocks पातरत चानो क अपय

4 weathering of igneous rocks आय चानो क अपय

Correct Answer -

weathering of igneous rocks आय चानो क अपय

Page 27: High School Teacher Eligibility Test- BOARD PROFESSIONAL ...peb.mp.gov.in/results/RESULT_18/HST_RES18/Final_anwser_key/HST… · M a ndl a / मंड ल ... Under the Madhya Pradesh

32)

33)

34)

Who publishes the topographical map of India भारत क थलाकितक मानिच को कौन कािशत करता ह

1 Geographical Survey of India भारत का भौगोिलक सवण

2 Government of India भारत सरकार

3 Geological Survey of India भारत क भगभय सवण

4 Survey of India भारत का सवण

Correct Answer -

Survey of India भारत का सवण

Who among the following claimed geography to be the lsquoEcology of Manrsquo

िनिलखत म स िकसन भगोल को मन का पारथितकी कहा ह

1 Alfred Hener अड हटनर

2 Vidal-de la Blache वाइडल-िड लॉ ॉश

3 Oo Schluter ओटो टर

4 Harlan Barrow हरलन बारो

Correct Answer -

Harlan Barrow हरलन बारो

Who among the following is regarded as the founder of humanistic approach in geography

िनिलखत म स िकस भगोल म मानवतावादी िकोण का सथापक माना जाता ह

1 William Bunge िविलयम बग

2 Yi-Fu-Tuan यी-फ- यान

3 Brain JL Berry न जएल बरी

4 Richard Peet रचड पीट

Correct Answer -

Yi-Fu-Tuan यी-फ- यान

Who prepared Lorenz curve लोरज व िकसन तयार िकया

1 Geddes गडस

2 None of these इनम स कोई नही

3 Griffith Taylor ििफथ टलर

35)

36)

37)

4 Max U Lorenz म य लोरज

Correct Answer -

Max U Lorenz म य लोरज

Gulf Streams are the currents of which of the following oceans

खाड़ी की धाराए िनिलखत महासागरो म स िकसकी धाराए ह

1 North Atlantic Ocean उरी अटलािटक महासागर

2 North Pacific Ocean उरी शात महासागर

3 Arabian Sea अरब सागर

4 South Pacific Ocean दिण शात महासागर

Correct Answer -

North Atlantic Ocean उरी अटलािटक महासागर

Disintegration wearing away and removal of rock material is generally referred as

िशला पदाथ (रॉक सामी) का टटना िमटना और हटना आमतौर पर ________ क प म सदिभत िकया जाता ह

1 Shattering िवसकारक

2 Denudation अनाादन

3 Fault श

4 Decomposition िवयोजन

Correct Answer -

Denudation अनाादन

Variations in the length of day time and night from season to season are due to

मौसम स मौसम परवतन पर िदन क समय और रात क समय की अविध म िभताए िन कारण स होती ह

1 The Earthrsquos revolution round the Sun in an elliptical manner पी का दीघवाकार तरीक स सय क चारो घणन

2 The Earthrsquos rotation on its axis पी का इसकी धरी पर घणन

3 Revolution of the Earth on a tilted axis नत अ पर पी का घणन

4 Latitudinal position of the place थान की अाश थित

Correct Answer -

Revolution of the Earth on a tilted axis नत अ पर पी का घणन

38)

39)

40)

Point out the correct sequence of mountain ranges from north to south

उर स दिण तक पवत खलाओ क सही अनम को इिगत कर

1 Great Himalaya Middle Himalaya Outer Himalaya Trans Himalaya

महान िहमालय म िहमालय बा िहमालय परा िहमालय

2 Middle Himalaya Great Himalaya Trans Himalaya Outer Himalaya

म िहमालय महान िहमालय परा िहमालय बा िहमालय

3 Outer Himalaya Middle Himalaya Great Himalaya Trans Himalaya

बा िहमालय म िहमालय महान िहमालय परा िहमालय

4 Trans Himalaya Great Himalaya Middle Himalaya Outer Himalaya

परा िहमालय महान िहमालय म िहमालय बा िहमालय

Correct Answer -

Trans Himalaya Great Himalaya Middle Himalaya Outer Himalaya

परा िहमालय महान िहमालय म िहमालय बा िहमालय

Sunrsquos halo is produced by the refraction of light in सय का भामडल ______ म काश क अपवतन ाराउ होता ह

1 Ice crystals in Cirrro-Cumulus clouds पाभ-कपास मघो क बफ िल

2 Ice crystal in Cirrus clouds पाभमघो क बफ िल

3 Dust particles in Stratus clouds री मघो क धल कण

4 Water vapour in Stratus clouds री मघो क जल वा

Correct Answer -

Ice crystal in Cirrus clouds पाभमघो क बफ िल

Read the given statements and answer which of the following options isare correct

(1) The minerals present in the rocks exposed to atmosphere are not subjected to alteration

(2) Oxidation is one of the processes of chemical weathering

िदए गए कथनो को पढ़ और उर द िक िन म स कौन सास िवक सही ह

(1) वायमल स अनावत शल म उपथत खिनज परवतन क अधीन नही होता ह

(2) ऑीकरण रासायिनक अपय की ियाओ म स एक ह

ो ो

41)

42)

1 Both statements are wrong दोनो कथन गलत ह

2 Both statements are correct दोनो कथन सही ह

3 First statement is wrong and second statement is correct पहला कथन गलत ह और दसरा कथन सही ह

4 First statement is correct and second statement is wrong पहला कथन सही ह और दसरा कथन गलत ह

Correct Answer -

First statement is wrong and second statement is correct पहला कथन गलत ह और दसरा कथन सही ह

Read the given statements and answer which of the following options isare correct

1 Sunrsquos short waves enter the earth partially heating the atmosphere

2 Heated earth surface from the sun produces broader waves which interacts and heats the atmosphere

िदए गए कथन को पढ़ और उर द िक िन म स कौन सास िवक सही ह

1 सय की छोटी तरग पी म आिशक प स वश करती ह और वायमडल को ऊत करती ह

2 सय स ऊत पी की सतह िवारत तरग उ करती ह जो परर भाव डालती ह और वायमडल कोऊत करती ह

1 Both Statements 1 and 2 are correct दोनो कथन 1 और 2 सही ह

2 Both Statements 1 and 2 are wrong दोनो कथन 1 और 2 गलत ह

3 Statement 1 is wrong and only Statement 2 is correct कथन 1 गलत ह और कवल कथन 2 सही ह

4 Only statement 1 is correct कवल कथन 1 सही ह

Correct Answer -

Both Statements 1 and 2 are correct दोनो कथन 1 और 2 सही ह

Read the given statements and answer which of the following options isare correct

(1)The rocks that get changed due to heat and pressure are termed as metamorphic rocks

(2)Slate is one such type of metamorphic rock

िदए गए कथनो को पढ़ और उर द िक िन म स कौन सास िवक सही ह

(1) शल जो ऊा और दाब क कारण परवितत हो जात ह उ कायातरक शलो क प म जाना जाता ह

(2) ट एक तरह का कायातरक शल ह

1 Both statements are wrong दोनो कथन गलत ह

2 Both statements are correct दोनो कथन सही ह

3 First statement is wrong and second statement is correct पहला कथन गलत ह और दसरा कथन सही ह

ी औ

43)

44)

4 First statement is correct and second statement is wrong पहला कथन सही ह और दसरा कथन गलत ह

Correct Answer -

Both statements are correct दोनो कथन सही ह

Read the given statements and answer which of the following options isare correct

1 Higher temperature anomaly is observed in the northern hemisphere

2 Differential heating is absent in Northern Hemisphere

िदए गए कथनो को पढ़ और उर द िक िन म स कौन सास िवक सही ह

1 उरी गोलाध म उ तापमान िवसगित पायी जाती ह

2 उरी गोलाध म अतर ऊन अनपथत होती ह

1 Both Statements 1 and 2 are correct दोनो कथन 1 और 2 सही ह

2 Both Statements 1 and 2 are wrong दोनो कथन 1 और 2 गलत ह

3 Statement 1 is wrong and Statement 2 is correct कथन 1 गलत ह और कथन 2 सही ह

4 Statement 1 is correct and Statement 2 is wrong कथन 1 सही ह और कथन 2 गलत ह

Correct Answer -

Statement 1 is correct and Statement 2 is wrong कथन 1 सही ह और कथन 2 गलत ह

Read the given statements and answer which of the following options isare correct

(1) Plutonic rocks are intrusive type of igneous rocks

(2) It cools very slowly because the surrounding rock serves as insulation around the intrusion of magma

िदए गए कथनो को पढ़ और उर द िक िन म स कौन सास िवक सही ह

(1) िवतलीय शल अतवधी कार क आश शल ह

(2) यह बत धीर-धीर ठडा होता ह ोिक आस-पास क शल मा क अतवधन क चारो ओर रोधन क प म कायकरत ह

1 Both statements are wrong दोनो कथन गलत ह

2 Both statements are correct दोनो कथन सही ह

3 First statement is wrong and second statement is correct पहला कथन गलत ह और दसरा कथन सही ह

4 First statement is correct and second statement is wrong पहला कथन सही ह और दसरा कथन गलत ह

Correct Answer -

Both statements are correct दोनो कथन सही ह

45)

46)

47)

48)

The dust and ash material hurled from the volcanoes are termed as

ालामखी स िनकलन वाली धल और राख सामी को _______ क प म कहा जाता ह

1 Pyroclasc पाइरोाक

2 Hyperclastic हाइपराक

3 Hepiroclastic हिपरोाक

4 Cirroclastic िसरोाक

Correct Answer -

Pyroclasc पाइरोाक

The vertical difference in elevation between a low tide and high tide is referred as

कम ार और उ ार क बीच ऊचाई म लबवत अतर _____ स सदिभत होता ह

1 Tidal slope ारीय ढलान

2 Tidal elevation ारीय उयन

3 Tidal range ारीय परास

4 Tidal height ारीय ऊचाई

Correct Answer -

Tidal range ारीय परास

The maximum biodiversity is found in which of the following regions िनिलखत ो म स अिधकतमजव िविवधता िकसम पायी जाती ह

1 Amazon Basin अमज़न बिसन

2 East Indies ई इडीज

3 Congo Basin कागो बिसन

4 West indies व इडीज

Correct Answer -

Amazon Basin अमज़न बिसन

The cultivation of rice crop produces_______ चावल की फसल की खती ______ का उादन करती ह

1 SO2

49)

50)

51)

2 CH4

3 CFCs

4 CO2

Correct Answer -

CH4

The pressure system with higher pressure at the centre is called__________

क म उ दबाव वाली दबाव णाली को _______ कहा जाता ह

1 front अ

2 depression अवनमन

3 cyclone चवात

4 anti-cyclone ितचवात

Correct Answer -

anti-cyclone ितचवात

The Himalayan region is poor in mineral resources because िहमालयी खिनज ससाधनो म समनही ह ोिक

1 The displacement of rock strata has disturbed the arrangement of rocks and made it complex

शलीय परत क िवथापन न चानो की वथा को अवथत कर िदया ह और इस जिटल बना िदया ह

2 The climate conditions are not suitable for exploitation of minerals

जलवाय की थित खिनजो क दोहन क िलए उपय नही ह

3 The terrain makes explanation of minerals difficult and very costly due to transportation difficulties

भ-भाग परवहन की किठनाइयो क कारण खिनजो का दोहन मल और बत महगा बना दता ह

4 It is made up of crystalline rocks यह िलीय चानो स बना ह

Correct Answer -

The displacement of rock strata has disturbed the arrangement of rocks and made it complex

शलीय परत क िवथापन न चानो की वथा को अवथत कर िदया ह और इस जिटल बना िदया ह

The process through which the moisture is added to the atmosphere by vegetation is termed as

वह िया िजसक माम स वनित ारा वातावरण म नमी िमलायी जाती ह _______ क प म जानी जाती ह

52)

53)

54)

1 Condensation सघनन

2 Evapotranspiration वान-उजन

3 Radiation िविकरण

4 Precipitation वषण

Correct Answer -

Evapotranspiration वान-उजन

The process through which the terrestrial heat is transferred to air by direct contact is termed as

वह िया िजसम सपक ारा थलीय ऊा वाय म थानातरत हो जाती ह ______ क प म जानी जाती ह

1 Conduction चालन

2 Convection सवहन

3 Insolation आतपन

4 Radiation िविकरण

Correct Answer -

Conduction चालन

The largest area under mangroves is in which of the following statesunion territory

मोव क अतगत िनिलखत राोसघ शािसत दशो म स सबस बड़ा कौन सा ह

1 Andaman and Nicobar अमान और िनकोबार

2 Andhra Pradesh आ दश

3 West Bengal पिम बगाल

4 Gujarat गजरात

Correct Answer -

West Bengal पिम बगाल

The longitudinal transverse and surface waves in an earthquake originate from

भकप म दशातर अनथ और सतह तरग यहा उ होती ह

1 The focus on the surface of the Earth पी क सतह पर क -िबद म

2 The focus within the body of the Earth पी क भीतर क -िबद म

3 The epicenter within the body of the Earth पी क भीतर उपरक म

55)

56)

57)

4 The epicenter on the surface of the Earth पी क सतह पर उपरक म

Correct Answer -

The focus within the body of the Earth पी क भीतर क -िबद म

The down slope movement of material due to gravity is called______

गाकषण क कारण पदाथ की अनढाल गित को ______ कहा जाता ह

1 mass movement पदाथ सचलन

2 deposition िनप

3 erosion रण

4 volcanic movement ालामखीय सचलन

Correct Answer -

mass movement पदाथ सचलन

Shimla is cooler than Amritsar although both are on the same latitude This is because

िशमला म अमतसर स अिधक ठड ह हालािक दोनो समान अाश पर ह ऐसा ह ोिक

1 Shimla is at a greater height above sea level than Amritsar अमतसर की तलना म िशमला सम तल स अिधकऊचाई पर ह

2 Shimla is further north िशमला उर की ओर ह

3 Shimla is farther from the equator िशमला भम रखा स आग ह

4 Their longitudes differ उनकी दशातर रखाए िभ ह

Correct Answer -

Shimla is at a greater height above sea level than Amritsar अमतसर की तलना म िशमला सम तल स अिधकऊचाई पर ह

lsquoTempo of Urbanizationrsquo measures which of the following

lsquoशहरीकरण का टपोrsquo िनिलखत म स कौन सा उपाय ह

1 Speed of urbanizaon शहरीकरण की गित

2 None of the above इनम स कोई नही

3 Inequality of urbanizaon शहरीकरण की असमानता

4 Current level of urbanizaon शहरीकरण का वतमान र

Correct Answer -

58)

59)

60)

Speed of urbanizaon शहरीकरण की गित

Out of the following options choose the INCORRECT statement

िनिलखत िवको म स गलत कथन का चयन कर

1 The clear tracts in the equatorial region recover rapidly भम रखा म भभाग तजी स ठीक हो जात ह

2 The stable communities include a redwood forest a pine forest at high elevations

थर समदायो म एक रडवड वन उ ऊचाई पर एक दवदार वन शािमल ह

3 Any ecosystem moves towards maximum biomass and stability to survive

कोई भी पारथितकी त जीिवत रहन क िलए अिधकतम जवसहित और थरता की तरफ असर होता ह

4 Tropical rain forests near equator are stable ecosystems

भम रखा क पास उकिटबधीय वषा वन थर पारथितक त ह

Correct Answer -

The clear tracts in the equatorial region recover rapidly भम रखा म भभाग तजी स ठीक हो जात ह

Seasonal contrasts are maximum in मौसमी िवषमता अिधकतम ह

1 Mid latitudes म अाश म

2 Low attitudes िन अाश म

3 High latitudes उ अाश म

4 Subtropics उपोकिटबधीय म

Correct Answer -

Mid latitudes म अाश म

In India which type of forest among the following occupies the largest area

भारत म िनिलखत म स िकस कार क वन सबस बड़ा फल आािदत करत ह

1 Sub-tropical Dry Evergreen Forest उप उकिटबधीय श सदाबहार वन

2 Mountain Wet Temperate Forest पवतीय आ शीतो वन

3 Tropical Moist Deciduous Forest उकिटबधीय आ पणपाती वन

4 Tropical Wet Evergreen Forest उकिटबधीय आ सदाबहार वन

Correct Answer -

Tropical Moist Deciduous Forest उकिटबधीय आ पणपाती वन

61)

62)

63)

64)

What is the proportion of lsquoJuvenile Populationrsquo (0-14 years) in India as per 2011Census

2011 की जनगणना क अनसार भारत म जवनाइल पॉपलशन यानी िकशोर जनस या (0-14 वष) का अनपात ाह

1 3076 of total population कल जनस या का 3076

2 2764 of total population कल जनस या का 2764

3 2933 of total population कल जनस या का 2933

4 3354 of total population कल जनस या का 3354

Correct Answer -

3076 of total population कल जनस या का 3076

What is the Belfast famous for बलफा िकसक िलए मशर ह

1 Belt of cotton textile industry कपास व उोग क

2 Ship-building industry जहाज िनमाण उोग

3 Agricultural machinery किष उपकरण

4 Aero planes manufacturing वाययान िनमाण

Correct Answer -

Ship-building industry जहाज िनमाण उोग

What is the most important occupation in tropical monsoon lands

उकिटबधीय मॉनसन भिम म सबस महपण वसाय ा ह

1 Mining खनन

2 Cattle rearing मवशी पालन

3 Agriculture किष

4 Nomadic herding नोमािडक जड़ी-बिटया

Correct Answer -

Agriculture किष

What is the most important characteristics of the islands (Indian) located in the Arabian sea

अरब सागर म थत ीपो (भारतीय) की सबस महपण िवशषता ा ह

ी ो

65)

66)

67)

1 There are all of coral origins सभी कोरल मल क ह

2 There are all very small in size य सभी आकार म बत छोट ह

3 They have a very dry climate इनकी जलवाय बत श ह

4 They are extended parts of the mainland व महाीप क िवारत िह ह

Correct Answer -

There are all of coral origins सभी कोरल मल क ह

What do the basalt layers of the Deccan indicate डन की बसा परत ा इिगत करती ह

1 All of the above उपरो सभी

2 Huge volcanic eruptions in the distant past दरथ अतीत म िवशाल ालामखीय िवोट

3 The immense erosional activity of the rivers निदयो की िवशाल रण गितिविध

4 The influence of weathering मौसम का भाव

Correct Answer -

Huge volcanic eruptions in the distant past दरथ अतीत म िवशाल ालामखीय िवोट

In the structure of planet Earth below the mantle the core is mainly made up of_____

पी ह की सरचना म मटल क नीच कोर म प स______ स िनिमत होती ह

1 aluminium एमीिनयम

2 silicon िसिलकॉन

3 chromium ोिमयम

4 iron लोहा

Correct Answer -

iron लोहा

One of the major Mid Oceanic Ridge is found in मख म-महासागर चोिटयो म स एक ______ म पायाजाता ह

1 Mid Pacific Ocean म शात महासागर

2 Mid Atlantic Ocean म अटलािटक महासागर

3 Mid Indian Ocean म भारतीय महासागर

4 Mid Arctic Ocean म आक िटक महासागर

68)

69)

70)

71)

Correct Answer -

Mid Atlantic Ocean म अटलािटक महासागर

Magma that reaches the Earthrsquos surface and then solidifies is called________

मा जो पी की सतह तक पचती ह और िफर ठोस हो जाती ह ________कहलाती ह

1 quartz ाटज

2 lava लावा

3 granite नाइट

4 silicates िसिलकट

Correct Answer -

lava लावा

Isotherms are the lines of equal_______ समताप रखाए समान _______की रखाए होती ह

1 pressure दाब

2 temperature तापमान

3 rainfall वषा

4 height ऊचाई

Correct Answer -

temperature तापमान

Mark the correct sequence of passes in the Western Ghats from north to south

पिमी घाटो म उर स दिण तक दर क सही अनम को िचित कर

1 Thalghat Palghat Bhorghat थलगघाट पालघाट भोरघाट

2 Thalghat Bhorghat Palghat थलघाट भोरघाट पालघाट

3 Bhorghat Thalghat Palghat भोरघाट थलघाट पालघाट

4 Palghat Bhorghat Thalghat पालघाट भोरघाट थलघाट

Correct Answer -

Thalghat Bhorghat Palghat थलघाट भोरघाट पालघाट

Which of the following does not have influence over the climate in India

ि ि ि ी ी

72)

73)

िनिलखत म स िकसका भाव भारत की जलवाय पर नही पड़ता ह

1 Ocean currents सागर की लहर

2 Nearness to equator भम रखा स िनकटता

3 Monsoons मानसन

4 Presence of Indian ocean भारतीय महासागर की उपथित

Correct Answer -

Ocean currents सागर की लहर

Which of the following cloud types has the characteristics like vertical tall narrow and puffy

िनिलखत म स िकस कार क मघो म लबवत लबी सकीण और थलता जसी िवशषताए ह

1 Cumulonimbus तफानी मघ

2 Cumulus मघ पज

3 Cirrocumulus पाभ कपासी मघ

4 Nimbostratus वषारी मघ

Correct Answer -

Cumulus मघ पज

Which of the following statement is INCORRECT about Crude Birth Rate

िनिलखत स कौन सा कथन अशोिधत ज दर क बार म सही नही ह

1 It cannot be used for comparing fertility level between two countries with different population characteristics

इसका उपयोग िविभ जनसा िवशषताओ वाल दो दशो क बीच जनन र की तलना क िलए नही िकया जा सकता ह

2 It is a standardized measure of fertility

यह जनन मता का मानकीकत उपाय ह

3 It is effected by the age-sex composition of the population

यह आबादी की आय-िलग सरचना स भािवत होता ह

4 It is expressed per 1000 population in a given geographical unit

यह िकसी दी गई भौगोिलक इकाई म ित 1000 जनसा पर िकया जाता ह

Correct Answer -

It is a standardized measure of fertility

यह जनन मता का मानकीकत उपाय ह

74)

75)

76)

77)

Which of the following state in India experienced negative decadal growth rate during 2001 to 2011census

भारत म िनिलखत म स िकस रा म वष 2001 स 2011 की जनगणना क दौरान नकाराक िगरावट दर ई

1 Tripura िपरा

2 Nagaland नागालड

3 Haryana हरयाणा

4 Odisha ओिडसा

Correct Answer -

Nagaland नागालड

Which of the following is NOT a characteristic of peninsular rivers

िनिलखत म स कौन सी िवशषता ायीपीय निदयो म नही होती ह

1 Flow through shallow valleys उथल घािटयो क माम स वाह

2 Seasonal flow मौसमी वाह

3 Little erosional activity थोड़ी कटावदार गितिविध

4 Meandering tendency often shifting their beds घमावदार वि अर अपन तटो को थानातरत करना

Correct Answer -

Meandering tendency often shifting their beds घमावदार वि अर अपन तटो को थानातरत करना

Which of the following gases in the atmosphere absorbs heat from the Sunrsquos radiation and the Earthssurface

वायमडल म िनिलखत म स कौन सी गस सय क िविकरण और पी की सतह स ऊा को अवशोिषत करती ह

1 Neon िनयॉन

2 Carbon dioxide काबन डाइऑाइड

3 Argon आगन

4 Nitrogen नाइट ोजन

Correct Answer -

Carbon dioxide काबन डाइऑाइड

Which of the following kind of settlement pattern is found at the confluence of rivers

ि ि ि ि ो

78)

79)

80)

िनिलखत म स िकस कार का वथापन पटन निदयो क सगम पर पाया जाता ह

1 Triangular Paern िकोणीय पटन

2 Circular or Semi-Circular Paern परप या अध-परप पटन

3 Nebular Paern नबलर पटन

4 Star ndashShaped Paern ार-आकार का पटन

Correct Answer -

Triangular Paern िकोणीय पटन

Which one was not the objective of the Biosphere Reserve Projects launched by the UNESCO

यनो ारा श की गई सरित जवमडल परयोजनाओ का उ इनम स कौन सा नही था

1 To promote teaching and research िशण और अनसधान को बढ़ावा दना

2 To make agriculture sustainable किष को दीघकािलक बनाना

3 To conserve ecosystems पारथितक त को सरित करना

4 To conserve genetic diversity for a longtime लब समय तक अनवािशक िविवधता को सरित करना

Correct Answer -

To make agriculture sustainable किष को दीघकािलक बनाना

Which region of the Earth surface receives the highest amount of insulation

पी सतह का कौन सा तापावरोधन की उतम माा ा करता ह

1 Land mass थलखड

2 Savannah region सवाना

3 Water bodies जल िनकाय

4 Tropical desert उकिटबधीय रिगान

Correct Answer -

Tropical desert उकिटबधीय रिगान

Which one of the following is not a biodiversity hotspot

िनिलखत म स कौन सा जव िविवधता का म जगह नही ह

1 Eastern Himalaya पव िहमालय

2 Eastern Ghats पव घाट

81)

82)

83)

3 Indo-Myanmar भारत-ामार

4 Westerm Ghats पिमी घाट

Correct Answer -

Eastern Ghats पव घाट

Which one of the following is NOT a part of the World Network of Biosphere Reserves based on theUNESCO Man and Biosphere Programme

यनो मन और बायोीयर कायम क आधार पर िनिलखत म स कौन बायोीयर रजव क िव नटवक कािहा नही ह

1 Gulf of Mannar मार की खाड़ी

2 Seshachalam शषाचलम

3 Sunderban सदरबन

4 Nilgiri नीलिगर

Correct Answer -

Seshachalam शषाचलम

Which one of the following is an example of ldquodesert vegetationrdquo

िनिलखत म स कौन मथलीय वनित का एक उदाहरण ह

1 Mosses and lichens दलदल और शवाल

2 Temperate grassland समशीतो घास क मदान

3 Coniferous forest शकधारी वन

4 Acacia and cactus एकािसया और कस

Correct Answer -

Acacia and cactus एकािसया और कस

Which one of the following reflects more sunlight िनिलखत म स कौन सा सय की रोशनी को अिधकपरावितत करता ह

1 Paddy crop land धान फसल भिम

2 Land covered with fresh snow ताजा बफ स आािदत भिम

3 Sand desert रतीली रिगान

4 Prairie land यरी भिम

84)

85)

86)

87)

Correct Answer -

Land covered with fresh snow ताजा बफ स आािदत भिम

Which layer of the atmosphere is in contact with the surface of the earthrsquos oceans

वायमडल की कौन सी परत पी क महासागरो की सतह क सपक म ह

1 Stratosphere समताप मडल

2 Mesosphere म मडल

3 Hydrosphere जलमडल

4 Troposphere ोभ मडल

Correct Answer -

Troposphere ोभ मडल

Mediterranean Sea is a border of which of the following countries भम सागर िनिलखत दशो म सिकसकी सीमा ह

1 None of these इनम स कोई नही

2 Iraq इराक

3 Lebanon लबनान

4 Jordan जॉडन

Correct Answer -

Lebanon लबनान

Benguela ocean currents are found along which coast बगएला महासागर धाराए िकस तट क साथ पायीजाती ह

1 East Coast of South America दिण अमरका क पव तट

2 East Coast of Africa अीका क पव तट

3 West Coast of South America दिण अमरका क पिमी तट

4 West Coast of Africa अीका क पिमी तट

Correct Answer -

West Coast of Africa अीका क पिमी तट

88)

89)

90)

Due to tension a block of land on one side being pushed up or upthrown relative to the downthrown blockis referred as

तनाव क कारण नीच फ क ए खड क साप भिम का एक खड एक ओर स ऊपर धकला जाता ह या ऊपर की ओरफ का जाता ह यह _____ क प म सदिभत ह

1 Thrust fault प श

2 Normal fault सामा श

3 Reverse fault म श

4 Strike slip fault नितलब सपण श

Correct Answer -

Normal fault सामा श

Inter-tropical doldrums is a zone of ______ अतर-उकिटबधीय डोलड ______ का एक ह

1 Frontolysis टोलायिसस

2 Convergence अिभसरण

3 Inter-tropical divergence zone अतर-उकिटबधीय िवचलन

4 Local wind थानीय वाय

Correct Answer -

Convergence अिभसरण

The Horse Latitudes are regions located at about _____ north and south of the equator

हॉस अाश भम रखा क उर और दिण म लगभग _____ पर थत ह

1 30ndash60 degree Latitude 30-60 िडी अाश

2 0ndash5 degree Latitude 0-5 िडी अाश

3 30 degree Latitude 30 िडी अाश

4 60ndash90 degree Latitude 60-90 िडी अाश

Correct Answer -

30 degree Latitude 30 िडी अाश

Generally evaporation is high over which part of the Earth

आम तौर पर पी क िकस भाग पर वाीकरण अिधक होता ह

1 Equatorial maritime भमवत समीय ी ी

91)

92)

2 Equatorial continental भमवत महाीपीय

3 Polar maritime वीय समीय

4 Polar continental वीय महाीपीय

Correct Answer -

Equatorial maritime भमवत समीय

A very high temperature during summer in north western India leads to what type of climaticcondition in south

उर पिमी भारत म गम क दौरान बत अिधक तापमान होन क कारण दिण म िकस कार की जलवाय थितउ करता ह

1 Depression over arabian sea अरब सागर पर अवनमन

2 Failure monsoon मानसन िवफलता

3 Successful monsoon मानसन सफलता

4 Cyclones चवात

Correct Answer -

Successful monsoon मानसन सफलता

Lightning and thunder are the resultant effect when तिड़त और गजन परणामी भाव ह जब

1 Two massive clouds hit powerfully each other first lightning is produced and later sound is produced

दो बड़ बादल एक दसर स शशाली ढग स टकरात ह पहल आकाशीय िवदयत उ होता ह और बाद म िन उहोती ह

2 Two massive clouds come into contact with the powerful wind collision this results into first sound and thenlightning

दो बड़ बादल शशाली पवन सघ क सपक म आत ह इसका परणामप पहल िन और िफर आकाशीय िवदयतउ होता ह

3 None of the above उपरो म स कोई भी नही

4 A high density cloud contains positively and negatively charged electric ions and when this interacts light andsound are simultaneously produced

एक उ घन बादल म धनाक और ऋणाक आविशत िवदयत आयन होत ह और जब यह परर भाव डालत ह तोकाश और िन एक साथ उािदत होती ह

Correct Answer -

A high density cloud contains positively and negatively charged electric ions and when this interacts light andsound are simultaneously produced

औ ि ि ो औ ो

93)

94)

95)

एक उ घन बादल म धनाक और ऋणाक आविशत िवदयत आयन होत ह और जब यह परर भाव डालत ह तोकाश और िन एक साथ उािदत होती ह

Doon Valley is able to grow rice because दन घाटी चावल उगान म सम ह ोिक

1 Other crops cannot be grown वहा अ फसलो को उगाया नही जा सकता ह

2 People in the valley are rice eaters घाटी म लोग चावल खान वाल ह

3 There is a huge export demand of rice वहा चावल की भारी िनयात माग ह

4 It has warm summer and snow melt waters for irrigation

वहा गिमया गम होती ह िसचाई क िलए बफ का िपघला आ पानी होता ह

Correct Answer -

It has warm summer and snow melt waters for irrigation

वहा गिमया गम होती ह िसचाई क िलए बफ का िपघला आ पानी होता ह

CANCELLED

In the geological time scale the Mesozoic Era DOES NOT contains which of the following periods

भगभय समय पमान पर मजीवी यग म िन कालो म स कौन नही ह

1 Triassic ट ाइऐिसक

2 Jurassic जरिसक

3 Cretaceous चाकमय

4 Carboniferous काबनी

Correct Answer -

Carboniferous काबनी

96)

1 P-3 Q-4 R-2 S-1

2 P-3 Q-4 R-1 S-2

3 P-3 Q-4 R-1 S-2

4 P-4 Q-3 R-2 S-1

Correct Answer -

P-4 Q-3 R-2 S-1

1 P-3 Q-1 R-4 S-2

2 P-3 Q-4 R-1 S-2

3 P-3 Q-2 R-4 S-1

97)

98)

4 P-2 Q-1 R-4 S-3

Correct Answer -

P-3 Q-4 R-1 S-2

1 P-3 Q-1 R-4 S-2

2 P-2 Q-3 R-4 S-1

3 P-2 Q-1 R-3 S-4

4 P-4 Q-2 R-1 S-3

Correct Answer -

P-3 Q-1 R-4 S-2

99)

100)

1 P-3 Q-2 R-4 S-1

2 P-1 Q-2 R-3 S-4

3 P-2 Q-3 R-1 S-4

4 P-4 Q-3 R-2 S-1

Correct Answer -

P-2 Q-3 R-1 S-4

ldquoHuman geography is the study of changing relationship between the unresting man and the unstableearthrdquo was defined by

lsquolsquoमानव भगोल ाकल आदमी और अथर पी क बीच सबध परवतन का अयन हrdquo ______ ारा परभािषत िकया गयाथा

1 J Brunches ज चस

2 EC Semple ईसी सल

3 HJ Mackinder एच ज मिकदर

4 PV Blache पीवी च

Correct Answer -

EC Semple ईसी सल

Sedimentary rocks are finally and ultimately derived from the____________

अवसादी चान अततः ________ स ा की जाती ह

1 action of earth movements पी की गितिविधयो

2 marine deposit समी िनप

3 weathering of metamorphic rocks पातरत चानो क अपय

4 weathering of igneous rocks आय चानो क अपय

Correct Answer -

weathering of igneous rocks आय चानो क अपय

Page 28: High School Teacher Eligibility Test- BOARD PROFESSIONAL ...peb.mp.gov.in/results/RESULT_18/HST_RES18/Final_anwser_key/HST… · M a ndl a / मंड ल ... Under the Madhya Pradesh

35)

36)

37)

4 Max U Lorenz म य लोरज

Correct Answer -

Max U Lorenz म य लोरज

Gulf Streams are the currents of which of the following oceans

खाड़ी की धाराए िनिलखत महासागरो म स िकसकी धाराए ह

1 North Atlantic Ocean उरी अटलािटक महासागर

2 North Pacific Ocean उरी शात महासागर

3 Arabian Sea अरब सागर

4 South Pacific Ocean दिण शात महासागर

Correct Answer -

North Atlantic Ocean उरी अटलािटक महासागर

Disintegration wearing away and removal of rock material is generally referred as

िशला पदाथ (रॉक सामी) का टटना िमटना और हटना आमतौर पर ________ क प म सदिभत िकया जाता ह

1 Shattering िवसकारक

2 Denudation अनाादन

3 Fault श

4 Decomposition िवयोजन

Correct Answer -

Denudation अनाादन

Variations in the length of day time and night from season to season are due to

मौसम स मौसम परवतन पर िदन क समय और रात क समय की अविध म िभताए िन कारण स होती ह

1 The Earthrsquos revolution round the Sun in an elliptical manner पी का दीघवाकार तरीक स सय क चारो घणन

2 The Earthrsquos rotation on its axis पी का इसकी धरी पर घणन

3 Revolution of the Earth on a tilted axis नत अ पर पी का घणन

4 Latitudinal position of the place थान की अाश थित

Correct Answer -

Revolution of the Earth on a tilted axis नत अ पर पी का घणन

38)

39)

40)

Point out the correct sequence of mountain ranges from north to south

उर स दिण तक पवत खलाओ क सही अनम को इिगत कर

1 Great Himalaya Middle Himalaya Outer Himalaya Trans Himalaya

महान िहमालय म िहमालय बा िहमालय परा िहमालय

2 Middle Himalaya Great Himalaya Trans Himalaya Outer Himalaya

म िहमालय महान िहमालय परा िहमालय बा िहमालय

3 Outer Himalaya Middle Himalaya Great Himalaya Trans Himalaya

बा िहमालय म िहमालय महान िहमालय परा िहमालय

4 Trans Himalaya Great Himalaya Middle Himalaya Outer Himalaya

परा िहमालय महान िहमालय म िहमालय बा िहमालय

Correct Answer -

Trans Himalaya Great Himalaya Middle Himalaya Outer Himalaya

परा िहमालय महान िहमालय म िहमालय बा िहमालय

Sunrsquos halo is produced by the refraction of light in सय का भामडल ______ म काश क अपवतन ाराउ होता ह

1 Ice crystals in Cirrro-Cumulus clouds पाभ-कपास मघो क बफ िल

2 Ice crystal in Cirrus clouds पाभमघो क बफ िल

3 Dust particles in Stratus clouds री मघो क धल कण

4 Water vapour in Stratus clouds री मघो क जल वा

Correct Answer -

Ice crystal in Cirrus clouds पाभमघो क बफ िल

Read the given statements and answer which of the following options isare correct

(1) The minerals present in the rocks exposed to atmosphere are not subjected to alteration

(2) Oxidation is one of the processes of chemical weathering

िदए गए कथनो को पढ़ और उर द िक िन म स कौन सास िवक सही ह

(1) वायमल स अनावत शल म उपथत खिनज परवतन क अधीन नही होता ह

(2) ऑीकरण रासायिनक अपय की ियाओ म स एक ह

ो ो

41)

42)

1 Both statements are wrong दोनो कथन गलत ह

2 Both statements are correct दोनो कथन सही ह

3 First statement is wrong and second statement is correct पहला कथन गलत ह और दसरा कथन सही ह

4 First statement is correct and second statement is wrong पहला कथन सही ह और दसरा कथन गलत ह

Correct Answer -

First statement is wrong and second statement is correct पहला कथन गलत ह और दसरा कथन सही ह

Read the given statements and answer which of the following options isare correct

1 Sunrsquos short waves enter the earth partially heating the atmosphere

2 Heated earth surface from the sun produces broader waves which interacts and heats the atmosphere

िदए गए कथन को पढ़ और उर द िक िन म स कौन सास िवक सही ह

1 सय की छोटी तरग पी म आिशक प स वश करती ह और वायमडल को ऊत करती ह

2 सय स ऊत पी की सतह िवारत तरग उ करती ह जो परर भाव डालती ह और वायमडल कोऊत करती ह

1 Both Statements 1 and 2 are correct दोनो कथन 1 और 2 सही ह

2 Both Statements 1 and 2 are wrong दोनो कथन 1 और 2 गलत ह

3 Statement 1 is wrong and only Statement 2 is correct कथन 1 गलत ह और कवल कथन 2 सही ह

4 Only statement 1 is correct कवल कथन 1 सही ह

Correct Answer -

Both Statements 1 and 2 are correct दोनो कथन 1 और 2 सही ह

Read the given statements and answer which of the following options isare correct

(1)The rocks that get changed due to heat and pressure are termed as metamorphic rocks

(2)Slate is one such type of metamorphic rock

िदए गए कथनो को पढ़ और उर द िक िन म स कौन सास िवक सही ह

(1) शल जो ऊा और दाब क कारण परवितत हो जात ह उ कायातरक शलो क प म जाना जाता ह

(2) ट एक तरह का कायातरक शल ह

1 Both statements are wrong दोनो कथन गलत ह

2 Both statements are correct दोनो कथन सही ह

3 First statement is wrong and second statement is correct पहला कथन गलत ह और दसरा कथन सही ह

ी औ

43)

44)

4 First statement is correct and second statement is wrong पहला कथन सही ह और दसरा कथन गलत ह

Correct Answer -

Both statements are correct दोनो कथन सही ह

Read the given statements and answer which of the following options isare correct

1 Higher temperature anomaly is observed in the northern hemisphere

2 Differential heating is absent in Northern Hemisphere

िदए गए कथनो को पढ़ और उर द िक िन म स कौन सास िवक सही ह

1 उरी गोलाध म उ तापमान िवसगित पायी जाती ह

2 उरी गोलाध म अतर ऊन अनपथत होती ह

1 Both Statements 1 and 2 are correct दोनो कथन 1 और 2 सही ह

2 Both Statements 1 and 2 are wrong दोनो कथन 1 और 2 गलत ह

3 Statement 1 is wrong and Statement 2 is correct कथन 1 गलत ह और कथन 2 सही ह

4 Statement 1 is correct and Statement 2 is wrong कथन 1 सही ह और कथन 2 गलत ह

Correct Answer -

Statement 1 is correct and Statement 2 is wrong कथन 1 सही ह और कथन 2 गलत ह

Read the given statements and answer which of the following options isare correct

(1) Plutonic rocks are intrusive type of igneous rocks

(2) It cools very slowly because the surrounding rock serves as insulation around the intrusion of magma

िदए गए कथनो को पढ़ और उर द िक िन म स कौन सास िवक सही ह

(1) िवतलीय शल अतवधी कार क आश शल ह

(2) यह बत धीर-धीर ठडा होता ह ोिक आस-पास क शल मा क अतवधन क चारो ओर रोधन क प म कायकरत ह

1 Both statements are wrong दोनो कथन गलत ह

2 Both statements are correct दोनो कथन सही ह

3 First statement is wrong and second statement is correct पहला कथन गलत ह और दसरा कथन सही ह

4 First statement is correct and second statement is wrong पहला कथन सही ह और दसरा कथन गलत ह

Correct Answer -

Both statements are correct दोनो कथन सही ह

45)

46)

47)

48)

The dust and ash material hurled from the volcanoes are termed as

ालामखी स िनकलन वाली धल और राख सामी को _______ क प म कहा जाता ह

1 Pyroclasc पाइरोाक

2 Hyperclastic हाइपराक

3 Hepiroclastic हिपरोाक

4 Cirroclastic िसरोाक

Correct Answer -

Pyroclasc पाइरोाक

The vertical difference in elevation between a low tide and high tide is referred as

कम ार और उ ार क बीच ऊचाई म लबवत अतर _____ स सदिभत होता ह

1 Tidal slope ारीय ढलान

2 Tidal elevation ारीय उयन

3 Tidal range ारीय परास

4 Tidal height ारीय ऊचाई

Correct Answer -

Tidal range ारीय परास

The maximum biodiversity is found in which of the following regions िनिलखत ो म स अिधकतमजव िविवधता िकसम पायी जाती ह

1 Amazon Basin अमज़न बिसन

2 East Indies ई इडीज

3 Congo Basin कागो बिसन

4 West indies व इडीज

Correct Answer -

Amazon Basin अमज़न बिसन

The cultivation of rice crop produces_______ चावल की फसल की खती ______ का उादन करती ह

1 SO2

49)

50)

51)

2 CH4

3 CFCs

4 CO2

Correct Answer -

CH4

The pressure system with higher pressure at the centre is called__________

क म उ दबाव वाली दबाव णाली को _______ कहा जाता ह

1 front अ

2 depression अवनमन

3 cyclone चवात

4 anti-cyclone ितचवात

Correct Answer -

anti-cyclone ितचवात

The Himalayan region is poor in mineral resources because िहमालयी खिनज ससाधनो म समनही ह ोिक

1 The displacement of rock strata has disturbed the arrangement of rocks and made it complex

शलीय परत क िवथापन न चानो की वथा को अवथत कर िदया ह और इस जिटल बना िदया ह

2 The climate conditions are not suitable for exploitation of minerals

जलवाय की थित खिनजो क दोहन क िलए उपय नही ह

3 The terrain makes explanation of minerals difficult and very costly due to transportation difficulties

भ-भाग परवहन की किठनाइयो क कारण खिनजो का दोहन मल और बत महगा बना दता ह

4 It is made up of crystalline rocks यह िलीय चानो स बना ह

Correct Answer -

The displacement of rock strata has disturbed the arrangement of rocks and made it complex

शलीय परत क िवथापन न चानो की वथा को अवथत कर िदया ह और इस जिटल बना िदया ह

The process through which the moisture is added to the atmosphere by vegetation is termed as

वह िया िजसक माम स वनित ारा वातावरण म नमी िमलायी जाती ह _______ क प म जानी जाती ह

52)

53)

54)

1 Condensation सघनन

2 Evapotranspiration वान-उजन

3 Radiation िविकरण

4 Precipitation वषण

Correct Answer -

Evapotranspiration वान-उजन

The process through which the terrestrial heat is transferred to air by direct contact is termed as

वह िया िजसम सपक ारा थलीय ऊा वाय म थानातरत हो जाती ह ______ क प म जानी जाती ह

1 Conduction चालन

2 Convection सवहन

3 Insolation आतपन

4 Radiation िविकरण

Correct Answer -

Conduction चालन

The largest area under mangroves is in which of the following statesunion territory

मोव क अतगत िनिलखत राोसघ शािसत दशो म स सबस बड़ा कौन सा ह

1 Andaman and Nicobar अमान और िनकोबार

2 Andhra Pradesh आ दश

3 West Bengal पिम बगाल

4 Gujarat गजरात

Correct Answer -

West Bengal पिम बगाल

The longitudinal transverse and surface waves in an earthquake originate from

भकप म दशातर अनथ और सतह तरग यहा उ होती ह

1 The focus on the surface of the Earth पी क सतह पर क -िबद म

2 The focus within the body of the Earth पी क भीतर क -िबद म

3 The epicenter within the body of the Earth पी क भीतर उपरक म

55)

56)

57)

4 The epicenter on the surface of the Earth पी क सतह पर उपरक म

Correct Answer -

The focus within the body of the Earth पी क भीतर क -िबद म

The down slope movement of material due to gravity is called______

गाकषण क कारण पदाथ की अनढाल गित को ______ कहा जाता ह

1 mass movement पदाथ सचलन

2 deposition िनप

3 erosion रण

4 volcanic movement ालामखीय सचलन

Correct Answer -

mass movement पदाथ सचलन

Shimla is cooler than Amritsar although both are on the same latitude This is because

िशमला म अमतसर स अिधक ठड ह हालािक दोनो समान अाश पर ह ऐसा ह ोिक

1 Shimla is at a greater height above sea level than Amritsar अमतसर की तलना म िशमला सम तल स अिधकऊचाई पर ह

2 Shimla is further north िशमला उर की ओर ह

3 Shimla is farther from the equator िशमला भम रखा स आग ह

4 Their longitudes differ उनकी दशातर रखाए िभ ह

Correct Answer -

Shimla is at a greater height above sea level than Amritsar अमतसर की तलना म िशमला सम तल स अिधकऊचाई पर ह

lsquoTempo of Urbanizationrsquo measures which of the following

lsquoशहरीकरण का टपोrsquo िनिलखत म स कौन सा उपाय ह

1 Speed of urbanizaon शहरीकरण की गित

2 None of the above इनम स कोई नही

3 Inequality of urbanizaon शहरीकरण की असमानता

4 Current level of urbanizaon शहरीकरण का वतमान र

Correct Answer -

58)

59)

60)

Speed of urbanizaon शहरीकरण की गित

Out of the following options choose the INCORRECT statement

िनिलखत िवको म स गलत कथन का चयन कर

1 The clear tracts in the equatorial region recover rapidly भम रखा म भभाग तजी स ठीक हो जात ह

2 The stable communities include a redwood forest a pine forest at high elevations

थर समदायो म एक रडवड वन उ ऊचाई पर एक दवदार वन शािमल ह

3 Any ecosystem moves towards maximum biomass and stability to survive

कोई भी पारथितकी त जीिवत रहन क िलए अिधकतम जवसहित और थरता की तरफ असर होता ह

4 Tropical rain forests near equator are stable ecosystems

भम रखा क पास उकिटबधीय वषा वन थर पारथितक त ह

Correct Answer -

The clear tracts in the equatorial region recover rapidly भम रखा म भभाग तजी स ठीक हो जात ह

Seasonal contrasts are maximum in मौसमी िवषमता अिधकतम ह

1 Mid latitudes म अाश म

2 Low attitudes िन अाश म

3 High latitudes उ अाश म

4 Subtropics उपोकिटबधीय म

Correct Answer -

Mid latitudes म अाश म

In India which type of forest among the following occupies the largest area

भारत म िनिलखत म स िकस कार क वन सबस बड़ा फल आािदत करत ह

1 Sub-tropical Dry Evergreen Forest उप उकिटबधीय श सदाबहार वन

2 Mountain Wet Temperate Forest पवतीय आ शीतो वन

3 Tropical Moist Deciduous Forest उकिटबधीय आ पणपाती वन

4 Tropical Wet Evergreen Forest उकिटबधीय आ सदाबहार वन

Correct Answer -

Tropical Moist Deciduous Forest उकिटबधीय आ पणपाती वन

61)

62)

63)

64)

What is the proportion of lsquoJuvenile Populationrsquo (0-14 years) in India as per 2011Census

2011 की जनगणना क अनसार भारत म जवनाइल पॉपलशन यानी िकशोर जनस या (0-14 वष) का अनपात ाह

1 3076 of total population कल जनस या का 3076

2 2764 of total population कल जनस या का 2764

3 2933 of total population कल जनस या का 2933

4 3354 of total population कल जनस या का 3354

Correct Answer -

3076 of total population कल जनस या का 3076

What is the Belfast famous for बलफा िकसक िलए मशर ह

1 Belt of cotton textile industry कपास व उोग क

2 Ship-building industry जहाज िनमाण उोग

3 Agricultural machinery किष उपकरण

4 Aero planes manufacturing वाययान िनमाण

Correct Answer -

Ship-building industry जहाज िनमाण उोग

What is the most important occupation in tropical monsoon lands

उकिटबधीय मॉनसन भिम म सबस महपण वसाय ा ह

1 Mining खनन

2 Cattle rearing मवशी पालन

3 Agriculture किष

4 Nomadic herding नोमािडक जड़ी-बिटया

Correct Answer -

Agriculture किष

What is the most important characteristics of the islands (Indian) located in the Arabian sea

अरब सागर म थत ीपो (भारतीय) की सबस महपण िवशषता ा ह

ी ो

65)

66)

67)

1 There are all of coral origins सभी कोरल मल क ह

2 There are all very small in size य सभी आकार म बत छोट ह

3 They have a very dry climate इनकी जलवाय बत श ह

4 They are extended parts of the mainland व महाीप क िवारत िह ह

Correct Answer -

There are all of coral origins सभी कोरल मल क ह

What do the basalt layers of the Deccan indicate डन की बसा परत ा इिगत करती ह

1 All of the above उपरो सभी

2 Huge volcanic eruptions in the distant past दरथ अतीत म िवशाल ालामखीय िवोट

3 The immense erosional activity of the rivers निदयो की िवशाल रण गितिविध

4 The influence of weathering मौसम का भाव

Correct Answer -

Huge volcanic eruptions in the distant past दरथ अतीत म िवशाल ालामखीय िवोट

In the structure of planet Earth below the mantle the core is mainly made up of_____

पी ह की सरचना म मटल क नीच कोर म प स______ स िनिमत होती ह

1 aluminium एमीिनयम

2 silicon िसिलकॉन

3 chromium ोिमयम

4 iron लोहा

Correct Answer -

iron लोहा

One of the major Mid Oceanic Ridge is found in मख म-महासागर चोिटयो म स एक ______ म पायाजाता ह

1 Mid Pacific Ocean म शात महासागर

2 Mid Atlantic Ocean म अटलािटक महासागर

3 Mid Indian Ocean म भारतीय महासागर

4 Mid Arctic Ocean म आक िटक महासागर

68)

69)

70)

71)

Correct Answer -

Mid Atlantic Ocean म अटलािटक महासागर

Magma that reaches the Earthrsquos surface and then solidifies is called________

मा जो पी की सतह तक पचती ह और िफर ठोस हो जाती ह ________कहलाती ह

1 quartz ाटज

2 lava लावा

3 granite नाइट

4 silicates िसिलकट

Correct Answer -

lava लावा

Isotherms are the lines of equal_______ समताप रखाए समान _______की रखाए होती ह

1 pressure दाब

2 temperature तापमान

3 rainfall वषा

4 height ऊचाई

Correct Answer -

temperature तापमान

Mark the correct sequence of passes in the Western Ghats from north to south

पिमी घाटो म उर स दिण तक दर क सही अनम को िचित कर

1 Thalghat Palghat Bhorghat थलगघाट पालघाट भोरघाट

2 Thalghat Bhorghat Palghat थलघाट भोरघाट पालघाट

3 Bhorghat Thalghat Palghat भोरघाट थलघाट पालघाट

4 Palghat Bhorghat Thalghat पालघाट भोरघाट थलघाट

Correct Answer -

Thalghat Bhorghat Palghat थलघाट भोरघाट पालघाट

Which of the following does not have influence over the climate in India

ि ि ि ी ी

72)

73)

िनिलखत म स िकसका भाव भारत की जलवाय पर नही पड़ता ह

1 Ocean currents सागर की लहर

2 Nearness to equator भम रखा स िनकटता

3 Monsoons मानसन

4 Presence of Indian ocean भारतीय महासागर की उपथित

Correct Answer -

Ocean currents सागर की लहर

Which of the following cloud types has the characteristics like vertical tall narrow and puffy

िनिलखत म स िकस कार क मघो म लबवत लबी सकीण और थलता जसी िवशषताए ह

1 Cumulonimbus तफानी मघ

2 Cumulus मघ पज

3 Cirrocumulus पाभ कपासी मघ

4 Nimbostratus वषारी मघ

Correct Answer -

Cumulus मघ पज

Which of the following statement is INCORRECT about Crude Birth Rate

िनिलखत स कौन सा कथन अशोिधत ज दर क बार म सही नही ह

1 It cannot be used for comparing fertility level between two countries with different population characteristics

इसका उपयोग िविभ जनसा िवशषताओ वाल दो दशो क बीच जनन र की तलना क िलए नही िकया जा सकता ह

2 It is a standardized measure of fertility

यह जनन मता का मानकीकत उपाय ह

3 It is effected by the age-sex composition of the population

यह आबादी की आय-िलग सरचना स भािवत होता ह

4 It is expressed per 1000 population in a given geographical unit

यह िकसी दी गई भौगोिलक इकाई म ित 1000 जनसा पर िकया जाता ह

Correct Answer -

It is a standardized measure of fertility

यह जनन मता का मानकीकत उपाय ह

74)

75)

76)

77)

Which of the following state in India experienced negative decadal growth rate during 2001 to 2011census

भारत म िनिलखत म स िकस रा म वष 2001 स 2011 की जनगणना क दौरान नकाराक िगरावट दर ई

1 Tripura िपरा

2 Nagaland नागालड

3 Haryana हरयाणा

4 Odisha ओिडसा

Correct Answer -

Nagaland नागालड

Which of the following is NOT a characteristic of peninsular rivers

िनिलखत म स कौन सी िवशषता ायीपीय निदयो म नही होती ह

1 Flow through shallow valleys उथल घािटयो क माम स वाह

2 Seasonal flow मौसमी वाह

3 Little erosional activity थोड़ी कटावदार गितिविध

4 Meandering tendency often shifting their beds घमावदार वि अर अपन तटो को थानातरत करना

Correct Answer -

Meandering tendency often shifting their beds घमावदार वि अर अपन तटो को थानातरत करना

Which of the following gases in the atmosphere absorbs heat from the Sunrsquos radiation and the Earthssurface

वायमडल म िनिलखत म स कौन सी गस सय क िविकरण और पी की सतह स ऊा को अवशोिषत करती ह

1 Neon िनयॉन

2 Carbon dioxide काबन डाइऑाइड

3 Argon आगन

4 Nitrogen नाइट ोजन

Correct Answer -

Carbon dioxide काबन डाइऑाइड

Which of the following kind of settlement pattern is found at the confluence of rivers

ि ि ि ि ो

78)

79)

80)

िनिलखत म स िकस कार का वथापन पटन निदयो क सगम पर पाया जाता ह

1 Triangular Paern िकोणीय पटन

2 Circular or Semi-Circular Paern परप या अध-परप पटन

3 Nebular Paern नबलर पटन

4 Star ndashShaped Paern ार-आकार का पटन

Correct Answer -

Triangular Paern िकोणीय पटन

Which one was not the objective of the Biosphere Reserve Projects launched by the UNESCO

यनो ारा श की गई सरित जवमडल परयोजनाओ का उ इनम स कौन सा नही था

1 To promote teaching and research िशण और अनसधान को बढ़ावा दना

2 To make agriculture sustainable किष को दीघकािलक बनाना

3 To conserve ecosystems पारथितक त को सरित करना

4 To conserve genetic diversity for a longtime लब समय तक अनवािशक िविवधता को सरित करना

Correct Answer -

To make agriculture sustainable किष को दीघकािलक बनाना

Which region of the Earth surface receives the highest amount of insulation

पी सतह का कौन सा तापावरोधन की उतम माा ा करता ह

1 Land mass थलखड

2 Savannah region सवाना

3 Water bodies जल िनकाय

4 Tropical desert उकिटबधीय रिगान

Correct Answer -

Tropical desert उकिटबधीय रिगान

Which one of the following is not a biodiversity hotspot

िनिलखत म स कौन सा जव िविवधता का म जगह नही ह

1 Eastern Himalaya पव िहमालय

2 Eastern Ghats पव घाट

81)

82)

83)

3 Indo-Myanmar भारत-ामार

4 Westerm Ghats पिमी घाट

Correct Answer -

Eastern Ghats पव घाट

Which one of the following is NOT a part of the World Network of Biosphere Reserves based on theUNESCO Man and Biosphere Programme

यनो मन और बायोीयर कायम क आधार पर िनिलखत म स कौन बायोीयर रजव क िव नटवक कािहा नही ह

1 Gulf of Mannar मार की खाड़ी

2 Seshachalam शषाचलम

3 Sunderban सदरबन

4 Nilgiri नीलिगर

Correct Answer -

Seshachalam शषाचलम

Which one of the following is an example of ldquodesert vegetationrdquo

िनिलखत म स कौन मथलीय वनित का एक उदाहरण ह

1 Mosses and lichens दलदल और शवाल

2 Temperate grassland समशीतो घास क मदान

3 Coniferous forest शकधारी वन

4 Acacia and cactus एकािसया और कस

Correct Answer -

Acacia and cactus एकािसया और कस

Which one of the following reflects more sunlight िनिलखत म स कौन सा सय की रोशनी को अिधकपरावितत करता ह

1 Paddy crop land धान फसल भिम

2 Land covered with fresh snow ताजा बफ स आािदत भिम

3 Sand desert रतीली रिगान

4 Prairie land यरी भिम

84)

85)

86)

87)

Correct Answer -

Land covered with fresh snow ताजा बफ स आािदत भिम

Which layer of the atmosphere is in contact with the surface of the earthrsquos oceans

वायमडल की कौन सी परत पी क महासागरो की सतह क सपक म ह

1 Stratosphere समताप मडल

2 Mesosphere म मडल

3 Hydrosphere जलमडल

4 Troposphere ोभ मडल

Correct Answer -

Troposphere ोभ मडल

Mediterranean Sea is a border of which of the following countries भम सागर िनिलखत दशो म सिकसकी सीमा ह

1 None of these इनम स कोई नही

2 Iraq इराक

3 Lebanon लबनान

4 Jordan जॉडन

Correct Answer -

Lebanon लबनान

Benguela ocean currents are found along which coast बगएला महासागर धाराए िकस तट क साथ पायीजाती ह

1 East Coast of South America दिण अमरका क पव तट

2 East Coast of Africa अीका क पव तट

3 West Coast of South America दिण अमरका क पिमी तट

4 West Coast of Africa अीका क पिमी तट

Correct Answer -

West Coast of Africa अीका क पिमी तट

88)

89)

90)

Due to tension a block of land on one side being pushed up or upthrown relative to the downthrown blockis referred as

तनाव क कारण नीच फ क ए खड क साप भिम का एक खड एक ओर स ऊपर धकला जाता ह या ऊपर की ओरफ का जाता ह यह _____ क प म सदिभत ह

1 Thrust fault प श

2 Normal fault सामा श

3 Reverse fault म श

4 Strike slip fault नितलब सपण श

Correct Answer -

Normal fault सामा श

Inter-tropical doldrums is a zone of ______ अतर-उकिटबधीय डोलड ______ का एक ह

1 Frontolysis टोलायिसस

2 Convergence अिभसरण

3 Inter-tropical divergence zone अतर-उकिटबधीय िवचलन

4 Local wind थानीय वाय

Correct Answer -

Convergence अिभसरण

The Horse Latitudes are regions located at about _____ north and south of the equator

हॉस अाश भम रखा क उर और दिण म लगभग _____ पर थत ह

1 30ndash60 degree Latitude 30-60 िडी अाश

2 0ndash5 degree Latitude 0-5 िडी अाश

3 30 degree Latitude 30 िडी अाश

4 60ndash90 degree Latitude 60-90 िडी अाश

Correct Answer -

30 degree Latitude 30 िडी अाश

Generally evaporation is high over which part of the Earth

आम तौर पर पी क िकस भाग पर वाीकरण अिधक होता ह

1 Equatorial maritime भमवत समीय ी ी

91)

92)

2 Equatorial continental भमवत महाीपीय

3 Polar maritime वीय समीय

4 Polar continental वीय महाीपीय

Correct Answer -

Equatorial maritime भमवत समीय

A very high temperature during summer in north western India leads to what type of climaticcondition in south

उर पिमी भारत म गम क दौरान बत अिधक तापमान होन क कारण दिण म िकस कार की जलवाय थितउ करता ह

1 Depression over arabian sea अरब सागर पर अवनमन

2 Failure monsoon मानसन िवफलता

3 Successful monsoon मानसन सफलता

4 Cyclones चवात

Correct Answer -

Successful monsoon मानसन सफलता

Lightning and thunder are the resultant effect when तिड़त और गजन परणामी भाव ह जब

1 Two massive clouds hit powerfully each other first lightning is produced and later sound is produced

दो बड़ बादल एक दसर स शशाली ढग स टकरात ह पहल आकाशीय िवदयत उ होता ह और बाद म िन उहोती ह

2 Two massive clouds come into contact with the powerful wind collision this results into first sound and thenlightning

दो बड़ बादल शशाली पवन सघ क सपक म आत ह इसका परणामप पहल िन और िफर आकाशीय िवदयतउ होता ह

3 None of the above उपरो म स कोई भी नही

4 A high density cloud contains positively and negatively charged electric ions and when this interacts light andsound are simultaneously produced

एक उ घन बादल म धनाक और ऋणाक आविशत िवदयत आयन होत ह और जब यह परर भाव डालत ह तोकाश और िन एक साथ उािदत होती ह

Correct Answer -

A high density cloud contains positively and negatively charged electric ions and when this interacts light andsound are simultaneously produced

औ ि ि ो औ ो

93)

94)

95)

एक उ घन बादल म धनाक और ऋणाक आविशत िवदयत आयन होत ह और जब यह परर भाव डालत ह तोकाश और िन एक साथ उािदत होती ह

Doon Valley is able to grow rice because दन घाटी चावल उगान म सम ह ोिक

1 Other crops cannot be grown वहा अ फसलो को उगाया नही जा सकता ह

2 People in the valley are rice eaters घाटी म लोग चावल खान वाल ह

3 There is a huge export demand of rice वहा चावल की भारी िनयात माग ह

4 It has warm summer and snow melt waters for irrigation

वहा गिमया गम होती ह िसचाई क िलए बफ का िपघला आ पानी होता ह

Correct Answer -

It has warm summer and snow melt waters for irrigation

वहा गिमया गम होती ह िसचाई क िलए बफ का िपघला आ पानी होता ह

CANCELLED

In the geological time scale the Mesozoic Era DOES NOT contains which of the following periods

भगभय समय पमान पर मजीवी यग म िन कालो म स कौन नही ह

1 Triassic ट ाइऐिसक

2 Jurassic जरिसक

3 Cretaceous चाकमय

4 Carboniferous काबनी

Correct Answer -

Carboniferous काबनी

96)

1 P-3 Q-4 R-2 S-1

2 P-3 Q-4 R-1 S-2

3 P-3 Q-4 R-1 S-2

4 P-4 Q-3 R-2 S-1

Correct Answer -

P-4 Q-3 R-2 S-1

1 P-3 Q-1 R-4 S-2

2 P-3 Q-4 R-1 S-2

3 P-3 Q-2 R-4 S-1

97)

98)

4 P-2 Q-1 R-4 S-3

Correct Answer -

P-3 Q-4 R-1 S-2

1 P-3 Q-1 R-4 S-2

2 P-2 Q-3 R-4 S-1

3 P-2 Q-1 R-3 S-4

4 P-4 Q-2 R-1 S-3

Correct Answer -

P-3 Q-1 R-4 S-2

99)

100)

1 P-3 Q-2 R-4 S-1

2 P-1 Q-2 R-3 S-4

3 P-2 Q-3 R-1 S-4

4 P-4 Q-3 R-2 S-1

Correct Answer -

P-2 Q-3 R-1 S-4

ldquoHuman geography is the study of changing relationship between the unresting man and the unstableearthrdquo was defined by

lsquolsquoमानव भगोल ाकल आदमी और अथर पी क बीच सबध परवतन का अयन हrdquo ______ ारा परभािषत िकया गयाथा

1 J Brunches ज चस

2 EC Semple ईसी सल

3 HJ Mackinder एच ज मिकदर

4 PV Blache पीवी च

Correct Answer -

EC Semple ईसी सल

Sedimentary rocks are finally and ultimately derived from the____________

अवसादी चान अततः ________ स ा की जाती ह

1 action of earth movements पी की गितिविधयो

2 marine deposit समी िनप

3 weathering of metamorphic rocks पातरत चानो क अपय

4 weathering of igneous rocks आय चानो क अपय

Correct Answer -

weathering of igneous rocks आय चानो क अपय

Page 29: High School Teacher Eligibility Test- BOARD PROFESSIONAL ...peb.mp.gov.in/results/RESULT_18/HST_RES18/Final_anwser_key/HST… · M a ndl a / मंड ल ... Under the Madhya Pradesh

38)

39)

40)

Point out the correct sequence of mountain ranges from north to south

उर स दिण तक पवत खलाओ क सही अनम को इिगत कर

1 Great Himalaya Middle Himalaya Outer Himalaya Trans Himalaya

महान िहमालय म िहमालय बा िहमालय परा िहमालय

2 Middle Himalaya Great Himalaya Trans Himalaya Outer Himalaya

म िहमालय महान िहमालय परा िहमालय बा िहमालय

3 Outer Himalaya Middle Himalaya Great Himalaya Trans Himalaya

बा िहमालय म िहमालय महान िहमालय परा िहमालय

4 Trans Himalaya Great Himalaya Middle Himalaya Outer Himalaya

परा िहमालय महान िहमालय म िहमालय बा िहमालय

Correct Answer -

Trans Himalaya Great Himalaya Middle Himalaya Outer Himalaya

परा िहमालय महान िहमालय म िहमालय बा िहमालय

Sunrsquos halo is produced by the refraction of light in सय का भामडल ______ म काश क अपवतन ाराउ होता ह

1 Ice crystals in Cirrro-Cumulus clouds पाभ-कपास मघो क बफ िल

2 Ice crystal in Cirrus clouds पाभमघो क बफ िल

3 Dust particles in Stratus clouds री मघो क धल कण

4 Water vapour in Stratus clouds री मघो क जल वा

Correct Answer -

Ice crystal in Cirrus clouds पाभमघो क बफ िल

Read the given statements and answer which of the following options isare correct

(1) The minerals present in the rocks exposed to atmosphere are not subjected to alteration

(2) Oxidation is one of the processes of chemical weathering

िदए गए कथनो को पढ़ और उर द िक िन म स कौन सास िवक सही ह

(1) वायमल स अनावत शल म उपथत खिनज परवतन क अधीन नही होता ह

(2) ऑीकरण रासायिनक अपय की ियाओ म स एक ह

ो ो

41)

42)

1 Both statements are wrong दोनो कथन गलत ह

2 Both statements are correct दोनो कथन सही ह

3 First statement is wrong and second statement is correct पहला कथन गलत ह और दसरा कथन सही ह

4 First statement is correct and second statement is wrong पहला कथन सही ह और दसरा कथन गलत ह

Correct Answer -

First statement is wrong and second statement is correct पहला कथन गलत ह और दसरा कथन सही ह

Read the given statements and answer which of the following options isare correct

1 Sunrsquos short waves enter the earth partially heating the atmosphere

2 Heated earth surface from the sun produces broader waves which interacts and heats the atmosphere

िदए गए कथन को पढ़ और उर द िक िन म स कौन सास िवक सही ह

1 सय की छोटी तरग पी म आिशक प स वश करती ह और वायमडल को ऊत करती ह

2 सय स ऊत पी की सतह िवारत तरग उ करती ह जो परर भाव डालती ह और वायमडल कोऊत करती ह

1 Both Statements 1 and 2 are correct दोनो कथन 1 और 2 सही ह

2 Both Statements 1 and 2 are wrong दोनो कथन 1 और 2 गलत ह

3 Statement 1 is wrong and only Statement 2 is correct कथन 1 गलत ह और कवल कथन 2 सही ह

4 Only statement 1 is correct कवल कथन 1 सही ह

Correct Answer -

Both Statements 1 and 2 are correct दोनो कथन 1 और 2 सही ह

Read the given statements and answer which of the following options isare correct

(1)The rocks that get changed due to heat and pressure are termed as metamorphic rocks

(2)Slate is one such type of metamorphic rock

िदए गए कथनो को पढ़ और उर द िक िन म स कौन सास िवक सही ह

(1) शल जो ऊा और दाब क कारण परवितत हो जात ह उ कायातरक शलो क प म जाना जाता ह

(2) ट एक तरह का कायातरक शल ह

1 Both statements are wrong दोनो कथन गलत ह

2 Both statements are correct दोनो कथन सही ह

3 First statement is wrong and second statement is correct पहला कथन गलत ह और दसरा कथन सही ह

ी औ

43)

44)

4 First statement is correct and second statement is wrong पहला कथन सही ह और दसरा कथन गलत ह

Correct Answer -

Both statements are correct दोनो कथन सही ह

Read the given statements and answer which of the following options isare correct

1 Higher temperature anomaly is observed in the northern hemisphere

2 Differential heating is absent in Northern Hemisphere

िदए गए कथनो को पढ़ और उर द िक िन म स कौन सास िवक सही ह

1 उरी गोलाध म उ तापमान िवसगित पायी जाती ह

2 उरी गोलाध म अतर ऊन अनपथत होती ह

1 Both Statements 1 and 2 are correct दोनो कथन 1 और 2 सही ह

2 Both Statements 1 and 2 are wrong दोनो कथन 1 और 2 गलत ह

3 Statement 1 is wrong and Statement 2 is correct कथन 1 गलत ह और कथन 2 सही ह

4 Statement 1 is correct and Statement 2 is wrong कथन 1 सही ह और कथन 2 गलत ह

Correct Answer -

Statement 1 is correct and Statement 2 is wrong कथन 1 सही ह और कथन 2 गलत ह

Read the given statements and answer which of the following options isare correct

(1) Plutonic rocks are intrusive type of igneous rocks

(2) It cools very slowly because the surrounding rock serves as insulation around the intrusion of magma

िदए गए कथनो को पढ़ और उर द िक िन म स कौन सास िवक सही ह

(1) िवतलीय शल अतवधी कार क आश शल ह

(2) यह बत धीर-धीर ठडा होता ह ोिक आस-पास क शल मा क अतवधन क चारो ओर रोधन क प म कायकरत ह

1 Both statements are wrong दोनो कथन गलत ह

2 Both statements are correct दोनो कथन सही ह

3 First statement is wrong and second statement is correct पहला कथन गलत ह और दसरा कथन सही ह

4 First statement is correct and second statement is wrong पहला कथन सही ह और दसरा कथन गलत ह

Correct Answer -

Both statements are correct दोनो कथन सही ह

45)

46)

47)

48)

The dust and ash material hurled from the volcanoes are termed as

ालामखी स िनकलन वाली धल और राख सामी को _______ क प म कहा जाता ह

1 Pyroclasc पाइरोाक

2 Hyperclastic हाइपराक

3 Hepiroclastic हिपरोाक

4 Cirroclastic िसरोाक

Correct Answer -

Pyroclasc पाइरोाक

The vertical difference in elevation between a low tide and high tide is referred as

कम ार और उ ार क बीच ऊचाई म लबवत अतर _____ स सदिभत होता ह

1 Tidal slope ारीय ढलान

2 Tidal elevation ारीय उयन

3 Tidal range ारीय परास

4 Tidal height ारीय ऊचाई

Correct Answer -

Tidal range ारीय परास

The maximum biodiversity is found in which of the following regions िनिलखत ो म स अिधकतमजव िविवधता िकसम पायी जाती ह

1 Amazon Basin अमज़न बिसन

2 East Indies ई इडीज

3 Congo Basin कागो बिसन

4 West indies व इडीज

Correct Answer -

Amazon Basin अमज़न बिसन

The cultivation of rice crop produces_______ चावल की फसल की खती ______ का उादन करती ह

1 SO2

49)

50)

51)

2 CH4

3 CFCs

4 CO2

Correct Answer -

CH4

The pressure system with higher pressure at the centre is called__________

क म उ दबाव वाली दबाव णाली को _______ कहा जाता ह

1 front अ

2 depression अवनमन

3 cyclone चवात

4 anti-cyclone ितचवात

Correct Answer -

anti-cyclone ितचवात

The Himalayan region is poor in mineral resources because िहमालयी खिनज ससाधनो म समनही ह ोिक

1 The displacement of rock strata has disturbed the arrangement of rocks and made it complex

शलीय परत क िवथापन न चानो की वथा को अवथत कर िदया ह और इस जिटल बना िदया ह

2 The climate conditions are not suitable for exploitation of minerals

जलवाय की थित खिनजो क दोहन क िलए उपय नही ह

3 The terrain makes explanation of minerals difficult and very costly due to transportation difficulties

भ-भाग परवहन की किठनाइयो क कारण खिनजो का दोहन मल और बत महगा बना दता ह

4 It is made up of crystalline rocks यह िलीय चानो स बना ह

Correct Answer -

The displacement of rock strata has disturbed the arrangement of rocks and made it complex

शलीय परत क िवथापन न चानो की वथा को अवथत कर िदया ह और इस जिटल बना िदया ह

The process through which the moisture is added to the atmosphere by vegetation is termed as

वह िया िजसक माम स वनित ारा वातावरण म नमी िमलायी जाती ह _______ क प म जानी जाती ह

52)

53)

54)

1 Condensation सघनन

2 Evapotranspiration वान-उजन

3 Radiation िविकरण

4 Precipitation वषण

Correct Answer -

Evapotranspiration वान-उजन

The process through which the terrestrial heat is transferred to air by direct contact is termed as

वह िया िजसम सपक ारा थलीय ऊा वाय म थानातरत हो जाती ह ______ क प म जानी जाती ह

1 Conduction चालन

2 Convection सवहन

3 Insolation आतपन

4 Radiation िविकरण

Correct Answer -

Conduction चालन

The largest area under mangroves is in which of the following statesunion territory

मोव क अतगत िनिलखत राोसघ शािसत दशो म स सबस बड़ा कौन सा ह

1 Andaman and Nicobar अमान और िनकोबार

2 Andhra Pradesh आ दश

3 West Bengal पिम बगाल

4 Gujarat गजरात

Correct Answer -

West Bengal पिम बगाल

The longitudinal transverse and surface waves in an earthquake originate from

भकप म दशातर अनथ और सतह तरग यहा उ होती ह

1 The focus on the surface of the Earth पी क सतह पर क -िबद म

2 The focus within the body of the Earth पी क भीतर क -िबद म

3 The epicenter within the body of the Earth पी क भीतर उपरक म

55)

56)

57)

4 The epicenter on the surface of the Earth पी क सतह पर उपरक म

Correct Answer -

The focus within the body of the Earth पी क भीतर क -िबद म

The down slope movement of material due to gravity is called______

गाकषण क कारण पदाथ की अनढाल गित को ______ कहा जाता ह

1 mass movement पदाथ सचलन

2 deposition िनप

3 erosion रण

4 volcanic movement ालामखीय सचलन

Correct Answer -

mass movement पदाथ सचलन

Shimla is cooler than Amritsar although both are on the same latitude This is because

िशमला म अमतसर स अिधक ठड ह हालािक दोनो समान अाश पर ह ऐसा ह ोिक

1 Shimla is at a greater height above sea level than Amritsar अमतसर की तलना म िशमला सम तल स अिधकऊचाई पर ह

2 Shimla is further north िशमला उर की ओर ह

3 Shimla is farther from the equator िशमला भम रखा स आग ह

4 Their longitudes differ उनकी दशातर रखाए िभ ह

Correct Answer -

Shimla is at a greater height above sea level than Amritsar अमतसर की तलना म िशमला सम तल स अिधकऊचाई पर ह

lsquoTempo of Urbanizationrsquo measures which of the following

lsquoशहरीकरण का टपोrsquo िनिलखत म स कौन सा उपाय ह

1 Speed of urbanizaon शहरीकरण की गित

2 None of the above इनम स कोई नही

3 Inequality of urbanizaon शहरीकरण की असमानता

4 Current level of urbanizaon शहरीकरण का वतमान र

Correct Answer -

58)

59)

60)

Speed of urbanizaon शहरीकरण की गित

Out of the following options choose the INCORRECT statement

िनिलखत िवको म स गलत कथन का चयन कर

1 The clear tracts in the equatorial region recover rapidly भम रखा म भभाग तजी स ठीक हो जात ह

2 The stable communities include a redwood forest a pine forest at high elevations

थर समदायो म एक रडवड वन उ ऊचाई पर एक दवदार वन शािमल ह

3 Any ecosystem moves towards maximum biomass and stability to survive

कोई भी पारथितकी त जीिवत रहन क िलए अिधकतम जवसहित और थरता की तरफ असर होता ह

4 Tropical rain forests near equator are stable ecosystems

भम रखा क पास उकिटबधीय वषा वन थर पारथितक त ह

Correct Answer -

The clear tracts in the equatorial region recover rapidly भम रखा म भभाग तजी स ठीक हो जात ह

Seasonal contrasts are maximum in मौसमी िवषमता अिधकतम ह

1 Mid latitudes म अाश म

2 Low attitudes िन अाश म

3 High latitudes उ अाश म

4 Subtropics उपोकिटबधीय म

Correct Answer -

Mid latitudes म अाश म

In India which type of forest among the following occupies the largest area

भारत म िनिलखत म स िकस कार क वन सबस बड़ा फल आािदत करत ह

1 Sub-tropical Dry Evergreen Forest उप उकिटबधीय श सदाबहार वन

2 Mountain Wet Temperate Forest पवतीय आ शीतो वन

3 Tropical Moist Deciduous Forest उकिटबधीय आ पणपाती वन

4 Tropical Wet Evergreen Forest उकिटबधीय आ सदाबहार वन

Correct Answer -

Tropical Moist Deciduous Forest उकिटबधीय आ पणपाती वन

61)

62)

63)

64)

What is the proportion of lsquoJuvenile Populationrsquo (0-14 years) in India as per 2011Census

2011 की जनगणना क अनसार भारत म जवनाइल पॉपलशन यानी िकशोर जनस या (0-14 वष) का अनपात ाह

1 3076 of total population कल जनस या का 3076

2 2764 of total population कल जनस या का 2764

3 2933 of total population कल जनस या का 2933

4 3354 of total population कल जनस या का 3354

Correct Answer -

3076 of total population कल जनस या का 3076

What is the Belfast famous for बलफा िकसक िलए मशर ह

1 Belt of cotton textile industry कपास व उोग क

2 Ship-building industry जहाज िनमाण उोग

3 Agricultural machinery किष उपकरण

4 Aero planes manufacturing वाययान िनमाण

Correct Answer -

Ship-building industry जहाज िनमाण उोग

What is the most important occupation in tropical monsoon lands

उकिटबधीय मॉनसन भिम म सबस महपण वसाय ा ह

1 Mining खनन

2 Cattle rearing मवशी पालन

3 Agriculture किष

4 Nomadic herding नोमािडक जड़ी-बिटया

Correct Answer -

Agriculture किष

What is the most important characteristics of the islands (Indian) located in the Arabian sea

अरब सागर म थत ीपो (भारतीय) की सबस महपण िवशषता ा ह

ी ो

65)

66)

67)

1 There are all of coral origins सभी कोरल मल क ह

2 There are all very small in size य सभी आकार म बत छोट ह

3 They have a very dry climate इनकी जलवाय बत श ह

4 They are extended parts of the mainland व महाीप क िवारत िह ह

Correct Answer -

There are all of coral origins सभी कोरल मल क ह

What do the basalt layers of the Deccan indicate डन की बसा परत ा इिगत करती ह

1 All of the above उपरो सभी

2 Huge volcanic eruptions in the distant past दरथ अतीत म िवशाल ालामखीय िवोट

3 The immense erosional activity of the rivers निदयो की िवशाल रण गितिविध

4 The influence of weathering मौसम का भाव

Correct Answer -

Huge volcanic eruptions in the distant past दरथ अतीत म िवशाल ालामखीय िवोट

In the structure of planet Earth below the mantle the core is mainly made up of_____

पी ह की सरचना म मटल क नीच कोर म प स______ स िनिमत होती ह

1 aluminium एमीिनयम

2 silicon िसिलकॉन

3 chromium ोिमयम

4 iron लोहा

Correct Answer -

iron लोहा

One of the major Mid Oceanic Ridge is found in मख म-महासागर चोिटयो म स एक ______ म पायाजाता ह

1 Mid Pacific Ocean म शात महासागर

2 Mid Atlantic Ocean म अटलािटक महासागर

3 Mid Indian Ocean म भारतीय महासागर

4 Mid Arctic Ocean म आक िटक महासागर

68)

69)

70)

71)

Correct Answer -

Mid Atlantic Ocean म अटलािटक महासागर

Magma that reaches the Earthrsquos surface and then solidifies is called________

मा जो पी की सतह तक पचती ह और िफर ठोस हो जाती ह ________कहलाती ह

1 quartz ाटज

2 lava लावा

3 granite नाइट

4 silicates िसिलकट

Correct Answer -

lava लावा

Isotherms are the lines of equal_______ समताप रखाए समान _______की रखाए होती ह

1 pressure दाब

2 temperature तापमान

3 rainfall वषा

4 height ऊचाई

Correct Answer -

temperature तापमान

Mark the correct sequence of passes in the Western Ghats from north to south

पिमी घाटो म उर स दिण तक दर क सही अनम को िचित कर

1 Thalghat Palghat Bhorghat थलगघाट पालघाट भोरघाट

2 Thalghat Bhorghat Palghat थलघाट भोरघाट पालघाट

3 Bhorghat Thalghat Palghat भोरघाट थलघाट पालघाट

4 Palghat Bhorghat Thalghat पालघाट भोरघाट थलघाट

Correct Answer -

Thalghat Bhorghat Palghat थलघाट भोरघाट पालघाट

Which of the following does not have influence over the climate in India

ि ि ि ी ी

72)

73)

िनिलखत म स िकसका भाव भारत की जलवाय पर नही पड़ता ह

1 Ocean currents सागर की लहर

2 Nearness to equator भम रखा स िनकटता

3 Monsoons मानसन

4 Presence of Indian ocean भारतीय महासागर की उपथित

Correct Answer -

Ocean currents सागर की लहर

Which of the following cloud types has the characteristics like vertical tall narrow and puffy

िनिलखत म स िकस कार क मघो म लबवत लबी सकीण और थलता जसी िवशषताए ह

1 Cumulonimbus तफानी मघ

2 Cumulus मघ पज

3 Cirrocumulus पाभ कपासी मघ

4 Nimbostratus वषारी मघ

Correct Answer -

Cumulus मघ पज

Which of the following statement is INCORRECT about Crude Birth Rate

िनिलखत स कौन सा कथन अशोिधत ज दर क बार म सही नही ह

1 It cannot be used for comparing fertility level between two countries with different population characteristics

इसका उपयोग िविभ जनसा िवशषताओ वाल दो दशो क बीच जनन र की तलना क िलए नही िकया जा सकता ह

2 It is a standardized measure of fertility

यह जनन मता का मानकीकत उपाय ह

3 It is effected by the age-sex composition of the population

यह आबादी की आय-िलग सरचना स भािवत होता ह

4 It is expressed per 1000 population in a given geographical unit

यह िकसी दी गई भौगोिलक इकाई म ित 1000 जनसा पर िकया जाता ह

Correct Answer -

It is a standardized measure of fertility

यह जनन मता का मानकीकत उपाय ह

74)

75)

76)

77)

Which of the following state in India experienced negative decadal growth rate during 2001 to 2011census

भारत म िनिलखत म स िकस रा म वष 2001 स 2011 की जनगणना क दौरान नकाराक िगरावट दर ई

1 Tripura िपरा

2 Nagaland नागालड

3 Haryana हरयाणा

4 Odisha ओिडसा

Correct Answer -

Nagaland नागालड

Which of the following is NOT a characteristic of peninsular rivers

िनिलखत म स कौन सी िवशषता ायीपीय निदयो म नही होती ह

1 Flow through shallow valleys उथल घािटयो क माम स वाह

2 Seasonal flow मौसमी वाह

3 Little erosional activity थोड़ी कटावदार गितिविध

4 Meandering tendency often shifting their beds घमावदार वि अर अपन तटो को थानातरत करना

Correct Answer -

Meandering tendency often shifting their beds घमावदार वि अर अपन तटो को थानातरत करना

Which of the following gases in the atmosphere absorbs heat from the Sunrsquos radiation and the Earthssurface

वायमडल म िनिलखत म स कौन सी गस सय क िविकरण और पी की सतह स ऊा को अवशोिषत करती ह

1 Neon िनयॉन

2 Carbon dioxide काबन डाइऑाइड

3 Argon आगन

4 Nitrogen नाइट ोजन

Correct Answer -

Carbon dioxide काबन डाइऑाइड

Which of the following kind of settlement pattern is found at the confluence of rivers

ि ि ि ि ो

78)

79)

80)

िनिलखत म स िकस कार का वथापन पटन निदयो क सगम पर पाया जाता ह

1 Triangular Paern िकोणीय पटन

2 Circular or Semi-Circular Paern परप या अध-परप पटन

3 Nebular Paern नबलर पटन

4 Star ndashShaped Paern ार-आकार का पटन

Correct Answer -

Triangular Paern िकोणीय पटन

Which one was not the objective of the Biosphere Reserve Projects launched by the UNESCO

यनो ारा श की गई सरित जवमडल परयोजनाओ का उ इनम स कौन सा नही था

1 To promote teaching and research िशण और अनसधान को बढ़ावा दना

2 To make agriculture sustainable किष को दीघकािलक बनाना

3 To conserve ecosystems पारथितक त को सरित करना

4 To conserve genetic diversity for a longtime लब समय तक अनवािशक िविवधता को सरित करना

Correct Answer -

To make agriculture sustainable किष को दीघकािलक बनाना

Which region of the Earth surface receives the highest amount of insulation

पी सतह का कौन सा तापावरोधन की उतम माा ा करता ह

1 Land mass थलखड

2 Savannah region सवाना

3 Water bodies जल िनकाय

4 Tropical desert उकिटबधीय रिगान

Correct Answer -

Tropical desert उकिटबधीय रिगान

Which one of the following is not a biodiversity hotspot

िनिलखत म स कौन सा जव िविवधता का म जगह नही ह

1 Eastern Himalaya पव िहमालय

2 Eastern Ghats पव घाट

81)

82)

83)

3 Indo-Myanmar भारत-ामार

4 Westerm Ghats पिमी घाट

Correct Answer -

Eastern Ghats पव घाट

Which one of the following is NOT a part of the World Network of Biosphere Reserves based on theUNESCO Man and Biosphere Programme

यनो मन और बायोीयर कायम क आधार पर िनिलखत म स कौन बायोीयर रजव क िव नटवक कािहा नही ह

1 Gulf of Mannar मार की खाड़ी

2 Seshachalam शषाचलम

3 Sunderban सदरबन

4 Nilgiri नीलिगर

Correct Answer -

Seshachalam शषाचलम

Which one of the following is an example of ldquodesert vegetationrdquo

िनिलखत म स कौन मथलीय वनित का एक उदाहरण ह

1 Mosses and lichens दलदल और शवाल

2 Temperate grassland समशीतो घास क मदान

3 Coniferous forest शकधारी वन

4 Acacia and cactus एकािसया और कस

Correct Answer -

Acacia and cactus एकािसया और कस

Which one of the following reflects more sunlight िनिलखत म स कौन सा सय की रोशनी को अिधकपरावितत करता ह

1 Paddy crop land धान फसल भिम

2 Land covered with fresh snow ताजा बफ स आािदत भिम

3 Sand desert रतीली रिगान

4 Prairie land यरी भिम

84)

85)

86)

87)

Correct Answer -

Land covered with fresh snow ताजा बफ स आािदत भिम

Which layer of the atmosphere is in contact with the surface of the earthrsquos oceans

वायमडल की कौन सी परत पी क महासागरो की सतह क सपक म ह

1 Stratosphere समताप मडल

2 Mesosphere म मडल

3 Hydrosphere जलमडल

4 Troposphere ोभ मडल

Correct Answer -

Troposphere ोभ मडल

Mediterranean Sea is a border of which of the following countries भम सागर िनिलखत दशो म सिकसकी सीमा ह

1 None of these इनम स कोई नही

2 Iraq इराक

3 Lebanon लबनान

4 Jordan जॉडन

Correct Answer -

Lebanon लबनान

Benguela ocean currents are found along which coast बगएला महासागर धाराए िकस तट क साथ पायीजाती ह

1 East Coast of South America दिण अमरका क पव तट

2 East Coast of Africa अीका क पव तट

3 West Coast of South America दिण अमरका क पिमी तट

4 West Coast of Africa अीका क पिमी तट

Correct Answer -

West Coast of Africa अीका क पिमी तट

88)

89)

90)

Due to tension a block of land on one side being pushed up or upthrown relative to the downthrown blockis referred as

तनाव क कारण नीच फ क ए खड क साप भिम का एक खड एक ओर स ऊपर धकला जाता ह या ऊपर की ओरफ का जाता ह यह _____ क प म सदिभत ह

1 Thrust fault प श

2 Normal fault सामा श

3 Reverse fault म श

4 Strike slip fault नितलब सपण श

Correct Answer -

Normal fault सामा श

Inter-tropical doldrums is a zone of ______ अतर-उकिटबधीय डोलड ______ का एक ह

1 Frontolysis टोलायिसस

2 Convergence अिभसरण

3 Inter-tropical divergence zone अतर-उकिटबधीय िवचलन

4 Local wind थानीय वाय

Correct Answer -

Convergence अिभसरण

The Horse Latitudes are regions located at about _____ north and south of the equator

हॉस अाश भम रखा क उर और दिण म लगभग _____ पर थत ह

1 30ndash60 degree Latitude 30-60 िडी अाश

2 0ndash5 degree Latitude 0-5 िडी अाश

3 30 degree Latitude 30 िडी अाश

4 60ndash90 degree Latitude 60-90 िडी अाश

Correct Answer -

30 degree Latitude 30 िडी अाश

Generally evaporation is high over which part of the Earth

आम तौर पर पी क िकस भाग पर वाीकरण अिधक होता ह

1 Equatorial maritime भमवत समीय ी ी

91)

92)

2 Equatorial continental भमवत महाीपीय

3 Polar maritime वीय समीय

4 Polar continental वीय महाीपीय

Correct Answer -

Equatorial maritime भमवत समीय

A very high temperature during summer in north western India leads to what type of climaticcondition in south

उर पिमी भारत म गम क दौरान बत अिधक तापमान होन क कारण दिण म िकस कार की जलवाय थितउ करता ह

1 Depression over arabian sea अरब सागर पर अवनमन

2 Failure monsoon मानसन िवफलता

3 Successful monsoon मानसन सफलता

4 Cyclones चवात

Correct Answer -

Successful monsoon मानसन सफलता

Lightning and thunder are the resultant effect when तिड़त और गजन परणामी भाव ह जब

1 Two massive clouds hit powerfully each other first lightning is produced and later sound is produced

दो बड़ बादल एक दसर स शशाली ढग स टकरात ह पहल आकाशीय िवदयत उ होता ह और बाद म िन उहोती ह

2 Two massive clouds come into contact with the powerful wind collision this results into first sound and thenlightning

दो बड़ बादल शशाली पवन सघ क सपक म आत ह इसका परणामप पहल िन और िफर आकाशीय िवदयतउ होता ह

3 None of the above उपरो म स कोई भी नही

4 A high density cloud contains positively and negatively charged electric ions and when this interacts light andsound are simultaneously produced

एक उ घन बादल म धनाक और ऋणाक आविशत िवदयत आयन होत ह और जब यह परर भाव डालत ह तोकाश और िन एक साथ उािदत होती ह

Correct Answer -

A high density cloud contains positively and negatively charged electric ions and when this interacts light andsound are simultaneously produced

औ ि ि ो औ ो

93)

94)

95)

एक उ घन बादल म धनाक और ऋणाक आविशत िवदयत आयन होत ह और जब यह परर भाव डालत ह तोकाश और िन एक साथ उािदत होती ह

Doon Valley is able to grow rice because दन घाटी चावल उगान म सम ह ोिक

1 Other crops cannot be grown वहा अ फसलो को उगाया नही जा सकता ह

2 People in the valley are rice eaters घाटी म लोग चावल खान वाल ह

3 There is a huge export demand of rice वहा चावल की भारी िनयात माग ह

4 It has warm summer and snow melt waters for irrigation

वहा गिमया गम होती ह िसचाई क िलए बफ का िपघला आ पानी होता ह

Correct Answer -

It has warm summer and snow melt waters for irrigation

वहा गिमया गम होती ह िसचाई क िलए बफ का िपघला आ पानी होता ह

CANCELLED

In the geological time scale the Mesozoic Era DOES NOT contains which of the following periods

भगभय समय पमान पर मजीवी यग म िन कालो म स कौन नही ह

1 Triassic ट ाइऐिसक

2 Jurassic जरिसक

3 Cretaceous चाकमय

4 Carboniferous काबनी

Correct Answer -

Carboniferous काबनी

96)

1 P-3 Q-4 R-2 S-1

2 P-3 Q-4 R-1 S-2

3 P-3 Q-4 R-1 S-2

4 P-4 Q-3 R-2 S-1

Correct Answer -

P-4 Q-3 R-2 S-1

1 P-3 Q-1 R-4 S-2

2 P-3 Q-4 R-1 S-2

3 P-3 Q-2 R-4 S-1

97)

98)

4 P-2 Q-1 R-4 S-3

Correct Answer -

P-3 Q-4 R-1 S-2

1 P-3 Q-1 R-4 S-2

2 P-2 Q-3 R-4 S-1

3 P-2 Q-1 R-3 S-4

4 P-4 Q-2 R-1 S-3

Correct Answer -

P-3 Q-1 R-4 S-2

99)

100)

1 P-3 Q-2 R-4 S-1

2 P-1 Q-2 R-3 S-4

3 P-2 Q-3 R-1 S-4

4 P-4 Q-3 R-2 S-1

Correct Answer -

P-2 Q-3 R-1 S-4

ldquoHuman geography is the study of changing relationship between the unresting man and the unstableearthrdquo was defined by

lsquolsquoमानव भगोल ाकल आदमी और अथर पी क बीच सबध परवतन का अयन हrdquo ______ ारा परभािषत िकया गयाथा

1 J Brunches ज चस

2 EC Semple ईसी सल

3 HJ Mackinder एच ज मिकदर

4 PV Blache पीवी च

Correct Answer -

EC Semple ईसी सल

Sedimentary rocks are finally and ultimately derived from the____________

अवसादी चान अततः ________ स ा की जाती ह

1 action of earth movements पी की गितिविधयो

2 marine deposit समी िनप

3 weathering of metamorphic rocks पातरत चानो क अपय

4 weathering of igneous rocks आय चानो क अपय

Correct Answer -

weathering of igneous rocks आय चानो क अपय

Page 30: High School Teacher Eligibility Test- BOARD PROFESSIONAL ...peb.mp.gov.in/results/RESULT_18/HST_RES18/Final_anwser_key/HST… · M a ndl a / मंड ल ... Under the Madhya Pradesh

41)

42)

1 Both statements are wrong दोनो कथन गलत ह

2 Both statements are correct दोनो कथन सही ह

3 First statement is wrong and second statement is correct पहला कथन गलत ह और दसरा कथन सही ह

4 First statement is correct and second statement is wrong पहला कथन सही ह और दसरा कथन गलत ह

Correct Answer -

First statement is wrong and second statement is correct पहला कथन गलत ह और दसरा कथन सही ह

Read the given statements and answer which of the following options isare correct

1 Sunrsquos short waves enter the earth partially heating the atmosphere

2 Heated earth surface from the sun produces broader waves which interacts and heats the atmosphere

िदए गए कथन को पढ़ और उर द िक िन म स कौन सास िवक सही ह

1 सय की छोटी तरग पी म आिशक प स वश करती ह और वायमडल को ऊत करती ह

2 सय स ऊत पी की सतह िवारत तरग उ करती ह जो परर भाव डालती ह और वायमडल कोऊत करती ह

1 Both Statements 1 and 2 are correct दोनो कथन 1 और 2 सही ह

2 Both Statements 1 and 2 are wrong दोनो कथन 1 और 2 गलत ह

3 Statement 1 is wrong and only Statement 2 is correct कथन 1 गलत ह और कवल कथन 2 सही ह

4 Only statement 1 is correct कवल कथन 1 सही ह

Correct Answer -

Both Statements 1 and 2 are correct दोनो कथन 1 और 2 सही ह

Read the given statements and answer which of the following options isare correct

(1)The rocks that get changed due to heat and pressure are termed as metamorphic rocks

(2)Slate is one such type of metamorphic rock

िदए गए कथनो को पढ़ और उर द िक िन म स कौन सास िवक सही ह

(1) शल जो ऊा और दाब क कारण परवितत हो जात ह उ कायातरक शलो क प म जाना जाता ह

(2) ट एक तरह का कायातरक शल ह

1 Both statements are wrong दोनो कथन गलत ह

2 Both statements are correct दोनो कथन सही ह

3 First statement is wrong and second statement is correct पहला कथन गलत ह और दसरा कथन सही ह

ी औ

43)

44)

4 First statement is correct and second statement is wrong पहला कथन सही ह और दसरा कथन गलत ह

Correct Answer -

Both statements are correct दोनो कथन सही ह

Read the given statements and answer which of the following options isare correct

1 Higher temperature anomaly is observed in the northern hemisphere

2 Differential heating is absent in Northern Hemisphere

िदए गए कथनो को पढ़ और उर द िक िन म स कौन सास िवक सही ह

1 उरी गोलाध म उ तापमान िवसगित पायी जाती ह

2 उरी गोलाध म अतर ऊन अनपथत होती ह

1 Both Statements 1 and 2 are correct दोनो कथन 1 और 2 सही ह

2 Both Statements 1 and 2 are wrong दोनो कथन 1 और 2 गलत ह

3 Statement 1 is wrong and Statement 2 is correct कथन 1 गलत ह और कथन 2 सही ह

4 Statement 1 is correct and Statement 2 is wrong कथन 1 सही ह और कथन 2 गलत ह

Correct Answer -

Statement 1 is correct and Statement 2 is wrong कथन 1 सही ह और कथन 2 गलत ह

Read the given statements and answer which of the following options isare correct

(1) Plutonic rocks are intrusive type of igneous rocks

(2) It cools very slowly because the surrounding rock serves as insulation around the intrusion of magma

िदए गए कथनो को पढ़ और उर द िक िन म स कौन सास िवक सही ह

(1) िवतलीय शल अतवधी कार क आश शल ह

(2) यह बत धीर-धीर ठडा होता ह ोिक आस-पास क शल मा क अतवधन क चारो ओर रोधन क प म कायकरत ह

1 Both statements are wrong दोनो कथन गलत ह

2 Both statements are correct दोनो कथन सही ह

3 First statement is wrong and second statement is correct पहला कथन गलत ह और दसरा कथन सही ह

4 First statement is correct and second statement is wrong पहला कथन सही ह और दसरा कथन गलत ह

Correct Answer -

Both statements are correct दोनो कथन सही ह

45)

46)

47)

48)

The dust and ash material hurled from the volcanoes are termed as

ालामखी स िनकलन वाली धल और राख सामी को _______ क प म कहा जाता ह

1 Pyroclasc पाइरोाक

2 Hyperclastic हाइपराक

3 Hepiroclastic हिपरोाक

4 Cirroclastic िसरोाक

Correct Answer -

Pyroclasc पाइरोाक

The vertical difference in elevation between a low tide and high tide is referred as

कम ार और उ ार क बीच ऊचाई म लबवत अतर _____ स सदिभत होता ह

1 Tidal slope ारीय ढलान

2 Tidal elevation ारीय उयन

3 Tidal range ारीय परास

4 Tidal height ारीय ऊचाई

Correct Answer -

Tidal range ारीय परास

The maximum biodiversity is found in which of the following regions िनिलखत ो म स अिधकतमजव िविवधता िकसम पायी जाती ह

1 Amazon Basin अमज़न बिसन

2 East Indies ई इडीज

3 Congo Basin कागो बिसन

4 West indies व इडीज

Correct Answer -

Amazon Basin अमज़न बिसन

The cultivation of rice crop produces_______ चावल की फसल की खती ______ का उादन करती ह

1 SO2

49)

50)

51)

2 CH4

3 CFCs

4 CO2

Correct Answer -

CH4

The pressure system with higher pressure at the centre is called__________

क म उ दबाव वाली दबाव णाली को _______ कहा जाता ह

1 front अ

2 depression अवनमन

3 cyclone चवात

4 anti-cyclone ितचवात

Correct Answer -

anti-cyclone ितचवात

The Himalayan region is poor in mineral resources because िहमालयी खिनज ससाधनो म समनही ह ोिक

1 The displacement of rock strata has disturbed the arrangement of rocks and made it complex

शलीय परत क िवथापन न चानो की वथा को अवथत कर िदया ह और इस जिटल बना िदया ह

2 The climate conditions are not suitable for exploitation of minerals

जलवाय की थित खिनजो क दोहन क िलए उपय नही ह

3 The terrain makes explanation of minerals difficult and very costly due to transportation difficulties

भ-भाग परवहन की किठनाइयो क कारण खिनजो का दोहन मल और बत महगा बना दता ह

4 It is made up of crystalline rocks यह िलीय चानो स बना ह

Correct Answer -

The displacement of rock strata has disturbed the arrangement of rocks and made it complex

शलीय परत क िवथापन न चानो की वथा को अवथत कर िदया ह और इस जिटल बना िदया ह

The process through which the moisture is added to the atmosphere by vegetation is termed as

वह िया िजसक माम स वनित ारा वातावरण म नमी िमलायी जाती ह _______ क प म जानी जाती ह

52)

53)

54)

1 Condensation सघनन

2 Evapotranspiration वान-उजन

3 Radiation िविकरण

4 Precipitation वषण

Correct Answer -

Evapotranspiration वान-उजन

The process through which the terrestrial heat is transferred to air by direct contact is termed as

वह िया िजसम सपक ारा थलीय ऊा वाय म थानातरत हो जाती ह ______ क प म जानी जाती ह

1 Conduction चालन

2 Convection सवहन

3 Insolation आतपन

4 Radiation िविकरण

Correct Answer -

Conduction चालन

The largest area under mangroves is in which of the following statesunion territory

मोव क अतगत िनिलखत राोसघ शािसत दशो म स सबस बड़ा कौन सा ह

1 Andaman and Nicobar अमान और िनकोबार

2 Andhra Pradesh आ दश

3 West Bengal पिम बगाल

4 Gujarat गजरात

Correct Answer -

West Bengal पिम बगाल

The longitudinal transverse and surface waves in an earthquake originate from

भकप म दशातर अनथ और सतह तरग यहा उ होती ह

1 The focus on the surface of the Earth पी क सतह पर क -िबद म

2 The focus within the body of the Earth पी क भीतर क -िबद म

3 The epicenter within the body of the Earth पी क भीतर उपरक म

55)

56)

57)

4 The epicenter on the surface of the Earth पी क सतह पर उपरक म

Correct Answer -

The focus within the body of the Earth पी क भीतर क -िबद म

The down slope movement of material due to gravity is called______

गाकषण क कारण पदाथ की अनढाल गित को ______ कहा जाता ह

1 mass movement पदाथ सचलन

2 deposition िनप

3 erosion रण

4 volcanic movement ालामखीय सचलन

Correct Answer -

mass movement पदाथ सचलन

Shimla is cooler than Amritsar although both are on the same latitude This is because

िशमला म अमतसर स अिधक ठड ह हालािक दोनो समान अाश पर ह ऐसा ह ोिक

1 Shimla is at a greater height above sea level than Amritsar अमतसर की तलना म िशमला सम तल स अिधकऊचाई पर ह

2 Shimla is further north िशमला उर की ओर ह

3 Shimla is farther from the equator िशमला भम रखा स आग ह

4 Their longitudes differ उनकी दशातर रखाए िभ ह

Correct Answer -

Shimla is at a greater height above sea level than Amritsar अमतसर की तलना म िशमला सम तल स अिधकऊचाई पर ह

lsquoTempo of Urbanizationrsquo measures which of the following

lsquoशहरीकरण का टपोrsquo िनिलखत म स कौन सा उपाय ह

1 Speed of urbanizaon शहरीकरण की गित

2 None of the above इनम स कोई नही

3 Inequality of urbanizaon शहरीकरण की असमानता

4 Current level of urbanizaon शहरीकरण का वतमान र

Correct Answer -

58)

59)

60)

Speed of urbanizaon शहरीकरण की गित

Out of the following options choose the INCORRECT statement

िनिलखत िवको म स गलत कथन का चयन कर

1 The clear tracts in the equatorial region recover rapidly भम रखा म भभाग तजी स ठीक हो जात ह

2 The stable communities include a redwood forest a pine forest at high elevations

थर समदायो म एक रडवड वन उ ऊचाई पर एक दवदार वन शािमल ह

3 Any ecosystem moves towards maximum biomass and stability to survive

कोई भी पारथितकी त जीिवत रहन क िलए अिधकतम जवसहित और थरता की तरफ असर होता ह

4 Tropical rain forests near equator are stable ecosystems

भम रखा क पास उकिटबधीय वषा वन थर पारथितक त ह

Correct Answer -

The clear tracts in the equatorial region recover rapidly भम रखा म भभाग तजी स ठीक हो जात ह

Seasonal contrasts are maximum in मौसमी िवषमता अिधकतम ह

1 Mid latitudes म अाश म

2 Low attitudes िन अाश म

3 High latitudes उ अाश म

4 Subtropics उपोकिटबधीय म

Correct Answer -

Mid latitudes म अाश म

In India which type of forest among the following occupies the largest area

भारत म िनिलखत म स िकस कार क वन सबस बड़ा फल आािदत करत ह

1 Sub-tropical Dry Evergreen Forest उप उकिटबधीय श सदाबहार वन

2 Mountain Wet Temperate Forest पवतीय आ शीतो वन

3 Tropical Moist Deciduous Forest उकिटबधीय आ पणपाती वन

4 Tropical Wet Evergreen Forest उकिटबधीय आ सदाबहार वन

Correct Answer -

Tropical Moist Deciduous Forest उकिटबधीय आ पणपाती वन

61)

62)

63)

64)

What is the proportion of lsquoJuvenile Populationrsquo (0-14 years) in India as per 2011Census

2011 की जनगणना क अनसार भारत म जवनाइल पॉपलशन यानी िकशोर जनस या (0-14 वष) का अनपात ाह

1 3076 of total population कल जनस या का 3076

2 2764 of total population कल जनस या का 2764

3 2933 of total population कल जनस या का 2933

4 3354 of total population कल जनस या का 3354

Correct Answer -

3076 of total population कल जनस या का 3076

What is the Belfast famous for बलफा िकसक िलए मशर ह

1 Belt of cotton textile industry कपास व उोग क

2 Ship-building industry जहाज िनमाण उोग

3 Agricultural machinery किष उपकरण

4 Aero planes manufacturing वाययान िनमाण

Correct Answer -

Ship-building industry जहाज िनमाण उोग

What is the most important occupation in tropical monsoon lands

उकिटबधीय मॉनसन भिम म सबस महपण वसाय ा ह

1 Mining खनन

2 Cattle rearing मवशी पालन

3 Agriculture किष

4 Nomadic herding नोमािडक जड़ी-बिटया

Correct Answer -

Agriculture किष

What is the most important characteristics of the islands (Indian) located in the Arabian sea

अरब सागर म थत ीपो (भारतीय) की सबस महपण िवशषता ा ह

ी ो

65)

66)

67)

1 There are all of coral origins सभी कोरल मल क ह

2 There are all very small in size य सभी आकार म बत छोट ह

3 They have a very dry climate इनकी जलवाय बत श ह

4 They are extended parts of the mainland व महाीप क िवारत िह ह

Correct Answer -

There are all of coral origins सभी कोरल मल क ह

What do the basalt layers of the Deccan indicate डन की बसा परत ा इिगत करती ह

1 All of the above उपरो सभी

2 Huge volcanic eruptions in the distant past दरथ अतीत म िवशाल ालामखीय िवोट

3 The immense erosional activity of the rivers निदयो की िवशाल रण गितिविध

4 The influence of weathering मौसम का भाव

Correct Answer -

Huge volcanic eruptions in the distant past दरथ अतीत म िवशाल ालामखीय िवोट

In the structure of planet Earth below the mantle the core is mainly made up of_____

पी ह की सरचना म मटल क नीच कोर म प स______ स िनिमत होती ह

1 aluminium एमीिनयम

2 silicon िसिलकॉन

3 chromium ोिमयम

4 iron लोहा

Correct Answer -

iron लोहा

One of the major Mid Oceanic Ridge is found in मख म-महासागर चोिटयो म स एक ______ म पायाजाता ह

1 Mid Pacific Ocean म शात महासागर

2 Mid Atlantic Ocean म अटलािटक महासागर

3 Mid Indian Ocean म भारतीय महासागर

4 Mid Arctic Ocean म आक िटक महासागर

68)

69)

70)

71)

Correct Answer -

Mid Atlantic Ocean म अटलािटक महासागर

Magma that reaches the Earthrsquos surface and then solidifies is called________

मा जो पी की सतह तक पचती ह और िफर ठोस हो जाती ह ________कहलाती ह

1 quartz ाटज

2 lava लावा

3 granite नाइट

4 silicates िसिलकट

Correct Answer -

lava लावा

Isotherms are the lines of equal_______ समताप रखाए समान _______की रखाए होती ह

1 pressure दाब

2 temperature तापमान

3 rainfall वषा

4 height ऊचाई

Correct Answer -

temperature तापमान

Mark the correct sequence of passes in the Western Ghats from north to south

पिमी घाटो म उर स दिण तक दर क सही अनम को िचित कर

1 Thalghat Palghat Bhorghat थलगघाट पालघाट भोरघाट

2 Thalghat Bhorghat Palghat थलघाट भोरघाट पालघाट

3 Bhorghat Thalghat Palghat भोरघाट थलघाट पालघाट

4 Palghat Bhorghat Thalghat पालघाट भोरघाट थलघाट

Correct Answer -

Thalghat Bhorghat Palghat थलघाट भोरघाट पालघाट

Which of the following does not have influence over the climate in India

ि ि ि ी ी

72)

73)

िनिलखत म स िकसका भाव भारत की जलवाय पर नही पड़ता ह

1 Ocean currents सागर की लहर

2 Nearness to equator भम रखा स िनकटता

3 Monsoons मानसन

4 Presence of Indian ocean भारतीय महासागर की उपथित

Correct Answer -

Ocean currents सागर की लहर

Which of the following cloud types has the characteristics like vertical tall narrow and puffy

िनिलखत म स िकस कार क मघो म लबवत लबी सकीण और थलता जसी िवशषताए ह

1 Cumulonimbus तफानी मघ

2 Cumulus मघ पज

3 Cirrocumulus पाभ कपासी मघ

4 Nimbostratus वषारी मघ

Correct Answer -

Cumulus मघ पज

Which of the following statement is INCORRECT about Crude Birth Rate

िनिलखत स कौन सा कथन अशोिधत ज दर क बार म सही नही ह

1 It cannot be used for comparing fertility level between two countries with different population characteristics

इसका उपयोग िविभ जनसा िवशषताओ वाल दो दशो क बीच जनन र की तलना क िलए नही िकया जा सकता ह

2 It is a standardized measure of fertility

यह जनन मता का मानकीकत उपाय ह

3 It is effected by the age-sex composition of the population

यह आबादी की आय-िलग सरचना स भािवत होता ह

4 It is expressed per 1000 population in a given geographical unit

यह िकसी दी गई भौगोिलक इकाई म ित 1000 जनसा पर िकया जाता ह

Correct Answer -

It is a standardized measure of fertility

यह जनन मता का मानकीकत उपाय ह

74)

75)

76)

77)

Which of the following state in India experienced negative decadal growth rate during 2001 to 2011census

भारत म िनिलखत म स िकस रा म वष 2001 स 2011 की जनगणना क दौरान नकाराक िगरावट दर ई

1 Tripura िपरा

2 Nagaland नागालड

3 Haryana हरयाणा

4 Odisha ओिडसा

Correct Answer -

Nagaland नागालड

Which of the following is NOT a characteristic of peninsular rivers

िनिलखत म स कौन सी िवशषता ायीपीय निदयो म नही होती ह

1 Flow through shallow valleys उथल घािटयो क माम स वाह

2 Seasonal flow मौसमी वाह

3 Little erosional activity थोड़ी कटावदार गितिविध

4 Meandering tendency often shifting their beds घमावदार वि अर अपन तटो को थानातरत करना

Correct Answer -

Meandering tendency often shifting their beds घमावदार वि अर अपन तटो को थानातरत करना

Which of the following gases in the atmosphere absorbs heat from the Sunrsquos radiation and the Earthssurface

वायमडल म िनिलखत म स कौन सी गस सय क िविकरण और पी की सतह स ऊा को अवशोिषत करती ह

1 Neon िनयॉन

2 Carbon dioxide काबन डाइऑाइड

3 Argon आगन

4 Nitrogen नाइट ोजन

Correct Answer -

Carbon dioxide काबन डाइऑाइड

Which of the following kind of settlement pattern is found at the confluence of rivers

ि ि ि ि ो

78)

79)

80)

िनिलखत म स िकस कार का वथापन पटन निदयो क सगम पर पाया जाता ह

1 Triangular Paern िकोणीय पटन

2 Circular or Semi-Circular Paern परप या अध-परप पटन

3 Nebular Paern नबलर पटन

4 Star ndashShaped Paern ार-आकार का पटन

Correct Answer -

Triangular Paern िकोणीय पटन

Which one was not the objective of the Biosphere Reserve Projects launched by the UNESCO

यनो ारा श की गई सरित जवमडल परयोजनाओ का उ इनम स कौन सा नही था

1 To promote teaching and research िशण और अनसधान को बढ़ावा दना

2 To make agriculture sustainable किष को दीघकािलक बनाना

3 To conserve ecosystems पारथितक त को सरित करना

4 To conserve genetic diversity for a longtime लब समय तक अनवािशक िविवधता को सरित करना

Correct Answer -

To make agriculture sustainable किष को दीघकािलक बनाना

Which region of the Earth surface receives the highest amount of insulation

पी सतह का कौन सा तापावरोधन की उतम माा ा करता ह

1 Land mass थलखड

2 Savannah region सवाना

3 Water bodies जल िनकाय

4 Tropical desert उकिटबधीय रिगान

Correct Answer -

Tropical desert उकिटबधीय रिगान

Which one of the following is not a biodiversity hotspot

िनिलखत म स कौन सा जव िविवधता का म जगह नही ह

1 Eastern Himalaya पव िहमालय

2 Eastern Ghats पव घाट

81)

82)

83)

3 Indo-Myanmar भारत-ामार

4 Westerm Ghats पिमी घाट

Correct Answer -

Eastern Ghats पव घाट

Which one of the following is NOT a part of the World Network of Biosphere Reserves based on theUNESCO Man and Biosphere Programme

यनो मन और बायोीयर कायम क आधार पर िनिलखत म स कौन बायोीयर रजव क िव नटवक कािहा नही ह

1 Gulf of Mannar मार की खाड़ी

2 Seshachalam शषाचलम

3 Sunderban सदरबन

4 Nilgiri नीलिगर

Correct Answer -

Seshachalam शषाचलम

Which one of the following is an example of ldquodesert vegetationrdquo

िनिलखत म स कौन मथलीय वनित का एक उदाहरण ह

1 Mosses and lichens दलदल और शवाल

2 Temperate grassland समशीतो घास क मदान

3 Coniferous forest शकधारी वन

4 Acacia and cactus एकािसया और कस

Correct Answer -

Acacia and cactus एकािसया और कस

Which one of the following reflects more sunlight िनिलखत म स कौन सा सय की रोशनी को अिधकपरावितत करता ह

1 Paddy crop land धान फसल भिम

2 Land covered with fresh snow ताजा बफ स आािदत भिम

3 Sand desert रतीली रिगान

4 Prairie land यरी भिम

84)

85)

86)

87)

Correct Answer -

Land covered with fresh snow ताजा बफ स आािदत भिम

Which layer of the atmosphere is in contact with the surface of the earthrsquos oceans

वायमडल की कौन सी परत पी क महासागरो की सतह क सपक म ह

1 Stratosphere समताप मडल

2 Mesosphere म मडल

3 Hydrosphere जलमडल

4 Troposphere ोभ मडल

Correct Answer -

Troposphere ोभ मडल

Mediterranean Sea is a border of which of the following countries भम सागर िनिलखत दशो म सिकसकी सीमा ह

1 None of these इनम स कोई नही

2 Iraq इराक

3 Lebanon लबनान

4 Jordan जॉडन

Correct Answer -

Lebanon लबनान

Benguela ocean currents are found along which coast बगएला महासागर धाराए िकस तट क साथ पायीजाती ह

1 East Coast of South America दिण अमरका क पव तट

2 East Coast of Africa अीका क पव तट

3 West Coast of South America दिण अमरका क पिमी तट

4 West Coast of Africa अीका क पिमी तट

Correct Answer -

West Coast of Africa अीका क पिमी तट

88)

89)

90)

Due to tension a block of land on one side being pushed up or upthrown relative to the downthrown blockis referred as

तनाव क कारण नीच फ क ए खड क साप भिम का एक खड एक ओर स ऊपर धकला जाता ह या ऊपर की ओरफ का जाता ह यह _____ क प म सदिभत ह

1 Thrust fault प श

2 Normal fault सामा श

3 Reverse fault म श

4 Strike slip fault नितलब सपण श

Correct Answer -

Normal fault सामा श

Inter-tropical doldrums is a zone of ______ अतर-उकिटबधीय डोलड ______ का एक ह

1 Frontolysis टोलायिसस

2 Convergence अिभसरण

3 Inter-tropical divergence zone अतर-उकिटबधीय िवचलन

4 Local wind थानीय वाय

Correct Answer -

Convergence अिभसरण

The Horse Latitudes are regions located at about _____ north and south of the equator

हॉस अाश भम रखा क उर और दिण म लगभग _____ पर थत ह

1 30ndash60 degree Latitude 30-60 िडी अाश

2 0ndash5 degree Latitude 0-5 िडी अाश

3 30 degree Latitude 30 िडी अाश

4 60ndash90 degree Latitude 60-90 िडी अाश

Correct Answer -

30 degree Latitude 30 िडी अाश

Generally evaporation is high over which part of the Earth

आम तौर पर पी क िकस भाग पर वाीकरण अिधक होता ह

1 Equatorial maritime भमवत समीय ी ी

91)

92)

2 Equatorial continental भमवत महाीपीय

3 Polar maritime वीय समीय

4 Polar continental वीय महाीपीय

Correct Answer -

Equatorial maritime भमवत समीय

A very high temperature during summer in north western India leads to what type of climaticcondition in south

उर पिमी भारत म गम क दौरान बत अिधक तापमान होन क कारण दिण म िकस कार की जलवाय थितउ करता ह

1 Depression over arabian sea अरब सागर पर अवनमन

2 Failure monsoon मानसन िवफलता

3 Successful monsoon मानसन सफलता

4 Cyclones चवात

Correct Answer -

Successful monsoon मानसन सफलता

Lightning and thunder are the resultant effect when तिड़त और गजन परणामी भाव ह जब

1 Two massive clouds hit powerfully each other first lightning is produced and later sound is produced

दो बड़ बादल एक दसर स शशाली ढग स टकरात ह पहल आकाशीय िवदयत उ होता ह और बाद म िन उहोती ह

2 Two massive clouds come into contact with the powerful wind collision this results into first sound and thenlightning

दो बड़ बादल शशाली पवन सघ क सपक म आत ह इसका परणामप पहल िन और िफर आकाशीय िवदयतउ होता ह

3 None of the above उपरो म स कोई भी नही

4 A high density cloud contains positively and negatively charged electric ions and when this interacts light andsound are simultaneously produced

एक उ घन बादल म धनाक और ऋणाक आविशत िवदयत आयन होत ह और जब यह परर भाव डालत ह तोकाश और िन एक साथ उािदत होती ह

Correct Answer -

A high density cloud contains positively and negatively charged electric ions and when this interacts light andsound are simultaneously produced

औ ि ि ो औ ो

93)

94)

95)

एक उ घन बादल म धनाक और ऋणाक आविशत िवदयत आयन होत ह और जब यह परर भाव डालत ह तोकाश और िन एक साथ उािदत होती ह

Doon Valley is able to grow rice because दन घाटी चावल उगान म सम ह ोिक

1 Other crops cannot be grown वहा अ फसलो को उगाया नही जा सकता ह

2 People in the valley are rice eaters घाटी म लोग चावल खान वाल ह

3 There is a huge export demand of rice वहा चावल की भारी िनयात माग ह

4 It has warm summer and snow melt waters for irrigation

वहा गिमया गम होती ह िसचाई क िलए बफ का िपघला आ पानी होता ह

Correct Answer -

It has warm summer and snow melt waters for irrigation

वहा गिमया गम होती ह िसचाई क िलए बफ का िपघला आ पानी होता ह

CANCELLED

In the geological time scale the Mesozoic Era DOES NOT contains which of the following periods

भगभय समय पमान पर मजीवी यग म िन कालो म स कौन नही ह

1 Triassic ट ाइऐिसक

2 Jurassic जरिसक

3 Cretaceous चाकमय

4 Carboniferous काबनी

Correct Answer -

Carboniferous काबनी

96)

1 P-3 Q-4 R-2 S-1

2 P-3 Q-4 R-1 S-2

3 P-3 Q-4 R-1 S-2

4 P-4 Q-3 R-2 S-1

Correct Answer -

P-4 Q-3 R-2 S-1

1 P-3 Q-1 R-4 S-2

2 P-3 Q-4 R-1 S-2

3 P-3 Q-2 R-4 S-1

97)

98)

4 P-2 Q-1 R-4 S-3

Correct Answer -

P-3 Q-4 R-1 S-2

1 P-3 Q-1 R-4 S-2

2 P-2 Q-3 R-4 S-1

3 P-2 Q-1 R-3 S-4

4 P-4 Q-2 R-1 S-3

Correct Answer -

P-3 Q-1 R-4 S-2

99)

100)

1 P-3 Q-2 R-4 S-1

2 P-1 Q-2 R-3 S-4

3 P-2 Q-3 R-1 S-4

4 P-4 Q-3 R-2 S-1

Correct Answer -

P-2 Q-3 R-1 S-4

ldquoHuman geography is the study of changing relationship between the unresting man and the unstableearthrdquo was defined by

lsquolsquoमानव भगोल ाकल आदमी और अथर पी क बीच सबध परवतन का अयन हrdquo ______ ारा परभािषत िकया गयाथा

1 J Brunches ज चस

2 EC Semple ईसी सल

3 HJ Mackinder एच ज मिकदर

4 PV Blache पीवी च

Correct Answer -

EC Semple ईसी सल

Sedimentary rocks are finally and ultimately derived from the____________

अवसादी चान अततः ________ स ा की जाती ह

1 action of earth movements पी की गितिविधयो

2 marine deposit समी िनप

3 weathering of metamorphic rocks पातरत चानो क अपय

4 weathering of igneous rocks आय चानो क अपय

Correct Answer -

weathering of igneous rocks आय चानो क अपय

Page 31: High School Teacher Eligibility Test- BOARD PROFESSIONAL ...peb.mp.gov.in/results/RESULT_18/HST_RES18/Final_anwser_key/HST… · M a ndl a / मंड ल ... Under the Madhya Pradesh

43)

44)

4 First statement is correct and second statement is wrong पहला कथन सही ह और दसरा कथन गलत ह

Correct Answer -

Both statements are correct दोनो कथन सही ह

Read the given statements and answer which of the following options isare correct

1 Higher temperature anomaly is observed in the northern hemisphere

2 Differential heating is absent in Northern Hemisphere

िदए गए कथनो को पढ़ और उर द िक िन म स कौन सास िवक सही ह

1 उरी गोलाध म उ तापमान िवसगित पायी जाती ह

2 उरी गोलाध म अतर ऊन अनपथत होती ह

1 Both Statements 1 and 2 are correct दोनो कथन 1 और 2 सही ह

2 Both Statements 1 and 2 are wrong दोनो कथन 1 और 2 गलत ह

3 Statement 1 is wrong and Statement 2 is correct कथन 1 गलत ह और कथन 2 सही ह

4 Statement 1 is correct and Statement 2 is wrong कथन 1 सही ह और कथन 2 गलत ह

Correct Answer -

Statement 1 is correct and Statement 2 is wrong कथन 1 सही ह और कथन 2 गलत ह

Read the given statements and answer which of the following options isare correct

(1) Plutonic rocks are intrusive type of igneous rocks

(2) It cools very slowly because the surrounding rock serves as insulation around the intrusion of magma

िदए गए कथनो को पढ़ और उर द िक िन म स कौन सास िवक सही ह

(1) िवतलीय शल अतवधी कार क आश शल ह

(2) यह बत धीर-धीर ठडा होता ह ोिक आस-पास क शल मा क अतवधन क चारो ओर रोधन क प म कायकरत ह

1 Both statements are wrong दोनो कथन गलत ह

2 Both statements are correct दोनो कथन सही ह

3 First statement is wrong and second statement is correct पहला कथन गलत ह और दसरा कथन सही ह

4 First statement is correct and second statement is wrong पहला कथन सही ह और दसरा कथन गलत ह

Correct Answer -

Both statements are correct दोनो कथन सही ह

45)

46)

47)

48)

The dust and ash material hurled from the volcanoes are termed as

ालामखी स िनकलन वाली धल और राख सामी को _______ क प म कहा जाता ह

1 Pyroclasc पाइरोाक

2 Hyperclastic हाइपराक

3 Hepiroclastic हिपरोाक

4 Cirroclastic िसरोाक

Correct Answer -

Pyroclasc पाइरोाक

The vertical difference in elevation between a low tide and high tide is referred as

कम ार और उ ार क बीच ऊचाई म लबवत अतर _____ स सदिभत होता ह

1 Tidal slope ारीय ढलान

2 Tidal elevation ारीय उयन

3 Tidal range ारीय परास

4 Tidal height ारीय ऊचाई

Correct Answer -

Tidal range ारीय परास

The maximum biodiversity is found in which of the following regions िनिलखत ो म स अिधकतमजव िविवधता िकसम पायी जाती ह

1 Amazon Basin अमज़न बिसन

2 East Indies ई इडीज

3 Congo Basin कागो बिसन

4 West indies व इडीज

Correct Answer -

Amazon Basin अमज़न बिसन

The cultivation of rice crop produces_______ चावल की फसल की खती ______ का उादन करती ह

1 SO2

49)

50)

51)

2 CH4

3 CFCs

4 CO2

Correct Answer -

CH4

The pressure system with higher pressure at the centre is called__________

क म उ दबाव वाली दबाव णाली को _______ कहा जाता ह

1 front अ

2 depression अवनमन

3 cyclone चवात

4 anti-cyclone ितचवात

Correct Answer -

anti-cyclone ितचवात

The Himalayan region is poor in mineral resources because िहमालयी खिनज ससाधनो म समनही ह ोिक

1 The displacement of rock strata has disturbed the arrangement of rocks and made it complex

शलीय परत क िवथापन न चानो की वथा को अवथत कर िदया ह और इस जिटल बना िदया ह

2 The climate conditions are not suitable for exploitation of minerals

जलवाय की थित खिनजो क दोहन क िलए उपय नही ह

3 The terrain makes explanation of minerals difficult and very costly due to transportation difficulties

भ-भाग परवहन की किठनाइयो क कारण खिनजो का दोहन मल और बत महगा बना दता ह

4 It is made up of crystalline rocks यह िलीय चानो स बना ह

Correct Answer -

The displacement of rock strata has disturbed the arrangement of rocks and made it complex

शलीय परत क िवथापन न चानो की वथा को अवथत कर िदया ह और इस जिटल बना िदया ह

The process through which the moisture is added to the atmosphere by vegetation is termed as

वह िया िजसक माम स वनित ारा वातावरण म नमी िमलायी जाती ह _______ क प म जानी जाती ह

52)

53)

54)

1 Condensation सघनन

2 Evapotranspiration वान-उजन

3 Radiation िविकरण

4 Precipitation वषण

Correct Answer -

Evapotranspiration वान-उजन

The process through which the terrestrial heat is transferred to air by direct contact is termed as

वह िया िजसम सपक ारा थलीय ऊा वाय म थानातरत हो जाती ह ______ क प म जानी जाती ह

1 Conduction चालन

2 Convection सवहन

3 Insolation आतपन

4 Radiation िविकरण

Correct Answer -

Conduction चालन

The largest area under mangroves is in which of the following statesunion territory

मोव क अतगत िनिलखत राोसघ शािसत दशो म स सबस बड़ा कौन सा ह

1 Andaman and Nicobar अमान और िनकोबार

2 Andhra Pradesh आ दश

3 West Bengal पिम बगाल

4 Gujarat गजरात

Correct Answer -

West Bengal पिम बगाल

The longitudinal transverse and surface waves in an earthquake originate from

भकप म दशातर अनथ और सतह तरग यहा उ होती ह

1 The focus on the surface of the Earth पी क सतह पर क -िबद म

2 The focus within the body of the Earth पी क भीतर क -िबद म

3 The epicenter within the body of the Earth पी क भीतर उपरक म

55)

56)

57)

4 The epicenter on the surface of the Earth पी क सतह पर उपरक म

Correct Answer -

The focus within the body of the Earth पी क भीतर क -िबद म

The down slope movement of material due to gravity is called______

गाकषण क कारण पदाथ की अनढाल गित को ______ कहा जाता ह

1 mass movement पदाथ सचलन

2 deposition िनप

3 erosion रण

4 volcanic movement ालामखीय सचलन

Correct Answer -

mass movement पदाथ सचलन

Shimla is cooler than Amritsar although both are on the same latitude This is because

िशमला म अमतसर स अिधक ठड ह हालािक दोनो समान अाश पर ह ऐसा ह ोिक

1 Shimla is at a greater height above sea level than Amritsar अमतसर की तलना म िशमला सम तल स अिधकऊचाई पर ह

2 Shimla is further north िशमला उर की ओर ह

3 Shimla is farther from the equator िशमला भम रखा स आग ह

4 Their longitudes differ उनकी दशातर रखाए िभ ह

Correct Answer -

Shimla is at a greater height above sea level than Amritsar अमतसर की तलना म िशमला सम तल स अिधकऊचाई पर ह

lsquoTempo of Urbanizationrsquo measures which of the following

lsquoशहरीकरण का टपोrsquo िनिलखत म स कौन सा उपाय ह

1 Speed of urbanizaon शहरीकरण की गित

2 None of the above इनम स कोई नही

3 Inequality of urbanizaon शहरीकरण की असमानता

4 Current level of urbanizaon शहरीकरण का वतमान र

Correct Answer -

58)

59)

60)

Speed of urbanizaon शहरीकरण की गित

Out of the following options choose the INCORRECT statement

िनिलखत िवको म स गलत कथन का चयन कर

1 The clear tracts in the equatorial region recover rapidly भम रखा म भभाग तजी स ठीक हो जात ह

2 The stable communities include a redwood forest a pine forest at high elevations

थर समदायो म एक रडवड वन उ ऊचाई पर एक दवदार वन शािमल ह

3 Any ecosystem moves towards maximum biomass and stability to survive

कोई भी पारथितकी त जीिवत रहन क िलए अिधकतम जवसहित और थरता की तरफ असर होता ह

4 Tropical rain forests near equator are stable ecosystems

भम रखा क पास उकिटबधीय वषा वन थर पारथितक त ह

Correct Answer -

The clear tracts in the equatorial region recover rapidly भम रखा म भभाग तजी स ठीक हो जात ह

Seasonal contrasts are maximum in मौसमी िवषमता अिधकतम ह

1 Mid latitudes म अाश म

2 Low attitudes िन अाश म

3 High latitudes उ अाश म

4 Subtropics उपोकिटबधीय म

Correct Answer -

Mid latitudes म अाश म

In India which type of forest among the following occupies the largest area

भारत म िनिलखत म स िकस कार क वन सबस बड़ा फल आािदत करत ह

1 Sub-tropical Dry Evergreen Forest उप उकिटबधीय श सदाबहार वन

2 Mountain Wet Temperate Forest पवतीय आ शीतो वन

3 Tropical Moist Deciduous Forest उकिटबधीय आ पणपाती वन

4 Tropical Wet Evergreen Forest उकिटबधीय आ सदाबहार वन

Correct Answer -

Tropical Moist Deciduous Forest उकिटबधीय आ पणपाती वन

61)

62)

63)

64)

What is the proportion of lsquoJuvenile Populationrsquo (0-14 years) in India as per 2011Census

2011 की जनगणना क अनसार भारत म जवनाइल पॉपलशन यानी िकशोर जनस या (0-14 वष) का अनपात ाह

1 3076 of total population कल जनस या का 3076

2 2764 of total population कल जनस या का 2764

3 2933 of total population कल जनस या का 2933

4 3354 of total population कल जनस या का 3354

Correct Answer -

3076 of total population कल जनस या का 3076

What is the Belfast famous for बलफा िकसक िलए मशर ह

1 Belt of cotton textile industry कपास व उोग क

2 Ship-building industry जहाज िनमाण उोग

3 Agricultural machinery किष उपकरण

4 Aero planes manufacturing वाययान िनमाण

Correct Answer -

Ship-building industry जहाज िनमाण उोग

What is the most important occupation in tropical monsoon lands

उकिटबधीय मॉनसन भिम म सबस महपण वसाय ा ह

1 Mining खनन

2 Cattle rearing मवशी पालन

3 Agriculture किष

4 Nomadic herding नोमािडक जड़ी-बिटया

Correct Answer -

Agriculture किष

What is the most important characteristics of the islands (Indian) located in the Arabian sea

अरब सागर म थत ीपो (भारतीय) की सबस महपण िवशषता ा ह

ी ो

65)

66)

67)

1 There are all of coral origins सभी कोरल मल क ह

2 There are all very small in size य सभी आकार म बत छोट ह

3 They have a very dry climate इनकी जलवाय बत श ह

4 They are extended parts of the mainland व महाीप क िवारत िह ह

Correct Answer -

There are all of coral origins सभी कोरल मल क ह

What do the basalt layers of the Deccan indicate डन की बसा परत ा इिगत करती ह

1 All of the above उपरो सभी

2 Huge volcanic eruptions in the distant past दरथ अतीत म िवशाल ालामखीय िवोट

3 The immense erosional activity of the rivers निदयो की िवशाल रण गितिविध

4 The influence of weathering मौसम का भाव

Correct Answer -

Huge volcanic eruptions in the distant past दरथ अतीत म िवशाल ालामखीय िवोट

In the structure of planet Earth below the mantle the core is mainly made up of_____

पी ह की सरचना म मटल क नीच कोर म प स______ स िनिमत होती ह

1 aluminium एमीिनयम

2 silicon िसिलकॉन

3 chromium ोिमयम

4 iron लोहा

Correct Answer -

iron लोहा

One of the major Mid Oceanic Ridge is found in मख म-महासागर चोिटयो म स एक ______ म पायाजाता ह

1 Mid Pacific Ocean म शात महासागर

2 Mid Atlantic Ocean म अटलािटक महासागर

3 Mid Indian Ocean म भारतीय महासागर

4 Mid Arctic Ocean म आक िटक महासागर

68)

69)

70)

71)

Correct Answer -

Mid Atlantic Ocean म अटलािटक महासागर

Magma that reaches the Earthrsquos surface and then solidifies is called________

मा जो पी की सतह तक पचती ह और िफर ठोस हो जाती ह ________कहलाती ह

1 quartz ाटज

2 lava लावा

3 granite नाइट

4 silicates िसिलकट

Correct Answer -

lava लावा

Isotherms are the lines of equal_______ समताप रखाए समान _______की रखाए होती ह

1 pressure दाब

2 temperature तापमान

3 rainfall वषा

4 height ऊचाई

Correct Answer -

temperature तापमान

Mark the correct sequence of passes in the Western Ghats from north to south

पिमी घाटो म उर स दिण तक दर क सही अनम को िचित कर

1 Thalghat Palghat Bhorghat थलगघाट पालघाट भोरघाट

2 Thalghat Bhorghat Palghat थलघाट भोरघाट पालघाट

3 Bhorghat Thalghat Palghat भोरघाट थलघाट पालघाट

4 Palghat Bhorghat Thalghat पालघाट भोरघाट थलघाट

Correct Answer -

Thalghat Bhorghat Palghat थलघाट भोरघाट पालघाट

Which of the following does not have influence over the climate in India

ि ि ि ी ी

72)

73)

िनिलखत म स िकसका भाव भारत की जलवाय पर नही पड़ता ह

1 Ocean currents सागर की लहर

2 Nearness to equator भम रखा स िनकटता

3 Monsoons मानसन

4 Presence of Indian ocean भारतीय महासागर की उपथित

Correct Answer -

Ocean currents सागर की लहर

Which of the following cloud types has the characteristics like vertical tall narrow and puffy

िनिलखत म स िकस कार क मघो म लबवत लबी सकीण और थलता जसी िवशषताए ह

1 Cumulonimbus तफानी मघ

2 Cumulus मघ पज

3 Cirrocumulus पाभ कपासी मघ

4 Nimbostratus वषारी मघ

Correct Answer -

Cumulus मघ पज

Which of the following statement is INCORRECT about Crude Birth Rate

िनिलखत स कौन सा कथन अशोिधत ज दर क बार म सही नही ह

1 It cannot be used for comparing fertility level between two countries with different population characteristics

इसका उपयोग िविभ जनसा िवशषताओ वाल दो दशो क बीच जनन र की तलना क िलए नही िकया जा सकता ह

2 It is a standardized measure of fertility

यह जनन मता का मानकीकत उपाय ह

3 It is effected by the age-sex composition of the population

यह आबादी की आय-िलग सरचना स भािवत होता ह

4 It is expressed per 1000 population in a given geographical unit

यह िकसी दी गई भौगोिलक इकाई म ित 1000 जनसा पर िकया जाता ह

Correct Answer -

It is a standardized measure of fertility

यह जनन मता का मानकीकत उपाय ह

74)

75)

76)

77)

Which of the following state in India experienced negative decadal growth rate during 2001 to 2011census

भारत म िनिलखत म स िकस रा म वष 2001 स 2011 की जनगणना क दौरान नकाराक िगरावट दर ई

1 Tripura िपरा

2 Nagaland नागालड

3 Haryana हरयाणा

4 Odisha ओिडसा

Correct Answer -

Nagaland नागालड

Which of the following is NOT a characteristic of peninsular rivers

िनिलखत म स कौन सी िवशषता ायीपीय निदयो म नही होती ह

1 Flow through shallow valleys उथल घािटयो क माम स वाह

2 Seasonal flow मौसमी वाह

3 Little erosional activity थोड़ी कटावदार गितिविध

4 Meandering tendency often shifting their beds घमावदार वि अर अपन तटो को थानातरत करना

Correct Answer -

Meandering tendency often shifting their beds घमावदार वि अर अपन तटो को थानातरत करना

Which of the following gases in the atmosphere absorbs heat from the Sunrsquos radiation and the Earthssurface

वायमडल म िनिलखत म स कौन सी गस सय क िविकरण और पी की सतह स ऊा को अवशोिषत करती ह

1 Neon िनयॉन

2 Carbon dioxide काबन डाइऑाइड

3 Argon आगन

4 Nitrogen नाइट ोजन

Correct Answer -

Carbon dioxide काबन डाइऑाइड

Which of the following kind of settlement pattern is found at the confluence of rivers

ि ि ि ि ो

78)

79)

80)

िनिलखत म स िकस कार का वथापन पटन निदयो क सगम पर पाया जाता ह

1 Triangular Paern िकोणीय पटन

2 Circular or Semi-Circular Paern परप या अध-परप पटन

3 Nebular Paern नबलर पटन

4 Star ndashShaped Paern ार-आकार का पटन

Correct Answer -

Triangular Paern िकोणीय पटन

Which one was not the objective of the Biosphere Reserve Projects launched by the UNESCO

यनो ारा श की गई सरित जवमडल परयोजनाओ का उ इनम स कौन सा नही था

1 To promote teaching and research िशण और अनसधान को बढ़ावा दना

2 To make agriculture sustainable किष को दीघकािलक बनाना

3 To conserve ecosystems पारथितक त को सरित करना

4 To conserve genetic diversity for a longtime लब समय तक अनवािशक िविवधता को सरित करना

Correct Answer -

To make agriculture sustainable किष को दीघकािलक बनाना

Which region of the Earth surface receives the highest amount of insulation

पी सतह का कौन सा तापावरोधन की उतम माा ा करता ह

1 Land mass थलखड

2 Savannah region सवाना

3 Water bodies जल िनकाय

4 Tropical desert उकिटबधीय रिगान

Correct Answer -

Tropical desert उकिटबधीय रिगान

Which one of the following is not a biodiversity hotspot

िनिलखत म स कौन सा जव िविवधता का म जगह नही ह

1 Eastern Himalaya पव िहमालय

2 Eastern Ghats पव घाट

81)

82)

83)

3 Indo-Myanmar भारत-ामार

4 Westerm Ghats पिमी घाट

Correct Answer -

Eastern Ghats पव घाट

Which one of the following is NOT a part of the World Network of Biosphere Reserves based on theUNESCO Man and Biosphere Programme

यनो मन और बायोीयर कायम क आधार पर िनिलखत म स कौन बायोीयर रजव क िव नटवक कािहा नही ह

1 Gulf of Mannar मार की खाड़ी

2 Seshachalam शषाचलम

3 Sunderban सदरबन

4 Nilgiri नीलिगर

Correct Answer -

Seshachalam शषाचलम

Which one of the following is an example of ldquodesert vegetationrdquo

िनिलखत म स कौन मथलीय वनित का एक उदाहरण ह

1 Mosses and lichens दलदल और शवाल

2 Temperate grassland समशीतो घास क मदान

3 Coniferous forest शकधारी वन

4 Acacia and cactus एकािसया और कस

Correct Answer -

Acacia and cactus एकािसया और कस

Which one of the following reflects more sunlight िनिलखत म स कौन सा सय की रोशनी को अिधकपरावितत करता ह

1 Paddy crop land धान फसल भिम

2 Land covered with fresh snow ताजा बफ स आािदत भिम

3 Sand desert रतीली रिगान

4 Prairie land यरी भिम

84)

85)

86)

87)

Correct Answer -

Land covered with fresh snow ताजा बफ स आािदत भिम

Which layer of the atmosphere is in contact with the surface of the earthrsquos oceans

वायमडल की कौन सी परत पी क महासागरो की सतह क सपक म ह

1 Stratosphere समताप मडल

2 Mesosphere म मडल

3 Hydrosphere जलमडल

4 Troposphere ोभ मडल

Correct Answer -

Troposphere ोभ मडल

Mediterranean Sea is a border of which of the following countries भम सागर िनिलखत दशो म सिकसकी सीमा ह

1 None of these इनम स कोई नही

2 Iraq इराक

3 Lebanon लबनान

4 Jordan जॉडन

Correct Answer -

Lebanon लबनान

Benguela ocean currents are found along which coast बगएला महासागर धाराए िकस तट क साथ पायीजाती ह

1 East Coast of South America दिण अमरका क पव तट

2 East Coast of Africa अीका क पव तट

3 West Coast of South America दिण अमरका क पिमी तट

4 West Coast of Africa अीका क पिमी तट

Correct Answer -

West Coast of Africa अीका क पिमी तट

88)

89)

90)

Due to tension a block of land on one side being pushed up or upthrown relative to the downthrown blockis referred as

तनाव क कारण नीच फ क ए खड क साप भिम का एक खड एक ओर स ऊपर धकला जाता ह या ऊपर की ओरफ का जाता ह यह _____ क प म सदिभत ह

1 Thrust fault प श

2 Normal fault सामा श

3 Reverse fault म श

4 Strike slip fault नितलब सपण श

Correct Answer -

Normal fault सामा श

Inter-tropical doldrums is a zone of ______ अतर-उकिटबधीय डोलड ______ का एक ह

1 Frontolysis टोलायिसस

2 Convergence अिभसरण

3 Inter-tropical divergence zone अतर-उकिटबधीय िवचलन

4 Local wind थानीय वाय

Correct Answer -

Convergence अिभसरण

The Horse Latitudes are regions located at about _____ north and south of the equator

हॉस अाश भम रखा क उर और दिण म लगभग _____ पर थत ह

1 30ndash60 degree Latitude 30-60 िडी अाश

2 0ndash5 degree Latitude 0-5 िडी अाश

3 30 degree Latitude 30 िडी अाश

4 60ndash90 degree Latitude 60-90 िडी अाश

Correct Answer -

30 degree Latitude 30 िडी अाश

Generally evaporation is high over which part of the Earth

आम तौर पर पी क िकस भाग पर वाीकरण अिधक होता ह

1 Equatorial maritime भमवत समीय ी ी

91)

92)

2 Equatorial continental भमवत महाीपीय

3 Polar maritime वीय समीय

4 Polar continental वीय महाीपीय

Correct Answer -

Equatorial maritime भमवत समीय

A very high temperature during summer in north western India leads to what type of climaticcondition in south

उर पिमी भारत म गम क दौरान बत अिधक तापमान होन क कारण दिण म िकस कार की जलवाय थितउ करता ह

1 Depression over arabian sea अरब सागर पर अवनमन

2 Failure monsoon मानसन िवफलता

3 Successful monsoon मानसन सफलता

4 Cyclones चवात

Correct Answer -

Successful monsoon मानसन सफलता

Lightning and thunder are the resultant effect when तिड़त और गजन परणामी भाव ह जब

1 Two massive clouds hit powerfully each other first lightning is produced and later sound is produced

दो बड़ बादल एक दसर स शशाली ढग स टकरात ह पहल आकाशीय िवदयत उ होता ह और बाद म िन उहोती ह

2 Two massive clouds come into contact with the powerful wind collision this results into first sound and thenlightning

दो बड़ बादल शशाली पवन सघ क सपक म आत ह इसका परणामप पहल िन और िफर आकाशीय िवदयतउ होता ह

3 None of the above उपरो म स कोई भी नही

4 A high density cloud contains positively and negatively charged electric ions and when this interacts light andsound are simultaneously produced

एक उ घन बादल म धनाक और ऋणाक आविशत िवदयत आयन होत ह और जब यह परर भाव डालत ह तोकाश और िन एक साथ उािदत होती ह

Correct Answer -

A high density cloud contains positively and negatively charged electric ions and when this interacts light andsound are simultaneously produced

औ ि ि ो औ ो

93)

94)

95)

एक उ घन बादल म धनाक और ऋणाक आविशत िवदयत आयन होत ह और जब यह परर भाव डालत ह तोकाश और िन एक साथ उािदत होती ह

Doon Valley is able to grow rice because दन घाटी चावल उगान म सम ह ोिक

1 Other crops cannot be grown वहा अ फसलो को उगाया नही जा सकता ह

2 People in the valley are rice eaters घाटी म लोग चावल खान वाल ह

3 There is a huge export demand of rice वहा चावल की भारी िनयात माग ह

4 It has warm summer and snow melt waters for irrigation

वहा गिमया गम होती ह िसचाई क िलए बफ का िपघला आ पानी होता ह

Correct Answer -

It has warm summer and snow melt waters for irrigation

वहा गिमया गम होती ह िसचाई क िलए बफ का िपघला आ पानी होता ह

CANCELLED

In the geological time scale the Mesozoic Era DOES NOT contains which of the following periods

भगभय समय पमान पर मजीवी यग म िन कालो म स कौन नही ह

1 Triassic ट ाइऐिसक

2 Jurassic जरिसक

3 Cretaceous चाकमय

4 Carboniferous काबनी

Correct Answer -

Carboniferous काबनी

96)

1 P-3 Q-4 R-2 S-1

2 P-3 Q-4 R-1 S-2

3 P-3 Q-4 R-1 S-2

4 P-4 Q-3 R-2 S-1

Correct Answer -

P-4 Q-3 R-2 S-1

1 P-3 Q-1 R-4 S-2

2 P-3 Q-4 R-1 S-2

3 P-3 Q-2 R-4 S-1

97)

98)

4 P-2 Q-1 R-4 S-3

Correct Answer -

P-3 Q-4 R-1 S-2

1 P-3 Q-1 R-4 S-2

2 P-2 Q-3 R-4 S-1

3 P-2 Q-1 R-3 S-4

4 P-4 Q-2 R-1 S-3

Correct Answer -

P-3 Q-1 R-4 S-2

99)

100)

1 P-3 Q-2 R-4 S-1

2 P-1 Q-2 R-3 S-4

3 P-2 Q-3 R-1 S-4

4 P-4 Q-3 R-2 S-1

Correct Answer -

P-2 Q-3 R-1 S-4

ldquoHuman geography is the study of changing relationship between the unresting man and the unstableearthrdquo was defined by

lsquolsquoमानव भगोल ाकल आदमी और अथर पी क बीच सबध परवतन का अयन हrdquo ______ ारा परभािषत िकया गयाथा

1 J Brunches ज चस

2 EC Semple ईसी सल

3 HJ Mackinder एच ज मिकदर

4 PV Blache पीवी च

Correct Answer -

EC Semple ईसी सल

Sedimentary rocks are finally and ultimately derived from the____________

अवसादी चान अततः ________ स ा की जाती ह

1 action of earth movements पी की गितिविधयो

2 marine deposit समी िनप

3 weathering of metamorphic rocks पातरत चानो क अपय

4 weathering of igneous rocks आय चानो क अपय

Correct Answer -

weathering of igneous rocks आय चानो क अपय

Page 32: High School Teacher Eligibility Test- BOARD PROFESSIONAL ...peb.mp.gov.in/results/RESULT_18/HST_RES18/Final_anwser_key/HST… · M a ndl a / मंड ल ... Under the Madhya Pradesh

45)

46)

47)

48)

The dust and ash material hurled from the volcanoes are termed as

ालामखी स िनकलन वाली धल और राख सामी को _______ क प म कहा जाता ह

1 Pyroclasc पाइरोाक

2 Hyperclastic हाइपराक

3 Hepiroclastic हिपरोाक

4 Cirroclastic िसरोाक

Correct Answer -

Pyroclasc पाइरोाक

The vertical difference in elevation between a low tide and high tide is referred as

कम ार और उ ार क बीच ऊचाई म लबवत अतर _____ स सदिभत होता ह

1 Tidal slope ारीय ढलान

2 Tidal elevation ारीय उयन

3 Tidal range ारीय परास

4 Tidal height ारीय ऊचाई

Correct Answer -

Tidal range ारीय परास

The maximum biodiversity is found in which of the following regions िनिलखत ो म स अिधकतमजव िविवधता िकसम पायी जाती ह

1 Amazon Basin अमज़न बिसन

2 East Indies ई इडीज

3 Congo Basin कागो बिसन

4 West indies व इडीज

Correct Answer -

Amazon Basin अमज़न बिसन

The cultivation of rice crop produces_______ चावल की फसल की खती ______ का उादन करती ह

1 SO2

49)

50)

51)

2 CH4

3 CFCs

4 CO2

Correct Answer -

CH4

The pressure system with higher pressure at the centre is called__________

क म उ दबाव वाली दबाव णाली को _______ कहा जाता ह

1 front अ

2 depression अवनमन

3 cyclone चवात

4 anti-cyclone ितचवात

Correct Answer -

anti-cyclone ितचवात

The Himalayan region is poor in mineral resources because िहमालयी खिनज ससाधनो म समनही ह ोिक

1 The displacement of rock strata has disturbed the arrangement of rocks and made it complex

शलीय परत क िवथापन न चानो की वथा को अवथत कर िदया ह और इस जिटल बना िदया ह

2 The climate conditions are not suitable for exploitation of minerals

जलवाय की थित खिनजो क दोहन क िलए उपय नही ह

3 The terrain makes explanation of minerals difficult and very costly due to transportation difficulties

भ-भाग परवहन की किठनाइयो क कारण खिनजो का दोहन मल और बत महगा बना दता ह

4 It is made up of crystalline rocks यह िलीय चानो स बना ह

Correct Answer -

The displacement of rock strata has disturbed the arrangement of rocks and made it complex

शलीय परत क िवथापन न चानो की वथा को अवथत कर िदया ह और इस जिटल बना िदया ह

The process through which the moisture is added to the atmosphere by vegetation is termed as

वह िया िजसक माम स वनित ारा वातावरण म नमी िमलायी जाती ह _______ क प म जानी जाती ह

52)

53)

54)

1 Condensation सघनन

2 Evapotranspiration वान-उजन

3 Radiation िविकरण

4 Precipitation वषण

Correct Answer -

Evapotranspiration वान-उजन

The process through which the terrestrial heat is transferred to air by direct contact is termed as

वह िया िजसम सपक ारा थलीय ऊा वाय म थानातरत हो जाती ह ______ क प म जानी जाती ह

1 Conduction चालन

2 Convection सवहन

3 Insolation आतपन

4 Radiation िविकरण

Correct Answer -

Conduction चालन

The largest area under mangroves is in which of the following statesunion territory

मोव क अतगत िनिलखत राोसघ शािसत दशो म स सबस बड़ा कौन सा ह

1 Andaman and Nicobar अमान और िनकोबार

2 Andhra Pradesh आ दश

3 West Bengal पिम बगाल

4 Gujarat गजरात

Correct Answer -

West Bengal पिम बगाल

The longitudinal transverse and surface waves in an earthquake originate from

भकप म दशातर अनथ और सतह तरग यहा उ होती ह

1 The focus on the surface of the Earth पी क सतह पर क -िबद म

2 The focus within the body of the Earth पी क भीतर क -िबद म

3 The epicenter within the body of the Earth पी क भीतर उपरक म

55)

56)

57)

4 The epicenter on the surface of the Earth पी क सतह पर उपरक म

Correct Answer -

The focus within the body of the Earth पी क भीतर क -िबद म

The down slope movement of material due to gravity is called______

गाकषण क कारण पदाथ की अनढाल गित को ______ कहा जाता ह

1 mass movement पदाथ सचलन

2 deposition िनप

3 erosion रण

4 volcanic movement ालामखीय सचलन

Correct Answer -

mass movement पदाथ सचलन

Shimla is cooler than Amritsar although both are on the same latitude This is because

िशमला म अमतसर स अिधक ठड ह हालािक दोनो समान अाश पर ह ऐसा ह ोिक

1 Shimla is at a greater height above sea level than Amritsar अमतसर की तलना म िशमला सम तल स अिधकऊचाई पर ह

2 Shimla is further north िशमला उर की ओर ह

3 Shimla is farther from the equator िशमला भम रखा स आग ह

4 Their longitudes differ उनकी दशातर रखाए िभ ह

Correct Answer -

Shimla is at a greater height above sea level than Amritsar अमतसर की तलना म िशमला सम तल स अिधकऊचाई पर ह

lsquoTempo of Urbanizationrsquo measures which of the following

lsquoशहरीकरण का टपोrsquo िनिलखत म स कौन सा उपाय ह

1 Speed of urbanizaon शहरीकरण की गित

2 None of the above इनम स कोई नही

3 Inequality of urbanizaon शहरीकरण की असमानता

4 Current level of urbanizaon शहरीकरण का वतमान र

Correct Answer -

58)

59)

60)

Speed of urbanizaon शहरीकरण की गित

Out of the following options choose the INCORRECT statement

िनिलखत िवको म स गलत कथन का चयन कर

1 The clear tracts in the equatorial region recover rapidly भम रखा म भभाग तजी स ठीक हो जात ह

2 The stable communities include a redwood forest a pine forest at high elevations

थर समदायो म एक रडवड वन उ ऊचाई पर एक दवदार वन शािमल ह

3 Any ecosystem moves towards maximum biomass and stability to survive

कोई भी पारथितकी त जीिवत रहन क िलए अिधकतम जवसहित और थरता की तरफ असर होता ह

4 Tropical rain forests near equator are stable ecosystems

भम रखा क पास उकिटबधीय वषा वन थर पारथितक त ह

Correct Answer -

The clear tracts in the equatorial region recover rapidly भम रखा म भभाग तजी स ठीक हो जात ह

Seasonal contrasts are maximum in मौसमी िवषमता अिधकतम ह

1 Mid latitudes म अाश म

2 Low attitudes िन अाश म

3 High latitudes उ अाश म

4 Subtropics उपोकिटबधीय म

Correct Answer -

Mid latitudes म अाश म

In India which type of forest among the following occupies the largest area

भारत म िनिलखत म स िकस कार क वन सबस बड़ा फल आािदत करत ह

1 Sub-tropical Dry Evergreen Forest उप उकिटबधीय श सदाबहार वन

2 Mountain Wet Temperate Forest पवतीय आ शीतो वन

3 Tropical Moist Deciduous Forest उकिटबधीय आ पणपाती वन

4 Tropical Wet Evergreen Forest उकिटबधीय आ सदाबहार वन

Correct Answer -

Tropical Moist Deciduous Forest उकिटबधीय आ पणपाती वन

61)

62)

63)

64)

What is the proportion of lsquoJuvenile Populationrsquo (0-14 years) in India as per 2011Census

2011 की जनगणना क अनसार भारत म जवनाइल पॉपलशन यानी िकशोर जनस या (0-14 वष) का अनपात ाह

1 3076 of total population कल जनस या का 3076

2 2764 of total population कल जनस या का 2764

3 2933 of total population कल जनस या का 2933

4 3354 of total population कल जनस या का 3354

Correct Answer -

3076 of total population कल जनस या का 3076

What is the Belfast famous for बलफा िकसक िलए मशर ह

1 Belt of cotton textile industry कपास व उोग क

2 Ship-building industry जहाज िनमाण उोग

3 Agricultural machinery किष उपकरण

4 Aero planes manufacturing वाययान िनमाण

Correct Answer -

Ship-building industry जहाज िनमाण उोग

What is the most important occupation in tropical monsoon lands

उकिटबधीय मॉनसन भिम म सबस महपण वसाय ा ह

1 Mining खनन

2 Cattle rearing मवशी पालन

3 Agriculture किष

4 Nomadic herding नोमािडक जड़ी-बिटया

Correct Answer -

Agriculture किष

What is the most important characteristics of the islands (Indian) located in the Arabian sea

अरब सागर म थत ीपो (भारतीय) की सबस महपण िवशषता ा ह

ी ो

65)

66)

67)

1 There are all of coral origins सभी कोरल मल क ह

2 There are all very small in size य सभी आकार म बत छोट ह

3 They have a very dry climate इनकी जलवाय बत श ह

4 They are extended parts of the mainland व महाीप क िवारत िह ह

Correct Answer -

There are all of coral origins सभी कोरल मल क ह

What do the basalt layers of the Deccan indicate डन की बसा परत ा इिगत करती ह

1 All of the above उपरो सभी

2 Huge volcanic eruptions in the distant past दरथ अतीत म िवशाल ालामखीय िवोट

3 The immense erosional activity of the rivers निदयो की िवशाल रण गितिविध

4 The influence of weathering मौसम का भाव

Correct Answer -

Huge volcanic eruptions in the distant past दरथ अतीत म िवशाल ालामखीय िवोट

In the structure of planet Earth below the mantle the core is mainly made up of_____

पी ह की सरचना म मटल क नीच कोर म प स______ स िनिमत होती ह

1 aluminium एमीिनयम

2 silicon िसिलकॉन

3 chromium ोिमयम

4 iron लोहा

Correct Answer -

iron लोहा

One of the major Mid Oceanic Ridge is found in मख म-महासागर चोिटयो म स एक ______ म पायाजाता ह

1 Mid Pacific Ocean म शात महासागर

2 Mid Atlantic Ocean म अटलािटक महासागर

3 Mid Indian Ocean म भारतीय महासागर

4 Mid Arctic Ocean म आक िटक महासागर

68)

69)

70)

71)

Correct Answer -

Mid Atlantic Ocean म अटलािटक महासागर

Magma that reaches the Earthrsquos surface and then solidifies is called________

मा जो पी की सतह तक पचती ह और िफर ठोस हो जाती ह ________कहलाती ह

1 quartz ाटज

2 lava लावा

3 granite नाइट

4 silicates िसिलकट

Correct Answer -

lava लावा

Isotherms are the lines of equal_______ समताप रखाए समान _______की रखाए होती ह

1 pressure दाब

2 temperature तापमान

3 rainfall वषा

4 height ऊचाई

Correct Answer -

temperature तापमान

Mark the correct sequence of passes in the Western Ghats from north to south

पिमी घाटो म उर स दिण तक दर क सही अनम को िचित कर

1 Thalghat Palghat Bhorghat थलगघाट पालघाट भोरघाट

2 Thalghat Bhorghat Palghat थलघाट भोरघाट पालघाट

3 Bhorghat Thalghat Palghat भोरघाट थलघाट पालघाट

4 Palghat Bhorghat Thalghat पालघाट भोरघाट थलघाट

Correct Answer -

Thalghat Bhorghat Palghat थलघाट भोरघाट पालघाट

Which of the following does not have influence over the climate in India

ि ि ि ी ी

72)

73)

िनिलखत म स िकसका भाव भारत की जलवाय पर नही पड़ता ह

1 Ocean currents सागर की लहर

2 Nearness to equator भम रखा स िनकटता

3 Monsoons मानसन

4 Presence of Indian ocean भारतीय महासागर की उपथित

Correct Answer -

Ocean currents सागर की लहर

Which of the following cloud types has the characteristics like vertical tall narrow and puffy

िनिलखत म स िकस कार क मघो म लबवत लबी सकीण और थलता जसी िवशषताए ह

1 Cumulonimbus तफानी मघ

2 Cumulus मघ पज

3 Cirrocumulus पाभ कपासी मघ

4 Nimbostratus वषारी मघ

Correct Answer -

Cumulus मघ पज

Which of the following statement is INCORRECT about Crude Birth Rate

िनिलखत स कौन सा कथन अशोिधत ज दर क बार म सही नही ह

1 It cannot be used for comparing fertility level between two countries with different population characteristics

इसका उपयोग िविभ जनसा िवशषताओ वाल दो दशो क बीच जनन र की तलना क िलए नही िकया जा सकता ह

2 It is a standardized measure of fertility

यह जनन मता का मानकीकत उपाय ह

3 It is effected by the age-sex composition of the population

यह आबादी की आय-िलग सरचना स भािवत होता ह

4 It is expressed per 1000 population in a given geographical unit

यह िकसी दी गई भौगोिलक इकाई म ित 1000 जनसा पर िकया जाता ह

Correct Answer -

It is a standardized measure of fertility

यह जनन मता का मानकीकत उपाय ह

74)

75)

76)

77)

Which of the following state in India experienced negative decadal growth rate during 2001 to 2011census

भारत म िनिलखत म स िकस रा म वष 2001 स 2011 की जनगणना क दौरान नकाराक िगरावट दर ई

1 Tripura िपरा

2 Nagaland नागालड

3 Haryana हरयाणा

4 Odisha ओिडसा

Correct Answer -

Nagaland नागालड

Which of the following is NOT a characteristic of peninsular rivers

िनिलखत म स कौन सी िवशषता ायीपीय निदयो म नही होती ह

1 Flow through shallow valleys उथल घािटयो क माम स वाह

2 Seasonal flow मौसमी वाह

3 Little erosional activity थोड़ी कटावदार गितिविध

4 Meandering tendency often shifting their beds घमावदार वि अर अपन तटो को थानातरत करना

Correct Answer -

Meandering tendency often shifting their beds घमावदार वि अर अपन तटो को थानातरत करना

Which of the following gases in the atmosphere absorbs heat from the Sunrsquos radiation and the Earthssurface

वायमडल म िनिलखत म स कौन सी गस सय क िविकरण और पी की सतह स ऊा को अवशोिषत करती ह

1 Neon िनयॉन

2 Carbon dioxide काबन डाइऑाइड

3 Argon आगन

4 Nitrogen नाइट ोजन

Correct Answer -

Carbon dioxide काबन डाइऑाइड

Which of the following kind of settlement pattern is found at the confluence of rivers

ि ि ि ि ो

78)

79)

80)

िनिलखत म स िकस कार का वथापन पटन निदयो क सगम पर पाया जाता ह

1 Triangular Paern िकोणीय पटन

2 Circular or Semi-Circular Paern परप या अध-परप पटन

3 Nebular Paern नबलर पटन

4 Star ndashShaped Paern ार-आकार का पटन

Correct Answer -

Triangular Paern िकोणीय पटन

Which one was not the objective of the Biosphere Reserve Projects launched by the UNESCO

यनो ारा श की गई सरित जवमडल परयोजनाओ का उ इनम स कौन सा नही था

1 To promote teaching and research िशण और अनसधान को बढ़ावा दना

2 To make agriculture sustainable किष को दीघकािलक बनाना

3 To conserve ecosystems पारथितक त को सरित करना

4 To conserve genetic diversity for a longtime लब समय तक अनवािशक िविवधता को सरित करना

Correct Answer -

To make agriculture sustainable किष को दीघकािलक बनाना

Which region of the Earth surface receives the highest amount of insulation

पी सतह का कौन सा तापावरोधन की उतम माा ा करता ह

1 Land mass थलखड

2 Savannah region सवाना

3 Water bodies जल िनकाय

4 Tropical desert उकिटबधीय रिगान

Correct Answer -

Tropical desert उकिटबधीय रिगान

Which one of the following is not a biodiversity hotspot

िनिलखत म स कौन सा जव िविवधता का म जगह नही ह

1 Eastern Himalaya पव िहमालय

2 Eastern Ghats पव घाट

81)

82)

83)

3 Indo-Myanmar भारत-ामार

4 Westerm Ghats पिमी घाट

Correct Answer -

Eastern Ghats पव घाट

Which one of the following is NOT a part of the World Network of Biosphere Reserves based on theUNESCO Man and Biosphere Programme

यनो मन और बायोीयर कायम क आधार पर िनिलखत म स कौन बायोीयर रजव क िव नटवक कािहा नही ह

1 Gulf of Mannar मार की खाड़ी

2 Seshachalam शषाचलम

3 Sunderban सदरबन

4 Nilgiri नीलिगर

Correct Answer -

Seshachalam शषाचलम

Which one of the following is an example of ldquodesert vegetationrdquo

िनिलखत म स कौन मथलीय वनित का एक उदाहरण ह

1 Mosses and lichens दलदल और शवाल

2 Temperate grassland समशीतो घास क मदान

3 Coniferous forest शकधारी वन

4 Acacia and cactus एकािसया और कस

Correct Answer -

Acacia and cactus एकािसया और कस

Which one of the following reflects more sunlight िनिलखत म स कौन सा सय की रोशनी को अिधकपरावितत करता ह

1 Paddy crop land धान फसल भिम

2 Land covered with fresh snow ताजा बफ स आािदत भिम

3 Sand desert रतीली रिगान

4 Prairie land यरी भिम

84)

85)

86)

87)

Correct Answer -

Land covered with fresh snow ताजा बफ स आािदत भिम

Which layer of the atmosphere is in contact with the surface of the earthrsquos oceans

वायमडल की कौन सी परत पी क महासागरो की सतह क सपक म ह

1 Stratosphere समताप मडल

2 Mesosphere म मडल

3 Hydrosphere जलमडल

4 Troposphere ोभ मडल

Correct Answer -

Troposphere ोभ मडल

Mediterranean Sea is a border of which of the following countries भम सागर िनिलखत दशो म सिकसकी सीमा ह

1 None of these इनम स कोई नही

2 Iraq इराक

3 Lebanon लबनान

4 Jordan जॉडन

Correct Answer -

Lebanon लबनान

Benguela ocean currents are found along which coast बगएला महासागर धाराए िकस तट क साथ पायीजाती ह

1 East Coast of South America दिण अमरका क पव तट

2 East Coast of Africa अीका क पव तट

3 West Coast of South America दिण अमरका क पिमी तट

4 West Coast of Africa अीका क पिमी तट

Correct Answer -

West Coast of Africa अीका क पिमी तट

88)

89)

90)

Due to tension a block of land on one side being pushed up or upthrown relative to the downthrown blockis referred as

तनाव क कारण नीच फ क ए खड क साप भिम का एक खड एक ओर स ऊपर धकला जाता ह या ऊपर की ओरफ का जाता ह यह _____ क प म सदिभत ह

1 Thrust fault प श

2 Normal fault सामा श

3 Reverse fault म श

4 Strike slip fault नितलब सपण श

Correct Answer -

Normal fault सामा श

Inter-tropical doldrums is a zone of ______ अतर-उकिटबधीय डोलड ______ का एक ह

1 Frontolysis टोलायिसस

2 Convergence अिभसरण

3 Inter-tropical divergence zone अतर-उकिटबधीय िवचलन

4 Local wind थानीय वाय

Correct Answer -

Convergence अिभसरण

The Horse Latitudes are regions located at about _____ north and south of the equator

हॉस अाश भम रखा क उर और दिण म लगभग _____ पर थत ह

1 30ndash60 degree Latitude 30-60 िडी अाश

2 0ndash5 degree Latitude 0-5 िडी अाश

3 30 degree Latitude 30 िडी अाश

4 60ndash90 degree Latitude 60-90 िडी अाश

Correct Answer -

30 degree Latitude 30 िडी अाश

Generally evaporation is high over which part of the Earth

आम तौर पर पी क िकस भाग पर वाीकरण अिधक होता ह

1 Equatorial maritime भमवत समीय ी ी

91)

92)

2 Equatorial continental भमवत महाीपीय

3 Polar maritime वीय समीय

4 Polar continental वीय महाीपीय

Correct Answer -

Equatorial maritime भमवत समीय

A very high temperature during summer in north western India leads to what type of climaticcondition in south

उर पिमी भारत म गम क दौरान बत अिधक तापमान होन क कारण दिण म िकस कार की जलवाय थितउ करता ह

1 Depression over arabian sea अरब सागर पर अवनमन

2 Failure monsoon मानसन िवफलता

3 Successful monsoon मानसन सफलता

4 Cyclones चवात

Correct Answer -

Successful monsoon मानसन सफलता

Lightning and thunder are the resultant effect when तिड़त और गजन परणामी भाव ह जब

1 Two massive clouds hit powerfully each other first lightning is produced and later sound is produced

दो बड़ बादल एक दसर स शशाली ढग स टकरात ह पहल आकाशीय िवदयत उ होता ह और बाद म िन उहोती ह

2 Two massive clouds come into contact with the powerful wind collision this results into first sound and thenlightning

दो बड़ बादल शशाली पवन सघ क सपक म आत ह इसका परणामप पहल िन और िफर आकाशीय िवदयतउ होता ह

3 None of the above उपरो म स कोई भी नही

4 A high density cloud contains positively and negatively charged electric ions and when this interacts light andsound are simultaneously produced

एक उ घन बादल म धनाक और ऋणाक आविशत िवदयत आयन होत ह और जब यह परर भाव डालत ह तोकाश और िन एक साथ उािदत होती ह

Correct Answer -

A high density cloud contains positively and negatively charged electric ions and when this interacts light andsound are simultaneously produced

औ ि ि ो औ ो

93)

94)

95)

एक उ घन बादल म धनाक और ऋणाक आविशत िवदयत आयन होत ह और जब यह परर भाव डालत ह तोकाश और िन एक साथ उािदत होती ह

Doon Valley is able to grow rice because दन घाटी चावल उगान म सम ह ोिक

1 Other crops cannot be grown वहा अ फसलो को उगाया नही जा सकता ह

2 People in the valley are rice eaters घाटी म लोग चावल खान वाल ह

3 There is a huge export demand of rice वहा चावल की भारी िनयात माग ह

4 It has warm summer and snow melt waters for irrigation

वहा गिमया गम होती ह िसचाई क िलए बफ का िपघला आ पानी होता ह

Correct Answer -

It has warm summer and snow melt waters for irrigation

वहा गिमया गम होती ह िसचाई क िलए बफ का िपघला आ पानी होता ह

CANCELLED

In the geological time scale the Mesozoic Era DOES NOT contains which of the following periods

भगभय समय पमान पर मजीवी यग म िन कालो म स कौन नही ह

1 Triassic ट ाइऐिसक

2 Jurassic जरिसक

3 Cretaceous चाकमय

4 Carboniferous काबनी

Correct Answer -

Carboniferous काबनी

96)

1 P-3 Q-4 R-2 S-1

2 P-3 Q-4 R-1 S-2

3 P-3 Q-4 R-1 S-2

4 P-4 Q-3 R-2 S-1

Correct Answer -

P-4 Q-3 R-2 S-1

1 P-3 Q-1 R-4 S-2

2 P-3 Q-4 R-1 S-2

3 P-3 Q-2 R-4 S-1

97)

98)

4 P-2 Q-1 R-4 S-3

Correct Answer -

P-3 Q-4 R-1 S-2

1 P-3 Q-1 R-4 S-2

2 P-2 Q-3 R-4 S-1

3 P-2 Q-1 R-3 S-4

4 P-4 Q-2 R-1 S-3

Correct Answer -

P-3 Q-1 R-4 S-2

99)

100)

1 P-3 Q-2 R-4 S-1

2 P-1 Q-2 R-3 S-4

3 P-2 Q-3 R-1 S-4

4 P-4 Q-3 R-2 S-1

Correct Answer -

P-2 Q-3 R-1 S-4

ldquoHuman geography is the study of changing relationship between the unresting man and the unstableearthrdquo was defined by

lsquolsquoमानव भगोल ाकल आदमी और अथर पी क बीच सबध परवतन का अयन हrdquo ______ ारा परभािषत िकया गयाथा

1 J Brunches ज चस

2 EC Semple ईसी सल

3 HJ Mackinder एच ज मिकदर

4 PV Blache पीवी च

Correct Answer -

EC Semple ईसी सल

Sedimentary rocks are finally and ultimately derived from the____________

अवसादी चान अततः ________ स ा की जाती ह

1 action of earth movements पी की गितिविधयो

2 marine deposit समी िनप

3 weathering of metamorphic rocks पातरत चानो क अपय

4 weathering of igneous rocks आय चानो क अपय

Correct Answer -

weathering of igneous rocks आय चानो क अपय

Page 33: High School Teacher Eligibility Test- BOARD PROFESSIONAL ...peb.mp.gov.in/results/RESULT_18/HST_RES18/Final_anwser_key/HST… · M a ndl a / मंड ल ... Under the Madhya Pradesh

49)

50)

51)

2 CH4

3 CFCs

4 CO2

Correct Answer -

CH4

The pressure system with higher pressure at the centre is called__________

क म उ दबाव वाली दबाव णाली को _______ कहा जाता ह

1 front अ

2 depression अवनमन

3 cyclone चवात

4 anti-cyclone ितचवात

Correct Answer -

anti-cyclone ितचवात

The Himalayan region is poor in mineral resources because िहमालयी खिनज ससाधनो म समनही ह ोिक

1 The displacement of rock strata has disturbed the arrangement of rocks and made it complex

शलीय परत क िवथापन न चानो की वथा को अवथत कर िदया ह और इस जिटल बना िदया ह

2 The climate conditions are not suitable for exploitation of minerals

जलवाय की थित खिनजो क दोहन क िलए उपय नही ह

3 The terrain makes explanation of minerals difficult and very costly due to transportation difficulties

भ-भाग परवहन की किठनाइयो क कारण खिनजो का दोहन मल और बत महगा बना दता ह

4 It is made up of crystalline rocks यह िलीय चानो स बना ह

Correct Answer -

The displacement of rock strata has disturbed the arrangement of rocks and made it complex

शलीय परत क िवथापन न चानो की वथा को अवथत कर िदया ह और इस जिटल बना िदया ह

The process through which the moisture is added to the atmosphere by vegetation is termed as

वह िया िजसक माम स वनित ारा वातावरण म नमी िमलायी जाती ह _______ क प म जानी जाती ह

52)

53)

54)

1 Condensation सघनन

2 Evapotranspiration वान-उजन

3 Radiation िविकरण

4 Precipitation वषण

Correct Answer -

Evapotranspiration वान-उजन

The process through which the terrestrial heat is transferred to air by direct contact is termed as

वह िया िजसम सपक ारा थलीय ऊा वाय म थानातरत हो जाती ह ______ क प म जानी जाती ह

1 Conduction चालन

2 Convection सवहन

3 Insolation आतपन

4 Radiation िविकरण

Correct Answer -

Conduction चालन

The largest area under mangroves is in which of the following statesunion territory

मोव क अतगत िनिलखत राोसघ शािसत दशो म स सबस बड़ा कौन सा ह

1 Andaman and Nicobar अमान और िनकोबार

2 Andhra Pradesh आ दश

3 West Bengal पिम बगाल

4 Gujarat गजरात

Correct Answer -

West Bengal पिम बगाल

The longitudinal transverse and surface waves in an earthquake originate from

भकप म दशातर अनथ और सतह तरग यहा उ होती ह

1 The focus on the surface of the Earth पी क सतह पर क -िबद म

2 The focus within the body of the Earth पी क भीतर क -िबद म

3 The epicenter within the body of the Earth पी क भीतर उपरक म

55)

56)

57)

4 The epicenter on the surface of the Earth पी क सतह पर उपरक म

Correct Answer -

The focus within the body of the Earth पी क भीतर क -िबद म

The down slope movement of material due to gravity is called______

गाकषण क कारण पदाथ की अनढाल गित को ______ कहा जाता ह

1 mass movement पदाथ सचलन

2 deposition िनप

3 erosion रण

4 volcanic movement ालामखीय सचलन

Correct Answer -

mass movement पदाथ सचलन

Shimla is cooler than Amritsar although both are on the same latitude This is because

िशमला म अमतसर स अिधक ठड ह हालािक दोनो समान अाश पर ह ऐसा ह ोिक

1 Shimla is at a greater height above sea level than Amritsar अमतसर की तलना म िशमला सम तल स अिधकऊचाई पर ह

2 Shimla is further north िशमला उर की ओर ह

3 Shimla is farther from the equator िशमला भम रखा स आग ह

4 Their longitudes differ उनकी दशातर रखाए िभ ह

Correct Answer -

Shimla is at a greater height above sea level than Amritsar अमतसर की तलना म िशमला सम तल स अिधकऊचाई पर ह

lsquoTempo of Urbanizationrsquo measures which of the following

lsquoशहरीकरण का टपोrsquo िनिलखत म स कौन सा उपाय ह

1 Speed of urbanizaon शहरीकरण की गित

2 None of the above इनम स कोई नही

3 Inequality of urbanizaon शहरीकरण की असमानता

4 Current level of urbanizaon शहरीकरण का वतमान र

Correct Answer -

58)

59)

60)

Speed of urbanizaon शहरीकरण की गित

Out of the following options choose the INCORRECT statement

िनिलखत िवको म स गलत कथन का चयन कर

1 The clear tracts in the equatorial region recover rapidly भम रखा म भभाग तजी स ठीक हो जात ह

2 The stable communities include a redwood forest a pine forest at high elevations

थर समदायो म एक रडवड वन उ ऊचाई पर एक दवदार वन शािमल ह

3 Any ecosystem moves towards maximum biomass and stability to survive

कोई भी पारथितकी त जीिवत रहन क िलए अिधकतम जवसहित और थरता की तरफ असर होता ह

4 Tropical rain forests near equator are stable ecosystems

भम रखा क पास उकिटबधीय वषा वन थर पारथितक त ह

Correct Answer -

The clear tracts in the equatorial region recover rapidly भम रखा म भभाग तजी स ठीक हो जात ह

Seasonal contrasts are maximum in मौसमी िवषमता अिधकतम ह

1 Mid latitudes म अाश म

2 Low attitudes िन अाश म

3 High latitudes उ अाश म

4 Subtropics उपोकिटबधीय म

Correct Answer -

Mid latitudes म अाश म

In India which type of forest among the following occupies the largest area

भारत म िनिलखत म स िकस कार क वन सबस बड़ा फल आािदत करत ह

1 Sub-tropical Dry Evergreen Forest उप उकिटबधीय श सदाबहार वन

2 Mountain Wet Temperate Forest पवतीय आ शीतो वन

3 Tropical Moist Deciduous Forest उकिटबधीय आ पणपाती वन

4 Tropical Wet Evergreen Forest उकिटबधीय आ सदाबहार वन

Correct Answer -

Tropical Moist Deciduous Forest उकिटबधीय आ पणपाती वन

61)

62)

63)

64)

What is the proportion of lsquoJuvenile Populationrsquo (0-14 years) in India as per 2011Census

2011 की जनगणना क अनसार भारत म जवनाइल पॉपलशन यानी िकशोर जनस या (0-14 वष) का अनपात ाह

1 3076 of total population कल जनस या का 3076

2 2764 of total population कल जनस या का 2764

3 2933 of total population कल जनस या का 2933

4 3354 of total population कल जनस या का 3354

Correct Answer -

3076 of total population कल जनस या का 3076

What is the Belfast famous for बलफा िकसक िलए मशर ह

1 Belt of cotton textile industry कपास व उोग क

2 Ship-building industry जहाज िनमाण उोग

3 Agricultural machinery किष उपकरण

4 Aero planes manufacturing वाययान िनमाण

Correct Answer -

Ship-building industry जहाज िनमाण उोग

What is the most important occupation in tropical monsoon lands

उकिटबधीय मॉनसन भिम म सबस महपण वसाय ा ह

1 Mining खनन

2 Cattle rearing मवशी पालन

3 Agriculture किष

4 Nomadic herding नोमािडक जड़ी-बिटया

Correct Answer -

Agriculture किष

What is the most important characteristics of the islands (Indian) located in the Arabian sea

अरब सागर म थत ीपो (भारतीय) की सबस महपण िवशषता ा ह

ी ो

65)

66)

67)

1 There are all of coral origins सभी कोरल मल क ह

2 There are all very small in size य सभी आकार म बत छोट ह

3 They have a very dry climate इनकी जलवाय बत श ह

4 They are extended parts of the mainland व महाीप क िवारत िह ह

Correct Answer -

There are all of coral origins सभी कोरल मल क ह

What do the basalt layers of the Deccan indicate डन की बसा परत ा इिगत करती ह

1 All of the above उपरो सभी

2 Huge volcanic eruptions in the distant past दरथ अतीत म िवशाल ालामखीय िवोट

3 The immense erosional activity of the rivers निदयो की िवशाल रण गितिविध

4 The influence of weathering मौसम का भाव

Correct Answer -

Huge volcanic eruptions in the distant past दरथ अतीत म िवशाल ालामखीय िवोट

In the structure of planet Earth below the mantle the core is mainly made up of_____

पी ह की सरचना म मटल क नीच कोर म प स______ स िनिमत होती ह

1 aluminium एमीिनयम

2 silicon िसिलकॉन

3 chromium ोिमयम

4 iron लोहा

Correct Answer -

iron लोहा

One of the major Mid Oceanic Ridge is found in मख म-महासागर चोिटयो म स एक ______ म पायाजाता ह

1 Mid Pacific Ocean म शात महासागर

2 Mid Atlantic Ocean म अटलािटक महासागर

3 Mid Indian Ocean म भारतीय महासागर

4 Mid Arctic Ocean म आक िटक महासागर

68)

69)

70)

71)

Correct Answer -

Mid Atlantic Ocean म अटलािटक महासागर

Magma that reaches the Earthrsquos surface and then solidifies is called________

मा जो पी की सतह तक पचती ह और िफर ठोस हो जाती ह ________कहलाती ह

1 quartz ाटज

2 lava लावा

3 granite नाइट

4 silicates िसिलकट

Correct Answer -

lava लावा

Isotherms are the lines of equal_______ समताप रखाए समान _______की रखाए होती ह

1 pressure दाब

2 temperature तापमान

3 rainfall वषा

4 height ऊचाई

Correct Answer -

temperature तापमान

Mark the correct sequence of passes in the Western Ghats from north to south

पिमी घाटो म उर स दिण तक दर क सही अनम को िचित कर

1 Thalghat Palghat Bhorghat थलगघाट पालघाट भोरघाट

2 Thalghat Bhorghat Palghat थलघाट भोरघाट पालघाट

3 Bhorghat Thalghat Palghat भोरघाट थलघाट पालघाट

4 Palghat Bhorghat Thalghat पालघाट भोरघाट थलघाट

Correct Answer -

Thalghat Bhorghat Palghat थलघाट भोरघाट पालघाट

Which of the following does not have influence over the climate in India

ि ि ि ी ी

72)

73)

िनिलखत म स िकसका भाव भारत की जलवाय पर नही पड़ता ह

1 Ocean currents सागर की लहर

2 Nearness to equator भम रखा स िनकटता

3 Monsoons मानसन

4 Presence of Indian ocean भारतीय महासागर की उपथित

Correct Answer -

Ocean currents सागर की लहर

Which of the following cloud types has the characteristics like vertical tall narrow and puffy

िनिलखत म स िकस कार क मघो म लबवत लबी सकीण और थलता जसी िवशषताए ह

1 Cumulonimbus तफानी मघ

2 Cumulus मघ पज

3 Cirrocumulus पाभ कपासी मघ

4 Nimbostratus वषारी मघ

Correct Answer -

Cumulus मघ पज

Which of the following statement is INCORRECT about Crude Birth Rate

िनिलखत स कौन सा कथन अशोिधत ज दर क बार म सही नही ह

1 It cannot be used for comparing fertility level between two countries with different population characteristics

इसका उपयोग िविभ जनसा िवशषताओ वाल दो दशो क बीच जनन र की तलना क िलए नही िकया जा सकता ह

2 It is a standardized measure of fertility

यह जनन मता का मानकीकत उपाय ह

3 It is effected by the age-sex composition of the population

यह आबादी की आय-िलग सरचना स भािवत होता ह

4 It is expressed per 1000 population in a given geographical unit

यह िकसी दी गई भौगोिलक इकाई म ित 1000 जनसा पर िकया जाता ह

Correct Answer -

It is a standardized measure of fertility

यह जनन मता का मानकीकत उपाय ह

74)

75)

76)

77)

Which of the following state in India experienced negative decadal growth rate during 2001 to 2011census

भारत म िनिलखत म स िकस रा म वष 2001 स 2011 की जनगणना क दौरान नकाराक िगरावट दर ई

1 Tripura िपरा

2 Nagaland नागालड

3 Haryana हरयाणा

4 Odisha ओिडसा

Correct Answer -

Nagaland नागालड

Which of the following is NOT a characteristic of peninsular rivers

िनिलखत म स कौन सी िवशषता ायीपीय निदयो म नही होती ह

1 Flow through shallow valleys उथल घािटयो क माम स वाह

2 Seasonal flow मौसमी वाह

3 Little erosional activity थोड़ी कटावदार गितिविध

4 Meandering tendency often shifting their beds घमावदार वि अर अपन तटो को थानातरत करना

Correct Answer -

Meandering tendency often shifting their beds घमावदार वि अर अपन तटो को थानातरत करना

Which of the following gases in the atmosphere absorbs heat from the Sunrsquos radiation and the Earthssurface

वायमडल म िनिलखत म स कौन सी गस सय क िविकरण और पी की सतह स ऊा को अवशोिषत करती ह

1 Neon िनयॉन

2 Carbon dioxide काबन डाइऑाइड

3 Argon आगन

4 Nitrogen नाइट ोजन

Correct Answer -

Carbon dioxide काबन डाइऑाइड

Which of the following kind of settlement pattern is found at the confluence of rivers

ि ि ि ि ो

78)

79)

80)

िनिलखत म स िकस कार का वथापन पटन निदयो क सगम पर पाया जाता ह

1 Triangular Paern िकोणीय पटन

2 Circular or Semi-Circular Paern परप या अध-परप पटन

3 Nebular Paern नबलर पटन

4 Star ndashShaped Paern ार-आकार का पटन

Correct Answer -

Triangular Paern िकोणीय पटन

Which one was not the objective of the Biosphere Reserve Projects launched by the UNESCO

यनो ारा श की गई सरित जवमडल परयोजनाओ का उ इनम स कौन सा नही था

1 To promote teaching and research िशण और अनसधान को बढ़ावा दना

2 To make agriculture sustainable किष को दीघकािलक बनाना

3 To conserve ecosystems पारथितक त को सरित करना

4 To conserve genetic diversity for a longtime लब समय तक अनवािशक िविवधता को सरित करना

Correct Answer -

To make agriculture sustainable किष को दीघकािलक बनाना

Which region of the Earth surface receives the highest amount of insulation

पी सतह का कौन सा तापावरोधन की उतम माा ा करता ह

1 Land mass थलखड

2 Savannah region सवाना

3 Water bodies जल िनकाय

4 Tropical desert उकिटबधीय रिगान

Correct Answer -

Tropical desert उकिटबधीय रिगान

Which one of the following is not a biodiversity hotspot

िनिलखत म स कौन सा जव िविवधता का म जगह नही ह

1 Eastern Himalaya पव िहमालय

2 Eastern Ghats पव घाट

81)

82)

83)

3 Indo-Myanmar भारत-ामार

4 Westerm Ghats पिमी घाट

Correct Answer -

Eastern Ghats पव घाट

Which one of the following is NOT a part of the World Network of Biosphere Reserves based on theUNESCO Man and Biosphere Programme

यनो मन और बायोीयर कायम क आधार पर िनिलखत म स कौन बायोीयर रजव क िव नटवक कािहा नही ह

1 Gulf of Mannar मार की खाड़ी

2 Seshachalam शषाचलम

3 Sunderban सदरबन

4 Nilgiri नीलिगर

Correct Answer -

Seshachalam शषाचलम

Which one of the following is an example of ldquodesert vegetationrdquo

िनिलखत म स कौन मथलीय वनित का एक उदाहरण ह

1 Mosses and lichens दलदल और शवाल

2 Temperate grassland समशीतो घास क मदान

3 Coniferous forest शकधारी वन

4 Acacia and cactus एकािसया और कस

Correct Answer -

Acacia and cactus एकािसया और कस

Which one of the following reflects more sunlight िनिलखत म स कौन सा सय की रोशनी को अिधकपरावितत करता ह

1 Paddy crop land धान फसल भिम

2 Land covered with fresh snow ताजा बफ स आािदत भिम

3 Sand desert रतीली रिगान

4 Prairie land यरी भिम

84)

85)

86)

87)

Correct Answer -

Land covered with fresh snow ताजा बफ स आािदत भिम

Which layer of the atmosphere is in contact with the surface of the earthrsquos oceans

वायमडल की कौन सी परत पी क महासागरो की सतह क सपक म ह

1 Stratosphere समताप मडल

2 Mesosphere म मडल

3 Hydrosphere जलमडल

4 Troposphere ोभ मडल

Correct Answer -

Troposphere ोभ मडल

Mediterranean Sea is a border of which of the following countries भम सागर िनिलखत दशो म सिकसकी सीमा ह

1 None of these इनम स कोई नही

2 Iraq इराक

3 Lebanon लबनान

4 Jordan जॉडन

Correct Answer -

Lebanon लबनान

Benguela ocean currents are found along which coast बगएला महासागर धाराए िकस तट क साथ पायीजाती ह

1 East Coast of South America दिण अमरका क पव तट

2 East Coast of Africa अीका क पव तट

3 West Coast of South America दिण अमरका क पिमी तट

4 West Coast of Africa अीका क पिमी तट

Correct Answer -

West Coast of Africa अीका क पिमी तट

88)

89)

90)

Due to tension a block of land on one side being pushed up or upthrown relative to the downthrown blockis referred as

तनाव क कारण नीच फ क ए खड क साप भिम का एक खड एक ओर स ऊपर धकला जाता ह या ऊपर की ओरफ का जाता ह यह _____ क प म सदिभत ह

1 Thrust fault प श

2 Normal fault सामा श

3 Reverse fault म श

4 Strike slip fault नितलब सपण श

Correct Answer -

Normal fault सामा श

Inter-tropical doldrums is a zone of ______ अतर-उकिटबधीय डोलड ______ का एक ह

1 Frontolysis टोलायिसस

2 Convergence अिभसरण

3 Inter-tropical divergence zone अतर-उकिटबधीय िवचलन

4 Local wind थानीय वाय

Correct Answer -

Convergence अिभसरण

The Horse Latitudes are regions located at about _____ north and south of the equator

हॉस अाश भम रखा क उर और दिण म लगभग _____ पर थत ह

1 30ndash60 degree Latitude 30-60 िडी अाश

2 0ndash5 degree Latitude 0-5 िडी अाश

3 30 degree Latitude 30 िडी अाश

4 60ndash90 degree Latitude 60-90 िडी अाश

Correct Answer -

30 degree Latitude 30 िडी अाश

Generally evaporation is high over which part of the Earth

आम तौर पर पी क िकस भाग पर वाीकरण अिधक होता ह

1 Equatorial maritime भमवत समीय ी ी

91)

92)

2 Equatorial continental भमवत महाीपीय

3 Polar maritime वीय समीय

4 Polar continental वीय महाीपीय

Correct Answer -

Equatorial maritime भमवत समीय

A very high temperature during summer in north western India leads to what type of climaticcondition in south

उर पिमी भारत म गम क दौरान बत अिधक तापमान होन क कारण दिण म िकस कार की जलवाय थितउ करता ह

1 Depression over arabian sea अरब सागर पर अवनमन

2 Failure monsoon मानसन िवफलता

3 Successful monsoon मानसन सफलता

4 Cyclones चवात

Correct Answer -

Successful monsoon मानसन सफलता

Lightning and thunder are the resultant effect when तिड़त और गजन परणामी भाव ह जब

1 Two massive clouds hit powerfully each other first lightning is produced and later sound is produced

दो बड़ बादल एक दसर स शशाली ढग स टकरात ह पहल आकाशीय िवदयत उ होता ह और बाद म िन उहोती ह

2 Two massive clouds come into contact with the powerful wind collision this results into first sound and thenlightning

दो बड़ बादल शशाली पवन सघ क सपक म आत ह इसका परणामप पहल िन और िफर आकाशीय िवदयतउ होता ह

3 None of the above उपरो म स कोई भी नही

4 A high density cloud contains positively and negatively charged electric ions and when this interacts light andsound are simultaneously produced

एक उ घन बादल म धनाक और ऋणाक आविशत िवदयत आयन होत ह और जब यह परर भाव डालत ह तोकाश और िन एक साथ उािदत होती ह

Correct Answer -

A high density cloud contains positively and negatively charged electric ions and when this interacts light andsound are simultaneously produced

औ ि ि ो औ ो

93)

94)

95)

एक उ घन बादल म धनाक और ऋणाक आविशत िवदयत आयन होत ह और जब यह परर भाव डालत ह तोकाश और िन एक साथ उािदत होती ह

Doon Valley is able to grow rice because दन घाटी चावल उगान म सम ह ोिक

1 Other crops cannot be grown वहा अ फसलो को उगाया नही जा सकता ह

2 People in the valley are rice eaters घाटी म लोग चावल खान वाल ह

3 There is a huge export demand of rice वहा चावल की भारी िनयात माग ह

4 It has warm summer and snow melt waters for irrigation

वहा गिमया गम होती ह िसचाई क िलए बफ का िपघला आ पानी होता ह

Correct Answer -

It has warm summer and snow melt waters for irrigation

वहा गिमया गम होती ह िसचाई क िलए बफ का िपघला आ पानी होता ह

CANCELLED

In the geological time scale the Mesozoic Era DOES NOT contains which of the following periods

भगभय समय पमान पर मजीवी यग म िन कालो म स कौन नही ह

1 Triassic ट ाइऐिसक

2 Jurassic जरिसक

3 Cretaceous चाकमय

4 Carboniferous काबनी

Correct Answer -

Carboniferous काबनी

96)

1 P-3 Q-4 R-2 S-1

2 P-3 Q-4 R-1 S-2

3 P-3 Q-4 R-1 S-2

4 P-4 Q-3 R-2 S-1

Correct Answer -

P-4 Q-3 R-2 S-1

1 P-3 Q-1 R-4 S-2

2 P-3 Q-4 R-1 S-2

3 P-3 Q-2 R-4 S-1

97)

98)

4 P-2 Q-1 R-4 S-3

Correct Answer -

P-3 Q-4 R-1 S-2

1 P-3 Q-1 R-4 S-2

2 P-2 Q-3 R-4 S-1

3 P-2 Q-1 R-3 S-4

4 P-4 Q-2 R-1 S-3

Correct Answer -

P-3 Q-1 R-4 S-2

99)

100)

1 P-3 Q-2 R-4 S-1

2 P-1 Q-2 R-3 S-4

3 P-2 Q-3 R-1 S-4

4 P-4 Q-3 R-2 S-1

Correct Answer -

P-2 Q-3 R-1 S-4

ldquoHuman geography is the study of changing relationship between the unresting man and the unstableearthrdquo was defined by

lsquolsquoमानव भगोल ाकल आदमी और अथर पी क बीच सबध परवतन का अयन हrdquo ______ ारा परभािषत िकया गयाथा

1 J Brunches ज चस

2 EC Semple ईसी सल

3 HJ Mackinder एच ज मिकदर

4 PV Blache पीवी च

Correct Answer -

EC Semple ईसी सल

Sedimentary rocks are finally and ultimately derived from the____________

अवसादी चान अततः ________ स ा की जाती ह

1 action of earth movements पी की गितिविधयो

2 marine deposit समी िनप

3 weathering of metamorphic rocks पातरत चानो क अपय

4 weathering of igneous rocks आय चानो क अपय

Correct Answer -

weathering of igneous rocks आय चानो क अपय

Page 34: High School Teacher Eligibility Test- BOARD PROFESSIONAL ...peb.mp.gov.in/results/RESULT_18/HST_RES18/Final_anwser_key/HST… · M a ndl a / मंड ल ... Under the Madhya Pradesh

52)

53)

54)

1 Condensation सघनन

2 Evapotranspiration वान-उजन

3 Radiation िविकरण

4 Precipitation वषण

Correct Answer -

Evapotranspiration वान-उजन

The process through which the terrestrial heat is transferred to air by direct contact is termed as

वह िया िजसम सपक ारा थलीय ऊा वाय म थानातरत हो जाती ह ______ क प म जानी जाती ह

1 Conduction चालन

2 Convection सवहन

3 Insolation आतपन

4 Radiation िविकरण

Correct Answer -

Conduction चालन

The largest area under mangroves is in which of the following statesunion territory

मोव क अतगत िनिलखत राोसघ शािसत दशो म स सबस बड़ा कौन सा ह

1 Andaman and Nicobar अमान और िनकोबार

2 Andhra Pradesh आ दश

3 West Bengal पिम बगाल

4 Gujarat गजरात

Correct Answer -

West Bengal पिम बगाल

The longitudinal transverse and surface waves in an earthquake originate from

भकप म दशातर अनथ और सतह तरग यहा उ होती ह

1 The focus on the surface of the Earth पी क सतह पर क -िबद म

2 The focus within the body of the Earth पी क भीतर क -िबद म

3 The epicenter within the body of the Earth पी क भीतर उपरक म

55)

56)

57)

4 The epicenter on the surface of the Earth पी क सतह पर उपरक म

Correct Answer -

The focus within the body of the Earth पी क भीतर क -िबद म

The down slope movement of material due to gravity is called______

गाकषण क कारण पदाथ की अनढाल गित को ______ कहा जाता ह

1 mass movement पदाथ सचलन

2 deposition िनप

3 erosion रण

4 volcanic movement ालामखीय सचलन

Correct Answer -

mass movement पदाथ सचलन

Shimla is cooler than Amritsar although both are on the same latitude This is because

िशमला म अमतसर स अिधक ठड ह हालािक दोनो समान अाश पर ह ऐसा ह ोिक

1 Shimla is at a greater height above sea level than Amritsar अमतसर की तलना म िशमला सम तल स अिधकऊचाई पर ह

2 Shimla is further north िशमला उर की ओर ह

3 Shimla is farther from the equator िशमला भम रखा स आग ह

4 Their longitudes differ उनकी दशातर रखाए िभ ह

Correct Answer -

Shimla is at a greater height above sea level than Amritsar अमतसर की तलना म िशमला सम तल स अिधकऊचाई पर ह

lsquoTempo of Urbanizationrsquo measures which of the following

lsquoशहरीकरण का टपोrsquo िनिलखत म स कौन सा उपाय ह

1 Speed of urbanizaon शहरीकरण की गित

2 None of the above इनम स कोई नही

3 Inequality of urbanizaon शहरीकरण की असमानता

4 Current level of urbanizaon शहरीकरण का वतमान र

Correct Answer -

58)

59)

60)

Speed of urbanizaon शहरीकरण की गित

Out of the following options choose the INCORRECT statement

िनिलखत िवको म स गलत कथन का चयन कर

1 The clear tracts in the equatorial region recover rapidly भम रखा म भभाग तजी स ठीक हो जात ह

2 The stable communities include a redwood forest a pine forest at high elevations

थर समदायो म एक रडवड वन उ ऊचाई पर एक दवदार वन शािमल ह

3 Any ecosystem moves towards maximum biomass and stability to survive

कोई भी पारथितकी त जीिवत रहन क िलए अिधकतम जवसहित और थरता की तरफ असर होता ह

4 Tropical rain forests near equator are stable ecosystems

भम रखा क पास उकिटबधीय वषा वन थर पारथितक त ह

Correct Answer -

The clear tracts in the equatorial region recover rapidly भम रखा म भभाग तजी स ठीक हो जात ह

Seasonal contrasts are maximum in मौसमी िवषमता अिधकतम ह

1 Mid latitudes म अाश म

2 Low attitudes िन अाश म

3 High latitudes उ अाश म

4 Subtropics उपोकिटबधीय म

Correct Answer -

Mid latitudes म अाश म

In India which type of forest among the following occupies the largest area

भारत म िनिलखत म स िकस कार क वन सबस बड़ा फल आािदत करत ह

1 Sub-tropical Dry Evergreen Forest उप उकिटबधीय श सदाबहार वन

2 Mountain Wet Temperate Forest पवतीय आ शीतो वन

3 Tropical Moist Deciduous Forest उकिटबधीय आ पणपाती वन

4 Tropical Wet Evergreen Forest उकिटबधीय आ सदाबहार वन

Correct Answer -

Tropical Moist Deciduous Forest उकिटबधीय आ पणपाती वन

61)

62)

63)

64)

What is the proportion of lsquoJuvenile Populationrsquo (0-14 years) in India as per 2011Census

2011 की जनगणना क अनसार भारत म जवनाइल पॉपलशन यानी िकशोर जनस या (0-14 वष) का अनपात ाह

1 3076 of total population कल जनस या का 3076

2 2764 of total population कल जनस या का 2764

3 2933 of total population कल जनस या का 2933

4 3354 of total population कल जनस या का 3354

Correct Answer -

3076 of total population कल जनस या का 3076

What is the Belfast famous for बलफा िकसक िलए मशर ह

1 Belt of cotton textile industry कपास व उोग क

2 Ship-building industry जहाज िनमाण उोग

3 Agricultural machinery किष उपकरण

4 Aero planes manufacturing वाययान िनमाण

Correct Answer -

Ship-building industry जहाज िनमाण उोग

What is the most important occupation in tropical monsoon lands

उकिटबधीय मॉनसन भिम म सबस महपण वसाय ा ह

1 Mining खनन

2 Cattle rearing मवशी पालन

3 Agriculture किष

4 Nomadic herding नोमािडक जड़ी-बिटया

Correct Answer -

Agriculture किष

What is the most important characteristics of the islands (Indian) located in the Arabian sea

अरब सागर म थत ीपो (भारतीय) की सबस महपण िवशषता ा ह

ी ो

65)

66)

67)

1 There are all of coral origins सभी कोरल मल क ह

2 There are all very small in size य सभी आकार म बत छोट ह

3 They have a very dry climate इनकी जलवाय बत श ह

4 They are extended parts of the mainland व महाीप क िवारत िह ह

Correct Answer -

There are all of coral origins सभी कोरल मल क ह

What do the basalt layers of the Deccan indicate डन की बसा परत ा इिगत करती ह

1 All of the above उपरो सभी

2 Huge volcanic eruptions in the distant past दरथ अतीत म िवशाल ालामखीय िवोट

3 The immense erosional activity of the rivers निदयो की िवशाल रण गितिविध

4 The influence of weathering मौसम का भाव

Correct Answer -

Huge volcanic eruptions in the distant past दरथ अतीत म िवशाल ालामखीय िवोट

In the structure of planet Earth below the mantle the core is mainly made up of_____

पी ह की सरचना म मटल क नीच कोर म प स______ स िनिमत होती ह

1 aluminium एमीिनयम

2 silicon िसिलकॉन

3 chromium ोिमयम

4 iron लोहा

Correct Answer -

iron लोहा

One of the major Mid Oceanic Ridge is found in मख म-महासागर चोिटयो म स एक ______ म पायाजाता ह

1 Mid Pacific Ocean म शात महासागर

2 Mid Atlantic Ocean म अटलािटक महासागर

3 Mid Indian Ocean म भारतीय महासागर

4 Mid Arctic Ocean म आक िटक महासागर

68)

69)

70)

71)

Correct Answer -

Mid Atlantic Ocean म अटलािटक महासागर

Magma that reaches the Earthrsquos surface and then solidifies is called________

मा जो पी की सतह तक पचती ह और िफर ठोस हो जाती ह ________कहलाती ह

1 quartz ाटज

2 lava लावा

3 granite नाइट

4 silicates िसिलकट

Correct Answer -

lava लावा

Isotherms are the lines of equal_______ समताप रखाए समान _______की रखाए होती ह

1 pressure दाब

2 temperature तापमान

3 rainfall वषा

4 height ऊचाई

Correct Answer -

temperature तापमान

Mark the correct sequence of passes in the Western Ghats from north to south

पिमी घाटो म उर स दिण तक दर क सही अनम को िचित कर

1 Thalghat Palghat Bhorghat थलगघाट पालघाट भोरघाट

2 Thalghat Bhorghat Palghat थलघाट भोरघाट पालघाट

3 Bhorghat Thalghat Palghat भोरघाट थलघाट पालघाट

4 Palghat Bhorghat Thalghat पालघाट भोरघाट थलघाट

Correct Answer -

Thalghat Bhorghat Palghat थलघाट भोरघाट पालघाट

Which of the following does not have influence over the climate in India

ि ि ि ी ी

72)

73)

िनिलखत म स िकसका भाव भारत की जलवाय पर नही पड़ता ह

1 Ocean currents सागर की लहर

2 Nearness to equator भम रखा स िनकटता

3 Monsoons मानसन

4 Presence of Indian ocean भारतीय महासागर की उपथित

Correct Answer -

Ocean currents सागर की लहर

Which of the following cloud types has the characteristics like vertical tall narrow and puffy

िनिलखत म स िकस कार क मघो म लबवत लबी सकीण और थलता जसी िवशषताए ह

1 Cumulonimbus तफानी मघ

2 Cumulus मघ पज

3 Cirrocumulus पाभ कपासी मघ

4 Nimbostratus वषारी मघ

Correct Answer -

Cumulus मघ पज

Which of the following statement is INCORRECT about Crude Birth Rate

िनिलखत स कौन सा कथन अशोिधत ज दर क बार म सही नही ह

1 It cannot be used for comparing fertility level between two countries with different population characteristics

इसका उपयोग िविभ जनसा िवशषताओ वाल दो दशो क बीच जनन र की तलना क िलए नही िकया जा सकता ह

2 It is a standardized measure of fertility

यह जनन मता का मानकीकत उपाय ह

3 It is effected by the age-sex composition of the population

यह आबादी की आय-िलग सरचना स भािवत होता ह

4 It is expressed per 1000 population in a given geographical unit

यह िकसी दी गई भौगोिलक इकाई म ित 1000 जनसा पर िकया जाता ह

Correct Answer -

It is a standardized measure of fertility

यह जनन मता का मानकीकत उपाय ह

74)

75)

76)

77)

Which of the following state in India experienced negative decadal growth rate during 2001 to 2011census

भारत म िनिलखत म स िकस रा म वष 2001 स 2011 की जनगणना क दौरान नकाराक िगरावट दर ई

1 Tripura िपरा

2 Nagaland नागालड

3 Haryana हरयाणा

4 Odisha ओिडसा

Correct Answer -

Nagaland नागालड

Which of the following is NOT a characteristic of peninsular rivers

िनिलखत म स कौन सी िवशषता ायीपीय निदयो म नही होती ह

1 Flow through shallow valleys उथल घािटयो क माम स वाह

2 Seasonal flow मौसमी वाह

3 Little erosional activity थोड़ी कटावदार गितिविध

4 Meandering tendency often shifting their beds घमावदार वि अर अपन तटो को थानातरत करना

Correct Answer -

Meandering tendency often shifting their beds घमावदार वि अर अपन तटो को थानातरत करना

Which of the following gases in the atmosphere absorbs heat from the Sunrsquos radiation and the Earthssurface

वायमडल म िनिलखत म स कौन सी गस सय क िविकरण और पी की सतह स ऊा को अवशोिषत करती ह

1 Neon िनयॉन

2 Carbon dioxide काबन डाइऑाइड

3 Argon आगन

4 Nitrogen नाइट ोजन

Correct Answer -

Carbon dioxide काबन डाइऑाइड

Which of the following kind of settlement pattern is found at the confluence of rivers

ि ि ि ि ो

78)

79)

80)

िनिलखत म स िकस कार का वथापन पटन निदयो क सगम पर पाया जाता ह

1 Triangular Paern िकोणीय पटन

2 Circular or Semi-Circular Paern परप या अध-परप पटन

3 Nebular Paern नबलर पटन

4 Star ndashShaped Paern ार-आकार का पटन

Correct Answer -

Triangular Paern िकोणीय पटन

Which one was not the objective of the Biosphere Reserve Projects launched by the UNESCO

यनो ारा श की गई सरित जवमडल परयोजनाओ का उ इनम स कौन सा नही था

1 To promote teaching and research िशण और अनसधान को बढ़ावा दना

2 To make agriculture sustainable किष को दीघकािलक बनाना

3 To conserve ecosystems पारथितक त को सरित करना

4 To conserve genetic diversity for a longtime लब समय तक अनवािशक िविवधता को सरित करना

Correct Answer -

To make agriculture sustainable किष को दीघकािलक बनाना

Which region of the Earth surface receives the highest amount of insulation

पी सतह का कौन सा तापावरोधन की उतम माा ा करता ह

1 Land mass थलखड

2 Savannah region सवाना

3 Water bodies जल िनकाय

4 Tropical desert उकिटबधीय रिगान

Correct Answer -

Tropical desert उकिटबधीय रिगान

Which one of the following is not a biodiversity hotspot

िनिलखत म स कौन सा जव िविवधता का म जगह नही ह

1 Eastern Himalaya पव िहमालय

2 Eastern Ghats पव घाट

81)

82)

83)

3 Indo-Myanmar भारत-ामार

4 Westerm Ghats पिमी घाट

Correct Answer -

Eastern Ghats पव घाट

Which one of the following is NOT a part of the World Network of Biosphere Reserves based on theUNESCO Man and Biosphere Programme

यनो मन और बायोीयर कायम क आधार पर िनिलखत म स कौन बायोीयर रजव क िव नटवक कािहा नही ह

1 Gulf of Mannar मार की खाड़ी

2 Seshachalam शषाचलम

3 Sunderban सदरबन

4 Nilgiri नीलिगर

Correct Answer -

Seshachalam शषाचलम

Which one of the following is an example of ldquodesert vegetationrdquo

िनिलखत म स कौन मथलीय वनित का एक उदाहरण ह

1 Mosses and lichens दलदल और शवाल

2 Temperate grassland समशीतो घास क मदान

3 Coniferous forest शकधारी वन

4 Acacia and cactus एकािसया और कस

Correct Answer -

Acacia and cactus एकािसया और कस

Which one of the following reflects more sunlight िनिलखत म स कौन सा सय की रोशनी को अिधकपरावितत करता ह

1 Paddy crop land धान फसल भिम

2 Land covered with fresh snow ताजा बफ स आािदत भिम

3 Sand desert रतीली रिगान

4 Prairie land यरी भिम

84)

85)

86)

87)

Correct Answer -

Land covered with fresh snow ताजा बफ स आािदत भिम

Which layer of the atmosphere is in contact with the surface of the earthrsquos oceans

वायमडल की कौन सी परत पी क महासागरो की सतह क सपक म ह

1 Stratosphere समताप मडल

2 Mesosphere म मडल

3 Hydrosphere जलमडल

4 Troposphere ोभ मडल

Correct Answer -

Troposphere ोभ मडल

Mediterranean Sea is a border of which of the following countries भम सागर िनिलखत दशो म सिकसकी सीमा ह

1 None of these इनम स कोई नही

2 Iraq इराक

3 Lebanon लबनान

4 Jordan जॉडन

Correct Answer -

Lebanon लबनान

Benguela ocean currents are found along which coast बगएला महासागर धाराए िकस तट क साथ पायीजाती ह

1 East Coast of South America दिण अमरका क पव तट

2 East Coast of Africa अीका क पव तट

3 West Coast of South America दिण अमरका क पिमी तट

4 West Coast of Africa अीका क पिमी तट

Correct Answer -

West Coast of Africa अीका क पिमी तट

88)

89)

90)

Due to tension a block of land on one side being pushed up or upthrown relative to the downthrown blockis referred as

तनाव क कारण नीच फ क ए खड क साप भिम का एक खड एक ओर स ऊपर धकला जाता ह या ऊपर की ओरफ का जाता ह यह _____ क प म सदिभत ह

1 Thrust fault प श

2 Normal fault सामा श

3 Reverse fault म श

4 Strike slip fault नितलब सपण श

Correct Answer -

Normal fault सामा श

Inter-tropical doldrums is a zone of ______ अतर-उकिटबधीय डोलड ______ का एक ह

1 Frontolysis टोलायिसस

2 Convergence अिभसरण

3 Inter-tropical divergence zone अतर-उकिटबधीय िवचलन

4 Local wind थानीय वाय

Correct Answer -

Convergence अिभसरण

The Horse Latitudes are regions located at about _____ north and south of the equator

हॉस अाश भम रखा क उर और दिण म लगभग _____ पर थत ह

1 30ndash60 degree Latitude 30-60 िडी अाश

2 0ndash5 degree Latitude 0-5 िडी अाश

3 30 degree Latitude 30 िडी अाश

4 60ndash90 degree Latitude 60-90 िडी अाश

Correct Answer -

30 degree Latitude 30 िडी अाश

Generally evaporation is high over which part of the Earth

आम तौर पर पी क िकस भाग पर वाीकरण अिधक होता ह

1 Equatorial maritime भमवत समीय ी ी

91)

92)

2 Equatorial continental भमवत महाीपीय

3 Polar maritime वीय समीय

4 Polar continental वीय महाीपीय

Correct Answer -

Equatorial maritime भमवत समीय

A very high temperature during summer in north western India leads to what type of climaticcondition in south

उर पिमी भारत म गम क दौरान बत अिधक तापमान होन क कारण दिण म िकस कार की जलवाय थितउ करता ह

1 Depression over arabian sea अरब सागर पर अवनमन

2 Failure monsoon मानसन िवफलता

3 Successful monsoon मानसन सफलता

4 Cyclones चवात

Correct Answer -

Successful monsoon मानसन सफलता

Lightning and thunder are the resultant effect when तिड़त और गजन परणामी भाव ह जब

1 Two massive clouds hit powerfully each other first lightning is produced and later sound is produced

दो बड़ बादल एक दसर स शशाली ढग स टकरात ह पहल आकाशीय िवदयत उ होता ह और बाद म िन उहोती ह

2 Two massive clouds come into contact with the powerful wind collision this results into first sound and thenlightning

दो बड़ बादल शशाली पवन सघ क सपक म आत ह इसका परणामप पहल िन और िफर आकाशीय िवदयतउ होता ह

3 None of the above उपरो म स कोई भी नही

4 A high density cloud contains positively and negatively charged electric ions and when this interacts light andsound are simultaneously produced

एक उ घन बादल म धनाक और ऋणाक आविशत िवदयत आयन होत ह और जब यह परर भाव डालत ह तोकाश और िन एक साथ उािदत होती ह

Correct Answer -

A high density cloud contains positively and negatively charged electric ions and when this interacts light andsound are simultaneously produced

औ ि ि ो औ ो

93)

94)

95)

एक उ घन बादल म धनाक और ऋणाक आविशत िवदयत आयन होत ह और जब यह परर भाव डालत ह तोकाश और िन एक साथ उािदत होती ह

Doon Valley is able to grow rice because दन घाटी चावल उगान म सम ह ोिक

1 Other crops cannot be grown वहा अ फसलो को उगाया नही जा सकता ह

2 People in the valley are rice eaters घाटी म लोग चावल खान वाल ह

3 There is a huge export demand of rice वहा चावल की भारी िनयात माग ह

4 It has warm summer and snow melt waters for irrigation

वहा गिमया गम होती ह िसचाई क िलए बफ का िपघला आ पानी होता ह

Correct Answer -

It has warm summer and snow melt waters for irrigation

वहा गिमया गम होती ह िसचाई क िलए बफ का िपघला आ पानी होता ह

CANCELLED

In the geological time scale the Mesozoic Era DOES NOT contains which of the following periods

भगभय समय पमान पर मजीवी यग म िन कालो म स कौन नही ह

1 Triassic ट ाइऐिसक

2 Jurassic जरिसक

3 Cretaceous चाकमय

4 Carboniferous काबनी

Correct Answer -

Carboniferous काबनी

96)

1 P-3 Q-4 R-2 S-1

2 P-3 Q-4 R-1 S-2

3 P-3 Q-4 R-1 S-2

4 P-4 Q-3 R-2 S-1

Correct Answer -

P-4 Q-3 R-2 S-1

1 P-3 Q-1 R-4 S-2

2 P-3 Q-4 R-1 S-2

3 P-3 Q-2 R-4 S-1

97)

98)

4 P-2 Q-1 R-4 S-3

Correct Answer -

P-3 Q-4 R-1 S-2

1 P-3 Q-1 R-4 S-2

2 P-2 Q-3 R-4 S-1

3 P-2 Q-1 R-3 S-4

4 P-4 Q-2 R-1 S-3

Correct Answer -

P-3 Q-1 R-4 S-2

99)

100)

1 P-3 Q-2 R-4 S-1

2 P-1 Q-2 R-3 S-4

3 P-2 Q-3 R-1 S-4

4 P-4 Q-3 R-2 S-1

Correct Answer -

P-2 Q-3 R-1 S-4

ldquoHuman geography is the study of changing relationship between the unresting man and the unstableearthrdquo was defined by

lsquolsquoमानव भगोल ाकल आदमी और अथर पी क बीच सबध परवतन का अयन हrdquo ______ ारा परभािषत िकया गयाथा

1 J Brunches ज चस

2 EC Semple ईसी सल

3 HJ Mackinder एच ज मिकदर

4 PV Blache पीवी च

Correct Answer -

EC Semple ईसी सल

Sedimentary rocks are finally and ultimately derived from the____________

अवसादी चान अततः ________ स ा की जाती ह

1 action of earth movements पी की गितिविधयो

2 marine deposit समी िनप

3 weathering of metamorphic rocks पातरत चानो क अपय

4 weathering of igneous rocks आय चानो क अपय

Correct Answer -

weathering of igneous rocks आय चानो क अपय

Page 35: High School Teacher Eligibility Test- BOARD PROFESSIONAL ...peb.mp.gov.in/results/RESULT_18/HST_RES18/Final_anwser_key/HST… · M a ndl a / मंड ल ... Under the Madhya Pradesh

55)

56)

57)

4 The epicenter on the surface of the Earth पी क सतह पर उपरक म

Correct Answer -

The focus within the body of the Earth पी क भीतर क -िबद म

The down slope movement of material due to gravity is called______

गाकषण क कारण पदाथ की अनढाल गित को ______ कहा जाता ह

1 mass movement पदाथ सचलन

2 deposition िनप

3 erosion रण

4 volcanic movement ालामखीय सचलन

Correct Answer -

mass movement पदाथ सचलन

Shimla is cooler than Amritsar although both are on the same latitude This is because

िशमला म अमतसर स अिधक ठड ह हालािक दोनो समान अाश पर ह ऐसा ह ोिक

1 Shimla is at a greater height above sea level than Amritsar अमतसर की तलना म िशमला सम तल स अिधकऊचाई पर ह

2 Shimla is further north िशमला उर की ओर ह

3 Shimla is farther from the equator िशमला भम रखा स आग ह

4 Their longitudes differ उनकी दशातर रखाए िभ ह

Correct Answer -

Shimla is at a greater height above sea level than Amritsar अमतसर की तलना म िशमला सम तल स अिधकऊचाई पर ह

lsquoTempo of Urbanizationrsquo measures which of the following

lsquoशहरीकरण का टपोrsquo िनिलखत म स कौन सा उपाय ह

1 Speed of urbanizaon शहरीकरण की गित

2 None of the above इनम स कोई नही

3 Inequality of urbanizaon शहरीकरण की असमानता

4 Current level of urbanizaon शहरीकरण का वतमान र

Correct Answer -

58)

59)

60)

Speed of urbanizaon शहरीकरण की गित

Out of the following options choose the INCORRECT statement

िनिलखत िवको म स गलत कथन का चयन कर

1 The clear tracts in the equatorial region recover rapidly भम रखा म भभाग तजी स ठीक हो जात ह

2 The stable communities include a redwood forest a pine forest at high elevations

थर समदायो म एक रडवड वन उ ऊचाई पर एक दवदार वन शािमल ह

3 Any ecosystem moves towards maximum biomass and stability to survive

कोई भी पारथितकी त जीिवत रहन क िलए अिधकतम जवसहित और थरता की तरफ असर होता ह

4 Tropical rain forests near equator are stable ecosystems

भम रखा क पास उकिटबधीय वषा वन थर पारथितक त ह

Correct Answer -

The clear tracts in the equatorial region recover rapidly भम रखा म भभाग तजी स ठीक हो जात ह

Seasonal contrasts are maximum in मौसमी िवषमता अिधकतम ह

1 Mid latitudes म अाश म

2 Low attitudes िन अाश म

3 High latitudes उ अाश म

4 Subtropics उपोकिटबधीय म

Correct Answer -

Mid latitudes म अाश म

In India which type of forest among the following occupies the largest area

भारत म िनिलखत म स िकस कार क वन सबस बड़ा फल आािदत करत ह

1 Sub-tropical Dry Evergreen Forest उप उकिटबधीय श सदाबहार वन

2 Mountain Wet Temperate Forest पवतीय आ शीतो वन

3 Tropical Moist Deciduous Forest उकिटबधीय आ पणपाती वन

4 Tropical Wet Evergreen Forest उकिटबधीय आ सदाबहार वन

Correct Answer -

Tropical Moist Deciduous Forest उकिटबधीय आ पणपाती वन

61)

62)

63)

64)

What is the proportion of lsquoJuvenile Populationrsquo (0-14 years) in India as per 2011Census

2011 की जनगणना क अनसार भारत म जवनाइल पॉपलशन यानी िकशोर जनस या (0-14 वष) का अनपात ाह

1 3076 of total population कल जनस या का 3076

2 2764 of total population कल जनस या का 2764

3 2933 of total population कल जनस या का 2933

4 3354 of total population कल जनस या का 3354

Correct Answer -

3076 of total population कल जनस या का 3076

What is the Belfast famous for बलफा िकसक िलए मशर ह

1 Belt of cotton textile industry कपास व उोग क

2 Ship-building industry जहाज िनमाण उोग

3 Agricultural machinery किष उपकरण

4 Aero planes manufacturing वाययान िनमाण

Correct Answer -

Ship-building industry जहाज िनमाण उोग

What is the most important occupation in tropical monsoon lands

उकिटबधीय मॉनसन भिम म सबस महपण वसाय ा ह

1 Mining खनन

2 Cattle rearing मवशी पालन

3 Agriculture किष

4 Nomadic herding नोमािडक जड़ी-बिटया

Correct Answer -

Agriculture किष

What is the most important characteristics of the islands (Indian) located in the Arabian sea

अरब सागर म थत ीपो (भारतीय) की सबस महपण िवशषता ा ह

ी ो

65)

66)

67)

1 There are all of coral origins सभी कोरल मल क ह

2 There are all very small in size य सभी आकार म बत छोट ह

3 They have a very dry climate इनकी जलवाय बत श ह

4 They are extended parts of the mainland व महाीप क िवारत िह ह

Correct Answer -

There are all of coral origins सभी कोरल मल क ह

What do the basalt layers of the Deccan indicate डन की बसा परत ा इिगत करती ह

1 All of the above उपरो सभी

2 Huge volcanic eruptions in the distant past दरथ अतीत म िवशाल ालामखीय िवोट

3 The immense erosional activity of the rivers निदयो की िवशाल रण गितिविध

4 The influence of weathering मौसम का भाव

Correct Answer -

Huge volcanic eruptions in the distant past दरथ अतीत म िवशाल ालामखीय िवोट

In the structure of planet Earth below the mantle the core is mainly made up of_____

पी ह की सरचना म मटल क नीच कोर म प स______ स िनिमत होती ह

1 aluminium एमीिनयम

2 silicon िसिलकॉन

3 chromium ोिमयम

4 iron लोहा

Correct Answer -

iron लोहा

One of the major Mid Oceanic Ridge is found in मख म-महासागर चोिटयो म स एक ______ म पायाजाता ह

1 Mid Pacific Ocean म शात महासागर

2 Mid Atlantic Ocean म अटलािटक महासागर

3 Mid Indian Ocean म भारतीय महासागर

4 Mid Arctic Ocean म आक िटक महासागर

68)

69)

70)

71)

Correct Answer -

Mid Atlantic Ocean म अटलािटक महासागर

Magma that reaches the Earthrsquos surface and then solidifies is called________

मा जो पी की सतह तक पचती ह और िफर ठोस हो जाती ह ________कहलाती ह

1 quartz ाटज

2 lava लावा

3 granite नाइट

4 silicates िसिलकट

Correct Answer -

lava लावा

Isotherms are the lines of equal_______ समताप रखाए समान _______की रखाए होती ह

1 pressure दाब

2 temperature तापमान

3 rainfall वषा

4 height ऊचाई

Correct Answer -

temperature तापमान

Mark the correct sequence of passes in the Western Ghats from north to south

पिमी घाटो म उर स दिण तक दर क सही अनम को िचित कर

1 Thalghat Palghat Bhorghat थलगघाट पालघाट भोरघाट

2 Thalghat Bhorghat Palghat थलघाट भोरघाट पालघाट

3 Bhorghat Thalghat Palghat भोरघाट थलघाट पालघाट

4 Palghat Bhorghat Thalghat पालघाट भोरघाट थलघाट

Correct Answer -

Thalghat Bhorghat Palghat थलघाट भोरघाट पालघाट

Which of the following does not have influence over the climate in India

ि ि ि ी ी

72)

73)

िनिलखत म स िकसका भाव भारत की जलवाय पर नही पड़ता ह

1 Ocean currents सागर की लहर

2 Nearness to equator भम रखा स िनकटता

3 Monsoons मानसन

4 Presence of Indian ocean भारतीय महासागर की उपथित

Correct Answer -

Ocean currents सागर की लहर

Which of the following cloud types has the characteristics like vertical tall narrow and puffy

िनिलखत म स िकस कार क मघो म लबवत लबी सकीण और थलता जसी िवशषताए ह

1 Cumulonimbus तफानी मघ

2 Cumulus मघ पज

3 Cirrocumulus पाभ कपासी मघ

4 Nimbostratus वषारी मघ

Correct Answer -

Cumulus मघ पज

Which of the following statement is INCORRECT about Crude Birth Rate

िनिलखत स कौन सा कथन अशोिधत ज दर क बार म सही नही ह

1 It cannot be used for comparing fertility level between two countries with different population characteristics

इसका उपयोग िविभ जनसा िवशषताओ वाल दो दशो क बीच जनन र की तलना क िलए नही िकया जा सकता ह

2 It is a standardized measure of fertility

यह जनन मता का मानकीकत उपाय ह

3 It is effected by the age-sex composition of the population

यह आबादी की आय-िलग सरचना स भािवत होता ह

4 It is expressed per 1000 population in a given geographical unit

यह िकसी दी गई भौगोिलक इकाई म ित 1000 जनसा पर िकया जाता ह

Correct Answer -

It is a standardized measure of fertility

यह जनन मता का मानकीकत उपाय ह

74)

75)

76)

77)

Which of the following state in India experienced negative decadal growth rate during 2001 to 2011census

भारत म िनिलखत म स िकस रा म वष 2001 स 2011 की जनगणना क दौरान नकाराक िगरावट दर ई

1 Tripura िपरा

2 Nagaland नागालड

3 Haryana हरयाणा

4 Odisha ओिडसा

Correct Answer -

Nagaland नागालड

Which of the following is NOT a characteristic of peninsular rivers

िनिलखत म स कौन सी िवशषता ायीपीय निदयो म नही होती ह

1 Flow through shallow valleys उथल घािटयो क माम स वाह

2 Seasonal flow मौसमी वाह

3 Little erosional activity थोड़ी कटावदार गितिविध

4 Meandering tendency often shifting their beds घमावदार वि अर अपन तटो को थानातरत करना

Correct Answer -

Meandering tendency often shifting their beds घमावदार वि अर अपन तटो को थानातरत करना

Which of the following gases in the atmosphere absorbs heat from the Sunrsquos radiation and the Earthssurface

वायमडल म िनिलखत म स कौन सी गस सय क िविकरण और पी की सतह स ऊा को अवशोिषत करती ह

1 Neon िनयॉन

2 Carbon dioxide काबन डाइऑाइड

3 Argon आगन

4 Nitrogen नाइट ोजन

Correct Answer -

Carbon dioxide काबन डाइऑाइड

Which of the following kind of settlement pattern is found at the confluence of rivers

ि ि ि ि ो

78)

79)

80)

िनिलखत म स िकस कार का वथापन पटन निदयो क सगम पर पाया जाता ह

1 Triangular Paern िकोणीय पटन

2 Circular or Semi-Circular Paern परप या अध-परप पटन

3 Nebular Paern नबलर पटन

4 Star ndashShaped Paern ार-आकार का पटन

Correct Answer -

Triangular Paern िकोणीय पटन

Which one was not the objective of the Biosphere Reserve Projects launched by the UNESCO

यनो ारा श की गई सरित जवमडल परयोजनाओ का उ इनम स कौन सा नही था

1 To promote teaching and research िशण और अनसधान को बढ़ावा दना

2 To make agriculture sustainable किष को दीघकािलक बनाना

3 To conserve ecosystems पारथितक त को सरित करना

4 To conserve genetic diversity for a longtime लब समय तक अनवािशक िविवधता को सरित करना

Correct Answer -

To make agriculture sustainable किष को दीघकािलक बनाना

Which region of the Earth surface receives the highest amount of insulation

पी सतह का कौन सा तापावरोधन की उतम माा ा करता ह

1 Land mass थलखड

2 Savannah region सवाना

3 Water bodies जल िनकाय

4 Tropical desert उकिटबधीय रिगान

Correct Answer -

Tropical desert उकिटबधीय रिगान

Which one of the following is not a biodiversity hotspot

िनिलखत म स कौन सा जव िविवधता का म जगह नही ह

1 Eastern Himalaya पव िहमालय

2 Eastern Ghats पव घाट

81)

82)

83)

3 Indo-Myanmar भारत-ामार

4 Westerm Ghats पिमी घाट

Correct Answer -

Eastern Ghats पव घाट

Which one of the following is NOT a part of the World Network of Biosphere Reserves based on theUNESCO Man and Biosphere Programme

यनो मन और बायोीयर कायम क आधार पर िनिलखत म स कौन बायोीयर रजव क िव नटवक कािहा नही ह

1 Gulf of Mannar मार की खाड़ी

2 Seshachalam शषाचलम

3 Sunderban सदरबन

4 Nilgiri नीलिगर

Correct Answer -

Seshachalam शषाचलम

Which one of the following is an example of ldquodesert vegetationrdquo

िनिलखत म स कौन मथलीय वनित का एक उदाहरण ह

1 Mosses and lichens दलदल और शवाल

2 Temperate grassland समशीतो घास क मदान

3 Coniferous forest शकधारी वन

4 Acacia and cactus एकािसया और कस

Correct Answer -

Acacia and cactus एकािसया और कस

Which one of the following reflects more sunlight िनिलखत म स कौन सा सय की रोशनी को अिधकपरावितत करता ह

1 Paddy crop land धान फसल भिम

2 Land covered with fresh snow ताजा बफ स आािदत भिम

3 Sand desert रतीली रिगान

4 Prairie land यरी भिम

84)

85)

86)

87)

Correct Answer -

Land covered with fresh snow ताजा बफ स आािदत भिम

Which layer of the atmosphere is in contact with the surface of the earthrsquos oceans

वायमडल की कौन सी परत पी क महासागरो की सतह क सपक म ह

1 Stratosphere समताप मडल

2 Mesosphere म मडल

3 Hydrosphere जलमडल

4 Troposphere ोभ मडल

Correct Answer -

Troposphere ोभ मडल

Mediterranean Sea is a border of which of the following countries भम सागर िनिलखत दशो म सिकसकी सीमा ह

1 None of these इनम स कोई नही

2 Iraq इराक

3 Lebanon लबनान

4 Jordan जॉडन

Correct Answer -

Lebanon लबनान

Benguela ocean currents are found along which coast बगएला महासागर धाराए िकस तट क साथ पायीजाती ह

1 East Coast of South America दिण अमरका क पव तट

2 East Coast of Africa अीका क पव तट

3 West Coast of South America दिण अमरका क पिमी तट

4 West Coast of Africa अीका क पिमी तट

Correct Answer -

West Coast of Africa अीका क पिमी तट

88)

89)

90)

Due to tension a block of land on one side being pushed up or upthrown relative to the downthrown blockis referred as

तनाव क कारण नीच फ क ए खड क साप भिम का एक खड एक ओर स ऊपर धकला जाता ह या ऊपर की ओरफ का जाता ह यह _____ क प म सदिभत ह

1 Thrust fault प श

2 Normal fault सामा श

3 Reverse fault म श

4 Strike slip fault नितलब सपण श

Correct Answer -

Normal fault सामा श

Inter-tropical doldrums is a zone of ______ अतर-उकिटबधीय डोलड ______ का एक ह

1 Frontolysis टोलायिसस

2 Convergence अिभसरण

3 Inter-tropical divergence zone अतर-उकिटबधीय िवचलन

4 Local wind थानीय वाय

Correct Answer -

Convergence अिभसरण

The Horse Latitudes are regions located at about _____ north and south of the equator

हॉस अाश भम रखा क उर और दिण म लगभग _____ पर थत ह

1 30ndash60 degree Latitude 30-60 िडी अाश

2 0ndash5 degree Latitude 0-5 िडी अाश

3 30 degree Latitude 30 िडी अाश

4 60ndash90 degree Latitude 60-90 िडी अाश

Correct Answer -

30 degree Latitude 30 िडी अाश

Generally evaporation is high over which part of the Earth

आम तौर पर पी क िकस भाग पर वाीकरण अिधक होता ह

1 Equatorial maritime भमवत समीय ी ी

91)

92)

2 Equatorial continental भमवत महाीपीय

3 Polar maritime वीय समीय

4 Polar continental वीय महाीपीय

Correct Answer -

Equatorial maritime भमवत समीय

A very high temperature during summer in north western India leads to what type of climaticcondition in south

उर पिमी भारत म गम क दौरान बत अिधक तापमान होन क कारण दिण म िकस कार की जलवाय थितउ करता ह

1 Depression over arabian sea अरब सागर पर अवनमन

2 Failure monsoon मानसन िवफलता

3 Successful monsoon मानसन सफलता

4 Cyclones चवात

Correct Answer -

Successful monsoon मानसन सफलता

Lightning and thunder are the resultant effect when तिड़त और गजन परणामी भाव ह जब

1 Two massive clouds hit powerfully each other first lightning is produced and later sound is produced

दो बड़ बादल एक दसर स शशाली ढग स टकरात ह पहल आकाशीय िवदयत उ होता ह और बाद म िन उहोती ह

2 Two massive clouds come into contact with the powerful wind collision this results into first sound and thenlightning

दो बड़ बादल शशाली पवन सघ क सपक म आत ह इसका परणामप पहल िन और िफर आकाशीय िवदयतउ होता ह

3 None of the above उपरो म स कोई भी नही

4 A high density cloud contains positively and negatively charged electric ions and when this interacts light andsound are simultaneously produced

एक उ घन बादल म धनाक और ऋणाक आविशत िवदयत आयन होत ह और जब यह परर भाव डालत ह तोकाश और िन एक साथ उािदत होती ह

Correct Answer -

A high density cloud contains positively and negatively charged electric ions and when this interacts light andsound are simultaneously produced

औ ि ि ो औ ो

93)

94)

95)

एक उ घन बादल म धनाक और ऋणाक आविशत िवदयत आयन होत ह और जब यह परर भाव डालत ह तोकाश और िन एक साथ उािदत होती ह

Doon Valley is able to grow rice because दन घाटी चावल उगान म सम ह ोिक

1 Other crops cannot be grown वहा अ फसलो को उगाया नही जा सकता ह

2 People in the valley are rice eaters घाटी म लोग चावल खान वाल ह

3 There is a huge export demand of rice वहा चावल की भारी िनयात माग ह

4 It has warm summer and snow melt waters for irrigation

वहा गिमया गम होती ह िसचाई क िलए बफ का िपघला आ पानी होता ह

Correct Answer -

It has warm summer and snow melt waters for irrigation

वहा गिमया गम होती ह िसचाई क िलए बफ का िपघला आ पानी होता ह

CANCELLED

In the geological time scale the Mesozoic Era DOES NOT contains which of the following periods

भगभय समय पमान पर मजीवी यग म िन कालो म स कौन नही ह

1 Triassic ट ाइऐिसक

2 Jurassic जरिसक

3 Cretaceous चाकमय

4 Carboniferous काबनी

Correct Answer -

Carboniferous काबनी

96)

1 P-3 Q-4 R-2 S-1

2 P-3 Q-4 R-1 S-2

3 P-3 Q-4 R-1 S-2

4 P-4 Q-3 R-2 S-1

Correct Answer -

P-4 Q-3 R-2 S-1

1 P-3 Q-1 R-4 S-2

2 P-3 Q-4 R-1 S-2

3 P-3 Q-2 R-4 S-1

97)

98)

4 P-2 Q-1 R-4 S-3

Correct Answer -

P-3 Q-4 R-1 S-2

1 P-3 Q-1 R-4 S-2

2 P-2 Q-3 R-4 S-1

3 P-2 Q-1 R-3 S-4

4 P-4 Q-2 R-1 S-3

Correct Answer -

P-3 Q-1 R-4 S-2

99)

100)

1 P-3 Q-2 R-4 S-1

2 P-1 Q-2 R-3 S-4

3 P-2 Q-3 R-1 S-4

4 P-4 Q-3 R-2 S-1

Correct Answer -

P-2 Q-3 R-1 S-4

ldquoHuman geography is the study of changing relationship between the unresting man and the unstableearthrdquo was defined by

lsquolsquoमानव भगोल ाकल आदमी और अथर पी क बीच सबध परवतन का अयन हrdquo ______ ारा परभािषत िकया गयाथा

1 J Brunches ज चस

2 EC Semple ईसी सल

3 HJ Mackinder एच ज मिकदर

4 PV Blache पीवी च

Correct Answer -

EC Semple ईसी सल

Sedimentary rocks are finally and ultimately derived from the____________

अवसादी चान अततः ________ स ा की जाती ह

1 action of earth movements पी की गितिविधयो

2 marine deposit समी िनप

3 weathering of metamorphic rocks पातरत चानो क अपय

4 weathering of igneous rocks आय चानो क अपय

Correct Answer -

weathering of igneous rocks आय चानो क अपय

Page 36: High School Teacher Eligibility Test- BOARD PROFESSIONAL ...peb.mp.gov.in/results/RESULT_18/HST_RES18/Final_anwser_key/HST… · M a ndl a / मंड ल ... Under the Madhya Pradesh

58)

59)

60)

Speed of urbanizaon शहरीकरण की गित

Out of the following options choose the INCORRECT statement

िनिलखत िवको म स गलत कथन का चयन कर

1 The clear tracts in the equatorial region recover rapidly भम रखा म भभाग तजी स ठीक हो जात ह

2 The stable communities include a redwood forest a pine forest at high elevations

थर समदायो म एक रडवड वन उ ऊचाई पर एक दवदार वन शािमल ह

3 Any ecosystem moves towards maximum biomass and stability to survive

कोई भी पारथितकी त जीिवत रहन क िलए अिधकतम जवसहित और थरता की तरफ असर होता ह

4 Tropical rain forests near equator are stable ecosystems

भम रखा क पास उकिटबधीय वषा वन थर पारथितक त ह

Correct Answer -

The clear tracts in the equatorial region recover rapidly भम रखा म भभाग तजी स ठीक हो जात ह

Seasonal contrasts are maximum in मौसमी िवषमता अिधकतम ह

1 Mid latitudes म अाश म

2 Low attitudes िन अाश म

3 High latitudes उ अाश म

4 Subtropics उपोकिटबधीय म

Correct Answer -

Mid latitudes म अाश म

In India which type of forest among the following occupies the largest area

भारत म िनिलखत म स िकस कार क वन सबस बड़ा फल आािदत करत ह

1 Sub-tropical Dry Evergreen Forest उप उकिटबधीय श सदाबहार वन

2 Mountain Wet Temperate Forest पवतीय आ शीतो वन

3 Tropical Moist Deciduous Forest उकिटबधीय आ पणपाती वन

4 Tropical Wet Evergreen Forest उकिटबधीय आ सदाबहार वन

Correct Answer -

Tropical Moist Deciduous Forest उकिटबधीय आ पणपाती वन

61)

62)

63)

64)

What is the proportion of lsquoJuvenile Populationrsquo (0-14 years) in India as per 2011Census

2011 की जनगणना क अनसार भारत म जवनाइल पॉपलशन यानी िकशोर जनस या (0-14 वष) का अनपात ाह

1 3076 of total population कल जनस या का 3076

2 2764 of total population कल जनस या का 2764

3 2933 of total population कल जनस या का 2933

4 3354 of total population कल जनस या का 3354

Correct Answer -

3076 of total population कल जनस या का 3076

What is the Belfast famous for बलफा िकसक िलए मशर ह

1 Belt of cotton textile industry कपास व उोग क

2 Ship-building industry जहाज िनमाण उोग

3 Agricultural machinery किष उपकरण

4 Aero planes manufacturing वाययान िनमाण

Correct Answer -

Ship-building industry जहाज िनमाण उोग

What is the most important occupation in tropical monsoon lands

उकिटबधीय मॉनसन भिम म सबस महपण वसाय ा ह

1 Mining खनन

2 Cattle rearing मवशी पालन

3 Agriculture किष

4 Nomadic herding नोमािडक जड़ी-बिटया

Correct Answer -

Agriculture किष

What is the most important characteristics of the islands (Indian) located in the Arabian sea

अरब सागर म थत ीपो (भारतीय) की सबस महपण िवशषता ा ह

ी ो

65)

66)

67)

1 There are all of coral origins सभी कोरल मल क ह

2 There are all very small in size य सभी आकार म बत छोट ह

3 They have a very dry climate इनकी जलवाय बत श ह

4 They are extended parts of the mainland व महाीप क िवारत िह ह

Correct Answer -

There are all of coral origins सभी कोरल मल क ह

What do the basalt layers of the Deccan indicate डन की बसा परत ा इिगत करती ह

1 All of the above उपरो सभी

2 Huge volcanic eruptions in the distant past दरथ अतीत म िवशाल ालामखीय िवोट

3 The immense erosional activity of the rivers निदयो की िवशाल रण गितिविध

4 The influence of weathering मौसम का भाव

Correct Answer -

Huge volcanic eruptions in the distant past दरथ अतीत म िवशाल ालामखीय िवोट

In the structure of planet Earth below the mantle the core is mainly made up of_____

पी ह की सरचना म मटल क नीच कोर म प स______ स िनिमत होती ह

1 aluminium एमीिनयम

2 silicon िसिलकॉन

3 chromium ोिमयम

4 iron लोहा

Correct Answer -

iron लोहा

One of the major Mid Oceanic Ridge is found in मख म-महासागर चोिटयो म स एक ______ म पायाजाता ह

1 Mid Pacific Ocean म शात महासागर

2 Mid Atlantic Ocean म अटलािटक महासागर

3 Mid Indian Ocean म भारतीय महासागर

4 Mid Arctic Ocean म आक िटक महासागर

68)

69)

70)

71)

Correct Answer -

Mid Atlantic Ocean म अटलािटक महासागर

Magma that reaches the Earthrsquos surface and then solidifies is called________

मा जो पी की सतह तक पचती ह और िफर ठोस हो जाती ह ________कहलाती ह

1 quartz ाटज

2 lava लावा

3 granite नाइट

4 silicates िसिलकट

Correct Answer -

lava लावा

Isotherms are the lines of equal_______ समताप रखाए समान _______की रखाए होती ह

1 pressure दाब

2 temperature तापमान

3 rainfall वषा

4 height ऊचाई

Correct Answer -

temperature तापमान

Mark the correct sequence of passes in the Western Ghats from north to south

पिमी घाटो म उर स दिण तक दर क सही अनम को िचित कर

1 Thalghat Palghat Bhorghat थलगघाट पालघाट भोरघाट

2 Thalghat Bhorghat Palghat थलघाट भोरघाट पालघाट

3 Bhorghat Thalghat Palghat भोरघाट थलघाट पालघाट

4 Palghat Bhorghat Thalghat पालघाट भोरघाट थलघाट

Correct Answer -

Thalghat Bhorghat Palghat थलघाट भोरघाट पालघाट

Which of the following does not have influence over the climate in India

ि ि ि ी ी

72)

73)

िनिलखत म स िकसका भाव भारत की जलवाय पर नही पड़ता ह

1 Ocean currents सागर की लहर

2 Nearness to equator भम रखा स िनकटता

3 Monsoons मानसन

4 Presence of Indian ocean भारतीय महासागर की उपथित

Correct Answer -

Ocean currents सागर की लहर

Which of the following cloud types has the characteristics like vertical tall narrow and puffy

िनिलखत म स िकस कार क मघो म लबवत लबी सकीण और थलता जसी िवशषताए ह

1 Cumulonimbus तफानी मघ

2 Cumulus मघ पज

3 Cirrocumulus पाभ कपासी मघ

4 Nimbostratus वषारी मघ

Correct Answer -

Cumulus मघ पज

Which of the following statement is INCORRECT about Crude Birth Rate

िनिलखत स कौन सा कथन अशोिधत ज दर क बार म सही नही ह

1 It cannot be used for comparing fertility level between two countries with different population characteristics

इसका उपयोग िविभ जनसा िवशषताओ वाल दो दशो क बीच जनन र की तलना क िलए नही िकया जा सकता ह

2 It is a standardized measure of fertility

यह जनन मता का मानकीकत उपाय ह

3 It is effected by the age-sex composition of the population

यह आबादी की आय-िलग सरचना स भािवत होता ह

4 It is expressed per 1000 population in a given geographical unit

यह िकसी दी गई भौगोिलक इकाई म ित 1000 जनसा पर िकया जाता ह

Correct Answer -

It is a standardized measure of fertility

यह जनन मता का मानकीकत उपाय ह

74)

75)

76)

77)

Which of the following state in India experienced negative decadal growth rate during 2001 to 2011census

भारत म िनिलखत म स िकस रा म वष 2001 स 2011 की जनगणना क दौरान नकाराक िगरावट दर ई

1 Tripura िपरा

2 Nagaland नागालड

3 Haryana हरयाणा

4 Odisha ओिडसा

Correct Answer -

Nagaland नागालड

Which of the following is NOT a characteristic of peninsular rivers

िनिलखत म स कौन सी िवशषता ायीपीय निदयो म नही होती ह

1 Flow through shallow valleys उथल घािटयो क माम स वाह

2 Seasonal flow मौसमी वाह

3 Little erosional activity थोड़ी कटावदार गितिविध

4 Meandering tendency often shifting their beds घमावदार वि अर अपन तटो को थानातरत करना

Correct Answer -

Meandering tendency often shifting their beds घमावदार वि अर अपन तटो को थानातरत करना

Which of the following gases in the atmosphere absorbs heat from the Sunrsquos radiation and the Earthssurface

वायमडल म िनिलखत म स कौन सी गस सय क िविकरण और पी की सतह स ऊा को अवशोिषत करती ह

1 Neon िनयॉन

2 Carbon dioxide काबन डाइऑाइड

3 Argon आगन

4 Nitrogen नाइट ोजन

Correct Answer -

Carbon dioxide काबन डाइऑाइड

Which of the following kind of settlement pattern is found at the confluence of rivers

ि ि ि ि ो

78)

79)

80)

िनिलखत म स िकस कार का वथापन पटन निदयो क सगम पर पाया जाता ह

1 Triangular Paern िकोणीय पटन

2 Circular or Semi-Circular Paern परप या अध-परप पटन

3 Nebular Paern नबलर पटन

4 Star ndashShaped Paern ार-आकार का पटन

Correct Answer -

Triangular Paern िकोणीय पटन

Which one was not the objective of the Biosphere Reserve Projects launched by the UNESCO

यनो ारा श की गई सरित जवमडल परयोजनाओ का उ इनम स कौन सा नही था

1 To promote teaching and research िशण और अनसधान को बढ़ावा दना

2 To make agriculture sustainable किष को दीघकािलक बनाना

3 To conserve ecosystems पारथितक त को सरित करना

4 To conserve genetic diversity for a longtime लब समय तक अनवािशक िविवधता को सरित करना

Correct Answer -

To make agriculture sustainable किष को दीघकािलक बनाना

Which region of the Earth surface receives the highest amount of insulation

पी सतह का कौन सा तापावरोधन की उतम माा ा करता ह

1 Land mass थलखड

2 Savannah region सवाना

3 Water bodies जल िनकाय

4 Tropical desert उकिटबधीय रिगान

Correct Answer -

Tropical desert उकिटबधीय रिगान

Which one of the following is not a biodiversity hotspot

िनिलखत म स कौन सा जव िविवधता का म जगह नही ह

1 Eastern Himalaya पव िहमालय

2 Eastern Ghats पव घाट

81)

82)

83)

3 Indo-Myanmar भारत-ामार

4 Westerm Ghats पिमी घाट

Correct Answer -

Eastern Ghats पव घाट

Which one of the following is NOT a part of the World Network of Biosphere Reserves based on theUNESCO Man and Biosphere Programme

यनो मन और बायोीयर कायम क आधार पर िनिलखत म स कौन बायोीयर रजव क िव नटवक कािहा नही ह

1 Gulf of Mannar मार की खाड़ी

2 Seshachalam शषाचलम

3 Sunderban सदरबन

4 Nilgiri नीलिगर

Correct Answer -

Seshachalam शषाचलम

Which one of the following is an example of ldquodesert vegetationrdquo

िनिलखत म स कौन मथलीय वनित का एक उदाहरण ह

1 Mosses and lichens दलदल और शवाल

2 Temperate grassland समशीतो घास क मदान

3 Coniferous forest शकधारी वन

4 Acacia and cactus एकािसया और कस

Correct Answer -

Acacia and cactus एकािसया और कस

Which one of the following reflects more sunlight िनिलखत म स कौन सा सय की रोशनी को अिधकपरावितत करता ह

1 Paddy crop land धान फसल भिम

2 Land covered with fresh snow ताजा बफ स आािदत भिम

3 Sand desert रतीली रिगान

4 Prairie land यरी भिम

84)

85)

86)

87)

Correct Answer -

Land covered with fresh snow ताजा बफ स आािदत भिम

Which layer of the atmosphere is in contact with the surface of the earthrsquos oceans

वायमडल की कौन सी परत पी क महासागरो की सतह क सपक म ह

1 Stratosphere समताप मडल

2 Mesosphere म मडल

3 Hydrosphere जलमडल

4 Troposphere ोभ मडल

Correct Answer -

Troposphere ोभ मडल

Mediterranean Sea is a border of which of the following countries भम सागर िनिलखत दशो म सिकसकी सीमा ह

1 None of these इनम स कोई नही

2 Iraq इराक

3 Lebanon लबनान

4 Jordan जॉडन

Correct Answer -

Lebanon लबनान

Benguela ocean currents are found along which coast बगएला महासागर धाराए िकस तट क साथ पायीजाती ह

1 East Coast of South America दिण अमरका क पव तट

2 East Coast of Africa अीका क पव तट

3 West Coast of South America दिण अमरका क पिमी तट

4 West Coast of Africa अीका क पिमी तट

Correct Answer -

West Coast of Africa अीका क पिमी तट

88)

89)

90)

Due to tension a block of land on one side being pushed up or upthrown relative to the downthrown blockis referred as

तनाव क कारण नीच फ क ए खड क साप भिम का एक खड एक ओर स ऊपर धकला जाता ह या ऊपर की ओरफ का जाता ह यह _____ क प म सदिभत ह

1 Thrust fault प श

2 Normal fault सामा श

3 Reverse fault म श

4 Strike slip fault नितलब सपण श

Correct Answer -

Normal fault सामा श

Inter-tropical doldrums is a zone of ______ अतर-उकिटबधीय डोलड ______ का एक ह

1 Frontolysis टोलायिसस

2 Convergence अिभसरण

3 Inter-tropical divergence zone अतर-उकिटबधीय िवचलन

4 Local wind थानीय वाय

Correct Answer -

Convergence अिभसरण

The Horse Latitudes are regions located at about _____ north and south of the equator

हॉस अाश भम रखा क उर और दिण म लगभग _____ पर थत ह

1 30ndash60 degree Latitude 30-60 िडी अाश

2 0ndash5 degree Latitude 0-5 िडी अाश

3 30 degree Latitude 30 िडी अाश

4 60ndash90 degree Latitude 60-90 िडी अाश

Correct Answer -

30 degree Latitude 30 िडी अाश

Generally evaporation is high over which part of the Earth

आम तौर पर पी क िकस भाग पर वाीकरण अिधक होता ह

1 Equatorial maritime भमवत समीय ी ी

91)

92)

2 Equatorial continental भमवत महाीपीय

3 Polar maritime वीय समीय

4 Polar continental वीय महाीपीय

Correct Answer -

Equatorial maritime भमवत समीय

A very high temperature during summer in north western India leads to what type of climaticcondition in south

उर पिमी भारत म गम क दौरान बत अिधक तापमान होन क कारण दिण म िकस कार की जलवाय थितउ करता ह

1 Depression over arabian sea अरब सागर पर अवनमन

2 Failure monsoon मानसन िवफलता

3 Successful monsoon मानसन सफलता

4 Cyclones चवात

Correct Answer -

Successful monsoon मानसन सफलता

Lightning and thunder are the resultant effect when तिड़त और गजन परणामी भाव ह जब

1 Two massive clouds hit powerfully each other first lightning is produced and later sound is produced

दो बड़ बादल एक दसर स शशाली ढग स टकरात ह पहल आकाशीय िवदयत उ होता ह और बाद म िन उहोती ह

2 Two massive clouds come into contact with the powerful wind collision this results into first sound and thenlightning

दो बड़ बादल शशाली पवन सघ क सपक म आत ह इसका परणामप पहल िन और िफर आकाशीय िवदयतउ होता ह

3 None of the above उपरो म स कोई भी नही

4 A high density cloud contains positively and negatively charged electric ions and when this interacts light andsound are simultaneously produced

एक उ घन बादल म धनाक और ऋणाक आविशत िवदयत आयन होत ह और जब यह परर भाव डालत ह तोकाश और िन एक साथ उािदत होती ह

Correct Answer -

A high density cloud contains positively and negatively charged electric ions and when this interacts light andsound are simultaneously produced

औ ि ि ो औ ो

93)

94)

95)

एक उ घन बादल म धनाक और ऋणाक आविशत िवदयत आयन होत ह और जब यह परर भाव डालत ह तोकाश और िन एक साथ उािदत होती ह

Doon Valley is able to grow rice because दन घाटी चावल उगान म सम ह ोिक

1 Other crops cannot be grown वहा अ फसलो को उगाया नही जा सकता ह

2 People in the valley are rice eaters घाटी म लोग चावल खान वाल ह

3 There is a huge export demand of rice वहा चावल की भारी िनयात माग ह

4 It has warm summer and snow melt waters for irrigation

वहा गिमया गम होती ह िसचाई क िलए बफ का िपघला आ पानी होता ह

Correct Answer -

It has warm summer and snow melt waters for irrigation

वहा गिमया गम होती ह िसचाई क िलए बफ का िपघला आ पानी होता ह

CANCELLED

In the geological time scale the Mesozoic Era DOES NOT contains which of the following periods

भगभय समय पमान पर मजीवी यग म िन कालो म स कौन नही ह

1 Triassic ट ाइऐिसक

2 Jurassic जरिसक

3 Cretaceous चाकमय

4 Carboniferous काबनी

Correct Answer -

Carboniferous काबनी

96)

1 P-3 Q-4 R-2 S-1

2 P-3 Q-4 R-1 S-2

3 P-3 Q-4 R-1 S-2

4 P-4 Q-3 R-2 S-1

Correct Answer -

P-4 Q-3 R-2 S-1

1 P-3 Q-1 R-4 S-2

2 P-3 Q-4 R-1 S-2

3 P-3 Q-2 R-4 S-1

97)

98)

4 P-2 Q-1 R-4 S-3

Correct Answer -

P-3 Q-4 R-1 S-2

1 P-3 Q-1 R-4 S-2

2 P-2 Q-3 R-4 S-1

3 P-2 Q-1 R-3 S-4

4 P-4 Q-2 R-1 S-3

Correct Answer -

P-3 Q-1 R-4 S-2

99)

100)

1 P-3 Q-2 R-4 S-1

2 P-1 Q-2 R-3 S-4

3 P-2 Q-3 R-1 S-4

4 P-4 Q-3 R-2 S-1

Correct Answer -

P-2 Q-3 R-1 S-4

ldquoHuman geography is the study of changing relationship between the unresting man and the unstableearthrdquo was defined by

lsquolsquoमानव भगोल ाकल आदमी और अथर पी क बीच सबध परवतन का अयन हrdquo ______ ारा परभािषत िकया गयाथा

1 J Brunches ज चस

2 EC Semple ईसी सल

3 HJ Mackinder एच ज मिकदर

4 PV Blache पीवी च

Correct Answer -

EC Semple ईसी सल

Sedimentary rocks are finally and ultimately derived from the____________

अवसादी चान अततः ________ स ा की जाती ह

1 action of earth movements पी की गितिविधयो

2 marine deposit समी िनप

3 weathering of metamorphic rocks पातरत चानो क अपय

4 weathering of igneous rocks आय चानो क अपय

Correct Answer -

weathering of igneous rocks आय चानो क अपय

Page 37: High School Teacher Eligibility Test- BOARD PROFESSIONAL ...peb.mp.gov.in/results/RESULT_18/HST_RES18/Final_anwser_key/HST… · M a ndl a / मंड ल ... Under the Madhya Pradesh

61)

62)

63)

64)

What is the proportion of lsquoJuvenile Populationrsquo (0-14 years) in India as per 2011Census

2011 की जनगणना क अनसार भारत म जवनाइल पॉपलशन यानी िकशोर जनस या (0-14 वष) का अनपात ाह

1 3076 of total population कल जनस या का 3076

2 2764 of total population कल जनस या का 2764

3 2933 of total population कल जनस या का 2933

4 3354 of total population कल जनस या का 3354

Correct Answer -

3076 of total population कल जनस या का 3076

What is the Belfast famous for बलफा िकसक िलए मशर ह

1 Belt of cotton textile industry कपास व उोग क

2 Ship-building industry जहाज िनमाण उोग

3 Agricultural machinery किष उपकरण

4 Aero planes manufacturing वाययान िनमाण

Correct Answer -

Ship-building industry जहाज िनमाण उोग

What is the most important occupation in tropical monsoon lands

उकिटबधीय मॉनसन भिम म सबस महपण वसाय ा ह

1 Mining खनन

2 Cattle rearing मवशी पालन

3 Agriculture किष

4 Nomadic herding नोमािडक जड़ी-बिटया

Correct Answer -

Agriculture किष

What is the most important characteristics of the islands (Indian) located in the Arabian sea

अरब सागर म थत ीपो (भारतीय) की सबस महपण िवशषता ा ह

ी ो

65)

66)

67)

1 There are all of coral origins सभी कोरल मल क ह

2 There are all very small in size य सभी आकार म बत छोट ह

3 They have a very dry climate इनकी जलवाय बत श ह

4 They are extended parts of the mainland व महाीप क िवारत िह ह

Correct Answer -

There are all of coral origins सभी कोरल मल क ह

What do the basalt layers of the Deccan indicate डन की बसा परत ा इिगत करती ह

1 All of the above उपरो सभी

2 Huge volcanic eruptions in the distant past दरथ अतीत म िवशाल ालामखीय िवोट

3 The immense erosional activity of the rivers निदयो की िवशाल रण गितिविध

4 The influence of weathering मौसम का भाव

Correct Answer -

Huge volcanic eruptions in the distant past दरथ अतीत म िवशाल ालामखीय िवोट

In the structure of planet Earth below the mantle the core is mainly made up of_____

पी ह की सरचना म मटल क नीच कोर म प स______ स िनिमत होती ह

1 aluminium एमीिनयम

2 silicon िसिलकॉन

3 chromium ोिमयम

4 iron लोहा

Correct Answer -

iron लोहा

One of the major Mid Oceanic Ridge is found in मख म-महासागर चोिटयो म स एक ______ म पायाजाता ह

1 Mid Pacific Ocean म शात महासागर

2 Mid Atlantic Ocean म अटलािटक महासागर

3 Mid Indian Ocean म भारतीय महासागर

4 Mid Arctic Ocean म आक िटक महासागर

68)

69)

70)

71)

Correct Answer -

Mid Atlantic Ocean म अटलािटक महासागर

Magma that reaches the Earthrsquos surface and then solidifies is called________

मा जो पी की सतह तक पचती ह और िफर ठोस हो जाती ह ________कहलाती ह

1 quartz ाटज

2 lava लावा

3 granite नाइट

4 silicates िसिलकट

Correct Answer -

lava लावा

Isotherms are the lines of equal_______ समताप रखाए समान _______की रखाए होती ह

1 pressure दाब

2 temperature तापमान

3 rainfall वषा

4 height ऊचाई

Correct Answer -

temperature तापमान

Mark the correct sequence of passes in the Western Ghats from north to south

पिमी घाटो म उर स दिण तक दर क सही अनम को िचित कर

1 Thalghat Palghat Bhorghat थलगघाट पालघाट भोरघाट

2 Thalghat Bhorghat Palghat थलघाट भोरघाट पालघाट

3 Bhorghat Thalghat Palghat भोरघाट थलघाट पालघाट

4 Palghat Bhorghat Thalghat पालघाट भोरघाट थलघाट

Correct Answer -

Thalghat Bhorghat Palghat थलघाट भोरघाट पालघाट

Which of the following does not have influence over the climate in India

ि ि ि ी ी

72)

73)

िनिलखत म स िकसका भाव भारत की जलवाय पर नही पड़ता ह

1 Ocean currents सागर की लहर

2 Nearness to equator भम रखा स िनकटता

3 Monsoons मानसन

4 Presence of Indian ocean भारतीय महासागर की उपथित

Correct Answer -

Ocean currents सागर की लहर

Which of the following cloud types has the characteristics like vertical tall narrow and puffy

िनिलखत म स िकस कार क मघो म लबवत लबी सकीण और थलता जसी िवशषताए ह

1 Cumulonimbus तफानी मघ

2 Cumulus मघ पज

3 Cirrocumulus पाभ कपासी मघ

4 Nimbostratus वषारी मघ

Correct Answer -

Cumulus मघ पज

Which of the following statement is INCORRECT about Crude Birth Rate

िनिलखत स कौन सा कथन अशोिधत ज दर क बार म सही नही ह

1 It cannot be used for comparing fertility level between two countries with different population characteristics

इसका उपयोग िविभ जनसा िवशषताओ वाल दो दशो क बीच जनन र की तलना क िलए नही िकया जा सकता ह

2 It is a standardized measure of fertility

यह जनन मता का मानकीकत उपाय ह

3 It is effected by the age-sex composition of the population

यह आबादी की आय-िलग सरचना स भािवत होता ह

4 It is expressed per 1000 population in a given geographical unit

यह िकसी दी गई भौगोिलक इकाई म ित 1000 जनसा पर िकया जाता ह

Correct Answer -

It is a standardized measure of fertility

यह जनन मता का मानकीकत उपाय ह

74)

75)

76)

77)

Which of the following state in India experienced negative decadal growth rate during 2001 to 2011census

भारत म िनिलखत म स िकस रा म वष 2001 स 2011 की जनगणना क दौरान नकाराक िगरावट दर ई

1 Tripura िपरा

2 Nagaland नागालड

3 Haryana हरयाणा

4 Odisha ओिडसा

Correct Answer -

Nagaland नागालड

Which of the following is NOT a characteristic of peninsular rivers

िनिलखत म स कौन सी िवशषता ायीपीय निदयो म नही होती ह

1 Flow through shallow valleys उथल घािटयो क माम स वाह

2 Seasonal flow मौसमी वाह

3 Little erosional activity थोड़ी कटावदार गितिविध

4 Meandering tendency often shifting their beds घमावदार वि अर अपन तटो को थानातरत करना

Correct Answer -

Meandering tendency often shifting their beds घमावदार वि अर अपन तटो को थानातरत करना

Which of the following gases in the atmosphere absorbs heat from the Sunrsquos radiation and the Earthssurface

वायमडल म िनिलखत म स कौन सी गस सय क िविकरण और पी की सतह स ऊा को अवशोिषत करती ह

1 Neon िनयॉन

2 Carbon dioxide काबन डाइऑाइड

3 Argon आगन

4 Nitrogen नाइट ोजन

Correct Answer -

Carbon dioxide काबन डाइऑाइड

Which of the following kind of settlement pattern is found at the confluence of rivers

ि ि ि ि ो

78)

79)

80)

िनिलखत म स िकस कार का वथापन पटन निदयो क सगम पर पाया जाता ह

1 Triangular Paern िकोणीय पटन

2 Circular or Semi-Circular Paern परप या अध-परप पटन

3 Nebular Paern नबलर पटन

4 Star ndashShaped Paern ार-आकार का पटन

Correct Answer -

Triangular Paern िकोणीय पटन

Which one was not the objective of the Biosphere Reserve Projects launched by the UNESCO

यनो ारा श की गई सरित जवमडल परयोजनाओ का उ इनम स कौन सा नही था

1 To promote teaching and research िशण और अनसधान को बढ़ावा दना

2 To make agriculture sustainable किष को दीघकािलक बनाना

3 To conserve ecosystems पारथितक त को सरित करना

4 To conserve genetic diversity for a longtime लब समय तक अनवािशक िविवधता को सरित करना

Correct Answer -

To make agriculture sustainable किष को दीघकािलक बनाना

Which region of the Earth surface receives the highest amount of insulation

पी सतह का कौन सा तापावरोधन की उतम माा ा करता ह

1 Land mass थलखड

2 Savannah region सवाना

3 Water bodies जल िनकाय

4 Tropical desert उकिटबधीय रिगान

Correct Answer -

Tropical desert उकिटबधीय रिगान

Which one of the following is not a biodiversity hotspot

िनिलखत म स कौन सा जव िविवधता का म जगह नही ह

1 Eastern Himalaya पव िहमालय

2 Eastern Ghats पव घाट

81)

82)

83)

3 Indo-Myanmar भारत-ामार

4 Westerm Ghats पिमी घाट

Correct Answer -

Eastern Ghats पव घाट

Which one of the following is NOT a part of the World Network of Biosphere Reserves based on theUNESCO Man and Biosphere Programme

यनो मन और बायोीयर कायम क आधार पर िनिलखत म स कौन बायोीयर रजव क िव नटवक कािहा नही ह

1 Gulf of Mannar मार की खाड़ी

2 Seshachalam शषाचलम

3 Sunderban सदरबन

4 Nilgiri नीलिगर

Correct Answer -

Seshachalam शषाचलम

Which one of the following is an example of ldquodesert vegetationrdquo

िनिलखत म स कौन मथलीय वनित का एक उदाहरण ह

1 Mosses and lichens दलदल और शवाल

2 Temperate grassland समशीतो घास क मदान

3 Coniferous forest शकधारी वन

4 Acacia and cactus एकािसया और कस

Correct Answer -

Acacia and cactus एकािसया और कस

Which one of the following reflects more sunlight िनिलखत म स कौन सा सय की रोशनी को अिधकपरावितत करता ह

1 Paddy crop land धान फसल भिम

2 Land covered with fresh snow ताजा बफ स आािदत भिम

3 Sand desert रतीली रिगान

4 Prairie land यरी भिम

84)

85)

86)

87)

Correct Answer -

Land covered with fresh snow ताजा बफ स आािदत भिम

Which layer of the atmosphere is in contact with the surface of the earthrsquos oceans

वायमडल की कौन सी परत पी क महासागरो की सतह क सपक म ह

1 Stratosphere समताप मडल

2 Mesosphere म मडल

3 Hydrosphere जलमडल

4 Troposphere ोभ मडल

Correct Answer -

Troposphere ोभ मडल

Mediterranean Sea is a border of which of the following countries भम सागर िनिलखत दशो म सिकसकी सीमा ह

1 None of these इनम स कोई नही

2 Iraq इराक

3 Lebanon लबनान

4 Jordan जॉडन

Correct Answer -

Lebanon लबनान

Benguela ocean currents are found along which coast बगएला महासागर धाराए िकस तट क साथ पायीजाती ह

1 East Coast of South America दिण अमरका क पव तट

2 East Coast of Africa अीका क पव तट

3 West Coast of South America दिण अमरका क पिमी तट

4 West Coast of Africa अीका क पिमी तट

Correct Answer -

West Coast of Africa अीका क पिमी तट

88)

89)

90)

Due to tension a block of land on one side being pushed up or upthrown relative to the downthrown blockis referred as

तनाव क कारण नीच फ क ए खड क साप भिम का एक खड एक ओर स ऊपर धकला जाता ह या ऊपर की ओरफ का जाता ह यह _____ क प म सदिभत ह

1 Thrust fault प श

2 Normal fault सामा श

3 Reverse fault म श

4 Strike slip fault नितलब सपण श

Correct Answer -

Normal fault सामा श

Inter-tropical doldrums is a zone of ______ अतर-उकिटबधीय डोलड ______ का एक ह

1 Frontolysis टोलायिसस

2 Convergence अिभसरण

3 Inter-tropical divergence zone अतर-उकिटबधीय िवचलन

4 Local wind थानीय वाय

Correct Answer -

Convergence अिभसरण

The Horse Latitudes are regions located at about _____ north and south of the equator

हॉस अाश भम रखा क उर और दिण म लगभग _____ पर थत ह

1 30ndash60 degree Latitude 30-60 िडी अाश

2 0ndash5 degree Latitude 0-5 िडी अाश

3 30 degree Latitude 30 िडी अाश

4 60ndash90 degree Latitude 60-90 िडी अाश

Correct Answer -

30 degree Latitude 30 िडी अाश

Generally evaporation is high over which part of the Earth

आम तौर पर पी क िकस भाग पर वाीकरण अिधक होता ह

1 Equatorial maritime भमवत समीय ी ी

91)

92)

2 Equatorial continental भमवत महाीपीय

3 Polar maritime वीय समीय

4 Polar continental वीय महाीपीय

Correct Answer -

Equatorial maritime भमवत समीय

A very high temperature during summer in north western India leads to what type of climaticcondition in south

उर पिमी भारत म गम क दौरान बत अिधक तापमान होन क कारण दिण म िकस कार की जलवाय थितउ करता ह

1 Depression over arabian sea अरब सागर पर अवनमन

2 Failure monsoon मानसन िवफलता

3 Successful monsoon मानसन सफलता

4 Cyclones चवात

Correct Answer -

Successful monsoon मानसन सफलता

Lightning and thunder are the resultant effect when तिड़त और गजन परणामी भाव ह जब

1 Two massive clouds hit powerfully each other first lightning is produced and later sound is produced

दो बड़ बादल एक दसर स शशाली ढग स टकरात ह पहल आकाशीय िवदयत उ होता ह और बाद म िन उहोती ह

2 Two massive clouds come into contact with the powerful wind collision this results into first sound and thenlightning

दो बड़ बादल शशाली पवन सघ क सपक म आत ह इसका परणामप पहल िन और िफर आकाशीय िवदयतउ होता ह

3 None of the above उपरो म स कोई भी नही

4 A high density cloud contains positively and negatively charged electric ions and when this interacts light andsound are simultaneously produced

एक उ घन बादल म धनाक और ऋणाक आविशत िवदयत आयन होत ह और जब यह परर भाव डालत ह तोकाश और िन एक साथ उािदत होती ह

Correct Answer -

A high density cloud contains positively and negatively charged electric ions and when this interacts light andsound are simultaneously produced

औ ि ि ो औ ो

93)

94)

95)

एक उ घन बादल म धनाक और ऋणाक आविशत िवदयत आयन होत ह और जब यह परर भाव डालत ह तोकाश और िन एक साथ उािदत होती ह

Doon Valley is able to grow rice because दन घाटी चावल उगान म सम ह ोिक

1 Other crops cannot be grown वहा अ फसलो को उगाया नही जा सकता ह

2 People in the valley are rice eaters घाटी म लोग चावल खान वाल ह

3 There is a huge export demand of rice वहा चावल की भारी िनयात माग ह

4 It has warm summer and snow melt waters for irrigation

वहा गिमया गम होती ह िसचाई क िलए बफ का िपघला आ पानी होता ह

Correct Answer -

It has warm summer and snow melt waters for irrigation

वहा गिमया गम होती ह िसचाई क िलए बफ का िपघला आ पानी होता ह

CANCELLED

In the geological time scale the Mesozoic Era DOES NOT contains which of the following periods

भगभय समय पमान पर मजीवी यग म िन कालो म स कौन नही ह

1 Triassic ट ाइऐिसक

2 Jurassic जरिसक

3 Cretaceous चाकमय

4 Carboniferous काबनी

Correct Answer -

Carboniferous काबनी

96)

1 P-3 Q-4 R-2 S-1

2 P-3 Q-4 R-1 S-2

3 P-3 Q-4 R-1 S-2

4 P-4 Q-3 R-2 S-1

Correct Answer -

P-4 Q-3 R-2 S-1

1 P-3 Q-1 R-4 S-2

2 P-3 Q-4 R-1 S-2

3 P-3 Q-2 R-4 S-1

97)

98)

4 P-2 Q-1 R-4 S-3

Correct Answer -

P-3 Q-4 R-1 S-2

1 P-3 Q-1 R-4 S-2

2 P-2 Q-3 R-4 S-1

3 P-2 Q-1 R-3 S-4

4 P-4 Q-2 R-1 S-3

Correct Answer -

P-3 Q-1 R-4 S-2

99)

100)

1 P-3 Q-2 R-4 S-1

2 P-1 Q-2 R-3 S-4

3 P-2 Q-3 R-1 S-4

4 P-4 Q-3 R-2 S-1

Correct Answer -

P-2 Q-3 R-1 S-4

ldquoHuman geography is the study of changing relationship between the unresting man and the unstableearthrdquo was defined by

lsquolsquoमानव भगोल ाकल आदमी और अथर पी क बीच सबध परवतन का अयन हrdquo ______ ारा परभािषत िकया गयाथा

1 J Brunches ज चस

2 EC Semple ईसी सल

3 HJ Mackinder एच ज मिकदर

4 PV Blache पीवी च

Correct Answer -

EC Semple ईसी सल

Sedimentary rocks are finally and ultimately derived from the____________

अवसादी चान अततः ________ स ा की जाती ह

1 action of earth movements पी की गितिविधयो

2 marine deposit समी िनप

3 weathering of metamorphic rocks पातरत चानो क अपय

4 weathering of igneous rocks आय चानो क अपय

Correct Answer -

weathering of igneous rocks आय चानो क अपय

Page 38: High School Teacher Eligibility Test- BOARD PROFESSIONAL ...peb.mp.gov.in/results/RESULT_18/HST_RES18/Final_anwser_key/HST… · M a ndl a / मंड ल ... Under the Madhya Pradesh

65)

66)

67)

1 There are all of coral origins सभी कोरल मल क ह

2 There are all very small in size य सभी आकार म बत छोट ह

3 They have a very dry climate इनकी जलवाय बत श ह

4 They are extended parts of the mainland व महाीप क िवारत िह ह

Correct Answer -

There are all of coral origins सभी कोरल मल क ह

What do the basalt layers of the Deccan indicate डन की बसा परत ा इिगत करती ह

1 All of the above उपरो सभी

2 Huge volcanic eruptions in the distant past दरथ अतीत म िवशाल ालामखीय िवोट

3 The immense erosional activity of the rivers निदयो की िवशाल रण गितिविध

4 The influence of weathering मौसम का भाव

Correct Answer -

Huge volcanic eruptions in the distant past दरथ अतीत म िवशाल ालामखीय िवोट

In the structure of planet Earth below the mantle the core is mainly made up of_____

पी ह की सरचना म मटल क नीच कोर म प स______ स िनिमत होती ह

1 aluminium एमीिनयम

2 silicon िसिलकॉन

3 chromium ोिमयम

4 iron लोहा

Correct Answer -

iron लोहा

One of the major Mid Oceanic Ridge is found in मख म-महासागर चोिटयो म स एक ______ म पायाजाता ह

1 Mid Pacific Ocean म शात महासागर

2 Mid Atlantic Ocean म अटलािटक महासागर

3 Mid Indian Ocean म भारतीय महासागर

4 Mid Arctic Ocean म आक िटक महासागर

68)

69)

70)

71)

Correct Answer -

Mid Atlantic Ocean म अटलािटक महासागर

Magma that reaches the Earthrsquos surface and then solidifies is called________

मा जो पी की सतह तक पचती ह और िफर ठोस हो जाती ह ________कहलाती ह

1 quartz ाटज

2 lava लावा

3 granite नाइट

4 silicates िसिलकट

Correct Answer -

lava लावा

Isotherms are the lines of equal_______ समताप रखाए समान _______की रखाए होती ह

1 pressure दाब

2 temperature तापमान

3 rainfall वषा

4 height ऊचाई

Correct Answer -

temperature तापमान

Mark the correct sequence of passes in the Western Ghats from north to south

पिमी घाटो म उर स दिण तक दर क सही अनम को िचित कर

1 Thalghat Palghat Bhorghat थलगघाट पालघाट भोरघाट

2 Thalghat Bhorghat Palghat थलघाट भोरघाट पालघाट

3 Bhorghat Thalghat Palghat भोरघाट थलघाट पालघाट

4 Palghat Bhorghat Thalghat पालघाट भोरघाट थलघाट

Correct Answer -

Thalghat Bhorghat Palghat थलघाट भोरघाट पालघाट

Which of the following does not have influence over the climate in India

ि ि ि ी ी

72)

73)

िनिलखत म स िकसका भाव भारत की जलवाय पर नही पड़ता ह

1 Ocean currents सागर की लहर

2 Nearness to equator भम रखा स िनकटता

3 Monsoons मानसन

4 Presence of Indian ocean भारतीय महासागर की उपथित

Correct Answer -

Ocean currents सागर की लहर

Which of the following cloud types has the characteristics like vertical tall narrow and puffy

िनिलखत म स िकस कार क मघो म लबवत लबी सकीण और थलता जसी िवशषताए ह

1 Cumulonimbus तफानी मघ

2 Cumulus मघ पज

3 Cirrocumulus पाभ कपासी मघ

4 Nimbostratus वषारी मघ

Correct Answer -

Cumulus मघ पज

Which of the following statement is INCORRECT about Crude Birth Rate

िनिलखत स कौन सा कथन अशोिधत ज दर क बार म सही नही ह

1 It cannot be used for comparing fertility level between two countries with different population characteristics

इसका उपयोग िविभ जनसा िवशषताओ वाल दो दशो क बीच जनन र की तलना क िलए नही िकया जा सकता ह

2 It is a standardized measure of fertility

यह जनन मता का मानकीकत उपाय ह

3 It is effected by the age-sex composition of the population

यह आबादी की आय-िलग सरचना स भािवत होता ह

4 It is expressed per 1000 population in a given geographical unit

यह िकसी दी गई भौगोिलक इकाई म ित 1000 जनसा पर िकया जाता ह

Correct Answer -

It is a standardized measure of fertility

यह जनन मता का मानकीकत उपाय ह

74)

75)

76)

77)

Which of the following state in India experienced negative decadal growth rate during 2001 to 2011census

भारत म िनिलखत म स िकस रा म वष 2001 स 2011 की जनगणना क दौरान नकाराक िगरावट दर ई

1 Tripura िपरा

2 Nagaland नागालड

3 Haryana हरयाणा

4 Odisha ओिडसा

Correct Answer -

Nagaland नागालड

Which of the following is NOT a characteristic of peninsular rivers

िनिलखत म स कौन सी िवशषता ायीपीय निदयो म नही होती ह

1 Flow through shallow valleys उथल घािटयो क माम स वाह

2 Seasonal flow मौसमी वाह

3 Little erosional activity थोड़ी कटावदार गितिविध

4 Meandering tendency often shifting their beds घमावदार वि अर अपन तटो को थानातरत करना

Correct Answer -

Meandering tendency often shifting their beds घमावदार वि अर अपन तटो को थानातरत करना

Which of the following gases in the atmosphere absorbs heat from the Sunrsquos radiation and the Earthssurface

वायमडल म िनिलखत म स कौन सी गस सय क िविकरण और पी की सतह स ऊा को अवशोिषत करती ह

1 Neon िनयॉन

2 Carbon dioxide काबन डाइऑाइड

3 Argon आगन

4 Nitrogen नाइट ोजन

Correct Answer -

Carbon dioxide काबन डाइऑाइड

Which of the following kind of settlement pattern is found at the confluence of rivers

ि ि ि ि ो

78)

79)

80)

िनिलखत म स िकस कार का वथापन पटन निदयो क सगम पर पाया जाता ह

1 Triangular Paern िकोणीय पटन

2 Circular or Semi-Circular Paern परप या अध-परप पटन

3 Nebular Paern नबलर पटन

4 Star ndashShaped Paern ार-आकार का पटन

Correct Answer -

Triangular Paern िकोणीय पटन

Which one was not the objective of the Biosphere Reserve Projects launched by the UNESCO

यनो ारा श की गई सरित जवमडल परयोजनाओ का उ इनम स कौन सा नही था

1 To promote teaching and research िशण और अनसधान को बढ़ावा दना

2 To make agriculture sustainable किष को दीघकािलक बनाना

3 To conserve ecosystems पारथितक त को सरित करना

4 To conserve genetic diversity for a longtime लब समय तक अनवािशक िविवधता को सरित करना

Correct Answer -

To make agriculture sustainable किष को दीघकािलक बनाना

Which region of the Earth surface receives the highest amount of insulation

पी सतह का कौन सा तापावरोधन की उतम माा ा करता ह

1 Land mass थलखड

2 Savannah region सवाना

3 Water bodies जल िनकाय

4 Tropical desert उकिटबधीय रिगान

Correct Answer -

Tropical desert उकिटबधीय रिगान

Which one of the following is not a biodiversity hotspot

िनिलखत म स कौन सा जव िविवधता का म जगह नही ह

1 Eastern Himalaya पव िहमालय

2 Eastern Ghats पव घाट

81)

82)

83)

3 Indo-Myanmar भारत-ामार

4 Westerm Ghats पिमी घाट

Correct Answer -

Eastern Ghats पव घाट

Which one of the following is NOT a part of the World Network of Biosphere Reserves based on theUNESCO Man and Biosphere Programme

यनो मन और बायोीयर कायम क आधार पर िनिलखत म स कौन बायोीयर रजव क िव नटवक कािहा नही ह

1 Gulf of Mannar मार की खाड़ी

2 Seshachalam शषाचलम

3 Sunderban सदरबन

4 Nilgiri नीलिगर

Correct Answer -

Seshachalam शषाचलम

Which one of the following is an example of ldquodesert vegetationrdquo

िनिलखत म स कौन मथलीय वनित का एक उदाहरण ह

1 Mosses and lichens दलदल और शवाल

2 Temperate grassland समशीतो घास क मदान

3 Coniferous forest शकधारी वन

4 Acacia and cactus एकािसया और कस

Correct Answer -

Acacia and cactus एकािसया और कस

Which one of the following reflects more sunlight िनिलखत म स कौन सा सय की रोशनी को अिधकपरावितत करता ह

1 Paddy crop land धान फसल भिम

2 Land covered with fresh snow ताजा बफ स आािदत भिम

3 Sand desert रतीली रिगान

4 Prairie land यरी भिम

84)

85)

86)

87)

Correct Answer -

Land covered with fresh snow ताजा बफ स आािदत भिम

Which layer of the atmosphere is in contact with the surface of the earthrsquos oceans

वायमडल की कौन सी परत पी क महासागरो की सतह क सपक म ह

1 Stratosphere समताप मडल

2 Mesosphere म मडल

3 Hydrosphere जलमडल

4 Troposphere ोभ मडल

Correct Answer -

Troposphere ोभ मडल

Mediterranean Sea is a border of which of the following countries भम सागर िनिलखत दशो म सिकसकी सीमा ह

1 None of these इनम स कोई नही

2 Iraq इराक

3 Lebanon लबनान

4 Jordan जॉडन

Correct Answer -

Lebanon लबनान

Benguela ocean currents are found along which coast बगएला महासागर धाराए िकस तट क साथ पायीजाती ह

1 East Coast of South America दिण अमरका क पव तट

2 East Coast of Africa अीका क पव तट

3 West Coast of South America दिण अमरका क पिमी तट

4 West Coast of Africa अीका क पिमी तट

Correct Answer -

West Coast of Africa अीका क पिमी तट

88)

89)

90)

Due to tension a block of land on one side being pushed up or upthrown relative to the downthrown blockis referred as

तनाव क कारण नीच फ क ए खड क साप भिम का एक खड एक ओर स ऊपर धकला जाता ह या ऊपर की ओरफ का जाता ह यह _____ क प म सदिभत ह

1 Thrust fault प श

2 Normal fault सामा श

3 Reverse fault म श

4 Strike slip fault नितलब सपण श

Correct Answer -

Normal fault सामा श

Inter-tropical doldrums is a zone of ______ अतर-उकिटबधीय डोलड ______ का एक ह

1 Frontolysis टोलायिसस

2 Convergence अिभसरण

3 Inter-tropical divergence zone अतर-उकिटबधीय िवचलन

4 Local wind थानीय वाय

Correct Answer -

Convergence अिभसरण

The Horse Latitudes are regions located at about _____ north and south of the equator

हॉस अाश भम रखा क उर और दिण म लगभग _____ पर थत ह

1 30ndash60 degree Latitude 30-60 िडी अाश

2 0ndash5 degree Latitude 0-5 िडी अाश

3 30 degree Latitude 30 िडी अाश

4 60ndash90 degree Latitude 60-90 िडी अाश

Correct Answer -

30 degree Latitude 30 िडी अाश

Generally evaporation is high over which part of the Earth

आम तौर पर पी क िकस भाग पर वाीकरण अिधक होता ह

1 Equatorial maritime भमवत समीय ी ी

91)

92)

2 Equatorial continental भमवत महाीपीय

3 Polar maritime वीय समीय

4 Polar continental वीय महाीपीय

Correct Answer -

Equatorial maritime भमवत समीय

A very high temperature during summer in north western India leads to what type of climaticcondition in south

उर पिमी भारत म गम क दौरान बत अिधक तापमान होन क कारण दिण म िकस कार की जलवाय थितउ करता ह

1 Depression over arabian sea अरब सागर पर अवनमन

2 Failure monsoon मानसन िवफलता

3 Successful monsoon मानसन सफलता

4 Cyclones चवात

Correct Answer -

Successful monsoon मानसन सफलता

Lightning and thunder are the resultant effect when तिड़त और गजन परणामी भाव ह जब

1 Two massive clouds hit powerfully each other first lightning is produced and later sound is produced

दो बड़ बादल एक दसर स शशाली ढग स टकरात ह पहल आकाशीय िवदयत उ होता ह और बाद म िन उहोती ह

2 Two massive clouds come into contact with the powerful wind collision this results into first sound and thenlightning

दो बड़ बादल शशाली पवन सघ क सपक म आत ह इसका परणामप पहल िन और िफर आकाशीय िवदयतउ होता ह

3 None of the above उपरो म स कोई भी नही

4 A high density cloud contains positively and negatively charged electric ions and when this interacts light andsound are simultaneously produced

एक उ घन बादल म धनाक और ऋणाक आविशत िवदयत आयन होत ह और जब यह परर भाव डालत ह तोकाश और िन एक साथ उािदत होती ह

Correct Answer -

A high density cloud contains positively and negatively charged electric ions and when this interacts light andsound are simultaneously produced

औ ि ि ो औ ो

93)

94)

95)

एक उ घन बादल म धनाक और ऋणाक आविशत िवदयत आयन होत ह और जब यह परर भाव डालत ह तोकाश और िन एक साथ उािदत होती ह

Doon Valley is able to grow rice because दन घाटी चावल उगान म सम ह ोिक

1 Other crops cannot be grown वहा अ फसलो को उगाया नही जा सकता ह

2 People in the valley are rice eaters घाटी म लोग चावल खान वाल ह

3 There is a huge export demand of rice वहा चावल की भारी िनयात माग ह

4 It has warm summer and snow melt waters for irrigation

वहा गिमया गम होती ह िसचाई क िलए बफ का िपघला आ पानी होता ह

Correct Answer -

It has warm summer and snow melt waters for irrigation

वहा गिमया गम होती ह िसचाई क िलए बफ का िपघला आ पानी होता ह

CANCELLED

In the geological time scale the Mesozoic Era DOES NOT contains which of the following periods

भगभय समय पमान पर मजीवी यग म िन कालो म स कौन नही ह

1 Triassic ट ाइऐिसक

2 Jurassic जरिसक

3 Cretaceous चाकमय

4 Carboniferous काबनी

Correct Answer -

Carboniferous काबनी

96)

1 P-3 Q-4 R-2 S-1

2 P-3 Q-4 R-1 S-2

3 P-3 Q-4 R-1 S-2

4 P-4 Q-3 R-2 S-1

Correct Answer -

P-4 Q-3 R-2 S-1

1 P-3 Q-1 R-4 S-2

2 P-3 Q-4 R-1 S-2

3 P-3 Q-2 R-4 S-1

97)

98)

4 P-2 Q-1 R-4 S-3

Correct Answer -

P-3 Q-4 R-1 S-2

1 P-3 Q-1 R-4 S-2

2 P-2 Q-3 R-4 S-1

3 P-2 Q-1 R-3 S-4

4 P-4 Q-2 R-1 S-3

Correct Answer -

P-3 Q-1 R-4 S-2

99)

100)

1 P-3 Q-2 R-4 S-1

2 P-1 Q-2 R-3 S-4

3 P-2 Q-3 R-1 S-4

4 P-4 Q-3 R-2 S-1

Correct Answer -

P-2 Q-3 R-1 S-4

ldquoHuman geography is the study of changing relationship between the unresting man and the unstableearthrdquo was defined by

lsquolsquoमानव भगोल ाकल आदमी और अथर पी क बीच सबध परवतन का अयन हrdquo ______ ारा परभािषत िकया गयाथा

1 J Brunches ज चस

2 EC Semple ईसी सल

3 HJ Mackinder एच ज मिकदर

4 PV Blache पीवी च

Correct Answer -

EC Semple ईसी सल

Sedimentary rocks are finally and ultimately derived from the____________

अवसादी चान अततः ________ स ा की जाती ह

1 action of earth movements पी की गितिविधयो

2 marine deposit समी िनप

3 weathering of metamorphic rocks पातरत चानो क अपय

4 weathering of igneous rocks आय चानो क अपय

Correct Answer -

weathering of igneous rocks आय चानो क अपय

Page 39: High School Teacher Eligibility Test- BOARD PROFESSIONAL ...peb.mp.gov.in/results/RESULT_18/HST_RES18/Final_anwser_key/HST… · M a ndl a / मंड ल ... Under the Madhya Pradesh

68)

69)

70)

71)

Correct Answer -

Mid Atlantic Ocean म अटलािटक महासागर

Magma that reaches the Earthrsquos surface and then solidifies is called________

मा जो पी की सतह तक पचती ह और िफर ठोस हो जाती ह ________कहलाती ह

1 quartz ाटज

2 lava लावा

3 granite नाइट

4 silicates िसिलकट

Correct Answer -

lava लावा

Isotherms are the lines of equal_______ समताप रखाए समान _______की रखाए होती ह

1 pressure दाब

2 temperature तापमान

3 rainfall वषा

4 height ऊचाई

Correct Answer -

temperature तापमान

Mark the correct sequence of passes in the Western Ghats from north to south

पिमी घाटो म उर स दिण तक दर क सही अनम को िचित कर

1 Thalghat Palghat Bhorghat थलगघाट पालघाट भोरघाट

2 Thalghat Bhorghat Palghat थलघाट भोरघाट पालघाट

3 Bhorghat Thalghat Palghat भोरघाट थलघाट पालघाट

4 Palghat Bhorghat Thalghat पालघाट भोरघाट थलघाट

Correct Answer -

Thalghat Bhorghat Palghat थलघाट भोरघाट पालघाट

Which of the following does not have influence over the climate in India

ि ि ि ी ी

72)

73)

िनिलखत म स िकसका भाव भारत की जलवाय पर नही पड़ता ह

1 Ocean currents सागर की लहर

2 Nearness to equator भम रखा स िनकटता

3 Monsoons मानसन

4 Presence of Indian ocean भारतीय महासागर की उपथित

Correct Answer -

Ocean currents सागर की लहर

Which of the following cloud types has the characteristics like vertical tall narrow and puffy

िनिलखत म स िकस कार क मघो म लबवत लबी सकीण और थलता जसी िवशषताए ह

1 Cumulonimbus तफानी मघ

2 Cumulus मघ पज

3 Cirrocumulus पाभ कपासी मघ

4 Nimbostratus वषारी मघ

Correct Answer -

Cumulus मघ पज

Which of the following statement is INCORRECT about Crude Birth Rate

िनिलखत स कौन सा कथन अशोिधत ज दर क बार म सही नही ह

1 It cannot be used for comparing fertility level between two countries with different population characteristics

इसका उपयोग िविभ जनसा िवशषताओ वाल दो दशो क बीच जनन र की तलना क िलए नही िकया जा सकता ह

2 It is a standardized measure of fertility

यह जनन मता का मानकीकत उपाय ह

3 It is effected by the age-sex composition of the population

यह आबादी की आय-िलग सरचना स भािवत होता ह

4 It is expressed per 1000 population in a given geographical unit

यह िकसी दी गई भौगोिलक इकाई म ित 1000 जनसा पर िकया जाता ह

Correct Answer -

It is a standardized measure of fertility

यह जनन मता का मानकीकत उपाय ह

74)

75)

76)

77)

Which of the following state in India experienced negative decadal growth rate during 2001 to 2011census

भारत म िनिलखत म स िकस रा म वष 2001 स 2011 की जनगणना क दौरान नकाराक िगरावट दर ई

1 Tripura िपरा

2 Nagaland नागालड

3 Haryana हरयाणा

4 Odisha ओिडसा

Correct Answer -

Nagaland नागालड

Which of the following is NOT a characteristic of peninsular rivers

िनिलखत म स कौन सी िवशषता ायीपीय निदयो म नही होती ह

1 Flow through shallow valleys उथल घािटयो क माम स वाह

2 Seasonal flow मौसमी वाह

3 Little erosional activity थोड़ी कटावदार गितिविध

4 Meandering tendency often shifting their beds घमावदार वि अर अपन तटो को थानातरत करना

Correct Answer -

Meandering tendency often shifting their beds घमावदार वि अर अपन तटो को थानातरत करना

Which of the following gases in the atmosphere absorbs heat from the Sunrsquos radiation and the Earthssurface

वायमडल म िनिलखत म स कौन सी गस सय क िविकरण और पी की सतह स ऊा को अवशोिषत करती ह

1 Neon िनयॉन

2 Carbon dioxide काबन डाइऑाइड

3 Argon आगन

4 Nitrogen नाइट ोजन

Correct Answer -

Carbon dioxide काबन डाइऑाइड

Which of the following kind of settlement pattern is found at the confluence of rivers

ि ि ि ि ो

78)

79)

80)

िनिलखत म स िकस कार का वथापन पटन निदयो क सगम पर पाया जाता ह

1 Triangular Paern िकोणीय पटन

2 Circular or Semi-Circular Paern परप या अध-परप पटन

3 Nebular Paern नबलर पटन

4 Star ndashShaped Paern ार-आकार का पटन

Correct Answer -

Triangular Paern िकोणीय पटन

Which one was not the objective of the Biosphere Reserve Projects launched by the UNESCO

यनो ारा श की गई सरित जवमडल परयोजनाओ का उ इनम स कौन सा नही था

1 To promote teaching and research िशण और अनसधान को बढ़ावा दना

2 To make agriculture sustainable किष को दीघकािलक बनाना

3 To conserve ecosystems पारथितक त को सरित करना

4 To conserve genetic diversity for a longtime लब समय तक अनवािशक िविवधता को सरित करना

Correct Answer -

To make agriculture sustainable किष को दीघकािलक बनाना

Which region of the Earth surface receives the highest amount of insulation

पी सतह का कौन सा तापावरोधन की उतम माा ा करता ह

1 Land mass थलखड

2 Savannah region सवाना

3 Water bodies जल िनकाय

4 Tropical desert उकिटबधीय रिगान

Correct Answer -

Tropical desert उकिटबधीय रिगान

Which one of the following is not a biodiversity hotspot

िनिलखत म स कौन सा जव िविवधता का म जगह नही ह

1 Eastern Himalaya पव िहमालय

2 Eastern Ghats पव घाट

81)

82)

83)

3 Indo-Myanmar भारत-ामार

4 Westerm Ghats पिमी घाट

Correct Answer -

Eastern Ghats पव घाट

Which one of the following is NOT a part of the World Network of Biosphere Reserves based on theUNESCO Man and Biosphere Programme

यनो मन और बायोीयर कायम क आधार पर िनिलखत म स कौन बायोीयर रजव क िव नटवक कािहा नही ह

1 Gulf of Mannar मार की खाड़ी

2 Seshachalam शषाचलम

3 Sunderban सदरबन

4 Nilgiri नीलिगर

Correct Answer -

Seshachalam शषाचलम

Which one of the following is an example of ldquodesert vegetationrdquo

िनिलखत म स कौन मथलीय वनित का एक उदाहरण ह

1 Mosses and lichens दलदल और शवाल

2 Temperate grassland समशीतो घास क मदान

3 Coniferous forest शकधारी वन

4 Acacia and cactus एकािसया और कस

Correct Answer -

Acacia and cactus एकािसया और कस

Which one of the following reflects more sunlight िनिलखत म स कौन सा सय की रोशनी को अिधकपरावितत करता ह

1 Paddy crop land धान फसल भिम

2 Land covered with fresh snow ताजा बफ स आािदत भिम

3 Sand desert रतीली रिगान

4 Prairie land यरी भिम

84)

85)

86)

87)

Correct Answer -

Land covered with fresh snow ताजा बफ स आािदत भिम

Which layer of the atmosphere is in contact with the surface of the earthrsquos oceans

वायमडल की कौन सी परत पी क महासागरो की सतह क सपक म ह

1 Stratosphere समताप मडल

2 Mesosphere म मडल

3 Hydrosphere जलमडल

4 Troposphere ोभ मडल

Correct Answer -

Troposphere ोभ मडल

Mediterranean Sea is a border of which of the following countries भम सागर िनिलखत दशो म सिकसकी सीमा ह

1 None of these इनम स कोई नही

2 Iraq इराक

3 Lebanon लबनान

4 Jordan जॉडन

Correct Answer -

Lebanon लबनान

Benguela ocean currents are found along which coast बगएला महासागर धाराए िकस तट क साथ पायीजाती ह

1 East Coast of South America दिण अमरका क पव तट

2 East Coast of Africa अीका क पव तट

3 West Coast of South America दिण अमरका क पिमी तट

4 West Coast of Africa अीका क पिमी तट

Correct Answer -

West Coast of Africa अीका क पिमी तट

88)

89)

90)

Due to tension a block of land on one side being pushed up or upthrown relative to the downthrown blockis referred as

तनाव क कारण नीच फ क ए खड क साप भिम का एक खड एक ओर स ऊपर धकला जाता ह या ऊपर की ओरफ का जाता ह यह _____ क प म सदिभत ह

1 Thrust fault प श

2 Normal fault सामा श

3 Reverse fault म श

4 Strike slip fault नितलब सपण श

Correct Answer -

Normal fault सामा श

Inter-tropical doldrums is a zone of ______ अतर-उकिटबधीय डोलड ______ का एक ह

1 Frontolysis टोलायिसस

2 Convergence अिभसरण

3 Inter-tropical divergence zone अतर-उकिटबधीय िवचलन

4 Local wind थानीय वाय

Correct Answer -

Convergence अिभसरण

The Horse Latitudes are regions located at about _____ north and south of the equator

हॉस अाश भम रखा क उर और दिण म लगभग _____ पर थत ह

1 30ndash60 degree Latitude 30-60 िडी अाश

2 0ndash5 degree Latitude 0-5 िडी अाश

3 30 degree Latitude 30 िडी अाश

4 60ndash90 degree Latitude 60-90 िडी अाश

Correct Answer -

30 degree Latitude 30 िडी अाश

Generally evaporation is high over which part of the Earth

आम तौर पर पी क िकस भाग पर वाीकरण अिधक होता ह

1 Equatorial maritime भमवत समीय ी ी

91)

92)

2 Equatorial continental भमवत महाीपीय

3 Polar maritime वीय समीय

4 Polar continental वीय महाीपीय

Correct Answer -

Equatorial maritime भमवत समीय

A very high temperature during summer in north western India leads to what type of climaticcondition in south

उर पिमी भारत म गम क दौरान बत अिधक तापमान होन क कारण दिण म िकस कार की जलवाय थितउ करता ह

1 Depression over arabian sea अरब सागर पर अवनमन

2 Failure monsoon मानसन िवफलता

3 Successful monsoon मानसन सफलता

4 Cyclones चवात

Correct Answer -

Successful monsoon मानसन सफलता

Lightning and thunder are the resultant effect when तिड़त और गजन परणामी भाव ह जब

1 Two massive clouds hit powerfully each other first lightning is produced and later sound is produced

दो बड़ बादल एक दसर स शशाली ढग स टकरात ह पहल आकाशीय िवदयत उ होता ह और बाद म िन उहोती ह

2 Two massive clouds come into contact with the powerful wind collision this results into first sound and thenlightning

दो बड़ बादल शशाली पवन सघ क सपक म आत ह इसका परणामप पहल िन और िफर आकाशीय िवदयतउ होता ह

3 None of the above उपरो म स कोई भी नही

4 A high density cloud contains positively and negatively charged electric ions and when this interacts light andsound are simultaneously produced

एक उ घन बादल म धनाक और ऋणाक आविशत िवदयत आयन होत ह और जब यह परर भाव डालत ह तोकाश और िन एक साथ उािदत होती ह

Correct Answer -

A high density cloud contains positively and negatively charged electric ions and when this interacts light andsound are simultaneously produced

औ ि ि ो औ ो

93)

94)

95)

एक उ घन बादल म धनाक और ऋणाक आविशत िवदयत आयन होत ह और जब यह परर भाव डालत ह तोकाश और िन एक साथ उािदत होती ह

Doon Valley is able to grow rice because दन घाटी चावल उगान म सम ह ोिक

1 Other crops cannot be grown वहा अ फसलो को उगाया नही जा सकता ह

2 People in the valley are rice eaters घाटी म लोग चावल खान वाल ह

3 There is a huge export demand of rice वहा चावल की भारी िनयात माग ह

4 It has warm summer and snow melt waters for irrigation

वहा गिमया गम होती ह िसचाई क िलए बफ का िपघला आ पानी होता ह

Correct Answer -

It has warm summer and snow melt waters for irrigation

वहा गिमया गम होती ह िसचाई क िलए बफ का िपघला आ पानी होता ह

CANCELLED

In the geological time scale the Mesozoic Era DOES NOT contains which of the following periods

भगभय समय पमान पर मजीवी यग म िन कालो म स कौन नही ह

1 Triassic ट ाइऐिसक

2 Jurassic जरिसक

3 Cretaceous चाकमय

4 Carboniferous काबनी

Correct Answer -

Carboniferous काबनी

96)

1 P-3 Q-4 R-2 S-1

2 P-3 Q-4 R-1 S-2

3 P-3 Q-4 R-1 S-2

4 P-4 Q-3 R-2 S-1

Correct Answer -

P-4 Q-3 R-2 S-1

1 P-3 Q-1 R-4 S-2

2 P-3 Q-4 R-1 S-2

3 P-3 Q-2 R-4 S-1

97)

98)

4 P-2 Q-1 R-4 S-3

Correct Answer -

P-3 Q-4 R-1 S-2

1 P-3 Q-1 R-4 S-2

2 P-2 Q-3 R-4 S-1

3 P-2 Q-1 R-3 S-4

4 P-4 Q-2 R-1 S-3

Correct Answer -

P-3 Q-1 R-4 S-2

99)

100)

1 P-3 Q-2 R-4 S-1

2 P-1 Q-2 R-3 S-4

3 P-2 Q-3 R-1 S-4

4 P-4 Q-3 R-2 S-1

Correct Answer -

P-2 Q-3 R-1 S-4

ldquoHuman geography is the study of changing relationship between the unresting man and the unstableearthrdquo was defined by

lsquolsquoमानव भगोल ाकल आदमी और अथर पी क बीच सबध परवतन का अयन हrdquo ______ ारा परभािषत िकया गयाथा

1 J Brunches ज चस

2 EC Semple ईसी सल

3 HJ Mackinder एच ज मिकदर

4 PV Blache पीवी च

Correct Answer -

EC Semple ईसी सल

Sedimentary rocks are finally and ultimately derived from the____________

अवसादी चान अततः ________ स ा की जाती ह

1 action of earth movements पी की गितिविधयो

2 marine deposit समी िनप

3 weathering of metamorphic rocks पातरत चानो क अपय

4 weathering of igneous rocks आय चानो क अपय

Correct Answer -

weathering of igneous rocks आय चानो क अपय

Page 40: High School Teacher Eligibility Test- BOARD PROFESSIONAL ...peb.mp.gov.in/results/RESULT_18/HST_RES18/Final_anwser_key/HST… · M a ndl a / मंड ल ... Under the Madhya Pradesh

72)

73)

िनिलखत म स िकसका भाव भारत की जलवाय पर नही पड़ता ह

1 Ocean currents सागर की लहर

2 Nearness to equator भम रखा स िनकटता

3 Monsoons मानसन

4 Presence of Indian ocean भारतीय महासागर की उपथित

Correct Answer -

Ocean currents सागर की लहर

Which of the following cloud types has the characteristics like vertical tall narrow and puffy

िनिलखत म स िकस कार क मघो म लबवत लबी सकीण और थलता जसी िवशषताए ह

1 Cumulonimbus तफानी मघ

2 Cumulus मघ पज

3 Cirrocumulus पाभ कपासी मघ

4 Nimbostratus वषारी मघ

Correct Answer -

Cumulus मघ पज

Which of the following statement is INCORRECT about Crude Birth Rate

िनिलखत स कौन सा कथन अशोिधत ज दर क बार म सही नही ह

1 It cannot be used for comparing fertility level between two countries with different population characteristics

इसका उपयोग िविभ जनसा िवशषताओ वाल दो दशो क बीच जनन र की तलना क िलए नही िकया जा सकता ह

2 It is a standardized measure of fertility

यह जनन मता का मानकीकत उपाय ह

3 It is effected by the age-sex composition of the population

यह आबादी की आय-िलग सरचना स भािवत होता ह

4 It is expressed per 1000 population in a given geographical unit

यह िकसी दी गई भौगोिलक इकाई म ित 1000 जनसा पर िकया जाता ह

Correct Answer -

It is a standardized measure of fertility

यह जनन मता का मानकीकत उपाय ह

74)

75)

76)

77)

Which of the following state in India experienced negative decadal growth rate during 2001 to 2011census

भारत म िनिलखत म स िकस रा म वष 2001 स 2011 की जनगणना क दौरान नकाराक िगरावट दर ई

1 Tripura िपरा

2 Nagaland नागालड

3 Haryana हरयाणा

4 Odisha ओिडसा

Correct Answer -

Nagaland नागालड

Which of the following is NOT a characteristic of peninsular rivers

िनिलखत म स कौन सी िवशषता ायीपीय निदयो म नही होती ह

1 Flow through shallow valleys उथल घािटयो क माम स वाह

2 Seasonal flow मौसमी वाह

3 Little erosional activity थोड़ी कटावदार गितिविध

4 Meandering tendency often shifting their beds घमावदार वि अर अपन तटो को थानातरत करना

Correct Answer -

Meandering tendency often shifting their beds घमावदार वि अर अपन तटो को थानातरत करना

Which of the following gases in the atmosphere absorbs heat from the Sunrsquos radiation and the Earthssurface

वायमडल म िनिलखत म स कौन सी गस सय क िविकरण और पी की सतह स ऊा को अवशोिषत करती ह

1 Neon िनयॉन

2 Carbon dioxide काबन डाइऑाइड

3 Argon आगन

4 Nitrogen नाइट ोजन

Correct Answer -

Carbon dioxide काबन डाइऑाइड

Which of the following kind of settlement pattern is found at the confluence of rivers

ि ि ि ि ो

78)

79)

80)

िनिलखत म स िकस कार का वथापन पटन निदयो क सगम पर पाया जाता ह

1 Triangular Paern िकोणीय पटन

2 Circular or Semi-Circular Paern परप या अध-परप पटन

3 Nebular Paern नबलर पटन

4 Star ndashShaped Paern ार-आकार का पटन

Correct Answer -

Triangular Paern िकोणीय पटन

Which one was not the objective of the Biosphere Reserve Projects launched by the UNESCO

यनो ारा श की गई सरित जवमडल परयोजनाओ का उ इनम स कौन सा नही था

1 To promote teaching and research िशण और अनसधान को बढ़ावा दना

2 To make agriculture sustainable किष को दीघकािलक बनाना

3 To conserve ecosystems पारथितक त को सरित करना

4 To conserve genetic diversity for a longtime लब समय तक अनवािशक िविवधता को सरित करना

Correct Answer -

To make agriculture sustainable किष को दीघकािलक बनाना

Which region of the Earth surface receives the highest amount of insulation

पी सतह का कौन सा तापावरोधन की उतम माा ा करता ह

1 Land mass थलखड

2 Savannah region सवाना

3 Water bodies जल िनकाय

4 Tropical desert उकिटबधीय रिगान

Correct Answer -

Tropical desert उकिटबधीय रिगान

Which one of the following is not a biodiversity hotspot

िनिलखत म स कौन सा जव िविवधता का म जगह नही ह

1 Eastern Himalaya पव िहमालय

2 Eastern Ghats पव घाट

81)

82)

83)

3 Indo-Myanmar भारत-ामार

4 Westerm Ghats पिमी घाट

Correct Answer -

Eastern Ghats पव घाट

Which one of the following is NOT a part of the World Network of Biosphere Reserves based on theUNESCO Man and Biosphere Programme

यनो मन और बायोीयर कायम क आधार पर िनिलखत म स कौन बायोीयर रजव क िव नटवक कािहा नही ह

1 Gulf of Mannar मार की खाड़ी

2 Seshachalam शषाचलम

3 Sunderban सदरबन

4 Nilgiri नीलिगर

Correct Answer -

Seshachalam शषाचलम

Which one of the following is an example of ldquodesert vegetationrdquo

िनिलखत म स कौन मथलीय वनित का एक उदाहरण ह

1 Mosses and lichens दलदल और शवाल

2 Temperate grassland समशीतो घास क मदान

3 Coniferous forest शकधारी वन

4 Acacia and cactus एकािसया और कस

Correct Answer -

Acacia and cactus एकािसया और कस

Which one of the following reflects more sunlight िनिलखत म स कौन सा सय की रोशनी को अिधकपरावितत करता ह

1 Paddy crop land धान फसल भिम

2 Land covered with fresh snow ताजा बफ स आािदत भिम

3 Sand desert रतीली रिगान

4 Prairie land यरी भिम

84)

85)

86)

87)

Correct Answer -

Land covered with fresh snow ताजा बफ स आािदत भिम

Which layer of the atmosphere is in contact with the surface of the earthrsquos oceans

वायमडल की कौन सी परत पी क महासागरो की सतह क सपक म ह

1 Stratosphere समताप मडल

2 Mesosphere म मडल

3 Hydrosphere जलमडल

4 Troposphere ोभ मडल

Correct Answer -

Troposphere ोभ मडल

Mediterranean Sea is a border of which of the following countries भम सागर िनिलखत दशो म सिकसकी सीमा ह

1 None of these इनम स कोई नही

2 Iraq इराक

3 Lebanon लबनान

4 Jordan जॉडन

Correct Answer -

Lebanon लबनान

Benguela ocean currents are found along which coast बगएला महासागर धाराए िकस तट क साथ पायीजाती ह

1 East Coast of South America दिण अमरका क पव तट

2 East Coast of Africa अीका क पव तट

3 West Coast of South America दिण अमरका क पिमी तट

4 West Coast of Africa अीका क पिमी तट

Correct Answer -

West Coast of Africa अीका क पिमी तट

88)

89)

90)

Due to tension a block of land on one side being pushed up or upthrown relative to the downthrown blockis referred as

तनाव क कारण नीच फ क ए खड क साप भिम का एक खड एक ओर स ऊपर धकला जाता ह या ऊपर की ओरफ का जाता ह यह _____ क प म सदिभत ह

1 Thrust fault प श

2 Normal fault सामा श

3 Reverse fault म श

4 Strike slip fault नितलब सपण श

Correct Answer -

Normal fault सामा श

Inter-tropical doldrums is a zone of ______ अतर-उकिटबधीय डोलड ______ का एक ह

1 Frontolysis टोलायिसस

2 Convergence अिभसरण

3 Inter-tropical divergence zone अतर-उकिटबधीय िवचलन

4 Local wind थानीय वाय

Correct Answer -

Convergence अिभसरण

The Horse Latitudes are regions located at about _____ north and south of the equator

हॉस अाश भम रखा क उर और दिण म लगभग _____ पर थत ह

1 30ndash60 degree Latitude 30-60 िडी अाश

2 0ndash5 degree Latitude 0-5 िडी अाश

3 30 degree Latitude 30 िडी अाश

4 60ndash90 degree Latitude 60-90 िडी अाश

Correct Answer -

30 degree Latitude 30 िडी अाश

Generally evaporation is high over which part of the Earth

आम तौर पर पी क िकस भाग पर वाीकरण अिधक होता ह

1 Equatorial maritime भमवत समीय ी ी

91)

92)

2 Equatorial continental भमवत महाीपीय

3 Polar maritime वीय समीय

4 Polar continental वीय महाीपीय

Correct Answer -

Equatorial maritime भमवत समीय

A very high temperature during summer in north western India leads to what type of climaticcondition in south

उर पिमी भारत म गम क दौरान बत अिधक तापमान होन क कारण दिण म िकस कार की जलवाय थितउ करता ह

1 Depression over arabian sea अरब सागर पर अवनमन

2 Failure monsoon मानसन िवफलता

3 Successful monsoon मानसन सफलता

4 Cyclones चवात

Correct Answer -

Successful monsoon मानसन सफलता

Lightning and thunder are the resultant effect when तिड़त और गजन परणामी भाव ह जब

1 Two massive clouds hit powerfully each other first lightning is produced and later sound is produced

दो बड़ बादल एक दसर स शशाली ढग स टकरात ह पहल आकाशीय िवदयत उ होता ह और बाद म िन उहोती ह

2 Two massive clouds come into contact with the powerful wind collision this results into first sound and thenlightning

दो बड़ बादल शशाली पवन सघ क सपक म आत ह इसका परणामप पहल िन और िफर आकाशीय िवदयतउ होता ह

3 None of the above उपरो म स कोई भी नही

4 A high density cloud contains positively and negatively charged electric ions and when this interacts light andsound are simultaneously produced

एक उ घन बादल म धनाक और ऋणाक आविशत िवदयत आयन होत ह और जब यह परर भाव डालत ह तोकाश और िन एक साथ उािदत होती ह

Correct Answer -

A high density cloud contains positively and negatively charged electric ions and when this interacts light andsound are simultaneously produced

औ ि ि ो औ ो

93)

94)

95)

एक उ घन बादल म धनाक और ऋणाक आविशत िवदयत आयन होत ह और जब यह परर भाव डालत ह तोकाश और िन एक साथ उािदत होती ह

Doon Valley is able to grow rice because दन घाटी चावल उगान म सम ह ोिक

1 Other crops cannot be grown वहा अ फसलो को उगाया नही जा सकता ह

2 People in the valley are rice eaters घाटी म लोग चावल खान वाल ह

3 There is a huge export demand of rice वहा चावल की भारी िनयात माग ह

4 It has warm summer and snow melt waters for irrigation

वहा गिमया गम होती ह िसचाई क िलए बफ का िपघला आ पानी होता ह

Correct Answer -

It has warm summer and snow melt waters for irrigation

वहा गिमया गम होती ह िसचाई क िलए बफ का िपघला आ पानी होता ह

CANCELLED

In the geological time scale the Mesozoic Era DOES NOT contains which of the following periods

भगभय समय पमान पर मजीवी यग म िन कालो म स कौन नही ह

1 Triassic ट ाइऐिसक

2 Jurassic जरिसक

3 Cretaceous चाकमय

4 Carboniferous काबनी

Correct Answer -

Carboniferous काबनी

96)

1 P-3 Q-4 R-2 S-1

2 P-3 Q-4 R-1 S-2

3 P-3 Q-4 R-1 S-2

4 P-4 Q-3 R-2 S-1

Correct Answer -

P-4 Q-3 R-2 S-1

1 P-3 Q-1 R-4 S-2

2 P-3 Q-4 R-1 S-2

3 P-3 Q-2 R-4 S-1

97)

98)

4 P-2 Q-1 R-4 S-3

Correct Answer -

P-3 Q-4 R-1 S-2

1 P-3 Q-1 R-4 S-2

2 P-2 Q-3 R-4 S-1

3 P-2 Q-1 R-3 S-4

4 P-4 Q-2 R-1 S-3

Correct Answer -

P-3 Q-1 R-4 S-2

99)

100)

1 P-3 Q-2 R-4 S-1

2 P-1 Q-2 R-3 S-4

3 P-2 Q-3 R-1 S-4

4 P-4 Q-3 R-2 S-1

Correct Answer -

P-2 Q-3 R-1 S-4

ldquoHuman geography is the study of changing relationship between the unresting man and the unstableearthrdquo was defined by

lsquolsquoमानव भगोल ाकल आदमी और अथर पी क बीच सबध परवतन का अयन हrdquo ______ ारा परभािषत िकया गयाथा

1 J Brunches ज चस

2 EC Semple ईसी सल

3 HJ Mackinder एच ज मिकदर

4 PV Blache पीवी च

Correct Answer -

EC Semple ईसी सल

Sedimentary rocks are finally and ultimately derived from the____________

अवसादी चान अततः ________ स ा की जाती ह

1 action of earth movements पी की गितिविधयो

2 marine deposit समी िनप

3 weathering of metamorphic rocks पातरत चानो क अपय

4 weathering of igneous rocks आय चानो क अपय

Correct Answer -

weathering of igneous rocks आय चानो क अपय

Page 41: High School Teacher Eligibility Test- BOARD PROFESSIONAL ...peb.mp.gov.in/results/RESULT_18/HST_RES18/Final_anwser_key/HST… · M a ndl a / मंड ल ... Under the Madhya Pradesh

74)

75)

76)

77)

Which of the following state in India experienced negative decadal growth rate during 2001 to 2011census

भारत म िनिलखत म स िकस रा म वष 2001 स 2011 की जनगणना क दौरान नकाराक िगरावट दर ई

1 Tripura िपरा

2 Nagaland नागालड

3 Haryana हरयाणा

4 Odisha ओिडसा

Correct Answer -

Nagaland नागालड

Which of the following is NOT a characteristic of peninsular rivers

िनिलखत म स कौन सी िवशषता ायीपीय निदयो म नही होती ह

1 Flow through shallow valleys उथल घािटयो क माम स वाह

2 Seasonal flow मौसमी वाह

3 Little erosional activity थोड़ी कटावदार गितिविध

4 Meandering tendency often shifting their beds घमावदार वि अर अपन तटो को थानातरत करना

Correct Answer -

Meandering tendency often shifting their beds घमावदार वि अर अपन तटो को थानातरत करना

Which of the following gases in the atmosphere absorbs heat from the Sunrsquos radiation and the Earthssurface

वायमडल म िनिलखत म स कौन सी गस सय क िविकरण और पी की सतह स ऊा को अवशोिषत करती ह

1 Neon िनयॉन

2 Carbon dioxide काबन डाइऑाइड

3 Argon आगन

4 Nitrogen नाइट ोजन

Correct Answer -

Carbon dioxide काबन डाइऑाइड

Which of the following kind of settlement pattern is found at the confluence of rivers

ि ि ि ि ो

78)

79)

80)

िनिलखत म स िकस कार का वथापन पटन निदयो क सगम पर पाया जाता ह

1 Triangular Paern िकोणीय पटन

2 Circular or Semi-Circular Paern परप या अध-परप पटन

3 Nebular Paern नबलर पटन

4 Star ndashShaped Paern ार-आकार का पटन

Correct Answer -

Triangular Paern िकोणीय पटन

Which one was not the objective of the Biosphere Reserve Projects launched by the UNESCO

यनो ारा श की गई सरित जवमडल परयोजनाओ का उ इनम स कौन सा नही था

1 To promote teaching and research िशण और अनसधान को बढ़ावा दना

2 To make agriculture sustainable किष को दीघकािलक बनाना

3 To conserve ecosystems पारथितक त को सरित करना

4 To conserve genetic diversity for a longtime लब समय तक अनवािशक िविवधता को सरित करना

Correct Answer -

To make agriculture sustainable किष को दीघकािलक बनाना

Which region of the Earth surface receives the highest amount of insulation

पी सतह का कौन सा तापावरोधन की उतम माा ा करता ह

1 Land mass थलखड

2 Savannah region सवाना

3 Water bodies जल िनकाय

4 Tropical desert उकिटबधीय रिगान

Correct Answer -

Tropical desert उकिटबधीय रिगान

Which one of the following is not a biodiversity hotspot

िनिलखत म स कौन सा जव िविवधता का म जगह नही ह

1 Eastern Himalaya पव िहमालय

2 Eastern Ghats पव घाट

81)

82)

83)

3 Indo-Myanmar भारत-ामार

4 Westerm Ghats पिमी घाट

Correct Answer -

Eastern Ghats पव घाट

Which one of the following is NOT a part of the World Network of Biosphere Reserves based on theUNESCO Man and Biosphere Programme

यनो मन और बायोीयर कायम क आधार पर िनिलखत म स कौन बायोीयर रजव क िव नटवक कािहा नही ह

1 Gulf of Mannar मार की खाड़ी

2 Seshachalam शषाचलम

3 Sunderban सदरबन

4 Nilgiri नीलिगर

Correct Answer -

Seshachalam शषाचलम

Which one of the following is an example of ldquodesert vegetationrdquo

िनिलखत म स कौन मथलीय वनित का एक उदाहरण ह

1 Mosses and lichens दलदल और शवाल

2 Temperate grassland समशीतो घास क मदान

3 Coniferous forest शकधारी वन

4 Acacia and cactus एकािसया और कस

Correct Answer -

Acacia and cactus एकािसया और कस

Which one of the following reflects more sunlight िनिलखत म स कौन सा सय की रोशनी को अिधकपरावितत करता ह

1 Paddy crop land धान फसल भिम

2 Land covered with fresh snow ताजा बफ स आािदत भिम

3 Sand desert रतीली रिगान

4 Prairie land यरी भिम

84)

85)

86)

87)

Correct Answer -

Land covered with fresh snow ताजा बफ स आािदत भिम

Which layer of the atmosphere is in contact with the surface of the earthrsquos oceans

वायमडल की कौन सी परत पी क महासागरो की सतह क सपक म ह

1 Stratosphere समताप मडल

2 Mesosphere म मडल

3 Hydrosphere जलमडल

4 Troposphere ोभ मडल

Correct Answer -

Troposphere ोभ मडल

Mediterranean Sea is a border of which of the following countries भम सागर िनिलखत दशो म सिकसकी सीमा ह

1 None of these इनम स कोई नही

2 Iraq इराक

3 Lebanon लबनान

4 Jordan जॉडन

Correct Answer -

Lebanon लबनान

Benguela ocean currents are found along which coast बगएला महासागर धाराए िकस तट क साथ पायीजाती ह

1 East Coast of South America दिण अमरका क पव तट

2 East Coast of Africa अीका क पव तट

3 West Coast of South America दिण अमरका क पिमी तट

4 West Coast of Africa अीका क पिमी तट

Correct Answer -

West Coast of Africa अीका क पिमी तट

88)

89)

90)

Due to tension a block of land on one side being pushed up or upthrown relative to the downthrown blockis referred as

तनाव क कारण नीच फ क ए खड क साप भिम का एक खड एक ओर स ऊपर धकला जाता ह या ऊपर की ओरफ का जाता ह यह _____ क प म सदिभत ह

1 Thrust fault प श

2 Normal fault सामा श

3 Reverse fault म श

4 Strike slip fault नितलब सपण श

Correct Answer -

Normal fault सामा श

Inter-tropical doldrums is a zone of ______ अतर-उकिटबधीय डोलड ______ का एक ह

1 Frontolysis टोलायिसस

2 Convergence अिभसरण

3 Inter-tropical divergence zone अतर-उकिटबधीय िवचलन

4 Local wind थानीय वाय

Correct Answer -

Convergence अिभसरण

The Horse Latitudes are regions located at about _____ north and south of the equator

हॉस अाश भम रखा क उर और दिण म लगभग _____ पर थत ह

1 30ndash60 degree Latitude 30-60 िडी अाश

2 0ndash5 degree Latitude 0-5 िडी अाश

3 30 degree Latitude 30 िडी अाश

4 60ndash90 degree Latitude 60-90 िडी अाश

Correct Answer -

30 degree Latitude 30 िडी अाश

Generally evaporation is high over which part of the Earth

आम तौर पर पी क िकस भाग पर वाीकरण अिधक होता ह

1 Equatorial maritime भमवत समीय ी ी

91)

92)

2 Equatorial continental भमवत महाीपीय

3 Polar maritime वीय समीय

4 Polar continental वीय महाीपीय

Correct Answer -

Equatorial maritime भमवत समीय

A very high temperature during summer in north western India leads to what type of climaticcondition in south

उर पिमी भारत म गम क दौरान बत अिधक तापमान होन क कारण दिण म िकस कार की जलवाय थितउ करता ह

1 Depression over arabian sea अरब सागर पर अवनमन

2 Failure monsoon मानसन िवफलता

3 Successful monsoon मानसन सफलता

4 Cyclones चवात

Correct Answer -

Successful monsoon मानसन सफलता

Lightning and thunder are the resultant effect when तिड़त और गजन परणामी भाव ह जब

1 Two massive clouds hit powerfully each other first lightning is produced and later sound is produced

दो बड़ बादल एक दसर स शशाली ढग स टकरात ह पहल आकाशीय िवदयत उ होता ह और बाद म िन उहोती ह

2 Two massive clouds come into contact with the powerful wind collision this results into first sound and thenlightning

दो बड़ बादल शशाली पवन सघ क सपक म आत ह इसका परणामप पहल िन और िफर आकाशीय िवदयतउ होता ह

3 None of the above उपरो म स कोई भी नही

4 A high density cloud contains positively and negatively charged electric ions and when this interacts light andsound are simultaneously produced

एक उ घन बादल म धनाक और ऋणाक आविशत िवदयत आयन होत ह और जब यह परर भाव डालत ह तोकाश और िन एक साथ उािदत होती ह

Correct Answer -

A high density cloud contains positively and negatively charged electric ions and when this interacts light andsound are simultaneously produced

औ ि ि ो औ ो

93)

94)

95)

एक उ घन बादल म धनाक और ऋणाक आविशत िवदयत आयन होत ह और जब यह परर भाव डालत ह तोकाश और िन एक साथ उािदत होती ह

Doon Valley is able to grow rice because दन घाटी चावल उगान म सम ह ोिक

1 Other crops cannot be grown वहा अ फसलो को उगाया नही जा सकता ह

2 People in the valley are rice eaters घाटी म लोग चावल खान वाल ह

3 There is a huge export demand of rice वहा चावल की भारी िनयात माग ह

4 It has warm summer and snow melt waters for irrigation

वहा गिमया गम होती ह िसचाई क िलए बफ का िपघला आ पानी होता ह

Correct Answer -

It has warm summer and snow melt waters for irrigation

वहा गिमया गम होती ह िसचाई क िलए बफ का िपघला आ पानी होता ह

CANCELLED

In the geological time scale the Mesozoic Era DOES NOT contains which of the following periods

भगभय समय पमान पर मजीवी यग म िन कालो म स कौन नही ह

1 Triassic ट ाइऐिसक

2 Jurassic जरिसक

3 Cretaceous चाकमय

4 Carboniferous काबनी

Correct Answer -

Carboniferous काबनी

96)

1 P-3 Q-4 R-2 S-1

2 P-3 Q-4 R-1 S-2

3 P-3 Q-4 R-1 S-2

4 P-4 Q-3 R-2 S-1

Correct Answer -

P-4 Q-3 R-2 S-1

1 P-3 Q-1 R-4 S-2

2 P-3 Q-4 R-1 S-2

3 P-3 Q-2 R-4 S-1

97)

98)

4 P-2 Q-1 R-4 S-3

Correct Answer -

P-3 Q-4 R-1 S-2

1 P-3 Q-1 R-4 S-2

2 P-2 Q-3 R-4 S-1

3 P-2 Q-1 R-3 S-4

4 P-4 Q-2 R-1 S-3

Correct Answer -

P-3 Q-1 R-4 S-2

99)

100)

1 P-3 Q-2 R-4 S-1

2 P-1 Q-2 R-3 S-4

3 P-2 Q-3 R-1 S-4

4 P-4 Q-3 R-2 S-1

Correct Answer -

P-2 Q-3 R-1 S-4

ldquoHuman geography is the study of changing relationship between the unresting man and the unstableearthrdquo was defined by

lsquolsquoमानव भगोल ाकल आदमी और अथर पी क बीच सबध परवतन का अयन हrdquo ______ ारा परभािषत िकया गयाथा

1 J Brunches ज चस

2 EC Semple ईसी सल

3 HJ Mackinder एच ज मिकदर

4 PV Blache पीवी च

Correct Answer -

EC Semple ईसी सल

Sedimentary rocks are finally and ultimately derived from the____________

अवसादी चान अततः ________ स ा की जाती ह

1 action of earth movements पी की गितिविधयो

2 marine deposit समी िनप

3 weathering of metamorphic rocks पातरत चानो क अपय

4 weathering of igneous rocks आय चानो क अपय

Correct Answer -

weathering of igneous rocks आय चानो क अपय

Page 42: High School Teacher Eligibility Test- BOARD PROFESSIONAL ...peb.mp.gov.in/results/RESULT_18/HST_RES18/Final_anwser_key/HST… · M a ndl a / मंड ल ... Under the Madhya Pradesh

78)

79)

80)

िनिलखत म स िकस कार का वथापन पटन निदयो क सगम पर पाया जाता ह

1 Triangular Paern िकोणीय पटन

2 Circular or Semi-Circular Paern परप या अध-परप पटन

3 Nebular Paern नबलर पटन

4 Star ndashShaped Paern ार-आकार का पटन

Correct Answer -

Triangular Paern िकोणीय पटन

Which one was not the objective of the Biosphere Reserve Projects launched by the UNESCO

यनो ारा श की गई सरित जवमडल परयोजनाओ का उ इनम स कौन सा नही था

1 To promote teaching and research िशण और अनसधान को बढ़ावा दना

2 To make agriculture sustainable किष को दीघकािलक बनाना

3 To conserve ecosystems पारथितक त को सरित करना

4 To conserve genetic diversity for a longtime लब समय तक अनवािशक िविवधता को सरित करना

Correct Answer -

To make agriculture sustainable किष को दीघकािलक बनाना

Which region of the Earth surface receives the highest amount of insulation

पी सतह का कौन सा तापावरोधन की उतम माा ा करता ह

1 Land mass थलखड

2 Savannah region सवाना

3 Water bodies जल िनकाय

4 Tropical desert उकिटबधीय रिगान

Correct Answer -

Tropical desert उकिटबधीय रिगान

Which one of the following is not a biodiversity hotspot

िनिलखत म स कौन सा जव िविवधता का म जगह नही ह

1 Eastern Himalaya पव िहमालय

2 Eastern Ghats पव घाट

81)

82)

83)

3 Indo-Myanmar भारत-ामार

4 Westerm Ghats पिमी घाट

Correct Answer -

Eastern Ghats पव घाट

Which one of the following is NOT a part of the World Network of Biosphere Reserves based on theUNESCO Man and Biosphere Programme

यनो मन और बायोीयर कायम क आधार पर िनिलखत म स कौन बायोीयर रजव क िव नटवक कािहा नही ह

1 Gulf of Mannar मार की खाड़ी

2 Seshachalam शषाचलम

3 Sunderban सदरबन

4 Nilgiri नीलिगर

Correct Answer -

Seshachalam शषाचलम

Which one of the following is an example of ldquodesert vegetationrdquo

िनिलखत म स कौन मथलीय वनित का एक उदाहरण ह

1 Mosses and lichens दलदल और शवाल

2 Temperate grassland समशीतो घास क मदान

3 Coniferous forest शकधारी वन

4 Acacia and cactus एकािसया और कस

Correct Answer -

Acacia and cactus एकािसया और कस

Which one of the following reflects more sunlight िनिलखत म स कौन सा सय की रोशनी को अिधकपरावितत करता ह

1 Paddy crop land धान फसल भिम

2 Land covered with fresh snow ताजा बफ स आािदत भिम

3 Sand desert रतीली रिगान

4 Prairie land यरी भिम

84)

85)

86)

87)

Correct Answer -

Land covered with fresh snow ताजा बफ स आािदत भिम

Which layer of the atmosphere is in contact with the surface of the earthrsquos oceans

वायमडल की कौन सी परत पी क महासागरो की सतह क सपक म ह

1 Stratosphere समताप मडल

2 Mesosphere म मडल

3 Hydrosphere जलमडल

4 Troposphere ोभ मडल

Correct Answer -

Troposphere ोभ मडल

Mediterranean Sea is a border of which of the following countries भम सागर िनिलखत दशो म सिकसकी सीमा ह

1 None of these इनम स कोई नही

2 Iraq इराक

3 Lebanon लबनान

4 Jordan जॉडन

Correct Answer -

Lebanon लबनान

Benguela ocean currents are found along which coast बगएला महासागर धाराए िकस तट क साथ पायीजाती ह

1 East Coast of South America दिण अमरका क पव तट

2 East Coast of Africa अीका क पव तट

3 West Coast of South America दिण अमरका क पिमी तट

4 West Coast of Africa अीका क पिमी तट

Correct Answer -

West Coast of Africa अीका क पिमी तट

88)

89)

90)

Due to tension a block of land on one side being pushed up or upthrown relative to the downthrown blockis referred as

तनाव क कारण नीच फ क ए खड क साप भिम का एक खड एक ओर स ऊपर धकला जाता ह या ऊपर की ओरफ का जाता ह यह _____ क प म सदिभत ह

1 Thrust fault प श

2 Normal fault सामा श

3 Reverse fault म श

4 Strike slip fault नितलब सपण श

Correct Answer -

Normal fault सामा श

Inter-tropical doldrums is a zone of ______ अतर-उकिटबधीय डोलड ______ का एक ह

1 Frontolysis टोलायिसस

2 Convergence अिभसरण

3 Inter-tropical divergence zone अतर-उकिटबधीय िवचलन

4 Local wind थानीय वाय

Correct Answer -

Convergence अिभसरण

The Horse Latitudes are regions located at about _____ north and south of the equator

हॉस अाश भम रखा क उर और दिण म लगभग _____ पर थत ह

1 30ndash60 degree Latitude 30-60 िडी अाश

2 0ndash5 degree Latitude 0-5 िडी अाश

3 30 degree Latitude 30 िडी अाश

4 60ndash90 degree Latitude 60-90 िडी अाश

Correct Answer -

30 degree Latitude 30 िडी अाश

Generally evaporation is high over which part of the Earth

आम तौर पर पी क िकस भाग पर वाीकरण अिधक होता ह

1 Equatorial maritime भमवत समीय ी ी

91)

92)

2 Equatorial continental भमवत महाीपीय

3 Polar maritime वीय समीय

4 Polar continental वीय महाीपीय

Correct Answer -

Equatorial maritime भमवत समीय

A very high temperature during summer in north western India leads to what type of climaticcondition in south

उर पिमी भारत म गम क दौरान बत अिधक तापमान होन क कारण दिण म िकस कार की जलवाय थितउ करता ह

1 Depression over arabian sea अरब सागर पर अवनमन

2 Failure monsoon मानसन िवफलता

3 Successful monsoon मानसन सफलता

4 Cyclones चवात

Correct Answer -

Successful monsoon मानसन सफलता

Lightning and thunder are the resultant effect when तिड़त और गजन परणामी भाव ह जब

1 Two massive clouds hit powerfully each other first lightning is produced and later sound is produced

दो बड़ बादल एक दसर स शशाली ढग स टकरात ह पहल आकाशीय िवदयत उ होता ह और बाद म िन उहोती ह

2 Two massive clouds come into contact with the powerful wind collision this results into first sound and thenlightning

दो बड़ बादल शशाली पवन सघ क सपक म आत ह इसका परणामप पहल िन और िफर आकाशीय िवदयतउ होता ह

3 None of the above उपरो म स कोई भी नही

4 A high density cloud contains positively and negatively charged electric ions and when this interacts light andsound are simultaneously produced

एक उ घन बादल म धनाक और ऋणाक आविशत िवदयत आयन होत ह और जब यह परर भाव डालत ह तोकाश और िन एक साथ उािदत होती ह

Correct Answer -

A high density cloud contains positively and negatively charged electric ions and when this interacts light andsound are simultaneously produced

औ ि ि ो औ ो

93)

94)

95)

एक उ घन बादल म धनाक और ऋणाक आविशत िवदयत आयन होत ह और जब यह परर भाव डालत ह तोकाश और िन एक साथ उािदत होती ह

Doon Valley is able to grow rice because दन घाटी चावल उगान म सम ह ोिक

1 Other crops cannot be grown वहा अ फसलो को उगाया नही जा सकता ह

2 People in the valley are rice eaters घाटी म लोग चावल खान वाल ह

3 There is a huge export demand of rice वहा चावल की भारी िनयात माग ह

4 It has warm summer and snow melt waters for irrigation

वहा गिमया गम होती ह िसचाई क िलए बफ का िपघला आ पानी होता ह

Correct Answer -

It has warm summer and snow melt waters for irrigation

वहा गिमया गम होती ह िसचाई क िलए बफ का िपघला आ पानी होता ह

CANCELLED

In the geological time scale the Mesozoic Era DOES NOT contains which of the following periods

भगभय समय पमान पर मजीवी यग म िन कालो म स कौन नही ह

1 Triassic ट ाइऐिसक

2 Jurassic जरिसक

3 Cretaceous चाकमय

4 Carboniferous काबनी

Correct Answer -

Carboniferous काबनी

96)

1 P-3 Q-4 R-2 S-1

2 P-3 Q-4 R-1 S-2

3 P-3 Q-4 R-1 S-2

4 P-4 Q-3 R-2 S-1

Correct Answer -

P-4 Q-3 R-2 S-1

1 P-3 Q-1 R-4 S-2

2 P-3 Q-4 R-1 S-2

3 P-3 Q-2 R-4 S-1

97)

98)

4 P-2 Q-1 R-4 S-3

Correct Answer -

P-3 Q-4 R-1 S-2

1 P-3 Q-1 R-4 S-2

2 P-2 Q-3 R-4 S-1

3 P-2 Q-1 R-3 S-4

4 P-4 Q-2 R-1 S-3

Correct Answer -

P-3 Q-1 R-4 S-2

99)

100)

1 P-3 Q-2 R-4 S-1

2 P-1 Q-2 R-3 S-4

3 P-2 Q-3 R-1 S-4

4 P-4 Q-3 R-2 S-1

Correct Answer -

P-2 Q-3 R-1 S-4

ldquoHuman geography is the study of changing relationship between the unresting man and the unstableearthrdquo was defined by

lsquolsquoमानव भगोल ाकल आदमी और अथर पी क बीच सबध परवतन का अयन हrdquo ______ ारा परभािषत िकया गयाथा

1 J Brunches ज चस

2 EC Semple ईसी सल

3 HJ Mackinder एच ज मिकदर

4 PV Blache पीवी च

Correct Answer -

EC Semple ईसी सल

Sedimentary rocks are finally and ultimately derived from the____________

अवसादी चान अततः ________ स ा की जाती ह

1 action of earth movements पी की गितिविधयो

2 marine deposit समी िनप

3 weathering of metamorphic rocks पातरत चानो क अपय

4 weathering of igneous rocks आय चानो क अपय

Correct Answer -

weathering of igneous rocks आय चानो क अपय

Page 43: High School Teacher Eligibility Test- BOARD PROFESSIONAL ...peb.mp.gov.in/results/RESULT_18/HST_RES18/Final_anwser_key/HST… · M a ndl a / मंड ल ... Under the Madhya Pradesh

81)

82)

83)

3 Indo-Myanmar भारत-ामार

4 Westerm Ghats पिमी घाट

Correct Answer -

Eastern Ghats पव घाट

Which one of the following is NOT a part of the World Network of Biosphere Reserves based on theUNESCO Man and Biosphere Programme

यनो मन और बायोीयर कायम क आधार पर िनिलखत म स कौन बायोीयर रजव क िव नटवक कािहा नही ह

1 Gulf of Mannar मार की खाड़ी

2 Seshachalam शषाचलम

3 Sunderban सदरबन

4 Nilgiri नीलिगर

Correct Answer -

Seshachalam शषाचलम

Which one of the following is an example of ldquodesert vegetationrdquo

िनिलखत म स कौन मथलीय वनित का एक उदाहरण ह

1 Mosses and lichens दलदल और शवाल

2 Temperate grassland समशीतो घास क मदान

3 Coniferous forest शकधारी वन

4 Acacia and cactus एकािसया और कस

Correct Answer -

Acacia and cactus एकािसया और कस

Which one of the following reflects more sunlight िनिलखत म स कौन सा सय की रोशनी को अिधकपरावितत करता ह

1 Paddy crop land धान फसल भिम

2 Land covered with fresh snow ताजा बफ स आािदत भिम

3 Sand desert रतीली रिगान

4 Prairie land यरी भिम

84)

85)

86)

87)

Correct Answer -

Land covered with fresh snow ताजा बफ स आािदत भिम

Which layer of the atmosphere is in contact with the surface of the earthrsquos oceans

वायमडल की कौन सी परत पी क महासागरो की सतह क सपक म ह

1 Stratosphere समताप मडल

2 Mesosphere म मडल

3 Hydrosphere जलमडल

4 Troposphere ोभ मडल

Correct Answer -

Troposphere ोभ मडल

Mediterranean Sea is a border of which of the following countries भम सागर िनिलखत दशो म सिकसकी सीमा ह

1 None of these इनम स कोई नही

2 Iraq इराक

3 Lebanon लबनान

4 Jordan जॉडन

Correct Answer -

Lebanon लबनान

Benguela ocean currents are found along which coast बगएला महासागर धाराए िकस तट क साथ पायीजाती ह

1 East Coast of South America दिण अमरका क पव तट

2 East Coast of Africa अीका क पव तट

3 West Coast of South America दिण अमरका क पिमी तट

4 West Coast of Africa अीका क पिमी तट

Correct Answer -

West Coast of Africa अीका क पिमी तट

88)

89)

90)

Due to tension a block of land on one side being pushed up or upthrown relative to the downthrown blockis referred as

तनाव क कारण नीच फ क ए खड क साप भिम का एक खड एक ओर स ऊपर धकला जाता ह या ऊपर की ओरफ का जाता ह यह _____ क प म सदिभत ह

1 Thrust fault प श

2 Normal fault सामा श

3 Reverse fault म श

4 Strike slip fault नितलब सपण श

Correct Answer -

Normal fault सामा श

Inter-tropical doldrums is a zone of ______ अतर-उकिटबधीय डोलड ______ का एक ह

1 Frontolysis टोलायिसस

2 Convergence अिभसरण

3 Inter-tropical divergence zone अतर-उकिटबधीय िवचलन

4 Local wind थानीय वाय

Correct Answer -

Convergence अिभसरण

The Horse Latitudes are regions located at about _____ north and south of the equator

हॉस अाश भम रखा क उर और दिण म लगभग _____ पर थत ह

1 30ndash60 degree Latitude 30-60 िडी अाश

2 0ndash5 degree Latitude 0-5 िडी अाश

3 30 degree Latitude 30 िडी अाश

4 60ndash90 degree Latitude 60-90 िडी अाश

Correct Answer -

30 degree Latitude 30 िडी अाश

Generally evaporation is high over which part of the Earth

आम तौर पर पी क िकस भाग पर वाीकरण अिधक होता ह

1 Equatorial maritime भमवत समीय ी ी

91)

92)

2 Equatorial continental भमवत महाीपीय

3 Polar maritime वीय समीय

4 Polar continental वीय महाीपीय

Correct Answer -

Equatorial maritime भमवत समीय

A very high temperature during summer in north western India leads to what type of climaticcondition in south

उर पिमी भारत म गम क दौरान बत अिधक तापमान होन क कारण दिण म िकस कार की जलवाय थितउ करता ह

1 Depression over arabian sea अरब सागर पर अवनमन

2 Failure monsoon मानसन िवफलता

3 Successful monsoon मानसन सफलता

4 Cyclones चवात

Correct Answer -

Successful monsoon मानसन सफलता

Lightning and thunder are the resultant effect when तिड़त और गजन परणामी भाव ह जब

1 Two massive clouds hit powerfully each other first lightning is produced and later sound is produced

दो बड़ बादल एक दसर स शशाली ढग स टकरात ह पहल आकाशीय िवदयत उ होता ह और बाद म िन उहोती ह

2 Two massive clouds come into contact with the powerful wind collision this results into first sound and thenlightning

दो बड़ बादल शशाली पवन सघ क सपक म आत ह इसका परणामप पहल िन और िफर आकाशीय िवदयतउ होता ह

3 None of the above उपरो म स कोई भी नही

4 A high density cloud contains positively and negatively charged electric ions and when this interacts light andsound are simultaneously produced

एक उ घन बादल म धनाक और ऋणाक आविशत िवदयत आयन होत ह और जब यह परर भाव डालत ह तोकाश और िन एक साथ उािदत होती ह

Correct Answer -

A high density cloud contains positively and negatively charged electric ions and when this interacts light andsound are simultaneously produced

औ ि ि ो औ ो

93)

94)

95)

एक उ घन बादल म धनाक और ऋणाक आविशत िवदयत आयन होत ह और जब यह परर भाव डालत ह तोकाश और िन एक साथ उािदत होती ह

Doon Valley is able to grow rice because दन घाटी चावल उगान म सम ह ोिक

1 Other crops cannot be grown वहा अ फसलो को उगाया नही जा सकता ह

2 People in the valley are rice eaters घाटी म लोग चावल खान वाल ह

3 There is a huge export demand of rice वहा चावल की भारी िनयात माग ह

4 It has warm summer and snow melt waters for irrigation

वहा गिमया गम होती ह िसचाई क िलए बफ का िपघला आ पानी होता ह

Correct Answer -

It has warm summer and snow melt waters for irrigation

वहा गिमया गम होती ह िसचाई क िलए बफ का िपघला आ पानी होता ह

CANCELLED

In the geological time scale the Mesozoic Era DOES NOT contains which of the following periods

भगभय समय पमान पर मजीवी यग म िन कालो म स कौन नही ह

1 Triassic ट ाइऐिसक

2 Jurassic जरिसक

3 Cretaceous चाकमय

4 Carboniferous काबनी

Correct Answer -

Carboniferous काबनी

96)

1 P-3 Q-4 R-2 S-1

2 P-3 Q-4 R-1 S-2

3 P-3 Q-4 R-1 S-2

4 P-4 Q-3 R-2 S-1

Correct Answer -

P-4 Q-3 R-2 S-1

1 P-3 Q-1 R-4 S-2

2 P-3 Q-4 R-1 S-2

3 P-3 Q-2 R-4 S-1

97)

98)

4 P-2 Q-1 R-4 S-3

Correct Answer -

P-3 Q-4 R-1 S-2

1 P-3 Q-1 R-4 S-2

2 P-2 Q-3 R-4 S-1

3 P-2 Q-1 R-3 S-4

4 P-4 Q-2 R-1 S-3

Correct Answer -

P-3 Q-1 R-4 S-2

99)

100)

1 P-3 Q-2 R-4 S-1

2 P-1 Q-2 R-3 S-4

3 P-2 Q-3 R-1 S-4

4 P-4 Q-3 R-2 S-1

Correct Answer -

P-2 Q-3 R-1 S-4

ldquoHuman geography is the study of changing relationship between the unresting man and the unstableearthrdquo was defined by

lsquolsquoमानव भगोल ाकल आदमी और अथर पी क बीच सबध परवतन का अयन हrdquo ______ ारा परभािषत िकया गयाथा

1 J Brunches ज चस

2 EC Semple ईसी सल

3 HJ Mackinder एच ज मिकदर

4 PV Blache पीवी च

Correct Answer -

EC Semple ईसी सल

Sedimentary rocks are finally and ultimately derived from the____________

अवसादी चान अततः ________ स ा की जाती ह

1 action of earth movements पी की गितिविधयो

2 marine deposit समी िनप

3 weathering of metamorphic rocks पातरत चानो क अपय

4 weathering of igneous rocks आय चानो क अपय

Correct Answer -

weathering of igneous rocks आय चानो क अपय

Page 44: High School Teacher Eligibility Test- BOARD PROFESSIONAL ...peb.mp.gov.in/results/RESULT_18/HST_RES18/Final_anwser_key/HST… · M a ndl a / मंड ल ... Under the Madhya Pradesh

84)

85)

86)

87)

Correct Answer -

Land covered with fresh snow ताजा बफ स आािदत भिम

Which layer of the atmosphere is in contact with the surface of the earthrsquos oceans

वायमडल की कौन सी परत पी क महासागरो की सतह क सपक म ह

1 Stratosphere समताप मडल

2 Mesosphere म मडल

3 Hydrosphere जलमडल

4 Troposphere ोभ मडल

Correct Answer -

Troposphere ोभ मडल

Mediterranean Sea is a border of which of the following countries भम सागर िनिलखत दशो म सिकसकी सीमा ह

1 None of these इनम स कोई नही

2 Iraq इराक

3 Lebanon लबनान

4 Jordan जॉडन

Correct Answer -

Lebanon लबनान

Benguela ocean currents are found along which coast बगएला महासागर धाराए िकस तट क साथ पायीजाती ह

1 East Coast of South America दिण अमरका क पव तट

2 East Coast of Africa अीका क पव तट

3 West Coast of South America दिण अमरका क पिमी तट

4 West Coast of Africa अीका क पिमी तट

Correct Answer -

West Coast of Africa अीका क पिमी तट

88)

89)

90)

Due to tension a block of land on one side being pushed up or upthrown relative to the downthrown blockis referred as

तनाव क कारण नीच फ क ए खड क साप भिम का एक खड एक ओर स ऊपर धकला जाता ह या ऊपर की ओरफ का जाता ह यह _____ क प म सदिभत ह

1 Thrust fault प श

2 Normal fault सामा श

3 Reverse fault म श

4 Strike slip fault नितलब सपण श

Correct Answer -

Normal fault सामा श

Inter-tropical doldrums is a zone of ______ अतर-उकिटबधीय डोलड ______ का एक ह

1 Frontolysis टोलायिसस

2 Convergence अिभसरण

3 Inter-tropical divergence zone अतर-उकिटबधीय िवचलन

4 Local wind थानीय वाय

Correct Answer -

Convergence अिभसरण

The Horse Latitudes are regions located at about _____ north and south of the equator

हॉस अाश भम रखा क उर और दिण म लगभग _____ पर थत ह

1 30ndash60 degree Latitude 30-60 िडी अाश

2 0ndash5 degree Latitude 0-5 िडी अाश

3 30 degree Latitude 30 िडी अाश

4 60ndash90 degree Latitude 60-90 िडी अाश

Correct Answer -

30 degree Latitude 30 िडी अाश

Generally evaporation is high over which part of the Earth

आम तौर पर पी क िकस भाग पर वाीकरण अिधक होता ह

1 Equatorial maritime भमवत समीय ी ी

91)

92)

2 Equatorial continental भमवत महाीपीय

3 Polar maritime वीय समीय

4 Polar continental वीय महाीपीय

Correct Answer -

Equatorial maritime भमवत समीय

A very high temperature during summer in north western India leads to what type of climaticcondition in south

उर पिमी भारत म गम क दौरान बत अिधक तापमान होन क कारण दिण म िकस कार की जलवाय थितउ करता ह

1 Depression over arabian sea अरब सागर पर अवनमन

2 Failure monsoon मानसन िवफलता

3 Successful monsoon मानसन सफलता

4 Cyclones चवात

Correct Answer -

Successful monsoon मानसन सफलता

Lightning and thunder are the resultant effect when तिड़त और गजन परणामी भाव ह जब

1 Two massive clouds hit powerfully each other first lightning is produced and later sound is produced

दो बड़ बादल एक दसर स शशाली ढग स टकरात ह पहल आकाशीय िवदयत उ होता ह और बाद म िन उहोती ह

2 Two massive clouds come into contact with the powerful wind collision this results into first sound and thenlightning

दो बड़ बादल शशाली पवन सघ क सपक म आत ह इसका परणामप पहल िन और िफर आकाशीय िवदयतउ होता ह

3 None of the above उपरो म स कोई भी नही

4 A high density cloud contains positively and negatively charged electric ions and when this interacts light andsound are simultaneously produced

एक उ घन बादल म धनाक और ऋणाक आविशत िवदयत आयन होत ह और जब यह परर भाव डालत ह तोकाश और िन एक साथ उािदत होती ह

Correct Answer -

A high density cloud contains positively and negatively charged electric ions and when this interacts light andsound are simultaneously produced

औ ि ि ो औ ो

93)

94)

95)

एक उ घन बादल म धनाक और ऋणाक आविशत िवदयत आयन होत ह और जब यह परर भाव डालत ह तोकाश और िन एक साथ उािदत होती ह

Doon Valley is able to grow rice because दन घाटी चावल उगान म सम ह ोिक

1 Other crops cannot be grown वहा अ फसलो को उगाया नही जा सकता ह

2 People in the valley are rice eaters घाटी म लोग चावल खान वाल ह

3 There is a huge export demand of rice वहा चावल की भारी िनयात माग ह

4 It has warm summer and snow melt waters for irrigation

वहा गिमया गम होती ह िसचाई क िलए बफ का िपघला आ पानी होता ह

Correct Answer -

It has warm summer and snow melt waters for irrigation

वहा गिमया गम होती ह िसचाई क िलए बफ का िपघला आ पानी होता ह

CANCELLED

In the geological time scale the Mesozoic Era DOES NOT contains which of the following periods

भगभय समय पमान पर मजीवी यग म िन कालो म स कौन नही ह

1 Triassic ट ाइऐिसक

2 Jurassic जरिसक

3 Cretaceous चाकमय

4 Carboniferous काबनी

Correct Answer -

Carboniferous काबनी

96)

1 P-3 Q-4 R-2 S-1

2 P-3 Q-4 R-1 S-2

3 P-3 Q-4 R-1 S-2

4 P-4 Q-3 R-2 S-1

Correct Answer -

P-4 Q-3 R-2 S-1

1 P-3 Q-1 R-4 S-2

2 P-3 Q-4 R-1 S-2

3 P-3 Q-2 R-4 S-1

97)

98)

4 P-2 Q-1 R-4 S-3

Correct Answer -

P-3 Q-4 R-1 S-2

1 P-3 Q-1 R-4 S-2

2 P-2 Q-3 R-4 S-1

3 P-2 Q-1 R-3 S-4

4 P-4 Q-2 R-1 S-3

Correct Answer -

P-3 Q-1 R-4 S-2

99)

100)

1 P-3 Q-2 R-4 S-1

2 P-1 Q-2 R-3 S-4

3 P-2 Q-3 R-1 S-4

4 P-4 Q-3 R-2 S-1

Correct Answer -

P-2 Q-3 R-1 S-4

ldquoHuman geography is the study of changing relationship between the unresting man and the unstableearthrdquo was defined by

lsquolsquoमानव भगोल ाकल आदमी और अथर पी क बीच सबध परवतन का अयन हrdquo ______ ारा परभािषत िकया गयाथा

1 J Brunches ज चस

2 EC Semple ईसी सल

3 HJ Mackinder एच ज मिकदर

4 PV Blache पीवी च

Correct Answer -

EC Semple ईसी सल

Sedimentary rocks are finally and ultimately derived from the____________

अवसादी चान अततः ________ स ा की जाती ह

1 action of earth movements पी की गितिविधयो

2 marine deposit समी िनप

3 weathering of metamorphic rocks पातरत चानो क अपय

4 weathering of igneous rocks आय चानो क अपय

Correct Answer -

weathering of igneous rocks आय चानो क अपय

Page 45: High School Teacher Eligibility Test- BOARD PROFESSIONAL ...peb.mp.gov.in/results/RESULT_18/HST_RES18/Final_anwser_key/HST… · M a ndl a / मंड ल ... Under the Madhya Pradesh

88)

89)

90)

Due to tension a block of land on one side being pushed up or upthrown relative to the downthrown blockis referred as

तनाव क कारण नीच फ क ए खड क साप भिम का एक खड एक ओर स ऊपर धकला जाता ह या ऊपर की ओरफ का जाता ह यह _____ क प म सदिभत ह

1 Thrust fault प श

2 Normal fault सामा श

3 Reverse fault म श

4 Strike slip fault नितलब सपण श

Correct Answer -

Normal fault सामा श

Inter-tropical doldrums is a zone of ______ अतर-उकिटबधीय डोलड ______ का एक ह

1 Frontolysis टोलायिसस

2 Convergence अिभसरण

3 Inter-tropical divergence zone अतर-उकिटबधीय िवचलन

4 Local wind थानीय वाय

Correct Answer -

Convergence अिभसरण

The Horse Latitudes are regions located at about _____ north and south of the equator

हॉस अाश भम रखा क उर और दिण म लगभग _____ पर थत ह

1 30ndash60 degree Latitude 30-60 िडी अाश

2 0ndash5 degree Latitude 0-5 िडी अाश

3 30 degree Latitude 30 िडी अाश

4 60ndash90 degree Latitude 60-90 िडी अाश

Correct Answer -

30 degree Latitude 30 िडी अाश

Generally evaporation is high over which part of the Earth

आम तौर पर पी क िकस भाग पर वाीकरण अिधक होता ह

1 Equatorial maritime भमवत समीय ी ी

91)

92)

2 Equatorial continental भमवत महाीपीय

3 Polar maritime वीय समीय

4 Polar continental वीय महाीपीय

Correct Answer -

Equatorial maritime भमवत समीय

A very high temperature during summer in north western India leads to what type of climaticcondition in south

उर पिमी भारत म गम क दौरान बत अिधक तापमान होन क कारण दिण म िकस कार की जलवाय थितउ करता ह

1 Depression over arabian sea अरब सागर पर अवनमन

2 Failure monsoon मानसन िवफलता

3 Successful monsoon मानसन सफलता

4 Cyclones चवात

Correct Answer -

Successful monsoon मानसन सफलता

Lightning and thunder are the resultant effect when तिड़त और गजन परणामी भाव ह जब

1 Two massive clouds hit powerfully each other first lightning is produced and later sound is produced

दो बड़ बादल एक दसर स शशाली ढग स टकरात ह पहल आकाशीय िवदयत उ होता ह और बाद म िन उहोती ह

2 Two massive clouds come into contact with the powerful wind collision this results into first sound and thenlightning

दो बड़ बादल शशाली पवन सघ क सपक म आत ह इसका परणामप पहल िन और िफर आकाशीय िवदयतउ होता ह

3 None of the above उपरो म स कोई भी नही

4 A high density cloud contains positively and negatively charged electric ions and when this interacts light andsound are simultaneously produced

एक उ घन बादल म धनाक और ऋणाक आविशत िवदयत आयन होत ह और जब यह परर भाव डालत ह तोकाश और िन एक साथ उािदत होती ह

Correct Answer -

A high density cloud contains positively and negatively charged electric ions and when this interacts light andsound are simultaneously produced

औ ि ि ो औ ो

93)

94)

95)

एक उ घन बादल म धनाक और ऋणाक आविशत िवदयत आयन होत ह और जब यह परर भाव डालत ह तोकाश और िन एक साथ उािदत होती ह

Doon Valley is able to grow rice because दन घाटी चावल उगान म सम ह ोिक

1 Other crops cannot be grown वहा अ फसलो को उगाया नही जा सकता ह

2 People in the valley are rice eaters घाटी म लोग चावल खान वाल ह

3 There is a huge export demand of rice वहा चावल की भारी िनयात माग ह

4 It has warm summer and snow melt waters for irrigation

वहा गिमया गम होती ह िसचाई क िलए बफ का िपघला आ पानी होता ह

Correct Answer -

It has warm summer and snow melt waters for irrigation

वहा गिमया गम होती ह िसचाई क िलए बफ का िपघला आ पानी होता ह

CANCELLED

In the geological time scale the Mesozoic Era DOES NOT contains which of the following periods

भगभय समय पमान पर मजीवी यग म िन कालो म स कौन नही ह

1 Triassic ट ाइऐिसक

2 Jurassic जरिसक

3 Cretaceous चाकमय

4 Carboniferous काबनी

Correct Answer -

Carboniferous काबनी

96)

1 P-3 Q-4 R-2 S-1

2 P-3 Q-4 R-1 S-2

3 P-3 Q-4 R-1 S-2

4 P-4 Q-3 R-2 S-1

Correct Answer -

P-4 Q-3 R-2 S-1

1 P-3 Q-1 R-4 S-2

2 P-3 Q-4 R-1 S-2

3 P-3 Q-2 R-4 S-1

97)

98)

4 P-2 Q-1 R-4 S-3

Correct Answer -

P-3 Q-4 R-1 S-2

1 P-3 Q-1 R-4 S-2

2 P-2 Q-3 R-4 S-1

3 P-2 Q-1 R-3 S-4

4 P-4 Q-2 R-1 S-3

Correct Answer -

P-3 Q-1 R-4 S-2

99)

100)

1 P-3 Q-2 R-4 S-1

2 P-1 Q-2 R-3 S-4

3 P-2 Q-3 R-1 S-4

4 P-4 Q-3 R-2 S-1

Correct Answer -

P-2 Q-3 R-1 S-4

ldquoHuman geography is the study of changing relationship between the unresting man and the unstableearthrdquo was defined by

lsquolsquoमानव भगोल ाकल आदमी और अथर पी क बीच सबध परवतन का अयन हrdquo ______ ारा परभािषत िकया गयाथा

1 J Brunches ज चस

2 EC Semple ईसी सल

3 HJ Mackinder एच ज मिकदर

4 PV Blache पीवी च

Correct Answer -

EC Semple ईसी सल

Sedimentary rocks are finally and ultimately derived from the____________

अवसादी चान अततः ________ स ा की जाती ह

1 action of earth movements पी की गितिविधयो

2 marine deposit समी िनप

3 weathering of metamorphic rocks पातरत चानो क अपय

4 weathering of igneous rocks आय चानो क अपय

Correct Answer -

weathering of igneous rocks आय चानो क अपय

Page 46: High School Teacher Eligibility Test- BOARD PROFESSIONAL ...peb.mp.gov.in/results/RESULT_18/HST_RES18/Final_anwser_key/HST… · M a ndl a / मंड ल ... Under the Madhya Pradesh

91)

92)

2 Equatorial continental भमवत महाीपीय

3 Polar maritime वीय समीय

4 Polar continental वीय महाीपीय

Correct Answer -

Equatorial maritime भमवत समीय

A very high temperature during summer in north western India leads to what type of climaticcondition in south

उर पिमी भारत म गम क दौरान बत अिधक तापमान होन क कारण दिण म िकस कार की जलवाय थितउ करता ह

1 Depression over arabian sea अरब सागर पर अवनमन

2 Failure monsoon मानसन िवफलता

3 Successful monsoon मानसन सफलता

4 Cyclones चवात

Correct Answer -

Successful monsoon मानसन सफलता

Lightning and thunder are the resultant effect when तिड़त और गजन परणामी भाव ह जब

1 Two massive clouds hit powerfully each other first lightning is produced and later sound is produced

दो बड़ बादल एक दसर स शशाली ढग स टकरात ह पहल आकाशीय िवदयत उ होता ह और बाद म िन उहोती ह

2 Two massive clouds come into contact with the powerful wind collision this results into first sound and thenlightning

दो बड़ बादल शशाली पवन सघ क सपक म आत ह इसका परणामप पहल िन और िफर आकाशीय िवदयतउ होता ह

3 None of the above उपरो म स कोई भी नही

4 A high density cloud contains positively and negatively charged electric ions and when this interacts light andsound are simultaneously produced

एक उ घन बादल म धनाक और ऋणाक आविशत िवदयत आयन होत ह और जब यह परर भाव डालत ह तोकाश और िन एक साथ उािदत होती ह

Correct Answer -

A high density cloud contains positively and negatively charged electric ions and when this interacts light andsound are simultaneously produced

औ ि ि ो औ ो

93)

94)

95)

एक उ घन बादल म धनाक और ऋणाक आविशत िवदयत आयन होत ह और जब यह परर भाव डालत ह तोकाश और िन एक साथ उािदत होती ह

Doon Valley is able to grow rice because दन घाटी चावल उगान म सम ह ोिक

1 Other crops cannot be grown वहा अ फसलो को उगाया नही जा सकता ह

2 People in the valley are rice eaters घाटी म लोग चावल खान वाल ह

3 There is a huge export demand of rice वहा चावल की भारी िनयात माग ह

4 It has warm summer and snow melt waters for irrigation

वहा गिमया गम होती ह िसचाई क िलए बफ का िपघला आ पानी होता ह

Correct Answer -

It has warm summer and snow melt waters for irrigation

वहा गिमया गम होती ह िसचाई क िलए बफ का िपघला आ पानी होता ह

CANCELLED

In the geological time scale the Mesozoic Era DOES NOT contains which of the following periods

भगभय समय पमान पर मजीवी यग म िन कालो म स कौन नही ह

1 Triassic ट ाइऐिसक

2 Jurassic जरिसक

3 Cretaceous चाकमय

4 Carboniferous काबनी

Correct Answer -

Carboniferous काबनी

96)

1 P-3 Q-4 R-2 S-1

2 P-3 Q-4 R-1 S-2

3 P-3 Q-4 R-1 S-2

4 P-4 Q-3 R-2 S-1

Correct Answer -

P-4 Q-3 R-2 S-1

1 P-3 Q-1 R-4 S-2

2 P-3 Q-4 R-1 S-2

3 P-3 Q-2 R-4 S-1

97)

98)

4 P-2 Q-1 R-4 S-3

Correct Answer -

P-3 Q-4 R-1 S-2

1 P-3 Q-1 R-4 S-2

2 P-2 Q-3 R-4 S-1

3 P-2 Q-1 R-3 S-4

4 P-4 Q-2 R-1 S-3

Correct Answer -

P-3 Q-1 R-4 S-2

99)

100)

1 P-3 Q-2 R-4 S-1

2 P-1 Q-2 R-3 S-4

3 P-2 Q-3 R-1 S-4

4 P-4 Q-3 R-2 S-1

Correct Answer -

P-2 Q-3 R-1 S-4

ldquoHuman geography is the study of changing relationship between the unresting man and the unstableearthrdquo was defined by

lsquolsquoमानव भगोल ाकल आदमी और अथर पी क बीच सबध परवतन का अयन हrdquo ______ ारा परभािषत िकया गयाथा

1 J Brunches ज चस

2 EC Semple ईसी सल

3 HJ Mackinder एच ज मिकदर

4 PV Blache पीवी च

Correct Answer -

EC Semple ईसी सल

Sedimentary rocks are finally and ultimately derived from the____________

अवसादी चान अततः ________ स ा की जाती ह

1 action of earth movements पी की गितिविधयो

2 marine deposit समी िनप

3 weathering of metamorphic rocks पातरत चानो क अपय

4 weathering of igneous rocks आय चानो क अपय

Correct Answer -

weathering of igneous rocks आय चानो क अपय

Page 47: High School Teacher Eligibility Test- BOARD PROFESSIONAL ...peb.mp.gov.in/results/RESULT_18/HST_RES18/Final_anwser_key/HST… · M a ndl a / मंड ल ... Under the Madhya Pradesh

93)

94)

95)

एक उ घन बादल म धनाक और ऋणाक आविशत िवदयत आयन होत ह और जब यह परर भाव डालत ह तोकाश और िन एक साथ उािदत होती ह

Doon Valley is able to grow rice because दन घाटी चावल उगान म सम ह ोिक

1 Other crops cannot be grown वहा अ फसलो को उगाया नही जा सकता ह

2 People in the valley are rice eaters घाटी म लोग चावल खान वाल ह

3 There is a huge export demand of rice वहा चावल की भारी िनयात माग ह

4 It has warm summer and snow melt waters for irrigation

वहा गिमया गम होती ह िसचाई क िलए बफ का िपघला आ पानी होता ह

Correct Answer -

It has warm summer and snow melt waters for irrigation

वहा गिमया गम होती ह िसचाई क िलए बफ का िपघला आ पानी होता ह

CANCELLED

In the geological time scale the Mesozoic Era DOES NOT contains which of the following periods

भगभय समय पमान पर मजीवी यग म िन कालो म स कौन नही ह

1 Triassic ट ाइऐिसक

2 Jurassic जरिसक

3 Cretaceous चाकमय

4 Carboniferous काबनी

Correct Answer -

Carboniferous काबनी

96)

1 P-3 Q-4 R-2 S-1

2 P-3 Q-4 R-1 S-2

3 P-3 Q-4 R-1 S-2

4 P-4 Q-3 R-2 S-1

Correct Answer -

P-4 Q-3 R-2 S-1

1 P-3 Q-1 R-4 S-2

2 P-3 Q-4 R-1 S-2

3 P-3 Q-2 R-4 S-1

97)

98)

4 P-2 Q-1 R-4 S-3

Correct Answer -

P-3 Q-4 R-1 S-2

1 P-3 Q-1 R-4 S-2

2 P-2 Q-3 R-4 S-1

3 P-2 Q-1 R-3 S-4

4 P-4 Q-2 R-1 S-3

Correct Answer -

P-3 Q-1 R-4 S-2

99)

100)

1 P-3 Q-2 R-4 S-1

2 P-1 Q-2 R-3 S-4

3 P-2 Q-3 R-1 S-4

4 P-4 Q-3 R-2 S-1

Correct Answer -

P-2 Q-3 R-1 S-4

ldquoHuman geography is the study of changing relationship between the unresting man and the unstableearthrdquo was defined by

lsquolsquoमानव भगोल ाकल आदमी और अथर पी क बीच सबध परवतन का अयन हrdquo ______ ारा परभािषत िकया गयाथा

1 J Brunches ज चस

2 EC Semple ईसी सल

3 HJ Mackinder एच ज मिकदर

4 PV Blache पीवी च

Correct Answer -

EC Semple ईसी सल

Sedimentary rocks are finally and ultimately derived from the____________

अवसादी चान अततः ________ स ा की जाती ह

1 action of earth movements पी की गितिविधयो

2 marine deposit समी िनप

3 weathering of metamorphic rocks पातरत चानो क अपय

4 weathering of igneous rocks आय चानो क अपय

Correct Answer -

weathering of igneous rocks आय चानो क अपय

Page 48: High School Teacher Eligibility Test- BOARD PROFESSIONAL ...peb.mp.gov.in/results/RESULT_18/HST_RES18/Final_anwser_key/HST… · M a ndl a / मंड ल ... Under the Madhya Pradesh

96)

1 P-3 Q-4 R-2 S-1

2 P-3 Q-4 R-1 S-2

3 P-3 Q-4 R-1 S-2

4 P-4 Q-3 R-2 S-1

Correct Answer -

P-4 Q-3 R-2 S-1

1 P-3 Q-1 R-4 S-2

2 P-3 Q-4 R-1 S-2

3 P-3 Q-2 R-4 S-1

97)

98)

4 P-2 Q-1 R-4 S-3

Correct Answer -

P-3 Q-4 R-1 S-2

1 P-3 Q-1 R-4 S-2

2 P-2 Q-3 R-4 S-1

3 P-2 Q-1 R-3 S-4

4 P-4 Q-2 R-1 S-3

Correct Answer -

P-3 Q-1 R-4 S-2

99)

100)

1 P-3 Q-2 R-4 S-1

2 P-1 Q-2 R-3 S-4

3 P-2 Q-3 R-1 S-4

4 P-4 Q-3 R-2 S-1

Correct Answer -

P-2 Q-3 R-1 S-4

ldquoHuman geography is the study of changing relationship between the unresting man and the unstableearthrdquo was defined by

lsquolsquoमानव भगोल ाकल आदमी और अथर पी क बीच सबध परवतन का अयन हrdquo ______ ारा परभािषत िकया गयाथा

1 J Brunches ज चस

2 EC Semple ईसी सल

3 HJ Mackinder एच ज मिकदर

4 PV Blache पीवी च

Correct Answer -

EC Semple ईसी सल

Sedimentary rocks are finally and ultimately derived from the____________

अवसादी चान अततः ________ स ा की जाती ह

1 action of earth movements पी की गितिविधयो

2 marine deposit समी िनप

3 weathering of metamorphic rocks पातरत चानो क अपय

4 weathering of igneous rocks आय चानो क अपय

Correct Answer -

weathering of igneous rocks आय चानो क अपय

Page 49: High School Teacher Eligibility Test- BOARD PROFESSIONAL ...peb.mp.gov.in/results/RESULT_18/HST_RES18/Final_anwser_key/HST… · M a ndl a / मंड ल ... Under the Madhya Pradesh

97)

98)

4 P-2 Q-1 R-4 S-3

Correct Answer -

P-3 Q-4 R-1 S-2

1 P-3 Q-1 R-4 S-2

2 P-2 Q-3 R-4 S-1

3 P-2 Q-1 R-3 S-4

4 P-4 Q-2 R-1 S-3

Correct Answer -

P-3 Q-1 R-4 S-2

99)

100)

1 P-3 Q-2 R-4 S-1

2 P-1 Q-2 R-3 S-4

3 P-2 Q-3 R-1 S-4

4 P-4 Q-3 R-2 S-1

Correct Answer -

P-2 Q-3 R-1 S-4

ldquoHuman geography is the study of changing relationship between the unresting man and the unstableearthrdquo was defined by

lsquolsquoमानव भगोल ाकल आदमी और अथर पी क बीच सबध परवतन का अयन हrdquo ______ ारा परभािषत िकया गयाथा

1 J Brunches ज चस

2 EC Semple ईसी सल

3 HJ Mackinder एच ज मिकदर

4 PV Blache पीवी च

Correct Answer -

EC Semple ईसी सल

Sedimentary rocks are finally and ultimately derived from the____________

अवसादी चान अततः ________ स ा की जाती ह

1 action of earth movements पी की गितिविधयो

2 marine deposit समी िनप

3 weathering of metamorphic rocks पातरत चानो क अपय

4 weathering of igneous rocks आय चानो क अपय

Correct Answer -

weathering of igneous rocks आय चानो क अपय

Page 50: High School Teacher Eligibility Test- BOARD PROFESSIONAL ...peb.mp.gov.in/results/RESULT_18/HST_RES18/Final_anwser_key/HST… · M a ndl a / मंड ल ... Under the Madhya Pradesh

99)

100)

1 P-3 Q-2 R-4 S-1

2 P-1 Q-2 R-3 S-4

3 P-2 Q-3 R-1 S-4

4 P-4 Q-3 R-2 S-1

Correct Answer -

P-2 Q-3 R-1 S-4

ldquoHuman geography is the study of changing relationship between the unresting man and the unstableearthrdquo was defined by

lsquolsquoमानव भगोल ाकल आदमी और अथर पी क बीच सबध परवतन का अयन हrdquo ______ ारा परभािषत िकया गयाथा

1 J Brunches ज चस

2 EC Semple ईसी सल

3 HJ Mackinder एच ज मिकदर

4 PV Blache पीवी च

Correct Answer -

EC Semple ईसी सल

Sedimentary rocks are finally and ultimately derived from the____________

अवसादी चान अततः ________ स ा की जाती ह

1 action of earth movements पी की गितिविधयो

2 marine deposit समी िनप

3 weathering of metamorphic rocks पातरत चानो क अपय

4 weathering of igneous rocks आय चानो क अपय

Correct Answer -

weathering of igneous rocks आय चानो क अपय